You are on page 1of 643

LABOR LAW REVIEW Atty.

Joyrich Golangco

Recruitment and Placement and OFWs

1. PEOPLE OF THE PHILIPPINES v. MELISSA CHUA


G.R. No. 184058 March 10, 2010

Doctrines:
A person convicted of illegal recruitment may, in addition, be convicted of Estafa as penalized under Article 315,
paragraph 2(a) of the Revised Penal Code, held that the elements thereof were sufficiently established, viz:
(a) that appellant deceived the complainants by assuring them of employment in Taiwan provided they pay
the required placement fee;
(b) that relying on such representation, the complainants paid appellant the amount demanded;
(c) that her representation turned out to be false because she failed to deploy them as promised; and
(d) that the complainants suffered damages when they failed to be reimbursed the amounts they paid.

ILLEGAL RECRUITMENT v. ESTAFA


Illegal recruitment is malum prohibitum, while estafa is malum in se. In the first, the criminal intent of the
accused is not necessary for conviction. In the second, such an intent is imperative. Estafa under Article 315,
paragraph 2, of the Revised Penal Code, is committed by any person who defrauds another by using fictitious
name, or falsely pretends to possess power, influence, qualifications, property, credit, agency, business or
imaginary transactions, or by means of similar deceits executed prior to or simultaneously with the commission
of fraud.

FACTS:
Sometime in 2002, accused Josie Campos (at large) and accused-appellant Chua promised to the complainants
deployment to Taiwan as factory workers. In connection with their deployment, accused-appellant Chua, who
was then a temporary cashier at Golden Gate (Recruitment Agency), collected from the complainants the
following amount as part of their placement fees:
ERIK DE GUIA TAN - P73,000.00
MARILYN D. MACARANAS - 83,000.00
NAPOLEON H. YU, JR. - 23,000.00
HARRY JAMES P. KING - 23,000.00
ROBERTO C. ANGELES - 23,000.00

Without valid reasons and without fault on the part of the said complainants, accused-appellant Chua failed to
actually deploy them and failed to reimburse expenses incurred in connection with their documentation and
processing for purposes of their deployment.

As her defense, appellant Chua maintains that Golden Gate was a licensed recruitment agency and that Josie,
who is her godmother, was an agent. Admitting having received P80,000 each from Marilyn and Tan, receipt of
which she issued but denying receiving any amount from King, she claimed that she turned over the money to
the documentation officer, one Arlene Vega, who in turn remitted the money to Marilyn Calueng whose present
whereabouts she did not know.

RTC RULING: Found CHUA GUILTY as principal of a large scale illegal recruitment and estafa three (3) counts.
She is sentenced to life imprisonment and to pay a fine of Five Hundred Thousand Pesos (P500,000.00) for
illegal recruitment. As regards Criminal Cases Nos. 04-222597 and 04-222599, both are dismissed for lack of
interest of complainants Roberto Angeles and Napoleon Yu, Jr.

CA RULING: AFFIRMED RTC

ISSUE: WON CHUA is guilty of Illegal Recruitment in a Large Scale and Estafa.

SC RULING:
YES. The recruitment activities were done at the time Golden Gates license had already expired. Further, the
illegal recruitment was made in a large scale because the following essential elements are present, to wit: (1)
the accused undertook a recruitment activity under Article 13(b) or any prohibited practice under Article 34 of the
Labor Code; (2) the accused did not have the license or the authority to lawfully engage in the recruitment and
placement of workers; and (3) the accused committed such illegal activity against three or more persons
individually or as a group.
San Beda College of Law 58
4S: 2015 - 2016
LABOR LAW REVIEW Atty. Joyrich Golangco

Even if CHUA were a mere temporary cashier of Golden Gate, that did not make her any less an employee to
be held liable for illegal recruitment as principal by direct participation, together with the employer, as it was
shown that she actively and consciously participated in the recruitment process.

Assuming arguendo that CHUA was unaware of the illegal nature of the recruitment business of Golden Gate,
that does not free her of liability either. Illegal Recruitment in Large Scale penalized under Republic Act No.
8042, or "The Migrant Workers and Overseas Filipinos Act of 1995," is a special law, a violation of which is
malum prohibitum, not malum in se. Intent is thus immaterial. And that explains why CHUA was, aside from
Estafa, convicted of such offense.

San Beda College of Law 59


4S: 2015 - 2016
LABOR LAW REVIEW Atty. Joyrich Golangco

2. PEOPLE OF THE PHILIPPINES v. RODOLFO GALLO y GADOT, FIDES PACARDO y JUNGCO and
PILAR MANTA y DUNGO
G.R. No. 184058 March 10, 2010
VELASCO, JR., J.:

Doctrine:
To commit syndicated illegal recruitment, three elements must be established: (1) the offender undertakes
either any activity within the meaning of "recruitment and placement" defined under Article 13(b), or any of the
prohibited practices enumerated under Art. 34 of the Labor Code; (2) he has no valid license or authority
8
required by law to enable one to lawfully engage in recruitment and placement of workers; and (3) the illegal
recruitment is committed by a group of three (3) or more persons conspiring or confederating with one
another.When illegal recruitment is committed by a syndicate or in large scale, i.e., if it is committed against
three (3) or more persons individually or as a group, it is considered an offense involving economic sabotage.

FACTS:
The present case involves two criminal cases, one for syndicated illegal recruitment and the other for estafa,
filed by Edgardo Dela Caza against 13 persons including the accused. However, only accused-appellant Gallo,
Pacardo and Manta have proceeded to trial because the others were at large.

Sometime in May 2011, Dela Caza was briefed by Mardeolyn, president of MPM Agency (non-licensed), about
the processing of his papers for a possible job opportunity in Korea, as well as their possible salary. One Yeo
Sin Ung, a Korean national, gave a briefing about the business and what to expect from the company. Then,
here comes accused-appellant Gallo who introduced himself as Mardeolyns relative and specifically told Dela
Caza of the fact that the agency was able to send many workers abroad. Dela Caza was even showed several
workers visas who were already allegedly deployed abroad. Later on, accused-appellant Gallo signed and
issued an official receipt acknowledging the down payment of Dela Caza. After two weeks, Dela Caza went
back to MPMs office only to find out that it had transferred to another place. After successfully locating the new
office, Dela Caza made a follow-up again but the agency failed to deploy him. Later on, MPM transferred to
another location without informing Dela Caza. After two months, MPM has still failed to deploy Dela Caza and
also failed to return his money despite repeated demands. Dela Caza and other applicants decided to take
action.

As his defense, accused-appellant Gallo denied having any part in the recruitment of Dela Caza. In fact, he
testified that he also applied with MPM Agency for deployment to Korea as a factory worker. He then worked for
MPM in order for him to fully pay the placement fee. Accused-appellant Gallo further avers that he cannot be
held criminally liable for illegal recruitment because he was neither an officer nor an employee of the recruitment
agency.

RTC RULING:
Found GALLO GUILTY of syndicated illegal recruitment and estafa. He is sentenced to life imprisonment and to
pay a fine of P 1, 000,000.00 for illegal recruitment. He is ordered to return P 45, 000 to Dela Caza.

Pacardo and Manta were acquitted for lack of evidence. It was only established that Pacardo acted as the
MPMs employee who was in charge of the records of the applicants. Manta, on the other hand, was also an
employee who was tasked to deliver documents to the Korean Embassy.

CA RULING: AFFIRMED RTC with MODIFICATION


In the Criminal Case for estafa, accused-appellant Gallo is sentenced to four (4) years of prision correccional to
ten (10) years of prision mayor (previous sentence was FOUR (4) years of prision correccional as minimum to
NINE (9) years of prision mayor as maximum)

ISSUE: WON GALLO is guilty of Syndicated Illegal Recruitment and Estafa.

SC RULING:
YES. All the elements for Syndicated Illegal Recruitment are present (see case doctrine). Further, the testimony
Dela Caza showed that accused-appellant Gallo made false misrepresentations and promises in assuring them
that after they paid the placement fee, jobs in Korea as factory workers were waiting for them and that they
would be deployed soon. In fact, Dela Caza personally talked to accused-appellant Gallo and gave him the
money and saw him sign and issue an official receipt as proof of his payment. Without a doubt, accused-
San Beda College of Law 60
4S: 2015 - 2016
LABOR LAW REVIEW Atty. Joyrich Golangco

appellant Gallos actions constituted illegal recruitment.

Additionally, accused-appellant Gallo cannot argue that the trial court erred in finding that he was indeed an
employee of the recruitment agency. On the contrary, his active participation in the illegal recruitment is
unmistakable. The fact that he was the one who issued and signed the official receipt belies his profession of
innocence.

San Beda College of Law 61


4S: 2015 - 2016
LABOR LAW REVIEW Atty. Joyrich Golangco

3. CLAUDIO S. YAP v. THENAMARIS SHIP'S MANAGEMENT and INTERMARE MARITIME AGENCIES,
INC.
G.R. No. 179532 May 30, 2011
NACHURA, J.:

Doctrine:
While this case was pending before the SC, we declared as unconstitutional the clause "or for three months for
every year of the unexpired term, whichever is less" provided in the 5th paragraph of Section 10 of R.A. No.
8042. The reason for declaring such clause unconstitutional is that it gives an erring employer the option to pay
an illegally dismissed migrant worker only three months for every year of the unexpired term of his contract
whereas under the Labor Code, he is guaranteed with reinstatement with full backwages computed from the
time compensation was withheld from them up to their actual reinstatement.

FACTS:
In August 23, 2001, Yap commenced the job as an electrician of the vessel, M/T SEASCOUT, with Intermare
as his agency and Vulture Shipping Ltd., as his principal. The duration of his contract was for 12 months.
However, on November 8, 2001, the vessel was sold, so Yap effectively lost his job. Capt. Adviento of Intermare
and Thenamaris informed the crew members, including Yap, that they are given the option to be transferred to
other vessels. Yap received bonuses and was offered payment equivalent to his one-month basic wage. He
refused to accept the payment of one-month basic wage, insisting that he is entitled to the payment of the
unexpired portion of his contract since he was illegally dismissed from employment. The refusal of Intermare
and Thenamaris to pay the unexpired portion of Yaps contract and their inaction in transferring him to other
vessels have lead Yap to file a complaint for Illegal Dismissal.

LA RULING:
Yap has been CONSTRUCTIVELY AND ILLEGALLY DISMISSED by respondents. The latter was ordered to
pay Yap the amount corresponding to the unexpired portion of his contract with damages.

LA found that respondents acted in bad faith when they assured Yap of re-embarkation and required him to
produce an electrician certificate during the period of his contract, but actually he was not able to board one
despite of respondents numerous vessels.

NLRC RULING: AFFIRMED LA with MODIFICATION


The NLRC ordered the respondent to pay Yap the amount equivalent to three (3) months basic salary pursuant
the clause "or for three months for every year of the unexpired term, whichever is less" provided in the 5th
paragraph of Section 10 of R.A. No. 8042.

CA RULING: AFFIRMED NLRC

ISSUE: WON Yap is entitled to the payment of the unexpired portion of his contract.

SC RULING:
YES. During the pendency of this case before the SC, the clause "or for three months for every year of the
unexpired term, whichever is less" has been declared unconstitutional. Thus, Yap is entitled to the payment of
the unexpired portion of his contract.

The doctrine of operative fact as an exception to Article 7 of the New Civil Code is not applicable in this case
because the respondents were in bad faith in the first place. The doctrine only applies as a matter of equity and
fair play. It nullifies the effects of an unconstitutional law by recognizing that the existence of a statute prior to a
determination of unconstitutionality is an operative fact and may have consequences which cannot always be
ignored.

The SC also ruled, as raised for the first time, that the tanker allowance is part of the basic salary. It is even
encapsulated in the basic salary clause found in Yaps contract.

San Beda College of Law 62


4S: 2015 - 2016
LABOR LAW REVIEW Atty. Joyrich Golangco

4. PEOPLE OF THE PHILIPPINES v. HON. DOMINGO PANIS, Presiding Judge of the Court of First
Instance of Zambales & Olongapo City, Branch III and SERAPIO ABUG
G.R. Nos. L-58674-77 July 11, 1990
CRUZ, J:

Doctrine:
The proviso That any person or entity which, in any manner, offers or promises for a fee employment to two or
more persons shall be deemed engaged in recruitment and placement found in Article 13(b) of the Labor Code
merely creates a presumption that an individual or entity is engaged in recruitment and placement whenever he
or it is dealing with two or more persons to whom, in consideration of a fee, an offer or promise of employment is
made in the course of the "canvassing, enlisting, contracting, transporting, utilizing, hiring or procuring (of)
workers. ". It does not require that there be at least two victims before the activity is to be considered as illegal
recruitment.

FACTS:
On January 9, 1981, four (4) Informations were filed against Abug charging him of Illegal Recruitment. Abug
then filed a Motion to Quash on the ground that the informations did not charge an offense because he was
accused of illegally recruiting only one person in each of the four informations. Under the proviso in Article 13(b),
he claimed, there would be illegal recruitment only "whenever two or more persons are in any manner promised
or offered any employment for a fee. " On the other hand, the prosecution argues that the requirement of two or
more persons is imposed only where the recruitment and placement consists of an offer or promise of
employment to such persons and always in consideration of a fee. The other acts mentioned in the body of the
article may involve even only one person and are not necessarily for profit.

ISSUE: WON the Labor Code requires that there be at least two victims before an activity is to be considered as
illegal recruitment.

SC RULING:
NO. The proviso merely lays down a rule of evidence that where a fee is collected in consideration of a promise
or offer of employment to two or more prospective workers, the individual or entity dealing with them shall be
deemed to be engaged in the act of recruitment and placement. The words "shall be deemed" create that
presumption.

This is not unlike the presumption in article 217 of the Revised Penal Code, for example, regarding the failure of
a public officer to produce upon lawful demand funds or property entrusted to his custody. Such failure shall be
prima facie evidence that he has put them to personal use; in other words, he shall be deemed to have
malversed such funds or property. In the instant case, the word "shall be deemed" should by the same token be
given the force of a disputable presumption or of prima facie evidence of engaging in recruitment and
placement.

San Beda College of Law 63


4S: 2015 - 2016
LABOR LAW REVIEW Atty. Joyrich Golangco

5. TRANS ACTION OVERSEAS CORPORATION v.THE HONORABLE SECRETARY OF LABOR and (33)
PRIVATE RESPONDENTS
G.R. No. 109583 September 5, 1997
ROMERO, J.:

Doctrine:
In view of the Court's disposition in the case of People v. Diaz, we rule that the power to suspend or cancel any
license or authority to recruit employees for overseas employment is concurrently vested with the POEA and the
Secretary of Labor.

FACTS:
From July 24 to September 9, 1987, Trans, a private fee-charging employment agency, scoured Iloilo City for
possible recruits for alleged job vacancies in Hongkong. Private respondents sought employment as domestic
helpers through Transs employees, Luzviminda Aragon, Ben Hur Domincil and his wife Cecille. The applicants
paid placement fees ranging from P1,000 to P14,000, but Trans failed to deploy them. Their demands for refund
proved unavailing; thus, they were constrained to institute complaints against Trans for violation of Articles 32
and 34(a) of the Labor Code, as amended. For their part, Trans claims that they have not receive the money
and that their employees had no authority to collect the same.

On April 5, 1991, Labor Usec. Nieves Confesor rendered the assailed Order cancelling the license of Trans.
According to Usec. Confesor, Trans is liable for twenty eight (28) counts of violation of Article 32 and five (5)
counts of Article 34 (a) with a corresponding suspension in the aggregate period of sixty six (66) months.
Considering however, that under the schedule of penalties, any suspension amounting to a period of 12 months
merits the imposition of the penalty of cancellation, prompting him to cancel Trans license.

Trans filed a petition under Rule 65. Trans contends that Usec. Confesor acted with grave abuse of discretion in
rendering the assailed orders on alternative grounds, viz.: (1) it is the Philippine Overseas Employment
Administration (POEA) which has the exclusive and original jurisdiction to hear and decide illegal recruitment
cases, including the authority to cancel recruitment licenses, or (2) the cancellation order based on the 1987
POEA Schedule of Penalties is not valid for non-compliance with the Revised Administrative Code of 1987
regarding its registration with the U.P. Law Center.

ISSUES:
(a) WON the DOLE, through Usec. Confesor, has the authority to cancel the license of Trans.
(b) WON the non-filing of the 1987 POEA Schedule of Penalties with the UP Law Center rendered it
ineffective.

SC RULING:
(a) YES. See Case Doctrine

(b) NO. The POEA Revised Rules on the Schedule of Penalties was issued pursuant to Article 34 of the
Labor Code, as amended. The same merely amplified and particularized the various violations of the rules and
regulations of the POEA and clarified and specified the penalties therefore. The questioned schedule of
penalties contains only a listing of offenses. It does not prescribe additional rules and regulations governing
overseas employment but only detailed the administrative sanctions imposable by this Office for some
enumerated prohibited acts.

Under the circumstances, the license of the respondent agency was cancelled on the authority of Article 35 of
the Labor Code, as amended, and not pursuant to the 1987 POEA Revised Rules on Schedule of Penalties.

San Beda College of Law 64


4S: 2015 - 2016
LABOR LAW REVIEW Atty. Joyrich Golangco

6. REPUBLIC OF THE PHILIPPINES v. PRINCIPALIA MANAGEMENT AND PERSONNEL CONSULTANTS,
INC.
G.R. No. 167639 April 19, 2006
YNARES-SANTIAGO, J.:

Doctrine:
Until such time that the appeal (as to the Order of Suspension) is resolved with finality by the DOLE, Principalia
has a clear and convincing right to operate as a recruitment agency.

FACTS:
This case stemmed from two separate complaints filed before the Philippine Overseas Employment
Administration (POEA) against Principalia Management and Personnel Consultants, Incorporated (Principalia)
for violation of the 2002 POEA Rules and Regulations.

The first complaint was filed by Ruth Yasmin Concha where she alleged that after paying P20,000.00 fee
required by Principalia which was not properly receipted, Principalia failed to deploy her for employment abroad
as caregiver or physical therapist. The Adjudication Office of the POEA found Principalia liable for violations of
the 2002 POEA Rules and Regulations.

The second complaint was filed by Rafael E. Baldoza. He alleged that Principalia assured him of employment in
Doha, Qatar as a machine operator with a monthly salary of $450.00. After paying P20,000.00 as placement
fee, he departed but when he arrived, he was made to work as welder. An alternative position as helper was
offered to him, which he refused. Thus, he was repatriated.

On November 12, 2003, Baldoza and Principalia entered into a compromise agreement with quitclaim and
release whereby the latter agreed to redeploy Baldoza for employment abroad. Principalia, however, failed to
deploy Baldoza as agreed hence, in an Order dated April 29, 2004, the POEA suspended Principalias
documentary processing. Principalia moved for reconsideration which the POEA granted on June 25, 2004. The
latter lifted its order suspending the documentary processing by Principalia after noting that it exerted efforts to
obtain overseas employment for Baldoza within the period stipulated in the settlement agreement but due to
Baldozas lack of qualification, his application was declined by its foreign principal.

Meanwhile, on June 14, 2004, or before the promulgation of POEAs order lifting the suspension, Principalia
filed a Complaint (Complaint) against Rosalinda D. Baldoz in her capacity as Administrator of POEA and Atty.
Jovencio R. Abara in his capacity as POEA Conciliator, before the RTC of Mandaluyong City for "Annulment of
Order for Suspension of Documentation Processing with Damages and Application for Issuance of a Temporary
Restraining Order and/or Writ of Preliminary Injunction, and a Writ of Preliminary Mandatory Injunction."
Principalia claimed that the suspension of its documentary processing would ruin its reputation and goodwill and
would cause the loss of its applicants, employers and principals. Thus, a writ of preliminary injunction and a writ
of mandatory injunction must be issued to prevent serious and irreparable damage to it.

On June 14, 2004, the Trial Court granted the prayer for a Temporary Restraining Order enjoining the
defendant[s] Rosalinda D. Baldoz and Atty. Jovencio R. Abara, from implementing the Orders of Suspension.

After the hearing on the preliminary injunction, the trial court held that the issue on the application for preliminary
mandatory injunction has become moot because POEA had already released the renewal of license of
Principalia. It however issue the Writ of Preliminary Prohibitory Injunction prayed for by the plaintiff, upon posting
of a bond in the amount of Php 500,000.00, stressing that the Order of Suspension dated March 15, 2004 is still
pending appeal before the Office of the Secretary of Labor and Employment, and that the said Order dated
March 15, 2004 does not categorically state that the suspension of Plaintiffs License is immediately executory
contrary to the contention of the defendants.

Counsel for POEA argued that the basis for the immediate implementation thereof is Section 5, Rule V, Part VI
of the 2002 POEA Rules and Regulation, which is quoted hereunder, as follows:

"Section 5. Stay of Execution. The decision of the Administration shall be stayed during the
pendency of the appeal; Provided that where the penalty imposed carried the maximum penalty
of twelve (12) months suspension o[r] cancellation of license, the decision shall be immediately
executory despite pendency of the appeal."
San Beda College of Law 65
4S: 2015 - 2016
LABOR LAW REVIEW Atty. Joyrich Golangco

The Order dated March 15, 2004 decreed Plaintiff as having violated Section 2 (a) (d) and (e) of Rule I, Part VI
of the POEA Rules and Regulations and the Plaintiffs was imposed the penalty of twelve (12) months
suspension of license (or in lieu, to pay fine of P120,000, it being its first offense).

Being a first offender, the plaintiff was imposed suspension of license for four (4) months for each violation or an
aggregate period of suspension for twelve (12) months for the three (3) violations.

POEA avers that the trial court gravely abused its discretion in granting the writ of preliminary prohibitory
injunction when the requirements to issue the same have not been met. It asserts that Principalia had no clear
and convincing right to the relief demanded as it had no proof of irreparable damage as required under the
Rules of Court.
ISSUE: Whether or not the trial court erred in issuing the writ of preliminary injunction?

SC RULING:
No. The trial court did not decree that the POEA, as the granting authority of Principalias license to recruit, is
not allowed to determine Principalias compliance with the conditions for the grant, as POEA would have us
believe. For all intents and purposes, POEA can determine whether the licensee has complied with the
requirements. In this instance, the trial court observed that the Order of Suspension dated March 15, 2004 was
pending appeal with the Secretary of the Department of Labor and Employment (DOLE). Thus, until such time
that the appeal is resolved with finality by the DOLE, Principalia has a clear and convincing right to
operate as a recruitment agency.

Furthermore, irreparable damage was duly proven by Principalia. Suspension of its license is not easily
quantifiable nor is it susceptible to simple mathematical computation, as alleged by POEA.
If the injunctive writ was not granted, Principalia would have been labeled as an untrustworthy
recruitment agency before there could be any final adjudication of its case by the DOLE. It would have
lost both its employer-clients and its prospective Filipino-applicants. Loss of the former due to a tarnished
reputation is not quantifiable.

Moreover, POEA would have no authority to exercise its regulatory functions over Principalia because the
matter had already been brought to the jurisdiction of the DOLE. Principalia has been granted the license to
recruit and process documents for Filipinos interested to work abroad. Thus, POEAs action of suspending
Principalias license before final adjudication by the DOLE would be premature and would amount to a
violation of the latters right to recruit and deploy workers.

San Beda College of Law 66


4S: 2015 - 2016
LABOR LAW REVIEW Atty. Joyrich Golangco

7. SANTOSA B. DATUMAN v. FIRST COSMOPOLITAN MANPOWER AND PROMOTION SERVICES, INC.
G.R. No. 156029 November 14, 2008
LEONARDO-DE CASTRO, J.:

Doctrine:
The signing of the "substitute" contracts with the foreign employer/principal before the expiration of the POEA-
approved contract and any continuation of petitioner's employment beyond the original one-year term, against
the will of petitioner, are continuing breaches of the original POEA-approved contract.

FACTS:
Sometime in 1989, respondent First Cosmopolitan Manpower & Promotion Services, Inc. recruited petitioner
Santosa B. Datuman to work abroad under the following terms and conditions:
Site of employment - Bahrain
Employees Classification/Position/Grade - Saleslady
Basic Monthly Salary - US$370.00
Duration of Contract - One (1) year
Foreign Employer - Mohammed Sharif Abbas Ghulam Hussain

On April 17, 1989, petitioner was deployed to Bahrain after paying the required placement fee. However, her
employer Mohammed Hussain took her passport when she arrived there; and instead of working as a saleslady,
she was forced to work as a domestic helper with a salary of Forty Bahrain Dinar (BD40.00), equivalent only to
US$100.00. This was contrary to the agreed salary of US$370.00 indicated in her Contract of Employment
signed in the Philippines and approved by the POEA.

On September 1, 1989, her employer compelled her to sign another contract, transferring her to another
employer as housemaid with a salary of BD40.00 for the duration of two (2) years. She pleaded with him to give
her a release paper and to return her passport but her pleas were unheeded. She continued working against her
will. Worse, she even worked without compensation from September 1991 to April 1993 because of her
employer's continued failure and refusal to pay her salary despite demand. In May 1993, she was able to finally
return to the Philippines through the help of the Bahrain Passport and Immigration Department.

In May 1995, petitioner filed a complaint before the POEA Adjudication Office against respondent for
underpayment and non-payment of salary, vacation leave pay and refund of her plane fare. While the case was
pending, she filed the instant case before the NLRC for underpayment of salary for a period of one year and six
months, non-payment of vacation pay and reimbursement of return airfare.

LA RULING:
Labor Arbiter Jovencio Mayor, Jr. rendered a Decision finding respondent liable for violating the terms of the
Employment Contract and ordering it to pay petitioner: (a) the amount of US$4,050.00 representing her salary
differentials for fifteen (15) months; and, (b) the amount of BD 180.00 representing the refund of plane ticket.

NLRC RULING: Affirmed LA with modification. NLRC reduced the award of salary differentials from
US$4,050.00 to US$2,970.00 ratiocinating as follows:

Accordingly, we find that the claims for salary differentials accruing earlier than April of 1993 had
indeed prescribed. This is so as complainant had filed her complaint on May 31, 1995 when she
arrived from the jobsite in April 1993. Since the cause of action for salary differential accrues at
the time when it falls due, it is clear that only the claims for the months of May 1993 to April 1994
have not yet prescribed. With an approved salary rate of US$370.00 vis--vis the amount of
salary received which was $100.00, complainant is entitled to the salary differential for the said
period in the amount of $2,970.00
CA RULING: CA issued the assailed Decision granting the petition and reversing the NLRC and the Labor
Arbiter.

It ruled that the provisions in number 2, Section 10 (a), Rule V, Book I of the Omnibus Rules Implementing the
Labor Code Section 1 (f), Rule II, Book II of the 1991 POEA Rules and Regulations were not made to make the
local agency a perpetual insurer against all untoward acts that may be done by the foreign principal or the direct
employer abroad. It is only as regards the principal contract to which it is privy shall its liability extend.

San Beda College of Law 67


4S: 2015 - 2016
LABOR LAW REVIEW Atty. Joyrich Golangco

ISSUE: Whether or not the CA erred in not holding respondent liable for petitioner's money claims pursuant to
their Contract of Employment.

SC RULING:
YES.

On whether respondent is solidarily liable for petitioner's monetary claims YES


Section 1 of Rule II of the POEA Rules and Regulations states that:
Section 1. Requirements for Issuance of License. - Every applicant for license to operate a
private employment agency or manning agency shall submit a written application together with
the following requirements:
xxx
f. A verified undertaking stating that the applicant:
xxx
(3) Shall assume joint and solidary liability with the employer for all claims and liabilities
which may arise in connection with the implementation of the contract; including but not
limited to payment of wages, death and disability compensation and repatriation. (emphasis
supplied).

The above provisions are clear that the private employment agency shall assume joint and solidary
liability with the employer. This Court has, time and again, ruled that private employment agencies are held
jointly and severally liable with the foreign-based employer for any violation of the recruitment agreement or
contract of employment. This joint and solidary liability imposed by law against recruitment agencies and foreign
employers is meant to assure the aggrieved worker of immediate and sufficient payment of what is due him.
This is in line with the policy of the state to protect and alleviate the plight of the working class.

We cannot agree with the view of the CA that the solidary liability of respondent extends only to the first.
The signing of the "substitute" contracts with the foreign employer/principal before the expiration of the POEA-
approved contract and any continuation of petitioner's employment beyond the original one-year term, against
the will of petitioner, are continuing breaches of the original POEA-approved contract.

To be sure, Republic Act No. 8042 explicitly prohibits the substitution or alteration to the prejudice of the
worker of employment contracts already approved and verified by the Department of Labor and Employment
(DOLE) from the time of actual signing thereof by the parties up to and including the period of the expiration of
the same without the approval of the DOLE.

In Placewell International Services Corporation v. Camote, we held that the subsequently executed side
agreement of an overseas contract worker with her foreign employer which reduced his salary below the amount
approved by the POEA is void because it is against our existing laws, morals and public policy. The said side
agreement cannot supersede the terms of the standard employment contract approved by the POEA. Hence, in
the present case, the diminution in the salary of petitioner from US$370.00 to US$100 (BD 40.00) per month is
void for violating the POEA-approved contract which set the minimum standards, terms, and conditions of her
employment. Consequently, the solidary liability of respondent with petitioner's foreign employer for petitioner's
money claims continues although she was forced to sign another contract in Bahrain. It is the terms of the
original POEA-approved employment contract that shall govern the relationship of petitioner with the respondent
recruitment agency and the foreign employer.

It is the recruitment agency's responsibility to ensure that the terms and conditions of the employment
contract, as approved by the POEA, are faithfully complied with and implemented properly by its foreign
client/principal.

On whether petitioner's claims for underpaid salaries have prescribed PARTLY Prescribed

Article 291 of the Labor Code which provides that:

Art. 291. Money Claims. - All money claims arising from employer-employee relations accruing
during the effectivity of this Code shall be filed within three years from the time that cause of
action accrued; otherwise, they shall be forever barred. (emphasis supplied)

San Beda College of Law 68


4S: 2015 - 2016
LABOR LAW REVIEW Atty. Joyrich Golangco

We do not agree with the CA when it held that the cause of action of petitioner had already prescribed
as the three-year prescriptive period should be reckoned from September 1, 1989 when petitioner was forced to
sign another contract against her will.

To determine for which months petitioner's right to claim salary differentials has not prescribed, we must
count three years prior to the filing of the complaint on May 31, 1995. Thus, only claims accruing prior to May
31, 1992 have prescribed when the complaint was filed on May 31, 1995. Petitioner is entitled to her claims for
salary differentials for the period May 31, 1992 to April 1993, or approximately eleven (11) months.

We find that the NLRC correctly computed the salary differential due to petitioner at US$2,970.00
(US$370.00 as approved salary rate - US$100.00 as salary received = US$290 as underpaid salary per month x
11 months). However, it should be for the period May 31, 1992 to April 1993 and not May 1993 to April 1994 as
erroneously stated in the NLRC's Decision.

San Beda College of Law 69


4S: 2015 - 2016
LABOR LAW REVIEW Atty. Joyrich Golangco

8. STOLT-NIELSEN TRANSPORTATION GROUP, INC. AND CHUNG GAI SHIP MANAGEMENT, vs.
SULPECIO MEDEQUILLO, JR.
G.R. No. 177498 January 18, 2012
PEREZ, J.:

Doctrine:
Even before the start of any employer-employee relationship, contemporaneous with the perfection of the
employment contract was the birth of certain rights and obligations, the breach of which may give rise to a cause
of action against the erring party.

FACTS:
On 6 March 1995, Sulpecio Madequillo (respondent) filed a complaint before the Adjudication Office of the
POEA against the petitioners for illegal dismissal under a first contract and for failure to deploy under a second
contract. In his complaint-affidavit, respondent alleged that:

1. Respondent was hired by Stolt-Nielsen Marine Services, Inc on behalf of its principal Chung-Gai Ship
Management of Panama as Third Assistant Engineer on board the vessel "Stolt Aspiration" for a
period of nine (9) for $1,212.00 per month commencing on 6 November 1991;
2. He then joined the vessel MV "Stolt Aspiration", but only after three (3) months, he was ordered by the
ships master to disembark the vessel and repatriated back to Manila for no reason or explanation;
3. Upon his return to Manila, he immediately proceeded to the petitioners office where he was
transferred employment with another vessel named MV "Stolt Pride" under the same terms and
conditions of the First Contract;
4. POEA approved the Second Contract, however, respondent was not deployed by petitioners despite the
commencement of the contract. POEA subsequently certified the Second Employment Contract without
the knowledge that petitioners failed to deploy the respondent.
5. Because of petitioners alleged non-compliance with the Second Contract, respondent Medequilla
demanded for the return of his passport and other employment documents from the petitioners. He
claimed that he was made to involuntarily sign a document in order to recover his employment papers.
Medequilla prayed for payment of damages as well as attorneys fees for his illegal dismissal and in view of
the Petitioners bad faith in not complying with the Second Contract. The case was transferred to the Labor
Arbiter of the DOLE upon the effectivity of the Migrant Workers and Overseas Filipinos Act of 1995.

LA RULING:
The LA declared the respondents guilty of constructively dismissing the complainant by not honoring the
employment contract. Accordingly, respondents are hereby ordered jointly and solidarily to pay complainant
$12,537.00 or its peso equivalent at the time of payment. The LA found the first contract entered into by and
between the complainant and the respondents to have been novated by the execution of the second
contract. In other words, respondents cannot be held liable for the first contract but are clearly and definitely
liable for the breach of the second contract. However, he ruled that there was no substantial evidence to
grant the prayer for moral and exemplary damages.

NLRC RULING: Affirmed with modification the Decision of the LA.


NLRC deleted the award of overtime pay in the total amount of US $3,636.00.

ISSUE: Whether or not petitioners have the obligation to deploy the respondent by virtue of the perfected
contract, and thus will be held liable for damages in case of non-deployment.

SC RULING:
Yes. The petitioners argue that under the POEA Contract, actual deployment of the seafarer is a suspensive
condition for the commencement of the employment. The Court agreed with petitioners on such point. However,
even without actual deployment, the perfected contract gives rise to obligations on the part of petitioners. Parties
are bound not only to the fulfillment of what has been expressly stipulated but also to all the consequences
which, according to their nature, may be in keeping with good faith, usage and law.

Thus, even if by the standard contract employment commences only "upon actual departure of the seafarer",
this does not mean that the seafarer has no remedy in case of non-deployment without any valid reason.
The Court further made a distinction between the perfection of the employment contract and the
commencement of the employer-employee relationship. The perfection of the contract occurred when petitioner
San Beda College of Law 70
4S: 2015 - 2016
LABOR LAW REVIEW Atty. Joyrich Golangco

and respondent agreed on the object and the cause, as well as the rest of the terms and conditions therein. The
commencement of the employer-employee relationship would have taken place had petitioner been actually
deployed from the point of hire. Thus, even before the start of any employer-employee relationship,
contemporaneous with the perfection of the employment contract was the birth of certain rights and obligations,
the breach of which may give rise to a cause of action against the erring party.

Respondent is thus liable to pay petitioner actual damages in the form of the loss of nine (9) months worth of
salary as provided in the contract. This is but proper because of the non-deployment of respondent without just
cause.

San Beda College of Law 71


4S: 2015 - 2016
LABOR LAW REVIEW Atty. Joyrich Golangco

9. PEOPLE OF THE PHILIPPINES vs. CAROL M. DELA PIEDRA
G.R. No. 121777 January 24, 2001
KAPUNAN, J.:

Doctrine: ILLEGAL RECRUITMENT


Illegal recruitment is committed when two elements concur. First, the offender has no valid license or authority
required by law to enable one to lawfully engage in recruitment and placement of workers. Second, he or she
undertakes either any activity within the meaning of "recruitment and placement" defined under Article 13 (b), or
38
any prohibited practices enumerated under Article 34 of the Labor Code. In case of illegal recruitment in large
scale, a third element is added: that the accused commits said acts against three or more persons, individually
or as a group.

A conviction for large scale illegal recruitment must be based on a finding in each case of illegal recruitment
of three or more persons whether individually or as a group.

FACTS:
Carol M. dela Piedra is convicted for illegal recruitment in large scale and assails, as well, the constitutionality of
the law defining and penalizing said crime.

On January 30, 1994, at exactly 10:00 in the morning, Erlie Ramos, Attorney II of the Philippine Overseas
Employment Agency (POEA), received a telephone call from an unidentified woman inquiring about the
legitimacy of the recruitment conducted by a certain Mrs. Carol Figueroa.

Bellotindos entered the house and pretended to be an applicant; She received a bio-data form from a Carol
Fegueroa Ramos contacted a friend, Mayeth Bellotindos, so they could both go to No. 26-D, Tetuan Highway,
Sta. Cruz, Zamboanga City, where the recruitment was reportedly being undertaken.

A raiding team was planned between POEA and CIS team led by Capt. Mendoza to commence the next day
with SPO2 Fermindoza posing as a would-be-applicant. Fermindoza talked personally with Carol and as the
latter was filling up the application form, Fermindoza signaled to the raiding party waiting outside the house.
Carol Fegueroa was caught holding filled up application forms.

The CIS asked Figueroa if she had a permit to recruit. Figueroa retorted that she was not engaged in
recruitment. Capt. Mendoza nevertheless proceeded to arrest Figueroa. He took the application forms she was
holding as the raiding party seized the other papers on the table. The CIS team then brought Figueroa, a certain
Jasmine Alejandro, and the three women suspected to be applicants, to the office for investigation.

In the course of their investigation, the CIS discovered that Carol Figueroa had many aliases, among them,
Carol Llena and Carol dela Piedra. The accused was not able to present any authority to recruit when
asked by the investigators. A check by Ramos with the POEA revealed that the acused was not licensed
or authorized to conduct recruitment. A certification dated February 2, 1994 stating thus was executed by
Renegold M. Macarulay, Officer-in-Charge of the POEA.

Accused was charged before RTC of Zamboanga in an information alleging:


That on or about January 30, 1994, in the City of Zamboanga, Philippines, Carol dela
Piedra, having no POEA license or authority to engage in recruitment and overseas
placement of workers willfully, unlawfully, and feloniously, offered and promised for a
fee an employment in Singapore to: Maria Lourdes Modesto [y] Gadrino, Nancy
Araneta y Aliwanag and Jennelyn Baez y Timbol Maria Lourdes had already advanced
P2k to accused in consideration of the promised employment.

The accused denied in court that she went to Jasmine's residence to engage in recruitment. She claimed she
came to Zamboanga City to visit her friends, to whom she could confide since she and her husband were having
some problems. She denied she knew Nancy Araneta or that she brought information sheets for job placement.
She also denied instructing Jasmine to collect P2,000 from alleged applicants as processing fee.

ISSUES:
1. Whether or not sec. 13 (b) of P.D. 442, as amended, otherwise known as the illegal
recruitment law is unconstitutional as it violates the due process clause.
San Beda College of Law 72
4S: 2015 - 2016
LABOR LAW REVIEW Atty. Joyrich Golangco

2. Whether or not there is illegal recruitment? Is it in large scale?
RTC RULING:
The accused Carol dela Piedra alias Carol Llena and Carol Figueroa guilty beyond reasonable doubt of
Illegal Recruitment committed in a large scale and hereby sentences her to suffer the penalty of LIFE
IMPRISONMENT and to pay a fine of P100,000.00, and also to pay the costs. Being a detention prisoner, the
said accused is entitled to the full time of the period of her detention during the pendency of this case under the
condition set forth in Article 29 of the Revised Penal Code.

SC RULING:
1. IT IS CONSTITUTIONAL.

In the first assigned error, appellant maintains that the law defining "recruitment and placement" violates due
process. Appellant also aver, that she was denied the equal protection of the laws.

Due process requires that the terms of a penal statute must be sufficiently explicit to inform those who are
subject to it what conduct on their part will render them liable to its penalties. As a rule, a statute or act may be
said to be vague when it lacks comprehensible standards that men "of common intelligence must necessarily
guess at its meaning and differ as to its application." It is repugnant to the Constitution in two respects: (1) it
violates due process for failure to accord persons, especially the parties targeted by it, fair notice of the conduct
to avoid; and (2) it leaves law enforcers unbridled discretion in carrying out its provisions and become an
arbitrary flexing of the Government muscle.

We added, however, that:


x x x the act must be utterly vague on its face, that is to say, it cannot be clarified by either a saving
clause or by construction.

As we see it, the proviso (see ARTICLE 13(B) of the Illegal Recruitment Law) was intended neither to impose a
condition on the basic rule nor to provide an exception thereto but merely to create a presumption. The
presumption is that the individual or entity is engaged in recruitment and placement whenever he or it is dealing
with two or more persons to whom, in consideration of a fee, an offer or promise of employment is made in the
course of the "canvassing, enlisting, contracting, transporting, utilizing, hiring or procuring (of) workers."

The number of persons dealt with is not an essential ingredient of the act of recruitment and placement
of workers. Any of the acts mentioned in the basic rule in Article 13(b) will constitute recruitment and placement
even if only one prospective worker is involved. The proviso merely lays down a rule of evidence that where a
fee is collected in consideration of a promise or offer of employment to two or more prospective workers, the
individual or entity dealing with them shall be deemed to be engaged in the act of recruitment and placement.
The words "shall be deemed" create that presumption.

2. THERE IS SIMPLE ILLEGAL RECRUITMENT

In this case, the first element of illegal recruitment is present. The certification of POEA Officer-in-Charge
Macarulay states that appellant is not licensed or authorized to engage in recruitment and placement.

The second element is also present. Appellant is presumed engaged in recruitment and placement under
Article 13 (b) of the Labor Code. Both Nancy Araneta and Lourdes Modesto testified that appellant promised
them employment for a fee. Their testimonies corroborate each other on material points: the briefing conducted
by appellant, the time and place thereof, t he fees involved.

Affirmative testimony of persons who are eyewitnesses of the fact asserted easily overrides negative testimony.

That appellant did not receive any payment for the promised or offered employment is of no moment.
From the language of the statute, the act of recruitment may be "for profit or not;" it suffices that the
accused "promises or offers for a fee employment" to warrant conviction for illegal recruitment.

Considering that the two elements of lack of license or authority and the undertaking of an activity
constituting recruitment and placement are present, appellant, at the very least, is liable for "simple"
illegal recruitment.

San Beda College of Law 73


4S: 2015 - 2016
LABOR LAW REVIEW Atty. Joyrich Golangco

She is not guilty of illegal recruitment in a large scale
A conviction for large scale illegal recruitment must be based on a finding in each case of illegal
recruitment of three or more persons whether individually or as a group. In this case, only two persons,
Araneta and Modesto, were proven to have been recruited by appellant. The third person named in the
complaint as having been promised employment for a fee, Jennelyn Baez, was not presented in court to
testify.

It is true that law does not require that at least three victims testify at the trial; nevertheless, it is necessary
that there is sufficient evidence proving that the offense was committed against three or more persons. In
this case, evidence that appellant likewise promised her employment for a fee is sketchy. The only
evidence that tends to prove this fact is the testimony of Nancy Araneta, who said that she and her
friends, Baez and Sandra Aquino, came to the briefing and that they (she and her "friends") filled up
application forms.

Baez affidavit executed with Araneta cannot support Aranetas testimony. Insofar as it purports to prove
that appellant recruited Baez, therefore, the affidavit is hearsay and inadmissible.

Neither can appellant be convicted for recruiting CIS agent Eileen Fermindoza or even the other persons
present in the briefing of January 30, 1994. Appellant is accused of recruiting only the three persons
named in the information Araneta, Modesto and Baez. The information does not include Fermindoza or
the other persons present in the briefing as among those promised or offered employment for a fee.

Section 19 (1), Article III of the Constitution states: "Excessive fines shall not be imposed, nor cruel,
degrading or inhuman punishment inflicted."

The penalty of life imprisonment imposed upon appellant must be reduced. Because the prosecution was
able to prove that appellant committed recruitment and placement against two persons only, she cannot be
convicted of illegal recruitment in large scale, which requires that recruitment be committed against three or
more persons.

Appellant can only be convicted of two counts of "simple" illegal recruitment, one for that committed against
Nancy Araneta, and another count for that committed against Lourdes Modesto. Appellant is sentenced, for
each count, to suffer the penalty of four (4) to six (6) years of imprisonment and to pay a fine of P30,000.00.

WHEREFORE, the decision of the regional trial court is MODIFIED. Appellant is hereby declared guilty of illegal
recruitment on two (2) counts and is sentenced, for each count, to suffer the penalty of four (4) to six (6) years of
imprisonment and to pay a fine of P30,000.00.

San Beda College of Law 74


4S: 2015 - 2016
LABOR LAW REVIEW Atty. Joyrich Golangco

10. ESTATE OF NELSON R. DULAY, represented by his wife MERRIDY JANE P. DULAY v. ABOITIZ
JEBSEN MARITIME, INC. and GENERAL CHARTERERS, INC.,
G.R. No. 172642 June 13, 2012
PERALTA, J.

Doctrine:
With respect to disputes involving claims of Filipino seafarers wherein the parties are covered by a collective
bargaining agreement, the dispute or claim should be submitted to the jurisdiction of a voluntary arbitrator or
panel of arbitrators. It is only in the absence of a collective bargaining agreement that parties may opt to submit
the dispute to either the NLRC or to voluntary arbitration. It is elementary that rules and regulations issued by
administrative bodies to interpret the law which they are entrusted to enforce, have the force of law, and are
[8]
entitled to great respect. Such rules and regulations partake of the nature of a statute and are just as binding
[9]
as if they have been written in the statute itself. In the instant case, the Court finds no cogent reason to depart
from this rule.

FACTS:
Nelson R. Dulay was employed by General Charterers Inc. (GCI), a subsidiary of co-petitioner Aboitiz Jebsen
Maritime Inc. since 1986. He initially worked as an ordinary seaman and later as bosun on a contractual basis.
From September 3, 1999 up to July 19, 2000, Nelson was detailed in petitioners vessel, the MV Kickapoo Belle.

After the completion of his employment contract, Nelson died due to acute renal failure secondary to septicemia.
At the time of his death, Nelson was a bona fide member of the Associated Marine Officers and Seamans
Union of the Philippines (AMOSUP), GCIs collective bargaining agent.

Nelsons widow, Merridy Jane, thereafter claimed for death benefits through the grievance procedure of the
Collective Bargaining Agreement (CBA) between AMOSUP and GCI. However, the grievance procedure was
declared deadlocked as petitioners refused to grant the benefits sought by the widow. Merridy Jane filed a
complaint with the NLRC Sub-Regional Arbitration Board in General Santos City against GCI for death and
medical benefits and damages. The amount claimed by Nelsons widow is $90,000 however GCI awarded
P20,000 in favor of the deceaseds brother. Merridy claims the remaining amount less the P20,000 her brother-
in-law received.

Respondents on the other hand, asserted that the NLRC had no jurisdiction over the action on account of the
absence of employer-employee relationship between GCI and Nelson at the time of the latters death. Nelson
also had no claims against petitioners for sick leave allowance/medical benefit by reason of the completion of
his contract with GCI. They further alleged that private respondent is not entitled to death benefits because
petitioners are only liable for such in case of death of the seafarer during the term of his contract pursuant to the
POEA contract and the cause of his death is not work-related. Petitioners admitted liability only with respect to
article 20(A) 2 [of the CBA].

ISSUE: Whether or not the Labor Arbiter has jurisdiction over the case.

LA RULING: The Labor Arbiter ruled in favor of private respondent. It took cognizance of the case by virtue of
Article 217 (a), paragraph 6 of the Labor Code and the existence of a reasonable causal connection between
the employer-employee relationship and the claim asserted. It ordered the petitioner to pay P4,621,300.00, the
equivalent of US$90,000.00 less P20,000.00, at the time of judgment. The Labor Arbiter also ruled that the
proximate cause of Nelsons death was not work-related.

NLRC RULING: On appeal, [the NLRC] affirmed the Labor Arbiters decision as to the grant of death
[3]
benefits under the CBA but reversed the latters ruling as to the proximate cause of Nelsons death.

CA RULING: The CA ruled that while the suit filed by Merridy Jane is a money claim, the same basically
involves the interpretation and application of the provisions in the subject CBA. As such, jurisdiction belongs to
the voluntary arbitrator and not the labor arbiter.

SC RULING:
JURISDICTION BELONGS TO THE VOLUNTARY ARBITRATOR AND NOT THE LABOR ARBITER.

San Beda College of Law 75


4S: 2015 - 2016
LABOR LAW REVIEW Atty. Joyrich Golangco

The Court agrees with petitioner's contention that the CBA is the law or contract between the parties. Article
13.1 of the CBA entered into by and between respondent GCI and AMOSUP, the union to which petitioner
belongs, provides as follows:

The Company and the Union agree that in case of dispute or conflict in the interpretation or application
of any of the provisions of this Agreement, or enforcement of Company policies, the same shall be
settled through negotiation, conciliation or voluntary arbitration.

In the same manner, Section 29 of the prevailing Standard Terms and Conditions Governing the Employment of
Filipino Seafarers on Board Ocean Going Vessels, promulgated by the Philippine Overseas Employment
Administration (POEA), provides as follows:

Section 29. Dispute Settlement Procedures. In cases of claims and disputes


arising from this employment, the parties covered by a collective bargaining
agreement shall submit the claim or dispute to the original and exclusive
jurisdiction of the voluntary arbitrator or panel of arbitrators.

It is clear from the above that the interpretation of the DOLE, in consultation with their counterparts in the
respective committees of the Senate and the House of Representatives, as well as the DFA and the POEA is
that with respect to disputes involving claims of Filipino seafarers wherein the parties are covered by a CBA, the
dispute or claim should be submitted to the jurisdiction of a voluntary arbitrator or panel of arbitrators. It is only in
the absence of a CBA that parties may opt to submit the dispute to either the NLRC or to voluntary arbitration.

On the basis of the foregoing, the Court finds no error in the ruling of the CA that the voluntary arbitrator
has jurisdiction over the instant case.

San Beda College of Law 76


4S: 2015 - 2016
LABOR LAW REVIEW Atty. Joyrich Golangco

11. SANTIAGO vs. CF SHARP CREW MANAGEMENT INC.
G.R. No. 162419 July 10, 2007
Tinga, J.:

Doctrine:
The jurisdiction of labor arbiters is not limited to claims arising from employer-employee relationships. Under
Section 10 of R.A. No. 8042 the Labor Arbiters of the National Labor Relations Commission (NLRC) shall also
have the original and exclusive jurisdiction to hear and decide, the claims arising xxx by virtue of any law or
contract involving Filipino workers for overseas deployment including claims for actual, moral, exemplary and
other forms of damages.

FACTS:
On 3 February 1998, Paul Santiago signed a new contract of employment with CF Sharp Crew Mgmt., Inc., with
the duration of nine (9) months. He was assured of a monthly salary of US$515.00, overtime pay and other
benefits. Santiago was to be deployed on board the "MSV Seaspread" which was scheduled to leave the port of
Manila for Canada on 13 February 1998.

A week before the scheduled date of departure, Capt. Pacifico Fernandez, CF Sharps Vice President, sent a
fax to the captain of "MSV Seaspread telling the latter that he received calls from various individuals about the
possibility that Santiago may jump ship in Canada like his brother did before him. On 9 February 1998,
Santiago was thus told that he would not be leaving for Canada anymore, but he was reassured that he might
be considered for deployment at some future date.

Consequently, Santiago filed a complaint for illegal dismissal, damages, and attorney's fees against CF Sharp
and its foreign principal. In defense, CF Sharp contends that there is no employer-employee relationship
between petitioner and respondent because under the POEA Standard Contract, the employment contract shall
commence upon actual departure of the seafarer from the airport or seaport at the point of hire. In the absence
of an employer-employee relationship between the parties, the claims for illegal dismissal, actual damages, and
attorneys fees should be dismissed as the NLRC does not have jurisdiction over the same.

ISSUE: WON the failure of CF Sharp to deploy Santiago without a valid contract entitles the latter to relief
sought despite the non-commencement of the employer-employee relationship?

SC RULING:
YES. The jurisdiction of labor arbiters is not limited to claims arising from employer-employee relationships.
Section 10 of R.A. No. 8042 (Migrant Workers Act), provides that the Labor Arbiters of the National Labor
Relations Commission (NLRC) shall have the original and exclusive jurisdiction to hear and decide, the claims
arising xxx by virtue of any law or contract involving Filipino workers for overseas deployment including claims
for actual, moral, exemplary and other forms of damages.

Here, since the present petition involves the employment contract entered into by petitioner for overseas
employment, his claims are cognizable by the labor arbiters of the NLRC.

Even before the start of any employer-employee relationship, contemporaneous with the perfection of the
employment contract was the birth of certain rights and obligations, the breach of which may give rise to a cause
of action against the erring party. Thus, if the reverse had happened, that is the seafarer failed or refused to be
deployed as agreed upon, he would be liable for damages.

San Beda College of Law 77


4S: 2015 - 2016
LABOR LAW REVIEW Atty. Joyrich Golangco

12. STO. TOMAS vs. SALAC
G.R. No. 152642 November 13, 2012
Abad, J.:

Doctrine:
Sections 29 and 30 have been repealed by R.A. No. 9422, thereby putting private overseas recruitment
activities under government regulation. On the other hand, Sections 6, 7, 9 and the last sentence of par.2, Sec.
10 of R.A. No. 8042 are not unconstitutional.

FACTS:
These are consolidated cases pertaining to questions being raised on the constitutionality of certain provisions
of R.A. No. 8042 or the Migrant Workers Act.

The first case questions the constitutionality of Sections 29 and 30 deregulating the business of handling the
recruitment and migration of overseas Filipino workers and phasing out within five years the regulatory functions
of the Philippine Overseas Employment Administration (POEA).

The second, on the other hand, questioned Sections 6, 7 and 9 of the same law. The issue raised as to Sec. 6
was that that its definition of "illegal recruitment" is vague as it fails to distinguish between licensed and non-
licensed recruiter and for that reason gives undue advantage to the non-licensed recruiters in violation of the
right to equal protection of those that operate with government licenses or authorities.

As to Sec. 7, it was held by an RTC as unconstitutional on the ground that its sweeping application of the
penalties failed to make any distinction as to the seriousness of the act committed for the application of the
penalty imposed on such violation. As an example, said the trial court, the mere failure to render a report under
Section 6(h) or obstructing the inspection by the Labor Department under Section 6(g) are penalized by
imprisonment for six years and one day and a minimum fine of P200,000.00 but which could unreasonably go
even as high as life imprisonment if committed by at least three persons.

The same RTC also held invalid Sec. 9 for on the ground that allowing the offended parties to file the criminal
case in their place of residence would negate the general rule on venue of criminal cases which is the place
where the crime or any of its essential elements were committed. Venue, said the RTC, is jurisdictional in penal
laws and, allowing the filing of criminal actions at the place of residence of the offended parties violates their
right to due process.

The third and final case questions the constitutionality of the last sentence of the par.2, Sec. 10, R.A. No. 8042,
which holds the corporate directors, officers and partners jointly and solidarily liable with their company for
money claims filed by OFWs against their employers and the recruitment firms.

ISSUE: WON the assailed provisions of the law are constitutional.

SC RULING:
YES. As to the first case, the issue has been rendered moot with the passage into law of R.A. 9422 which
expressly repealed Sections 29 and 30 of R.A. 8042 and adopted the policy of close government regulation of
the recruitment and deployment of OFWs.

Coming to the second case, illegal recruitment" as defined in Section 6 is clear and unambiguous and, contrary
to the RTCs finding, actually makes a distinction between licensed and non-licensed recruiters. By its terms,
persons who engage in "canvassing, enlisting, contracting, transporting, utilizing, hiring, or procuring workers"
without the appropriate government license or authority are guilty of illegal recruitment whether or not they
commit the wrongful acts enumerated in that section. On the other hand, recruiters who engage in the
canvassing, enlisting, etc. of OFWs, although with the appropriate government license or authority, are guilty of
illegal recruitment only if they commit any of the wrongful acts enumerated in Section 6.

Neither is Sec. 7 unconstitutional as the law can impose such grave penalties upon what it believed were
specific acts that were not as condemnable as the others in the lists. But, in fixing uniform penalties for each of
the enumerated acts under Section 6, Congress was within its prerogative to determine what individual acts are
equally reprehensible, consistent with the State policy of according full protection to labor, and deserving of the
same penalties. It is not within the power of the Court to question the wisdom of this kind of choice.
San Beda College of Law 78
4S: 2015 - 2016
LABOR LAW REVIEW Atty. Joyrich Golangco

As to Sec. 9, there is nothing arbitrary or unconstitutional in Congress fixing an alternative venue for violations of
Section 6 of R.A. 8042 that differs from the venue established by the Rules on Criminal Procedure. Indeed,
Section 15(a), Rule 110 of the latter Rules allows exceptions provided by laws. Section 9 of R.A. 8042, as an
exception to the rule on venue of criminal actions is, consistent with that laws declared policy of providing a
criminal justice system that protects and serves the best interests of the victims of illegal recruitment.

On to the third case, the Court has already held that the liability of corporate directors and officers is not
automatic. To make them jointly and solidarily liable with their company, there must be a finding that they were
remiss in directing the affairs of that company, such as sponsoring or tolerating the conduct of illegal activities.

13. SAMEER OVERSEAS PLACEMENT AGENCY, INC. vs. JOY C. CABILES


G.R. No. 170139 August 5, 2014
Leonen, J.:

Doctrine:
The re-enactment of the last paragraph of Sec. 10 of R.A. No. 8042 did not erase its unconstitutionality and a
central bank circular on interest rates cannot repeal a positive provision R.A. No. 8042, as regards interest rates
on certain pecuniary awards granted to illegally dismissed OFWs.

FACTS:
Joy C. Cabiles, submitted her application for a quality control job in Taiwan via the Sameer Overseas Placement
Agency, Inc. Joys application was accepted. Joy was later asked to sign a one-year employment contract for a
monthly salary of NT$15,360.00. She alleged that Sameer Overseas Agency required her to pay a placement
fee of P70,000.00 when she signed the employment contract.

Joy was deployed to work for Taiwan Wacoal, Co. Ltd. (Wacoal) on June 26, 1997. Sameer Overseas
Placement Agency claims that on July 14, 1997, Wacoal informed Joy, without prior notice, that she was
terminated and that "she should immediately report to their office to get her salary and passport." She was
asked to "prepare for immediate repatriation."

Consequently, Joy filed a complaint with the NLRC against Sameer and Wacoal. She claimed that she was
illegally dismissed. She asked for the return of her placement fee, the withheld amount for repatriation costs,
payment of her salary for 23 months as well as moral and exemplary damages. Sameer, on the other hand,
posits among others that Petitioner Wacoal's accreditation with it had already been transferred to the Pacific
Manpower & Management Services, Inc. (Pacific) as of August 6, 1997. Thus, it asserts that it was already
substituted by Pacific Manpower.

Joy won before the NLRC and the CA who both awarded Joy a mere (3) months worth of salary,
reimbursement of withheld repatriation expense, and attorneys fees. Sameer brought a petition for review with
the SC. Brought to the fore is the issue on the constitutionality of the last paragraph of Sec. 10 of R.A. No. 8042
which has been declared unconstitutional in Serrano v. Gallant Maritime but re-enacted by legislation
subsequent to Serrano. Also brought for discussion is the proper application of the newly imposed interest rates
on the pecuniary awards given to Joy in view of Central Bank Circular No. 799.

ISSUE(S): 1. WON the award of 3-months worth of salary to Joy was legal in view of the re-enactment of the
last paragraph of Sec. 10 of R.A. No. 8042 which was previously declared as unconstitutional.

2. WON the new Central Bank circular on interest rates applies to the award of reimbursement of
placement fee and other deductions.

SC RULING:
1. NO. the reinstated clause, this time as provided in Republic Act. No. 10022, violates the constitutional rights
to equal protection and due process.Sameer as well as the Solicitor General have failed to show any compelling
change in the circumstances that would warrant us to revisit the precedent. Again, limiting wages that should be
recovered by an illegally dismissed overseas worker to three months is both a violation of due process and the
equal protection clauses of the Constitution.

The adoption of the reinstated clause in Republic Act No. 8042 subjected the money claims of illegally
dismissed overseas workers with an unexpired term of at least a year to a cap of three months worth of their
San Beda College of Law 79
4S: 2015 - 2016
LABOR LAW REVIEW Atty. Joyrich Golangco

salary. There was no such limitation on the money claims of illegally terminated local workers with fixed-term
employment.

Also, the subject clause creates a sub-layer of discrimination among OFWs whose contract periods are for more
than one year: those who are illegally dismissed with less than one year left in their contracts shall be entitled to
their salaries for the entire unexpired portion thereof, while those who are illegally dismissed with one year or
more remaining in their contracts shall be covered by the reinstated clause, and their monetary benefits limited
to their salaries for three months only

These classifications do not rest on any real or substantial distinctions that would justify different treatments in
terms of the computation of money claims resulting from illegal termination.

As such, Joy Cabiles is entitled to her salary for the unexpired portion of her contract, in accordance with
Section 10 of Republic Act No. 8042. The award of the three-month equivalence of her salary must be modified
accordingly. Since she started working on June 26, 1997 and was terminated on July 14, 1997, respondent is
entitled to her salary from July 15, 1997 to June 25, 1998.

2. NO. A Central Bank Circular cannot repeal a law. Only a law can repeal another law. For example, Section 10
of Republic Act No. 8042 provides that unlawfully terminated overseas workers are entitled to the
reimbursement of his or her placement fee with an interest of 12% per annum. Since Bangko Sentral ng
Pilipinas circulars cannot repeal Republic Act No. 8042, the issuance of Circular No. 799 does not have the
effect of changing the interest on awards for reimbursement of placement fees from 12% to 6%. This is despite
Section 1 of Circular No. 799, which provides that the 6% interest rate applies even to judgments.

However, the same cannot be said for awards of salary for the unexpired portion of the employment contract
under Republic Act No. 8042. These awards are covered by Circular No. 799 because the law does not provide
for a specific interest rate that should apply.

San Beda College of Law 80


4S: 2015 - 2016
LABOR LAW REVIEW Atty. Joyrich Golangco

14. MAERSK-FILIPINAS CREWING, INC., A.P. MOLLER SINGAPORE PTE. LIMITED, AND JESUS
AGBAYANI vs. TORIBIO C. AVESTRUZ
G.R. No. 207010 February 18, 2015

Doctrine
Except in cases of clear and existing danger to the crew or vessel, an erring seaman should be given a written
notice of the charge against him and should be afforded an opportunity to explain or defend himself. Should
sanctions be imposed, then a written notice of penalty and the reasons for it shall be furnished the erring
seafarer. In all cases, a complete report should be sent to the manning agency, supported by substantial
evidence of the findings.

FACTS:
Maersk-Filipinas Crewing, Inc. (Maersk), on behalf of its foreign principal hired Avestruz as Chief Cook on board
the vessel M/V Nedlloyd Drake for a period of six (6) months
Avestruz boarded the vessel on May 4, 2011.

On June 22, 2011, in the course of the weekly inspection of the vessels galley, Captain Charles C. Woodward
(Captain Woodward) noticed that the cover of the garbage bin in the kitchen near the washing area was oily.
Woodward called Avestruz. An altercation ensued.

On the very same day, Captain Woodward informed Avestruz that he would be dismissed from service and be
disembarked in India. On July 3, 2011, Avestruz was disembarked in Colombo, Sri Lanka and arrived in the
Philippines on July 4, 2011.

Consequently, Avestruz filed a complaint for illegal dismissal, payment for the unexpired portion of his contract,
damages, and attorneys fees against Maersk et al. and that Captain Woodward failed to observe the provisions
under Section 17 of the Philippine Overseas Employment Administration (POEA) Standard Employment
Contract (POEA-SEC) on disciplinary procedures. Maersk maintained that Avestruz was dismissed for a just
and valid cause and is, therefore, not entitled to recover his salary for the unexpired portion of his contract. They
contend that Avestruz was dismissed on the ground of insubordination, consisting of his repeated failure to
obey his superiors order to maintain cleanliness in the galley of the vessel as well as his act of insulting a
superior officer by words or deeds. In support of this contention, petitioners presented as evidence the e-mails
sent by Captain Woodward, both dated June 22, 2011, and time-stamped 10:07 a.m. and 11:40 a.m.,
respectively

The CA held for Avestruz and directed petitioners to pay him, jointly and severally, the full amount of his
placement fee and deductions made, with interest at twelve percent (12%) per annum, as well as his salaries for
the unexpired portion of his contract, and attorneys fees of ten percent (10%) of the total award. All other
money claims were denied for lack of merit.

ISSUE(S): 1. WON Avestruz dismissal was proper pursuant to the POEA-SEC.


2. WON the pecuniary awards given to Avestruz was proper.

SC RULING:
1. NO. An erring seaman is given a written notice of the charge against him and is afforded an opportunity to
explain or defend himself. Should sanctions be imposed, then a written notice of penalty and the reasons for it
shall be furnished the erring seafarer. It is only in the exceptional case of clear and existing danger to the safety
of the crew or vessel that the required notices are dispensed with; but just the same, a complete report should
be sent to the manning agency, supported by substantial evidence of the findings.

In this case, there is dearth of evidence to show that Avestruz had been given a written notice of the charge
against him, or that he was given the opportunity to explain or defend himself. The statement given by Captain
Woodward requiring him to explain in writing the events that transpired at the galley in the morning of June 22,
2011 hardly qualifies as a written notice of the charge against him, nor was it an opportunity for Avestruz to
explain or defend himself. While Captain Woodward claimed in his e-mail that he conducted a disciplinary
hearing informing Avestruz of his inefficiency, no evidence was presented to support the same. Neither was
Avestruz given a written notice of penalty and the reasons for its imposition.

2. YES. It is in consonance with Section 10 of RA 8042, as amended by RA 10022, which reads:


San Beda College of Law 81
4S: 2015 - 2016
LABOR LAW REVIEW Atty. Joyrich Golangco

Section 10. Money claims. x x x

x x x x In case of termination of overseas employment without just, valid or authorized cause as defined
by law or contract, or any unauthorized deductions from the migrant workers salary, the worker shall be
entitled to the full reimbursement of his placement fee and the deductions made with interest at twelve
percent (12%) per annum, plus his salaries for the unexpired portion of his employment contract or for
three (3) months for every year of the unexpired term, whichever is less.

Similarly, the Court affirmed the grant of attorneys fees of ten percent (10%) of the total award. All other
monetary awards were denied for lack of merit.

San Beda College of Law 82


4S: 2015 - 2016
LABOR LAW REVIEW Atty. Joyrich Golangco

15. SUNACE INTERNATIONAL MANAGEMENT SERVICES, INC. vs. NLRC
G.R. No. 161757 January 25, 2006
Carpio-Morales, J.:

Doctrine:
The theory of imputed knowledge ascribes the knowledge of the agent to the principal not the other way around.
Also, the agency between a foreign principal and its local recruitment agent is revoked if the foreign principal
directly manages the business (hiring of employee) entrusted to the local recruitment agent, dealing directly with
third persons.

FACTS:
Sunace International Management Services (Sunace), deployed to Taiwan Divina A. Montehermozo (Divina) as
a domestic helper under a 12-month contract effective February 1, 1997. After her 12-month contract expired on
February 1, 1998, Divina continued working for her Taiwanese employer, Hang Rui Xiong, for two more years,
after which she returned to the Philippines on February 4, 2000.

Shortly after her return or on February 14, 2000, Divina filed a complaint before the National Labor Relations
Commission (NLRC) against Sunace and three others including the employer-foreign principal alleging that she
was jailed for three months and that she was underpaid.

Divina filed also claimed that under her original one-year contract and the 2-year extended contract which was
with the knowledge and consent of Sunace, amounts representing income tax and savings were deducted from
her salary and while the amounts deducted in 1997 were refunded to her, those deducted in 1998 and 1999
were not.

For its part, Sunace alleged that Divinas 2-year extension of her contract was without its knowledge and
consent, hence, it had no liability attaching to any claim arising therefrom, and Divina in fact executed a
Waiver/Quitclaim and Release of Responsibility and an Affidavit of Desistance. The CA affirmed the Labor
Arbiter and NLRCs finding that Sunace knew of and impliedly consented to the extension of Divinas 2-year
contract. It went on to state that It is undisputed that Sunace was continually communicating with Divinas
foreign employer. It thus concluded that as agent of the foreign principal, petitioner cannot profess ignorance of
such extension as obviously, the act of the principal extending complainants employment contract necessarily
bound it.

ISSUE: WON Sunace was solidarily liable with the foreign principal to Divina as to the events which transpired
during her 2-year extension in Taiwan.

SC RULING:
NO. The theory of imputed knowledge ascribes the knowledge of the agent, Sunace, to the principal, employer
Xiong, not the other way around. The knowledge of the principal-foreign employer cannot, therefore, be imputed
to its agent Sunace. Also, the agency is revoked if the principal directly manages the business entrusted to the
agent, dealing directly with third persons.

Here, there is no substantial proof that Sunace knew of and consented to be bound under the 2-year
employment contract extension, it cannot be said to be privy thereto. As such, it cannot be held solidarily liable
for any of Divinas claims arising from the 2-year employment extension. Furthermore, Sunace correctly points
out, there was an implied revocation of its agency relationship with its foreign principal when, after the
termination of the original employment contract, the foreign principal directly negotiated with Divina and entered
into a new and separate employment contract in Taiwan.

San Beda College of Law 83


4S: 2015 - 2016
LABOR LAW REVIEW Atty. Joyrich Golangco

ARTICLE 22-78

16. HON. PATRICIA A. STO.TOMAS, et al. vs. REY SALAC, et al.


G.R. No. 152642 November 13, 2012
ABAD, J.:

FACTS:
These consolidated cases pertain to the constitutionality of certain provisions of RA 8042, or the Migrant
Workers and Overseas Filipinos Act of 1995, enacted on June 7, 1995, which sets the Governments policies on
overseas employment and establishes a higher standard of protection and promotion of the welfare of migrant
workers, their families, and overseas Filipinos in distress.

*Constitutionality of Sections 29 and 30 of R.A. 8042


Sections 29 and 30 commanded the Department of Labor and Employment (DOLE) to begin deregulating within
one year of its passage the business of handling the recruitment and migration of OFWs and phase out within
five years the regulatory functions of the Philippine Overseas Employment Administration (POEA).

On January 8, 2002, Rey Salac, et al. filed a petition for certiorari, prohibition and mandamus with application for
TRO and preliminary injunction against Patricia Sto. Tomas as DOLE Secretary, et al. in RTC Quezon City.
They sought to: 1) nullify DOLE Department Order 10 and POEA Memorandum Circular 15; 2) prohibit the
DOLE, POEA, and TESDA from implementing the same and from further issuing rules and regulations that
would regulate the recruitment and placement of OFWs; and 3) also enjoin them to comply with the policy of
deregulation mandated under Sections 29 and 30 of Republic Act 8042.

On March 20, the Quezon City RTC granted Salac, et al.s petition and ordered the government agencies
mentioned to deregulate the recruitment and placement of OFWs. The RTC also annulled DOLE DO 10, POEA
MC 15, and all other orders, circulars and issuances that are inconsistent with the policy of deregulation under
R.A. 8042. Hence, the petition seeking to annul the RTCs decision and have the same enjoined pending action
on the petition.

On April 17, the Philippine Association of Service Exporters, Inc. intervened, claiming that Decision gravely
affected them since it paralyzed the deployment abroad of OFWs and performing artists. So did the
Confederated Association of Licensed Entertainment Agencies, Incorporated (CALEA). On May 23, the
Court issued a TRO, enjoining the Quezon City RTC from enforcing its decision.

In a parallel case, Asian Recruitment Council Philippine Chapter, Inc. and others (Arcophil, et al.) filed a petition
for certiorari and prohibition with application for TRO and preliminary injunction to enjoin the implementation of
the 2002 Rules and Regulations Governing the Recruitment and Employment of Overseas Workers and to
cease and desist from issuing other orders, circulars, and policies that tend to regulate the recruitment and
placement of OFWs in violation of the policy of deregulation provided in Sections 29 and 30 of R.A. 8042. It was
granted.

On December 4, 2008, however, the Republic informed the Court that on April 10, 2007 former President Gloria
Macapagal-Arroyo signed into law R.A. 9422 which expressly repealed Sections 29 and 30 of R.A. 8042 and
adopted the policy of close government regulation of the recruitment and deployment of OFWs.

The repeal of Sections 29 and 30 of R.A. 8042 renders the issues moot and academic. Hence, they should be
dismissed.

*Constitutionality of Sections 6, 7, and 9 of R.A. 8042


On August 21, 1995, PASEI filed a petition for declaratory relief and prohibition with prayer for issuance
of TRO and writ of preliminary injunction in RTC Manila, seeking to annul Sections 6, 7, and 9 of R.A. 8042 for
being unconstitutional. Section 6 defines the crime of "illegal recruitment" and enumerates the acts constituting
the same. Section 7 provides the penalties for prohibited acts. Section 9 allowed the filing of criminal actions
arising from "illegal recruitment" before the RTC of the province or city where the offense was committed or
where the offended party actually resides at the time of the commission of the offense.

San Beda College of Law 84


4S: 2015 - 2016
LABOR LAW REVIEW Atty. Joyrich Golangco

RTC Manila declared Section 6 unconstitutional on the ground that its definition of "illegal recruitment" is vague.
But "illegal recruitment" as defined in Section 6 is clear and unambiguous and, contrary to the RTCs finding,
actually makes a distinction between licensed and non-licensed recruiters.

RTC Manila also declared Section 7 unconstitutional on the ground that its sweeping application of the penalties
failed to make any distinction as to the seriousness of the act committed for the application of the penalty
imposed on such violation. But, in fixing uniform penalties for each of the enumerated acts under Section 6,
Congress was within its prerogative to determine what individual acts are equally reprehensible, consistent with
the State policy of according full protection to labor, and deserving of the same penalties. It is not within the
power of the Court to question the wisdom of this kind of choice. Notably, this legislative policy has been further
stressed in July 2010 with the enactment of R.A. 10022 which increased even more the duration of the penalties
of imprisonment and the amounts of fine for the commission of the acts listed under Section 7.

RTC Manila also invalidated Section 9 on the ground that allowing the offended parties to file the criminal case
in their place of residence would negate the general rule on venue of criminal cases which is the place where
the crime or any of its essential elements were committed. Venue, said the RTC, is jurisdictional in penal laws
and, allowing the filing of criminal actions at the place of residence of the offended parties violates their right to
due process. But there is nothing arbitrary or unconstitutional in Congress fixing an alternative venue for
violations. Section 9, as an exception to the rule on venue of criminal actions is consistent with that laws
declared policy of providing a criminal justice system that protects and serves the best interests of the victims of
illegal recruitment.

*Constitutionality of Section 10, last sentence of 2nd paragraph


Spouses Simplicio and Mila Cuaresma filed a claim for death and insurance benefits and damages against
Becmen Service Exporter and Promotion, Inc. and White Falcon Services, Inc. for the death of their daughter
Jasmin while working as staff nurse in Riyadh, Saudi Arabia.

The Labor Arbiter dismissed the claim. However, the National Labor Relations Commission found Becmen and
White Falcon jointly and severally liable for Jasmins death and ordered them to pay the Cuaresmas the amount
of US$113,000.00 as actual damages. The NLRC relied on the Cabanatuan City Health Offices autopsy finding
that Jasmin died of criminal violence and rape. CA upheld.

SC found Jasmins death not work-related or work-connected since her rape and death did not occur while she
was on duty at the hospital or doing acts incidental to her employment. The Court deleted the award of actual
damages but ruled that Becmens corporate directors and officers are solidarily liable with their company for its
failure to investigate the true nature of her death. Becmen and White Falcon abandoned their legal, moral, and
social duty to assist the Cuaresmas in obtaining justice for their daughter.

The corporate directors and officers of Becmen, namely, Eufrocina Gumabay, et al. filed a motion for leave to
Intervene. They questioned the constitutionality of the last sentence of the second paragraph of Section 10, R.A.
8042 which holds the corporate directors, officers and partners jointly and solidarily liable with their company for
money claims filed by OFWs against their employers and the recruitment firms.

The key issue that Gumabay, et al. present is whether or not the 2nd paragraph of Section 10, R.A. 8042, which
holds the corporate directors, officers, and partners of recruitment and placement agencies jointly and solidarily
liable for money claims and damages that may be adjudged against the latter agencies, is unconstitutional.

Absent sufficient proof that the corporate officers and directors of the erring company had knowledge of and
allowed the illegal recruitment, making them automatically liable would violate their right to due process of law.
The liability of corporate directors and officers is not automatic. To make them jointly and solidarily liable with
their company, there must be a finding that they were remiss in directing the affairs of that company, such as
sponsoring or tolerating the conduct of illegal activities. In the case of Becmen and White Falcon, while there is
evidence that these companies were at fault in not investigating the cause of Jasmins death, there is no
mention of any evidence in the case against them that intervenors Gumabay, et al., Becmens corporate officers
and directors, were personally involved in their companys particular actions or omissions in Jasmins case.

SC RULING:
R.A. 8042 is a police power measure intended to regulate the recruitment and deployment of OFWs. It aims to
curb, if not eliminate, the injustices and abuses suffered by numerous OFWs seeking to work abroad. The rule is
San Beda College of Law 85
4S: 2015 - 2016
LABOR LAW REVIEW Atty. Joyrich Golangco

settled that every statute has in its favor the presumption of constitutionality. The Court cannot inquire into the
wisdom or expediency of the laws enacted by the Legislative Department. Hence, in the absence of a clear and
unmistakable case that the statute is unconstitutional, the Court must uphold its validity.

San Beda College of Law 86


4S: 2015 - 2016
LABOR LAW REVIEW Atty. Joyrich Golangco

17. CENTURY CANNING CORPORATION vs. CA and GLORIA C. PALAD
G.R. No. 152894 August 17, 2007
CARPIO, J.:

FACTS:
On 15 July 1997, Century Canning Corporation hired Gloria C. Palad as fish cleaner at its tuna and sardines
factory. Palad signed an apprenticeship agreement and received an apprentice allowance of P138.75 daily. On
25 July, CCC submitted its apprenticeship program for approval to the Technical Education and Skills
Development Authority (TESDA) of the Department of Labor and Employment (DOLE). On 26 September, the
TESDA approved the same.

According to CCC, a performance evaluation was conducted on 15 November, where CCC gave Palad a rating
of N.I. or needs improvement since she scored only 27.75% based on a 100% performance indicator.
Furthermore, according to the performance evaluation, Palad incurred numerous tardiness and absences. As a
consequence, CCC issued a termination
noticehttp://sc.judiciary.gov.ph/jurisprudence/2007/august2007/152894.htm - _ftn5 dated 22 November to Palad,
informing her of her termination effective at the close of business hours of 28 November.

Palad then filed a complaint for illegal dismissal, underpayment of wages, and non-payment of pro-rated
th
13 month pay for the year 1997.

On 25 February 1999, the Labor Arbiter dismissed the complaint for lack of merit but ordered CCC to pay Palad
th
her last salary and her pro-rated 13 month pay. On appeal, the National Labor Relations Commission affirmed
with modification. Upon denial of Palads motion for reconsideration, Palad filed a special civil action
for certiorari with the CA. On 12 November 2001, CA set aside the decision of NLRC, and found that the
dismissal was illegal.

CA held that the apprenticeship agreement which Palad signed was not valid and binding because it was
executed more than two months before the TESDA approved CCCs apprenticeship program. CA also held that
CCC illegally dismissed Palad. CA ruled that CCC failed to show that Palad was properly apprised of the
required standard of performance. CA likewise held that Palad was not afforded due process because CCC did
not comply with the twin requirements of notice and hearing.

ISSUES:
1. Whether Palad was an apprentice. No
2. Whether CCC had adequately proven the existence of a valid cause in terminating the service of palad.
No

SC RULING:
The Labor Code defines an apprentice as a worker who is covered by a written apprenticeship agreement with
an employer. One of the objectives of Title II (Training and Employment of Special Workers) of the Labor Code
is to establish apprenticeship standards for the protection of apprentices. (Articles 60 and 61)

In Nitto Enterprises v. National Labor Relations Commission, the Court cited Article 61 of the Labor Code and
held that an apprenticeship program should first be approved by the DOLE before an apprentice may be hired,
otherwise the person hired will be considered a regular employee.

Based on the evidence, CCC did not comply with the requirements of the law. Prior approval by the Department
of Labor and Employment of the proposed apprenticeship program is a condition sine qua non before an
apprenticeship agreement can be validly entered into.

Hence, since the apprenticeship agreement between CCC and Palad has no force and effect in the absence of
a valid apprenticeship program duly approved by the DOLE, Palads assertion that she was hired not as an
apprentice deserves credence. She should rightly be considered as a regular employee of CCC as defined by
Article 280 of the Labor Code.

RA 7796, which created the TESDA, has transferred the authority over apprenticeship programs from the
Bureau of Local Employment of the DOLE to the TESDA. RA 7796 emphasizes TESDAs approval of the
apprenticeship program as a pre-requisite for the hiring of apprentices. Such intent is clear under Section 4.
San Beda College of Law 87
4S: 2015 - 2016
LABOR LAW REVIEW Atty. Joyrich Golangco

Clearly, the apprenticeship agreement was enforced even before the TESDA approved CCCs apprenticeship
program. Thus, the apprenticeship agreement is void because it lacked prior approval from the TESDA. The
requisite TESDA approval of the apprenticeship program prior to the hiring of apprentices was further
emphasized by the DOLE with the issuance of Department Order No. 68-04 on 18 August 2004.

Since Palad is not considered an apprentice because the apprenticeship agreement was enforced before the
TESDAs approval of CCCs apprenticeship program, Palad is deemed a regular employee performing the job of
a fish cleaner. Clearly, the job of a fish cleaner is necessary in CCCs business as a tuna and sardines factory.

No clear and sufficient evidence exist to warrant Palads dismissal as an apprentice during the agreed
period. Besides the absence of any written warnings given to Palad reminding her of poor performance, CCCs
evidence in this respect consisted of an indecipherable or unauthenticated xerox of the performance evaluation
allegedly conducted on Palad. This is of doubtful authenticity and/or credibility.

Under Article 227 of the Labor Code, the employer has the burden of proving that the termination was for a valid
or authorized cause. CCC failed to substantiate its claim that Palad was terminated for valid reasons. It was
likewise not shown that CCC ever apprised Palad of the performance standards set by the company. When the
alleged valid cause for the termination of employment is not clearly proven, as in this case, the law considers
the matter a case of illegal dismissal. Furthermore, Palad was not accorded due process. CCC failed to warn
Palad of her alleged poor performance. The records are bereft of evidence to show that petitioner ever gave
Palad the opportunity to explain and defend herself. Clearly, the two requisites for a valid dismissal are lacking
in this case. CA decision AFFIRMED.

San Beda College of Law 88


4S: 2015 - 2016
LABOR LAW REVIEW Atty. Joyrich Golangco

18. MARITES BERNARDO, et al. vs. NATIONAL LABOR RELATIONS COMMISSION & FAR EAST BANK
AND TRUST COMPANY
G.R. No. 122917; July 12, 1999
PANGANIBAN, J.:

FACTS:
Complainants Marites Bernardo, et al., numbering 43, are deaf-mutes who were hired on various periods from
1988 to 1993 by Far East Bank and Trust Co. as Money Sorters and Counters through a uniformly worded
agreement called Employment Contract for Handicapped Workers.

By the time this case arose, there were 56 deaf-mutes who were employed by FEBTC under the said
employment agreement. Disclaiming that complainants were regular employees, FEBTC maintained that
complainants who are a special class of workers - the hearing impaired employees were hired temporarily under
a special employment arrangement which was a result of overtures made by some civic and political
personalities to FEBTC; that complainants were hired due to pakiusap which must be considered in the light of
the context of FEBTCs corporate philosophy; that the idea of hiring handicapped workers was acceptable to
them only on a special arrangement basis; that it adopted the special program to help tide over a group of
handicapped workers such as deaf-mutes like the complainants who could do manual work for the Bank, among
others.

The labor arbiter and, on appeal, the NLRC ruled against the complainants petition to be deemed regular
employees under Art. 280 of the Labor Code because their task as money sorters and counters was necessary
and desirable to the business of the bank. Hence, the recourse to the Supreme Court.

NLRC said that Art. 280 is not controlling herein as complainants were hired as an accommodation to the
recommendation of civic oriented personalities and the contracts of employment were with special provisions on
duration as specified under Art. 80. The NLRC also declared that the Magna Carta for Disabled Persons was
not applicable, considering the prevailing circumstances/milieu of the case.

ISSUE: Whether the NLRC committed grave abuse of discretion in holding that the bernardo, et al. were not
regular employees.

SC RULING:
YES. The employees, who worked for more than six months and whose contracts were renewed, are deemed
regular. Hence, their dismissal from employment was illegal. The Supreme Court appreciates the nobility of
FEBTC effort to provide employment to physically impaired individuals and to make them more productive
members of society. However, SC cannot allow it to elude the legal consequences of that effort, simply because
it now deems their employment irrelevant. The facts, viewed in light of the Labor Code and the Magna Carta for
Disabled Persons, indubitably show that the Bernardo, et al., except 16 of them, should be deemed regular
employees. As such, they have acquired legal rights that SC is duty-bound to protect and uphold, not as a
matter of compassion but as a consequence of law and justice.

The stipulations in the employment contracts indubitably conform with Art. 80 of the Labor Code. Succeeding
events and the enactment of RA No. 7277, however, justify the application of Article 280. Verily, the renewal of
the contracts of the handicapped workers and the hiring of others lead to the conclusion that their tasks were
beneficial and necessary to the bank. More important, these facts show that they were qualified to perform the
responsibilities of their positions. In other words, their disability did not render them unqualified or unfit for the
tasks assigned to them.

In this light, the Magna Carta for Disabled Persons mandates that a qualified disabled employee should be
given the same terms and conditions of employment as a qualified able-bodied person. (Section 5: Equal
Opportunity for Employment)

The fact that the employees were qualified disabled persons necessarily removes the employment contracts
from the ambit of Article 80. Since the Magna Carta accords them the rights of qualified able-bodied persons,
they are thus covered by Article 280 of the Labor Code.

San Beda College of Law 89


4S: 2015 - 2016
LABOR LAW REVIEW Atty. Joyrich Golangco

Without a doubt, the task of counting and sorting bills is necessary and desirable to the business of
FEBTC. With the exception of 16 of them, Bernardo, et al. performed these tasks for more than six
months. Thus, they should be deemed regular employees.

Because FEBTC failed to show such cause, Bernardo, et al. are deemed illegally dismissed and therefore
entitled to back wages and reinstatement without loss of seniority rights and other privileges. But considering the
allegation of FEBTC that the job of money sorting is no longer available because it has been assigned back to
the tellers to whom it originally belonged, Bernardo, et al. are awarded separation pay in lieu of reinstatement.

The noble objectives of Magna Carta for Disabled Persons are not based merely on charity or accommodation,
but on justice and the equal treatment of qualified persons, disabled or not. In the present case, the handicap is
not a hindrance to their work. The eloquent proof of this statement is the repeated renewal of their employment
contracts. The Court believes, that, after showing their fitness for the work assigned to them, they should be
treated and granted the same rights like any other regular employees. Petition GRANTED.

San Beda College of Law 90


4S: 2015 - 2016
LABOR LAW REVIEW Atty. Joyrich Golangco

Art. 82 (EMPLOYER-EMPLOYEE RELATIONSHIP)

19. ANGELINA FRANCISCO vs. NATIONAL LABOR RELATIONS COMMISSION


G.R. No. 170087 August 31, 2006
YNARES-SANTIAGO, J.:

Doctrine:
When the control test is not sufficient to give a complete picture of the relationship between the parties, two-
tiered test must be applied.

The proper standard of economic dependence is whether the worker is dependent on the alleged employer for
his continued employment in that line of business.

FACTS:
In 1995, petitioner was hired by Kasei Corporation during its incorporation stage. She was designated as
Accountant and Corporate Secretary and was assigned as Liaison Officer. In 1996, petitioner was designated as
Acting Manager and was assigned to handle recruitment of all employees and perform management
administration functions; represent the company in all dealings with government agencies, especially with the
BIR, SSS and in the city government of Makati; and to administer all other matters pertaining to the operation of
Kasei Restaurant which is owned and operated by Kasei Corporation. For five years, petitioner performed the
duties of Acting Manager. Her salary was P27,500.00 plus P3,000.00 housing allowance and a 10% share in the
profit of Kasei Corporation.

In January 2001, petitioner was replaced as Manager and reduced her salary by P2,500.00 a month. On
October 15, 2001, petitioner was informed that she is no longer connected with the company.

ISSUE: Whether or not there is an employer-employee relationship between petitioner and Kasei Corp.

SC RULING:
YES, there is an employer-employee relationship between petitioner and Kasei Corporation.

There are instances when, aside from the employers power to control the employee with respect to the means
and methods by which the work is to be accomplished, economic realities of the employment relations help
provide a comprehensive analysis of the true classification of the individual, whether as employee, independent
contractor, corporate officer or some other capacity.

The better approach would therefore be to adopt a two-tiered test involving: (1) the putative employers power to
control the employee with respect to the means and methods by which the work is to be accomplished; and (2)
the underlying economic realities of the activity or relationship.

The determination of the relationship between employer and employee depends upon the circumstances of the
whole economic activity, such as: (1) the extent to which the services performed are an integral part of the
employers business; (2) the extent of the workers investment in equipment and facilities; (3) the nature and
degree of control exercised by the employer; (4) the workers opportunity for profit and loss; (5) the amount of
initiative, skill, judgment or foresight required for the success of the claimed independent enterprise; (6) the
permanency and duration of the relationship between the worker and the employer; and (7) the degree of
dependency of the worker upon the employer for his continued employment in that line of business.

The proper standard of economic dependence is whether the worker is dependent on the alleged employer for
his continued employment in that line of business.
By applying the control test, there is no doubt that petitioner is an employee of Kasei Corporation
because she was under the direct control and supervision of Seiji Kamura, the corporations Technical
Consultant.

Under the broader economic reality test, the petitioner can likewise be said to be an employee of respondent
corporation because she had served the company for six years before her dismissal, receiving check vouchers
indicating her salaries/wages, benefits, 13th month pay, bonuses and allowances, as well as deductions and
Social Security contributions from August 1, 1999 to December 18, 2000. It is therefore apparent that petitioner

San Beda College of Law 91


4S: 2015 - 2016
LABOR LAW REVIEW Atty. Joyrich Golangco

is economically dependent on respondent corporation for her continued employment in the latters line of
business.

Based on the foregoing, there can be no other conclusion that petitioner is an employee of respondent Kasei
Corporation. She was selected and engaged by the company for compensation, and is economically dependent
upon respondent for her continued employment in that line of business.

San Beda College of Law 92


4S: 2015 - 2016
LABOR LAW REVIEW Atty. Joyrich Golangco

20. JOSE Y. SONZA vs. ABS-CBN BROADCASTING CORPORATION
G.R. No. 138051 June 10, 2004

Doctrine:
The control test is the most important test our courts apply in distinguishing an employee from an independent
contractor. This test is based on the extent of control the hirer exercises over a worker. The greater the
supervision and control the hirer exercises, the more likely the worker is deemed an employee, and the less
control the hirer exercises, the more likely the worker is considered an independent contractor.

FACTS:
In May 1994, respondent ABS-CBN signed an Agreement with the Mel and Jay Management and Development
Corporation (MJMDC). ABS-CBN was represented by its corporate officers while MJMDC was represented by
SONZA, as President and General Manager, and Tiangco, as EVP and Treasurer. Referred to in the Agreement
as AGENT, MJMDC agreed to provide SONZAs services exclusively to ABS-CBN as talent for radio and
television. ABS-CBN agreed to pay for SONZAs services a monthly talent fee of P310,000 for the first year and
P317,000 for the second and third year of the Agreement. ABS-CBN would pay the talent fees on the 10th and
25th days of the month.
On 30 April 1996, SONZA filed a complaint against ABS-CBN before the DOLE. SONZA complained
that ABS-CBN did not pay his salaries, separation pay, service incentive leave pay, 13th month pay, signing
bonus, travel allowance and amounts due under the Employees Stock Option Plan (ESOP). ABS-CBN filed a
Motion to Dismiss on the ground that no employer-employee relationship existed between the parties.

ISSUE: Whether or not there is an employer-employee relationship between petitioner and ABS-CBN.

LA RULING: There is no employer-employee relationship between petitioner and ABS-CBN.

It must be noted that complainant was engaged by respondent by reason of his peculiar skills and talent as a TV
host and a radio broadcaster. Unlike an ordinary employee, he was free to perform the services he undertook to
render in accordance with his own style. The fact that per the May 1994 Agreement complainant was accorded
some benefits normally given to an employee is inconsequential. Whatever benefits complainant enjoyed arose
from specific agreement by the parties and not by reason of employer-employee relationship. The fact that
complainant was made subject to respondents Rules and Regulations, likewise, does not detract from the
absence of employer-employee relationship.

NLRC and CA RULING: There is no employer-employee relationship between petitioner and ABS-CBN.

SC RULING:
NO, there is no employer-employee relationship between petitioner and ABS-CBN.

Case law has consistently held that the elements of an employer-employee relationship are: (a) the selection
and engagement of the employee; (b) the payment of wages; (c) the power of dismissal; and (d) the employers
power to control the employee on the means and methods by which the work is accomplished. The last element,
the so-called control test, is the most important element.

As to the selection and engagement of the employee: ABS-CBN engaged SONZAs services to co-host its
television and radio programs because of SONZAs peculiar skills, talent and celebrity status. Independent
contractors often present themselves to possess unique skills, expertise or talent to distinguish them from
ordinary employees. The specific selection and hiring of SONZA, because of his unique skills, talent and
celebrity status not possessed by ordinary employees, is a circumstance indicative, but not conclusive, of an
independent contractual relationship.

As to payment of wages: All the talent fees and benefits paid to SONZA were the result of negotiations that led
to the Agreement. If SONZA were ABS-CBNs employee, there would be no need for the parties to stipulate on
th
benefits such as SSS, Medicare, x x x and 13 month pay which the law automatically incorporates into every
employer-employee contract. Whatever benefits SONZA enjoyed arose from contract and not because of an
employer-employee relationship.

San Beda College of Law 93


4S: 2015 - 2016
LABOR LAW REVIEW Atty. Joyrich Golangco

As to power of dismissal: For violation of any provision of the Agreement, either party may terminate their
relationship. SONZA failed to show that ABS-CBN could terminate his services on grounds other than breach of
contract, such as retrenchment to prevent losses as provided under labor laws.

As to power of control, which is the most important: Applying the control test to the present case, we find that
SONZA is not an employee but an independent contractor. This test is based on the extent of control the hirer
exercises over a worker. The greater the supervision and control the hirer exercises, the more likely the worker
is deemed an employee. The converse holds true as well the less control the hirer exercises, the more likely the
worker is considered an independent contractor. ABS-CBN did not assign any other work to SONZA. How
SONZA delivered his lines, appeared on television, and sounded on radio were outside ABS-CBNs
control. SONZA did not have to render eight hours of work per day. The clear implication is that SONZA had a
free hand on what to say or discuss in his shows provided he did not attack ABS-CBN or its interests.

San Beda College of Law 94


4S: 2015 - 2016
LABOR LAW REVIEW Atty. Joyrich Golangco

21. BITOY JAVIER vs. FLY ACE CORPORATION/FLORDELYN CASTILLO
G.R. No. 192558 February 15, 2012
MENDOZA, J.:

Doctrine:
Whoever claims entitlement to the benefits provided by law should establish his or her right thereto. Hence, a
person who claims to be an employee must establish such claim.

FACTS:
Javier filed a complaint before the NLRC for underpayment of salaries and other labor standard benefits. He
alleged that he was an employee of Fly Ace since September 2007, performing various tasks at the respondents
warehouse such as cleaning and arranging the canned items before their delivery to certain locations, except in
instances when he would be ordered to accompany the companys delivery vehicles, as pahinante; that he
reported for work from Monday to Saturday from 7:00 oclock in the morning to 5:00 oclock in the afternoon; that
during his employment, he was not issued an identification card and payslips by the company; that thereafter,
Javier was terminated from his employment without notice; and that he was neither given the opportunity to
refute the cause/s of his dismissal from work.

For its part, Fly Ace denied that Javier is its employee and averred that it was engaged in the business of
importation and sales of groceries. Javier was contracted by its employee, Mr. Ong, as extra helper on a
pakyaw basis. Mr. Ong contracted Javier roughly 5 to 6 times only in a month whenever the vehicle of its
contracted hauler, Milmar Hauling Services, was not available.

ISSUE: Whether or not the petitioner is a an employee of Fly Ace Corporation.

RULING IN LA: Petitioner is not an employee of Fly Ace Corporation. It ruled that Javier has no employee ID
showing his employment with the Respondent nor any document showing that he received the benefits
accorded to regular employees of the Respondents. Respondent Fly Ace is not engaged in trucking business
but in the importation and sales of groceries. Since there is a regular hauler to deliver its products, we give
credence to Respondents claim that complainant was contracted on pakiao basis.

RULING IN NLRC: The NLRC reversed the decision of the LA and ruled that the LA skirted the argument of
Javier and immediately concluded that he was not a regular employee simply because he failed to present
proof. It was of the view that a pakyaw-basis arrangement did not preclude the existence of employer-employee
relationship. Payment by result x x x is a method of compensation and does not define the essence of the
relation. It is a mere method of computing compensation, not a basis for determining the existence or absence
of an employer-employee relationship.

RULING IN CA:The CA annulled the NLRC findings that Javier was indeed a former employee of Fly Ace and
reinstated the dismissal of Javiers complaint as ordered by the LA.

SC RULING:
NO, petitioner is not an employee of Fly Ace Corporation.

No particular form of evidence is required to prove the existence of such employer-employee relationship. Any
competent and relevant evidence to prove the relationship may be admitted. The rule of thumb remains: the
onus probandi falls on petitioner to establish or substantiate such claim by the requisite quantum of evidence.
Whoever claims entitlement to the benefits provided by law should establish his or her right thereto x x x. Sadly,
Javier failed to adduce substantial evidence as basis for the grant of relief.

In this case, the LA and the CA both concluded that Javier failed to establish his employment with Fly Ace. By
way of evidence on this point, all that Javier presented were his self-serving statements purportedly showing his
activities as an employee of Fly Ace. Clearly, Javier failed to pass the substantiality requirement to support his
claim. Hence, the Court sees no reason to depart from the findings of the CA.

San Beda College of Law 95


4S: 2015 - 2016
LABOR LAW REVIEW Atty. Joyrich Golangco

22. SAN MIGUEL CORPORATION EMPLOYEES UNION-PTGWO, ET AL vs. HON. JESUS G. BERSAMIRA,
IN HIS CAPACITY AS PRESIDING JUDGE OF BRANCH 166, RTC, PASIG, and SAN MIGUEL CORP.
G.R. No. 87700 June 13, 1990
MELENCIO-HERRERA, J.

Doctrine:
A labor dispute can nevertheless exist regardless of whether the disputants stand in the proximate relationship
of employer and employee. The existence of a labor dispute is not negative by the fact that the plaintiffs and
defendants do not stand in the proximate relation of employer and employee.

FACTS:
Sometime in 1983 and 1984, SanMig entered into contracts for merchandising services with Lipercon and
D'Rite. These companies are independent contractors duly licensed by the DOLE. In said contracts, it was
expressly understood and agreed that the workers employed by the contractors were to be paid by the latter and
that none of them were to be deemed employees or agents of SanMig. There was to be no employer-employee
relation between the contractors and/or its workers, on the one hand, and SanMig on the other.

Petitioner SMCEU is the duly authorized representative of the monthly paid rank-and-file employees of SanMig
with whom the latter executed a CBA.

In a letter, the Union advised SanMig that some Lipercon and D'Rite workers had signed up for union
membership and sought the regularization of their employment with SMC. The Union alleged that this group of
employees, while appearing to be contractual workers supposedly independent contractors, have been
continuously working for SanMig for a period ranging from 6 months to 15 years and that their work is neither
casual nor seasonal as they are performing work or activities necessary or desirable in the usual business or
trade of SanMig. Thus, it was contended that there exists a "labor-only" contracting situation. It was then
demanded that the employment status of these workers be regularized.

SMC filed a verified Complaint for Injunction and Damages before respondent Court. The Union filed a Motion to
Dismiss SanMig's Complaint on the ground of lack of jurisdiction over the case/nature of the action, which
motion was opposed by SanMig.

ISSUE: Whether or not there the Respondent Court has jurisdiction in issuing the injunction.

RULING IN THE RTC: The respondent Court issued injunction. The absence of employer-employee relationship
negates the existence of labor dispute. Verily, this court (RTC) has jurisdiction to take cognizance of Sanmig's
grievance.

The evidence so far presented indicates that plaintiff has contracts for services with Lipercon and D'Rite. The
application and contract for employment of the defendants' witnesses are either with Lipercon or D'Rite. What
could be discerned is that there is no employer-employee relationship between plaintiff and the contractual
workers employed by Lipercon and D'Rite. This, however, does not mean that a final determination regarding
the question of the existence of employer-employee relationship has already been made. To finally resolve this
dispute, the court must extensively consider and delve into the manner of selection and engagement of the
putative employee; the mode of payment of wages; the presence or absence of a power of dismissal; and the
Presence or absence of a power to control the putative employee's conduct.
SC RULING:
NO, the respondent Court has no jurisdiction in issuing the injunction.

A "labor dispute" as defined in Article 212 (1) of the Labor Code includes "any controversy or matter concerning
terms and conditions of employment or the association or representation of persons in negotiating, fixing,
maintaining, changing, or arranging the terms and conditions of employment, regardless of whether the
disputants stand in the proximate relation of employer and employee." A labor dispute, as defined by the law,
does exist herein is evident.

Whether or not the Union demands are valid; whether or not SanMig's contracts with Lipercon and D'Rite
constitute "labor-only" contracting and, therefore, a regular employer-employee relationship may, in fact, be said
to exist; whether or not the Union can lawfully represent the workers of Lipercon and D'Rite in their demands
against SanMig in the light of the existing CBA; whether or not the notice of strike was valid and the strike itself
San Beda College of Law 96
4S: 2015 - 2016
LABOR LAW REVIEW Atty. Joyrich Golangco

legal when it was allegedly instigated to compel the employer to hire strangers outside the working unit; those
are issues the resolution of which call for the application of labor laws, and SanMig's causes of action in the
Court below are inextricably linked with those issues.

As the case is indisputably linked with a labor dispute, jurisdiction belongs to the labor tribunals. As explicitly
provided for in Article 217 of the Labor Code, prior to its amendment by R.A. No. 6715 on 21 March 1989, since
the suit below was instituted on 6 March 1989, Labor Arbiters have original and exclusive jurisdiction to hear
and decide the following cases involving all workers.

San Beda College of Law 97


4S: 2015 - 2016
LABOR LAW REVIEW Atty. Joyrich Golangco

23. RAUL G. LOCSIN and EDDIE B. TOMAQUIN v. PHILIPPINE LONG DISTANCE TELEPHONE COMPANY
G.R. No. 185251 October 2, 2009
VELASCO, JR., J.:

Doctrine:
The power of control, in this case, has been explained as the right to control not only the end to be achieved
but also the means to be used in reaching such end. With the conclusion that respondent directed petitioners to
remain at their posts and continue with their duties, it is clear that respondent exercised the power of control
over them; thus, the existence of an employer-employee relationship.

FACTS:
Respondent PLDT and the SSCP entered into a Security Services Agreement whereby SSCP would provide
armed security guards to PLDT to be assigned to its various offices. Pursuant to such agreement, petitioners
Raul Locsin and Eddie Tomaquin, among other security guards, were posted at a PLDT office.

Then respondent issued a Letter terminating the Agreement. Despite the termination of the Agreement,
however, petitioners continued to secure the premises of their assigned office. They were allegedly directed to
remain at their post by representatives of respondent. In support of their contention, petitioners provided the
Labor Arbiter with copies of petitioner Locsins pay slips for the period after the said termination of Agreement.

Then, after a year, petitioners services were terminated. Thus, petitioners filed a complaint before the Labor
Arbiter for illegal dismissal and recovery of money claims.

ISSUE: Whether or not there is an employer-employee relationship between the petitioners and PLDT.

RULING IN LA: Petitioners were found to be employees of PLDT and not of SSCP. Such conclusion was
arrived at with the factual finding that petitioners continued to serve as guards of PLDTs offices. As such
employees, petitioners were entitled to substantive and procedural due process before termination of
employment. The Labor Arbiter held that respondent failed to observe such due process requirements.

RULING IN NLRC: The NLRC affirmed the decision of the LA.

RULING IN CA: The CA reversed the decision of LA and NLRC. The CA applied the four-fold test in order to
determine the existence of an employer-employee relationship between the parties but did not find such
relationship. It determined that SSCP was not a labor-only contractor and was an independent contractor having
substantial capital to operate and conduct its own business. The CA further bolstered its decision by citing the
Agreement whereby it was stipulated that there shall be no employer-employee relationship between the
security guards and PLDT.

SC RULING:
YES, there is employer-employee relationship between the petitioners and PLDT. From the foregoing
circumstances, reason dictates that we conclude that petitioners remained at their post under the instructions of
respondent. We can further conclude that respondent dictated upon petitioners that the latter perform their
regular duties to secure the premises during operating hours. This, to our mind and under the circumstances, is
sufficient to establish the existence of an employer-employee relationship.

To reiterate, while respondent and SSCP no longer had any legal relationship with the termination of the
Agreement, petitioners remained at their post securing the premises of respondent while receiving their salaries,
allegedly from SSCP. Clearly, such a situation makes no sense, and the denials proffered by respondent do not
shed any light to the situation. It is but reasonable to conclude that, with the behest and, presumably, directive of
respondent, petitioners continued with their services. Evidently, such are indicia of control that respondent
exercised over petitioners.

Such power of control has been explained as the right to control not only the end to be achieved but also the
means to be used in reaching such end. With the conclusion that respondent directed petitioners to remain at
their posts and continue with their duties, it is clear that respondent exercised the power of control over them;
thus, the existence of an employer-employee relationship.

San Beda College of Law 98


4S: 2015 - 2016
LABOR LAW REVIEW Atty. Joyrich Golangco

Evidently, respondent having the power of control over petitioners must be considered as petitioners employer
from the termination of the Agreement on wards as this was the only time that any evidence of control was
exhibited by respondent over petitioners.

24. PEOPLE'S BROADCASTING (BOMBO RADYO PHILS., INC.) vs. SECRETARY OF DOLE
G.R. No. 179652 May 8, 2009
TINGA, J.:

Doctrine:
The Department of Labor and Employment is fully empowered to make a determination as to the existence of an
employer-employee relationship in the exercise of its visitorial and enforcement power.

FACTS:
Private respondent Jandeleon Juezan filed a complaint against petitioner before (DOLE) Regional Office, for
illegal deduction, nonpayment of service incentive leave, 13th month pay, premium pay for holiday and rest day
and illegal diminution of benefits, delayed payment of wages and non-coverage of SSS, PAG-IBIG and
Philhealth. After summary investigation, DOLE found that private respondent was an employee of petitioner, and
was entitled to his money. Bombo Radyo appealed the decision, but the DOLE dismissed the same. The Court
of Appeals (CA) afirmed such dismissal.

When the matter reached the Supreme Court, the CA decision was reversed and set aside. The Court found
that there was no employer-employee relationship between Bombo Radyo and Juezan. It was held that while
the DOLE may make a determination of the existence of an employer-employee relationship, this function could
not be co-extensive with the visitorial and enforcement power provided in Art. 128(b) of the Labor Code, as
amended by RA 7730. The National Labor Relations Commission (NLRC) was held to be the primary agency in
determining the existence of an employer-employee relationship. From this decision, the Public Attorneys Ofice
(PAO) filed a Motion for Clariication of Decision (with Leave of Court). The PAO sought to clarify as to when the
visitorial and enforcement power of the DOLE can be considered as co-extensive with the power to determine
the existence of an employer-employee relationship. The Court treated the Motion for Clarification as a second
motion for reconsideration, granting said motion and reinstating the petition.

ISSUE: Whether or not the Department of Labor and Employment has the power to determine the existence of
employer-employee relationship in its exercise of its visitorial and its enforcement power.

RULING:
No limitation in the law was placed upon the power of the DOLE to determine the existence of an employer-
employee relationship. No procedure was laid down where the DOLE would only make a preliminary inding,
that the power was primarily held by the NLRC. The law did not say that the DOLE would first seek the NLRCs
determination of the existence of an employer-employee relationship, or that should the existence of the
employer-employee relationship be disputed, the DOLE would refer the matter to the NLRC. The DOLE must
have the power to determine whether or not an employer-employee relationship exists, and from there to decide
whether or not to issue compliance orders in accordance with Art. 128(b) of the Labor Code, as amended by RA
7730.

The determination of the existence of an employer-employee relationship by the DOLE must be respected. The
expanded visitorial and enforcement power of the DOLE granted by RA 7730 would be rendered nugatory if the
alleged employer could, by the simple expedient of disputing the employer- employee relationship, force the
referral of the matter to the NLRC. The Court issued the declaration that at least a prima facie showing of the
absence of an employer-employee relationship be made to oust the DOLE of jurisdiction. But it is precisely the
DOLE that will be faced with that evidence, and it is the DOLE that will weigh it, to see if the same does
successfully refute the existence of an employer-employee relationship.

San Beda College of Law 99


4S: 2015 - 2016
LABOR LAW REVIEW Atty. Joyrich Golangco

25. ERNESTO G. YMBONG vs. ABS-CBN BROADCASTING CORPORATION
G.R. No. 184885 March 7, 2012
VILLARAMA, JR., J.:

Doctrine:
A company is estopped from denying the existence of employer- employee relationship after applying a
company policy to all its employees including those under talent contracts.

FACTS:
Ernesto G. Ymbong worked for ABS-CBN Broadcasting Corporation (ABS-CBN at its regional station in Cebu
as a television talent which extended to radio when ABS-CBN Cebu launched its AM station DYAB where he
worked as drama and voice talent, spinner, scriptwriter and public affairs program anchor. Leandro Patalinghug
also worked for ABS-CBN Cebu as talent, director and scriptwriter for various radio programs aired over DYAB.

ABS-CBN head office issued Policy No. HR-ER-016 (Policy on Employees Seeking Public Office) requiring an
employee (1) to resign if he intends to run for a public position (2) to file a leave of absence if he intends to join a
political group/party or even with no political affiliation but who intends to openly and aggressively campaign for
a candidate or group of candidates (e.g. publicly speaking/endorsing candidate, recruiting campaign workers,
etc.) Dante Luzon, Assistant Station Manager of DYAB issued a memorandum stating that those who intend to
run shall file a leave of absent not consonance with the Policy.

Ymbong then file a leave of absence although he ran for a public office. Patalinghug filed a resignation.
Unfortunately both lost in the election. They then tried to go back to work with the ABS. However, they were only
allowed to wind up their programs. After such, they were informed of their automatic termination because of the
policy.

Ymbong filed complaint for illegal dismissal, in contrast contended that


after the expiration of his leave of absence, he reported back to work as a regular talent and in fact
continued to receive his salary;
The ground cited by ABS-CBN for his dismissal was not among those enumerated in the Labor Code, as
amended; and
The company policy violates his constitutional right to suffrage.

In their defense, complaints arguing that there is no employer-employee relationship between the
company and Ymbong and Patalinghug. ABS-CBN contended that they are not employees but talents as
evidenced by their talent contracts.

LA RULING: There exists employer-employee relationship based on the appointment letters/talent contracts
imposed conditions in the performance of their work, specifically on attendance and punctuality, which
effectively placed them under the control of ABS-CBN.
Also, the policy was not made known to them and superseded by the memo of Luzon.

NLRC RULING: The NLRC held that ABS-CBN wielded the power of control over Ymbong and Patalinghug,
thereby proving the existence of an employer-employee relationship between them. However, the two were
treated differently (1) Patalinghug was considered resigned while (2) Ymbong was not considered resigned. In
the latter, it was ruled that the memorandum of Luzon shall prevail ever the policy under the principle of social
justice.

CA RULING: The CA ruled that ABS-CBN is estopped from claiming that Ymbong was not its employee after
applying the provisions of Policy No. HR-ER-016 to him. The CA likewise held that the subject company policy is
the controlling guideline and therefore, Ymbong should be considered resigned from ABS-CBN.

ISSUES:
1)Whether or not Ymbong and Patalinghug were employees of ABS-CBN.
2)Whether or not Ymbong was terminated.

SC RULING:

1) Yes. They were employees. The Court upheld the decision of CA as to employment issue.
San Beda College of Law 100
4S: 2015 - 2016
LABOR LAW REVIEW Atty. Joyrich Golangco

2) Ymbong was not terminated. He resigned.

a) The Policy was a valid exercise of management prerogative. In the instant case, ABS-CBN validly
justified the implementation of Policy No. HR-ER-016. It is well within its rights to ensure that it
maintains its objectivity and credibility and freeing itself from any appearance of impartiality so that
the confidence of the viewing and listening public in it will not be in any way eroded.

b) Ymbongs overt act of running for councilor of Lapu-Lapu City is tantamount to resignation on his
part. He was separated from ABS-CBN not because he was dismissed but because he resigned.
Since there was no termination to speak of, the requirement of due process in dismissal cases
cannot be applied to Ymbong. Thus, ABS-CBN is not duty-bound to ask him to explain why he did
not tender his resignation before he ran for public office as mandated by the subject company
policy.

San Beda College of Law 101


4S: 2015 - 2016
LABOR LAW REVIEW Atty. Joyrich Golangco

26. PROFESSIONAL SERVICES, INC. vs. COURT OF APPEALS and NATIVIDAD and ENRIQUE AGANA
G.R. No. 126297 February 11, 2008
SANDOVAL-GUTIERREZ, J.:

Doctrine:
"Consultant" physicians can be considered as employee of the hospital if there is an actual control over their
selection, hiring and termination.

FACTS:
Dr. Fuentes and Dr. Ampil performed and completed the hysterectomy upon Natividad Agana who was
suffering from "cancer of the sigmoid." However, the operation appeared to be flawed. Natividad complained of
excruciating pain in her anal region. She consulted both Dr. Ampil and Dr. Fuentes about it. They told her that
the pain was the natural consequence of the surgical operation. She then sought treatment at the Polymedic
General Hospital wherein Dr. Ramon Gutierrez detected the presence of a foreign object in her vagina -- a foul-
smelling gauze which badly infected her vaginal vault. Two surgeries were performed to remedy the infection.

Natividad and her husband filed with the RTC QC a complaint for damages against Professional Service Inc
(PSI) (owner of Medical City), Dr. Ampil and Dr. Fuentes. Pending the outcome of the above case, Natividad
died. She was duly substituted by her above-named children (the Aganas).

RTC RULING:
PSI, Dr. Ampil and Dr. Fuentes are severally liable.

CA RULING:
The court affirmed the assailed judgment with modification in the sense that the complaint against Dr. Fuentes
was dismissed.PSI, Dr. Ampil and the Aganas filed with this Court separate petitions for review on certiorari.

SUPREME COURT FIRST DIVISION:


The Court, through its First Division, rendered a Decision holding that PSI is jointly and severally liable;
1. there is an employer-employee relationship between Medical City and Dr. Ampil for purposes of
apportioning responsibility in medical negligence cases;
2. PSIs act of publicly displaying in the lobby of the Medical City the names and specializations of its
accredited physicians, including Dr. Ampil, estopped it from denying the existence of an employer-
employee relationship between them under the doctrine of ostensible agency or agency by estoppel;
3. PSIs failure to supervise Dr. Ampil and its resident physicians and nurses and to take an active step in
order to remedy their negligence rendered it directly liable under the doctrine of corporate negligence.

Arguments of PSI in the MR:


1. PSI contends that there's no employer-employee relationship between it and its consultant, Dr. Ampil.
PSI stressed that the Courts Decision in Ramos holding that "an employer-employee relationship in
effect exists between hospitals and their attending and visiting physicians for the purpose of
apportioning responsibility".
2. PSI maintains that consultants, like Dr. Ampil, are "independent contractors," not employees of the
hospital.
3.

ISSUES:
1. Whether or not there's an employee - employer relationship for solidary liability to attach.
2. Whether or not Dr. Ampil an independent contractor-physician hence liability is personal.

SC RULING:
Yes, employer-employee relationship "in effect" exists between the Medical City and Dr. Ampil. Consequently,
both are jointly and severally liable to the Aganas.

First, hospitals exercise significant control in the hiring and firing of consultants and in the conduct of
their work within the hospital premises. The applicant for "consultant" required to submit;
1. proof of completion of residency;
2. their educational qualifications;
3. generally, evidence of accreditation by the appropriate board (diplomate);
San Beda College of Law 102
4S: 2015 - 2016
LABOR LAW REVIEW Atty. Joyrich Golangco

4. evidence of fellowship in most cases, and
5. references.

After a physician is accepted, either as a visiting or attending consultant, he is normally required to attend
clinico-pathological conferences, rounds and patient audits. In addition to these, the physicians performance as
a specialist is generally evaluated by a peer review committee on the basis of mortality and morbidity statistics,
and feedback from patients, nurses, interns and residents.
Hence, private hospitals hire, fire and exercise real control over their attending and visiting "consultant" staff.
While "consultants" are not, technically employees, a point which respondent hospital asserts in denying all
responsibility for the patients condition, the control exercised, the hiring, and the right to terminate consultants
all fulfill the important hallmarks of an employer-employee relationship, with the exception of the payment of
wages. In assessing whether such a relationship in fact exists, the control test is determining. Accordingly, on
the basis of the foregoing, we rule that for the purpose of allocating responsibility in medical negligence cases,
an employer-employee relationship in effect exists between hospitals and their attending and visiting physicians.

Second, even assuming that Dr. Ampil is not an employee of Medical City, but an independent contractor, still
the said hospital is liable to the Aganas based on the "doctrine of apparent authority." There are two factors to
consider:
1. Whether the hospital acted in a manner which would lead a reasonable person to conclude that the
individual who was alleged to be negligent was an employee or agent of the hospital; and
2. Whether the plaintiff acted in reliance upon the conduct of the hospital or its agent, consistent with
ordinary care and prudence.

In this case, PSI is estopped from passing the blame solely to Dr. Ampil. Its act of displaying his name and
those of the other physicians in the public directory at the lobby of the hospital amounts to holding out to the
public that it offers quality medical service through the listed physicians. This justifies Atty. Aganas belief that
Dr. Ampil was a member of the hospitals staff. It must be stressed that under the doctrine of apparent authority,
the question in every case is whether the principal has by his voluntary act placed the agent in such a situation
that a person of ordinary prudence, conversant with business usages and the nature of the particular business,
is justified in presuming that such agent has authority to perform the particular act in question.

San Beda College of Law 103


4S: 2015 - 2016
LABOR LAW REVIEW Atty. Joyrich Golangco

27. SOUTH EAST INTERNATIONAL RATTAN INC. v. COMING
G.R. No. 126297 February 11, 2008
VILLARAMA, JR., J.:

Doctrine:
Payroll not conclusive proof of existence or absence of ER-EE relationship.

FACTS:
Respondent Jesus J. Coming filed a complaint for illegal dismissal, underpayment of wages, non-payment of
holiday pay, 13th month pay and service incentive leave pay, with prayer for reinstatement, back wages,
damages and attorneys fees against South East International Rattan Inc (SEIRI). He alleged that he was hired
by petitioners as Sizing Machine Operator on March 17, 1984. His work schedule is from 8:00 a.m. to 5:00 p.m.
Initially, his compensation was on "pakiao" basis but sometime in June 1984, it was fixed at P150.00 per day
which was paid weekly. In 1990, he was told not to work for two months for no reason. After two months, he
reported back to wrk only to be later on terminated because the company is not doing well financially and that
he would be called back to work only if they need his services again. Respondent waited for almost a year but
petitioners did not call him back to work. Hence, he filed a complaint before the regional arbitration branch. To
bolster his claim, respondent submitted an affidavit signed by five former co-workers stating that respondent
was one of the pioneer employees who worked in SEIRI for almost twenty years.

In their defense, petitioners denied having hired respondent. They stressed that respondent was not included in
the list of employees submitted to the Social Security System (SSS). There's also an affidavit of Comings's
brother attesting that he worked for another employer.

LA RULING: Respondent is a regular employee of SEIRI and that the termination of his employment was illegal.
Labor Arbiter Carreon found that respondents work as sizing machine operator is usually necessary and
desirable to the rattan furniture business of petitioners and their failure to include respondent in the employment
report to SSS is not conclusive proof that respondent is not their employee.

NLRC RULING: Set aside the decision of LA ruling that, complainant failed to present a single payslip, voucher
or a copy of a company payroll showing that he rendered service during the period indicated therein. The appeal
to (NLRC)-Cebu City submitted the following additional evidence:
(1) copies of SEIRIs payrolls and individual pay records of employees;
(2) affidavit15 of SEIRIs Treasurer, Angelina Agbay; and
(3) second affidavit16 of Vicente Coming.

CA RULING: Reinstated the decision of LA. The CA gave more credence to the declarations of the five former
employees of petitioners that respondent was their co-worker in SEIRI. As to the absence of respondents name
in the payroll and SSS employment report, the CA observed that the payrolls submitted were only from January
1, 1999 to December 29, 2000 and not the entire period of eighteen years when respondent claimed he worked
for SEIRI. It further noted that the names of the five affiants, whom petitioners admitted to be their former
employees, likewise do not appear in the aforesaid documents. According to the CA, it is apparent that
petitioners maintained a separate payroll for certain employees or willfully retained a portion of the payroll.

As to the control test, records show that:


(1) they required him to work within the company premises;
(2) they obliged petitioner to report every day of the week and tasked him to usually perform the same job;
(3) they enforced the observance of definite hours of work from 8 oclock in the morning to 5 oclock in the
afternoon;
(4) the mode of payment of petitioners salary was under their discretion, at first paying him on pakiao basis
and thereafter, on daily basis;
(5) they implemented company rules and regulations;
(6) [Estanislao] Agbay directly paid petitioners salaries and controlled all aspects of his employment and
(7) petitioner rendered work necessary and desirable in the business of the respondent company.

ISSUE: Whether or not an employer-employee relationship exists.

SC RULING:
Yes. The Court affirmed the ruling of the CA.
San Beda College of Law 104
4S: 2015 - 2016
LABOR LAW REVIEW Atty. Joyrich Golangco

To ascertain the existence of an employer-employee relationship jurisprudence has invariably adhered


to the four-fold test, to wit:
(1) the selection and engagement of the employee;
(2) the payment of wages;
(3) the power of dismissal; and
(4) the power to control the employees conduct, or the so-called "control test."

[Evidence]
In resolving the issue of whether such relationship exists in a given case, substantial evidence that
amount of relevant evidence which a reasonable mind might accept as adequate to justify a conclusion is
sufficient. Although no particular form of evidence is required to prove the existence of the relationship, and any
competent and relevant evidence to prove the relationship may be admitted, a finding that the relationship exists
must nonetheless rest on substantial evidence.

As to the SSS or payroll list:


The Court reiterated that in Tan v. Lagrama, the fact that a worker was not reported as an employee to the SSS
is not conclusive proof of the absence of employer-employee relationship. Otherwise, an employer would be
rewarded for his failure or even neglect to perform his obligation.

For a payroll to be utilized to disprove the employment of a person, it must contain a true and complete list of
the employee. In this case, the exhibits offered by petitioners before the NLRC consisting of copies of payrolls
and pay earnings records are only for the years 1999 and 2000; they do not cover the entire 18-year period
during which respondent supposedly worked for SEIRI.

As to the certifications issued by Mayol and Apondar asserting that respondent worked for them and
not for SEIRI:
The Court ruled that the certifications did not prove any fact that respondent was not an employee of SEIRI. The
certifications only claimed that (1) respondent worked under Mayor on his own discretion and (2) under Apondar
as his sideline but only after regular working hours and "off and on" basis. Even assuming the truth of the
foregoing statements, these do not foreclose respondents regular or full-time employment with SEIRI.

As to the affidavit of former co-workers submitted by respondent:


The petitioner claimed that the affiants were employees of their suppliers Mayol and Apondar. However, they did
not submit proof that the latter were indeed independent contractors; clearly, petitioners failed to discharge their
burden of proving their own affirmative allegation.

Hence, respondent Coming was a regular employee and unlawfully dismissed.

Decision regarding BACKWAGES and reinstatement


Respondent, whose employment was terminated without valid cause by petitioners, is entitled to reinstatement
without loss of seniority rights and other privileges and to his full back wages, inclusive of allowances and other
benefits or their monetary equivalent, computed from the time his compensation was withheld from him up to the
time of his actual reinstatement. Where reinstatement is no longer viable as an option, back wages shall be
computed from the time of the illegal termination up to the finality of the decision. Separation pay equivalent to
one month salary for every year of service should likewise be awarded as an alternative in case reinstatement in
not possible.

San Beda College of Law 105


4S: 2015 - 2016
LABOR LAW REVIEW Atty. Joyrich Golangco

28. TENAZAS ET.AL v. R. VILLEGAS TAXI TRANSPORTATION
G.R. No. 192998 April 2, 2014
REYES, J.:

Doctrine:
The employee must present evidence to establish the existence of employer-employee relationship.

FACTS:
Tenazas, Francisco and Edraca were complainants in a consolidated case for illegal dismissal against R.
Villegas Taxi Transport and Romualdo Villegas before the Labor Arbiter of. In their positions papers, they
alleged they were hired as taxi drivers on a boundary system by the respondents.
1. The taxi Tenazas was driving was sideswiped by another vehicle. When he reported the matter to the
company, he was scolded by the respondents and told to leave the garage as he was already fired.
2. Francisco alleged that he was terminated on suspicion that he was organising a labor union, hence he
was terminated without due process.
3. Isidro alleged that he was terminated when he fell short of the required boundary after he brought his
unit to an auto repair shop for an urgent repair. When he returned to the garage his drivers license was
confiscated and he was no longer allowed to drive a taxi despite his pleas.

In their defense, the company admitted Tenazas and Edraca were regular and spare drivers respectively, but
denied employing Francisco as a driver. Tenzas was never terminated by the company. He was informed that
his unit was due for overhaul and advised to wait for further notice from the company if his unit was already
fixed. Despite being informed on July 8, 2007 that his unit was ready for release, Tenazas did not return. On
Edraca, the company alleged he was a spare driver of the company from 2001, substituting whenever a driver is
not around. They could not have terminated him in 2006 since he stopped reporting for work in 2003.

LA RULING: The LA dismissed their complaint, finding no employer-employee relationship between them and
the company. The company having denied the existence thereof, it was incumbent upon complainants to prove
the existence of the employer-employee relationship.

NLRC RULING: On appeal to the NLRC, however, the commission, relying on the newly discovered evidence
submitted by the complainant Tenazas, ruled them illegally dismissed. It ordered payment of their back wages
from the time of dismissal, as well as payment of separation pay and attorneys fees.

CA RULING: On petition for certiorari with the CA, the latter affirmed the NLRC judgment but deleted the award
of separation pay and ordered their reinstatement. It also deleted the award in favour of Francisco, who, the CA
averred, failed to prove that he was an employee of the respondent. Thus, the petitioners elevated their case to
the Supreme Court to review the CA decision dismissing Franciscos complaint and deleting the award of
separation pay to the other petitioners.

ISSUE: Whether or not employer-employee relations exist between the Jaime Francisco and the company.

SC RULING:
The petition lacks merit.Pivotal to the resolution of the instant case is the determination of the existence of
employer-employee relationship and whether there was an illegal dismissal.

Unlike the other complainant, Tenazas who submitted proof of SSS contribution, affidavit of co-drivers and
pictures wearing company shirt, Francisco failed to present sufficient evidence to prove regular employment
such as company ID, SSS membership, withholding tax certificates or similar articles.

The Court ruled that in determining the presence or absence of an employer-employee relationship the following
requisites must be present;
(a) the selection and engagement of the employee;
(b) the payment of wages;
(c) the power of dismissal; and
(d) the employers power to control the employee on the means and methods by which the work is
accomplished. The last element, the so-called control test, is the most important element.

San Beda College of Law 106


4S: 2015 - 2016
LABOR LAW REVIEW Atty. Joyrich Golangco

There is no hard and fast rule designed to establish the aforesaid elements. Any competent and relevant
evidence to prove the relationship may be admitted. Identification cards, cash vouchers, social security
registration, appointment letters or employment contracts, payrolls, organization charts, and personnel lists,
serve as evidence of employee status.

In this case, however, Francisco failed to present any proof substantial enough to establish his relationship with
the respondents.
He failed to present any attendance logbook, payroll, SSS record or any personnel file that could somehow
depict his status as an employee;
He was not issued with employment records, he could have, at least, produced his social security records
which state his contributions, name and address of his employer, as his co-petitioner Tenazas did.
There's no testimonial evidence showing the respondents exercise of control over the means and methods
by which he undertakes his work.
The employment was being claimed by Emmanuel who executed an affidavit alleging that Francisco was
employed as a spare driver in his taxi garage, a fact that the latter failed to deny or question in any of the
pleadings attached to the records of this case.

In Opulencia Ice Plant and Storage v. NLRC, the Court emphasized, that there's no particular form of evidence
is required to prove the existence of an employer-employee relationship. However in this case, Francisco simply
relied on his allegation that he was an employee of the company without any other evidence supporting his
claim.

Hence, CA correctly ruled that Francisco could not be considered an employee of the respondents.

THE CASE ALSO DISCUSS THE APPLICATION OF BACKWAGES AND REINSTATEMENT.


The CAs order of reinstatement of Tenazas and Endraca, instead of the payment of separation pay, is also well
in accordance with prevailing jurisprudence. In Macasero v. Southern Industrial Gases Philippines,14 the Court
reiterated, thus:

[A]n illegally dismissed employee is entitled to two reliefs: backwages and reinstatement. The two reliefs
provided are separate and distinct. In instances where reinstatement is no longer feasible because of strained
relations between the employee and the employer, separation pay is granted. In effect, an illegally dismissed
employee is entitled to either reinstatement, if viable, or separation pay if reinstatement is no longer viable, and
backwages.

The normal consequences of respondents illegal dismissal, then, are reinstatement without loss of seniority
rights, and payment of backwages computed from the time compensation was withheld up to the date of actual
reinstatement. Where reinstatement is no longer viable as an option, separation pay equivalent to one (1)
month salary for every year of service should be awarded as an alternative. The payment of separation pay is
in addition to payment of backwages. (Emphasis supplied)

Clearly, it is only when reinstatement is no longer feasible that the payment of separation pay is ordered in lieu
thereof. For instance, if reinstatement would only exacerbate the tension and strained relations between the
parties, or where the relationship between the employer and the employee has been unduly strained by reason
of their irreconcilable differences, it would be more prudent to order payment of separation pay instead of
reinstatement.16

This doctrine of strained relations, however, should not be used recklessly or applied loosely17 nor be based
on impression alone. It bears to stress that reinstatement is the rule and, for the exception of strained relations
to apply, it should be proved that it is likely that if reinstated, an atmosphere of antipathy and antagonism
would be generated as to adversely affect the efficiency and productivity of the employee concerned.18

Moreover, the existence of strained relations, it must be emphasized, is a question of fact. In Golden Ace
Builders v. Talde, the Court underscored:

Strained relations must be demonstrated as a fact, however, to be adequately supported by evidence


substantial evidence to show that the relationship between the employer and the employee is indeed strained as
a necessary consequence of the judicial controversy (Citations omitted and emphasis ours)

San Beda College of Law 107


4S: 2015 - 2016
LABOR LAW REVIEW Atty. Joyrich Golangco

After a perusal of the NLRC decision, this Court failed to find the factual basis of the award of separation pay to
the petitioners. The NLRC decision did not state the facts which demonstrate that reinstatement is no longer a
feasible option that could have justified the alternative relief of granting separation pay instead.

The petitioners themselves likewise overlooked to allege circumstances which may have rendered their
reinstatement unlikely or unwise and even prayed for reinstatement alongside the payment of separation pay in
their position paper. A bare claim of strained relations by reason of termination is insufficient to warrant the
granting of separation pay. Likewise, the filing of the complaint by the petitioners does not necessarily translate
to strained relations between the parties. As a rule, no strained relations should arise from a valid and legal act
asserting ones right. Although litigation may also engender a certain degree of hostility, the understandable
strain in the parties relation would not necessarily rule out reinstatement which would, otherwise, become the
rule rather the exception in illegal dismissal cases. Thus, it was a prudent call for the CA to delete the award of
separation pay and order for reinstatement instead, in accordance with the general rule stated in Article 279 of
the Labor Code.

Finally, the Court finds the computation of the petitioners backwages at the rate of P800.00 daily reasonable
and just under the circumstances. The said rate is consistent with the ruling of this Court in Hyatt Taxi Services,
Inc. v. Catinoy, which dealt with the same matter.

WHEREFORE, in view of the foregoing disquisition, the petition for review on certiorari is DENIED. The
Decision dated March 11, 2010 and Resolution dated June 28, 2010 of the Court of Appeals.

San Beda College of Law 108


4S: 2015 - 2016
LABOR LAW REVIEW Atty. Joyrich Golangco

29. GREGORIO V. TONGKO, petitioner vs. THE MANUFACTURERS LIFE INSURANCE CO. (PHILS.), INC.
G.R. No. 167622 November 7, 2008
VELASCO, JR., J.:

Doctrines:
1. An employer-employee relationship exists where the person for whom the services are performed
reserves the right to control not only the end to be achieved but also the means to be used in reaching
such end.
2. An employer may terminate the services of an employee for just cause and this must be supported by
substantial evidence.
FACTS:
Manufacturers Life Insurance Co. (Phils.), Inc. (Manulife) is a domestic corporation engaged in life insurance
business. It executed a Career Agents Agreement with Gregorio Tongko whereby the latter agreed to be an
independent contracor for the canvass of insurance policies and other products offered by the company. Tongko
was promoted to Unit Manager in 1983 and Branch Manager in 1990. However, Tongko received a letter in
2001 from Manulife President and Chief Executive Officer, Ranato Vergel De Dios, regarding a Metro North
Sales Managers Meeting. The said letter stated that the region of Tongko is the lowest performer in terms of
recruiting and provided for measures to address such issue. Subsequently, Tongko received another letter form
De Dios terminating his Agents Contract for his failure to align his directions with the Managements avowed
agency growth policy. Tongko then filed a complaint for illegal dismissal against Manulife before the NLRC.

ISSUES
1. Whether or not an Employer-Employee relationship exist between Tongko and Manulife.
2. Whether or not Tongko was illegaly dismissed.

LA RULING: No. The LA ruled that no Employer-Employee relationship was found in applying the four-fold test.

NLRC RULING:nYes. The NLRC ruled that an Employer-Employee relationship existed because Manulife
exercised control over Tongko as evidence by a letter of De Dios, which contained various directives to Tongko.
The NLRC also held Manulife liable for illegal Dismissal.

CA RULING: No. The CA ruled that no Employer-Employee relationship existed because Manulife did not
exercise control over Tongko that would render the latter an employee of the former.

SC RULING:
1. Yes. Manulife had the power of control over Tongko. Under the Agreement executed between Tongko
and Manulife in 1977, the former must comply with the following requirements: (1) compliance with the
regulations and requirements of the company; (2) maintenance of a level of knowledge of the
companys products that is satisfactory to the company; and (3) compliance with a quota of new
businesses. Tongko was required to comply with the different codes of conduct of Manulife and he was
also tasked to perform administrative duties that established his employment.

2. Yes. Manulife fialed to cite a single iota of evidence to support its claims that there was gross and
habitual neglect of duties, inefficiency as well as willful disobedince of the lawful order of Manulife on the
part of Tongko. An employer may only terminate the services of an employee for a just cause which
must be supported by substantial evidence.

San Beda College of Law 109


4S: 2015 - 2016
LABOR LAW REVIEW Atty. Joyrich Golangco

30. TELEVISION AND PRODUCTION EXPONENTS, INC. and/or ANTONIO P. TUVIERA, petitioners, vs.
ROBERTO C. SERVAA
G.R. No. 167648 January 28, 2008
TINGA, J.:

Doctrine:
There is an employer-employee relationship when the person for whom the services are performed reserves the
right to control not only the end achieved but also the manner and means used to achieve that end.
FACTS:
TAPE is a domestic corporation engaged in the production of television programs while Antonio Tuviera serves
as its president. Roberto Servaa served as security guard for TAPE from 1987 until his services were
termitated on 3 March 2000. Servaa filed a complaint for illegal dismissal agianst TAPE.He alleged that he was
first connected with Agro-Commercial Security Agency but was later on absorbed by TAPE as a regular
company guard. His services were terminated on account of TAPEs decision to contract the services of a
professional security agency. Tape, on the other hand, alleged that Servaa was an independent contractor
falling under the talent group category and was working under a special arrangement. It alleged that it was
agreed that Servaa would render his services unitil such time that the company shall have engaged the
services of a professional security agency.

ISSUE: Whether or not there is an Employer-Employee relationship between TAPE and Servaa?

LA RULING: Yes. The Labor Arbiter ruled that Servaa was a regular employee of Tape on account of the
nature of the work of Servaa, which is securing and maintaining order in the studio, as necessary and desirable
in the usual business of TAPE. However, the Labor Aribter ruled the termination valid on the ground of
redundancy.

NLRC RULING: No. The NLRC reversed the ruling of the Labor Arbiter on the ground security services may not
be deemed necessary and desirable in the usual business of TAPE.

CA RULING: Yes. The CA ruled that that Servaa was a regular employee considering the nature and length of
his service.

SC RULING:
Yes. Jurisprudence is abound with cases that recite the factors to be considered in determining the existence of
employer-employee relationship, namely: (a) the selection and engagement of the employee; (b) the payment of
wages; (c) the power of dismissal; and (d) the employer's power to control the employee with respect to the
means and method by which the work is to be accomplished.
Servaa was hired by TAPE when the latter absorbed him upon the expiration of his security agency contract
with RPN-9. The monthly salary received by Servaa is considered wages despite being designated as talent
fees by TAPE. The Memorandum informing Servaa of discontinuance of his services also proves that TAPE
had the power to dismiss him. Control is also manifested in the bundy cards submitted by Servaa. He was
required to report daily and observe definite work hours. He is also considered a regular employee by reason of
his 5 year continuous service regardless of whether or not respondent had been performing work that is
necessary or desirable to the usual business of TAPE. Thus being a regular employee, his services may not be
terminated except for a just or authorized cause. TAPE is liable for illegal dismissal for it failure to comply the 1
month requirement for termination of services as required by law.

However, with respect to the liability of petitioner Tuviera, president of TAPE, absent any showing that he acted
with malice or bad faith in terminating respondent, he cannot be held solidarily liable with TAPE.

San Beda College of Law 110


4S: 2015 - 2016
LABOR LAW REVIEW Atty. Joyrich Golangco

31. ENCYCLOPAEDIA BRITANNICA (PHILIPPINES), INC. v. NATIONAL LABOR RELATIONS
COMMISSION, HON. LABOR ARBITER TEODORICO L. ROGELIO and BENJAMIN LIMJOCO
G.R. No. 87098 November 4, 1996
TORRES, JR., J.:

Doctrine:
The mere issuance of memoranda does not establish an Employer-Employee relationship.

FACTS:
Respondent Benjami Limjoco was a Sales Division Manager of petitioner Encyclopaedia Britannica. He was in
charge of selling its products through some sales represenatives and received commisions from the products
sold by his agents. His office expenses were deducted from his commissions and he was informed by petitioner
of appointment, promotions and transfers of employees in his district. He resigned from the said office on 14
June 1974 to pursue his private business but on 30 October 1975, he filed a complaint against petitioner for
non-payment of separation pay and other benefits and also illegal deduction form his sales commision.

Petitioner alleged that respondent is not its employee but an independent dealer. He did not have any salary
and his income from petitioner is depended on the volume of sales accomplished. He also maintained his own
office and his expenses are chargeable to his commissions. Petitioner further alleges that it had no control and
supervision over the respondent. Respondent, on the other hand, alleges that he was hired by petitioner and
was assigned in the sales department with an average of Php 4,000.00 monthly as earnings. He was under the
supervision of petitioner through the issuances of memoranda, guidelines on company policies, instructions and
other orders.

ISSUE: Whether or not there is an Employer-Employee Relationship between Encyclopaedia Britannica and
Limjoco?

LA RULING:
Yes. The Labor Arbiter ruled that Limjoco was under the control of the petitioner since he was required
to make periodic reports of his sales activities to the company and all transactions were subject to the final
approval of the petitioner.

NLRC RULING:
Yes. The NLRC found no evidence supporting the allegation that Limjoco was an independent
contractor or dealer. The petitioner dictated Limjoco how and where to sell its products.

SC RULING:
No. The fact that petitioner issued memoranda to Limjoco and to other division sales managers did not prove
that petitioner had actual control over them. These were merely guidelines on company policies, which the sales
managers follow and impose on their respective agents. Independent authorized agents who did not receive
regular compensations but commissions based on the sale of products primarily conducted the sales operation.
They also financed their own expenses and maintained their own staff.

The prices of the products may have been fixed but the independent agents still had free rein in the means and
methods for conducting the marketing operations. He was free to conduct his work and he was free to engage in
other means of livelihood. In fact, he was also a director and later president of the Farmers Rural Bank while he
was connected with the petitioner.

San Beda College of Law 111


4S: 2015 - 2016
LABOR LAW REVIEW Atty. Joyrich Golangco

32. ATOK BIG WEDGE COMPANY, INC. vs. JESUS P. GISON
G.R. No. 169510 August 8, 2011
PERALTA, J.:

Doctrine:
Article 280 of the Labor Code, is not the yardstick for determining the existence of an employment relationship
because it merely distinguished two kinds of employees that is regular and casual employees.

FACTS:
Atok Big Wedge Company, Inc. through it then Asst. Vice-President and Acting Manager, Rutillo Torres
engaged Jesus Gison as part-time consultant on retainer basis. He assisted Atok with matters pertaining to the
prosecution of cases against illegal surface occupants within the area covered by the companys mineral claims.
He likewise performed liaison work with several government agencies. He was not required to report to office on
a regular basis except when requested by the management. He received Php 3,000.00 a month as retainer fee,
which was delivered to him in his residence or in a local restaurant.

Gison requested Atok to cause his registration with the SSS considering he was getting old. Atok, however,
ignored his request, which prompted him to file a complaint against Atok before the SSS. Afterwards, his
services was terminated by Atok on the ground his services were no longer necessary. This prompted Gison to
file a complaint for illegal dismissal before the NLRC against Atok.

ISSUE: Whether or not there is an Employer-Employee Relationship between Atok and Gison?

LA RULING: No. The Labor Arbiter ruled that there is no Employer-Employee relationship.

NLRC RULING: No. The NLRC affirmed the ruling of the Labor Arbiter.

CA RULING: Yes. The CA annulled and set aside the decision of the NLRC. The CA opined that applying
Article 280 of the Labor Code Gison is deemed a regular employee of the petitioner after the lapse of one year
from his employment.

SC RULING:
No. There is the absence of the element of control on the part of Atok, which results to the conclusion of an
Employer-Employee relationship. He was not required to report everyday during regular office hours and his
monthly retainer fees were paid to him either at his residence or a local restaurant. He was also assigned tasks
to perform but Atok did not control the manner and methods by which Gison performed these tasks.

Article 280 of the Labor, which was used by the CA to support its findings, is not applicable in the case at bar.
The said provision merely distinguishes between two kinds of employment, i.e., regular employees and casual
employees, for the purposes of determining the right of an employee to certain benefits. It does not apply where
the existence of an employment relationship is in dispute. Therefore, it was erroneous for the CA to rely on the
said provision in determining whether an Employer-Employee relationship exists between Atok and Gison.

San Beda College of Law 112


4S: 2015 - 2016
LABOR LAW REVIEW Atty. Joyrich Golangco

33. DUMPIT-MURILLO vs. COURT OF APPEALS
G.R. No. 164652 June 8, 2007
QUISUMBING, J.:

Doctrine:
The assertion that a talent contract exists does not necessarily prevent a regular employment status.

FACTS:
Associated Broadcasting Company (ABC) hired Thelma Dumpit-Murillo as a newscaster and co-anchor for
Balitang-Balita under a Talent Contract for a period of three-months. The said contract was renewed multiple
times. When the last contract expired, Dumpit-Murillo sent a letter to Jose Javire, Vice President for News and
Public Affairs of ABC, informing the latter of her interest in renewing her contract. She sent another letter stating
that she was not able to receive any reply from her previous letter. She also stated that she considered the
failure of a formal response on the part of the company as her constructive dismissal. She then sent a demand
letter requesting her reinstatement and payment of unpaid wages and other benefits. ABC replied that the
checks for her talent fees are being processed but claimed that the other claims hand no basis. Dumpit-Murillo
filed a complaint against ABC for illegal dismissal before the NLRC.

ISSUES:
1. Whether or not there is an employer-employee relationship between ABC and Dumpit-Murillo?
2. Whether or not Dumpit-Murillo is a regular employee?

LA RULING: No. The Labor Arbiter dismissed the Complaint.

NLRC RULING: Yes. The NLRC held that an employer-employee relationship existed between Dumpit-Murillo
and ABS; that the subject talent contract was void; and that she was a regular employee illegally dismissed.

CA RULING: No. The CA reversed the decision of the NLRC. It ruled that Dumpit-Murillo was a fixed-term
employee and not a regular employee and should not be allowed to renege from the stipulation she had
voluntarily and knowingly executed.

SC RULING:
1. Yes. The practice of having fixed-term contracts in the industry does not automatically make all talent
contracts valid and compliant with labor law. The assertion that a talent contract exists does not
necessarily prevent a regular employment status. The duties of Dumpit-Murillo as enumerated in her
employment contract indicate that ABC had control over the work of petitioner. Aside from control, ABC
also dictated the work assignments and payment of her wages. ABC also had the power to dismiss.

2. Yes. Dumpit-Murillos work was necessary or desirable in the usual business or trade of the employer,
which includes its participation in the governments news and public information dissemination. Her
work was continuous for a period of four years and her repeated engagement under contract of hire is
indicative of the necessity and desirability of her work in ABCs business.

There is no valid fixed-term employment between Dumpit-Murillo and ABC. Fixed-term employment will
not be considered valid where, from the circumstances, it is apparent that periods have been imposed
to preclude acquisition of tenurial security by the employee. It should satisfactorily appear that the
employer and the employee dealt with each other on more or less equal terms with no moral dominance
being exercised by the employer over the employee. Patently, Dumpit-Murillo occupied a position of
weakness vis--vis the employer. She was merely one of the numerous newscasters/broadcasters of
ABC and she was left with no choice but to affix her signature of conformity on each renewal of her
contract or risk the loss of her job.

San Beda College of Law 113


4S: 2015 - 2016
LABOR LAW REVIEW Atty. Joyrich Golangco

34. JOSE MEL BERNARTE vs. PHILIPPINE BASKETBALL ASSOCIATION (PBA), JOSE EMMANUEL M.
EALA, and PERRY MARTINEZ
G.R. No. 192084 September 14, 2011
CARPIO, J

FACTS:
Complainants Jose Mel Bernarte and Renato Guevara were referees of the PBA. They claim that they had been
made to sign contracts on a year to year basis until 2003, when Bernarte was made to sign a one and a half
month contract for the period of July 1st to August 5th 2003. In January 2004, Bernarte Received a letter
advising him that his contract would not be renewed citing his unsatisfactory performance on and off the court.
Bernarte was shocked, and felt that the dismissal was caused by his refusal to fix a game upon order of Ernie
De Leon.

Complainant Guevarra, a referee since 2001, was likewise no longer made to sign a contract beginning
February 2004. Complainants aver that they were employees of the PBA and were illegally dismissed.

Respondents PBA aver that the complainants entered into contracts of retainer with the PBA which after the
lapse of their respective periods, were not renewed. Respondents argue that complainants were not illegally
dismissed because they were not employees of the PBA, that their respective contracts were simply not
renewed, and that the PBA had the prerogative of whether or not to renew their contracts.

ISSUE: Whether or not complainants are employees of the PBA.

LA RULING: In her 31 March 2005 Decision, the Labor Arbiter declared petitioner an employee whose
dismissal by respondents was illegal. Accordingly, the Labor Arbiter ordered the reinstatement of petitioner and
the payment of backwages, moral and exemplary damages and attorney's fees.

NLRC RULING: Affirmed the decision of the LA.


CA RULING: The Court of Appeals found petitioner an independent contractor since respondents did not
exercise any form of control over the means and methods by which petitioner performed his work as a
basketball referee. The Court of Appeals held:

While the NLRC agreed that the PBA has no control over the referees acts of blowing the whistle and
making calls during basketball games, it, nevertheless, theorized that the said acts refer to the means
and methods employed by the referees in officiating basketball games for the illogical reason that said
acts refer only to the referees skills. How could a skilled referee perform his job without blowing a
whistle and making calls? Worse, how can the PBA control the performance of work of a referee without
controlling his acts of blowing the whistle and making calls?
SC RULING:
The Petition is bereft of merit. To determine the existence of an employer-employee relationship, case law has
consistently applied the four-fold test, to wit: (a) the selection and engagement of the employee; (b) the payment
of wages; (c) the power of dismissal; and (d) the employers power to control the employee on the means and
methods by which the work is accomplished. The so-called control test is the most important indicator of the
presence or absence of an employer-employee relationship.

The contractual stipulations do not pertain to, much less dictate, how and when petitioner will blow the whistle
and make calls. On the contrary, they merely serve as rules of conduct or guidelines in order to maintain the
integrity of the professional basketball league. As correctly observed by the Court of Appeals, how could a
skilled referee perform his job without blowing a whistle and making calls? x x x [H]ow can the PBA control the
performance of work of a referee without controlling his acts of blowing the whistle and making calls?

We agree with respondents that once in the playing court, the referees exercise their own independent
judgment, based on the rules of the game, as to when and how a call or decision is to be made. The referees
decide whether an infraction was committed, and the PBA cannot overrule them once the decision is made on
the playing court. The referees are the only, absolute, and final authority on the playing court. Respondents or
any of the PBA officers cannot and do not determine which calls to make or not to make and cannot
control the referee when he blows the whistle because such authority exclusively belongs to the
referees. The very nature of petitioners job of officiating a professional basketball game undoubtedly calls for
freedom of control by respondents.
San Beda College of Law 114
4S: 2015 - 2016
LABOR LAW REVIEW Atty. Joyrich Golangco

Further, unlike regular employees who ordinarily report for work eight hours per day for five days a week,
petitioner is required to report for work only when PBA games are scheduled or three times a week at two hours
per game. In addition, there are no deductions for contributions to the Social Security System, Philhealth or Pag-
Ibig, which are the usual deductions from employees salaries. These undisputed circumstances buttress the fact
that petitioner is an independent contractor, and not an employee of respondents.

In addition, the fact that PBA repeatedly hired petitioner does not by itself prove that petitioner is an employee of
the former. For a hired party to be considered an employee, the hiring party must have control over the means
and methods by which the hired party is to perform his work, which is absent in this case. The continuous
rehiring by PBA of petitioner simply signifies the renewal of the contract between PBA and petitioner, and
highlights the satisfactory services rendered by petitioner warranting such contract renewal. Conversely, if PBA
decides to discontinue petitioners services at the end of the term fixed in the contract, whether for unsatisfactory
services, or violation of the terms and conditions of the contract, or for whatever other reason, the same merely
results in the non-renewal of the contract, as in the present case. The non-renewal of the contract between the
parties does not constitute illegal dismissal of petitioner by respondents.

San Beda College of Law 115


4S: 2015 - 2016
LABOR LAW REVIEW Atty. Joyrich Golangco

35. ANGEL JARDIN vs. NATIONAL LABOR RELATIONS COMMISSION (NLRC) and GOODMAN TAXI
(PHILJAMA INTERNATIONAL, INC.)
G.R. No. 119268 February 23, 2000
QUISUMBING, J.

FACTS:
Petitioners were drivers of private respondent, Philjama International Inc., a domestic corporation engaged in
the operation of "Goodman Taxi." Petitioners used to drive private respondent's taxicabs every other day on a
24-hour work schedule under the boundary system. Under this arrangement, the petitioners earned an average
of P400.00 daily. Nevertheless, private respondent admittedly regularly deducts from petitioners, daily earnings
the amount of P30.00 supposedly for the washing of the taxi units. Believing that the deduction is illegal,
petitioners decided to form a labor union to protect their rights and interests.

Upon learning about the plan of petitioners, private respondent refused to let petitioners drive their taxicabs
when they reported for work on August 6, 1991, and on succeeding days. Petitioners suspected that they were
singled out because they were the leaders and active members of the proposed union. Aggrieved, petitioners
filed with the labor arbiter a complaint against private respondent for unfair labor practice, illegal dismissal and
illegal deduction of washing fees.

ISSUE: Whether or not petitioners are employees of the respondent.

LA RULING: Dismissed the complaint for lack of merit.

NLRC RULING: At first, the NLRC reversed and set aside the judgment of the LA and declared that petitioners
are employees of private respondent, and as such, their dismissal must be for just cause and after due process.
However, after TWO motions for reconsideration, the NLRC ruled that it lacks jurisdiction over the case
as petitioners and private respondent have NO employer employee relationship. It held that the relationship of
the parties is leasehold which is covered by the Civil Code rather than the Labor Code.

SC RULING:
The petition is impressed with merit. The SC declared that the NLRC should not have entertained the
respondent's second motion for reconsideration, the same being a prohibited pleading under the NLRC rules.

As to the substantive issue, the SC ruled as follows:

In a number of cases decided by this Court, we ruled that the relationship between jeepney
owners/operators on one hand and jeepney drivers on the other under the boundary system is
that of employer-employee and not of lessor-lessee. We explained that in the lease of chattels,
the lessor loses complete control over the chattel leased although the lessee cannot be
reckless in the use thereof, otherwise he would be responsible for the damages to the lessor.

In the case of jeepney owners/operators and jeepney drivers, the former exercise supervision and
control over the latter. The management of the business is in the owner's hands. The owner as holder of the
certificate of public convenience must see to it that the driver follows the route prescribed by the franchising
authority and the rules promulgated as regards its operation. Now, the fact that the drivers do not receive fixed
wages but get only that in excess of the so-called "boundary" they pay to the owner/operator is not sufficient to
withdraw the relationship between them from that of employer and employee. We have applied by analogy the
abovestated doctrine to the relationships between bus owner/operator and bus conductor, auto-calesa
owner/operator and driver, and recently between taxi owners/operators and taxi drivers. Hence, petitioners are
undoubtedly employees of private respondent because as taxi drivers they perform activities which are usually
necessary or desirable in the usual business or trade of their employer.

San Beda College of Law 116


4S: 2015 - 2016
LABOR LAW REVIEW Atty. Joyrich Golangco

36. CHAVEZ v. NLRC
G.R. No. 146530 January 17, 2005
CALLEJO, SR., J.

Doctrine:
Of the four elements of the employer-employee relationship, the control test is the most important.

Compared to an employee, an independent contractor is one who carries on a distinct and independent
business and undertakes to perform the job, work, or service on its own account and under its own responsibility
according to its own manner and method, free from the control and direction of the principal in all matters
connected with the performance of the work except as to the results thereof. Hence, while an independent
contractor enjoys independence and freedom from the control and supervision of his principal, an employee is
subject to the employers power to control the means and methods by which the employees work is to be
performed and accomplished.

FACTS:
The respondent company, Supreme Packaging, Inc., is in the business of manufacturing cartons and other
packaging materials for export and distribution. On 1984, it engaged the services of the petitioner, Pedro
Chavez, as truck driver and as such, he was tasked to deliver the respondent companys products from its
factory to its various customers, mostly in Metro Manila.

Sometime in 1992, Chavez asked respondent companys plant manager his desire to avail himself of the
benefits that regular employees were receiving such as overtime pay, nightshift differential pay, and 13th month
pay, among others but the same was never given.

On 1995, Chavez filed a complaint for regularization with the Regional Arbitration Branch 3 but before the case
could be heard, respondent company terminated the services of Chavez prompting Chavez to amend the
complaint against the respondents for illegal dismissal, unfair labor practice and non-payment of overtime pay,
nightshift differential pay, 13th month pay, among others.

The respondents, for their part, denied the existence of an employer-employee relationship between the
respondent company and the petitioner. They averred that the petitioner was an independent contractor as
evidenced by the contract of service which he and the respondent company entered into.

ISSUE: Whether or not Chavez was respondent companys employee or was a private contractor.

LA RULING: The LA found Chavez to be respondent companys employee thus finding respondent guilty of
illegal dismissal. It held that the petitioner was a regular employee of the respondent company as he was
performing a service that was necessary and desirable to the latters business. Moreover, it was noted that the
petitioner had discharged his duties as truck driver for the respondent company for a continuous and
uninterrupted period of more than ten years.

NLRC RULING: The NLRC initially affirmed the LAs decision but later on reversed it decision declaring that no
employer-employee relationship existed. The NLRC stated that the respondents did not exercise control over
the means and methods by which the petitioner accomplished his delivery services. It upheld the validity of the
contract of service as it pointed out that said contract was silent as to the time by which the petitioner was to
make the deliveries and that the petitioner could hire his own helpers whose wages would be paid from his own
account.

CA RULING: Initially, the CA reversed the NLRCs decision ruling in favor of Chavez but later reconsidered the
same ruling in favor of respondent company. In reconsidering its decision, the CA explained that the extent of
control exercised by the respondents over the petitioner was only with respect to the result but not to the means
and methods used by him. The CA cited the following circumstances: (1) the respondents had no say on how
the goods were to be delivered to the customers; (2) the petitioner had the right to employ workers who would
be under his direct control; and (3) the petitioner had no working time.

The fact that the petitioner had been with the respondent company for more than ten years was, according to
the CA, of no moment because his status was determined not by the length of service but by the contract of
service. This contract, not being contrary to morals, good customs, public order or public policy, should be given
San Beda College of Law 117
4S: 2015 - 2016
LABOR LAW REVIEW Atty. Joyrich Golangco

the force and effect of law as between the respondent company and the petitioner. Consequently, the CA
reinstated the July 10, 1998 Decision of the NLRC dismissing the petitioners complaint for illegal dismissal.

SC RULING:
The court held that an employer-employee relationship existed and that Chavez was not a mere private
contractor.

Applying the four-fold test, the SC found:

First. Undeniably, it was the respondents who engaged the services of the petitioner without the
intervention of a third party.

Second. That the petitioner was paid on a per trip basis is not significant. This is merely a method of
computing compensation and not a basis for determining the existence or absence of employer-
employee relationship. One may be paid on the basis of results or time expended on the work, and may
or may not acquire an employment status, depending on whether the elements of an employer-
employee relationship are present or not. In this case, it cannot be gainsaid that the petitioner received
compensation from the respondent company for the services that he rendered to the latter.

Third. The respondents power to dismiss the petitioner was inherent in the fact that they engaged the
services of the petitioner as truck driver. They exercised this power by terminating the petitioners
services albeit in the guise of severance of contractual relation due allegedly to the latters breach of his
contractual obligation.

Fourth. As earlier opined, of the four elements of the employer-employee relationship, the control test is
the most important. Compared to an employee, an independent contractor is one who carries on a
distinct and independent business and undertakes to perform the job, work, or service on its own
account and under its own responsibility according to its own manner and method, free from the control
and direction of the principal in all matters connected with the performance of the work except as to the
results thereof. Hence, while an independent contractor enjoys independence and freedom from the
control and supervision of his principal, an employee is subject to the employers power to control the
means and methods by which the employees work is to be performed and accomplished.

San Beda College of Law 118


4S: 2015 - 2016
LABOR LAW REVIEW Atty. Joyrich Golangco

37. COCA-COLA BOTTLERS PHILS., INC. v. CLIMACO
G.R. No. 146881 February 5, 2007
AZCUNA, J.

Doctrine:
The Court, in determining the existence of an employer-employee relationship, has invariably adhered to the
four-fold test: (1) the selection and engagement of the employee; (2) the payment of wages; (3) the power of
dismissal; and (4) the power to control the employees conduct, or the so-called control test, considered to be
the most important element. The issuance by the principal of guidelines does not establish control by principal.

FACTS:
Dr. Climaco is a medical doctor who was hired by the petitioner by virtue of retainer agreement. The agreement
states that there is no employer-employee relationship between the parties. The retainer agreement was
renewed annually. The last one expired on Dec. 31, 1993. Despite of the non-renewal of the agreement,
respondent continued to perform his functions as company doctor until he received a letter in March 1995
concluding their retainer agreement.

Respondent filed a complaint before the NLRC seeking recognition as a regular employee of the petitioner
company and prayed for the payment of all benefits of a regular employee.

ISSUE: Whether or not an employer-employee relationship existed between petitioner Coca-Cola Bottlers and
respondent Dr. Climaco.

LA AND NLRC RULING: The Labor Arbiter and the NLRC found that the company lacked the power of control
over Dr. Climaco, therefore no employer-employee relationship existed.

CA RULING: Court of Appeals ruled that there existed an employer-employee relationship. It held that Coca-
Colas power to control petitioner is present because the particular objectives and activities to be observed and
accomplished by the latter are fixed and set under the Comprehensive Medical Plan which was made an
integral part of the retainer agreement. Moreover, the times for accomplishing these objectives and activities are
likewise controlled and determined by the company. Petitioner is subject to definite hours of work, and due to
this, he performs his duties to Coca-Cola not at his own pleasure but according to the schedule dictated by the
company.

The CA added that Dr. Climaco should be classified as a regular employee having rendered 6 years of
service as plant physician by virtue of several renewed retainer agreements.

SC RULING:
The court held no, upholding the decisions of both the LA and the NLRC. The Court, in determining the
existence of an employer-employee relationship, has invariably adhered to the four-fold test: (1) the selection
and engagement of the employee; (2) the payment of wages; (3) the power of dismissal; and (4) the power to
control the employees conduct, or the so-called "control test," considered to be the most important element.

The Labor Arbiter and the NLRC correctly found that Coca Cola lacked the power of control over the
performance by respondent of his duties. The petitioner company, through the Comprehensive Medical Plan,
provided guidelines merely to ensure that the end result was achieved, but did not control the means and
methods by which respondent performed his assigned tasks.

The NLRC affirmed the findings of the Labor Arbiter and stated that it is precisely because the company lacks
the power of control that the contract provides that respondent shall be directly responsible to the employee
concerned and their dependents for any injury, harm or damage caused through professional negligence,
incompetence or other valid causes of action.

In addition, the Court finds that the schedule of work and the requirement to be on call for emergency cases do
not amount to such control, but are necessary incidents to the Retainership Agreement.

The Court agrees with the Labor Arbiter and the NLRC that there is nothing wrong with the employment of
respondent as a retained physician of petitioner company and upholds the validity of the Retainership
Agreement which clearly stated that no employer-employee relationship existed between the parties.
San Beda College of Law 119
4S: 2015 - 2016
LABOR LAW REVIEW Atty. Joyrich Golangco

38. GABRIEL v. BRILON
G.R. No. 146989 February 7, 2007
AZCUNA, J.:

Doctrine:
The relationship between jeepney owners/operators and jeepney drivers under the boundary system is that of
employer-employee and not of lessor-lessee because in the lease of chattels the lessor loses complete control
over the chattel leased although the lessee cannot be reckless in the use thereof, otherwise he would be
responsible for the damages to the lessor. In the case of jeepney owners/operators and jeepney drivers, the
former exercises supervision and control over the latter.

FACTS:
Petitioner, represented by his surviving spouse, Flordeliza V. Gabriel, was the owner-operator of a public
transport business, "Gabriel Jeepney," with a fleet of 54 jeepneys plying the Baclaran-Divisoria-Tondo route.
Petitioner had a pool of drivers, which included respondents, operating under a "boundary system" of P400 per
day.

Respondents alleged that they were regular drivers of Gabriel Jeepney under a boundary system of P400 per
day, plying Baclaran to Divisoria via Tondo, and vice versa. They added that despite the fact that there is no law
providing that the operator can require the drivers to pay police protection, deposit, washing, and garage fees,
they were forced to pay additional P55.00 per day for the following: a) P20.00 police protection; b) P20.00
washing; c) P10.00 deposit; and [d)] P5.00 garage fees. Respondents further alleged that on April 1995,
petitioner told them not to drive anymore, and when they went to the garage to report for work the next day, they
were not given a unit to drive.

Based on the foregoing, respondents filed an action for illegal dismissal, illegal deductions, and separation pay
against petitioner Gabriel with the NLRC.

ISSUE: Whether or not an employer-employee relationship existed between Gabriel and its jeepney drivers
considering that the latter worked for the former under a boundary system.

LA RULING: The Labor Arbiter ruled in favor of the respondents-jeepney drivers declaring the illegality of
respondents dismissal and ordered Melencio Gabriel to pay the respondents the sum of PHP1,034,000
representing respondents backwages and separation pay.

NLRC RULING: The NLRC Division reversed and set aside the LAs decision for lack of employer-employee
relationship.

CA RULING: The CA reversed the NLRCs decision and held that an employer-employee relationship existed
between Gabriel and the respondent-jeepney drivers. The CA iterated that the NLRCs decision is egregiously
wrong insofar as it was anchored on the absence of an employer-employee relationship. Well-settled is the rule
that the boundary system used in jeepney and (taxi) operations presupposes an employer-employee
relationship (National Labor Union v. Dinglasan, 98 Phil. 649)

SC RULING:
The SC upheld the CAs decision reiterating that the relationship between jeepney owners/operators and
jeepney drivers under the boundary system is that of employer-employee and not of lessor-lessee because in
the lease of chattels the lessor loses complete control over the chattel leased although the lessee cannot be
reckless in the use thereof, otherwise he would be responsible for the damages to the lessor. In the case of
jeepney owners/operators and jeepney drivers, the former exercises supervision and control over the latter.
The fact that the drivers do not receive fixed wages but get only that in excess of the so-called "boundary" that
they pay to the owner/operator is not sufficient to withdraw the relationship between them from that of employer
and employee. Thus, private respondents were employees because they had been engaged to perform
activities which were usually necessary or desirable in the usual business or trade of the employer.

The Court also agrees with the labor arbiter and the CA that respondents were illegally dismissed by petitioner.
Respondents were not accorded due process. Moreover, petitioner failed to show that the cause for termination
falls under any of the grounds enumerated in Article 282 of the Labor Code. Consequently, respondents are
entitled to reinstatement without loss of seniority rights and other privileges and to their full backwages
San Beda College of Law 120
4S: 2015 - 2016
LABOR LAW REVIEW Atty. Joyrich Golangco

computed from the date of dismissal up to the time of their actual reinstatement in accordance with Article 279
of the Labor Code.

The SC also awarded reinstatement if favor of the respondents ruling that Reinstatement is obtainable in this
case because it has not been shown that there is an ensuing "strained relations" between petitioner and
respondents. This is pursuant to the principle laid down in Globe-Mackay Cable and Radio Corporation v. NLRC
as quoted earlier in the CA decision.

San Beda College of Law 121


4S: 2015 - 2016
LABOR LAW REVIEW Atty. Joyrich Golangco

39. FELIX VS. BUENSANEDA
G.R. No. 109704 January 17, 1995
KAPUNAN, J.:

Doctrine:
A residency or resident physician position in a medical specialty is never a permanent one. Residency connotes
training and temporary status. It is the step taken by a physician right after post-graduate internship (and after
hurdling the Medical Licensure Examinations) prior to his recognition as a specialist or sub-specialist in a given
field.

FACTS:
Petitioner Dr. Alfredo Felix, after passing the Physician's Licensure Examinations given by the Professional
Regulation Commission in June of 1979, joined the National Center for Mental Health (then the National Mental
Hospital) on May 26, 1980 as a Resident Physician. He was later on promoted to the position of Senior Resident
Physician in a temporary capacity immediately after he and other employees of the NCMH allegedly tendered
their courtesy resignations to the Secretary of Health on January 1983 pursuant to a reorganization act, EO No.
119. He was again promoted to the position of Medical Specialist I (Temporary Status), which position was
renewed the following year on August 1988.

In the same year, 1988, the DOH subsequently issued Department Order No. 347 which required board
certification as a prerequisite for renewal of specialist positions in various medical centers, hospitals and
agencies of the said department. Petitioner was one of the hundreds of government medical specialist who was
subjected to such certification requirement for them to enable to continue to work in their present positions.

On 1991, after reviewing petitioner's service record and performance, the Medical Credentials Committee of the
National Center for Mental Health recommended non-renewal of his appointment as Medical Specialist I. He
was, however, allowed to continue in the service, and receive his salary, allowances and other benefits even
after being informed of the termination of his appointment. A subsequent meeting took place and discussed the
Dr. Felixs status. Dr. Felixs immediate supervisor, pointed out his poor performance, frequent tardiness and
inflexibility as among the factors responsible for the recommendation not to renew his appointment. With one
exception, other department heads present in the meeting expressed the same opinion, and the overwhelming
concensus was for non-renewal.

After having been issued a memorandum ordering Dr. Felix to vacate his cottage, he filed a petition with the
Merit System Protection Board (MSPB) complaining about the alleged harassment by respondents and
questioning the non-renewal of his appointment.

MSPB RULING: The MPSB dismissed Dr. Felixs complaint for lack of merit finding that as an apparent incident
of the power to appoint, the renewal of a temporary appointment upon or after its expiration is a matter largely
addressed to the sound discretion of the appointing authority. Complainant therefore, has no basis in law to
assail the non-renewal of his expired temporary appointment much less invoke the aid of this Board cannot
substitute its judgment to that of the appointing authority nor direct the latter to issue an appointment in the
complainant's favor. Dr. Felix then appealed to the Civil Service Commission.

CSC DECISION: The CSC dismissed the appeal and denied Dr. Felixs motion for reconsideration.

ISSUE: Whether or not Dr. Alfredo Felixs dismissal was illegal and violative of the constitutional provision on
security of tenure allegedly because his removal was made pursuant to an invalid reorganization.

SC DECISION:
The court held no. The court held that the patent absurdity of petitioner's posture is readily obvious.
A residency or resident physician position in a medical specialty is never a permanent one. Residency connotes
training and temporary status. It is the step taken by a physician right after post-graduate internship (and after
hurdling the Medical Licensure Examinations) prior to his recognition as a specialist or sub-specialist in a given
field.

Petitioner's insistence on being reverted back to the status quo prior to the reorganizations made pursuant to
Executive Order No. 119 would therefore be akin to a college student asking to be sent back to high school and
staying there. From the position of senior resident physician, which he held at the time of the government
San Beda College of Law 122
4S: 2015 - 2016
LABOR LAW REVIEW Atty. Joyrich Golangco

reorganization, the next logical step in the stepladder process was obviously his promotion to the rank of
Medical Specialist I, a position which he apparently accepted not only because of the increase in salary and
rank but because of the prestige and status which the promotion conferred upon him in the medical community.

Such status, however, clearly carried with it certain professional responsibilities including the responsibility of
keeping up with the minimum requirements of specialty rank, the responsibility of keeping abreast with current
knowledge in his specialty rank, the responsibility of completing board certification requirements within a
reasonable period of time. The evaluation made by the petitioner's peers and superiors clearly showed that he
was deficient in a lot of areas, in addition to the fact that at the time of his non-renewal, he was not even board-
certified.

The court also took notice of the fact that petitioner made no attempt to oppose earlier renewals of his
temporary Specialist I contracts, clearly demonstrating his acquiescence to if not his unqualified acceptance
of the promotion (albeit of a temporary nature) made in 1988. Whatever objections petitioner had against the
earlier change from the status of permanent senior resident physician to temporary senior physician were
neither pursued nor mentioned at or after his designation as Medical Specialist I (Temporary).

The court ruled then that he is therefore estopped from insisting upon a right or claim which he had plainly
abandoned when he, from all indications, enthusiastically accepted the promotion. His negligence to assert his
claim within a reasonable time, coupled with his failure to repudiate his promotion to a temporary position,
warrants a presumption, in the words of this Court in Tijam vs. Sibonghanoy, that he "either abandoned (his
claim) or declined to assert it."

San Beda College of Law 123


4S: 2015 - 2016
LABOR LAW REVIEW Atty. Joyrich Golangco

40. AUTO BUS TRANSPORT SYSTEM, INC. VS. BAUTISTA
G.R. No. 156367 May 16, 2005
CHICO-NAZARIO, J.:

Doctrine:
The term field personnel is not merely concerned with the location where the employee regularly performs his
duties but also with the fact that the employees performance is unsupervised by the employer. Thus, in order to
conclude whether an employee is a field employee, it is also necessary to ascertain if actual hours of work in the
field can be determined with reasonable certainty by the employer.

FACTS:
Respondent Antonio Bautista was employed with petitioner Auto Bus Transport System, Inc. since May 24,
1995 as a driver-conductor of the latters bus. Bautista was paid on commission basis per travel on a twice a
month basis. On January 3, 2000, the bus driven by Bautista accidentally bumped another bus owned by the
respondent. As a result, Auto Bus did not allow Bautista to work until he paid the cost of the repair of the
damaged bus. Bautista failed to pay and after given the opportunity to explain his side, Auto Bus sent him a
letter for termination. Bautista then instituted a Complaint for Illegal Dismissal with Money Claims for
nonpayment of 13th month pay and service incentive leave pay (SILP) against Auto Bus.

ISSUE: Whether or not Antonio Bautista is considered a field personnel thus determinative of his service
incentive leave pay entitlement.

LA RULING: Labor Arbiter Tabingan decided on the case in favor of Auto Bus, dimissing the Complaint of
Bautista. However, the LA ordered Auto Bus to pay Bautista his 13th month pay from the date of his hiring to the
date of his dismissal and his SILP for all the years he has been in service for the former.

NLRC RULING: The NLRC affirmed with modification the LAs decision. It held that Bautista, being an
employee paid on commission basis, was not entitled for 13th month pay in accordance with Section 3 of the
Rules and Regulations Implementing PD No. 851, leaving Bautista with a claim for his SILP.

The NLRC also denied petitioners motion for reconsideration in which petitioner denied their liability to pay
Bautista of his SILP contending that that Bautista, being a field personnel, was an exception to the rule that
employees are entitled to SILP. As a legal basis, petitioner cited Section 1(d), Rule V, Book 3 of the
Implementing Rules and Regulations of the Labor Code which delimits the grant of the SIL, excluding among
others field personnel and other employees whose performance is unsupervised by the employer including
those who are engaged on task or contract basis, purely commission basis, or those who are paid in a fixed
amount for performing work irrespective of the time consumed in the performance thereof.

CA RULING: The CA affirmed the NLRCs decision.

SC RULING:
The Court held no. According to Article 82 of the Labor Code, field personnel shall refer to non-agricultural
employees who regularly perform their duties away from the principal place of business or branch office of the
employer and whose actual hours of work in the field cannot be determined with reasonable certainty. The term
field personnel is not merely concerned with the location where the employee regularly performs his duties but
also with the fact that the employees performance is unsupervised by the employer. Thus, in order to conclude
whether an employee is a field employee, it is also necessary to ascertain if actual hours of work in the field can
be determined with reasonable certainty by the employer.

In the case of Bautista, it was observed in the facts found by the LA that he must be at a specific place in a
specified time to be able to observe prompt departure and arrival from his point of origin to his point of
destination. In each and every depot, there is always a dispatcher whose function is to see to it that Bautistas
bus and its crew leave the premises at specific time and arrive at the estimated proper time. Therefore, Bautista
was under constant supervision while in the performance of his work. In conclusion, he was not a field personnel
but a regular employee who performs tasks usually necessary and desirable to the usual trade of Auto Bus.
Thus, being a regular employee, he has the right to claim service incentive leave pay under Article 95 of the
Labor Code.

San Beda College of Law 124


4S: 2015 - 2016
LABOR LAW REVIEW Atty. Joyrich Golangco

41. ARIEL L. DAVID, doing business under the name and style "YIELS HOG DEALER vs. JOHN G.
MACASIO
G.R. No. 195466 July 2, 2014
BRION, J.:

Doctrine:
Engagement in a pakyaw or task basis does not negate the existence of employer-employee relationship.

FACTS:
Macasio filed a complaint against David, doing business under the name and style Yiels Hog Dealer, for non-
payment of overtime pay, holiday pay, 13th month pay, and SIL plus moral and exemplary damages and
attorneys fees.

Macasio alleged that he has been working as a butcher for David. Macasio claimed that David exercised control
and supervision over his work because David:
1. Set the work day, reporting time and hogs to be chopped, as well as the manner by which he was to
perform his work;
2. Daily paid his salary of P700.00;
3. Approved and disapproved his leaves; and
4. Owned the hogs delivered for chopping, as well as the work tools and implements and also rented the
workplace.
On the other hand, David claimed that he hired Macasio on pakyaw or task basis thus he is not entitled to the
benefits claimed. David pointed out that Macasios work starts at 10:00pm-2:00am depending on the volume of
hogs delivered. Macasio was paid a fixed amount regardless of the number of hogs chopped but was not
engaged to work, and accordingly not paid, when no hogs are delivered.

To support his claims, Macasio presented the Certificate of Employment (COE) issued to him by David and
likewise faulted David for not presenting as evidence the DTRs and payrolls which could have easily
established Macasios claims. David, however, insists that Macasio was not his employee, as he was engaged
in a pakyaw or task basis and that the COE was issued only for overseas employment purposes.

LA RULING: The LA dimissed the complaint banking on the argument of David that Macasio was merely
engaged in a pakyaw or task basis. Accordingly, Macasio is not entitled to the monetary awards.

NLRC RULING: Affirmed LA ruling. It ruled that Macasio was not covered by the Labor Standards on the
awards claimed because he was paid by results.

CA RULING: The CA modified the NLRC ruling. While agreeing that Macasio was paid by results, this did not
preclude the award of the benefits sought by Macasio. The CA ruled that he will only be excluded from the
th
coverage of the holiday, SIL, 13 month pay only if he is a field personnel, which are lacking in Macasios case.

On appeal to the SC, David alleges, among others, engagement on a pakyaw or task basis precludes the
creation of employer-employee relationship.

ISSUE: Whether engagement on pakyaw or task basis negates the existence of employer-employee
relationship between them the parties involved.

SC RULING:
No. Engagement in pakyaw or task basis does not characterize the relationship between the parties whether
employment or independent contractorship. It only determines the manner of calculation of the wages due to the
employee which, is in this case, is the quantity or quality of work done.
Moreover, employing the control test, employer-employee relationship exists in this case as shown by the
following circumstances:
1. David engaged the services of Macasio;
2. David paid Macasios wages;
3. David had been setting the day and time when Macasio should report for work;
4. David rents the place where Macasio had been performing his tasks;
5. Macasio would leave the workplace only after he had finished chopping all of the hog meats given to
him for the days task; and
San Beda College of Law 125
4S: 2015 - 2016
LABOR LAW REVIEW Atty. Joyrich Golangco

6. David would still engage Macasios services and have him report for work even during the days when
only few hogs were delivered for butchering.
The totality of the surrounding circumstances of the present case sufficiently points to an employer-
employee relationship existing between David and Macasio.

San Beda College of Law 126


4S: 2015 - 2016
LABOR LAW REVIEW Atty. Joyrich Golangco

42. BEGINO V. ABS-CBN
G.R. No. 199166 April 20, 2015
PEREZ, J.:

Doctrine:
Exclusivity Clause and Prohibitions in talent contracts are indicative of control by the employer if it does not
concern well-known television and radio personality who can legitimately be considered as talent and
compensated as such.

FACTS:
ABS-CBN employed Begino and Del Valle sometime in 1996 as Cameramen/Editors for TV Broadcasting.
Sumayao Avila-Llorin were similarly engaged as reporters sometime in 1996 and 2002, respectively. [hereinafter
referred to as petitioners] Petitioner were engaged through Talent Contracts which, though regularly renewed
over the years, provided terms ranging from three (3) months to one (1) year. Petitioners were given Project
Assignment Forms which detailed, among other matters, the duration of a particular project as well as the
budget and the daily technical requirements thereof. In the aforesaid capacities, petitioners were tasked with
coverage of news items for subsequent daily airings in respondents TV Patrol Bicol Program.

The Talent Contract specified the absence of employer-employee relationship between the parties and
mandated compliance with the professional standards of ABS-CBN and its policies and guidelines as well as the
rules of KBP. It also prohibited the petitioners from engaging in similar work for persons or entities in direct or
indirect competition with ABS-CBN. Petitioners compensation were termed as Talent Fees and were results
oriented in nature, thus petitioners were not required to observe normal working hours.

Claiming that they were regular employees, petitioners filed a complaint against ABS-CBN before the NLRC S-
RAB Naga City. Petitioners claimed that they performed functions necessary and desirable in ABS-CBN's
business. Petitioners averred that they worked under the direct control and supervision of Villafuerte, ABS-
CBNs manager, because they were mandated to wear company IDs and the latter provided all the equipment
they needed, and, at the end of each day, were informed about the news to be covered the following day, the
routes they were to take and, whenever the subject of their news coverage is quite distant, even the start of their
workday. Moreover, noncompliance with the company policies will merit dismissal. Petitioners were constantly
evaluated and were subjected to annual competency assessment alongside other ABS-CBN employees.

As a result of their denomination as talents, they merely earned an average of P7,000.00 to P8,000.00 per
month, or decidedly lower than the P21,773.00 monthly salary ABS-CBN paid its regular rank-and-file
employees.

ABS-CBN contends that, due to the lack of manpower to produce its own programs, it is necessary to hire
independent contractors who offered their services in relation to a particular program. Due to the unpredictability
of viewer preferences, their payment usually depends on the budget allocation for a project.

It argued that its control is limited to the imposition of general guidelines on conduct and performance, simply for
the purpose of upholding the standards of the company and the strictures of the industry. There is no control or
restrictions over the means and methods by which they performed or discharged the tasks for which their
services were engaged. Petitioners were, at most, briefed whenever necessary regarding the general
requirements of the project to be executed.

LA RULING: The LA ruled that petitioners were regular employees having rendered services necessary and
related to ABS-CBNs business for more than a year. It ruled that the exclusivity and prohibitions in the contract
showed ABS-CBNs control over petitioners.

NLRC RULING: The NLRC affirmed LA decision.

CA RULING: The CA discounted the existence of an employer-employee relation between the parties upon the
following findings and conclusions: (a) petitioners, were engaged by respondents as talents for periods, work
and the program specified in the Talent Contracts and/or Project Assignment Forms concluded between them;
(b) petitioners were paid talent fees depending on the budget allocated for the program to which they were
assigned; (c) being respondents did not exercise control over the manner and method by which petitioner
accomplished their work but only ensured that they complied with the standards of the company, the KBP and
San Beda College of Law 127
4S: 2015 - 2016
LABOR LAW REVIEW Atty. Joyrich Golangco

the industry; and, (d) the existence of an employer-employee relationship is not necessarily established by the
exclusivity clause and prohibitions which are but terms and conditions on which the parties are allowed to freely
stipulate.

ISSUE: Whether an employer-employee relationship exists between petitioners and ABS-CBN.

SC RULING:
Yes. Notwithstanding the nomenclature of their Talent Contracts and/or Project Assignment Forms and the
terms and condition embodied therein, petitioners are regular employees of ABS-CBN because they perform
functions necessary and essential to ABS-CBNs business. Respondents repeated hiring of petitioners for its
long-running news program positively indicates that the latter were ABS-CBNs regular employees.

Petitioners were subject to the control and supervision of respondents which, first and foremost, provided them
with the equipments essential for the discharge of their functions. The talent contracts specifically provide that
ABS-CBN shall retain all creative, administrative, financial and legal control of the programs which were
assigned to petitioners. They were likewise required to attend and participate in all promotional or
merchandising campaigns, activities or events for the Program, and to perform their functions at such locations
and Performance/Exhibition Schedules. Such terms demonstrate the control over petitioners not only over the
results but also over the means employed to achieve the same.

While it is true that in Sonza, where similar exclusivity clause and restrictions were held not to be indicative of
control and lead to the conclusion that Sonza was an independent contractor, such cannot be applied in this
case. The said case enunciated that guidelines for the achievement of mutually desired results are not
tantamount to control. It cannot not be applied in this case because Sonza case involved a well-known television
and radio personality who was legitimately considered a talent and amply compensated as such. While
possessed of skills for which they were modestly recompensed by respondents, petitioners lay no claim to fame
and/or unique talents for which talents like actors and personalities are hired and generally compensated in the
broadcast industry.

San Beda College of Law 128


4S: 2015 - 2016
LABOR LAW REVIEW Atty. Joyrich Golangco

ARTICLE 95

43. AUTO BUS TRANSPORT SYSTEM INC. V. BAUTISTA


G.R. No. 156367 May 16, 2005
CHICO-NAZARIO, J.:

Doctrine:
The three (3)-year prescriptive period for SIL commences, not at the end of the year when the employee
becomes entitled to the commutation of his SIL, but from the time when the employer refuses to pay its
monetary equivalent after demand of commutation or upon termination of the employees services, as the case
may be.

FACTS:
Antonio Bautista has been employed by petitioner Auto Bus Transport Systems, Inc. (Autobus), as driver-
conductor and was paid on commission basis at the rate of 7% of the total gross income per travel. Bautista,
while driving petitioners bus along Sta. Fe, Nueva Vizcaya, the bus he was driving accidentally bumped the rear
portion of another bus of petitioner, as the latter vehicle suddenly stopped at a sharp curve without giving any
warning.

After a month, Bautista was terminated for failing to pay 30% of the cost of the repairs. Thus, Bautista instituted
th
a complaint for Illegal dismissal with Money Claims for nonpayment of 13 month pay and service incentive
leave pay against Autobus.

Autobus, on the other hand, maintained that Bautistas employment was replete with offenses involving reckless
imprudence, gross negligence, and dishonesty. To support its claim, petitioner presented copies of letters,
memos, irregularity reports, and warrants of arrest pertaining to several incidents wherein respondent was
involved. It likewise claimed to have afforded Bautista opportunity to explain his side.
th
LA RULING: The LA ruled that there was no illegal dismissal but ordered petitioner to pay Bautista his 13
month pay and SIL.
th
NLRC RULING: The NLRC deleted the award of 13 month pay but retained the award of SIL.

CA RULING: The CA affirmed in toto the decision of the NLRC.

ISSUES:
1. Whether Bautista is entitled to SIL.
2. Whether 3 year prescriptive period under Art. 291 is applicable to Bautistas SIL.

SC RULING:
1. Yes. As a rule, SIL shall not apply to employees classified as field personnel. The phrase other employees
whose performance is unsupervised by the employer must not be understood as a separate classification of
employees to which service incentive leave shall not be granted. Rather, it serves as an amplification of the
interpretation of the definition of field personnel under the Labor Code as those whose actual hours of work in
the field cannot be determined with reasonable certainty.

The same is true with respect to the phrase those who are engaged on task or contract basis, purely
commission basis. Said phrase should be related with field personnel, applying the rule on ejusdem generis that
general and unlimited terms are restrained and limited by the particular terms that they follow. Hence,
employees engaged on task or contract basis or paid on purely commission basis are not automatically
exempted from the grant of service incentive leave, unless, they fall under the classification of field personnel.

Accordingly, the mere fact that Bautista is paid purely on a commission basis does not deprive him entitlement
to SIL.

Bautista cannot be considered as field personnel because the definition of field personnel is not merely
concerned with the location where the employee regularly performs his duties but also with the fact that the
employees performance is unsupervised by the employer.

San Beda College of Law 129


4S: 2015 - 2016
LABOR LAW REVIEW Atty. Joyrich Golangco

Along the routes that are plied by these bus companies, there are its inspectors assigned at strategic places
who board the bus and inspect the passengers, the punched tickets, and the conductors reports. There is also
the mandatory once-a-week car barn or shop day, where the bus is regularly checked as to its mechanical,
electrical, and hydraulic aspects, whether or not there are problems thereon as reported by the driver and/or
conductor. They too, must be at specific place as specified time, as they generally observe prompt departure
and arrival from their point of origin to their point of destination. In each and every depot, there is always the
Dispatcher whose function is precisely to see to it that the bus and its crew leave the premises at specific times
and arrive at the estimated proper time. Bautista, was therefore under constant supervision while in the
performance of this work.

2. Yes. As such, in the computation of the three-year prescriptive period, a determination must be made as to
the period when the act constituting a violation of the workers right to the benefits being claimed was
committed. In the case of service incentive leave, the employee may choose to either use his leave credits or
commute it to its monetary equivalent if not exhausted at the end of the year. Furthermore, if the employee
entitled to service incentive leave does not use or commute the same, he is entitled upon his resignation or
separation from work to the commutation of his accrued service incentive leave.

Thus, the three (3)-year prescriptive period commences, not at the end of the year when the employee becomes
entitled to the commutation of his service incentive leave, but from the time when the employer refuses to pay its
monetary equivalent after demand of commutation or upon termination of the employees services, as the case
may be.

Bautista had not made use of his service incentive leave nor demanded for its commutation until his
employment was terminated by petitioner. Neither did he compensate his accumulated service incentive leave
pay at the time of his dismissal. It was only upon his filing of a complaint for illegal dismissal, one month from
the time of his dismissal, that respondent demanded from his former employer commutation of his accumulated
leave credits. His cause of action to claim the payment of his accumulated service incentive leave thus accrued
from the time when his employer dismissed him and failed to pay his accumulated leave credits. It cannot be
denied that his cause of action did not prescribe.

San Beda College of Law 130


4S: 2015 - 2016
LABOR LAW REVIEW Atty. Joyrich Golangco

ARTICLE 97

44. SONGCO ET. AL V. NATIONAL LABOR RELATIONS COMMISSION


G.R. No. 50999-51000 March 23, 1990
MEDIALDEA J.:

Doctrines:
1. In computing the basis for paying separation pay, commissions and allowances shall be added to the
basic monthly salary.

2. The average commissions earned by a salesman during their last year of employment should be used
in computing the separation pay.

FACTS:
Petitioners are in the sales force of Zuellig. They received monthly salaries of at least P400.00. In addition, they
received commissions for every sale they made.

Zuellig filed with the DOLE an application seeking clearance to terminate the services of petitioners allegedly on
the ground of retrenchment due to financial losses. Initially, petitioners opposed the dismissal on the ground that
they are dismissed for being part of the union. Later, they agreed that the sole issue to be resolved is the basis
of the separation pay due to them.

Petitioners maintain that their earned sales commissions and allowances should be added together with their
salary to arrive at the basis for computing separation pay, citing Article 97(f) of the Labor Code. Zuellig on the
other hand argues that in the said article the term wage, commission is used only as one of the features or
designations attached to the word remuneration or earnings.

ISSUES:
1. Whether the allowances should be included in the monthly salary of petitioners for the purpose of
computation of their separation pay; and
2. Whether the sales commissions should be included in the monthly salary of petitioners for the purpose of
computation of their separation pay.

LA RULING: The basis of separation pay shall be equivalent to their one month salary (exclusive of
commissions, allowances, etc.) for every year in service that they have worked in with the company.

The appeal to the NLRC was dismissed for lack of merit.

SC RULING:
1. Yes. In computing the basis for separation pay of a dismissed employee, allowances should be included in
the monthly salary. This has been settled in the case of Santos v. NLRC, et al. (GR No. 76721. September 21,
1987) where the SC ruled that in the computation of backwages and separation pay, account must be taken not
only of the basic salary but also of her transportation and emergency living allowances.

2. Yes. Article 97(f) by itself is explicit that commission is included in the definition of the term wage. The law
speaks in clear and categorical language, there is no room for interpretation or construction. Said Article
provides:

(f) Wage paid to any employee shall mean the remuneration or earnings, however designated,
capable of being expressed in terms of money, whether fixed or ascertained on a time, task, piece,
or commission basis, or other method calculating the same, which is payable by an employer to an
employee under a written or unwritten contract of employment for wok done or to be done, or for
services rendered or to be rendered, and includes the fair and reasonable value, as determined by
the Secretary of Labor, of board, lodging, or other facilities customarily furnished by the employer to
the employee. Fair and reasonable value shall not include any profit to the employer or to any
person affiliated with the employer.

Granting, in grantia argumenti, that the commissions were in the form of incentives or encouragement, so that
the petitioners would be inspired to put a little more industry on the jobs particularly assigned to them, still these
San Beda College of Law 131
4S: 2015 - 2016
LABOR LAW REVIEW Atty. Joyrich Golangco

commissions are direct remunerations for services rendered which contributed to the increase of income of
Zuellig. Commission is the recompense, compensation or reward of an agent, salesman, executor, trustee,
receiver, factor, broker or bailee, when the same is calculated as a percentage on the amount of his
transactions or on the profit to the Principal. The nature of the work of a salesman and the reason for such tyoe
of remuneration for services rendered demonstrate clearly that commissions are part of petitioners wage or
salary.

In computation thereof, what should be taken into account is the average commissions earned during their last
year of employment.
45. MILLARES ET AL V. NLRC
GR No. 122827 March 29, 1999
BELLOSILLO, J.:

Doctrines:
1. In determining whether a privilege is a facility, the criterion is not so much its kind but its purpose.
2. The Sec. of Labor may from time to time fix in appropriate issuances the fair and reasonable value of
board, lodging and other facilities customarily furnished by an employer to his employees.
3. Separation pay when awarded to an illegally dismissed employee should be computed based not only on
the basic salary but also on the regular allowances that the employee had been receiving.

FACTS:
Petitioners numbering 116 occupied positions of Technical Staff, Unit Manager, Section Manager, Department
Manager, Division Manager and Vice President in the mill site of PICOP in Bislig, Surigao del Sur.

Their services were terminated when the company undertook a retrenchment program. They received
separation pay at the rate of one (1) month basic salary for every year in service.

They lodged a complaint for separation pay differentials believing that the allowances they allegedly regularly
received on a monthly basis during their employment should have been included in the computation of their
separation pay.

The allowances pertained to the following:

1. Staff/Managers Allowance

a. PICOP provides free housing facilities to supervisory and managerial employees


assigned in Bislig. The privilege includes free water and electric consumption.

b. Owing to the shortage of such facilities, PICOP was constrained to grant Staff
allowance instead to those who live in rented houses outside but near the vicinity of the
mill site. The allowance ceases whenever a vacancy occurs in the companys housing
facilities.

2. Transportation Allowance Transportation allowance is granted to key officers and Managers


assigned in the mill site who use their own vehicles in the performance of their duties. It is a
conditional grant such that when the conditions no longer obtain, the privilege is discontinued.

3. Bislig Allowance Given to Division Managers and corporate officers assigned in Bislig on
account of the hostile environment. But once the recipient is transferred elsewhere outside Bislig, the
allowance ceases.

Petitioners maintain that the said allowances are included in the definition of facilities in Art. 97, par. (f), of the
Labor Code, being necessary and indispensable for their existence and subsistence. Furthermore, they claim
that their availment of the monetary equivalent of those facilities on a monthly basis was characterized by
permanency, regularity and customariness.

ISSUES:
1. Whether the receipt of the above-mentioned allowances, on a monthly basis, ipso facto characterize it
as regular and forming part of salary; and
San Beda College of Law 132
4S: 2015 - 2016
LABOR LAW REVIEW Atty. Joyrich Golangco

2. Whether the above-mentioned can be considered as facilitites and therefore included in the
computation of separation pay as wage.

LA RULING: Yes, the allowances are to be characterized as being received regularly and forming part of salary.
It is also to be considered as facilities under Art 97, par. (f) of the Labor Code for purposes of computing
separation pay. The LA cited Santos v NLRC and Soriano v NLRC that in the computation of separation pay
account should be taken not just of the basic salary but also of the regular allowances that the employee had
been receiving.

NLRC RULING: No, the NLRC reversed the Labor Arbiter. The NLRC found that petitioners allowances were
contingency-based and thus not included in their salaries.

SC RULING:
The decision of the NLRC is affirmed. The allowances are not to be included in the computation of wage for
purposes of paying separation pay

1. The receipt of an allowance on a monthly basis does not ipso facto characterize it as regular and
forming part of salary because the nature of the grant is a factor worth considering. The subject allowances
were temporarily, not regularly, received by petitioners. Petitioners continuous enjoyment of the disputed
allowances was based on contingencies the occurrence of which wrote finis to such enjoyment.

For housing allowance, the same is discontinued once a vacancy occurs in the company-provided
housing accommodations.

Transportation allowance is given only to employees who have personal cars in the form of advances
for actual transportation expenses subject to liquidation.
Bislig allowance is- once the officer is transferred outside Bislig, the allowance stops.

2. The Staff/Managers allowance may fall under lodging but the transportation and Bislig allowances
are not embraced in facilitites on the main consideration that they are granted as well as the Staff/Managers
allowance for respondent PICOPs benefit and convenience, i.e. to insure that petitioners render quality
performance. In determining whether a privilege is a facility, the criterion is not so much its kind but its purpose.

San Beda College of Law 133


4S: 2015 - 2016
LABOR LAW REVIEW Atty. Joyrich Golangco

46. SLL INTERNATIONAL CABLES SPECIALIST AND SONNY LAGON V. NLRC
GR No. 172161. March 2, 2011
MENDOZA, J.:

Doctrines:
1. Before the value of facilities can be deducted from the employees wages, 3 requisites must concur: (1)
proof must be shown that such facilities are customarily furnished by the trade; (2) the provision of
deductible facilities must be voluntarily accepted in writing by the employee; and (3) facilities must be
charged at reasonable value.

2. Distinction between Facilities and Supplements. Supplements constitute extra remuneration or


special privileges or benefits given to or received by the laborers over and above their ordinary earnings or
wages. Facilities are items of expense necessary for the laborers and his familys existence and
subsistence so that they form part of the wage and when furnished by the employer are deductible
therefrom, since if they are not so furnished, the laborer would spend and pay for them just the same.

FACTS:
Private respondents Lopez, Canete and Zuniga were hired by petitioner, initially as trainee cable/lineman. They
were paid the full minimum wage. After their training, they were repeatedly hired by petitioner as project
employees. They were not paid the required minimum wage. In their last project with petitioner, respondents
were not allowed to render overtime work. This prompted respondents to return home and leave their work.
th
They filed a complaint for illegal dismissal, non-payment of wages, holiday pay, 13 month pay and damages.

In its answer, petitioner argued that respondents are not regular employees. It also reasoned that the food
allowance, allowance for lodging house, transportation. Electricity, water and snacks should be added to their
basic pay. With these, petitioners claimed that private respondents received higher wage rate than that
prescribed in their areas of work.

ISSUE: Whether the said allowances are to be considered as facilities and are therefore deductible from the
wage of the respondent employees.

LA RULING: No. The free board and lodging, electricity, water and food enjoyed by respondents could not be
included in the computation of their wages because these were given without their written consent.

NLRC RULING: No. decision of the Labor Arbiter affirmed.

CA RULING: No. decision of the Labor Arbiter affirmed.

SC RULING:
No. Before the value of facilities can be deducted from the employees wages, 3 requisites must concur: (1)
proof must be shown that such facilities are customarily furnished by the trade; (2) the provision of deductible
facilities must be voluntarily accepted in writing by the employee; and (3) facilities must be charged at
reasonable value. These requirements have not been met in this case.

San Beda College of Law 134


4S: 2015 - 2016
LABOR LAW REVIEW Atty. Joyrich Golangco

47. OUR HAUS REALTY DEVELOPMENT CORPORATION vs.ALEXANDER PARIAN, JAY C. ERINCO,
ALEXANDER CANLAS, BERNARD TENEDERO and JERRY SABULAO
G.R. No. 204651 August 6, 2014
BRION J.:

Doctrines:
1. To be able to deduct facilities to the wage of an employee, three requisites must concur:
a. proof must be shown that such facilities are customarily furnished by the trade;
b. the provision of deductible facilities must be voluntarily accepted in writing by the employee; and
c. The facilities must be charged at fair and reasonable value.

2. DOLE DO No. 56, series of 2005, which sets out the guidelines for the implementation of DOLE DO No.
13, mandates that the cost of the implementation of the requirements for the construction safety and
health of workers, shall be integrated to the overall project cost. The rationale behind this is to ensure
that the living accommodation of the workers is not substandard and is strictly compliant with the
DOLEs OSH criteria.

As part of the project cost that construction companies already charge to their clients, the value of the
housing of their workers cannot be charged again to their employees salaries. Our Haus cannot pass
the burden of the OSH costs of its construction projects to its employees by deducting it as facilities.
This is Our Haus obligation under the law.

3. Lastly, even if a benefit is customarily provided by the trade, it must still pass the purpose test set by
jurisprudence. Under this test, if a benefit or privilege granted to the employee is clearly for the
employers convenience, it will not be considered as a facility but a supplement. Here, careful
consideration is given to the nature of the employers business in relation to the work performed by the
employee. This test is used to address inequitable situations wherein employers consider a benefit
deductible from the wages even if the factual circumstances show that it clearly redounds to the
employers greater advantage.
FACTS:
Respondents Alexander Parian, Jay Erinco, Alexander Canlas, Jerry Sabulao and Bernardo Tenederowere all
laborers working for petitioner Our Haus Realty Development Corporation (Our Haus), a company engaged in
the construction business.

They claimed that they were not paid the required minimum wage. Petitioner argues that aside from subsidizing
their meals (3 times a day), it also gave them free lodging near the construction project they were assigned to. In
determining the total amount of the respondents daily wages, the value of these benefits should be considered,
in line with Article 97(f) of the Labor Code.

The respondents pointed out that Our Haus never presented any proof that they agreed in writing to the
inclusion of their meals value in their wages. Also, Our Haus failed to prove that the value of the facilities it
furnished was fair and reasonable. Finally, instead of deducting the maximum amount of 70% of the value of the
meals, Our Haus actually withheld its full value (which was Php290.00 per week for each employee).

ISSUE: Whether the amounts for food subsidy and lodging should be considered as part of the daily wages of a
construction worker

LA RULING: Yes. The LA ruled in favor of Our Haus. He held that if the reasonable values of the board and
lodging would be taken into account, the respondents daily wages would meet the minimum wage rate

NLRC RULING: No. The NLRC reversed the LA. Citing the case of Mayon Hotel & Restaurant v. Adana, the
NLRC noted that the respondents did not authorize Our Haus in writing to charge the values of their board and
lodging to their wages. Thus, the samecannot be credited.

CA RULING: No. The CA dismissed Our Haus certiorari petition and affirmed the NLRC rulings in toto.

SC RULING:
No. As the CA correctly ruled, the requirements for deducting the value of facilities to the wages of an
employee, as summarized in Mabeza, are the following:
San Beda College of Law 135
4S: 2015 - 2016
LABOR LAW REVIEW Atty. Joyrich Golangco

a. proof must be shown that such facilities are customarily furnished by the trade;
b. the provision of deductible facilities must be voluntarily accepted in writing by the employee; and
c. The facilities must be charged at fair and reasonable value

None of these are proven to have existed by petitioner.

Our Haus could not really be expected to prove compliance with the first requirement since the living
accommodation of workers in the construction industry is not simply a matter of business practice. Peculiar to
the construction business are the occupational safety and health (OSH) services which the law itself mandates
employers to provide to their workers. This isto ensure the humane working conditions of construction
employees despite their constant exposure to hazardous working environments. Under Section 16 of DOLE
43
Department Order (DO) No. 13, series of 1998, employers engaged in the construction business are required
to providethe following welfare amenities:
16.1 Adequate supply of safe drinking water
16.2 Adequate sanitaryand washing facilities
16.3 Suitable living accommodation for workers, and as may be applicable, for their families
16.4 Separate sanitary, washing and sleeping facilitiesfor men and women workers.

Moreover, DOLE DO No. 56, series of 2005, which sets out the guidelines for the implementation ofDOLE DO
No. 13, mandates that the cost of the implementation of the requirements for the construction safety and health
44
of workers, shall be integrated to the overall project cost. The rationale behind this isto ensure that the living
accommodation of the workers is not substandard and is strictly compliant with the DOLEs OSH criteria.

As part of the project cost that construction companies already charge to their clients, the value of the housing
of their workers cannot be charged again to their employees salaries. Our Haus cannot pass the burden of the
OSH costs of its construction projects to its employees by deducting it as facilities. This is Our Haus obligation
under the law.

As to the second requirement, Our Haus belatedly submitted five kasunduans, supposedly executed by the
respondents, containing their conformity to the inclusion of the values of the meals and housing to their total
wages. Oddly, Our Haus only offered these documents when the NLRC had already ruled that respondents did
not accomplish any written authorization, to allow deduction from their wages. These five kasunduans were also
undated, making us wonder if they had reallybeen executed when respondents first assumed their jobs.

Moreover, in the earlier sinumpaang salaysay by Our Haus four employees, it was not mentioned that they also
executed a kasunduanfor their board and lodging benefits. Because of these surrounding circumstances and the
suspicious timing when the five kasunduanswere submitted as evidence, we agree withthe CA that the NLRC
committed no grave abuse of discretion in disregarding these documents for being self serving.

As to the third requirement, Our Haus never explained how it came up with the values it assigned for the
benefits it provided; it merely listed its supposed expenses without any supporting document. Since Our Haus is
using these additional expenses (cooks salary, water and LPG) to support its claim that it did not withhold the
full amount of the meals value, Our Haus is burdened to present evidence to corroborate its claim. The records
however, are bereft of any evidence to support Our Haus meal expense computation. Even the value it
assigned for the respondents living accommodations was not supported by any documentary evidence. Without
any corroborative evidence, it cannot be said that Our Haus complied with this third requisite.

San Beda College of Law 136


4S: 2015 - 2016
LABOR LAW REVIEW Atty. Joyrich Golangco

ARTICLE 100

48. AMERICAN WIRE AND CABLE DAILY RATED EMPLOYEES UNION V. AMERICAN WIRE AND CABLE
CO., INC.
GR No. 155059 April 29, 2005
CHICO-NAZARIO J.

Doctrines:
1. The granting of a bonus is a management prerogative, something given in addition to what is ordinarily
received by or strictly due the recipient. Thus, a bonus is not a demandable and enforceable obligation,
except when it is made part of the wage, salary or compensation of the employee.

2. For a bonus to be enforceable, it must have been promised by the employer and expressly agreed upon by
the parties, or it must have had a fixed amount and had been a long ad regular practice on the part of the
employer.

3. To be considered a regular practice, the giving of the bonus should have been done over a long period of
time, and must be shown to have been consistent and deliberate.

FACTS:
The American Wire and Cable Co. Inc., has been giving its employees certain benefits and entitlements. These
include the following:

a. Service Award
b. 35% premium pay of an employees basic pay for work rendered during Holy Monday, Holy
Tuesday, Holy Wednesday, Dec. 23, 26, 27, 28 and 29
c. Christmas Party
d. Promotional Increase

All the said benefits are no part of the CBA and the grant thereof was based upon the financial performance of
the company. Moreover, the grant of the 35% premium pay was only made for a period of two years with the
express condition that it is based on the financial situation of the company.

Over the years, there has been a downtrend in the giving of service awards and its amount and holding of
Christmas parties.

When the financial situation of the company worsened, the company unilaterally stopped giving the said
benefits.

The unions (petitioners), filed a complaint alleging that the company violated Article 100 of the Labor Code. It
argues that the benefits and incentives can no longer be withdrawn since it has ripened into a company practice.

The company answered by arguing that the said benefits are in the nature of bonuses which it can withdraw
unilaterally.

ISSUES:
1. Whether the said benefits are in the nature of bonuses which can be withdrawn unilaterally by respondent
company

2. If considered as bonuses, whether it can be considered as part of the wage or salary or compensation
making them enforceable obligations

LA RULING: Yes, therefore the company is not guilty of violating Art 100 of the Labor Code.

CA RULING: Yes. The decision of the VA is affirmed and upheld.

SC RULING:
San Beda College of Law 137
4S: 2015 - 2016
LABOR LAW REVIEW Atty. Joyrich Golangco

1. Yes. A bonus is an amount granted and paid to an employee for his industry and loyalty which contributed to
the success of the employers business and made possible the realization of profits. The granting of a bonus is a
management prerogative, something given in addition to what is ordinarily received by or strictly due the
recipient. Thus, a bonus is not a demandable and enforceable obligation, except when it is made part of the
wage, salary or compensation of the employee.

All the said benefits are in excess of what the law requires each employer to give its employees. Since they are
above what is strictly due to the members of the union, the granting of the same was a management
prerogative, which, whenever management sees necessary, may be withdrawn, unless they have been made a
part of the wage or salary or compensation of the employees.

2. No. For a bonus to be enforceable, it must have been promised by the employer and expressly agreed upon
by the parties, or it must have had a fixed amount and had been a long and regular practice on the part of the
employer.

The benefits in question were never subjects of any express agreement between the parties. They were never
incorporated in the CBA. The Christmas parties and its incidental benefits and the giving of case incentive
together with the service award cannot be said to have fixed amounts, To be considered a regular practice, the
givng of the bonus should have been done over a long period of time, and must be shown to have been
consistent and deliberate. The downtrend in the grant of these two bonuses over the years demonstrate that
there is nothing consistent about it.

San Beda College of Law 138


4S: 2015 - 2016
LABOR LAW REVIEW Atty. Joyrich Golangco

49. TSPIC CORPORATION V. TSPIC EMPLOYEES UNION
GR No. 163419 February 13, 2008
VELASCO, JR., J.

Doctrines:
1. Diminution of benefits is the unilateral withdrawal by the employer of benefits already enjoyed by the
employees. There is diminution of benefits when it is shown that: (1) the grant or benefit is founded on a
policy or has ripened into a practice over a long period; (2) the practice is consistent and deliberate; (3) the
practice is not due to error in the construction or application of a doubtful or difficult question of law; and (4)
the diminution or discontinuance is done unilaterally by the employer

2. An erroneously granted benefit may be withdrawn without violating the prohibition against non-diminution of
benefits

FACTS:
Respondent Union is the registered bargaining agent of petitioner TSPIC. The two entered into a CBA for the
years 2000-2004. The CBA included a provision on yearly salary increases starting January 2000 until January
2002. Under the CBA, different rates of wage increases in the duration of the CBA, are given to different sets of
employees. The increases for the second year and third year of implementation of the CBA are deemed to be
inclusive of any Wage Increase ordered by the Wage Boards and as correction of any wage distortion that may
have been brought about by future Wage Orders (crediting provision.)

When Wage Order No. 8 was implemented increasing the minimum wage of regular employees, an error in the
automated payroll system occurred and TSPIC claims that 24 employees were overpaid. They were notified that
the overpayment would be deducted from their salaries in a staggered basis. TSPIC explained that the
correction of the erroneous computation was based on the crediting provision of the CBA.

The Union asserted that there was no error and that the deduction constituted diminution of pay. The Union
insists that the crediting provision of the CBA finds no application in the present case, since at the time the
Wage Order was issued, the probationary employees were not yet covered by the CBA, particularly by the
crediting provision.

ISSUE: Whether charging the overpayments made to the respondents through staggered deductions constitute
diminution of benefits.

LA RULING: Yes. The unilateral deduction made by TSPIC violated Art. 100 of the Labor Code

CA RULING: Yes, the decision of the VA is affirmed in toto

SC RULING:
No. Diminution of benefits is the unilateral withdrawal by the employer of benefits already enjoyed by the
employees. There is diminution of benefits when it is shown that: (1) the grant or benefit is founded on a policy
or has ripened into a practice over a long period; (2) the practice is consistent and deliberate; (3) the practice is
not due to error in the construction or application of a doubtful or difficult question of law; and (4) the diminution
or discontinuance is done unilaterally by the employer

As correctly pointed out by TSPIC, the overpayment of its employees was a result of an error. This error
was immediately rectified by TSPIC upon its discovery. We have ruled before that an erroneously granted
benefit may be withdrawn without violating the prohibition against non-diminution of benefits.

San Beda College of Law 139


4S: 2015 - 2016
LABOR LAW REVIEW Atty. Joyrich Golangco

50. LEPANTO CERAMICS, INC., V. LEPANTO CERAMICS EMPLOYEES ASSOCIATION
GR No. 180866 March 2, 2010
PEREZ, J.:

Doctrine:
A bonus that has been incorporated in the CBA becomes more than just an act of generosity on the part of the
employer but a contractual obligation it has undertaken.

FACTS:
In December 1998, petitioner gave a P3,000.00 bonus to its employees, members of the respondent
Association. In September 1999, the two entered into a CBA which provides for the grant of a Christmas gift
package/bonus to the members of the respondent Association. The Christmas bonus was one of the
enumerated existing benefit, practice of traditional rights which shall remain in full force and effect.

In 2002, the year-end cash benefit was only P600.00. The Association objected arguing that such act was a
violation of the CBA. After failure to settle, the Association filed a Notice of Strike. The case was referred to the
Voluntary Arbitrator.

The Association insisted that it has been the company practice grant members Christmas bonuses in the
amount of P3,000.00. Thus it argues that failure on the part of the company to give said amount was in violation
of the CBA.

Petitioner argues that the said amount is in the form of a bonus and is thus not demandable. It argued that the
giving of extra compensation was based on the companys available resources for a given year and the workers
are not entitled to a bonus if the company does not make profits. Petitioner avers that it is debt ridden and could
not give out the bonus.

ISSUE: Whether the amount of P3,000 Christmas gift/bonus is demandable for being included in the CBA

LA RULING: Yes. The CBA is a binding contract and constitutes the law between the parties.

CA RULING: The decision of the VA is affirmed in toto

SC RULING:
Yes. Generally, a bonus is not a demandable and enforceable obligation. For a bonus to be enforceable, it must
have been promised by the employer and expressly agreed upon by the parties. Given that the bonus in this
case is integrated in the CBA, the same partakes the nature of a demandable obligation. Verily, by virtue of its
incorporation in the CBA, the Christmas bonus due to respondent Association has become more than just an act
of generosity on the part of the petitioner but a contractual obligation it has undertaken.

San Beda College of Law 140


4S: 2015 - 2016
LABOR LAW REVIEW Atty. Joyrich Golangco

51. EASTERN TELECOMMUNICATIONS PHIL. INC. V. EASTERN TELECOMS EMPLOYEES UNION
GR No. 185665. February 8, 2012
MENDOZA, J.:

Doctrines:
1. Whether or not a bonus forms part of wages depends upon the circumstances and conditions for its
payment. If it is additional compensation which the employer promised and agreed to give without any
conditions imposed for its payment, such as success of business or greater production or output, then it is
part of the wage, But if its paid only if profits are realized or if a certain level of productivity is achieved, it
cannot be considered part of wage. Where it is not payable to all but only to some employees and only
when their labor becomes more efficient or more productive, it is only an inducement for efficiency, a prize
therefore, not a part of wage.

2. A bonus may be granted on equitable consideration when the giving of such bonus has been the
companys long and regular practice.

3. The principle of non-diminution of benefits is founded on the constitutional mandate to protect the rights of
workers and to promote their welfare and to afford labor full protection.

FACTS:
Respondent Union is the exclusive bargaining agent in the establishment of Petitioner Company. Since 1975,
th th th
the company has been giving its employees 14 , 15 and 16 month bonuses. In 2001, the two signed a side
th th th
agreement which provides that the 14 , 15 and 16 month bonuses are granted.

Due to continuing financial losses which started in 2000, the company, decided in 2003, to defer the payment of
the said bonuses. The Union opposed the said plan. The Union argues that the bonuses are now legally
demandable for being included in the CBA. Furthermore, the giving of the said bonuses has now ripened into
company practice and can no longer be withdrawn without violating Article 100 of the Labor Code.

The company argues that the said bonuses are not legally demandable. It argues that the giving of said
bonuses are dependent on the financial capability of the company. Since it has been sustaining losses since
2000, it no longer has the capacity to give such bonuses.

ISSUE: Whether the said bonuses are legally demandable

NLRC RULING: No. the payment of these bonuses are management prerogative, being an act of generosity
and munificence on the part of the company and contingent upon the realization of profits. The company may
not be obliged to pay extra compensation in view of the substantial decline in its financial condition.

CA RULING: Yes, the Side Agreement in the CBA granting the bonuses are contractual obligations on the
company without qualification or condition. Also, the grant of the said bonuses has already ripened into a
company practice and their denial would amount to diminution of the employees benefits.

SC RULING:
Yes. A bonus becomes a demandable or enforceable obligation when it is made part of the wage or salary or
compensation of the employee. It is indubitable that the company and the union agreed on the inclusion of a
provision for the grant of the bonuses in the Side Agreement. There were no conditions specified in the CBA
Side Agreement for the grant of the benefits. By its inclusion in the CBA Side Agreements, the bonuses has
become more than just an act of generosity on the part of the company but a contractual obligation it has
undertaken.

Granting arguendo that the CBA Side Agreement does not contractually bid the company, its act of granting the
same has become an established company practice such that it has virtually become part of the employees
salary or wage. A bonus may be granted on equitable consideration when the giving of such bonus has been
the companys long and regular practice.
In this case, the company has been giving the said bonuses since 1975 whether it earned profits or not.

San Beda College of Law 141


4S: 2015 - 2016
LABOR LAW REVIEW Atty. Joyrich Golangco

ARTICLE 106

52. GSIS vs. NLRC


G.R. No. 180045 November 17, 2010
NACHURA, J.:

JOINT AND SOLIDARY LIABILITY OF THE PRINCIPAL

FACTS:
Respondents were employed as security guards by DNL Security Agency. By virtue of the service contract
entered into by DNL Security and GSIS, respondents were assigned to GSIS Tacloban City office.

However, DNL Security informed respondents that its service contract with GSIS was terminated.
Notwithstanding, DNL Security instructed respondents to continue reporting for work to GSIS. Respondents
worked as instructed but without receiving their wages; after which, they were terminated from employment.
Hence, respondents filed with the LA a complaint against DNL Security and GSIS.

ISSUE: Is GSIS liable for payment of the respondents unpaid salary and other monetary benefits?

LA RULING: LA rendered a decision against DNL Security and GSIS ordering both as jointly and solidarily
liable to respondent for the unpaid salary.

NLRC RULING: NLRC treated DNL Securitys motion for reconsideration as an appeal, but dismissed the
same, as it was not legally perfected. GSIS filed a petition for certiorari before the CA.

CA RULING: CA affirmed the NLRC ruling. GSIS averred that it has no actual and direct employer-employee
relationship between it and the respondents.

SC RULING:
GSIS is jointly and severally liable with DNS Security with respect to respondents claims. When GSIS
contracted DNL Securitys services, it became an indirect employer of respondents, pursuant to Article 107 of
LC. After DNL Security failed to pay respondents the correct wages and other monetary benefits, GSIS, as
principal, became jointly and severally liable, as provided in Articles 106 and 109 of LC.

While it is true that respondents continued working for GSIS after the expiration of their contract, based on the
instruction of DNL Security, GSIS did not object to such assignment and allowed respondents to render service.
Thus, GSIS impliedly approved the extension of respondents services. Accordingly, GSIS is bound by the
provisions of the LC on indirect employment. So long as the work, task, job, or project has been performed for
its benefit or on its behalf, the liability accrues for such services. However, the solidary liability of GSIS does not
preclude the application of Article 1217 of the Civil Code on the right of reimbursement from its co-debtor, DNS
Security.

GSIS liability, however, cannot extend to the payment of separation pay. An order to pay separation pay is
invested with a punitive character, such that an indirect employer should not be made liable without a finding
that it had conspired in the illegal dismissal of the employees.

San Beda College of Law 142


4S: 2015 - 2016
LABOR LAW REVIEW Atty. Joyrich Golangco

53. ALIVIADO, et.al. vs. PROCTOR AND GAMBLE
G.R. No. 160506 June 6, 2011
DEL CASTILLO, J.:

CONTROL TEST IS MERELY ONE OF THE ELEMENTS TO DETERMINE EXISTENCE OF LABOR-ONLY


CONTRACTING

FACTS:
(The full text of the case does not include the facts since it only resolved the 2nd MR filed by P&G to SC.) On
March 9, 2010, the SC rendered a Decision holding that Promm-Gem is a legitimate independent contractor;
that Sales and Promotions Services (SAPS) is a labor-only contractor consequently its employees are
considered employees of Procter & Gamble Phils., Inc. (P&G); that Promm-Gem is guilty of illegal dismissal;
that SAPS/P&G is likewise guilty of illegal dismissal; that petitioners are entitled to reinstatement; and that the
dismissed employees of SAPS/P&G are entitled to moral damages and attorneys fees there being bad faith in
their dismissal.

P&G filed a Motion for Reconsideration but was denied by the SC. P&G filed a second MR. P&G claimed that
the SC erred in not applying the four-fold test, particularly the control test in determining whether SAPS is a
legitimate independent contractor or a labor-only contractor.

ISSUE: Whether SAPS is a labor-only contractor?

SC RULING:
The SC correctly determined SAPS as a labor-only contractor. As discussed in the March 9, 2010 SC Decision,
the applicable rules are Article 106 of the LC and Rule VIII-A, Book III of the Omnibus Rules Implementing the
LC, as amended by D.O. No. 18-06. The said DO provides that labor-only contracting exists when any of the
two elements is present: (1) the contractor or subcontractor does not have substantial capital or investment
which relates to the job, work or service to be performed and the employees recruited, supplied or placed by
such contractor or subcontractor are performing activities which are directly related to the main business of the
principal; OR (2) the contractor does not exercise the right to control over the performance of the work of the
contractual employee.

Therefore, the control test is merely one of the factors to consider. It was already established that SAPS has no
substantial capitalization and it was performing merchandising and promotional activities which are directly
related to P&G's business. Since SAPS met one of the requirements, it was enough basis for SC to hold that it
is a labor-only contractor. Consequently, its principal, P&G, is considered the employer of its employees. This is
pursuant to the ruling in Aklan v. San Miguel Corporation[27] where it was held that [a] finding that a contractor
is a labor-only contractor, as opposed to permissible job contracting, is equivalent to declaring that there is an
employer-employee relationship between the principal and the employees of the supposed contractor, and
the labor-only contractor is considered as a mere agent of the principal, the real employer.

San Beda College of Law 143


4S: 2015 - 2016
LABOR LAW REVIEW Atty. Joyrich Golangco

54. MANDAUE GALLEON TRADE INC. vs. ANDALES et.al.
G.R. No. 159668 March 7, 2008
AUSTRIA-MARTINEZ, J.:

FACTS:
Respondent Vicente Andales filed a complaint with the Labor Arbiter (LA) against petitioners Mandaue Galleon
Trade, Inc. (MGTI) and Gamallosons Traders, Inc. (GTI) for illegal dismissal and non-payment of 13th month
pay and service incentive leave pay. Respondents alleged that MGTI hired them on various dates as weavers,
grinders, sanders and finishers but they were dismissed without notice and just cause.

Respondents further alleged that they are regular employees of MGTI because: (a) they performed their work
inside the company premises; (b) they were issued uniforms by MGTI and were told to strictly follow company
rules and regulations; (c) they were under the supervision of MGTI's foremen, quality control personnel and
checkers; (d) MGTI supplied the materials, designs, tools and equipment in the production of furniture; (e) MGTI
conducts orientations on how the work was to be done and the safe and efficient use of tools and equipment; (f)
MGTI issues memoranda regarding absences and waste of materials; and (g) MGTI exercises the power to
discipline them.

On the other hand, MGTI denied the existence of employer-employee relationship with complainants, claiming
that they are workers of independent contractors whose services were engaged temporarily and seasonally
when the demands for its products are high and could not be met by its regular workforce; the independent
contractors recruited and hired the complainants, prepared the payroll and paid their wages, supervised and
directed their work, and had authority to dismiss them.

LA RULING: LA held that the respondents are regular piece-rate employees of MGTI since they were made to
perform functions which are necessary to MGTI's rattan furniture manufacturing business. The independent
contractors were not properly identified. The absence of proof that the independent contractors have work
premises of their own, substantial capital or investment in the form of tools, equipment and machineries make
them only labor contractors.

NLRC RULING: It affirmed the decision of LA. It held that labor-only contracting and not job-contracting was
present since the alleged contractors did not have substantial capital in the form of equipment, machineries and
work premises.

CA RULING: MGTI is liable to the respondents because the alleged contractors are not independent contractors
but labor-only contractors.

ISSUE: Whether or not MGIT is a labor-only contractor?

SC RULING:
MGIT is a labor-only contractor. Based on Article 106 of the Labor Code and Sections 5 and 7 of the
Implementing Rules, labor-only contracting exists when the following criteria are present: (1) where the
contractor or subcontractor supplying workers to an employer does not have substantial capital or investment in
the form of tools, equipment, machineries, work premises, among other things; and the workers recruited and
placed by the contractor or subcontractor are performing activities which are directly related to the principal
business of such employer; or (2) where the contractor does not exercise the right to control the performance of
the work of the contractual employee.

First, respondents work as weavers, grinders, sanders and finishers is directly related to MGTI's principal
business of rattan furniture manufacturing. Where the employees are tasked to undertake activities usually
desirable or necessary in the usual business of the employer, the contractor is considered as a labor-only
contractor and such employees are considered as regular employees of the employer.

Second, MGTI was unable to present any proof that its contractors had substantial capital. There was no
evidence pertaining to the contractors' capitalization; nor to their investment in tools, equipment or implements
actually used in the performance or completion of the job, work, or service that they were contracted to render.

Thus, the contractors are labor-only contractors since they do not have substantial capital or investment which
relates to the service performed and respondents performed activities which were directly related to MGTI's
San Beda College of Law 144
4S: 2015 - 2016
LABOR LAW REVIEW Atty. Joyrich Golangco

main business. MGTI, the principal employer, is solidarily liable with the labor-only contractors, for the rightful
claims of the employees. Under this set-up, labor-only contractors are deemed agents of the principal, MGTI,
and the law makes the principal responsible to the employees of the labor-only contractor as if the principal itself
directly hired or employed the employees.

San Beda College of Law 145


4S: 2015 - 2016
LABOR LAW REVIEW Atty. Joyrich Golangco

55. SPIC N SPAN SERVICES CORPORATION vs. PAJE et.al.
G.R. No. 174084 August 25, 2010
BRION, J.:

REQUIREMENTS OF LEGITIMATE CONTRACTING/SUBCONTRACTING

FACTS:
Swift Foods, Inc. (Swift) manufactures and processes meat products and other food products. Petitioner SNSs
business is to supply manpower services to its clients for a fee. Swift and SNS have a contract to promote Swift
products. Respondents worked as Deli/Promo Girls of Swift products in supermarkets. They were all dismissed
from their employment on February 28, 1998. They filed two complaints for illegal dismissal against SNS and
Swift before NLRC. Swift moved to dismiss the complaints on the ground that it entered into an independent
labor contract with SNS for the promotion of its products. It alleged that the respondents were the employees of
SNS, not of Swift.

RULING OF LA: LA found SNS to be the agent of Swift. First, the agreement between SNS and Swift shows
that the latter exercised control over the promo girls and/or merchandisers through the services of coordinators.
Second, it cannot be said that SNS has substantial capital. Third, the duties of the petitioners were directly
related, necessary and vital to the day-to-day operations of Swift. Lastly, the uniform and identification cards
used by the petitioners were subject to the approval of Swift.

RULING OF NLRC: NLRC ruled that SNS is an independent contractor. First, there is no evidence that Swift
exercised the power of control over the petitioners. Rather, it is SNS who exercised direct control and
supervision over the nature and performance of the works of herein petitioners. Second, by law, Swift and SNS
have distinct and separate juridical personality from each other.

RULING OF CA: CA dismissed the appeal. It concluded that SNS was merely an agent of Swift; thus, the latter
should not be exempt from liability.

ISSUE: Whether SNS is merely an agent of Swift?

SC RULING:
SNS is considered merely an agent of Swift which does not exempt the latter from liability. To be legitimate,
contracting or subcontracting must satisfy the following requirements: 1) The contractor or subcontractor carries
on a distinct and independent business and undertakes to perform the job, work or service on its own account
and under its own responsibility; 2) the contractor or subcontractor has substantial capital or investment; and 3)
the agreement between the principal and contractor or subcontractor assures the contractual employees
entitlement to all labor and occupational safety and health standards, free exercise of right to self-organization,
security of tenure, and social and welfare benefit.

Nowhere in the decisions of both the LA and the NLRC show that SNS had full control of the means and
methods of the performance of their work. Moreover, as found by the LA, there was no evidence that SNS has
substantial capital or investment. Lastly, there was no finding by the LA nor the NLRC that the agreement
between the principal (Swift) and contractor (SNS) assures the contractual employees entitlement to all labor
and occupational safety and health standards, free exercise of right to self-organization, security of tenure, and
social and welfare benefit.

San Beda College of Law 146


4S: 2015 - 2016
LABOR LAW REVIEW Atty. Joyrich Golangco

56. VIGILLA et.al. vs. PCCI
G.R. No. 200094 June 10, 2013
MENDOZA, JR:

A QUITCLAIM EXECUTED IN FAVOR OF THE LABOR-ONLY CONTRACTOR WILL REDOUND TO THE


BENEFIT OF THE PRINCIPAL EMPLOYER; A LABOR-ONLY CONTRACTOR IS SOLIDARILY LIABLE
WITH THE EMPLOYER

FACTS:
PCCI is a non-stock educational institution, while the petitioners were janitors, janitresses and supervisor in the
Maintenance Department of PCCI under the supervision and control of Atty. Seril, PCCIs Senior Vice President
for Administration. The petitioners, however, were made to understand, upon application with PCCI, that they
were under Metropolitan Building Services, Inc. (MBMSI), a corporation engaged in providing janitorial services
to clients. Atty. Seril is also the President and General Manager of MBMSI.

PCCI, citing the revocation of MBMSI Articles of Incorporation, terminated its relationship with MBMSI, resulting
in the dismissal of the petitioners.

In their complainants before LA, petitioners alleged that it was the school, not MBMSI, which was their real
employer because PCCI had direct control over MBMSIs operations and the selection and hiring of employees
were undertaken by PCCI.

On the other hand, PCCI contended that it could not have illegally dismissed the complainants because it was
not their direct employer; (b) MBMSI was the one who had complete and direct control over the complainants;
and (c) PCCI had a contractual agreement with MBMSI, thus, making the latter their direct employer. Also, PCCI
submitted before LA the releases, waivers and quitclaims in favor of MBMSI executed by the respondents to
prove that they were employees of MBMSI and not PCCI.

LA RULING: LA found that PCCI was the real principal employer of the complainants and that MBMSI was a
mere adjunct or alter ego/labor-only contractor. LA explained that PCCI was actually the one which exercised
control over the means and methods of the work of the petitioners, thru Atty. Seril, who was acting, throughout
the time in his capacity as Senior Vice President for Administration of PCCI, not in any way or time as the
supposed employer/general manager or president of MBMSI. However, LA did not touch on the validity and
effect of the quitclaims.

NLRC RULING: NLRC affirmed the LAs findings. Nevertheless, the respondents were excused from their
liability by virtue of the releases, waivers and quitclaims executed by the petitioners. Hence, petitioners filed an
appeal before CA.

CA RULING: CA affirmed the NLRC decision. Petitioners argue that there is no solidary liability to speak of in
case of an existence of a labor-only contractor. Petitioners contend that under Article 106 of the LC, a labor-only
contractors liability is not solidary as it is the employer who should be directly responsible to the supplied
worker. Hence, the said releases, waivers and quitclaims which they purportedly issued in favor of MBMSI and
Atty. Seril do not automatically release respondents from their liability.

ISSUES: Whether the quitclaims executed in favour of MBMSI redounded to the benefit of PCCI?

SC RULING:
The NLRC and the CA correctly ruled that the releases, waivers and quitclaims executed by petitioners in favor
of MBMSI redounded to the benefit of PCCI pursuant to Article 1217 of the New Civil Code. The reason is that
MBMSI is solidarily liable with the respondents for the valid claims of petitioners pursuant to Article 109 of the
Labor Code.

The issue of whether there is solidary liability between the labor-only contractor and the employer is crucial in
this case. If a labor-only contractor is solidarily liable with the employer, then the releases, waivers and
quitclaims in favor of MBMSI will redound to the benefit of PCCI. On the other hand, if a labor-only contractor is
not solidarily liable with the employer, the latter being directly liable, then the releases, waivers and quitclaims in
favor of MBMSI will not extinguish the liability of PCCI.

San Beda College of Law 147


4S: 2015 - 2016
LABOR LAW REVIEW Atty. Joyrich Golangco

There is solidary liability between the principal and labor-only contractor. In labor-only contracting, the statute
creates an employer-employee relationship for a comprehensive purpose: to prevent a circumvention of labor
laws. The contractor is considered merely an agent of the principal employer and the latter is responsible to the
employees of the labor-only contractor as if such employees had been directly employed by the principal
employer. The principal employer therefore becomes solidarily liable with the labor-only contractor for all the
rightful claims of the employees.

San Beda College of Law 148


4S: 2015 - 2016
LABOR LAW REVIEW Atty. Joyrich Golangco

57. BABAS, ET AL. V. LORENZO SHIPPING CORP.
GR. No. 186091 December 15, 2010
NACHURA, J.:

Doctrine:
In labor-only contracting, a prohibited act, the following elements are present: (a) the contractor or subcontractor
does not have substantial capital or investment to actually perform the job, work, or service under its own
account and responsibility; and (b) the employees recruited, supplied, or placed by such contractor or
subcontractor perform activities which are directly related to the main business of the principal.

On the other hand, permissible job contracting or subcontracting refers to an arrangement whereby a principal
agrees to put out or farm out with the contractor or subcontractor the performance or completion of a specific
job, work, or service within a definite or predetermined period, regardless of whether such job, work, or service
is to be performed or completed within or outside the premises of the principal.

FACTS:
Respondent LSC entered into a General Equipment Maintenance Repair and Management Services Agreement
(Agreement) with Best Manpower Services, Inc. (BMSI). Under the Agreement, BMSI undertook to provide
maintenance and repair services to LSCs container vans, heavy equipment, trailer chassis, and generator
sets. BMSI further undertook to provide checkers to inspect all containers received for loading to and/or
unloading from its vessels.

Simultaneous with the execution of the Agreement, LSC leased its equipment, tools, and tractors to BMSI. The
period of lease was coterminous with the Agreement.

BMSI then hired petitioners on various dates to work at LSC as checkers, welders, utility men, clerks, forklift
operators, motor pool and machine shop workers, technicians, trailer drivers, and mechanics. Six years later,
LSC entered into another contract with BMSI, this time, a service contract.

In September 2003, petitioners filed with the Labor Arbiter (LA) a complaint for regularization against LSC and
BMSI. On October 1, 2003, LSC terminated the Agreement. Consequently, petitioners lost their employment.

BMSI asserted that it is an independent contractor. It averred that it was willing to regularize petitioners;
however, some of them lacked the requisite qualifications for the job. BMSI was willing to reassign petitioners
who were willing to accept reassignment.

LSC, on the other hand, averred that petitioners were employees of BMSI and were assigned to LSC by virtue
of the Agreement. The Agreement between LSC and BMSI constituted legitimate job contracting. Thus,
petitioners were employees of BMSI and not of LSC.

LA RULING: LA found that petitioners were employees of BMSI.

NLRC RULING: Reversing the LA, the NLRC held: BMSI is not engaged in legitimate job contracting. BMSI has
no equipment, no office premises, no capital and no investments as shown in the Agreement itself. BMSI has no
independent business or activity or job to perform in respondent LSC free from the control of respondent LSC
except as to the results thereof. LSC [petitioners] performed work that was necessary and desirable to the main
business of respondent LSC. BMSI has no other client but respondent LSC.

Consequently, respondent Lorenzo Shipping Corp. is ordered to reinstate [petitioners] to their former
positions as regular employees and pay their wage differentials and benefits. If reinstatement is not
feasible, both respondents Lorenzo Shipping Corp. and Best Manpower Services are adjudged jointly
and solidarily to pay [petitioners] separation pay.

CA RULING: CA rendered the now challenged Decision, reversing the NLRC. According to the CA, the
fact that BMSI entered into a contract of lease with LSC did not ipso facto make BMSI a labor-only
contractor; on the contrary, it proved that BMSI had substantial capital. The CA was of the view that the
law only required substantial capital or investment.

ISSUE: Whether or not LSC the employer of the petitioners as BSMI is not an independent contractor.
San Beda College of Law 149
4S: 2015 - 2016
LABOR LAW REVIEW Atty. Joyrich Golangco

SC RULING:
YES. In distinguishing between prohibited labor-only contracting and permissible job contracting, the totality of
the facts and the surrounding circumstances of the case are to be considered.

In labor-only contracting, a prohibited act, the following elements are present: (a) the contractor or subcontractor
does not have substantial capital or investment to actually perform the job, work, or service under its own
account and responsibility; and (b) the employees recruited, supplied, or placed by such contractor or
subcontractor perform activities which are directly related to the main business of the principal.

A person is considered engaged in legitimate job contracting or subcontracting if the following conditions concur:

(a) The contractor carries on a distinct and independent business and undertakes the contract work on his
account under his own responsibility according to his own manner and method, free from the control
and direction of his employer or principal in all matters connected with the performance of his work
except as to the results thereof;

(b) The contractor has substantial capital or investment; and

(c) The agreement between the principal and the contractor or subcontractor assures the contractual
employees' entitlement to all labor and occupational safety and health standards, free exercise of the
right to self-organization, security of tenure, and social welfare benefits.

The Court sustains the petitioners contention that BMSI is engaged in labor-only contracting.

First, petitioners worked at LSCs premises, and nowhere else. There was no showing that it was BMSI which
established petitioners working procedure and methods, which supervised petitioners in their work, or which
evaluated the same. There was absolute lack of evidence that BMSI exercised control over them or their work.

Second, LSC was unable to present proof that BMSI had substantial capital. What is clear was that the
equipment used by BMSI were owned by, and merely rented from, LSC.

Third, petitioners performed activities which were directly related to the main business of LSC.
Lastly, BMSI had no other client except for LSC. A Certificate of Registration issued by the Department of Labor
and Employment is not conclusive evidence of status of independent contractor. The fact of registration simply
prevents the legal presumption of being a mere labor-only contractor from arising.

Consequently, the workers that BMSI supplied to LSC became regular employees of the latter. Having gained
regular status, petitioners were entitled to security of tenure and could only be dismissed for just or authorized
causes and after they had been accorded due process.

San Beda College of Law 150


4S: 2015 - 2016
LABOR LAW REVIEW Atty. Joyrich Golangco

58. FIRST PHILIPPINE INDUSTRIAL CORPORATION v. RAQUEL M. CALIMBAS AND LUISA P. MAHILOM
G.R. No. 179256 July 10, 2013
PERALTA, J.:

Doctrine:
There is labor-only contracting where the person supplying workers to an employer does not have substantial
capital or investment in the form of tools, equipment, machineries, work premises, among others, and the
workers recruited and placed by such person are performing activities which are directly related to the principal
business of such employer. In such cases, the person or intermediary shall be considered merely as an agent of
the employer who shall be responsible to the workers in the same manner and extent as if the latter were
directly employed by him.

FACTS:
(FPIC) is a domestic corporation primarily engaged in the transportation of petroleum products by pipeline.
[DGMS] is engaged in the business of supplying manpower to render general clerical, building and grounds
maintenance, and janitorial and utility services.

FPIC, entered into a Contract of Special Services with DGMS, wherein the latter agreed to undertake some
aspects of building and grounds maintenance at FPICs premises, offices and facilities, as well as to provide
clerical and other utility services as may be required from time to time by FPIC.

Pursuant to the said Contract, petitioner Raquel Calimbas and Luisa Mahilom were engaged by the DGMS to
render services to FPIC. Thereat, petitioner Calimbas was assigned as a department secretary at the Technical
Services Department while petitioner Mahilom served as a clerk at the Money Movement Section of the Finance
Division.

FPIC, through its Human Resources Manager, Lorna Young, informed the petitioners that their services to the
company would no longer be needed as a result of the Pace-Setting Study conducted by an outside
consultant. Accordingly, Treasurer of DGMS, formally notified both the petitioners that their respective work
assignments in FPIC were no longer available to them citing the termination of the Project Contract with FPIC as
the main reason thereof. Calimbas and Mahilom signed quitclaims, releasing and discharging DGMS from
whatever claims that they might have against it by virtue of their past employment.

Petitioners still filed a Complaint against FPIC for illegal dismissal and for the collection of monetary benefits,
alleging that they were regular employees of FPIC after serving almost five (5) years, rendering services which
were usually necessary or desirable in the usual business or trade of FPIC and that they were dismissed without
cause.

In their Position Paper, petitioners maintained that their real employer was FPIC, and that DGMS was merely its
agent for having been engaged in prohibited labor-only contracting. The petitioners averred that DGMS did not
have substantial capital.

FPIC insisted that the Labor Arbiter had no jurisdiction over the case because there was absolutely no
employer-employee relationship between it and the petitioners; and that they executed quitclaims in favor of
DGMS
4
LA RULING: Labor Arbiter rendered a Decision holding that respondents were regular employees of FPIC, and
that they were illegally dismissed.

NLRC RULING: NLRC the Labor Arbiters decision. However, in a Resolution after MR by FPIC, the NLRC
reversed its decision. The CA finds no legal basis to deem DGMS a labor-only contracting entity as maintained
by complainants. The fact that DGMS had only a capitalization of P75,000.00, without an investment in tools,
equipment, etc., does not necessarily constitute the latter as labor-only contractor. Labor Arbiter is
hereby REINSTATED.

ISSUE: Whether or not respondents are employees of FPIC.

San Beda College of Law 151


4S: 2015 - 2016
LABOR LAW REVIEW Atty. Joyrich Golangco

SC RULING:
YES. Article 106 of the Labor Code and Sections 8 and 9 of DOLE Department Order No. 10, Series of 1997 are
the standards to apply.

There is labor-only contracting where the person supplying workers to an employer does not have substantial
capital or investment in the form of tools, equipment, machineries, work premises, among others, and the
workers recruited and placed by such person are performing activities which are directly related to the principal
business of such employer. In such cases, the person or intermediary shall be considered merely as an agent of
the employer who shall be responsible to the workers in the same manner and extent as if the latter were
directly employed by him.

Respondents are petitioners employees and that DGMS is engaged in labor-only contracting.
12
First, in Vinoya v. National Labor Relations Commission, this Court categorically stated that the actual paid-in
capital of P75,000.00 could not be considered as substantial capital. Thus, DGMSs actual paid-in capital in the
amount of P75,000.00 does not constitute substantial capital essential to carry out its business as an
independent job contractor. DGMS has no substantial equipment in the form of tools, equipment and machinery.
As a matter of fact, respondents were using office equipment and materials owned by petitioner while they were
rendering their services at its offices.

Second, FPIC exercised the power of control and supervision over the respondents. The fact that DGMS did
not assign representatives to supervise over respondents work in petitioners company tends to disprove the
independence of DGMS. Respondents were subjected to the control and supervision of petitioner while they
were performing their jobs.

Third, also worth stressing are the points highlighted by respondents: Respondents worked only at petitioners
offices for an uninterrupted period of five years, occupying the same position at the same department under the
supervision of company officials; FPICs HR Manager Lorna Young notified respondents, in a closed-door
meeting, that their services to the company would be terminated; The direct superiors of respondents were
managerial employees of petitioner, and had direct control over all the work-related activities of the latter.

All told, an employer-employee relationship exists between petitioner and respondents. And having served for
almost five years at petitioners company, respondents had already attained the status of regular employees.

In the present case, petitioners failed to show any valid or just cause under the Labor Code on which it may
justify the termination of services of respondents. Also, apart from notifying that their services had already been
terminated, petitioner failed to comply with the rudimentary requirement of notifying respondents regarding the
acts or omissions which led to the termination of their services as well as giving them an ample opportunity to
contest the legality of their dismissal. Having failed to establish compliance with the requirements of termination
of employment under the Labor Code, respondents dismissal is tainted with illegality.

San Beda College of Law 152


4S: 2015 - 2016
LABOR LAW REVIEW Atty. Joyrich Golangco

59. AVELINO S. ALILIN, ET. AL. vs. PETRON CORPORATION
G.R. No. 177592 June 9, 2014
DEL CASTILLO, J.

Doctrine:
A contractor is presumed to be a labor-only contractor, unless it proves that it has the substantial capital,
investment, tools and the like. However, where the principal is the one claiming that the contractor is a legitimate
contractor, the burden of proving the supposed status of the contractor rests on the principal.

FACTS:
Romualdo D. Gindang Contractor, started recruiting laborers for fielding to Petrons Mandaue Bulk Plant. When
Romualdo died, his son Romeo D. Gindang through Romeo D. Gindang Services(RDG), took over the business
and continued to provide manpower services to Petron. Petitioners were among those recruited by Romualdo D.
Gindang Contractor and RDG to work in the premises of the said bulk plant.
9
Petron and RDG entered into a Contract for Services for the period from June 1, 2000 to May 31, 2002,
whereby RDG undertook to provide Petron with janitorial, maintenance, tanker receiving, packaging and other
utility services in its Mandaue Bulk Plant. This contract was extended on July 31, 2002 and further extended
until September 30, 2002. Upon expiration thereof, no further renewal of the service contract was done.

Alleging that they were barred from continuing their services on October 16, 2002, petitioners filed a
Complaint for illegal dismissal, underpayment of wages, damages and attorneys fees against Petron and RDG.

Petitioners did not deny that RDG hired them and paid their salaries. They, however, claimed that the latter is a
labor-only contractor, which merely acted as an agent of Petron, their true employer. They asseverated that their
jobs, which are directly related to Petrons business, entailed them to work inside the premises of Petron using
the required equipment and tools furnished by it and that they were subject to Petrons supervision.

RDG corroborated petitioners claim that they are regular employees of Petron.

Petron, on the other hand, maintained that RDG is an independent contractor and the real employer of the
petitioners. It was RDG which hired and selected petitioners, paid their salaries and wages, and directly
supervised their work. And not being the employer, Petron cannot be held liable for petitioners claim of illegal
dismissal.

LA RULING: Labor Arbiter ruled that petitioners are regular employees of Petron; also found that Petron merely
utilized RDG in its attempt to hide the existence of employee-employer relationship between it and petitioners
and avoid liability under labor laws. The Labor Arbiter declared them to have been illegally dismissed. Petron
was thus held solidarily liable with Romeo for the payment of petitioners separation pa.

NLRC RULING: NLRC affirmed the ruling of the LA.

CA RULING: The CA found no employer-employee relationship between the parties. The CA also found RDG to
be an independent labor contractor with sufficient capitalization and investment as shown by its financial
statement for year-end 2000.

ISSUE: Whether RDG is a labor-only contractor (prohibited) as such, petitioners are regular employees of
Petron.

SC RULING: YES. The prevailing rule on labor-only contracting at the time Petron and RDG entered into the
Contract for Services in June 2000 is DOLE Department Order No. 10, series of 1997.

"Permissible job contracting or subcontracting refers to an arrangement whereby a principal


agrees to farm out with a contractor or subcontractor the performance of a specific job, work, or
service within a definite or predetermined period, regardless of whether such job, work or,
service is to be performed or completed within or outside the premises of the principal.

Labor-only contracting, on the other hand, is a prohibited act, defined as "supplying workers to an employer who
does not have substantial capital or investment in the form of tools, equipment, machineries, work premises,
San Beda College of Law 153
4S: 2015 - 2016
LABOR LAW REVIEW Atty. Joyrich Golangco

among others, and the workers recruited and placed by such person are performing activities which are directly
45
related to the principal business of such employer." "[I]n distinguishing between prohibited labor-only
contracting and permissible job contracting, the totality of the facts and the surrounding circumstances of the
46
case shall be considered." Generally, the contractor is presumed to be a labor-only contractor, unless such
contractor overcomes the burden of proving that it has the substantial capital, investment, tools and the like.
However, where the principal is the one claiming that the contractor is a legitimate contractor, as in the present
47
case, said principal has the burden of proving that supposed status. It is thus incumbent upon Petron, and not
48
upon petitioners as Petron insists, to prove that RDG is an independent contractor.

Petron failed to discharge the burden of proving that RDG is a legitimate contractor. Hence, the presumption
that RDG is a labor-only contractor stands.

Here, the audited financial statements and other financial documents of RDG for the years 1999 to 2001
establish that it does have sufficient working capital to meet the requirements of its service contract. The
evidence adduced merely proves that RDG was financially qualified as a legitimate contractor but only with
respect to its last service contract with Petron in the year 2000.

While Petron was able to establish that RDG was financially capable as a legitimate contractor at the time of the
execution of the service contract in 2000, it nevertheless failed to establish the financial capability of RDG at the
time when petitioners actually started to work for Petron in 1968, 1979, 1981, 1987, 1990,1992 and 1993.

Petrons power of control over petitioners exists in this case.


The facts that petitioners were hired by Romeo or his father and that their salaries were paid by them do not
detract from the conclusion that there exists an employer-employee relationship between the parties due to
Petrons power of control over the petitioners. One manifestation of the power of control is the power to transfer
55
employees from one work assignment to another. Here, Petron could order petitioners to do work outside of
their regular "maintenance/utility" job. Also, petitioners were required to report for work everyday at the bulk
plant, observe an 8:00 a.m. to 5:00 p.m. daily work schedule, and wear proper uniform and safety helmets as
prescribed by the safety and security measures being implemented within the bulk plant. All these imply control.

Petitioners already attained regular status as employees of Petron.


Petitioners were given various work assignments. While the jobs performed by petitioners may be menial and
mechanical, they are nevertheless necessary and related to Petrons business operations. If not for these tasks,
Petrons products will not reach the consumers in their proper state. Indeed, petitioners roles were vital
inasmuch as they involve the preparation of the products that Petron will distribute to its consumers.
In view of these, and considering further that petitioners length of service entitles them to become regular
employees under the Labor Code, petitioners are deemed by law to have already attained the status as Petrons
regular employees. As such, Petron could not terminate their services on the pretext that the service contract it
entered with RDG has already lapsed.

Petron therefore, being the principal employer and RDG, being the labor-only contractor, are solidarily liable for
petitioners' illegal dismissal and monetary claims.

San Beda College of Law 154


4S: 2015 - 2016
LABOR LAW REVIEW Atty. Joyrich Golangco

1
60. FONTERRA BRANDS PHILS., INC. v. LEONARDO LARGADO AND TEOTIMO ESTRELLADO
G.R. No. 205300 March 18, 2015
VELASCO JR., J.

Doctrine:
A person is considered engaged in legitimate job contracting or subcontracting if the following conditions concur:
1. The contractor or subcontractor carries on a distinct and independent business and undertakes to
perform the job, work or service on its own account and under its own responsibility according to its own
manner and method, and free from the control and direction of the principal in all matters connected
with the performance of the work except as to the results thereof;
2. The contractor or subcontractor has substantial capital or investment; and
3. The agreement between the principal and contractor or subcontractor assures the contractual
employees entitlement to all labor and occupational safety and health standards, free exercise of the
right to self-organization, security of tenure, and social and welfare benefits.

FACTS:
Fonterra Brands Phils., Inc. (Fonterra) contracted the services of Zytron Marketing and Promotions Corp.
(Zytron) for the marketing and promotion of its milk and dairy products. Zytron provided Fonterra with trade
merchandising representatives (TMRs), including respondents Leonardo Largado (Largado) and Teotimo
Estrellado (Estrellado). The engagement of their services began on September 15, 2003 and May 27, 2002,
respectively.

On May 3, 2006, Fonterra sent Zytron a letter terminating its promotions contract. Fonterra then entered into an
agreement for manpower supply with A.C. Sicat Marketing and Promotional Services (A.C. Sicat). Desirous of
continuing their work as TMRs, respondents submitted their job applications with A.C. Sicat, which hired them
for a term of five (5) months.

When respondents 5-month contracts with A.C. Sicat were about to expire, they allegedly sought renewal
thereof, but were allegedly refused. This prompted respondents to file complaints for illegal dismissal,
regularization.

LA RULING: Labor Arbiter dismissed the complaint and ruled that: (1) respondents were not illegally dismissed
and (2) they were consecutively employed by Zytron and A.C. Sicat, not by Fonterra.

NLRC RULING: NLRC affirmed the Labor Arbiter.

CA RULING: The CA, found that A.C. Sicat satisfies the requirements of legitimate job contracting, but Zytron
does not. According to the CA: Zytrons paid-in capital of P250,000 cannot be considered as substantial capital;
its claim that it has the necessary tools and equipment for its business is unsubstantiated. Therefore, according
to the CA, respondents were Fonterras employees.

Additionally, the CA held that respondents were illegally dismissed since Fonterra itself failed to prove that their
dismissal is lawful. However, the illegal dismissal should be reckoned from the termination of their supposed
employment with Zytron on June 6, 2006.

ISSUE: Whether or not Zytron and A.C. Sicat are labor-only contractors, making Fonterra the employer of
herein respondents.

SC RULING:
NO. Fonterra is not the employer and respondents were not illegally dismissed. As correctly held by the Labor
Arbiter and the NLRC, the termination of respondents employment with Zytron was brought about by the
cessation of their contracts with the latter. By refusing to renew their contracts with Zytron, respondents
effectively resigned from the latter.

Respondents voluntarily terminated their employment with Zytron by refusing to renew their employment
contracts with the latter, applying with A.C. Sicat, and working as the latters employees, thereby abandoning
their previous employment with Zytron. Too, it is well to mention that for obvious reasons, resignation is
inconsistent with illegal dismissal. This being the case, Zytron cannot be said to have illegally dismissed
respondents.
San Beda College of Law 155
4S: 2015 - 2016
LABOR LAW REVIEW Atty. Joyrich Golangco

A.C. Sicat is as a legitimate job contractor, seeing that it is consistent with the rules on job contracting and is
sufficiently supported by the evidence on record.

A person is considered engaged in legitimate job contracting or subcontracting if the following conditions concur:
1. The contractor or subcontractor carries on a distinct and independent business and undertakes to
perform the job, work or service on its own account and under its own responsibility according to its own
manner and method, and free from the control and direction of the principal in all matters connected
with the performance of the work except as to the results thereof;
2. The contractor or subcontractor has substantial capital or investment; and
3. The agreement between the principal and contractor or subcontractor assures the contractual
employees entitlement to all labor and occupational safety and health standards, free exercise of the
right to self-organization, security of tenure, and social and welfare benefits.

A.C. Sicat has substantial capital, having assets totaling P5,926,155.76 as of December 31, 2006. Too, its
Agreement with Fonterra clearly sets forth that A.C. Sicat shall be liable for the wages and salaries of its
employees or workers, including benefits, premiums, and protection due them.

We agree with the findings of the CA that the termination of respondents employment with the latter was simply
brought about by the expiration of their employment contracts.

Foremost, respondents were fixed-term employees. It is clear that respondents were employed by A.C. Sicat as
project employees. In their employment contract with the latter, it is clearly stated that [A.C. Sicat is] temporarily
employing [respondents] as TMRs.

Respondents, by accepting the conditions of the contract with A.C. Sicat, were well aware of and even acceded
to the condition that their employment thereat will end on said pre-determined date of termination. They cannot
now argue that they were illegally dismissed by the latter when it refused to renew their contracts after its
expiration.

San Beda College of Law 156


4S: 2015 - 2016
LABOR LAW REVIEW Atty. Joyrich Golangco

ARTICLE 110

61. DEVELOPMENT BANK OF THE PHILIPPINES vs. NLRC


G.R. No. 108031 March 1, 1995
BELLOSILLO, J.

Doctrine:
A declaration of bankruptcy or a judicial liquidation must be present before the worker's preference may be
enforced.

FACTS:
Leonor Ang was an employee of Tropical Philippines Wood Industries, Inc. (TPWII). DBP, as mortgagee of
TPWII, foreclosed its plant facilities and equipment. It took possession of the foreclosed properties. From then
on the company ceased its operations. As a consequence private respondent was verbally terminated from the
service. Private respondent Ang filed with Labor Arbiter a complaint for separation pay, 13th month pay,
vacation and sick leave pay, salaries and allowances against TPWII, its General Manager, and petitioner. LA
awarded Angs separation pay and vacation and sick leave pay and held DBP subsidiarily liable in the even the
company failed to satisfy the judgment. The Labor Arbiter rationalized that the right of an employee to be paid
benefits due him from the properties of his employer is superior to the right of the latter's mortgage. NLRC
affirmed the ruling of LA.

ISSUE: Whether or not the declaration of bankruptcy or judicial liquidation required before the worker's
preference may be invoked under Art. 110 of the Labor Code.

SC RULING:
Yes, a declaration of bankruptcy or a judicial liquidation must be present before the worker's preference may be
enforced. In the event of insolvency, a principal objective should be to effect an equitable distribution of the
insolvents property among his creditors. To accomplish this there must first be some proceeding where notice to
all of the insolvent's creditors may be given and where the claims of preferred creditors may be bindingly
adjudicated. A preference applies only to claims which do not attach to specific properties. A lien creates a
charge on a particular property. The right of first preference as regards unpaid wages recognized by Article 110
does not constitute a lien on the property of the insolvent debtor in favor of workers.

It is but a preference of credit in their favor, a preference in application. It is a method adopted to determine and
specify the order in which credits should be paid in the final distribution of the proceeds of the insolvent's assets.
It is a right to a first preference in the discharge of the funds of the judgment debtor. Article 110 of the Labor
Code does not purport to create a lien in favor of workers or employees for unpaid wages either upon all of the
properties or upon any particular property owned by their employer. Claims for unpaid wages do not therefore
fall at all within the category of specially preferred claims established under Articles 2241 and 2242 of the Civil
Code, except to the extent that such claims for unpaid wages are already covered by Article 2241, number 6:
"claims for laborers: wages, on the goods manufactured or the work done;" or by Article 2242, number 3, "claims
of laborers and other workers engaged in the construction reconstruction or repair of buildings, canals and other
works, upon said buildings, canals and other works . . . . To the extent that claims for unpaid wages fall outside
the scope of Article 2241, number 6, and 22421 number 3, they would come within the ambit of the category of
ordinary preferred credits under Article 2244.

San Beda College of Law 157


4S: 2015 - 2016
LABOR LAW REVIEW Atty. Joyrich Golangco

ARTICLE 113

62. SHS PERFORATED MATERIALS, INC. vs. DIAZ


G.R. No. 185814 October 13, 2010

Doctrine:
Absent a showing that the withholding of complainants wages falls under the exceptions provided in Article 113,
the withholding thereof is thus unlawful.

FACTS:
Manuel Diaz was hired by petitioner SHS as Manager for Business Development on probationary status. During
his employment in the said company, Mr. Hartmannshenn, the companys president, was often abroad and sent
instructions to respondent either by electronic mail or through telephone or mobile phone. During meetings with
respondent, Hartmannshen expressed his dissatisfaction over respondents poor performance. When
Hartmannshenn arrived in the Philippines, he notified respondent of his arrival through electronic mail
messages, which the respondent claimed he never received, but the respondent refused to respond and to meet
with him. Hartmannshenn instructed not to release respondents salary. Later that afternoon, respondent called
and inquired about his salary but he was informed that it was being withheld and that he had to immediately
communicate with Hartmannshen. The next day, respondent served a demand letter and a resignation letter. In
the evening of the same day, respondent met with Hartmannshenn in Alabang. The latter told him that he was
extremely disappointed for the following reasons: his poor work performance; his unauthorized leave and
malingering from November 16 to November 30, 2005; and failure to immediately meet Hartmannshenn upon
his arrival from Germany. Respondent, on the other hand, claimed that the meeting with Hartmannshenn took
place in the evening of December 1, 2005, at which meeting the latter insulted him and rudely demanded that he
accept P25,000.00 instead of his accrued wage and stop working for SHS, which demands he refused. Later
that same night, he sent Hartmannshenn and Schumacher an electronic mail message appealing for the release
of his salary. Respondent filed a Complaint against the petitioners for illegal dismissal; non-payment of
th
salaries/wages and 13 month pay with prayer for reinstatement and full backwages; exemplary damages, and
attorneys fees, costs of suit, and legal interest.

LA RULING: The Labor Arbiter rendered a decision declaring complainant as having been illegally dismissed
and further ordering his immediate reinstatement without loss of seniority rights and benefits. The LA found that
respondent was constructively dismissed because the withholding of his salary was contrary to Article 116 of the
Labor Code as it was not one of the exceptions for allowable wage deduction by the employer under Article 113
of the Labor Code. He had no other alternative but to resign because he could not be expected to continue
working for an employer who withheld wages without valid cause

NLRC RULING: On appeal, NLRC reversed the decision of LA. The NLRC explained that the withholding of
respondents salary was a valid exercise of management prerogative. The act was deemed justified as it was
reasonable to demand an explanation for failure to report to work and to account for his work accomplishments.
The NLRC held that the respondent voluntarily resigned as evidenced by the language used in his resignation
letter and demand letters. CA reversed NLRC resolution and held that withholding respondents salary was not a
valid exercise of management prerogative as there is no such thing as a management prerogative to withhold
wages temporarily.

ISSUE: Whether or not the temporary withholding of salary of employee by employer is a valid exercise of
management prerogative

SC RULING: No. Management prerogative refers to the right of an employer to regulate all aspects of
employment, such as the freedom to prescribe work assignments, working methods, processes to be followed,
regulation regarding transfer of employees, supervision of their work, lay-off and discipline, and dismissal and
recall of work. Although management prerogative refers to the right to regulate all aspects of employment, it
cannot be understood to include the right to temporarily withhold salary/wages without the consent of the
employee. Any withholding of an employees wages by an employer may only be allowed in the form of wage
deductions under the circumstances provided in Article 113 of the Labor Code. As correctly pointed out by the
LA, absent a showing that the withholding of complainants wages falls under the exceptions provided in Article
113, the withholding thereof is thus unlawful.

San Beda College of Law 158


4S: 2015 - 2016
LABOR LAW REVIEW Atty. Joyrich Golangco

What made it impossible, unreasonable or unlikely for respondent to continue working for SHS was the unlawful
withholding of his salary. For said reason, he was forced to resign. It is of no moment that he served his
resignation letter on November 30, 2005, the last day of the payroll period and a non-working holiday, since his
salary was already due him on November 29, 2005, being the last working day of said period. In fact, he was
then informed that the wages of all the other SHS employees were already released, and only his was being
withheld. What is significant is that the respondent prepared and served his resignation letter right after he was
informed that his salary was being withheld. It would be absurd to require respondent to tolerate the unlawful
withholding of his salary for a longer period before his employment can be considered as so impossible,
unreasonable or unlikely as to constitute constructive dismissal. Even granting that the withholding of
respondents salary on November 30, 2005, would not constitute an unlawful act, the continued refusal to
release his salary after the payroll period was clearly unlawful. The petitioners claim that they prepared the
check ready for pick-up cannot undo the unlawful withholding.

San Beda College of Law 159


4S: 2015 - 2016
LABOR LAW REVIEW Atty. Joyrich Golangco

ARTICLE 124 Standards/Criteria for Minimum Wage Fixing

63. P.I. MANUFACTURING, INC. v. P.I. MANUFACTURING SUPERVISORS AND FOREMAN ASSOCIATION
and the NATIONAL LABORUNION
G.R. No. 167217 February 4, 2008
SANDOVAL-GUTIERREZ

DOCTRINE:
The Court adopts the policy that requires recognition and validation of wage increases given by employers
either unilaterally or as a result of collective bargaining negotiations in an effort to correct wage distortions.

FACTS:
P.I. Manufacturing, Inc. is a domestic corporation engaged in the manufacture and sale of household
appliances. P.I. Manufacturing Supervisors and Foremen Association (PIMASUFA) is an organization of
supervisors and foremen, joined in this case by its federation, the National Labor Union (NLU). In 1987, the
President signed into law RA 6640 providing, among others, an increase in the statutory minimum wage and
salary rates of employees and workers in the private sector.

Section 2 provides: The statutory minimum wage rates of workers and employees in the private sector, whether
agricultural or non-agricultural, shall be increased by ten pesos (P10.00) per day, except non-agricultural
workers and employees outside Metro Manila who shall receive an increase of eleven pesos (P11.00) per
day: Provided, That those already receiving above the minimum wage up to one hundred pesos
(P100.00) shall receive an increase of ten pesos (P10.00) per day. Excepted from the provisions of this Act are
domestic helpers and persons employed in the personal service of another.

P.I. and PIMASUFA entered into a new Collective Bargaining Agreement (1987 CBA) whereby the supervisors
were granted an increase of P625.00 per month and the foremen, P475.00 per month. The increases were
made retroactive prior to the passage of R.A. No. 6640, and every year thereafter until July 26, 1989.

In 1989, PIMASUFA and NLU filed a complaint with the Arbitration Branch of the NLRC, charging P.I. with
violation of R.A. No. 6640. Theyhttp://sc.judiciary.gov.ph/jurisprudence/2008/feb2008/167217.htm -
_ftn3 attached to their complaint a numerical illustration of wage distortion resulting from the implementation of
R.A. No. 6640.

LABOR ARBITER RULING: in favor of PIMASUFA.

NLRC RULING: affirmed the Labor Arbiters judgment.

CA RULING: affirmed the Decision of the NLRC with modification by raising the 13.5% wage increase
to 18.5%.

ISSUE: Whether the increase resulting from any wage distortion caused by the implementation of Republic Act
6640 is waivable.

SC RULING:
YES. R.A. No. 6727, otherwise known as the Wage Rationalization Act, explicitly defines wage distortion as: a
situation where an increase in prescribed wage rates results in the elimination or severe contraction of
intentional quantitative differences in wage or salary rates between and among employee groups in an
establishment as to effectively obliterate the distinctions embodied in such wage structure based on skills,
length of service, or other logical bases of differentiation.

In this case, the Court of Appeals correctly ruled that a wage distortion occurred due to the implementation of
R.A. No. 6640. Significantly, the 1987 CBA wage increases almost doubled that of the P10.00 increase under
R.A. No. 6640. Clearly, the gap between the wage rates of the supervisors and those of the foremen was
inevitably re-established. It continued to broaden through the years.

Interestingly, such gap as re-established by virtue of the CBA is more than a substantial compliance with R.A.
No. 6640. The CA erred in not taking into account the provisions of the CBA viz-a-viz the wage increase under

San Beda College of Law 160


4S: 2015 - 2016
LABOR LAW REVIEW Atty. Joyrich Golangco

the said law. To direct petitioner to grant an across-the-board increase to all of them, regardless of the amount
of wages they are already receiving, would be harsh and unfair to the former.

To compel employers simply to add on legislative increases in salaries or allowances without regard to what is
already being paid, would be to penalize employers who grant their workers more than the statutory prescribed
minimum rates of increases. Clearly, this would be counter-productive so far as securing the interests of labor is
concerned.

At this juncture, it must be stressed that a CBA constitutes the law between the
parties when freely and voluntarily entered
into.http://sc.judiciary.gov.ph/jurisprudence/2008/feb2008/167217.htm - _ftn13 Here, it has not been shown that
respondent PIMASUFA was coerced or forced to sign the 1987 CBA. All of its 13 officers signed the CBA with
the assistance of respondent NLU.

San Beda College of Law 161


4S: 2015 - 2016
LABOR LAW REVIEW Atty. Joyrich Golangco

64. BANKARD EMPLOYEES UNION-WORKERS ALLIANCE TRADE UNIONS v. NATIONAL LABOR
RELATIONS COMMISSION and BANKARD, INC.
G.R. No. 140689 February 17, 2004
CARPIO MORALES, J.:

ARTICLE 124

DOCTRINE:
The four elements of wage distortion are: (1.) An existing hierarchy of positions with corresponding salary rates;
(2) A significant change in the salary rate of a lower pay class without a concomitant increase in the salary rate
of a higher one; (3) The elimination of the distinction between the two levels; and (4) The existence of the
distortion in the same region of the country.

FACTS:
Bankard, Inc. classifies its employees by levels, to wit: Level I to V. In 1993, its Board of Directors approved a
New Salary Scale for the purpose of making its hiring rate competitive in the industrys labor market. The New
Salary Scale increased the hiring rates of new employees, to wit: Levels I and V by P1,000, and Levels II, III and
IV by P900. Accordingly, the salaries of employees who fell below the new minimum rates were also adjusted to
reach such rates under their levels.

Bankards move drew the Bankard Employees Union-WATU, the duly certified exclusive bargaining agent of the
regular rank and file employees of Bankard, to press for the increase in the salary of its old, regular employees.
Bankard took the position, however, that there was no obligation on the part of the management to grant to all
its employees the same increase in an across-the-board manner.

As the continued request remained unheeded, it filed a Notice of Strike on the ground of discrimination and
other acts of Unfair Labor Practice.

NLRC RULING: finding no wage distortion, dismissed the case for lack of merit.

CA RULING: denied the same for lack of merit. Hence, the present petition.

ISSUE: Whether the unilateral adoption by an employer of an upgraded salary scale that increased the hiring
rates of new employees without increasing the salary rates of old employees resulted in wage distortion within
the contemplation of Article 124 of the Labor Code.

SC RULING:
NO. Upon the enactment of R.A. No. 6727 (WAGE RATIONALIZATION ACT, amending, among others, Article
124 of the Labor Code) on June 9, 1989, the term wage distortion was explicitly defined as:
... a situation where an increase in prescribed wage rates results in the elimination or severe contraction of
intentional quantitative differences in wage or salary rates between and among employee groups in an
establishment as to effectively obliterate the distinctions embodied in such wage structure based on skills,
length of service, or other logical bases of differentiation.

The four elements of wage distortion are: (1.) An existing hierarchy of positions with corresponding salary rates;
(2) A significant change in the salary rate of a lower pay class without a concomitant increase in the salary rate
of a higher one; (3) The elimination of the distinction between the two levels; and (4) The existence of the
distortion in the same region of the country.

Involved in the classification of employees are various factors such as the degrees of responsibility, the skills
and knowledge required, the complexity of the job, or other logical basis of differentiation. The differing wage
rate for each of the existing classes of employees reflects this classification.

Petitioner maintains that for purposes of wage distortion, the classification is not one based on levels or ranks
but on two groups of employees, the newly hired and the old, in each and every level, and not between and
among the different levels or ranks in the salary structure.

The employees of Bankard have been historically classified into levels, i.e. I to V, and not on the basis of their
length of service. The Union cannot make a contrary classification of Bankards employees without encroaching
San Beda College of Law 162
4S: 2015 - 2016
LABOR LAW REVIEW Atty. Joyrich Golangco

upon recognized management prerogative of formulating a wage structure, in this case, one based on level. It is
thus clear that there is no hierarchy of positions between the newly hired and regular employees of Bankard,
hence, the first element of wage distortion is wanting. For purposes of determining the existence of wage
distortion, employees cannot create their own independent classification and use it as a basis to demand an
across-the-board increase in salary.

Even assuming that there is a decrease in the wage gap between the pay of the old employees and the newly
hired employees, said gap is not significant as to obliterate or result in severe contraction of the intentional
quantitative differences in the salary rates between the employee group. The classification under the wage
structure is based on the rank of an employee, not on seniority. For this reason, wage distortion does not appear
to exist.

San Beda College of Law 163


4S: 2015 - 2016
LABOR LAW REVIEW Atty. Joyrich Golangco

th
13 MONTH PAY

65. CENTRAL AZUCARERA DE TARLAC v. CENTRAL AZUCARERA DE TARLAC LABOR UNION-NLU


G.R. No. 188949 July 26, 2010
NACHURA, J.:

DOCTRINE:
The term basic salary of an employee for the purpose of computing the 13th-month pay was interpreted to
include all remuneration or earnings paid by the employer for services rendered, but does not include
allowances and monetary benefits which are not integrated as part of the regular or basic salary, such as the
cash equivalent of unused vacation and sick leave credits, overtime, premium, night differential and holiday pay,
and cost-of-living allowances. However, these salary-related benefits should be included as part of the basic
salary in the computation of the 13th-month pay if, by individual or collective agreement, company practice or
policy, the same are treated as part of the basic salary of the employees.

FACTS:
Central Azucarera de Tarlac is a domestic corporation engaged in the business of sugar manufacturing, while
Central Azucarera de Tarlac Labor Union-NLU is a legitimate labor organization which serves as the exclusive
bargaining representative of the Central's rank-and-file employees. The controversy stems from the
interpretation of the term basic pay, essential in the computation of the 13th-month pay.

In compliance with P.D. No. 851, the Central granted its employees the mandatory 13th month pay since 1975.
The formula used was: Total Basic Annual Salary divided by 12. Included in the computation of the Total Basic
Annual Salary were the following: basic monthly salary; first 8 hours overtime pay on Sunday and legal/special
holiday; night premium pay; and vacation and sick leaves for each year. Throughout the years, the Central used
this computation until 2006.

After a strike staged by the Union, the Central gave the employees their 13th-month pay based on the
employees total earnings during the year divided by 12. The latter objected to this computation. The Union filed
a complaint against for money claims based on the alleged diminution of benefits/erroneous computation of
13th-month pay before the Regional Arbitration Branch of the NLRC.

LA RULING: dismissed the complaint and declared that the Central had the right to rectify the error in the
computation of the 13th-month pay of its employees.

NLRC RULING: reversed the Labor Arbiter.

CA RULING: affirmed the decision and resolution of the NLRC. Hence, the petition.

ISSUE: Whether there was an error in the computation of the employees' 13th month pay.

SC RULING:
YES. The 13th-month pay mandated by P.D. No. 851 represents an additional income based on wage but not
part of the wage. It is equivalent to one-twelfth (1/12) of the total basic salary earned by an employee within a
calendar year. All rank-and-file employees, regardless of their designation or employment status and
irrespective of the method by which their wages are paid, are entitled to this benefit, provided that they have
worked for at least one month during the calendar year. If the employee worked for only a portion of the year,
the 13th-month pay is computed pro rata.

It is clear that there could have no erroneous interpretation or application of what is included in the term basic
salary for purposes of computing the 13th-month pay of employees. From the inception of P.D. No. 851 on
December 16, 1975, clear-cut administrative guidelines have been issued to insure uniformity in the
interpretation, application, and enforcement of the provisions of P.D. No. 851 and its implementing regulations.

As correctly ruled by the CA, the practice of the Central in giving 13th-month pay based on the employees gross
annual earnings which included the basic monthly salary, premium pay for work on rest days and special
holidays, night shift differential pay and holiday pay continued for almost thirty (30) years and has ripened into a
company policy or practice which cannot be unilaterally withdrawn.

San Beda College of Law 164


4S: 2015 - 2016
LABOR LAW REVIEW Atty. Joyrich Golangco

The argument of the Central that the grant of the benefit was not voluntary and was due to error in the
interpretation of what is included in the basic salary deserves scant consideration. No doubtful or difficult
question of law is involved in this case. The voluntariness of the grant of the benefit was manifested by the
number of years the employer had paid the benefit to its employees. The Central only changed the formula in
the computation of the 13th-month pay after almost 30 years and only after the dispute between the
management and employees erupted. This act of changing the formula at this time cannot be sanctioned, as it
indicates a badge of bad faith.

San Beda College of Law 165


4S: 2015 - 2016
LABOR LAW REVIEW Atty. Joyrich Golangco

ARTICLE 128 Visitorial and Enforcement Power

66. PEOPLES BROADCASTING SERVICE (BOMBO RADYO PHILS., INC.) vs. THE SECRETARY OF THE
DEPARTMENT OF LABOR AND EMPLOYMENT, THE REGIONAL DIRECTOR, DOLE REGION VII, and
JANDELEON JUEZAN
G.R. No. 179652 March 6, 2012
VELASCO, JR., J.:

DOCTRINE:
Under Art. 128(b) of the Labor Code, as amended by RA 7730, the DOLE is fully empowered to make a
determination as to the existence of an employer-employee relationship in the exercise of its visitorial and
enforcement power, subject to judicial review, not review by the NLRC.

FACTS:
Jandeleon Juezan filed a complaint against Peoples Broadcasting Service (Bombo) with the DOLE Regional
Office No. VII, Cebu City, for illegal deduction, nonpayment of service incentive leave, 13th month pay, among
others. After the conduct of summary investigations, and after the parties submitted their position papers, the
DOLE Regional Director found that Juezan was an employee of Bombo, and was entitled to his money
claims. Bombo sought reconsideration of the Directors Order, but failed.

The Acting DOLE Secretary dismissed Bombos appeal. When the matter was brought before the CA, where
Bombo claimed that it had been denied due process, it was held that Bombo was accorded due process as it
had been given the opportunity to be heard, and that the DOLE Secretary had jurisdiction over the matter, as
the jurisdictional limitation imposed by Article 129 of the Labor Code on the power of the DOLE Secretary under
Art. 128(b) of the Code had been repealed by Republic Act No. (RA) 7730.

SC reversed the CA Decision and the complaint against Bombo was dismissed. The Court found that there was
no employer-employee relationship between Bombo and Juezan. It was held that while the DOLE may make a
determination of the existence of an employer-employee relationship, this function could not be co-extensive
with the visitorial and enforcement power provided in Art. 128(b) of the Labor Code, as amended by RA
7730. The NLRC was held to be the primary agency in determining the existence of an employer-employee
relationship. From this Decision, the Public Attorneys Office (PAO) filed a Motion for Clarification of Decision
(with Leave of Court). The PAO sought to clarify as to when the visitorial and enforcement power of the DOLE
be not considered as co-extensive with the power to determine the existence of an employer-employee
relationship. The SC revisits its former conclusion.

ISSUE: Whether DOLE can make a determination of the existence of employer-employee relationship.

SC RULING:
YES. No limitation in the law was placed upon the power of the DOLE to determine the existence of an
employer-employee relationship. No procedure was laid down where the DOLE would only make a preliminary
finding, that the power was primarily held by the NLRC. The law did not say that the DOLE would first seek the
NLRCs determination of the existence of an employer-employee relationship, or that should the existence of the
employer-employee relationship be disputed, the DOLE would refer the matter to the NLRC. The DOLE must
have the power to determine whether or not an employer-employee relationship exists, and from there to decide
whether or not to issue compliance orders in accordance with Art. 128(b) of the Labor Code, as amended by RA
7730.

The determination of the existence of an employer-employee relationship by the DOLE must be respected. The
expanded visitorial and enforcement power of the DOLE granted by RA 7730 would be rendered nugatory if the
alleged employer could, by the simple expedient of disputing the employer-employee relationship, force the
referral of the matter to the NLRC. If the DOLE makes a finding that there is an existing employer-employee
relationship, it takes cognizance of the matter, to the exclusion of the NLRC. The DOLE would have no
jurisdiction only if the employer-employee relationship has already been terminated, or it appears, upon review,
that no employer-employee relationship existed in the first place.

If a complaint is brought before the DOLE to give effect to the labor standards provisions of the Labor Code or
other labor legislation, and there is a finding by the DOLE that there is an existing employer-employee
relationship, the DOLE exercises jurisdiction to the exclusion of the NLRC. If the DOLE finds that there is no
San Beda College of Law 166
4S: 2015 - 2016
LABOR LAW REVIEW Atty. Joyrich Golangco

employer-employee relationship, the jurisdiction is properly with the NLRC. If a complaint is filed with the DOLE,
and it is accompanied by a claim for reinstatement, the jurisdiction is properly with the Labor Arbiter, under Art.
217(3) of the Labor Code. If a complaint is filed with the NLRC, and there is still an existing employer-employee
relationship, the jurisdiction is properly with the DOLE. The findings of the DOLE, however, may still be
questioned through a petition for certiorari under Rule 65 of the Rules of Court.

San Beda College of Law 167


4S: 2015 - 2016
LABOR LAW REVIEW Atty. Joyrich Golangco

67. MRS. ALBERTA YANSON v. SECRETARY OF LABOR AND EMPLOYMENT
G.R. No. 159026 February 11, 2008
AUSTRIA-MARTINEZ, J.:

ARTICLE 128

DOCTRINE:
The posting of the proper amount of the appeal bond under Article 128 (b) is mandatory for the perfection of an
appeal from a monetary award in labor standard cases.

FACTS:
In 1998, Mardy Cabigo and 40 other workers filed with DOLE Bacolod a request for payroll inspection of
Hacienda Valentin Balabag owned by Alberta Yanson. DOLE Bacolod conducted an inspection of the
establishment and issued a Notice of Inspection Report, finding Yanson liable for the following violations of labor
standard laws:

1. Underpayment of salaries and wages (workers being paid a daily rate of P90.00 since 1997 and P75.00
prior to such year);
2. Non-payment of 13th month pay for two (2) years;
3. Non-payment of Social Amelioration Bonus (SAB) for two (2) years;
4. Non-payment of employers 1/3 carabao share.

In addition, DOLE Bacolod scheduled a summary investigation. Yanson did not appear in any of the scheduled
hearings, or present any pleading or document. In a Compliance Order, DOLE Bacolod directed Yanson to pay
a total of P372,444 and to correct existing violations of occupational safety and health standards. It then issued
a Writ of Execution. Yanson filed with public respondent a Verified Appeal and posted a bond.

SECRETARY OF LABOR RULING: dismissed the appeal.

CA RULING: Petition for Certiorari was denied due course and dismissed. Hence, the present recourse.

ISSUE:
1. Whether the compliance order by DOLE Bacolod, in the exercise of its visitorial and enforcement power,
was proper. YES
2. Whether the appeal was perfected. NO.

SC RULING:
For its perfection, the appeal was subject to the requirements prescribed under Article 128 of the Labor Code,
as amended by Republic Act No. 7730, viz.:

Art. 128. Visitorial and Enforcement Power. - x x x (b) Notwithstanding the provisions of Articles 129 and
217 of this Code to the contrary, and in cases where the relationship of employer-employee still exists,
the Secretary of Labor and Employment or his duly authorized representatives shall have the power to
issue compliance orders to give effect to the labor standards provisions of this Code and other labor
legislation based on the findings of labor employment and enforcement officers or industrial safety
engineers made in the course of inspection. The Secretary or his duly authorized representatives shall
issue writs of execution to the appropriate authority for the enforcement of their orders, except in cases
where the employer contests the findings of the labor employment and enforcement officer and raises
issues supported by documentary proofs which were not considered in the course of inspection.

An order issued by the duly authorized representative of the Secretary of Labor and Employment under this
article may be appealed to the latter. In case said order involves a monetary award, an appeal by the employer
may be perfected only upon the posting of a cash or surety bond issued by a reputable bonding company duly
accredited by the Secretary of Labor and Employment in the amount equivalent to the monetary award in the
order appealed from.

When Yanson filed her Verified Appeal and Supplement to the Verified Appeal, Public respondent rejected said
appeal for insufficiency of the appeal bond. The posting of the proper amount of the appeal bond under Article
128 (b) is mandatory for the perfection of an appeal from a monetary award in labor standard cases. Also
San Beda College of Law 168
4S: 2015 - 2016
LABOR LAW REVIEW Atty. Joyrich Golangco

applying the Implementing Rules, there is one other reason for holding that Yanson failed to perfect her appeal.
It is of record that she received Compliance Order issued by DOLE-Bacolod. She was put on actual notice not
only of the existence of the Compliance Order but also of the summary investigation of her establishment. It
behooves her to file a timely appeal to public respondent or object to the conduct of the investigation. Yanson
did neither, opting instead to sit idle and wait until the following year to question the investigation and resultant
order, in the guise of opposing the writ of execution.

In fine, the CA was correct in holding that public respondent did not commit grave abuse of discretion in
rejecting the appeal due to the insufficiency of her appeal bond.

Even on its substance, her appeal would still not prosper. The determination made by DOLE-Bacolod on this
matter binds the Court, especially as it was not reversed by public respondent and the CA.

San Beda College of Law 169


4S: 2015 - 2016
LABOR LAW REVIEW Atty. Joyrich Golangco

68. BALLADARES ET. AL. v. PEAK VENTURES CORPORATION
G.R. No. 161794 June 16, 2009
Nachura, J.:

ART. 128: VISITORIAL AND ENFORCEMENT POWERS OF THE DOLE REGIONAL DIRECTOR

DOCTRINE:
The visitorial and enforcement powers of the DOLE Regional Director to order and enforce compliance with
labor standard laws can be exercised even when the individual claim exceeds P5,000. However, if the labor
standards case is covered by the exception clause in Article 128 (b) of the Labor Code, then the Regional
Director will have to endorse the case to the appropriate Arbitration Branch of the NLRC.

FACTS:
Petitioners Nestor J. Balladares et al., were employed by respondent Peak Ventures Corp as security guards
and were assigned at the premises of respondent YMOAA. They filed a complaint for underpayment of wages
against their employer, Peak Ventures, with the DOLE. Acting on the complaint, DOLE conducted an inspection
of Peak Ventures and the following violations were noted: underpayment of the minimum wage and other
auxiliary benefits; pertinent employment records were not available at the time of inspection.

A Notice of Inspection Result was issued to Peak Ventures instructing them to effect restitution and/or file its
objections within five working days from receipt thereof. Respondent failed to correct the violations or contest
the findings as required, hence, the parties were summoned for hearing. Peak Ventures moved to implead its
client YMOAA, claiming that any underpayment of wages arose from the failure of YMOAA to pay Peak
Ventures the amount due petitioners as prescribed by various wage orders.

After the hearing, DOLE Regional Director Maximo Lim rendered judgment in favor of petitioners and ruled that
the contractor was jointly and severally liable with the principal. Lim averred that because Peak Ventures failed
to controvert the complaint and its repeated denial to give access to records, it is deemed to have waived its
constitutional right to due process. Petitioners were awarded P1,106,298. Peak Ventures filed a motion for
reconsideration, but the same was denied prompting them to appeal to the CA.

CA RULING: The CA granted the petition, ruling that the Regional Director had no jurisdiction to hear and
decide the case, because the claims of each of the petitioners exceeded P5,000.00, and the power to adjudicate
such claims belonged to the Labor Arbiter, pursuant to Servandos, Inc. v. Secretary of Labor. The appellate
court ratiocinated that this exclusive jurisdiction of the Labor Arbiters was confirmed by Article 129 of the Labor
Code, which excludes from the jurisdiction of the Regional Directors or any hearing officer of the DOLE the
power to hear and decide claims of employees arising from employer-employee relations exceeding the amount
of P5,000.00 for each employee.

ISSUE: Does DOLE Regional Director has jurisdiction to even though the claims of the complainants exceeded
P5,000?

SC RULING:
YES. Yes. The Supreme Court ruled that the visitorial and enforcement powers of the DOLE Regional Director
to order and enforce compliance with labor standard laws can be exercised even when the individual claim
exceeds P5,000. However, if the labor standards case is covered by the exception clause in Article 128 (b) of
the Labor Code, then the Regional Director will have to endorse the case to the appropriate Arbitration Branch
of the NLRC. In order to divest the Regional Director or his representatives of jurisdiction, the following
elements must be present: (a) that the employer contests the findings of the labor regulations officer and
raises issues thereon; (b) that in order to resolve such issues, there is a need to examine evidentiary matters;
and (c) that such matters are not verifiable in the normal course of inspection. The rules also provide that the
employer shall raise such objections during the hearing of the case or at any time after receipt of the notice of
inspection results.

In the case at bar, Peak Ventures did not contest the findings of the labor regulations officer during the hearing
or after receipt of notice of the inspection results. Accordingly, we find no sufficient reason to warrant the
certification of the instant case to the LA and divest the Regional Director of jurisdiction. Respondent did not
contest the findings of the labor regulations officer. Even during the hearing, respondent never denied that
petitioners were not paid correct wages and benefits.
San Beda College of Law 170
4S: 2015 - 2016
LABOR LAW REVIEW Atty. Joyrich Golangco

69. ALLIED INVESTIGATION BUREAU, INC., v. SECRETARY OF LABOR


GR No. 122006 November 24, 1999

ART. 128 VISITORIAL AND ENFORCEMENT POWERS OF THE DOLE REGIONAL DIRECTOR

DOCTRINE:
While it is true that under Articles 129 and 217of the Labor Code, the Labor Arbiter has jurisdiction to hear and
decide cases where the aggregate money claims of each employee exceedsP5,000.00, said provisions of law
do not contemplate nor cover the visitorial and enforcement powers of the Secretary of Labor or his duly
authorized representatives. Said powers are defined and set forth in Art. 128 of the Labor Code.

FACTS:
Petitioner Allied Investigation Bureau is a security agency which entered into a security contract with Novelty
Philippines Inc (NPI). Private respondents Melvin Pelayo and Samuel Sucanel, two of the security guards
assigned by petitioner to NPI, filed a complaint with the Office of respondent Regional Director Romeo Young,
charging petitioner with non-compliance with a wage order increasing the minimum daily pay of workers.
Regional Director Young conducted inspection visits at petitioners establishment and found that Petitioner failed
to implement the wage increase. Petitioner was required to effect restitution and/or correction of the foregoing
within five calendar days, or challenge the findings within five working days. Thereafter, a series of conferences
and hearings were scheduled by the Regional Director to facilitate amicable settlement. However, despite due
notice, petitioner failed to appear in any of said hearings. As a result, the Regional Director ruled in favor of
private respondents and awarded them P807,570.

Petitioners appealed the Order to respondent Secretary of Labor, without posting a cash or surety bond, as such
the appeal was dismissed. Petitioner argues that the power to adjudicate money claims belongs to the Labor
Arbiter who has exclusive jurisdiction over employees claims where the aggregate amount of the claims of each
employee exceeds P5,000.

ISSUE: Whether or not the DOLE Regional Director acted without jurisdiction in adjudicating the private
respondents claims which were in excess of P5,000.

RULING:
Yes. While it is true that under Articles 129 and 217of the Labor Code, the Labor Arbiter has jurisdiction to hear
and decide cases where the aggregate money claims of each employee exceedsP5,000.00, said provisions of
law do not contemplate nor cover the visitorial and enforcement powers of the Secretary of Labor or his duly
authorized representatives. Said powers are defined and set forth in Art. 128 of the Labor Code.
Art. 128. Visitorial and enforcement power.
(a) The Secretary of Labor or his duly authorized representatives, including labor regulation officers, shall have
access to employers records and premises at any time of the day or night whenever work is being
undertaken therein, and the right to copy therefrom, to question any employee and investigate any fact,
condition or matter which may be necessary to determine violations or which may aid in the enforcement of
this Code and of any labor law, wage order or rules and regulations issued pursuant thereto.
(b) Notwithstanding the provisions of Articles 129 and 217 of this Code to the contrary, and in cases where the
relationship of employer-employee exists, the Secretary of Labor and Employment or his duly authorized
representatives shall have the power to issue compliance orders to give effect to the labor standards
provisions of this Code and other labor legislation based on the findings of labor employment and
enforcement officers or industrial safety engineers made in the course of inspection. The Secretary or his
duly authorized representatives shall issue writs of execution to the appropriate authority for the
enforcement of their orders, except in cases where the employer contests the findings of the labor
employment and enforcement officer and raises issues supported by documentary proofs which were not
considered in the course of inspection.
San Beda College of Law 171
4S: 2015 - 2016
LABOR LAW REVIEW Atty. Joyrich Golangco

An order issued by the duly authorized representatives of the Secretary of Labor and Employment under this
article may be appealed to the latter. In case said order involves a monetary award, an appeal by the employer
may be perfected only upon the posting of a cash or surety bond issued by a reputable bonding company duly
accredited by the Secretary of Labor and Employment in the amount equivalent to the monetary award in the
order appealed from.

In the case at bar, the Office of respondent Regional Director conducted inspection visits at petitioners
establishment on February 9 and 14, 1995 in accordance with the above-mentioned provision of law. In the
course of said inspection, several violations of the labor standard provisions of the Labor Code were discovered
and reported by Senior Labor Enforcement Officer Eduvigis A. Acero in his Notice of Inspection Results. It was
on the bases of the aforesaid findings (which petitioner did not contest), that respondent Regional Director
issued the assailed Order for petitioner to pay private respondents the respective wage differentials due them.

Clearly, as the duly authorized representative of respondent Secretary of Labor, and in the lawful exercise of the
Secretarys visitorial and enforcement powers under Article 128 of the Labor Code, respondent Regional
Director had jurisdiction to issue his impugned Order.

70. URBANES v. SECRETARY OF LABOR


GR No. 122791 February 19, 2003

ART. 128 VISITORIAL AND ENFORCEMENT POWERS OF THE DOLE REGIONAL DIRECTOR

DOCTRINE:
It is well settled in law and jurisprudence that where no employer-employee relationship exists between the
parties and no issue is involved which may be resolved by reference to the Labor Code, other labor statutes or
any collective bargaining agreement, it is the Regional Trial Court that has jurisdiction. In its complaint, private
respondent is not seeking any relief under the Labor Code but seeks payment of a sum of money and damages
on account of petitioner's alleged breach of its obligation under their Guard Service Contract. The action is
within the realm of civil law hence jurisdiction over the case belongs to the regular courts. While the resolution of
the issue involves the application of labor laws, reference to the labor code was only for the determination of the
solidary liability of the petitioner to the respondent where no employer-employee relation exists.

FACTS:
Petitioner Placido O. Urbanes, Jr., doing business under the name and style of Catalina Security Agency,
entered into an agreement to provide security services to respondent Social Security System (SSS). During the
effectivity of the agreement, petitioner, by letter of May 16, 1994, requested the SSS for the upward adjustment
of their contract rate in view of Wage Order No. NCR-03. As SSS refused to comply, On June 29, 1994,
petitioner filed a complaint with the DOLE-NCR against the SSS seeking the implementation of Wage Order No.
NCR-03. In its position paper, the SSS prayed for the dismissal of the complaint on the ground that petitioner is
not the real party in interest and has no legal capacity to file the same. In any event, it argued that if it had any
obligation, it was to the security guards. On the other hand, petitioner in his position paper, citing Eagle Security
Agency, Inc. v. NLRC, contended that the security guards assigned to the SSS do not have any legal basis to
file a complaint against it for lack of contractual privity.

Finding for petitioner, the Regional Director of the DOLE-NCR issued an Order for SSS to pay petitioner P1.6
million. SSS appealed to the Secretary of Labor, claiming that the Regional Director has no jurisdiction to issue
the assailed order. The Secretary set aside the order and remanded the case. Petitioner filed the present
petition for certiorari with the Supreme Court asserting that the Secretary of Labor does not have jurisdiction to
review appeals from decisions of the Regional Directors in complaints filed under Art. 129 of the Labor Code.
They claim that appeals from orders of Regional directors should be made with the NLRC.

ISSUE: Whether or not the Secretary of Labor has jurisdiction to review appeals from decisions of the Regional
Directors in complaints filed under Art. 129.

SC RULING:
No. Neither the petitioners contention nor the SSSs is impressed with merit, rather, it is the RTC that has
jurisdiction over the subject matter of the present case. It is well settled in law and jurisprudence that where no
employer-employee relationship exists between the parties and no issue is involved which may be resolved by
San Beda College of Law 172
4S: 2015 - 2016
LABOR LAW REVIEW Atty. Joyrich Golangco

reference to the Labor Code, other labor statutes or any collective bargaining agreement, it is the Regional
Trial Court that has jurisdiction. In its complaint, private respondent is not seeking any relief under the Labor
Code but seeks payment of a sum of money and damages on account of petitioner's alleged breach of its
obligation under their Guard Service Contract. The action is within the realm of civil law hence jurisdiction over
the case belongs to the regular courts. While the resolution of the issue involves the application of labor laws,
reference to the labor code was only for the determination of the solidary liability of the petitioner to the
respondent where no employer-employee relation exists.

In the case at bar, even if petitioner filed the complaint on his and also on behalf of the security guards, the relief
sought has to do with the enforcement of the contract between him and the SSS which was deemed amended
by virtue of Wage Order No. NCR-03. The controversy subject of the case at bar is thus a civil dispute, the
proper forum for the resolution of which is the civil courts.
Even if the petition was filed with the proper forum, it must still be dismissed for lack of cause of action. Under
Art. 106 of the Labor Code: In the event that the contractor or subcontractor fails to pay the wage of his
employees in accordance with this Code, the employer shall be jointly and severally liable with his contractor or
subcontractor to such employees to the extent of the work performed under the contract, in the same manner
and extent that he is liable to employees directly employed by him.

It is only when [the] contractor pays the increases mandated that it can claim an adjustment from the principal to
cover the increases payable to the security guards. The conclusion that the right of the contractor (as
principal debtor) to recover from the principal (as solidary co-debtor) arises only if he has paid the
amounts for which both of them are jointly and severally liable is in line with Article 1217 of the Civil
Code.

In fine, the liability of the SSS to reimburse petitioner arises only if and when petitioner pays his employee-
security guards the increases mandated by Wage Order No. NCR-03.
ARTICLE 136 (now Art. 134) Stipulation Against Marriage

71. ZIALCITA, ET AL. v. PAL


RO4-3-398-76. February 20, 1977

FACTS:
Complainant Zialcita, an international flight stewardess of PAL, was discharged from the service on account of
her marriage. In separating Zialcita, PAL invoked its policy which stated that flight attendants must be single,
and shall be automatically separated from employment in the event they subsequently get married. They
claimed that this policy was in accordance with Article 132 of the Labor Code. On the other hand, Zialcita
questioned her termination on account of her marriage, invoking Article 136 of the same law.

ISSUE: Was Zialcita validly terminated on account of her marriage?

SC RULING:
NO. When Presidential Decree No. 148, otherwise known as the Women and Child Labor Law, was
promulgated in 13 March 1973, PALs policy had met its doom. However, since no one challenged its validity,
the said policy was able to obtain a momentary reprieve. Section 8 of PD148 is exactly the same provision
reproduced verbatim in Article 136 of the Labor Code, which was promulgated on 1 May 1974 and took effect
six months later. Although Article 132 enjoins the Secretary of Labor to establish standards that will ensure the
safety and health of women employees and in appropriate cases shall by regulation require employers
to determine appropriate minimum standards for termination in special occupations, such as those of flight
attendants, it is logical to presume that, in the absence of said standards or regulations which are yet to
be established, the policy of PAL against marriage is patently illegal.

Article 136 is not intended to apply only to women employed in ordinary occupations,
or it should have categorically expressed so. The sweepingintendment of the law, be it on special or ordinary
occupations, is reflected in the whole text and supported by Article 135 that speaks of non-discrimination on the
employment of women.

San Beda College of Law 173


4S: 2015 - 2016
LABOR LAW REVIEW Atty. Joyrich Golangco

San Beda College of Law 174


4S: 2015 - 2016
LABOR LAW REVIEW Atty. Joyrich Golangco

72. STAR PAPER CORPORATION v. SIMBOL
GR No. 164774 April 12, 2006
Puno, J.

ART. 136 OF THE LABOR CODE AND DISCRIMINATION.

DOCTRINE:
The questioned policy may not facially violate Article 136 of the Labor Code but it creates a disproportionate
effect and under the disparate impact theory, the only way it could pass judicial scrutiny is a showing that it
is reasonable despite the discriminatory, albeit disproportionate, effect. The failure of petitioners to prove a
legitimate business concern in imposing the questioned policy cannot prejudice the employees right to be free
from arbitrary discrimination based upon stereotypes of married persons working together in one company.

FACTS: Petitioner Star Paper Corporation promulgated a company policy which states:
1. New applicants will not be allowed to be hired if in case he/she has [a] relative, up to [the] 3rd degree of
relationship, already employed by the company.
2. In case of two of our employees (both singles [sic], one male and another female) developed a friendly
relationship during the course of their employment and then decided to get married, one of them should
resign to preserve the policy stated above.
Respondents Ronaldo Simbol, and Wilfreda Comia are employees of Star Paper who claim that they were
compelled to resign in view of an illegal company policy, after they married fellow co-workers. They filed a
complaint for unfair labor practice and constructive dismissal against Star Paper Corp.

Respondents submit that their dismissal violates the above provision. Petitioners allege that its policy "may
appear to be contrary to Article 136 of the Labor Code" but it assumes a new meaning if read together with the
first paragraph of the rule. The rule does not require the woman employee to resign. The employee spouses
have the right to choose who between them should resign. Further, they are free to marry persons other than
co-employees. Hence, it is not the marital status of the employee, per se, that is being discriminated. It is only
intended to carry out its no-employment-for-relatives-within-the-third-degree-policy which is within the ambit of
the prerogatives of management.

LA RULING: The LA dismissed the complaint ruling that the policy against marriage between employees was a
valid management prerogative. The ruling was affirmed by the NLRC.

CA RULING: The CA reversed the above rulings and held that the dismissal of the respondents was illegal and
ordering their reinstatement.

ISSUE: Whether or not the policy of Star Paper requiring the resignation of either spouse-employee is in
violation of Art. 136 of the Labor Code.

SC RULING:
Yes. The Supreme Court ruled that Petitioners sole contention that "the company did not just want to have two
(2) or more of its employees related between the third degree by affinity and/or consanguinity is lame. That the
second paragraph was meant to give teeth to the first paragraph of the questioned rule is evidently not the valid
reasonable business necessity required by the law.

It is significant to note that in the case at bar, respondents were hired after they were found fit for the job, but
were asked to resign when they married a co-employee. Petitioners failed to show how the marriage of Simbol,
then a Sheeting Machine Operator, to Alma Dayrit, then an employee of the Repacking Section, could be
detrimental to its business operations. Neither did petitioners explain how this detriment will happen in the case
of Wilfreda Comia, then a Production Helper in the Selecting Department, who married Howard Comia, then a
helper in the cutter-machine. The policy is premised on the mere fear that employees married to each other will
be less efficient. If we uphold the questioned rule without valid justification, the employer can create policies
based on an unproven presumption of a perceived danger at the expense of an employees right to security of
tenure.

Petitioners contend that their policy will apply only when one employee marries a co-employee, but they are free
to marry persons other than co-employees. The questioned policy may not facially violate Article 136 of the
Labor Code but it creates a disproportionate effect and under the disparate impact theory, the only way it could
San Beda College of Law 175
4S: 2015 - 2016
LABOR LAW REVIEW Atty. Joyrich Golangco

pass judicial scrutiny is a showing that it is reasonable despite the discriminatory, albeit disproportionate, effect.
The failure of petitioners to prove a legitimate business concern in imposing the questioned policy cannot
prejudice the employees right to be free from arbitrary discrimination based upon stereotypes of married
persons working together in one company.

San Beda College of Law 176


4S: 2015 - 2016
LABOR LAW REVIEW Atty. Joyrich Golangco

SEXUAL HARASSMENT

73. MA. LOURDES T. DOMINGO v. ROGELIO I. RAYALA


G.R. No. 155831 February 18, 2008
NACHURA, J.:

DOCTRINES:
1. It is not necessary that the demand, request or requirement of a sexual favor be articulated in a
categorical oral or written statement to be considered as sexual harassment.
2. The Chief Executive does not have unfettered discretion to impose a penalty other than the penalty
provided by law for sexual harassment.

FACTS:
Petitioner Ma. Lourdes T. Domingo, then Stenographic Reporter III at the NLRC, filed a Compliant for sexual
harassment against Respondent, NLRC Chairman Rogelio I. Rayala before the Secretary of Labor and
Employment. She claimed that the respondent committed the following acts:
1. Holding and squeezing her shoulders;
2. Running his fingers across her neck and tickling her ear;
3. Having inappropriate conversations wither her;
4. Giving her money allegedly for school expenses with a promise of future privileges; and
5. Making statements with unmistakable sexual overtones.

A committee was created to investigate the said allegations. The said committee found the respondent guilty of
the offense charged and recommended the imposition of the minimum penalty provided under AO 250, which it
erroneously stated as suspension for 6 months. However, the Secretary of Labor and employment
recommended that the penalty should be suspension for 6 months and 1 day, in accordance with AO 250. The
Office of the President, through Executive Secretary Zamora, concurred with the findings of the Committee but
imposed the penalty of dismissal. The respondent assailed the decision claiming his acts do not constitute
sexual harassment.

CA RULING: The CA held that there was sufficient evidence on record to create moral certainty that the
Respondent committed the acts he was charged with.

ISSUE:
1. Whether or not the Respondent is guilty of sexual harassment?
2. Whether or not the Office of the President may impose the penalty of dismissal?

SC RULING:
1. Yes. If we were to test Rayalas acts strictly by the standards set in Section 3, RA 7877, he would still
be administratively liable. It is true that this provision calls for a "demand, request or requirement of a
sexual favor." But it is not necessary that the demand, request or requirement of a sexual favor be
articulated in a categorical oral or written statement. It may be discerned, with equal certitude, from the
acts of the offender. Holding and squeezing Domingos shoulders, running his fingers across her neck
and tickling her ear, having inappropriate conversations with her, giving her money allegedly for school
expenses with a promise of future privileges, and making statements with unmistakable sexual
overtones all these acts of Rayala resound with deafening clarity the unspoken request for a sexual
favor.

Likewise, contrary to Rayalas claim, it is not essential that the demand, request or requirement be
made as a condition for continued employment or for promotion to a higher position. It is enough that
the respondents acts result in creating an intimidating, hostile or offensive environment for the
employee.45 That the acts of Rayala generated an intimidating and hostile environment for Domingo is
clearly shown by the common factual finding of the Investigating Committee, the OP and the CA that
Domingo reported the matter to an officemate and, after the last incident, filed for a leave of absence
and requested transfer to another unit.
2. No. Under AO 250, the penalty for the first offense is suspension for six (6) months and one (1) day to
one (1) year, while the penalty for the second offense is dismissal.52 On the other hand, Section 22(o),
Rule XVI of the Omnibus Rules Implementing Book V of the Administrative Code of 198753 and Section
San Beda College of Law 177
4S: 2015 - 2016
LABOR LAW REVIEW Atty. Joyrich Golangco

52 A(15) of the Revised Uniform Rules on Administrative Cases in the Civil Service54 both provide that
the first offense of disgraceful and immoral conduct is punishable by suspension of six (6) months and
one (1) day to one (1) year. A second offense is punishable by dismissal.

Under the Labor Code, the Chairman of the NLRC shall hold office during good behavior until he or she
reaches the age of sixty-five, unless sooner removed for cause as provided by law or becomes
incapacitated to discharge the duties of the office.55

In this case, it is the President of the Philippines, as the proper disciplining authority, who would
determine whether there is a valid cause for the removal of Rayala as NLRC Chairman. This power,
however, is qualified by the phrase "for cause as provided by law." Thus, when the President found that
Rayala was indeed guilty of disgraceful and immoral conduct, the Chief Executive did not have
unfettered discretion to impose a penalty other than the penalty provided by law for such offense. As
cited above, the imposable penalty for the first offense of either the administrative offense of sexual
harassment or for disgraceful and immoral conduct is suspension of six (6) months and one (1) day to
one (1) year. Accordingly, it was error for the Office of the President to impose upon Rayala the penalty
of dismissal from the service, a penalty, which can only be imposed upon commission of a second
offense.

San Beda College of Law 178


4S: 2015 - 2016
LABOR LAW REVIEW Atty. Joyrich Golangco

74. PHILIPPINE AEOLUS AUTOMOTIVE UNITED CORPORATION and/or FRANCIS CHUA v. NATIONAL
LABOR RELATIONS COMMISSION and ROSALINDA C. CORTEZ, respondents.
G.R. No. 124617 April 28, 2000
BELLOSILLO, J.:

SEXUAL HARASSMENT

DOCTRINE:
The gravamen of the offense in sexual harassment is not the violation of the employee's sexuality but the abuse
of power by the employer.
FACTS:
Petitioner Philippine Aeolus Automotive United Corporation (PAAUC) is a corporation duly organized and
existing under Philippine laws, petitioner, Francis Chua is its President while private respondent Rosalinda C.
Cortez was a company nurse of Petitioner Corporation until her termination.

PAAUC dismissed Private Respondent from service on the ground of serious misconduct, gross habitual neglect
and fraud or willful breach of trust. Among the acts she allegedly committed is throwing a stapler at Plant
Manager William Chua, her superior and uttering invectives against him. She filed with the Labor Arbiter a
th
complaint for illegal dismissal, non-payment of annual service incentive leave, 13 month pay and damages
against PAAUC and its President Francis Chua. She claimed as a defense for the offense charged against her
that William Chua manifested a special liking for her. She claimed that William Chua would oftentimes invite her
for a date, make sexual advances touching her hands, putting his arms around her shoulders, running his
finger on her arms and telling her she looked beautiful. The special treatment and sexual advances continued
during her employment for 4 years but she never reciprocated his flirtations, until finally, she noticed that his
attitude toward her changed. He made her understand that if she would not give in to his sexual advances he
would cause her termination from the service; and he made good his threat when he started harassing her.

LA RULING:
The Labor Arbiter rendered a decision holding the termination of Cortez as valid and legal, at the same time
dismissing her claim for damages for lack of merit.

NLRC RULING:
The NLRC disbelieved the explanation proffered by private respondent on the ground she never filed a
complaint against William Chua for more than 4 years.

ISSUE:
Whether or not William Chua committed acts of sexual harassment against Cortez?

SC RULING:
Yes. The gravamen of the offense in sexual harassment is not the violation of the employee's sexuality but the
abuse of power by the employer. Any employee, male or female, may rightfully cry "foul" provided the claim is
well substantiated. Strictly speaking, there is no time period within which he or she is expected to complain
through the proper channels. The time to do so may vary depending upon the needs, circumstances, and more
importantly, the emotional threshold of the employee.
Private respondent admittedly allowed four (4) years to pass before finally coming out with her employer's
sexual impositions. Not many women, especially in this country, are made of the stuff that can endure the agony
and trauma of a public, even corporate, scandal. If petitioner corporation had not issued the third memorandum
that terminated the services of private respondent, we could only speculate how much longer she would keep
her silence. Moreover, few persons are privileged indeed to transfer from one employer to another. The dearth
of quality employment has become a daily "monster" roaming the streets that one may not be expected to give
up one's employment easily but to hang on to it, so to speak, by all tolerable means. Perhaps, to private
respondent's mind, for as long as she could outwit her employer's ploys she would continue on her job and
consider them as mere occupational hazards. This uneasiness in her place of work thrived in an atmosphere of
tolerance for four (4) years, and one could only imagine the prevailing anxiety and resentment, if not bitterness,
that beset her all that time. But William Chua faced reality soon enough. Since he had no place in private
respondent's heart, so must she have no place in his office. So, he provoked her, harassed her, and finally
dislodged her; and for finally venting her pent-up anger for years, he "found" the perfect reason to terminate her.

San Beda College of Law 179


4S: 2015 - 2016
LABOR LAW REVIEW Atty. Joyrich Golangco

ARTICLE 141 (now Art. 139) - Coverage

75. APEX MINING COMPANY, INC. v. NATIONAL LABOR RELATIONS COMMISSION and SINCLITICA
CANDIDO
G.R. No. 94951 April 22, 1991
GANCAYCO, J.:

HOUSEHELPER OR DOMESTIC SERVANT

DOCTRINE:
The definition provided by the Labor Code of the terms househelper or domestic servant cannot be
interpreted to include househelp or laundrywomen working in staffhouses of a company, like petitioner who
attends to the needs of the company's guest and other persons availing of said facilities

FACTS:
Private respondent Sinclita Candida was employed by petitioner Apex Mining Company, Inc. to perform laundry
services at its staff house. In the beginning, she was paid on a piece rate basis. However, she was then paid on
a monthly basis at Php 250.00 a month, which was ultimately increased, to Php 575.00 a month. On 18
December 1987, while she was attending to her assigned task and she was hanging her laundry, she
accidentally slipped and hit her back on a stone. As a result of which, she was not able to continue her work.
She was offered the amount of Php 2,000.00, which was eventually increased to Php 5,000.00, to persuade her
to quit her job, but she refused. The petitioner subsequently disallowed her to return to work. She filed a request
for assistance with the DOLE.

LA RULING:
th
The Labor Arbiter ordered the petitioner to pay her the following: 1) salary; 2) Emergency Living; 3) 13 Month
Pay; and 4) Separation Pay. The petitioner appealed the decision to the NLRC.

NLRC RULING:
The NLRC dismissed the appeal for lack of merit and affirmed the appealed decision. A motion for
reconsideration thereof was likewise denied.

ISSUE: Whether or not private responded should be treated as a mere househelper or domestic servant and not
as a regular employee entitled to the amounts granted by the Labor Arbiter?

SC RULING:
No. Under Rule XIII, Section l(b), Book 3 of the Labor Code, as amended, the terms "househelper" or "domestic
servant" are defined as follows:

The term "househelper" as used herein is synonymous to the term "domestic servant" and shall refer to
any person, whether male or female, who renders services in and about the employer's home and which
services are usually necessary or desirable for the maintenance and enjoyment thereof, and ministers
exclusively to the personal comfort and enjoyment of the employer's family.

The definition cannot be interpreted to include househelp or laundrywomen working in staffhouses of a


company, like petitioner who attends to the needs of the company's guest and other persons availing of said
facilities. By the same token, it cannot be considered to extend to then driver, houseboy, or gardener exclusively
working in the company, the staffhouses and its premises. They may not be considered as within the meaning of
a "househelper" or "domestic servant" as above-defined by law.

The criteria is the personal comfort and enjoyment of the family of the employer in the home of said employer.
While it may be true that the nature of the work of a househelper, domestic servant or laundrywoman in a home
or in a company staffhouse may be similar in nature, the difference in their circumstances is that in the former
instance they are actually serving the family while in the latter case, whether it is a corporation or a single
proprietorship engaged in business or industry or any other agricultural or similar pursuit, service is being
rendered in the staffhouses or within the premises of the business of the employer. In such instance, they are
employees of the company or employer in the business concerned entitled to the privileges of a regular
employee.

San Beda College of Law 180


4S: 2015 - 2016
LABOR LAW REVIEW Atty. Joyrich Golangco

BOOK IV Healt, Safety and Social Welfare Benefits

76. GOVERNMENT SERVICE INSURANCE SYSTEM (GSIS) v. COURT OF APPEALS and HEIRS OF
ABRAHAM CATE, represented by DOROTHY CATE
G.R. No. 124208 January 28, 2008
AZCUNA, J.:

LIBERAL CONSTRUCTION OF EMPLOYEES COMPENSATION ACT

DOCTRINE:
If in the present state of science, the proof referred by law to be present by the deceased claimant was
unavailable and impossible to comply with, the condition must be deemed as not imposed.

FACTS:
On March 6, 1974, Abraham Cate joined the military service as a Rifleman of the Philippine Navy. In 1975, he
was designated as Action Clerk. On Feburary 22, 1986, he was transferred to the now defunct Philippine
Constabulary with the rank of Technical Sergeant and was later promoted to Master Sergeant. On January 2,
1991, he was absorbed in the Philippine National Police with the rank of Senior Police Officer IV.

He was diagnosed of having Osteoblastic Osteosarcoma in his left cheek. He underwent Total Maxillectomy
with Orbital Exenteration in the PGH. However, his disease recurred and he underwent debulking of the recent
tumor at PGH. The post-operative course was uneventful and he underwent radiotherapy. He was then
compulsorily retired from the PNP. He filed a claim for income benefits with the GSIS under PD No. 626 as
amended. However, his claim was denied by the GSIS on the ground Osteosarcoma is not considered an
occupational disease. After his death, his wife and 2 children appealed the decision of the GSIS to the ECC.

ECC Ruling: The ECC affirmed the decision of the GSIS and dismissed the case for lack of merit.

CA RULING: The CA reversed and set aside the decision of the ECC on the ground the Employees
Compensation Act should be liberally construed in favor of applicant.

ISSUE:
Whether or not the ailment of the late Abraham is compensable under the present law on employees
compensation?

SC RULING:
Yes. The present law on compensation allows certain diseases to be compensable if it is sufficiently proven that
the risk of contract it is increased by the working conditions. The application of the rules would mean that absent
any proof that the risk of contracting the ailment was increased by the working conditions of the late Abraham,
private respondents would not be entitled to compensation. Considering, however, that it is practically
undisputed that under the present state of science, the proof referred by the law to be presented by the
deceased private respondent claimant was unavailable and impossible to comply with, the condition must be
deemed as not imposed.
Petitioners failure to present positive evidence of a causal relation of the illness and his working conditions is
due to the pure and simple lack of available proof to be offered in evidence. Verily, to deny compensation to
osteosarcoma victims who will definitely be unable to produce a single piece of proof to that effect is unrealistic,
illogical and unfair. At the very least, on a very exceptional circumstance, the rule on compensability should be
relaxed and be allowed to apply to such situations. To disallow the benefit will even more add up to the
sufferings, this time, for the ignorance of the inability of mankind to discover the real truth about cancer.

San Beda College of Law 181


4S: 2015 - 2016
LABOR LAW REVIEW Atty. Joyrich Golangco

77. DOMINGA A. SALMONE v. COMPENSATION COMMISSION and SOCIAL SECURITY SYSTEM
G.R. No. 142392 September 26, 2000
PARDO, J.:

DEGREE OF PROOF REQUIRED UNDER PD NO. 626

DOCTRINE:
The claimant must show, at least, by substantial evidence that the development of the disease is brought largely
by the conditions present in the nature of the job. What the law requires is a reasonable work-connection and
not a direct causal relation.

FACTS:
The Paul Geneve Entertainment Corporation employed petitioner Dominga A. Salmone as a sewer. She was
later promoted as the officer-in-charge and the over-all custodian in the Sewing Department. However, she
started to feel chest pains, which forced her to file a leave of absence from work because they have become
unbearable. Upon medical examination, she was diagnosed with Atherosclerotic heart disease, Atrial Fibrillation,
Cardiac Arrhythmia. Upon recommendation of her doctor, she resigned from her work hoping that with a much-
need complete rest, she will be cured. She then filed a disability claim with the SSS from the Employees
compensation fund. However, the SSS denied her claim including her motion for reconsideration. Thus she
appealed the said decision to the ECC.

ECC Ruling: The ECC dismissed her appeal for want of merit.

CA RULING: The CA dismissed the petition on the ground that the petitioners illness was not compensable
because petitioner failed to adduce substantial evidence proving any of the condition of compensability.

ISSUE: Whether or not the petitioners illness is compensable?

SC RULING:
Yes. Under the Labor Code, as amended, the law applicable to the case at bar, in order for the employee to be
entitled to sickness or death benefits, the sickness or death resulting therefrom must be or must have resulted
from either (a) any illness definitely accepted as an occupational disease listed by the Commission, or (b) any
illness caused by employment, subject to proof that the risk of contracting the same is increased by working
conditions.
In this case, petitioner has shown by uncontroverted evidence that in the course of her employment, due to work
related stress, she suffered from severe chest pains which caused her to take a rest, per physician's advice, and
ultimately to resign from her employment. She was diagnosed as suffering from "atherosclerotic heart disease,
atrial fibrillation, cardiac arrhythmia" which, as heretofore stated, is included within the term cardiovascular
diseases.
Indisputably, cardiovascular diseases, which, as herein above-stated include atherosclerotic heart disease, atrial
fibrillation, cardiac arrhythmia, are listed as compensable occupational diseases in the Rules of the Employees'
Compensation Commission, hence, no further proof of casual relation between the disease and claimant's work
is necessary.
The degree of proof required under P. D. No. 626, is merely substantial evidence, which means, "such relevant
evidence as a reasonable mind might accept as adequate to support a conclusion." The claimant must show, at
least, by substantial evidence that the development of the disease is brought largely by the conditions present in
the nature of the job. What the law requires is a reasonable work-connection and not a direct causal relation. It
is enough that the hypothesis on which the workmen's claim is based is probable. Medical opinion to the
contrary can be disregarded especially where there is some basis in the facts for inferring a work- connection.
Probability, not certainty, is the touchtone.

San Beda College of Law 182


4S: 2015 - 2016
LABOR LAW REVIEW Atty. Joyrich Golangco

78. HEIRS OF DEAUNA v. FIL STAR MARITIME CORPORATION
G.R. No. 191563 June 20, 2012
REYES, J.:

THE DEATH OF A SEAFARER IS COMPENSABLE WHEN IT OCCURS WHILE STILL IN THE EMPLOYENT
OF THE EMPLOYER.

DOCTRINE:
Article 22.1(b) considers an employment as terminated if a seafarer signs off from the vessel due to sickness,
but subject to the provisions of Article 29.

Article 29.1 of the IBF/AMOSUP/IMMAJ CBA provides that the death of a seafarer, for any cause, is
compensable when it occurs while he is in the employment of the company. Article 29.4, on the other hand,
clarifies that the seafarer shall be considered as in the employment of the company for so long as the provisions
of Articles 25 and 26 apply and provided the death is directly attributable to sickness or injury that caused the
seafarer's employment to be terminated in accordance with Article 22.1(b).

Under Article 25.3, a seafarer repatriated to the port of his engagement, unfit as a result of sickness, shall be
entitled to medical attention at the company's expense for up to a maximum period of 130 days after
repatriation, subject to the submission of satisfactory medical reports. Article 26.2 further states that a seafarer
shall likewise be entitled to sick pay at the rate equivalent to his basic wage while he remains sick up to a
maximum of 130 days. Article 26.4 allows continued entitlement to sick pay beyond the 130 day period,
reckoned from repatriation, provided satisfactory medical reports shall be submitted and endorsed where
necessary, by a company-appointed doctor.

FACTS:
Edwin boarded on August 1, 2004 for a nine-month engagement as Chief Engineer of the Sanko. He suffered
from abdominal pains and was found to have kidney stones for which he was given medication. Edwin was then
repatriated. Respondents claimed that Edwin requested for an early termination while petitioners averred that
Edwin was repatriated due to the latter's body weakness and head heaviness. Edwin was discovered to
have Glioblastoma WHO Grade 4 (GBM) . It was then noted that Edwin could have acquired the cancer as a
result of radiation or vinyl products, or had worked in the vicinity of power lines.

Respondent claimed that out of compassion and intent to avoid legal battles, they extended to Edwin an
allowance of US$6,033.36. They also offered the payment of US$60,000.00 disability benefits despite having no
obligation to do so on their part as GBM can only be considered as work-related if a person who suffers
therefrom had exposures to radiation or vinyl products, or had worked in the vicinity of power lines. The
respondents claimed that Edwin did not have such exposure while under their employ. Petitioners then asked
for disability benefits, but were denied by respondents. They then filed a complaint for disability benefits,
medical and transportation reimbursements, moral and exemplary damages and attorney's fees were filed
before the National Labor Relations Commission (NLRC). Edwin died on April 13, 2006 during the pendency of
the proceedings. He was substituted therein by the petitioners who sought the payment of death benefits under
the International Bargaining Forum/Associated Marine Officers and Seamens Union of the
Philippines/International Mariners Management Association of Japan Collective Bargaining Agreement
(IBF/AMOSUP/IMMAJ CBA).

Voluntary Arbitrator Rene Ofreneo (VA Ofreneo), invoking the provisions of the Philippine Overseas
Employment Administration Standard Employment Contract (POEA SEC) and the IBF/AMOSUP/IMMAJ CBA,
awarded death benefits to the petitioners. The Court of Appeals reversed the decision of VA Ofreneo.
Petitioners contend that they are entitled to death benefits.

ISSUE: Whether or not within the purview of the IBF/AMOSUP/IMMAJ CBA, Edwin's death on April 13, 2006, or
more than a year from his repatriation, can be considered as one occurring while he was still in the employment
of the respondents.

SC RULING:
YES. Edwin's death can be considered can be considered as one occuring while he was still in the employment
of respondents. Under the IBF/AMOSUP/IMMAJ CBA provisions, Edwin's death a little more than a year from
his repatriation can still be considered as one occurring while he was still under the respondents' employ.
San Beda College of Law 183
4S: 2015 - 2016
LABOR LAW REVIEW Atty. Joyrich Golangco

From the foregoing, the SC concluded that at the time of Edwin's death on April 13, 2006 due to GBM, he was
still in the employment of the respondents. While it is true that Article 22.1 of the IBF/AMOSUP/IMMAJ CBA
considers a seafarer as terminated when he signs off from the vessel due to sickness, the foregoing is subject to
the provisions of Article 29. Under Article 29, a seafarer remains under the respondents' employ as long as the
former is still entitled to medical assistance and sick pay, and provided that the death which eventually occurs is
directly attributable to the sickness which caused the seafarer's employment to be terminated. As discussed
above, the company-designated physician, Dr. Cruz, in effect admitted that Edwin was repatriated due to
symptoms which a person suffering from GBM normally exhibits. The petitioners are, however, not entitled to
moral and exemplary damages and attorney's fees.

79. DEBAUDIN v. SSS


G.R. No. 148308 September 21, 2007
AZCUNA, J.:

FOR A NON-OCCUPATIONAL DISEASE(those not included in the list of occupational disease


enumerated under Annex "A" of P.D. 626) TO BE COMPENSABLE, THE CLAIMANT MUST
SUBSTANTIATE HIS CLAIM WITH EVIDENCE THAT HIS EMPLOYMENT OR WORKING CONDITIONS
CONTRIBUTED IN CONTRACTING THE AILMENT

DOCTRINE:
An employee is entitled to compensation benefits if the sickness is a result of an occupational disease listed
under the Rules on Employees' Compensation; or in case of any other illness, if it is caused by employment,
subject to proof that the risk of contracting the same is increased by the working conditions. This is as it should
be because for an illness to be compensable, it must be (1) directly caused by such employment; (2) aggravated
by the employment; or (3) the result of the nature of such employment. Jurisprudence provides that to establish
compensability of a non-occupational disease, reasonable proof of work-connection and not direct causal
relation is required.

FACTS:
Petitioner, Roberto Debaudin, is a seaman by profession as a utility staff who performed cleaning chemical-spill-
oil on deck, slat dislodging, and spraying naphtha chemical and washing dirt and rusts inside the tank. 18 years
after, Debaudin sought medical assistance after he experienced episodes of bilateral blurring of vision and was
later diagnosed of chronic open angle glaucoma.

On account of his ailment, petitioner filed before the SSS a claim for compensation benefits under P.D. No. 626
claiming that the strenuous tasks required climbing, bending over and running for so many times acts which a
medical book considered as contributory factors that would increase intraocular pressure which causes
glaucoma. He also adds that he was also subjected to emotional strains of going through the perils of the sea
and homesickness for being away from his family during the entire duration of the contracts. He, thus, alleges
that his employment as a seaman contributed, even in a small degree, to the development of his ailment.
His claim, however, was denied by SSS on the ground that there is no causal relationship between the illness
and his job as a seaman.

EMPLOYEES COMPENSTAION COMMISSION (EEC): EEC denied Debaudins motion for reconsideration
holding that Debaudins Chronic Open Angle Glaucoma is not an occupational disease under the law. Thus, he
is required to show by substantial evidence that the nature of his job as a Seaman had increased the risk of
contracting the disease. However, appellant failed to discharge the burden of proof required by the law.

CA RULING: The CA dismissed the case on the ground that petitioner failed to adduce substantial evidence
supporting the conclusion that the working conditions as a seaman increased the risk of contracting his chronic
open angle glaucoma.

ISSUE: Whether or not the work of Debaudin as a seaman contributed even in a small degree in or had
increased the risk of contracting his chronic open angle glaucoma.

SC RULING:
NO. In the present case, petitioners chronic open angle glaucoma is not listed as an occupational disease;
hence, he has the burden of proving by substantial evidence, or such relevant evidence which a reasonable

San Beda College of Law 184


4S: 2015 - 2016
LABOR LAW REVIEW Atty. Joyrich Golangco

mind might accept as adequate to justify a conclusion, that the nature of his employment or working conditions
increased the risk of contracting the ailment or that its progression or aggravation was brought about thereby.

It is enough that the hypothesis on which the workmen's claim is based is probable. Probability, not the ultimate
degree of certainty, is the test of proof in compensation proceedings since in carrying out and interpreting the
provisions of the Labor Code and its implementing rules and regulations the primordial and paramount
consideration is the employees' welfare.

Other than positing petitioners allegations, petitioner presented no competent medical history, records or
physicians report to objectively substantiate the claim that there is a reasonable nexus between his work and
his ailment. Without saying more, his bare allegations do not ipso facto make his illness compensable. Awards
of compensation cannot rest on speculations or presumptions. The claimant must present concrete evidence to
prove a positive proposition.

San Beda College of Law 185


4S: 2015 - 2016
LABOR LAW REVIEW Atty. Joyrich Golangco

80. AUSTRIA v. CA and EMPLOYEES COMPENSATION COMMISSION
G.R. No. 146636 August 12, 2002
PUNO, J.:

THE DISABILITY IS TOTAL AND PERMANENT IF AS A RESULT OF THE INJURY OR SICKNESS, THE
EMPLOYEE IS UNABLE TO PERFORM ANY GAINFUL OCCUPATION FOR A CONTINUOUS PERIOD
EXCEEDING 120 DAYS;

DOCTRINE:
The disability is total and permanent if as a result of the injury or sickness, the employee is unable to perform
any gainful occupation for a continuous period exceeding 120 days; and It is partial and permanent if as a result
of the injury or sickness, the employee suffers a permanent partial loss of the use of any part of his body.

FACTS:
Petitioner Pablo A. Austria was employed as bag piler at Central Azucarera de Tarlac from June 1, 1977 to July
20, 1997. In 1994, petitioner began to feel severe back pain. In 1998, it was revealed that he was suffering from
osteoarthritis of the lumbar spine. Thus petitioner filed with the SSS a claim for compensation benefits under PD
626 as amended. The claim was granted and petitioner was awarded permanent partial disability benefits.
Petitioner thereafter requested the SSS for conversion of his permanent partial disability benefit to permanent
total disability benefit. The SSS denied the request

ECC RULING: On appeal, the ECC affirmed the decision of the SSS. The ECC held that considering the degree
of his disability at the time he was separated from the service, petitioner has already availed of the maximum
benefits to which he is entitled on account of his osteoarthritis.

CA RULING: The appellate court dismissed the petition, ruling that the law does not allow the conversion of
permanent partial disability to permanent total disability

ISSUE: Whether or not the Honorable Court of Appeals erred in denying the claim for additional benefits in favor
of the petitioner and not allowing the conversion of his (petitioner) permanent partial disability to permanent total
disability

SC RULING:
YES. The test of whether or not an employee suffers from permanent total disability is a showing of the capacity
of the employee to continue performing his work notwithstanding the disability he incurred. Thus, if by reason of
the injury or sickness he sustained, the employee is unable to perform his customary job for more than 120 days
and he does not come within the coverage of Rule X of the Amended Rules on Employees Compensability
(which, in more detailed manner, describes what constitutes temporary total disability), then the said employee
undoubtedly suffers from permanent total disability regardless of whether or not he loses the use of any part of
his body.

PD 626 as amended provides three types of disability benefits to qualified employees: (1) temporary total
disability, (2) permanent total disability, and (3) permanent partial disability. In the case at bar, petitioner was
granted by the SSS, as affirmed by the ECC, permanent partial disability benefit, but he seeks to avail of
permanent total disability benefit. Under Section 2 Rule VII of the Amended Rules on Employees
Compensation, a disability is total and permanent if as a result of the injury or sickness, the employee is unable
to perform any gainful occupation for a continuous period exceeding 120 days; and a disability is partial and
permanent if as a result of the injury or sickness, the employee suffers a permanent partial loss of the use of any
part of his body.

Total disability does not require that the employee be absolutely disabled, or totally paralyzed. What is
necessary is that the injury must be such that she cannot pursue her usual work and earn therefrom. Applying
the foregoing standards, we find petitioner entitled to permanent total disability benefit under the law. Petitioner
has been employed as bag piler for twenty (20) years at the Central Azucarera de Tarlac. His duties require him
to carry heavy loads of refined sugar and to perform other manual work. Since his work obviously taxes so much
on his back, his illness which affects his lumbar spine renders him incapable of doing his usual work as bag
piler. Hence, his disability to perform his regular duties may be considered total and permanent.

San Beda College of Law 186


4S: 2015 - 2016
LABOR LAW REVIEW Atty. Joyrich Golangco

81. GATUS v. SSS


G.R. No. 174725 January 26, 2011
LEONARDO-DE CASTRO, J.:

A CLAIMANT MUST SHOW, AT LEAST BY SUBSTANTIAL EVIDENCE, THAT THE DEVELOPMENT OF


THE DISEASE WAS BROUGHT ABOUT LARGELY BY THE CONDITIONS PRESENT IN THE NATURE OF
THE JOB.

DOCTRINE:
His disease not being listed as an occupational disease, he was expected to show that the illness or the fatal
disease was caused by his employment and the risk of contracting the disease was increased or aggravated by
the working conditions. His proof would constitute a reasonable basis for arriving at a conclusion that the
conditions of his employment had caused the disease or that such working conditions had aggravated the risk of
contracting the illness or the fatal disease.

FACTS:
Gatus worked at the Central Azucarera de Tarlac beginning 1972. He was a covered member of the SSS and
was certified as being fit to work before employment. He optionally retired from Central Azucarera de Tarlac
upon reaching 30 years of service at the age of 62. He was diagnosed to be suffering from Coronary Artery
Disease (CAD): Triple Vessel and Unstable Angina in 1995. His medical records showed him to be hypertensive
for 10 years and a smoker. Thus he was given by the SSS EC/SSS Permanent Partial Disability (PPD) benefits.
In 2003, an SSS audit revealed the need to recover the EC benefits already paid to him on the ground that his
CAD, being attributed to his chronic smoking, was not work-related. He elevated the matter to the ECC, which
denied his appeal on December 10, 2004, essentially ruling that although his CAD was a cardiovascular disease
listed as an occupational disease under Annex A of the Implementing Rules on Employees Compensation,
nothing on record established the presence of the qualifying circumstances for responsibility; that it was
incumbent upon him to prove that the nature of his previous employment and the conditions prevailing therein
had increased the risk of contracting his CAD; and that he had failed to prove this requisite. Hence, this
recourse, wherein he contends that he had contracted the disease due to the presence of harmful fuel smoke
emission of methane gas from a nearby biological waste digester and a railway terminal.

CA RULING: CA affirmed the ruling of SSS ruling that petitioner failed to submit substantial evidence that might
have shown that he was entitled to the benefits he had applied for.

ISSUE: Whether the Court of Appeals committed grave abuse of discretion in affirming the finding of the ECC
that petitioners ailment is not compensable under Presidential Decree No. 626, as amended

SC RULING: The court held NO. Gatus was diagnosed to have suffered from CAD; Triple Vessel and Unstable
Angina, diseases or conditions falling under the category of Cardiovascular Diseases which are not considered
occupational diseases under the Amended Rules on Employees Compensation. His disease not being listed as
an occupational disease, he was expected to show that the illness or the fatal disease was caused by his
employment and the risk of contracting the disease was increased or aggravated by the working conditions. His
proof would constitute a reasonable basis for arriving at a conclusion that the conditions of his employment had
caused the disease or that such working conditions had aggravated the risk of contracting the illness or the fatal
disease. While he might have been exposed to various smoke emissions at work for 30 years, he did not submit
satisfactory evidence proving that the exposure had contributed to the development of his disease or had
increased the risk of contracting the illness. Neither did he show that the disease had progressed due to
conditions in his job as a factory worker. In fact, he did not present any physicians report in order to substantiate
his allegation that the working conditions had increased the risk of acquiring the cardiovascular disease.
San Beda College of Law 187
4S: 2015 - 2016
LABOR LAW REVIEW Atty. Joyrich Golangco

San Beda College of Law 188


4S: 2015 - 2016
LABOR LAW REVIEW Atty. Joyrich Golangco

82. REPUBLIC OF THE PHILIPPINES v. MARIANO
G.R. No. 139455 March 28, 2003
QUISUMBING, J.:

WHERE IT WAS ESTABLISHED THAT THE CLAIMANTS AILMENT OCCURRED DURING AND IN THE
COURSE OF HIS EMPLOYMENT, IT MUST BE PRESUMED THAT THE NATURE OF THE CLAIMANTS
EMPLOYMENT IS THE CAUSE OF THE DISEASE

DOCTRINE:
For the sickness to be compensable, the same must be an occupational disease included in the list provided,
with the conditions set therein satisfied; otherwise, the claimant must show proof that the risk of contracting it is
increased by the working conditions.

FACTS:
For an eleven-year period, respondent Pedro Mariano was an employee of LGP Printing Press. During his
employment, Mariano worked in various capacities, including that of a machine operator, paper cutter,
monotype composer, film developer, and supervisor of the printing press. Sometime in February 1994, Marianos
service abruptly ended when he could no longer perform any work due to a heart ailment. An electrocardiograph
test revealed that he was suffering from Incomplete Right Bundle Branch Block. Respondent had consulted Dr.
Rogelio Mariano, whose diagnosis showed he was suffering from Parkinsons disease and hypertension.
Mariano filed a claim for employees compensation benefit with the SSS. In its medical evaluation dated April 15,
1997, SSS denied his claim on the ground that there was no causal connection between his ailment and his job
as film developer. The ECC ultimately dismissed the case on the ground that the claimant failed to establish a
causal connection between Parkinsons disease and the conditions of the printing press.

CA RULING: The Court of Appeals found that the nature of petitioners work at LGP resulted in his exposure to
various toxic chemicals, which is a possible cause of Parkinsons Disease. As to his hypertension, the appellate
court ruled that the respondents duties as machine operator and paper cutter involved physical pressure and
restlessness, since he was required to meet urgent deadlines for rush print orders. This in turn caused
respondent to suffer from stress and anxiety. In sum, the appellate court held that respondent had substantially
established the connection between the cause of his ailments and the nature of his work.

ISSUE: Whether or not Mariano was able to prove that his employment had a causal relation that with his
ailments: Parkinson's and Hypertension.

SC RULING:
YES. Workmens Compensation cases are governed by the law in force at the time the claimant contracted his
illness. In the instant case, the applicable rule is Section 1 (b), Rule III, of the Rules Implementing P.D. No. 626.
Under said Rule, for the sickness to be compensable, the same must be an occupational disease included in the
list provided, with the conditions set therein satisfied; otherwise, the claimant must show proof that the risk of
contracting it is increased by the working conditions.

As to Parkinsons disease, while it is true that this disease is not included in the list of compensable diseases
under the law then prevailing, it was found by the Court of Appeals that the conditions prevailing at LGP largely
led to the progression of the ailment. The respondents functions entailed constant exposure to hazardous or
toxic chemicals such as carbon disulfate, carbon monoxide, or manganese. As the ECC itself admitted in its
judgment, the exposure to these toxic substances is among the possible causes of this disease. Where it was
established that the claimants ailment occurred during and in the course of his employment, it must be
presumed that the nature of the claimants employment is the cause of the disease.

Second, even if we were to assume that Parkinsons Disease is not compensable, there can be no question that
Essential Hypertension is a compensable illness, following our ruling in Government Service Insurance System
v. Gabriel, that hypertension and heart ailments are compensable illnesses.

In upholding respondent Marianos claim, the Court of Appeals found that among the various jobs the
respondent performed were those of a machine operator, paper cutter, monotype composer, and later as
supervisor, most of which are physical and stressful in character. In established cases of Essential
Hypertension, the blood pressure fluctuates widely in response to emotional stress and physical activity. Given
the nature of his assigned job and the printing business, with its tight deadlines entailing large amounts of rush
San Beda College of Law 189
4S: 2015 - 2016
LABOR LAW REVIEW Atty. Joyrich Golangco

work, indeed the emotional and physical stress of respondents work at the printing press caused, and then
exacerbated, his hypertension. On this score, we hold that the Court of Appeals did not err in liberally construing
the rules implementing P.D. No. 626. In matters of labor and social legislation, it is well established that doubts
in the interpretation and application of the law are resolved liberally in favor of the worker and strictly against the
employer.

San Beda College of Law 190


4S: 2015 - 2016
LABOR LAW REVIEW Atty. Joyrich Golangco

83. MAGSAYSAY MARITIME CORPORATION and/or WASTFEL-LARSEN MANAGEMENT A/S v. OBERTO
LOBUSTA
G.R. No. 177578 January 25, 2012
VILLARAMA, JR., J.

TEMPORARY TOTAL DISABILITY

DOCTRINE:
A temporary total disability only becomes permanent when so declared by the company physician within the
periods he is allowed to do so, or upon the expiration of the maximum 240-day medical treatment period without
a declaration of either fitness to work or the existence of a permanent disability.

FACTS:
Petitioner Magsaysay Maritime Corporation is a domestic corporation and the local manning agent of the vessel
MV "Fossanger" and of petitioner Wastfel-Larsen Management A/S. Respondent Oberto S. Lobusta is a
seaman who has worked for Magsaysay Maritime Corporation. Lobusta boarded MV "Fossanger" on March 16,
1998. After two months, he complained of breathing difficulty and back pain. On May 12, 1998, while the vessel
was in Singapore, Lobusta was admitted at Gleneagles Maritime Medical Center and was diagnosed to be
suffering from severe acute bronchial asthma with secondary infection and lumbosacral muscle strain. Dr. C K
Lee certified that Lobusta was fit for discharge on May 21, 1998, for repatriation for further treatment. Upon
repatriation, Lobusta was referred to Metropolitan Hospital. The medical coordinator, Dr. Robert Lim, reported
that Lobusta has been diagnosed to have a moderate obstructive pulmonary disease which tends to be a
chronic problem, such that Lobusta needs to be on medications indefinitely. Petitioners "then faced the need for
confirmation and grading by a second opinion" and "it took the parties time to agree on a common doctor, until
they agreed on Dr. Camilo Roa." According to Dr. Roa, Lobusta is not physically fit to resume his normal work
as a seaman due to the persistence of his symptoms. Magsaysay Maritime Corporation suggested that Lobusta
be examined by another company-designated doctor for an independent medical examination. Dr. David opined
that Mr. Lobusta ought not to be considered fit to return to work as an Able Seaman. As no settlement was
reached despite the above findings, the Labor Arbiter ordered the parties to file their respective position papers.

LA RULING: ordered petitioners to pay Lobusta (a) US$2,060 as medical allowance, (b) US$20,154 as
disability benefits, and (c) 5% of the awards as attorneys fees. The Labor Arbiter held that provisions of the
Labor Code, as amended, on permanent total disability do not apply to overseas seafarers.

NLRC RULING: Lobusta appealed. The NLRC dismissed his appeal and affirmed the Labor Arbiters decision.
The NLRC ruled that Lobustas condition may only be considered permanent partial disability.

CA RULING: The CA ruled that Lobusta's disability brought about by his bronchial asthma is permanent and
total as he had been unable to work since May 14, 1998 up to the present or for more than 120 days, and
because Dr. David found him not fit to return to work as an able seaman.

ISSUE: Does the poea contract considers the mere lapse of more than one hundred twenty (120) days as total
and permanent disability?

SC RULING:
No. A temporary total disability only becomes permanent when so declared by the company physician within the
periods he is allowed to do so, or upon the expiration of the maximum 240-day medical treatment period without
a declaration of either fitness to work or the existence of a permanent disability.

Upon sign-off from the vessel for medical treatment, the seafarer is entitled to sickness allowance equivalent to
his basic wage until he is declared fit to work or the degree of permanent disability has been assessed by the
company-designated physician[,] but in no case shall this period exceed one hundred twenty (120) days.

Upon repatriation, Lobusta was first examined by the Pulmonologist and Orthopedic Surgeon on May 22, 1998.
The maximum 240-day (8-month) medical-treatment period expired, but no declaration was made that Lobusta
is fit to work. Nor was there a declaration of the existence of Lobustas permanent disability. On February 16,
1999, Lobusta was still prescribed medications for his lumbosacral pain and was advised to return for
reevaluation. May 22, 1998 to February 16, 1999 is 264 days or 6 days short of 9 months.

San Beda College of Law 191


4S: 2015 - 2016
LABOR LAW REVIEW Atty. Joyrich Golangco

Dr. Roas clinical summary also shows that as of December 16, 1999, Lobusta was still unfit to resume his
normal work as a seaman due to the persistence of his symptoms. But neither did Dr. Roa declare the existence
of Lobustas permanent disability. Again, the maximum 240-day medical treatment period had already expired.
May 22, 1998 to December 16, 1999 is 19 months or 570 days. In Remigio, unfitness to work for 11-13 months
was considered permanent total disability. So it must be in this case. And Dr. Davids much later report that
Lobusta "ought not to be considered fit to return to work as an Able Seaman" validates that his disability is
permanent and total as provided under the POEA Standard Employment Contract and the Labor Code, as
amended.

In fact, the CA has found that Lobusta was not able to work again as a seaman and that his disability is
permanent "as he has been unable to work since 14 May 1998 to the present or for more than 120 days." This
period is more than eight years, counted until the CA decided the case in August 2006. On the CA ruling that
Lobustas disability is permanent since he was unable to work "for more than 120 days," SC have clarified in
Vergara that this "temporary total disability period may be extended up to a maximum of 240 days."

San Beda College of Law 192


4S: 2015 - 2016
LABOR LAW REVIEW Atty. Joyrich Golangco

84. MAGSAYSAY MITSUI OSK MARINE, INC. and/or MOL TANKSHIP MANAGEMENT (ASIA) PTE LTD. v.
JUANITO G. BENGSON
G.R. No. 198528 October 13, 2014

WORK-RELATED COMPENSABLE ILLNESS

DOCTRINE:
Time and again, this Court has held that cardiovascular disease, coronary artery disease, and other heart
ailments are work-related and, thus, compensable.

FACTS:
Since 1986, Juanito Bengson has been working as a seafarer for Magsaysay, Inc. He entered into his 22nd
contract of employment with Magsaysay, Inc. Prior to his deployment, Bengson underwent and passed the Pre-
Employment Medical Examination (PEME) and was found to be "fit for sea duty. On October 5, 2007, after
doing his usual duties on board the vessel, [Bengson] suddenly experienced difficulty in breathing and
numbness on half of his body. He was examined in Izola General Hospital in Slovenia. Due to his incapacity to
work, his immediate repatriation was arranged. Upon arrival in the Philippines, he was immediately brought to
the Manila Doctors Hospital for confinement under the supervision of company-designated-physician Dr.
Benigno F. Agbayani, Jr. Bengsons Medical Abstract/Discharge Summary showed that he had a stroke. Dr.
Agbayani issued an Initial Out-Patient Consult Report which stated that Bengsons illness was not work-related.
Thus, Magsaysay, Inc. did not anymore issue any assessment on [Bengsons] disability grade. [Bengson], on
the other hand, continuously took medications and was unable to return to his work as a seaman due to the
severity of his disability. [Bengson] thus filed his disability compensation claim against x x x Magsaysay, Inc.
However, during the grievance proceedings before the Associated Marine Officers and Seamens Union of the
Philippines (AMOSUP), his claim was outrightly denied by x x x Magsaysay, Inc.

LA RULING: illness of Bengson is related to his work and the strenuous nature of his work and the conditions
he was subjected to while working on board petitioners vessel caused his illness.

NLRC RULING: under the POEA-SEC, hematoma is not included in the list of compensable illnesses; this being
the case, Bengson should have proved that such illness was work-related and compensable, and it is not
enough for him to claim or show that it was contracted during his employment with petitioners.

CA RULING: Bengsons exposure to different hazards on board petitioners vessel, the performance of his
functions as Third Mate, and the extraordinary physical and mental strain required by his position caused him to
suffer his present illness. Therefore, his illness is work-related.

ISSUE: Is cardiovascular disease an occupational disease and and, thus, compensable?

SC RULING:
YES. In many cases decided in the past, this Court has held that cardiovascular disease, coronary artery
disease, and other heart ailments are compensable.

In the present case, petitioners flatly claim that Bengsons hypertensive cardio-vascular disease is not
compensable on the sole basis of its company-designated physician Agbayanis declaration that such illness is
not work-related.

However, the Court finds that Bengsons illness is work-related. The undisputed facts indicate that respondent
37
has been working for petitioners since 1988; that per his service record, he has been serving as Third Mate for
twelve (12) years; and that as Third Mate, he was saddled with heavy responsibilities relative to navigation of
the vessel, ship safety and management of emergencies. It is beyond doubt that respondent was subjected to
physical and mental stress and strain: as Third Mate, he is the ships fourth in command, and he is the ships
safety officer; these responsibilities have been heavy burdens on respondents shoulders all these years, and
certainly contributed to the development of his illness. Besides, "[i]t is already recognized that any kind of work
or labor produces stress and strain normally resulting in wear and tear of the human body." "Notably, it is a
matter of judicial notice that an overseas worker, having to ward off homesickness by reason of being physically
separated from his family for the entire duration of his contract, bears a great degree of emotional strain while
making an effort to perform his work well. The strain is even greater in the case of a seaman who is constantly
subjected to the perils of the sea while at work abroad and away from his family."
San Beda College of Law 193
4S: 2015 - 2016
LABOR LAW REVIEW Atty. Joyrich Golangco

Having worked for petitioners since 1988 under employment contracts that were continuously renewed, it can be
said that respondent spent much of his productive years with petitioners; his years of service certainly took a toll
on his body, and he could not have contracted his illness elsewhere except while working for petitioners. To be
sure, the Court has ruled that "the list of illnesses/diseases in Section 32-A does not preclude other
illnesses/diseases not so listed from being compensable. The POEA-SEC cannot be presumed to contain all the
possible injuries that render a seafarer unfit for further sea duties." And equally significant, "it is not the injury
which is compensated, but rather it is the incapacity to work resulting in the impairment of ones earning
capacity."

San Beda College of Law 194


4S: 2015 - 2016
LABOR LAW REVIEW Atty. Joyrich Golangco

85. PEDRO LIBANG, JR. vs. INDOCHINA SHIP MANAGEMENT, INC., MR. MIGUEL SANTOS and
MAJESTIC CARRIERS, INC.
G.R. No. 189863 September 17, 2014
REYES, J.

ASSESSMENT BY COMPANY-DESIGNATED DOCTOR VS. ASSESSMENT BY CLAIMANTS DOCTOR

DOCTRINE:
The respondents could not be allowed to benefit from their physicians inaction or refusal to disclose the results
of the diagnostic tests performed upon Libang, the extent of the patients illnesses, and the effect of the severity
of these illnesses on his fitness or disability.

FACTS:
Libang entered into a nine-month employment contract with ISMI, a domestic manning agency that acted for and
in behalf of its foreign shipping company, Majestic. Libang was engaged as a Cook 1 for the vessel M/V
Baltimar Orion. While Libang was on board M/V Baltimar Orion, he experienced numbness on the left side of his
face, difficulty in hearing from his left ear, blurred vision of his left eye and speech problem. Libang was
eventually repatriated. Two days later, he reported to ISMI and was endorsed for medical attention to the
company-designated physician, Dr. Robert Lim (Dr. Lim) of the Marine Medical Services in Metropolitan
Hospital. Dr. Lim issued to Libang a medical certificate which states that the hypertension of Libang could be
pre-existing. Considering Dr. Lims failure to assess Libangs disability despite his health status, the latter
sought medical attention and assessment from another doctor, Dr. Efren R. Vicaldo (Dr. Vicaldo) of the
Philippine Heart Center. A medical certificate issued by Dr. Vicaldo states that Libang has Hypertensive
Cardiovascular Disease, Diabetes Mellitus and S/P Cerebrovascular accident and gave Impediment Grade VI
(50%). Libang filed with NLRC a complaint for disability benefit. The respondents disputed any liability arguing
that the disability was pre-existing.

LA RULING: granted claim for disability benefit. Without doubt, [Libang] had gone through a thorough and rigid
screening process of [ISMI and Santos] (medical examinations included) before an agreement or the contract of
employment between the parties was reached and actualized. This is precisely the reason why [ISMI and
Santos], should not be allowed to make use of the argument that [Libang] is not entitled to any disability
benefits as he was already suffering from a pre-existing illness when he entered into a contract of
employment with [ISMI and Santos].

NLRC RULING: In sustaining the LAs finding that Libang was entitled to disability benefit, the NLRC considered
the reasonable connection between the nature of Libangs work as a cook and the development of his illness.

CA RULING: For the CA, the lone assessment made by Dr. Vicaldo could not have justified the LAs and
NLRCs finding of a Grade VI disability. The Philippine Overseas Employment Administration-Standard
Employment Contract (POEA-SEC) requires the company-designated physician to be the one to make a
disability assessment of a seafarer.

ISSUE: Is Libang entitled to disability benefit?

SC RULING:
YES. Rather than making a full assessment of Libangs health condition, disability or fitness, Dr. Lim only
reasoned in his medical certificate dated that Libangs hypertension could be pre-existing and that it was
difficult to say whether his diabetes mellitus and small pontine infarct are pre-existing or not. His assessment
was evidently uncertain and the extent of his examination for a proper medical diagnosis was incomplete. The
alleged concealment by Libang of his hypertension during his pre-employment medical examination was also
unsubstantiated, but was a mere hearsay purportedly relayed to Dr. Lim by one Dr. Aileen Corbilla, his co-
attending physician. A categorical statement from Dr. Lim that Libangs illnesses were pre-existing and non-
work-related was made only in his affidavit dated July 16, 2004, or after the subject labor complaint had been
filed. Still, Dr. Lim gave no explanation for his statement that Libangs illnesses were not work-related.

Given the failure of Dr. Lim to fully evaluate Libangs illness, disability or fitness to work, the seafarer was
justified in seeking the medical expertise of his physician of choice. The NLRC did not commit grave abuse of
discretion in considering Dr. Vicaldos assessment. As against an incomplete evaluation by Dr. Lim, the medical
certificate issued by Dr. Vicaldo included a determination of the disability grade that applied to Libangs
San Beda College of Law 195
4S: 2015 - 2016
LABOR LAW REVIEW Atty. Joyrich Golangco

condition. Libang was diagnosed to have both Hypertensive Cardiovascular Disease and Diabetes Mellitus with
an Impediment Grade VI. He was declared to be unfit to resume to work as a seafarer in any capacity.

The respondents could not be allowed to benefit from their physicians inaction or refusal to disclose the results
of the diagnostic tests performed upon Libang, the extent of the patients illnesses, and the effect of the severity
of these illnesses on his fitness or disability. The respondents even failed to sufficiently dispute the finding of the
LA and NLRC that Libangs illnesses had resulted in a Grade VI disability.

San Beda College of Law 196


4S: 2015 - 2016
LABOR LAW REVIEW Atty. Joyrich Golangco

86. INTERORIENT MARITIME ENTERPRISES, INC. vs. VICTOR M. CREER III
G.R. No. 181921 September 17, 2014
DEL CASTILLO, J.:

ELEMENTS FOR DISABILITY TO BE COMPENSABLE

DOCTRINE:
For an illness to be compensable, Section 20(B)(6) of the 2000 Amended Standard Terms and Conditions
Governing the Employment of Filipino Seafarers on Board Ocean-Going Vessels (2000 Amended Standard
Terms and Conditions), deemed incorporated in the POEA Contract, requires the concurrence of two elements:
first, that the illness mustbe work-related; and second, that the work- related illness must have existed during
the term of the seafarers employment contract.

FACTS:
InterOrient hired Victor as Galley Boy on board the vessel M/V MYRTO owned by Calidero Shipping Company,
Ltd. (Calidero). Victor alleged that when he was about to get provisions from the cold storage sometime in
November 2001, he felt a sudden pain in his chest that radiated to his back. Since then, he experienced
incessant cough, nasal congestion, difficulty in breathing, physical weakness, chills and extreme apprehension.
According to him, this condition persisted until the expiration of his contract on May 7, 2002. On May 9, 2002,
Victor arrived in Manila. The following day, he reported to the office of InterOrient and informed the company
about the pain he experienced while he was on board. Victor averred that InterOrient merely advised him to
consult a doctor without giving him any doctors referral. He did, however, sign a Receipt and Release where he
acknowledged receipt of the full payment of his monetary entitlements under the employment contract.
According to him, he underwent medical examinations in different hospitals and that he shouldered all the
expenses. Victor consulted another physician, Dr. Vicaldo, at the Philippine Heart Center. After conducting a
medical examination and evaluation, Dr. Vicaldo issued a medical certificate indicating that Victor was
diagnosed with Hypertension, Stage II, and Pulmonary Tuberculosis. He gave Victor an impediment grade VIII
(33.59%) and further declared him unfit to resume work as a seaman in any capacity, and that his illness was
considered work-aggravated. Victor claimed for disability benefit.

LA RULING: denied claim. Labor Arbiter noted that there is nothing on record to show that Victor ever made
any formal claim for sickness allowance, medical benefits and disability benefits while on board the vessel or
immediately after his repatriation. Neither did he submit to, nor apply for any post-employment medical
examination within three days from his repatriation a requirement for claims for sickness and disability
benefits.

NLRC RULING: affirmed in toto the Decision of the Labor Arbiter and dismissed Victors appeal.

CA RULING: granted the same and awarded him permanent disability benefits and attorneys fees. Applying
Section 32-A of the POEA Contract, the CA declared Victors illness, pulmonary tuberculosis, included inthe list
of occupational diseases. It found that Victor was overworked and over-fatigued as a result of the long hours of
work required by his duties and that he was exposed todaily rapid variations in temperature.

ISSUE: Is Victor entitled to disability benefits?

SC RULING:
No. For a seamans claim for disability to prosper, it is mandatory that within three days from his repatriation, he
is examined by a company-designated physician. Non-compliance with this mandatory requirement results in
the forfeiture of the right to claim for compensation and disability benefits. It is undisputed that on May 7, 2002,
Victors employment contract was completed. He arrived in Manila on May 9, 2002; the following day, or on May
10, 2002, he reported to the office of InterOrient. Although he averred that he informed InterOrient about the
pain he experienced whileon board the vessel, the company allegedly only advised him to consult a doctor but
did not give any referral.

SC is not persuaded. His repatriation was not due to any medical reasons but because his employment contract
had already expired. Other than his self-serving allegation that he experienced pain while on board, he was not
able to substantiate the same. There was no showing that he reported his injury to his officers while on board
the vessel; neither did he prove that he sought medical attention but was refused. Likewise, other than his bare
and self-serving assertion that he informed InterOrient about his pain, he presented no evidence ortangible
San Beda College of Law 197
4S: 2015 - 2016
LABOR LAW REVIEW Atty. Joyrich Golangco

proof that he indeed requested for medical attention, much more that he was rebuffed.

On the contrary, the records show that when he reported to InterOrient immediately after his repatriation, he
signed a Receipt and Release stating that he has not contracted or suffered any illness or injury from work and
that he was discharged in good and perfect health.

Victors illness is not compensable.


Even if we disregard the mandatory three-day rule on post-employment medical examination by the company-
designated physician, Victors claim for disability benefits must still fail for not being compensable. For an illness
to be compensable, Section 20(B)(6) of the 2000 Amended Standard Terms and Conditions Governing the
Employment of Filipino Seafarers on Board Ocean-Going Vessels (2000 Amended StandardTerms and
Conditions), deemed incorporated in the POEA Contract, requires the concurrence of two elements: first, that
the illness mustbe work-related; and second, that the work- related illness must have existed during the term of
the seafarers employment contract.

a) Victor failed to show that his illness existed during the term of his contract - As already mentioned, the
reason for Victors repatriation was the completion/expiration of his contract and not because of any
sickness. Other than his uncorroborated and self-serving assertion that he experienced chest pains
while on board the vessel, there was absolutely no proof at all that he consulted a doctor while on
board, or that he reported the same to his superiors so that he will be provided with medical assistance.
On the contrary, upon repatriation, he signed a Receipt and Release wherein he acknowledged that he
worked under normal conditions on board the vessel; that he did not contract or suffer any injury; and
that he was discharged in good health. Victor never alleged that he was coerced into signing the
Receipt and Release or that he did not understand the same.
b) Victor failed to show that his illness is work-related - While pulmonary tuberculosis is listed as an
occupational disease, the Court is not convinced that Victors pulmonary tuberculosis is work-acquired
or work-aggravated because if it were so, then at the outset, Victor should have already been diagnosed
with pulmonary tuberculosis when he sought medical help one month from his repatriation. Instead, Dr.
Ayuyao diagnosed him with Community Acquired Pneumonia I and Bronchial Asthma sicknesses
which aside from being different from pulmonary tuberculosis, were not shown to have any relation
thereto.

San Beda College of Law 198


4S: 2015 - 2016
LABOR LAW REVIEW Atty. Joyrich Golangco

87. RICARDO A. DALUSONG vs. EAGLE CLARC SHIPPING PHILIPPINES, INC., NORFIELD OFFSHORE
AS, and/or CAPT. LEOPOLDO T. ARCILLAR, and COURT OF APPEALS
G.R. No. 204233, September 3, 2014
CARPIO, Acting C.J.

ASSESSMENT BY COMPANY-DESIGNATED DOCTOR VS. ASSESSMENT BY CLAIMANTS DOCTOR

DOCTRINE:
The doctor who have had a personal knowledge of the actual medical condition, having closely, meticulously
and regularly monitored and actually treated the seafarers illness, is more qualified to assess the seafarers
disability.

FACTS:
Private respondents hired petitioner as Able Seaman on board their vessel MV Malene Ostervold with a basic
salary of US$800 per month. On 13 December 2009, while petitioner was drilling to attach an overboard safety
equipment on the vessel, a sudden swell caused some movement ofthe vessel. As a result, one of the crew fell
directly on petitioner, inflicting injury on petitioners right foot. Petitioner was repatriated to the Philippines for
further examination and medical treatment.

Dr. Nicomedes Cruz, the company-designated doctor, gave petitioner an interim disability grading based on the
Philippine Overseas Employment Administration (POEA) schedule of disability of "grade 8 that is moderate
rigidity or one third loss of motion or lifting power of the trunk." Petitioner disagreed with the disability
assessment and consulted Dr. Nicanor Escutin, a physician of his own choice, who found petitioner to be
suffering from "PARTIAL PERMANENT DISABILITY and concluded that petitioner is "unfit for seaduty in
whatever capacity as seaman."
Petitioner filed with the NLRC a complaint against private respondents, claiming disability benefits, sick wages,
damages, and attorneys fees. Petitioner maintained that he is entitled to full disability benefits of US$80,000,
while private respondents insisted that petitioner is only entitled to US$12,551 based on the disability
assessment of the company-designated doctor.

ISSUE: Is Dalusong entitled to full disability benefits?

LA RULING: ruled in favor of private respondents. The Labor Arbiter did not give probative value to the medical
report presented by petitioner.

NLRC RULING: modified the Labor Arbiters decision.

CA RULING: ruled that it is the company-designated doctor who initially determines the degree of disability of
petitioner.

SC RULING:
NO. SC agree with the Court of Appeals ruling, giving more credence to the medical findings of the company-
designated doctor. Contrary to the ruling of the NLRC, petitioners doctor did not categorically give petitioner a
grade 1 disability rating which is equivalent to total and permanent disability. Petitioners physician found
petitioner to be suffering from "PARTIAL PERMANENT DISABILITY," and "is UNFIT FOR SEA DUTY in
whatever capacity as seaman." Aside from this seemingly inconsistent assessment by petitioners doctor, there
was no evidence submitted of medical procedures, examinations or tests which would support his conclusion
that petitioner is unfit for sea duty in whatever capacity as a seaman. In contrast, the company-designated
doctor gave petitioner a final disability grading under the POEA schedule of disabilities of "grade 11-complete
immobility of an ankle joint in normal position," only after petitioner had undergone a series of medical tests and
examinations, and physical therapy over a period of six months, during which the company designated doctor
issued periodic medical reports. As the Court aptly stated in Philman Marine Agency, Inc. (now DOHLE-
PHILMAN Manning Agency, Inc.) v. Cabanban, "the doctor who have had a personal knowledge of the actual
medical condition, having closely, meticulously and regularly monitored and actually treated the seafarers
illness, is more qualified to assess the seafarers disability." Based on the Disability Report of petitioners doctor,
it appears that he only conducted a physical examination on petitioner before issuing his final diagnosis and
disability rating on petitioners condition. Clearly, the findings of the company-designated doctor, who, with his
team of specialists which included an orthopedic surgeon and a physical therapist, periodically treated petitioner
for months and monitored his condition, deserve greater evidentiary weight than the single medical report of
San Beda College of Law 199
4S: 2015 - 2016
LABOR LAW REVIEW Atty. Joyrich Golangco

petitioners doctor, who appeared to have examined petitioner only once.

Petitioner argues that since his treatment lasted for more than 120 days, then his disability is deemed total and
permanent. Petitioners contention is not entirely correct.

Upon sign-off from the vessel for medical treatment, the seafarer is entitled to sickness allowance equivalent to
his basic wage until he is declared fit to work or the degree of permanent disability has been assessed by the
company-designated physician but in no case shall this period exceed one hundred twenty (120) days.

Just because the seafarer is unable to perform his job and is undergoing medical treatment for more than 120
days does not automatically entitle the seafarer to total and permanent disability compensation. In this case,
petitioner's medical treatment lasted more than 120 days but less than 240 days, after which the company-
designated doctor gave petitioner a final disability grading under the POEA schedule of disabilities of "grade 11 -
complete immobility of an ankle joint in normal position." Thus, before the maximum 240-day medical treatment
period expired, petitioner was issued a final disability grade 11 which is merely equivalent to a permanent partial
disability, since under Section 32 of the POEA-SEC, only those classified under grade 1 are considered total
and permanent disability. Clearly, petitioner is only entitled to permanent partial disability compensation, since
his condition cannot be considered as permanent total disability.

San Beda College of Law 200


4S: 2015 - 2016
LABOR LAW REVIEW Atty. Joyrich Golangco

88. ROBERT KUA, CAROLINE N. KUA, AND MA. TERESITA N. KUA, v. GREGORIO SACUPAYO AND
MAXIMINIANO PANERIO
G.R. No. 191237 September 24, 2014
PEREZ, J.:

DOCTRINE:
Probable cause is affirmed against an employer who failed to remit SSS contributions and payments on
loans of its employees if it was only under threat of criminal liability that the employers subsequently remitted
what they had long deducted from the wages of said employees.
FACTS:
Petitioners are members of the Board of Directors and the officers of Vicmar Development Corporation (Vicmar).
Respondents Sacupayo and Panerio were VICMAR employees.

As required by law, Vicmar, deducted the SSS contributions of respondents from their wages. A certain amount
from Sacupayos wage representing the monthly amortization from a he loan he obtained from the SSS. The
deductions were initially remitted to SSS.

However, sometime in 2003 and 2004, unknown to respondents and despite the continued SSS and
amortization deductions from their wages, Vicmar stopped remitting the same. Meantime in 2004, Sacupayo
and Panerio were dismissed from employment. Both filed complaints for illegal dismissal.

Panerio was thereafter afflicted with Chronic Persistent Asthma but when he applied for sickness benefits before
the SSS the same was denied for the reason that no contributions or payments were made for 12 months prior
to the semester of confinement. Sacupayo, for his part, filed another loan application but this was also denied
outright for non-payment of a previous loan which should have been fully paid if not for the failure of Vicmar to
remit the amounts due to the SSS.

Aggrieved respondents filed complaints before the Office of the City Prosecutor. Vicmar then remitted to SSS
the contributions and loan payments of respondents. Nevertheless 3 separate Informations were filed against
petitioners officers of Vicmar for violation of Section 22 (a) in relation to Section 28 (e) of RA 8282 otherwise
known as the Social Security Act of 1997.

MTC RULING: Dismissed outright for lack of jurisdiction

RTC RULING: Given due course but later on granted the Motion of petitioners to withdraw the criminal cases.

ISSUE: Validity of the order of the trial court directing the withdrawal from its dockets of Criminal Case Nos.
2006-072, 2006-073 and 2006-074 for violation of Sec. 22 (a) and (d) in relation to Sec. 28 (e) of R.A. No. 8282.

SC RULING:
The factual milieu obtaining herein does not denote a simple delay in payment. Again, petitioners initially failed
to remit the SSS contributions and payments of respondents such that respondents were denied benefits under
the SS Law which they wanted to avail of. It was only under threat of criminal liability that petitioners
subsequently remitted what they had long deducted from the wages of respondents.

The culpability of the accused under the indictment is not yet before us. Yet to be determined during the
ensuing trial are considerations such as the extent and reason for the delay, the date of actual remittance and
all other circumstances that attended such remittance. All these are matters of defense that need proof during
trial.

WHEREFORE, the petition is DENIED. The Decision of the Court of Appeals in CA-G.R. SP No. 01569-MIN
is AFFIRMED. Criminal Case Nos. 2006-072, 2006-073 and 2006-074 pending before the Regional Trial Court,
Branch 20, Cagayan de Oro City are REINSTATED and the Presiding Judge thereof is DIRECTED to dispose of
the cases with dispatch.

San Beda College of Law 201


4S: 2015 - 2016
LABOR LAW REVIEW Atty. Joyrich Golangco

89. GOVERNMENT SERVICE INSURANCE SYSTEM v. JOSE M. CAPACITE
G.R. No. 199780 September 24, 2014
BRION, J.:

DOCTRINE:
For sickness and the resulting death of an employee to be compensable, the claimant must show either: (1) that
it is a result of an occupational disease listed under Annex "A" of the Amended Rules on Employees'
Compensation with the conditions set therein satisfied; or (2) if not so listed, that the risk of contracting the
disease was increased by the working conditions.

FACTS:
Provincial Office who successively held the following positions: Junior Statistician, Bookkeeper, Bookkeeper II,
and finally as Accountant I. Due to persistent cough coupled with abdominal pain, Elma was admitted at the
Bethany Hospital where the pathology examination showed that she was suffering from Adenocarcinoma,
moderately differentiated, probably cecal origin with metastases to mesenteric lymph node and seeding of the
peritoneal surface.c i Elma died due to Respiratory Failure secondary to Metastatic Cancer to the lungs; Bowel
cancer with Hepatic and Intraperitoneal Seeding and Ovarian cancer. Elmas surviving spouse, Jose, filed a
claim for ECC death benefits before the GSIS Branch Office, alleging that Elmas stressful working condition
caused the cancer that eventually led to her death.
GSIS: denied Joses claim for failure to present direct evidence to prove a causal connection between Elmas
illness and her work.

ECC: also denied it holding that colorectal cancer is not listed as an occupational and compensable disease
under Annex A of the Amended Rules on Employees Compensation. Although its item 17 provides that cancer
of the lungs, liver and brain shall be compensable, the rules required that it had been incurred by employees
working as vinyl chloride workers, or plastic workers.

CA RULING: reversed ECC. That it was enough that the nature of her employment contributed to the
development of the disease. As a bookkeeper, Elma had been exposed to voluminous dusty records and
other harmful substances that aggravated her respiratory disease.

ISSUE: Whether CA erred in ruling that Metastasized to the lungs is an ailment akin to respiratory disease
under ANNEX A of P.D. NO. 626, as amended, o that such disease is work-related.

SC RULING:
PD 626, as amended, defines compensable sickness as any illness definitely accepted as an occupational
disease listed by the Commission, or any illness caused by employment subject to proof by the employee that
the risk of contracting the same is increased by the working conditions. Of particular significance in this
definition is the use of the conjunction or, which indicates alternative situations.

Based on this definition, we ruled in GSIS v. Vicencio that for sickness and the resulting death of an employee
to be compensable, the claimant must show either: (1) that it is a result of an occupational disease listed under
Annex "A" of the Amended Rules on Employees' Compensation with the conditions set therein satisfied; or (2) if
not so listed, that the risk of contracting the disease was increased by the working conditions.

While item 17, Annex A of the Amended Rules of Employees Compensation considers lung cancer to be a
compensable occupational disease, it likewise provides that the employee should be employed as a vinyl
chloride worker or a plastic worker. In this case, however, Elma did not work in an environment involving the
manufacture of chlorine or plastic, for her lung cancer to be considered an occupational disease. There was,
therefore, no basis for the CA to simply categorize her illness as an occupational disease without first
establishing the nature of Elmas work. Both the law and the implementing rules clearly state that the given
alternative conditions must be satisfied for a disease to be compensable.

Aside from Joses general allegations proving the stressful duties of his late wife, no reasonable proof exists to
support the claim that her respiratory disease, which is similar to lung cancer, was aggravated by her working
conditions. The records do not support the contention that she had been exposed to voluminous and dusty
records, nor do they provide any definite picture of her working environment.

San Beda College of Law 202


4S: 2015 - 2016
LABOR LAW REVIEW Atty. Joyrich Golangco

90. OSG SHIPMANAGEMENT MANILA, INC., MERCEDES M. RAVANOPOLOUS, OSG SHIPMANAGEMENT


(UK) LTD. & M/T DELPHINA, v. JOSELITO B. PELLAZAR
G.R. No. 198367 August 06, 2014
BRION, J.:

DOCTRINE:
In the present case, since there is a conflict in the assessment of the company-designated physicians and an
employees physician of choice, the matter should have been referred to a third doctor for final determination as
required by the POEA-SEC and the parties CBA. Since the employee was responsible for the non-referral to
the third doctor because of his failure to inform the manning agency that he would be consulting a doctor of his
choice, he should suffer the consequences of the absence of a binding third opinion.

FACTS:
Pellazar was deployed to the M/T Delphina under an employment contract for eight months. While he was on
duty onboard the vessel, his right hand was injured after it was struck by a solid iron pipe. He was given
medical attention in a hospital in Braziland was later on medically repatriated.

Upon his arrival in Manila, Pellazar reported to OSG Manila and was referred to the company-designated
physicians, Dr. De Guzman and Dr. Banaga. Pellazars working diagnosis was complete fracture, distal part of
5th finger, right hand post-casting. The company-designated physicians gave Pellazar a Grade 10 disability
rating7 for loss of grasping power for large objects between fingers and palm of one hand.

Pellazar consulted a physician of his choice,Dr. Sabado who diagnosed him with loss of grasping power of 5th
finger, loss of opposition between finger and thumb (r) and ankylosis of the 5thfinger (r), and certified that he
was permanently unfit for any sea duty.

Petitioners denied liability alleging that Pellazar failed to comply with his duty to observe the dispute resolution
provisions of the CBA. Also, that Pellazar was not entitled to disability compensation higher than what was
provided under a Grade 10 disability rating as that was the company-designated physicians assessment of his
disability. A Grade 10 disability is compensated US$10,075.00 under the POEA Standard Employment Contract
(POEA-SEC).

LA RULING: in favor of Pellazar

NLRC: affirmed but modified the labor arbiters decision ruling that Pellazar is entitled only to an award of
$10,075.01 which is the equivalent of a Grade 10 disability in accordance with the disability rating given to him
by the company-designated physicians

CA RULING: reversed the challenged NLRC rulings and, reinstated LAs award of permanent total disability
benefits to Pellazar
ISSUE: Whether Pellazar is entitled to a Grade 10 disability or a permanent total disability.

SC RULING:
Entitlement to disability benefits by seamen on overseas work is a matter governed, not only by medical findings
but, by Philippine law and by the contract between the parties. The material statutory provisions are Articles 191
to 193 under Chapter VI (Disability Benefits) of the Labor Code, in relation with Rule X of the Rules and
Regulations Implementing Book IV of the Labor Code. By contract, Department Order No. 4, series of 2000 of
the Department of Labor and Employment (the POEA Standard Employment Contract) and the parties' CBA
bind the seaman and his employer to each other. The terms under the POEA-SEC are to be read in accordance
with what the Philippine law provides.

Under the POEA-SEC and the AMOSUP/IMEC TCCC CBA, the degree of disability arising from a work-
connected injury or illness of a seafarer or his fitness to work shall be assessed by the company-designated
physician to make the employer liable. Controversy arose, however, when Pellazar consulted a physician of his
San Beda College of Law 203
4S: 2015 - 2016
LABOR LAW REVIEW Atty. Joyrich Golangco

choice, whose findings are in conflict with those of the company-designated physicians. This conflict invariably
leads to the question of whose findings should prevail.

In the present case, since there is a conflict in the assessment of the company-designated physicians and Dr.
Sabados certification in relation to Pellazars fitness or unfitness to work, the matter should have been referred
to a third doctor for final determination as required by the POEA-SEC and the parties CBA. Since Pellazar was
responsible for the non-referral to the third doctor because of his failure to inform the manning agency that he
would be consulting Dr. Sabado, he should suffer the consequences of the absence of a binding third opinion.
Thus, the NLRC was well within the bounds of its jurisdiction, in upholding the disability assessment of Drs. De
Guzman and Banaga as against Pellazars physician of choice.

Since the company-designated physicians gave Pellazar only a Grade 10 disability - and not a permanent total
disability - he cannot be entitled to the full disability benefits of US$75,000.00 under the CBA
91. ESTRELLA D. S. BAEZ v. SOCIAL SECURITY SYSTEM AND DE LA SALLE UNIVERSITY
G.R. No. 189574 July 18, 2014

DOCTRINE:
For death benefits the law requires proof by substantial evidence, or such relevant evidence which a reasonable
mind might accept as adequate to justify a conclusion, that the nature of his employment or working conditions
increased the risk of contracting the ailment or that its progression or aggravation was brought about thereby.

FACTS:
Baylon, the husband of petitioner, was employed by DLSU. From 21991-2006, Baylon worked as a Laboratory
Technician at the Chemistry Department.

In 2006, Baylon was confined at Manila Doctors Hospital due to fever, weakness, dysuria and flank pains. He
was diagnosed to be suffering from urinary tract infection. A month later he was confined again for functional
dyspepsia. Later, he was diagnosed to be suffering from Systemic Lupus Erythematosus (SLE).

Dr. Castillo prepared a clinical abstract/toxicologic assessment on Baylon and she stated that based on the
occupational history of the patient, x x x the probability of a chemically induced disease cannot be discounted.

Baylon succumbed to the complications of his disease on 27 August 2006. Baylons attending physician, Dr.
Torres, issued a Medical Certificate stating that Baylon who was confined and expired in Medical Center Manila
for Systemic Lupus Erythematosus may have been precipitated by the chronic exposure to chemicals which is
an occupational hazard in his performance of being a laboratory technician. Based on medical opinions of Dr.
Castillo and Dr. Torres, petitioner filed a claim for death benefits under the Employees Compensation Law
before the Social Security System (SSS).

SSS: Denied claim on two grounds: 1) the cause of death, cardiac complication of SLE, is not considered work-
related; and 2) SLE is not included in the list of occupational diseases.

ECC: Also denied claim on the ground that SLE is caused by a genetic tendency to mount an abnormal
immune response against ones own tissues or organs leading to their destruction or malfunction.

CA RULING: dismissed petition for review for being filed out of time.

ISSUE: Whether petitioners claim should prosper.

SC RULING:
NO. In order for the beneficiary of an employee to be entitled to death benefits under the SSS, the cause of
death of the employee must be a sickness listed as an occupational disease by ECC; or any other illness
caused by employment, subject to proof that the risk of contracting the same is increased by the working
conditions.
It is undisputed that SLE is not listed as an occupational disease under Annex A of the Rules on Employees
Compensation. Thus, petitioner has to prove by substantial evidence the causal relationship between her
husbands illness and his working conditions.

While there are certain chemicals accepted as increasing the risks of contracting SLE such as chlorinated
San Beda College of Law 204
4S: 2015 - 2016
LABOR LAW REVIEW Atty. Joyrich Golangco

pesticides and crystalline silica, the law requires proof by substantial evidence, or such relevant evidence which
a reasonable mind might accept as adequate to justify a conclusion, that the nature of his employment or
working conditions increased the risk of contracting the ailment or that its progression or aggravation was
brought about thereby.

Petitioner relied unqualifiedly on the toxicological report which failed to prove the causal relationship between
Baylons work and his illness. The report made an indirect link between SLE and chemicals through drug-
induced lupus.

SLE and Drug-Induced Lupus Erythematosus are both autoimmune diseases. Drug-induced lupus is a
temporary and mild form of lupus caused by certain prescription medications. They include some types of high
blood pressure drugs (such as hydralazine, ACE inhibitors, and calcium channel blockers)
and diuretics (hydrochlorothiazide). Symptoms resolve once the medication is stopped.

Furthermore, the toxicological report made mention of certain drugs with chemical structures related to
aromatic amines or substituted hydrazines, listed in the inventory of the school, can affect the immune system.
This would include Benzenes, Naphthylamine, Toluene, Dinitrophenylhydrazine, etc. However, these drugs
were not proven to have been administered on Baylon. These substances which can induce the disease all
pertain to drugs which are orally administered on the patient. There is no showing that the drugs given to
Baylon had increased his risk of contracting Drug-Induced Lupus and SLE.
92. ALPHA SHIP MANAGEMENT CORPORATION/JUNEL M CHAN and/or CHUO-KAIUN COMPANY,
LIMITED v. ELEOSIS v. CALO
G.R. No. 192034 January 13, 2014
DEL CASTILLO, J.:
DOCTRINE:
An employees disability becomes permanent and total when so declared by the company-designated physician,
or, in case of absence of such a declaration either of fitness or permanent total disability, upon the lapse of the
120 or 24045-day treatment period, while the employees disability continues and he is unable to engage in
gainful employment during such period, and the company-designated physician fails to arrive at a definite
assessment of the employees fitness or disability. This is true "regardless of whether the employee loses the
use of any part of his body.

FACTS:
Respondent Calo worked for petitioners Alpha Ship, Junel M. Chan and their foreign principal, (CKCL) under 7
employment contracts.

While MV Iris was in China, respondent suffered back pain on the lower part of his lumbar region and urinated
with solid particles. On checkup, the doctor found him suffering from urinary tract infection and renal colic, and
was given antibiotics. When respondents condition did not improve, he consulted another doctor in Chile and
was found to have kidney problems and urinary tract infection but was declared fit for work on a "light duty"
basis. In Japan, respondent was diagnosed with suspected renal and/or ureter calculus and was declared "unfit
for work.

Respondent was thus repatriated and was referred by petitioners to Dr. Cruz, the company-designated
physician who continously examined respondent from 2004-2005.
Respondent, who felt that his condition has not improved consulted another specialist in internal medicine, Dr.
Vicaldo, who issued the following diagnosis: that it was Impediment Grade X, that he is now unfit to resume
work as seaman in any capacity and that his illness is considered work aggravated/related. Respondent filed a
claim for disability benefits with petitioners, but the claim was denied.

LA: granted permanent total disability benefits and attorneys fees to respondent, but denied his claim for moral
and exemplary damages.

NLRC: Appeal is granted. The decision of the Labor Arbiter was vacated and set aside. The complaint for
dismissed for lack of merit.

CA RULING: NLRC decision was reversed. Decision of the Labor Arbiter was reinstated.
ISSUE: Whether respondents claim for disability benefits should prosper.
SC RULING:
San Beda College of Law 205
4S: 2015 - 2016
LABOR LAW REVIEW Atty. Joyrich Golangco

YES. An employees disability becomes permanent and total when so declared by the company-designated
physician, or, in case of absence of such a declaration either of fitness or permanent total disability, upon the
lapse of the 120 or 24045-day treatment period, while the employees disability continues and he is unable to
engage in gainful employment during such period, and the company-designated physician fails to arrive at a
definite assessment of the employees fitness or disability. This is true "regardless of whether the employee
loses the use of any part of his body."

Respondent was repatriated on October 12, 2004 and underwent treatment by the company-designated
physician, Dr. Cruz, until October 14, 2005, or for a continuous period of over one year or for more than the
statutory 120-day47 or even 240-day48 period. During said treatment period, Dr. Cruz did not arrive at a definite
assessment of respondents fitness or disability; thus, respondents medical condition remained unresolved. It
was only on July 18, 2006 that respondent was declared fit to work by Dr. Cruz. Such declaration, however,
became irrelevant, for by then, respondent had been under medical treatment and unable to engage in gainful
employment for more than 240 days. Pursuant to the doctrine in Kestrel, the conclusive presumption that the
respondent is totally and permanently disabled thus arose.

In the same manner, the issue of which among the two diagnoses or opinions should prevail that of Dr. Cruz
or Dr. Vicaldo is rendered irrelevant in view of the lapse of the said 240-day period. As far as the parties are
concerned, respondents medical treatment and disability continued for more than 240 days without any finding
or diagnosis by the company-designated physician that he was fit to resume work. Thus, consonant with law
and jurisprudence, respondent is entitled to a declaration of permanent total disability, as well as the
corresponding benefit attached thereto in the amount of US$60,000.00.
93. INC. SHIPMANAGEMENT, INC., CAPTAIN SIGFREDO E. MONTERROYO AND/OR INTERORIENT
NAVIGATION LIMITED, v. ALEXANDER L. MORADAS G.R. No. 178564 January 15, 2014
PAYMENT OF DISABILITY BENEFITS

DOCTRINE:
An employer shall be liable for the injury or illness suffered by a seafarer during the term of his contract. There is
no need to show that such injury is work-related except that it must be proven to have been contracted during
the term of the contract. The rule, however, is not absolute and the employer may be exempt from liability if he
can successfully prove that the cause of the seamans injury was directly attributable to his deliberate or willful
act.

FACTS:
Respondent was employed as wiper for the vessel MV Commander by petitioner INC Shipmanagement, Inc. for
its principal, petitioner Interorient Navigation, Ltd. Respondent claimed while working, certain chemicals
splashed all over his body because of an explosion. Respondent demanded for the payment of his full disability
benefits under Section 20 (B) in relation to Sections 30 and 30-A of the Philippine Overseas Employment
Agency (POEA) Standard Employment Contract (POEA-SEC), in the amount of US$60,000.00, which
petitioners refused to heed. Thus, respondent filed a complaint against petitioners for the same.

Petitioners denied respondents claims, contending that his injury was self-inflicted and, hence, not
compensable under Section 20 (D) of the POEA-SEC. They denied that there was an explosion and claimed
that respondent poured thinners on himself and set himself on fire. They averred that he was led to commit such
act because he was to be dismissed for stealing supplies. They also stated that before they discovered
respondent burning, he caused flooding in the engine room.

LA RULING: The LA ruled in favor of petitioners, dismissing respondents complaint for lack of merit. The LA
held that respondents injury was self-inflicted and that no incinerator explosion occurred that would have
caused the latters injuries.

NLRC RULING: The NLRC sustained the findings of the LA. It pointed out that respondents mental or physical
fitness was not at issue since he was motivated to inflict injury to himself for reasons related to his impending
discharge and not because of his disposition.

CA RULING: CA found that the NLRC gravely abused its discretion. It found no logical and causal connection
between the act of pilferage as well as the act of causing the flooding in the engine room and the conclusion that
respondents injury was self-inflicted. It added that it was contrary to human nature and experience for
respondent to burn himself.
San Beda College of Law 206
4S: 2015 - 2016
LABOR LAW REVIEW Atty. Joyrich Golangco

ISSUE: Is the petitioner liable to pay the permanent total disability benefits?

SC RULING:
NO. The prevailing rule under Section 20 (B) of the 1996 POEA-SEC on compensation and benefits for injury or
illness was that an employer shall be liable for the injury or illness suffered by a seafarer during the term of his
contract. There was no need to show that such injury was work-related except that it must be proven to have
been contracted during the term of the contract. The rule, however, is not absolute and the employer may be
exempt from liability if he can successfully prove that the cause of the seamans injury was directly attributable
to his deliberate or willful act as provided under Section 20 (D) thereof.

Petitioners have successfully discharged the burden of proving by substantial evidence that respondents injury
was directly attributable to himself.

First, records bear out circumstances which all lead to the reasonable conclusion that respondent was
responsible for the flooding and burning incidents. The LA and NLRC gave credence to the corroborating
testimonies of the crewmen pointing to respondent as the person who deliberately caused the flooding incident.
Second, respondents version that the burning was caused by an accident is hardly supported by the evidence
on record. In addition to testimonies, an inspection of the incinerator after the incident showed that there were
unburnt cardboard cartons found inside with no sign of explosion and the steel plates surrounding it were cool to
the touch. Third, petitioners theory that respondents burns were self-inflicted gains credence through the
existence of motive. Both the LA and the NLRC made a factual finding that prior to the burning incident,
respondent was caught pilfering the vessels supplies for which he was told that he was to be relieved from his
duties. This adequately supports the reasonable conclusion that respondent may have harbored a grudge
against the captain and the chief steward who denied giving him the questioned items. At the very least, it was
natural for him to brood over feelings of resentment considering his impending dismissal. These incidents shore
up the theory that he was motivated to commit an act of sabotage which, however, backfired into his own
burning.

All told, petitioners having established through substantial evidence that respondents injury was self-inflicted
and, hence, not compensable pursuant to Section 20 (D) of the 1996 POEA-SEC.

San Beda College of Law 207


4S: 2015 - 2016
LABOR LAW REVIEW Atty. Joyrich Golangco

94. UNITED PHILIPPINE LINES, INC. AND HOLLAND AMERICA LINE, v. GENEROSO E. SIBUG
G.R. No. 201072 April 2, 2014
VILLARAMA, JR., J.:

PHYSICIAN ASSESMENT RE: PERMANENT AND TOTAL DISABILITY

DOCTRINE:
Company-designated physicians must arrive at a definite assessment of the seafarers fitness to work or
permanent disability within the period of 120 or 240 days. If he fails to do so and the seafarers medical
condition remains unresolved, the latter shall be deemed totally and permanently disabled.

FACTS:
Petitioners hired Sibug as waste handler on the vessel M/S Volendam. While cleaning, Sibug fell from a ladder
and suffered from Anterior Cruciate Ligament (ACL) which required surgery. After being declared fit for work,
Sibug was rehired by petitioners for the vessel M/S Ryndam. Sibug met another accident injuring his right hand
and wrist. He was repatriated and arrived in the Philippines on Jan. 15, 2007. On Sep. 7, 2007, the company
doctor issued a medical report that Sibug has a permanent but incomplete disability. In an email dated Sep. 28,
2007, the company doctor classified Sibugs disability as a grade 10 disability. Sibug filed two complaints for
disability benefits, illness allowance, damages and attorneys fees against petitioners.

LA RULING: Dismissed the Volendam case on the ground that Sibug was declared fit to work after his ACL
reconstruction surgery. As regards the Ryndam case, the Labor Arbiter awarded to Sibug US$10,075 which is
the equivalent award for the grade 10 disability rating issued by the company-designated doctor.

NLRC RULING: Reversed the LAs Decision granting Sibug permanent and total disability benefit of US$60,000
for his Volendam injury and another US$60,000 for his Ryndam injury. On reconsideration it reinstated the LA
decision.

CA RULING: Reinstated the NLRCs first decision ruling that Sibug was unable to perform his customary work
for more than 120 days on account of his Volendam and Ryndam injuries. Thus, he is entitled to permanent and
total disability benefit for both injuries.

ISSUE: Is Sibug entitled to permanent and total disability benefits?

SC RULING:
Volendam Injury No.
Ryndam Injury Yes.

Sibug is not entitled to permanent and total disability benefit for his Volendam injury since he became already fit
to work again as a seaman. As regards his Ryndam injury, Sibug is entitled to permanent and total disability
benefit amounting to US$60,000. The company-designated doctor failed to issue a certification with a
definite assessment of the degree of Sibugs disability for his Ryndam injury within 240 days. In Fil-Pride
Shipping Company, Inc., et al. v. Balasta, we held that the "company-designated physician must arrive at a
definite assessment of the seafarers fitness to work or permanent disability within the period of 120 or 240 days,
pursuant to Article 192 (c)(1) of the Labor Code and Rule X, Section 2 of the Amended Rules on Employees
Compensation. If he fails to do so and the seafarers medical condition remains unresolved, the latter shall be
deemed totally and permanently disabled." This definite assessment of the seamans permanent disability must
include the degree of his disability, as required by Section 20-B of the POEA-SEC.

In this case, Sibug was repatriated and arrived in the country on January 15, 2007 after his Ryndam injury. On
September 7, 2007, the company-designated doctor issued a medical report that Sibug has a permanent but
incomplete disability. But this medical report failed to state the degree of Sibugs disability. Only in an email
dated September 28, 2007, copy of which was attached as Annex 3 of petitioners position paper, was Sibugs
disability from his Ryndam injury classified as a grade 10 disability by the company-designated doctor. By that
time, however, the 240-day extended period when the company-designated doctor must give the definite
assessment of Sibugs disability had lapsed. From January 15, 2007 to September 28, 2007 is 256 days. Hence,
Sibugs disability is already deemed permanent and total.

San Beda College of Law 208


4S: 2015 - 2016
LABOR LAW REVIEW Atty. Joyrich Golangco

95. MAGSAYSAY MARITIME CORPORATION, v. OSCAR D. CHIN, JR.
G.R. No. 199022 April 7, 2014
ABAD, J.:

PERMANENT AND TOTAL DISABILITY BENEFIT LOSS OF EARNING CAPACITY

DOCTRINE:
After an award of disability compensation, an additional award for loss of earnings will result in double recovery.
In a catena of cases, the Court has consistently ruled that disability should not be understood more on its
medical significance but on the loss of earning capacity. Permanent total disability means disablement of an
employee to earn wages in the same kind of work, or work of similar nature that he was trained for or
accustomed to perform, or any kind of work which a person of his mentality and attainment could do. Disability,
therefore, is not synonymous with "sickness" or "illness." What is compensated is ones incapacity to work
resulting in the impairment of his earning capacity.

FACTS:
Thome Ship Management Pte. Ltd., acting through its agent petitioner Magsaysay Maritime Corporation hired
respondent Chin as seaman on board MV Star Siranger. Chin sustained injuries while working on his job aboard
the vessel. Chin filed a claim for disability with Pandiman Phils., Inc. which is the local agent of P&I Club of
which Magsaysay Maritime is a member. Pandiman offered US$30,000.00 as disability compensation which
Chin accepted. He then executed a Release and Quitclaim in favor of Magsaysay Maritime. Chin later filed a
complaint with (NLRC), claiming underpayment of disability benefits and attorneys fees.

The LA dismissed it for lack of merit, which the NLRC affirmed. The CA reversed the NLRC and ruled that Chin
was entitled to permanent total disability benefit of US$60,000.00. It remanded the case to the LA for
determination of other monetary awards. Magsaysay paid the deficiency award of US$30,000.00.

LA RULING: The LA ordered Magsaysay to pay Chin: a) P19,279.75 as reimbursement for medical expenses;
b) US$147,026.43 as loss of future wages; c) P200,000.00 as moral damages; d) P75,000.00 as exemplary
damages; and e) 10% of the total award as attorneys fees.

NLRC RULING: modified the Labor Arbiters Decision by deleting the awards of loss of future wages and moral
and exemplary damages for lack of factual and legal bases.

CA RULING: reversed NLRC; reinstated LA ruling

ISSUE: Is Chin entitled to an award of loss of future earnings on top of his disability benefits?

SC RULING:
NO. The Labor Arbiters award of loss of earning is unwarranted since Chin had already been given disability
compensation for loss of earning capacity. An additional award for loss of earnings will result in double recovery.
In a catena of cases, the Court has consistently ruled that disability should not be understood more on its
medical significance but on the loss of earning capacity. Permanent total disability means disablement of an
employee to earn wages in the same kind of work, or work of similar nature that he was trained for or
accustomed to perform, or any kind of work which a person of his mentality and attainment could do. Disability,
therefore, is not synonymous with "sickness" or "illness." What is compensated is ones incapacity to work
resulting in the impairment of his earning capacity.

Moreover, the award for loss of earning lacks basis since the Philippine Overseas Employment Agency (POEA)
Standard Contract of Employment (POEA SCE), the governing law between the parties, does not provide for
such a grant. What Section 20, paragraph (G) of the POEA SCE provides is that payment for injury, illness,
incapacity, disability, or death of the seafarer covers "all claims arising from or in relation with or in the course of
the seafarers employment, including but not limited to damages arising from the contract, tort, fault or
negligence under the laws of the Philippines or any other country." The permanent disability compensation of
US$60,000 clearly amounts to reasonable compensation for the injuries and loss of earning capacity of the
seafarer

San Beda College of Law 209


4S: 2015 - 2016
LABOR LAW REVIEW Atty. Joyrich Golangco

96. CARLO F. SUNGA, v. VIRJEN SHIPPING CORPORATION, NISSHO ODYSSEY SHIP MANAGEMENT
PTE. LTD., and/or CAPT. ANGEL ZAMBRANO,
G.R. No. 198640 April 23, 2014
BRION, J.:

DISABILITY BENEFITS

DOCTRINE:
An accident pertains to an unforeseen event in which no fault or negligence attaches to the defendant. It is "a
fortuitous circumstance, event or happening; an event happening without any human agency, or if happening
wholly or partly through human agency, an event which under the circumstances is unusual or unexpected by
the person to whom it happens."

FACTS:
Sunga was hired as fitter by Virjen Shipping Corporation (Virjen), acting in behalf of its foreign principal, Nissho
Odyssey Ship Management Pte. Ltd. While on board the MT Sunway vessel, Sunga started to experience an
on-and-off right flank pain, making it difficult for him to work. Dr. Cruz issued a medical certificate recommending
a Grade 8 disability based on the POEA Standard Employment Contract; and another recommending a disability
rating of 25% in accordance with the CBA. Based on these two certificates, Virjen offered US$ 16,795.00 in
accordance with the POEA-SEC. Sunga rejected the offer and demanded disability benefits pursuant to the
CBA. Virjen denied Sungas demand prompting the latter to file a complaint for disability benefits. Virjen claimed
that the CBA requires that for permanent disability to be compensable, the disability should be the result of an
accident incurred during the course of the seafarers employment. Virjen argued that Sunga failed to present
any proof that his disability was indeed the result of an accident.

LA RULING: In favour of Sunga. Ordered Virjen to pay US$110,000 pursuant to the CBA. The result of the MRI
revealed that Sunga had a herniated disc is already a manifestation that the injury resulted from an accident,
commonly incurred through falling or by lifting heavy objects.

NLRC RULING: Affirmed the LA

CA RULING: Reversed the NLRC. The injury was not accidental since carrying heavy objects can cause injury
and that lifting and carrying heavy objects are part of his duties as fitter. Thus, a back injury is reasonably
anticipated. It cannot serve as basis for Sunga to be entitled to disability benefits.

ISSUE: Is Sunga entitled to the benefits under the CBA?

SC RULING:
YES. Sunga did not incur the injury while solely performing his regular duties; an intervening event transpired
which brought upon the injury. To repeat, the two other oilers who were supposed to help carry the weight of the
200-kilogram globe valve lost their grasp of the globe valve. As a result, Sungas back snapped when the entire
weight of the item fell upon him. Notably, this incident cannot be considered as foreseeable, nor can it be
reasonably anticipated. Sungas duty as a fitter involved changing the valve, not to routinely carry a 200-
kilogram globe valve singlehandedly.

In Jarco Marketing Corporation, et al., v. Court of Appeals, we ruled that an accident pertains to an unforeseen
event in which no fault or negligence attaches to the defendant. It is "a fortuitous circumstance, event or
happening; an event happening without any human agency, or if happening wholly or partly through human
agency, an event which under the circumstances is unusual or unexpected by the person to whom it happens."

Since Sunga encountered an accident on board MT Sunway, the CA thus grossly misappreciated and misread
the ruling of the NLRC, leading the appellate court to find a grave abuse of discretion sufficient for a reversal of
the NLRC ruling. In other words, as the NLRC found, Sunga's disability benefits should fall within the coverage
of the parties' CBA.

San Beda College of Law 210


4S: 2015 - 2016
LABOR LAW REVIEW Atty. Joyrich Golangco

97. D. M. CONSUNJI, INC., v. COURT OF APPEALS and MARIA J. JUEGO
G.R. No. 137873 April 20, 2001
KAPUNAN, J.:

EXCEPTION TO THE WAIVER BY ELECTION

DOCTRINE:
An injured worker has a choice of either to recover from the employer the fixed amounts set by the Workmens
Compensation Act or to prosecute an ordinary civil action against the tortfeasor for higher damages but he
cannot pursue both courses of action simultaneously. However, if the choice of the first remedy was based on
ignorance or a mistake of fact, the choice is nullified as it was not an intelligent choice.

FACTS:
On Nov. 2, 1990, Jose Juego, a construction worker of D. M. Consunji, Inc., fell 14 floors from the Renaissance
Tower, resulting to his death. Jose Juegos widow, Maria, filed in the RTC a complaint for damages against the
deceaseds employer, D.M. Consunji, Inc. The employer raised, among other defenses, the widows prior
availment of the benefits from the State Insurance Fund. Petitioner argues that private respondent had
previously availed of the death benefits provided under the Labor Code and is, therefore, precluded from
claiming from the deceaseds employer damages under the Civil Code.

RTC RULING: Ruled in favour of Juego, awarding among others, damages.

CA RULING: Affirmed the RTC.

ISSUE: Is Juego precluded from recovering damages?

SC RULING:
No. An injured worker has a choice of either to recover from the employer the fixed amounts set by the
Workmens Compensation Act or to prosecute an ordinary civil action against the tortfeasor for higher damages
but he cannot pursue both courses of action simultaneously. However, if the choice of the first remedy was
based on ignorance or a mistake of fact, the choice is nullified as it was not an intelligent choice.

When a party having knowledge of the facts makes an election between inconsistent remedies, the election is
final and bars any action, suit, or proceeding inconsistent with the elected remedy, in the absence of fraud by
the other party. The choice of a party between inconsistent remedies results in a waiver by election. The
claimant, by his choice of one remedy, is deemed to have waived the other. However, ignorance of a material
fact negates waiver. Waiver cannot be established by a consent given under a mistake or misapprehension of
fact. That lack of knowledge of a fact that nullifies the election of a remedy is the basis for the exception.

It bears stressing that what negates waiver is lack of knowledge or a mistake of fact.
Private respondents case came under the exception because private respondent was unaware of petitioners
negligence when she filed her claim for death benefits from the State Insurance Fund. Private respondent filed
the civil complaint for damages after she received a copy of the police investigation report and the Prosecutors
Memorandum dismissing the criminal complaint against petitioners personnel.
There is also no showing that private respondent knew of the remedies available to her when the claim before
the ECC was filed. On the contrary, private respondent testified that she was not aware of her rights.

San Beda College of Law 211


4S: 2015 - 2016
LABOR LAW REVIEW Atty. Joyrich Golangco

98. THE HEIRS OF THE LATE DELFIN DELA CRUZ, REPRESENTED BY HIS SPOUSE, CARMELITA DELA
CRUZ v. PHILIPPINE TRANSMARINE CARRIERS, INC., REPRESENTED BY MR. CARLOS C. SALINAS
AND/OR TECTO BELGIUM N.V.
G.R. No. 196357 April 20, 2015
DEL CASTILLO, J.

THE 3-DAY MANDATORY REPORTING REQUIREMENT MUST BE STRICTLY OBSERVED.

DOCTRINE:
The 3-day mandatory reporting requirement must be strictly observed since within 3 days from repatriation, it
would be fairly manageable for the physician to identity whether the disease was contracted during the term of
his employment or that his working conditions increased the risk of contracting the ailment.

FACTS:
The late Delfin Dela Cruz was contracted for the position of Oiler by Philippine Transmarine Carriers, a local
manning agent for and in behalf of the latter's principal, Tecto Belgium N.V. Delfin was declared Fit for Sea
Servce and left the Philippines on 16 August 2000 and immediately embarked the vessel "Lady Hilde" on 17
August 2000.

While on board, he felt gradual chest pains and pain in his upper abdominal region. In 2001, while performing
his regular duties, he was hit by a metal board on his back. He, thereafter, requested medical attention and was
given medications and advised to be given light duties for the rest of the week. Upon the vessel's arrival at a
convenient port on 16 August 2001, his contract expired and he was signed off from the vessel. He reported to
respondents as required. He also sought medical assistance but was not extended such.

On 13 November 2003, Delfin sought for proper medical attention. Afterwards, he was not employed by
respondents because he was already incapacitated to engage in his customary work. He filed his claim for
sickness allowance from the same manning agency but the same was not granted. His condition deteriorated
and was later diagnosed to be suffering from malignant peripheral nerve sheath tumor [MPNST].

On 4 December 2003, he filed a complaint before the NLRC to, claim payment for sickness allowance and
disability compensation. Delfin averred that he is entitled to sickness allowance because his inability to work and
perform his usual occupation after he acquired the sickness while on board, lasted for more than 120 days.

Respondents, on the other hand, averred that the medical condition of Delfin was not acquired or suffered
during the term of his employment, that said medical condition is not work-related, and, therefore, the said
illness is not compensable under the POEA Standard Employment Contract. Furthermore, respondents
asseverated that more than two years had elapsed from the time of the termination of Delfin's employment in
August 2001 up to the time the claim was filed in November 2003, and thus the illness was not acquired during
the period of employment.

LA RULING: Delfin is ENTITLED to his claims. The LA opined that Delfin contracted his illness during the
period of his employment with respondents and that such illness is a compensable occupational disease.
Hence,

NLRC RULING: It REVERSED the LA decision.

CA RULING: AFFIRMED NLRC

ISSUE: Are petitioners, in behalf of the late Delfin Dela Cruz, entitled to permanent disability benefits and
sickness allowance?

SC RULING:
NO. The 1996 POEA SEC clearly provides that a seafarer must submit himself to a post-employment medical
examination within three days from his arrival in the Philippines (mandatory reporting requirement) so that his
claim for disability and sickness allowance can prosper. The 3-day mandatory reporting requirement must be
strictly observed since within 3 days from repatriation, it would be fairly manageable for the physician to identity
whether the disease was contracted during the term of his employment or that his working conditions increased
the risk of contracting the ailment.
San Beda College of Law 212
4S: 2015 - 2016
LABOR LAW REVIEW Atty. Joyrich Golangco

Whoever claims entitlement to the benefits provided by law should establish his right to the benefits by
substantial evidence" or "such relevant evidence as a reasonable mind might accept as adequate to support a
conclusion, even if other equally reasonable minds might conceivably opine otherwise." Absent a showing
thereof, any decision set forth will only be based on unsubstantiated allegations. Accordingly, the Court cannot
grant a claim for disability benefits without adequate substantiation for to do so will offend due process.

Petitioners failed to show the steps supposedly undertaken by Delfin to comply with the mandatory reporting
requirement. To the Court's mind, this lapse on petitioners' part only demonstrates that Delfin did not comply
with what was incumbent upon him. The reasonable conclusion, therefore, is that at the time of his repatriation,
Delfin was not suffering from any physical disability requiring immediate medical attendance. Otherwise, and
even if his request for medical assistance went unheeded, he would have submitted himself for check-up with
his personal physician. After all, the injury complained of by Delfin was a serious one and it would seem illogical
for him to just suffer in silence and bear the pain for a considerable length of time. Moreover, while the rule on
mandatory reporting requirement is not absolute as a seafarer may show that he was physically incapable to
comply with the same by submitting a written notice to the agency within the same three-day period, nowhere in
the records does it show that Delfin submitted any such notice. Clearly, petitioners failed to show that Delfin
complied with the mandatory reporting requirement. Thus, he is deemed to have forfeited his right to claim
disability benefits and sickness allowance.

San Beda College of Law 213


4S: 2015 - 2016
LABOR LAW REVIEW Atty. Joyrich Golangco

Social Security System (SSS) Law

99. SOCIAL SECURITY COMMISSION v. EDNA A. AZOTE


G.R. No. 209741 April 15, 2015
MENDOZA, J.

DESIGNATION OF BENEFICIARY MUST CONFORM TO THE STATUTE.

DOCTRINE:
Although an SSS member is free to designate a beneficiary, the designation must always conform to the
statute. To blindly rely on the form submitted by the deceased-member would subject the entire social security
system to the whims and caprices of its members and would render the SS Law inutile.

FACTS:
On June 19, 1992, respondent Edna and Edgardo, a member of the Social Security System (SSS), were
married. Their union produced six children. In 1994, Edgardo submitted Form E-4 to the SSS with Edna and
their three older children as designated beneficiaries. Thereafter or on September 7, 2001, Edgardo submitted
another Form E-4 to the SSS designating his three younger children as additional beneficiaries.

On January 13, 2005, Edgardo passed away. Shortly thereafter, Edna filed her claim for death benefits with the
SSS as the wife of a deceased-member. It appeared, however, from the SSS records that Edgardo had earlier
submitted another Form E-4 on November 5, 1982 with a different set of beneficiaries, namely: Rosemarie
Azote (Rosemarie), as his spouse; and Elmer Azote (Elmer), as dependent, born on October 9,
1982. Consequently, Ednas claim was denied. Her children were adjudged as beneficiaries and she was
considered as the legal guardian of her minor children. The benefits, however, would be stopped once a child
would attain the age of 21.

On March 13, 2007, Edna filed a petition with the SSC to claim the death benefits, lump sum and monthly
7
pension of Edgardo. She insisted that she was the legitimate wife of Edgardo. In its answer, the SSS averred
that there was a conflicting information in the forms submitted by the deceased. Summons was published in a
newspaper of general circulation directing Rosemarie to file her answer. Despite the publication, no answer was
filed and Rosemarie was subsequently declared in default.

SSC RULING: Edna is NOT ENTITLED to the benefits. The SSC dismissed Ednas petition for lack of
merit. Citing Section 24(c) of the SS Law, it explained that although Edgardo filed the Form E-4 designating
Edna and their six children as beneficiaries, he did not revoke the designation of Rosemarie as his wife-
beneficiary, and Rosemarie was still presumed to be his legal wife.

CA RULING: Reversed SSC decision.

ISSUE: Is respondent entitled to claim the SSS death benefit and pension of Edgardo?

SC RULING:
NO. Under R. A. No. 8282, the law in force at the time of Edgardos death, only the legal spouse of the
deceased-member is qualified to be the beneficiary of the latters SS benefits. In this case, there is a concrete
proof that Edgardo contracted an earlier marriage with another individual as evidenced by their marriage
contract and Edgardos acknowledgment of his married status when he filled out the 1982 Form E-4 designating
Rosemarie as his spouse.

The updated Form E-4 of Edgardo was not determinative of Ednas status and eligibility to claim the death
benefits of deceased-member. Although an SSS member is free to designate a beneficiary, the designation
must always conform to the statute. To blindly rely on the form submitted by the deceased-member would
subject the entire social security system to the whims and caprices of its members and would render the SS
Law inutile.

Although the SSC is not intrinsically empowered to determine the validity of marriages, it is required by Section
4(b) (7) of R.A. No. 8282 to examine available statistical and economic data to ensure that the benefits fall into
the rightful beneficiaries. The existence of two Form E-4s designating, on two different dates, two different
women as his spouse is already an indication that only one of them can be the legal spouse. It should be
San Beda College of Law 214
4S: 2015 - 2016
LABOR LAW REVIEW Atty. Joyrich Golangco

emphasized that the SSC determined Ednas eligibility on the basis of available statistical data and documents
on their database as expressly permitted by Section 4(b) (7) of R.A. No. 8282.

San Beda College of Law 215


4S: 2015 - 2016
LABOR LAW REVIEW Atty. Joyrich Golangco

100. SOCIAL SECURITY COMMISSION and SOCIAL SECURITY SYSTEM vs. TERESA G. FAVILA
G.R. No. 170195 March 28, 2011
DEL CASTILLO, J.

FACTORS FOR SPOUSE TO BE CONSIDERED AS PRIMARY BENEFICIARY

DOCTRINE:
A spouse who claims entitlement to death benefits as a primary beneficiary under the Social Security Law must
establish two qualifying factors, to wit: (1) that he/she is the legitimate spouse; and (2) that he/she is dependent
upon the member for support.

FACTS:
On August 5, 2002, respondent Teresa G. Favila (Teresa) filed a Petition before petitioner SSC. She averred
therein that after she was married to Florante Favila (Florante) on January 17, 1970, the latter designated her as
the sole beneficiary in the E-1 Form he submitted before petitioner Social Security System (SSS). When they
begot their children Jofel, Floresa and Florante II, her husband likewise designated each one of them as
beneficiaries. When Florante died on February 1, 1997, his pension benefits under the SSS were given to their
only minor child at that time, Florante II, but only until his emancipation at age 21. Believing that as the surviving
legal wife she is likewise entitled to receive Florantes pension benefits, Teresa subsequently filed her claim for
said benefits before the SSS.

In its Answer, SSS averred that the claim for Florantes pension benefits was initially settled in favor of Teresa
as guardian of the minor Florante II. SSS also alleged that Estelita Ramos, sister of Florante, wrote a letter
stating that her brother had long been separated from Teresa. She alleged therein that the couple lived together
for only ten years and then decided to go their separate ways because Teresa had an affair with a married man.

SSC RULING: The SSC ruled that she is DISQUALIFIED from claiming the death benefits because she was
deemed not dependent for support from Florante due to marital infidelity.

CA RULING: The CA REVERSED the SSC decision. It gave weight to the fact that she is a primary beneficiary
because she is the lawful surviving spouse of Florante and in addition, she was designated by Florante as such
beneficiary.

ISSUE:
Is Teresa a primary beneficiary in contemplation of the Social Security Law as to be entitled to death benefits
accruing from the death of Florante?

SC RULING:
NO. A spouse who claims entitlement to death benefits as a primary beneficiary under the Social Security Law
must establish two qualifying factors, to wit: (1) that he/she is the legitimate spouse; and (2) that he/she is
dependent upon the member for support.

There is no question that Teresa was Florantes legal wife. However, Teresa failed to show that despite their
separation she was dependent upon Florante for support at the time of his death. Aside from Teresas bare
allegation that she was dependent upon her husband for support and her misplaced reliance on the presumption
of dependency by reason of her valid and then subsisting marriage with Florante, Teresa has not presented
sufficient evidence to discharge her burden of proving that she was dependent upon her husband for support at
the time of his death. She could have done this by submitting affidavits of reputable and disinterested persons
who have knowledge that during her separation with Florante, she does not have a known trade, business,
profession or lawful occupation from which she derives income sufficient for her support and such other
evidence tending to prove her claim of dependency. While we note from the abovementioned SSS
Memorandum that Teresa submitted affidavits executed by Napoleon Favila and Josefina Favila, same only
pertained to the fact that she never remarried nor cohabited with another man. On the contrary, what is clear is
that she and Florante had already been separated for about 17 years prior to the latters death as Florante was
in fact, living with his common law wife when he died. Suffice it to say that "whoever claims entitlement to the
benefits provided by law should establish his or her right thereto by substantial evidence."

San Beda College of Law 216


4S: 2015 - 2016
LABOR LAW REVIEW Atty. Joyrich Golangco

101. ROMARICO J. MENDOZA vs. PEOPLE OF THE PHILIPPINES
G.R. No. 183891 August 3, 2010
CARPIO MORALES, J.

MANAGING HEAD- MEANING

DOCTRINE:
The term "managing head" in Section 28(f) is used, in its broadest connotation, not to any specific organizational
or managerial nomenclature.

FACTS:
An Information was filed against petitioner, being the proprietor of Summa Alta Tierra Industries, Inc. (SATII), for
failure and/or refusal to remit the SSS premium contributions in favor of its employees, in violation of Sec. 22(a)
and (d) in relation to Sec. 28 of Republic Act No. 8282, as amended.

The monthly premium contributions of SATII employees to SSS which petitioner admittedly failed to remit
covered the period August 1998 to July 1999 amounting to P421, 151.09 inclusive of penalties. After petitioner
was advised by the SSS to pay the above-said amount, he proposed to settle it over a period of 18
months which proposal the SSS approved.

Despite the grant of petitioners request for several extensions of time to settle the delinquency in installments,
petitioner failed, hence, his indictment.

Petitioner maintains that the managing head or president or general manager of a corporation is not among
those specifically mentioned as liable in the above-quoted Section 28(f). And he calls attention to an alleged
congenital infirmity in the Information in that he was charged as "proprietor" and not as director of SATII.

RTC RULING: Found Mendoza GUILTY for failure to remit the Social Security System (SSS) premium
contributions of employees of the SATII of which he was president.

CA RULING: AFFIRMED the RTC decision

ISSUE:
Is Mendoza guilty of violation of RA 8282 (SSS Law)?

SC RULING:
YES. Section 28(f) of the Act reads:
(f) If the act or omission penalized by this Act be committed by an association, partnership, corporation
or any other institution, its managing head, directors or partners shall be liable for the penalties
provided in this Act for the offense.

The provision of the law being clear and unambiguous, petitioners interpretation that a "proprietor," as he was
designated in the Information, is not among those specifically mentioned under Sec. 28(f) as liable, does not lie.
For the word connotes management, control and power over a business entity. No need to resort to statutory
construction for Section 28(f) of the Social Security Law imposes penalty on: (1) the managing head; (2)
directors; or (3) partners, for offenses committed by a juridical person. The term "managing head" in Section
28(f) is used, in its broadest connotation, not to any specific organizational or managerial nomenclature. To
heed petitioners reasoning would allow unscrupulous businessmen to conveniently escape liability by the
creative adoption of managerial titles.

San Beda College of Law 217


4S: 2015 - 2016
LABOR LAW REVIEW Atty. Joyrich Golangco

102. YOLANDA SIGNEY vs. SOCIAL SECURITY SYSTEM, EDITHA ESPINOSA-CASTILLO, and GINA
SERVANO, representative of GINALYN and RODELYN SIGNEY
G.R. No. 173582 January 28, 2008
TINGA, J.

QUALIFIED DEPENDENTS UNDER SSS LAW

DOCTRINE:
The dependent shall be the following:

(1) The legal spouse entitled by law to receive support from the member;
(2) The legitimate, legitimated, or legally adopted, and illegitimate child who is unmarried, not gainfully
employed and has not reached twenty-one years (21) of age, or if over twenty-one (21) years of age, he
is congenitally or while still a minor has been permanently incapacitated and incapable of self-support,
physically or mentally; and
(3) The parent who is receiving regular support from the member.

FACTS:
Rodolfo Signey, Sr., a member of the SSS, died on 21 May 2001. In his members records, he had designated
Yolanda Signey (petitioner) as primary beneficiary and his four children with her as secondary beneficiaries. On
6 July 2001, petitioner filed a claim for death benefits with the public respondent SSS. She revealed in her SSS
claim that the deceased had a common-law wife, Gina Servano (Gina), with whom he had two minor children
namey, Ginalyn Servano (Ginalyn), born on 13 April 1996, and Rodelyn Signey (Rodelyn), born on 20 April
2000.

Petitioners declaration was confirmed when Gina herself filed a claim for the same death benefits on 13 July
2001 in which she also declared that both she and petitioner were common-law wives of the deceased and that
Editha Espinosa (Editha) was the legal wife.

In addition, in October 2001, Editha also filed an application for death benefits with the SSS stating that she was
the legal wife of the deceased.

The SSS denied the death benefit claim of petitioner. Thereafter, petitioner filed a petition with the SSC in which
she attached a waiver of rights executed by Editha.

SSC RULING: DENIED the claim of petitioner Yolanda. The SSC gave more weight to the SSS field
investigation and the confirmed certification of marriage showing that the deceased was married to Editha on 29
October 1967, than to the aforestated declarations of Editha in her waiver of rights.

CA RULING: AFFIRMED the SSC decision.

ISSUE: Who is entitled to the social security benefits of a Social Security System (SSS) member who was
survived not only by his legal wife, but also by two common-law wives with whom he had six children?

SC RULING:
Ginalyn and Rodelyn, the minor children of the deceased with Gina.
The records disclosed that the deceased had one legitimate child, Ma. Evelyn Signey, who predeceased him,
and several illegitimate children with petitioner and with Gina. Based on their respective certificates of live birth,
the deceased SSS members four illegitimate children with petitioner could no longer be considered dependents
at the time of his death because all of them were over 21 years old when he died on 21 May 2001, the youngest
having been born on 31 March 1978. On the other hand, the deceased SSS members illegitimate children with
Gina were qualified to be his primary beneficiaries for they were still minors at the time of his death, Ginalyn
having been born on 13 April 1996, and Rodelyn on 20 April 2000.
Section 8(e) and (k) of R.A. No. 8282 provides:
SEC. 8. Terms Defined.For the purposes of this Act, the following terms shall, unless the context
indicates otherwise, have the following meanings:
xxx
(e) Dependents The dependent shall be the following:
(1) The legal spouse entitled by law to receive support from the member;
San Beda College of Law 218
4S: 2015 - 2016
LABOR LAW REVIEW Atty. Joyrich Golangco

2) The legitimate, legitimated, or legally adopted, and illegitimate child who is unmarried, not
gainfully employed and has not reached twenty-one years (21) of age, or if over twenty-one (21)
years of age, he is congenitally or while still a minor has been permanently incapacitated and incapable
of self-support, physically or mentally; and
3) The parent who is receiving regular support from the member.
xxx
(k) Beneficiaries The dependent spouse until he or she remarries, the dependent legitimate,
legitimated or legally adopted, and illegitimate children, who shall be the primary beneficiaries of the
member: Provided, That the dependent illegitimate children shall be entitled to fifty percent (50%) of the
share of the legitimate, legitimated or legally adopted children: Provided, further, That in the absence of
the dependent legitimate, legitimated or legally adopted children of the member, his/her dependent
illegitimate children shall be entitled to one hundred percent (100%) of the benefits. In their absence, the
dependent parents who shall be the secondary beneficiaries of the member. In the absence of all of
the foregoing, any other person designated by the member as his/her secondary beneficiary.
Whoever claims entitlement to the benefits provided by law should establish his or her right thereto by
substantial evidence. Since petitioner is disqualified to be a beneficiary and because the deceased has no
legitimate child, it follows that the dependent illegitimate minor children of the deceased shall be entitled to the
death benefits as primary beneficiaries. The SSS Law is clear that for a minor child to qualify as a "dependent,"
the only requirements are that he/she must be below 21 years of age, not married nor gainfully employed.
In this case, the minor illegitimate children Ginalyn and Rodelyn were born on 13 April 1996 and 20 April 2000,
respectively. Had the legitimate child of the deceased and Editha survived and qualified as a dependent under
the SSS Law, Ginalyn and Rodelyn would have been entitled to a share equivalent to only 50% of the share of
the said legitimate child. Since the legitimate child of the deceased predeceased him, Ginalyn and Rodelyn, as
the only qualified primary beneficiaries of the deceased, are entitled to 100% of the benefits.

San Beda College of Law 219


4S: 2015 - 2016
LABOR LAW REVIEW Atty. Joyrich Golangco

103. SSS v. TERESITA JARQUE VDA DE BAILON
G.R. No. 165545 March 24, 2006
CARPIO MORALES, J.:

DOCTRINE:
SSS and/or SSC has no jurisdiction to declare a marriage null and void.

FACTS:
1955, Clemente Bailon (Bailon) and Alice Diaz (Alice) contracted marriage in Barcelona, Sorsogon. After 15
year Alice Diaz was declared presumptively dead. 13 years after his wife Alice was declared presumptively
dead, Bailon contracted marriage with Teresita Jarque (respondent). After the death of Bailon, Jarque filed a
claim for funeral benefits, and was granted P12,000 by the SSS.

However, after coming to knowledge of the claim, Alice reappeared contesting the release of funeral benefits
and pension to Jarque asking that the benefits be granter to her as the lawful wife.

SSS RULING: SSS advised respondent of the cancellation of her monthly pension for death benefits and
requested respondent to return the monthly pension she had received from the SSS because her marriage with
Bailon was void as it was contracted while the latters marriage with Alice was still subsisting. Jarque then
elevated the decision to the SSC (Commission).

SSC RULING: By Resolution, the SSC found that the marriage of respondent to Bailon was void and, therefore,
she was "just a common-law-wife affirmed the decision of SSS.

CA RULING: Decision reversing that of SSC


According to the CA, SSS/SSC has no jurisdiction to declare the second marriage null and void on the basis
alone of its own investigation and declare that the decision of the RTC declaring one to be presumptively dead
is without basis. Respondent SSS cannot arrogate upon itself the authority to review the decision of the regular
courts under the pretext of determining the actual and lawful beneficiaries of its members.

ISSUE: Can the SSS and Commission validly declare the first marriage subsisting and the second marriage null
and void?

SC RULING:
No. Although SSC is empowered to settle any dispute with respect to SSS coverage, benefits and
contributions, in so exercising such power, however, it cannot review, much less reverse, decisions rendered by
courts of law as it did in the case at bar when it declared that the CFI Order was obtained through fraud and
subsequently disregarded the same, making its own findings with respect to the validity of Bailon and Alices
marriage on the one hand and the invalidity of Bailon and respondents marriage on the other.
In interfering with and passing upon the CFI Order, the SSC virtually acted as an appellate court. The law does
not give the SSC unfettered discretion to trifle with orders of regular courts in the exercise of its authority to
determine the beneficiaries of the SSS.

San Beda College of Law 220


4S: 2015 - 2016
LABOR LAW REVIEW Atty. Joyrich Golangco

Government Service Insurance System (GSIS) Law

104. GERSIP ASSOCIATION, INC., LETICIA ALMAZAN, ANGELA NARVAEZ, MARIA B. PINEDA, LETICIA
DE MESA AND ALFREDO D. PINEDA, v. GOVERNMENT INSURANCE SERVICE SYSTEM
G.R. No. 18982 October 16, 2013
VILLARAMA, J.:

DOCTRINE:
GRF creates a trust and not a co-ownership between the employees and GSIS.

FACTS:
GSIS Board of Trustees (GSIS Board) approved the proposed GSIS Provident Fund Plan (Plan) to provide
supplementary benefits to GSIS employees upon their retirement, disability or separation from the service, and
payment of definite amounts to their beneficiaries in the event of death. It likewise adopted the "Provident Fund
Rules and Regulations" (PFRR).

Under the Plan, employees who are members of the Provident Fund (Fund) contribute through salary deduction
a sum equivalent to five percent (5%) of their monthly salary while GSIS monthly contribution is fixed at 45% of
each members monthly salary.

Out of the earnings realized by the Fund, twenty percent (20%) of the proportionate earnings of GSIS
contributions is deducted and credited to a General Reserve Fund (GRF) and the remainder is credited to the
accounts of the members in proportion to the amounts standing to their credit at the beginning of each quarter.
Upon retirement, members are entitled to withdraw the entire amount of their contributions and proportionate
share of the accumulated earnings thereon, and 100% of respondents contributions with its proportionate
earnings.

GERSIP Association, Inc. (GERSIP), composed of retired GSIS employees and officers, wrote the President
and General Manager of respondent requesting the liquidation and partition of the General Reserve Fund
(GRF). Petitioners initially filed a civil suit before the RTC but on motion of respondent said case was dismissed
on the ground that it is the GSIS Board which has jurisdiction over the controversy. Petitioners filed a Petition
with the GSIS Board alleging that they have not been paid their portion of the GRF upon their retirement, to
which they are entitled as "co-owners" of the Fund.

Contention of GERSIP
(1) GSIS Provident Fund is an employee fringe benefit package incorporated in the (CBA), the members
own not only their personal contributions to the Fund but also 100% of GSIS management contributions
remitted in their names and for their benefit, plus all the earnings of both personal contributions and the
earnings of the management contribution, 20% of which is allotted by respondent to the GRF.

(2) Upon the remittance by GSIS of its contributions to the Fund, the same ceased to be part of
management funds but becomes part of the equity of the members for whom they were remitted as a
contractual obligation.

(3) Members are entitled also to that part of earnings from respondents contributions which are remitted to
the GRF, or at least the remaining balance thereof pertaining to the share of each member.

(4) GSIS has no legal title to the funds and it has no basis to impose any condition on how to avail of the
Fund benefits, or to refuse its accounting and audit.

GSIS RULING: This was affirmed by CA.

GSIS Board denied the petition for lack of merit. It held that the execution of the Trust Agreement between
respondent and the Committee is a clear indication that the parties intended to establish an express trust, not a
co-ownership, with respondent as Trustor, the Committee as Trustee of the Fund and the members as
Beneficiaries. As to the GRF, the Board said that it answers only for the contingent claims and there is no
requirement for the accounting and partition of GRF.

ISSUE: What is the nature of the funds contributed and its accumulated earnings under the Plan?
San Beda College of Law 221
4S: 2015 - 2016
LABOR LAW REVIEW Atty. Joyrich Golangco

SC RULING:
A provident fund is a type of retirement plan where both the employer and employee make fixed contributions.
Out of the accumulated fund and its earnings, employees receive benefits upon their retirement, separation from
service or disability.

The GSIS Provident Fund was established through Resolution No. 201 of the GSIS Board.1wphi1 The GSIS
Board likewise adopted a set of rules and regulations (PFRR) to govern the membership, fund contributions and
investment, payment of benefits and the trustees.

On July 23, 1981, a Trust Agreement was executed between respondent and the Committee. The latter was
tasked to administer, manage and invest the Fund, out of which it shall pay the benefits due to members or their
beneficiaries in accordance with the policies, rules and regulations approved by respondent. The Agreement
likewise explicitly declares:

SECTION 2. - The COMMITTEE OF TRUSTEES shall hold title and manage the FUND in trust
for the exclusive benefit of the members and their beneficiaries as provided for in the PLAN. No
part of the FUND shall be used for, or diverted to any purpose or purposes other than for the
exclusive benefits of such members and their beneficiaries. (Emphasis supplied.)

Respondents contention that it had thereby created an express trust was upheld by the GSIS Board
and the CA. The appellate court further ruled that the rules on co-ownership do not apply and there is
nothing in the PFRR that allows the distribution of the GRF in proportion to the members share therein.

We sustain the rulings of the GSIS Board and CA.

Trust is the legal relationship between one person having an equitable ownership in property and another
person owning the legal title to such property, the equitable ownership of the former entitling him to the
performance of certain duties and the exercise of certain powers by the latter. A trust fund refers to money or
property set aside as a trust for the benefit of another and held by a trustee.Under the Civil Code, trusts are
classified as either express or implied. An express trust is created by the intention of the trustor or of the parties,
while an implied trust comes into being by operation of
law.http://www.lawphil.net/judjuris/juri2013/oct2013/gr_189827_2013.html - fnt21
There is no doubt that respondent intended to establish a trust fund from the employees contributions (5% of
monthly salary) and its own contributions (45% of each members monthly salary and all unremitted Employees
Welfare contributions). We cannot accept petitioners submission that respondent could not impose terms and
conditions on the availment of benefits from the Fund on the ground that members already own respondents
contributions from the moment such was remitted to their account. Petitioners assertion that the Plan was a
purely contractual obligation on the part of respondent is likewise mistaken.

Republic Act No. 8291, otherwise known as "The Government Service Insurance System Act of 1997,"
mandated respondent to maintain a provident fund subject to rules and regulations it may adopt. Thus:
SECTION 41. Powers and Functions of the GSIS. The GSIS shall exercise the following
powers and functions:
xxxx
(s) to maintain a provident fund , which consists of contributions made by both the
GSIS and its officials and employees and their earnings, for the payment of benefits to
such officials and employees or their heirs under such terms and conditions as it may
prescribe; (Emphasis supplied.)

In Development Bank of the Philippines v. Commission on Audit,this Court recognized DBPs establishment of a
trust fund to cover the retirement benefits of certain employees. We noted that as the trustor, DBP vested in the
trustees legal title over the Fund as well as control over the investment of the money and assets of the Fund.
The Trust Agreement therein also stated that the principal and income must be used to satisfy all of the liabilities
to the beneficiary officials and employees under the Gratuity
Plan.http://www.lawphil.net/judjuris/juri2013/oct2013/gr_189827_2013.html - fnt23
Here, petitioners as beneficiaries of the Fund contend that they became co-owners of the entire Fund including
respondents contributions and its accumulated earnings. On this premise, they demand a proportionate share
in the GRF which was deducted from the earnings on respondents contributions.
San Beda College of Law 222
4S: 2015 - 2016
LABOR LAW REVIEW Atty. Joyrich Golangco

Under the PFRR, however, the GRF is allocated for specific purposes and not intended for distribution to
members. Section 8, Article IV thus provides:

Section 8. Earnings. At the beginning of each quarter, the earnings realized by the Fund in the
previous quarter just ended shall be credited to the accounts of the members in proportion to
the amounts standing to their credit as of the beginning of the same quarter after deducting
therefrom twenty per cent (20%) of the proportionate earnings of the Systems contributions,
which deduction shall be credited to a General Reserve Fund. Whenever circumstances
warrant, however, the Committee may reduce the percentage to be credited to the General
Reserve Fund for any given quarter; provided that in no case shall such percentage be lower
than five per cent (5%) of the proportionate earnings of the Systems contributions for the
quarter. When and as long as the total amount in the General Reserve Fund is equivalent to at
least ten per cent (10%) of the total assets of the Fund, the Committee may authorize all the
earnings for any given quarter to be credited to the members.

The General Reserve Fund shall be used for the following purposes:
(a) To cover the deficiency, if any, between the amount standing to the credit of a member who dies or is
separated from the service due to permanent and total disability, and the amount due him under Article
V Section 4;
(b) To make up for any investment losses and write-offs of bad debts, in accordance with policies to be
promulgated by the Board;
(c) To pay the benefits of separated employees in accordance with Article IV, Section 3; and (d) For other
purposes as may be approved by the Board, provided that such purposes is consistent with Article IV,
Section 4

It is clear that while respondents monthly contributions are credited to the account of each member,
and the same were received by petitioners upon their retirement, they were entitled to only a
proportionate share of the earnings thereon. The benefits of retiring members of the Fund are covered
by Section 1(b), Article V which states:

(b) Retirement. In the event the separation from the System is due to retirement under existing
laws, such as P.D. 1146, R.A. 660 or R.A. 1616, irrespective of the length of membership to
the Fund, the retiree shall be entitled to withdraw the entire amount of his contributions to the
Fund, as well as the corresponding proportionate share of the accumulated earnings thereon,
and in addition, 100% of the Systems contributions, plus the proportionate earnings thereon.

We find nothing illegal or anomalous in the creation of the GRF to address certain contingencies and ensure the
Funds continuing viability. Petitioners right to receive retirement benefits under the Plan was subject to well-
defined rules and regulations that were made known to and accepted by them when they applied for
membership in the Fund.

Petitioners have the right to demand for an accounting of the Fund including the GRF. Under Section 5, Article
VIII of the PFRR, the Committee is required to prepare an annual report showing the income and expenses and
the financial condition of the Fund as of the end of each calendar year. Said report shall be submitted to the
GSIS Board and shall be available to members. There is, however, no allegation or evidence that the
Committee failed to comply with the submission of such annual report, or that such report was not made
available to members.

San Beda College of Law 223


4S: 2015 - 2016
LABOR LAW REVIEW Atty. Joyrich Golangco

105. GOVERNMENT SERVICE INSURANCE SYSTEM, Petitioner, v. FERNANDO P. DE LEON,
G.R. No. 186560 November 17, 2010
NACHURA, J.:

DOCTRINE:
Disqualification from receiving retirement benefits under R.A. No. 910 does not mean that he is disqualified from
receiving any retirement benefit under any other existing retirement law.

FACTS:
Respondent Fernando P. de Leon retired as Chief State Prosecutor of the Department of Justice (DOJ), after 44
years of service to the government. He applied for retirement under Republic Act (R.A.) No. 910, invoking R.A.
No. 3783, as amended by R.A. No. 4140, which provides that chief state prosecutors hold the same rank as
judges. The application was approved by GSIS and for more than 9 years, respondent continuously received his
retirement benefits, until he failed to receive his monthly pension.

De Leon learned that GSIS cancelled the payment of his pension because the Department of Budget and
Management (DBM) informed GSIS that respondent was not qualified to retire under R.A. No. 910 since it only
applies to justices and judges not to prosecutors. Thus, GSIS stopped the payment of respondents monthly
pension. Because of the discontinuance of his pension, respondent sought to convert his retirement under R.A.
No. 910 to one under another law administered by GSIS .He then wrote a letter to GSIS regarding the
continuation of his pension.

GSIS RULING: Discontinuance of the pension. De Leon already retired and received benefits and pension
under Republic Act No. 910. However DBM already refused to release the funds for your pension benefit on the
ground that Chief State Prosecutors are not covered by R.A. 910. Since De Leon retired and received benefits
under the said law, he cannot seek to secure benefits under any other applicable GSIS law. There is nothing in
the GSIS law which sanctions double retirement unless the retiree is first re-employed and qualifies once again
to retire under GSIS law. In fact, Section 55 of Republic Act No. 8291 provides for exclusivity of benefits which
means that a retiree may choose only one retirement scheme available to him to the exclusion of all others.

CA RULING: The CA found that GSIS allowed respondent to retire under R.A. No. 910, following precedents
which allowed non-judges to retire under the said law. The CA said that it was not respondents fault that he was
allowed to avail of the benefits under R.A. No. 910; and that, even if his retirement under that law was
erroneous, respondent was, nonetheless, entitled to a monthly pension under the GSIS Act. The CA held that
this was not a case of double retirement, but merely a continuation of the payment of respondents pension
benefit to which he was clearly entitled. Since the error in the award of retirement benefits under R.A. 910 was
not attributable to respondent, it was incumbent upon GSIS to continue defraying his pension in accordance with
the appropriate law which might apply to him. It was unjust for GSIS to entirely stop the payment of respondents
monthly pension without providing any alternative sustenance to him.

ISSUE: Whether or not the GSIS can stop defraying the pension without specifying other pension scheme.

SC RULING:
NO.

(1) Retirement laws, in particular, are liberally construed in favor of the retiree because their objective is
to provide for the retirees sustenance and, hopefully, even comfort, when he no longer has the
capability to earn a livelihood. The liberal approach aims to achieve the humanitarian purposes of the law in
order that efficiency, security, and well-being of government employees may be enhanced.Indeed, retirement
laws are liberally construed and administered in favor of the persons intended to be benefited, and all doubts
are resolved in favor of the retiree to achieve their humanitarian purpose.

In this case, as adverted to above, respondent was able to establish that he has a clear legal right to the
reinstatement of his retirement benefits.

(2) In stopping the payment of respondents monthly pension, GSIS relied on the memorandum of the
DBM and because respondent had been mistakenly allowed to receive retirement benefits under R.A. No. 910,
GSIS erroneously concluded that respondent was not entitled to any retirement benefits at all, not even
under any other extant retirement law. This is flawed logic.
San Beda College of Law 224
4S: 2015 - 2016
LABOR LAW REVIEW Atty. Joyrich Golangco

Respondents disqualification from receiving retirement benefits under R.A. No. 910 does not mean that
he is disqualified from receiving any retirement benefit under any other existing retirement law.

(3) To grant respondent these benefits does not equate to double retirement, as GSIS mistakenly claims.
Since respondent has been declared ineligible to retire under R.A. No. 910, GSIS should simply apply the
proper retirement law to respondents claim, in substitution of R.A. No. 910. In this way, GSIS would be faithful
to its mandate to administer retirement laws in the spirit in which they have been enacted, i.e., to provide
retirees the wherewithal to live a life of relative comfort and security after years of service to the government.
Respondent will not receive --- and GSIS is under no obligation to give him --- more than what is due him under
the proper retirement law.

It must be emphasized that P.D. No. 1146 specifically mandates that a retiree is entitled to monthly pension for
life. As this Court previously held:

Considering the mandatory salary deductions from the government employee, the government pensions do not
constitute mere gratuity but form part of compensation.

In a pension plan where employee participation is mandatory, the prevailing view is that employees have
contractual or vested rights in the pension where the pension is part of the terms of employment. The reason for
providing retirement benefits is to compensate service to the government. Retirement benefits to government
employees are part of emolument to encourage and retain qualified employees in the government service.
Retirement benefits to government employees reward them for giving the best years of their lives in the service
of their country.

Thus, where the employee retires and meets the eligibility requirements, he acquires a vested right to benefits
that is protected by the due process clause. Retirees enjoy a protected property interest whenever they acquire
a right to immediate payment under pre-existing law. Thus, a pensioner acquires a vested right to benefits that
have become due as provided under the terms of the public employees pension statute. No law can deprive
such person of his pension rights without due process of law, that is, without notice and opportunity to be heard.

It must also be underscored that GSIS itself allowed respondent to retire under R.A. No. 910, following
jurisprudence laid down by this Court.

One could hardly fault respondent, though a seasoned lawyer, for relying on petitioners interpretation of the
pertinent retirement laws, considering that the latter is tasked to administer the governments retirement system.
He had the right to assume that GSIS personnel knew what they were doing.

Since the change in circumstances was through no fault of respondent, he cannot be prejudiced by the same.
His right to receive monthly pension from the government cannot be jeopardized by a new interpretation of the
law.

(4) GSIS argument that respondent has already been enormously benefited under R.A. No. 910 misses
the point.

Retirement benefits are a form of reward for an employees loyalty and service to the employer, and are
intended to help the employee enjoy the remaining years of his life, lessening the burden of having to worry
about his financial support or upkeep. A pension partakes of the nature of "retained wages" of the retiree for a
dual purpose: to entice competent people to enter the government service; and to permit them to retire from the
service with relative security, not only for those who have retained their vigor, but more so for those who have
been incapacitated by illness or accident.

Surely, giving respondent what is due him under the law is not unjust enrichment.

(5) As to GSIS contention that what respondent seeks is conversion of his retirement mode, which is
prohibited under R.A. No. 8291, the Court agrees with the CA that this is not a case of conversion within
the contemplation of the law. The conversion under the law is one that is voluntary, a choice to be made by
the retiree. Here, respondent had no choice but to look for another law under which to claim his pension benefits
because the DBM had decided not to release the funds needed to continue payment of his monthly pension.
San Beda College of Law 225
4S: 2015 - 2016
LABOR LAW REVIEW Atty. Joyrich Golangco

Respondent himself admitted that, if the DBM had not suspended the payment of his pension, he would not
have sought any other law under which to receive his benefits. The necessity to "convert" was not a voluntary
choice of respondent but a circumstance forced upon him by the government itself.

(6) Finally, GSIS would like this Court to believe that because it has returned respondents premium
contributions, it is now legally impossible for it to comply with the CAs directive.

Given the fact that respondent is ineligible to retire under R.A. No. 910, the refund by GSIS of respondents
premium payments was erroneous. Hence, GSIS can demand the return of the erroneous payment or it may opt
to deduct the amount earlier received by respondent from the benefits which he will receive in the future.
Considering its expertise on the matter, GSIS can device a scheme that will facilitate either the reimbursement
or the deduction in the most cost-efficient and beneficial manner.

The foregoing disquisition draws even greater force from subsequent developments. While this case was
pending, the Congress enacted Republic Act No. 10071, the Prosecution Service Act of 2010. By virtue of this
express provision, respondent is covered by R.A. No. 10071. In addition, he is now entitled to avail of the
benefits provided by Section 23, that "all pension benefits of retired prosecutors of the National Prosecution
Service shall be automatically increased whenever there is an increase in the salary and allowance of the same
position from which he retired."

Respondent, as former Chief State Prosecutor, albeit the position has been renamed "Prosecutor General,"
should enjoy the same retirement benefits as the Presiding Justice of the CA, pursuant to Section 14 of R.A. No.
10071, to wit:

Section 14. Qualifications, Rank and Appointment of the Prosecutor General. - The Prosecutor General shall
have the same qualifications for appointment, rank, category, prerogatives, salary grade and salaries,
allowances, emoluments, and other privileges, shall be subject to the same inhibitions and disqualifications, and
shall enjoy the same retirement and other benefits as those of the Presiding Justice of the Court of Appeals and
shall be appointed by the President.34

Furthermore, respondent should also benefit from the application of Section 16 of the law, which states:

Section 16. Qualifications, Ranks, and Appointments of Prosecutors, and other Prosecution Officers. x x x.

Any increase after the approval of this Act in the salaries, allowances or retirement benefits or any upgrading of
the grades or levels thereof of any or all of the Justices or Judges referred to herein to whom said emoluments
are assimilated shall apply to the corresponding prosecutors.

Lastly, and most importantly, by explicit fiat of R.A. No. 10071, members of the National Prosecution Service
have been granted the retirement benefits under R.A. No. 910, to wit:

Section 25. Applicability. - All benefits heretofore extended under Republic Act No. 910, as amended, and all
other benefits that may be extended by the way of amendment thereto shall likewise be given to the prosecutors
covered by this Act.

Hence, from the time of the effectivity of R.A. No. 10071, respondent should be entitled to receive retirement
benefits granted under R.A. No. 910.

Consequently, GSIS should compute respondents retirement benefits from the time the same were withheld
until April 7, 2010 in accordance with P.D. No. 1146; and his retirement benefits from April 8, 2010 onwards in
accordance with R.A. No. 910.

A final note. The Court is dismayed at the cavalier manner in which GSIS handled respondents claims,
keeping respondent in the dark as to the real status of his retirement benefits for so long. That the agency
tasked with administering the benefits of retired government employees could so unreasonably treat one of its
beneficiaries, one who faithfully served our people for over 40 years, is appalling. It is well to remind GSIS of its
mandate to promote the efficiency and welfare of the employees of our government, and to perform its tasks not
only with competence and proficiency but with genuine compassion and concern.
San Beda College of Law 226
4S: 2015 - 2016
LABOR LAW REVIEW Atty. Joyrich Golangco

San Beda College of Law 227


4S: 2015 - 2016
LABOR LAW REVIEW Atty. Joyrich Golangco

106. GOVERNMENT SERVICE INSURANCE SYSTEM v. MARILOU ALCZARAZ
G.R. No. 187474 February 06, 2013
BRION, J.:

DOCTRINE:
It is not necessary that the disease causing the death of the employee be directly connected to the work.
Substantial evidence that the development of the disease is brought largely by the conditions present in the
nature of the job is sufficient.

FACTS:
Bernardo was employed for almost twenty-nine (29) years by the (MMDA) in Makati City. He worked at the
MMDA as labourer. Bernardo was diagnosed with Pulmonary Tuberculosis (PTB) and Community Acquired
Pneumonia (CAP). He was discharged on May 19, 2004 with the following diagnosis: Acute Diffuse
Anterolateral Wall Myocardial Infarction. A year Bernardo was found dead at the basement of the MMDA
building. His body was brought to the Southern Police District Crime Laboratory in Makati City for an autopsy.
Medico-Legal Officer Ma. Cristina B. Freyra performed the autopsy and concluded that Bernardo died of
Myocardial Infarction, old and recent.Bernardos widow, Marilou, subsequently filed a claim for death benefits
with the GSIS.

GSIS RULING: The GSIS denied the claim for death benefits on the ground that myocardial infarction, the
cause of Bernardos death, was directly related to diabetes which is not considered a work-connected illness;
hence, its complications, such as myocardial infarction, are not work-related. This decision of GSIS was affirmed
by ECC. The GSIS insists that myocardial infarction which caused Bernardos death cannot be said to have
been aggravated by the nature of his duties. It stresses that on the contrary, there was no evidence showing
that it was the performance of his duties that caused the development of myocardial infarction as it was a mere
complication of diabetes mellitus, a non-occupational disease. His heart ailment, therefore, cannot be
considered an occupational disease.

CA RULING: The CA granted the petition and set aside the ECC ruling. It pointed out that, as this Court held in
Salmone v. Employees Compensation Commission, "[t]he claimant must show, at least, by substantial evidence
that the development of the disease is brought largely by the conditions present in the nature of the job."

The CA found sufficient proof of work-connection between Bernardos ailment and his working conditions. It
believed that his work as laborer and metro aide must have substantially contributed to his illness.

The CA ordered the GSIS to pay Bernardos heirs the proper benefits for his death consistent with the State
policy to extend the applicability of the employees compensation law, Presidential Decree No. 626, to a greater
number of employees who can avail of the benefits under the law, in consonance with the avowed policy of the
State to give maximum aid and protection to labor.

ISSUE: Whether or not the indirect relation of the cause of death of Bernardo to his work negates the award of
benefits to his dependents.

SC RULING:
No. Diabetes mellitus not the sole predisposing factor to myocardial infarction
Bernardo died after almost three decades of service with the MMDA (July 1, 1976 to January 15, 2005). His
death occurred within his employers premises, at the basement of the MMDA building while he was at work.
The GSIS and the ECC denied the claim of his widow for death benefits on the ground that his death was due to
myocardial infarction which they declared to be non-compensable; they opined that it is not work-related as it is
simply a complication of diabetes mellitus. They pointed out that diabetes mellitus is not in the list of
occupational diseases and, for this reason, its complications such as myocardial infarction, are not work-related.

We disagree with the GSISs position. The conclusions of the two agencies totally disregarded the stressful
and strenuous conditions under which Bernardo toiled for almost 29 long years as a laborer and as a metro
aide. By so doing, they closed the door to other influences that caused or contributed to Bernardos fatal heart
problem an ailment aggravated with the passage of time by the risks present in the difficult working conditions
that Bernardo had to bear from day to day in his employment.

San Beda College of Law 228


4S: 2015 - 2016
LABOR LAW REVIEW Atty. Joyrich Golangco

The CA vividly captured Bernardos hazardous working environment (the streets of Makati City) and its effects
on his health when it stated:

Petitioner contends that the ECC erred in ruling that petitioner is not entitled to claim benefits
for her husbands death. She pointed out that as early as May 3, 2004, the deceased was
already complaining of shortness of breath and dizziness; that despite such condition, he still
continued performing his work until he was confined at the Ospital ng Makati from May l3 to
19, 2004 where he was diagnosed with Acute Diffuse Anterlateral Wall Myocardial Infarction;
that the short intervening period between his confinement at the hospital and his last day of
duty with the MMDA on January 14, 2005, indicate that he had been suffering from such
disease at the time that he was employed; that his [everyday] exposure under the sweltering
heat of the sun during summer and his constant exposure to rain during the rainy season,
aggravated by his contact to smoke emitted by vehicles passing as he cleaned the streets of
Makati, are enough proofs of the strenuous nature of his work; that his everyday exposure to
these elements not only resulted to his developing myorcardial infarction, but also aggravated
pre-existing illness which were pulmonary tuberculosis and community acquired pneumonia.
http://www.lawphil.net/judjuris/juri2013/feb2013/gr_187474_2013.html - fnt14
While diabetes mellitus was indeed a complicating factor in Bernardos health condition and indisputably
aggravated his heart problem, we cannot discount other employment factors, mental and physical, that had
been indisputably present; they contributed, if not as a direct cause of the heart condition itself, as aggravation
that worsened and hastened his fatal myocardial infarction.

For instance, it is undisputed that Bernardo was earlier diagnosed with CAP which could also be a predisposing
factor to myocardial infarction.There is also stress due to the nature of Bernardos work. As Marilou pointed out,
this Court recognized that stress could influence the onset of myocardial infarction.1wphi1 The Court declared
inGoverment Service Insurance System (GSIS) v. Cuanang:"Myocardial infarction, also known as coronary
occlusion or just a coronary, is a life threatening condition. Predisposing factors for myocardial infarction are
the same for all forms of Coronary Artery Disease, and these factors include stress. Stress appears to be
associated with elevated blood pressure."

The CA, therefore, is correct in holding that there is substantial evidence supporting the conclusion that
myocardial infarction in Bernardos case is work-related.

Cardio-vascular disease compensable


The CAs conclusion is bolstered by the fact that the ECC itself, the government agency tasked by law to
implement the employees compensation program (together with the GSIS in the public sector and the Social
Security System [SSS] in the private sector), included cardio-vascular diseases in the list of occupational
diseases, making them compensable, subject to any of the conditions stated in its enabling Resolution No.
432.With the resolution, it should be obvious that by itself, a heart disease, such as myocardial infarction, can be
considered work-related, with or without the complicating factors of other non-occupational illnesses. Thus, the
20
Court so ruled in Raises v. ECC, where it emphasized that the incidence of acute myocardial infarction,
whether or not associated with a non-listed ailment, is enough basis for compensation.

Resolution No. 432 provides (as one of the conditions) that a heart disease is compensable if it was known to
have been present during employment, there must be proof that an acute exacerbation was clearly precipitated
by the unusual strain by reason of the nature of his work. Based on the evidence on record, we find as the
CA did, that the nature of Bernardos duties and the conditions under which he worked were such as to
eventually cause the onset of his myocardial infarction. The stresses, the strain, and the exposure to street
pollution and to the elements that Bernardo had to bear for almost 29 years are all too real to be ignored. They
cannot but lead to a deterioration of health particularly with the contributing factors of diabetes and pulmonary
disease.

Bernardo had in fact been a walking time bomb ready to explode towards the end of his employment days.
Records show that the debilitating effect of Bernardos working conditions on his health manifested itself several
months before his death. As early as May 3, 2004, Bernardo was already complaining of shortness of breath
and dizziness. From May 13 to 19, 2004, he had to be confined at the Ospital ng Makati and was diagnosed
with acute myocardial infarction which caused his death on January 15, 2005 while he was at work. To be sure,
a reasonable mind analyzing these facts cannot but arrive at the conclusion that the risks present in his work

San Beda College of Law 229


4S: 2015 - 2016
LABOR LAW REVIEW Atty. Joyrich Golangco

environment for the entire duration of his employment precipitated the acute myocardial infarction that led to his
death.

We thus find no merit in the petition. The CA committed no reversible error nor any grave abuse of discretion in
awarding death benefits to Bernardos heirs. As a final point, we take this occasion to reiterate that as an
agency charged by law with the implementation of social justice guaranteed and secured by the Constitution
the ECC (as well as the GSIS and the SSS) should adopt a liberal attitude in favor of the employees in
deciding claims for compensability, especially where there is some basis in the facts for inferring a work-
connection to the accident or to the illness. This is what the Constitution dictates.

San Beda College of Law 230


4S: 2015 - 2016
LABOR LAW REVIEW Atty. Joyrich Golangco

ARTICLE 212 (now Art. 219) Definition of Labor Dispute

107. CITIBANK, N. A. v. CA (Third Division), AND CITIBANK INTEGRATED GUARDS LABOR ALLIANCE
(CIGLA) SEGATUPAS/FSM LOCAL CHAPTER No. 1394
G.R. No. 108961 November 27, 1998
PARDO, J.:

DOCTRINE:
Non-renewal of Security Guard Service agreement is a civil dispute and not a labor dispute.

FACTS:
Citibank and El Toro Security Agency, Inc. (hereafter El Toro) entered into a contract for the latter to provide
security and protective services. In 1990, the contract between Citibank and El Toro expired.
Integrated Guards Labor Alliance-SEGA-TUPAS/FSM (hereafter CIGLA) filed with the National Conciliation and
Mediation Board (NCMB) a request for preventive mediation citing Citibank as respondent therein giving as
issues for preventive mediation the following: (1) Unfair labor practice (2) Dismissal of union officers/members;
and (3) Union busting.

Three days after, Citibank served on El Toro a written notice that the bank would not renew anymore the service
agreement with the latter. Simultaneously, Citibank hired another security agency, the Golden Pyramid Security
Agency, to render security services at Citibank's premises.

Hence, CIGLA filed a manifestation with the NCMB that it was converting its request for preventive mediation
into a notice of strike for failure of the parties to reach a mutually acceptable settlement of the issues, which it
followed with a supplemental notice of strike alleging as supplemental issue the mass dismissal of all union
officers and members.

The following day the guards of El Toro were replaced by guards of the Golden Pyramid Security Agency. They
threatened to go on strike against Citibank and picket its premises. CIGLA filed a notice of strike directed at the
premises of the Citibank main office.

Citibank filed with the Regional Trial Court, Makati, a complaint for injunction and damages to which respondent
CIGLA filed with the trial court a motion to dismiss the complaint. The motion alleged that the Court had no
jurisdiction, this being labor dispute.

RTC RULING: The trial court denied respondent CIGLA's motion to dismiss because plaintiff's complaint there
are allegations, which negate any employer-employee relationship between it and the CIGLA members.
Respondent CIGLA filed with the Court of Appeals a petition for certiorari with preliminary injunction assailing
the validity of the proceedings had before the regional trial court.

CA RULING: It declared the proceedings before the RTC null and void.

ISSUE:
(1) The basic issue involved is whether it is the labor tribunal or the regional trial court that has jurisdiction over
the subject matter of the complaint filed by Citibank with the trial court.

(2) Is there a labor dispute between Citibank and the security guards, members of respondent CIGLA,
regardless of whether they stand in the relation of employer and employees?

SC RULING:
(1) Yes.
The Court sustained the petitioner's contention. This Court has held in many cases that "in determining the
existence of an employer-employee relationship, the following elements are generally considered: 1) the
selection and engagement of the employee; 2) the payment of wages; 3) the power of dismissal; and 4) the
employer's power to control the employee with respect to the means and methods by which the work is to be
6
accomplished". It has been decided also that the Labor Arbiter has no jurisdiction over a claim filed where no
employer-employee relationship existed between a company and the security guards assigned to it by a security
7
service contractor. In this case, it was the security agency El Toro that recruited, hired and assigned the

San Beda College of Law 231


4S: 2015 - 2016
LABOR LAW REVIEW Atty. Joyrich Golangco

watchmen to their place of work. It was the security agency that was answerable to Citibank for the conduct of
its guards.

2. No. It is a civil dispute.


Article 212, paragraph l of the Labor Code provides the definition of a "labor dispute". It "includes any
controversy or matter concerning terms or conditions of employment or the association or representation of
persons in negotiating, fixing, maintaining, changing or arranging the terms and conditions of employment,
regardless of whether the disputants stand in the proximate relation of employer and employee."

If at all, the dispute between Citibank and El Toro security agency is one regarding the termination or non-
8
renewal of the contract of services. This is a civil dispute . El Toro was an independent contractor. Thus, no
employer-employee relationship existed between Citibank and the security guard members of the union in the
security agency who were assigned to secure the bank's premises and property. Hence, there was no labor
dispute and no right to strike against the bank.

It is a basic rule of procedure that "jurisdiction of the court over the subject matter of the action is determined by
the allegations of the complaint, irrespective of whether or not the plaintiff is entitled to recover upon all or some
of the claims asserted therein. The jurisdiction of the court can not be made to depend upon the defenses set up
in the answer or upon the motion to dismiss, for otherwise, the question of jurisdiction would almost entirely
9
depend upon the defendant." "What determines the jurisdiction of the court is the nature of the action pleaded
as appearing from the allegations in the complaint. The averments therein and the character of the relief sought
are the ones to be consulted."

In the complaint filed with the trial court, petitioner alleged that in 1983, it entered into a contract with El Toro, a
security agency, for security and protection service. The parties renewed the contract yearly until April 22, 1990.
Petitioner further alleged that from June 11, 1990, until the filing of the complaint, El Toro security guards
formerly assigned to guard Citibank premises loitered around the bank's premises in large groups and
threatened to stage a strike, which would hamper its operations and the normal conduct of its business and that
the bank would suffer damages should a strike push through.

On the basis of the allegations of the complaint, it is safe to conclude that the dispute involved is a civil one, not
a labor dispute. Consequently, we rule that jurisdiction over the subject matter of the complaint lies with the
regional trial court.

San Beda College of Law 232


4S: 2015 - 2016
LABOR LAW REVIEW Atty. Joyrich Golangco

108. PHILIPPINE AIRLINES, INC. vs. NATIONAL LABOR RELATIONS COMMISSION, FERDINAND PINEDA
and GODOFREDO CABLING
G.R. No. 120567 20 March 1998
Martinez, J.:

DEFINITION OF A LABOR DISPUTE

DOCTRINE:
The power of the NLRC to issue an injunctive writ originates from "any labor dispute. The term "labor dispute" is
defined as "any controversy or matter concerning terms and conditions of employment or the association or
representation of persons in negotiating, fixing, maintaining, changing, or arranging the terms and conditions of
employment regardless of whether or not the disputants stand in the proximate relation of employers and
employees. There is no labor dispute when there has yet been no complaint for illegal dismissal filed with the
labor arbiter.

FACTS:
Ferdinand Pineda and Godofredo Cabling, flight stewards of PAL, were dismissed by the latter from the service
for their alleged involvement in the currency smuggling in Hong Kong. Aggrieved by said dismissal, they went
directly to the NLRC and filed a petition for injunction with the object of making PAL withhold its orders of
dismissal and reinstate them to work. The NLRC granted their petition.

Displeased, PAL challenged the NLRC through a motion for reconsideration questioning its jurisdiction to issue
an injunction or restraining order since this may be issued only under Article 218 of the Labor Code if the case
involves or arises from labor disputes.

NLRC RULING: It denied PALs motion for reconsideration and upheld its jurisdiction to issue the mandatory
injunctive writ ordering PAL to withhold the enforcement of the orders of dismissal and reinstate Pineda and
Cabling.

ISSUE: Can the NLRC, even without a complaint for illegal dismissal filed before the labor arbiter, entertain an
action for injunction and issue such writ?

SC RULING:
NO. Generally, injunction is not a cause of action in itself but merely a provisional remedy, an adjunct to a main
suit. Relative to this, the power of the NLRC to issue an injunctive writ originates from "any labor dispute.

The term "labor dispute" is defined as "any controversy or matter concerning terms and conditions of
employment or the association or representation of persons in negotiating, fixing, maintaining, changing, or
arranging the terms and conditions of employment regardless of whether or not the disputants stand in the
proximate relation of employers and employees."

The term "controversy" is likewise defined as "a litigated question; adversary proceeding in a court of law; a civil
action or suit, either at law or in equity; a justiciable dispute."

A "justiciable controversy" is "one involving an active antagonistic assertion of a legal right on one side and a
denial thereof on the other concerning a real, and not a mere theoretical question or issue."

From the foregoing definitions, it is therefore an essential requirement that there must first be a labor dispute
between the contending parties before the labor arbiter.

In the present case, there is no labor dispute between PAL and respondents Pineda and Cabling as there has
yet been no complaint for illegal dismissal filed with the labor arbiter by them against the PAL. The petition for
injunction directly filed before the NLRC is in reality an action for illegal dismissal. This is clear from the
allegations in the petition which prays for their reinstatement; award of full backwages, moral and exemplary
damages; and attorney's fees. As such, the petition should have been filed with the labor arbiter who has the
original and exclusive jurisdiction to hear and decide the following cases involving all workers, whether
agricultural or non-agricultural.

San Beda College of Law 233


4S: 2015 - 2016
LABOR LAW REVIEW Atty. Joyrich Golangco

Managerial Employee

109. CHARLITO PEARANDA v. BAGANGA PLYWOOD CORPORATION and HUDSON CHUA


G.R. No. 159577 3 May 2006
Panganiban, C.J.:

MEMBERS OF THE MANAGERIAL STAFF

DOCTRINE:
Members of the managerial staff are those who customarily and regularly exercise discretion and independent
judgment. Members of the managerial staff are exempted from the provisions of the Labor Code on labor
standards.

FACTS:
Charlito Pearanda was hired as an employee of Baganga Plywood Corporation (BPC) to take charge of the
operations and maintenance of its steam plant boiler. Subsequently, Pearanda filed a Complaint for illegal
dismissal with money claims against BPC and its general manager, Hudson Chua, before the NLRC. Pearanda
claims, among others, that he was not a managerial employee, and therefore, entitled to the award granted by
the labor arbiter.

LA RULING: There was no illegal dismissal and that petitioners Complaint was premature because he was still
employed by BPC. The temporary closure of BPCs plant did not terminate his employment; hence, he need not
reapply when the plant reopened. Nevertheless, the labor arbiter found Pearanda entitled to overtime pay,
premium pay for working on rest days

NLRC RULING: Deleted the award of overtime pay and premium pay for working on rest days. According to the
Commission, petitioner was not entitled to these awards because he was a managerial employee.

CA RULING: Denied Pearandas petition on purely procedural grounds, which prompted him to seek recourse
with the SC.

ISSUE: Is Pearanda a managerial employee?

SC RULING:
NO. He was a member of the managerial staff. The Implementing Rules of the Labor Code define members of a
managerial staff as those who customarily and regularly exercise discretion and independent judgment.

As borne out by the facts, Pearanda supervised the engineering section of the steam plant boiler. His work
involved overseeing the operation of the machines and the performance of the workers in the engineering
section. This work necessarily required the use of discretion and independent judgment to ensure the proper
functioning of the steam plant boiler. As supervisor, he is deemed a member of the managerial staff.

Members of the managerial staff are exempted from the provisions of the Labor Code on labor standards. Since
Pearanda belongs to this class of employees, he is not entitled to overtime pay and premium pay for working
on rest days.

San Beda College of Law 234


4S: 2015 - 2016
LABOR LAW REVIEW Atty. Joyrich Golangco

110. SAMAHANG MANGGAGAWA SA CHARTER CHEMICAL SOLIDARITY OF UNIONS IN THE
PHILIPPINES FOR EMPOWERMENT AND REFORMS (SMCC-SUPER), ZACARRIAS JERRY VICTORIO-
Union President v. CHARTER CHEMICAL and COATING CORPORATION
G.R. No. 169717 16 March 2011
Del Castillo, J.:

MANAGERIAL EMPLOYEES

DOCTRINE:
After a labor organization has been registered, it may exercise all the rights and privileges of a legitimate labor
organization. Any mingling between supervisory and rank-and-file employees in its membership cannot affect its
legitimacy for that is not among the grounds for cancellation of its registration, unless such mingling was brought
about by misrepresentation, false statement or fraud under Article 239 of the Labor Code.

FACTS:
Samahang Manggagawa sa Charter Chemical Solidarity of Unions in the Philippines for Empowerment and
Reforms (petitioner union) filed a petition for certification election among the regular rank-and-file employees of
Charter Chemical and Coating Corporation (respondent company) with the Mediation Arbitration Unit of the
DOLE, National Capital Region.

Respondent company filed an Answer with Motion to Dismiss on the ground that petitioner union is not a
legitimate labor organization because of (1) failure to comply with the documentation requirements set by law,
and (2) the inclusion of supervisory employees within petitioner union.

MED-ARBITER RULING: Sided with the company.

DOLE RULING: Granted the unions petition for a certification election.

CA RULING: Reversed the DOLE and upheld the Med-Arbiters Ruling.

ISSUE: Does the commingling of supervisory and rank-and-file employees in a union divest it of its personality
as a legitimate labor organization?

SC RULING:
NO. After a labor organization has been registered, it may exercise all the rights and privileges of a legitimate
labor organization. Any mingling between supervisory and rank-and-file employees in its membership cannot
affect its legitimacy for that is not among the grounds for cancellation of its registration, unless such mingling
was brought about by misrepresentation, false statement or fraud under Article 239 of the Labor Code.

Applying this principle to the case at bar, petitioner union was not divested of its status as a legitimate labor
organization even if some of its members were supervisory employees. It had the right to file the subject petition
for certification election. Besides, the legal personality of the union cannot be collaterally attacked by the
company in the certification election proceedings the latter being in the eyes of the law a mere bystander in
such proceedings.

111. PAMELA FLORENTINA JUMUAD, Petitioner v. HI-FLYER FOOD, INC. and/or JESUS R.
MONTEMAYOR
G.R. No. 187877 September 2011
Mendoza, J.:

MANAGERIAL EMPLOYEES

DOCTRINE:

As long as there is some basis for loss of confidence, such as when the employer has reasonable ground to
believe that the employee concerned is responsible for the purported misconduct, and the nature of his
participation therein renders him unworthy of the trust and confidence demanded of his position, a managerial
employee may be dismissed.

San Beda College of Law 235


4S: 2015 - 2016
LABOR LAW REVIEW Atty. Joyrich Golangco

FACTS:
Pamela Florentina Jumuad was employed as Area Manager in Visayas by Hi-Flyer, Inc., the company managing
Kentucky Fried Chicken stores throughout the country. Later on, the company discovered lapses on the part of
Jumuad in doing her job. Jumuad was given the opportunity to explain the reason these. Nonetheless, the
company still terminated her employment on the ground of neglect of duty and breach of trust and confidence.
This prompted Jumuad to file a complaint against Hi-Flyer for illegal dismissal.

LA RULING: After finding that no serious cause for termination existed, the LA ruled that Jumuad was illegally
dismissed.

NLRC RULING: Affirmed the LA

CA RULING: Reversed the NLRC. CA was of the opinion that the requirements of substantive and procedural
due process were complied with affording Jumuad an opportunity to be heard first, when she submitted her
written explanation and then, when she was informed of the decision and the basis of her termination.

ISSUE: Was Jumuad Illegally dismissed?

SC RULING:
NO. As long as there is some basis for loss of confidence, such as when the employer has reasonable ground
to believe that the employee concerned is responsible for the purported misconduct, and the nature of his
participation therein renders him unworthy of the trust and confidence demanded of his position, a managerial
employee may be dismissed.

Here, there is ample evidence that Jumuad indeed committed acts justifying loss of trust and confidence of Hi-
Flyer, which resulted to her dismissal from service. Her mismanagement and negligence in supervising the
effective operation of KFC branches in the span of less than a year, resulting in the closure of KFC-Gaisano due
to deplorable sanitary conditions, cash shortages in KFC-Bohol, in which the said branch, at the time of
discovery, was only several months into operation, and the poor sanitation at KFC-Cocomall. The glaring fact
that three (3) out of the seven (7) branches under her area were neglected cannot be glossed over by her
explanation that there was no negligence on her part as the sanitation problem was structural, that she had
been usually busy conducting management team meetings in several branches of KFC in her area or that she
had no participation whatsoever in the alleged cash shortages.

As the employer, Hi-Flyer has the right to regulate, according to its discretion and best judgment, all aspects of
employment, including work assignment, working methods, processes to be followed, working regulations,
transfer of employees, work supervision, lay-off of workers and the discipline, dismissal and recall of workers.

San Beda College of Law 236


4S: 2015 - 2016
LABOR LAW REVIEW Atty. Joyrich Golangco

ARTICLE 217 (now Art. 224) Jurisdiction of Labor Arbiters

112. PEOPLES BROADCASTING SERVICE (BOMBO RADYO PHILS., INC.) v. THE SECRETARY OF THE
DEPARTMENT OF LABOR AND EMPLOYMENT, THE REGIONAL DIRECTOR, DOLE REGION VII, and
JANDELEON JUEZAN
G.R. No. 179652 6 March 2012
Velasco, Jr., J.:

JURISDICTION OF THE LABOR ARBITER

DOCTRINE:
If a complaint is brought before the DOLE to give effect to the labor standards provisions of the Labor Code or
other labor legislation, and there is a finding by the DOLE that there is an existing employer-employee
relationship, the DOLE exercises jurisdiction to the exclusion of the NLRC. If the DOLE finds that there is no
employer-employee relationship, the jurisdiction is properly with the NLRC. If a complaint is filed with the DOLE,
and it is accompanied by a claim for reinstatement, the jurisdiction is properly with the Labor Arbiter, under Art.
217(3) of the Labor Code, which provides that the Labor Arbiter has original and exclusive jurisdiction over
those cases involving wages, rates of pay, hours of work, and other terms and conditions of employment, if
accompanied by a claim for reinstatement. If a complaint is filed with the NLRC, and there is still an existing
employer-employee relationship, the jurisdiction is properly with the DOLE. The findings of the DOLE, however,
may still be questioned through a petition for certiorari under Rule 65 of the Rules of Court.

FACTS:
Jandeleon Juezan filed a complaint against petitioner with the Department of Labor and Employment (DOLE),
for illegal deduction, nonpayment of service incentive leave, 13th month pay, premium pay for holiday and rest
day and illegal diminution of benefits, delayed payment of wages and noncoverage of SSS, PAG-IBIG and
Philhealth. The DOLE Regional Director found that private respondent was an employee of petitioner, and was
entitled to his money claims.

When the matter was brought before the CA it was held that PBS was accorded due process as it had been
given the opportunity to be heard, and that the DOLE Secretary had jurisdiction over the matter, as the
jurisdictional limitation imposed by Article 129 of the Labor Code on the power of the DOLE Secretary under Art.
128(b) of the Code had been repealed by Republic Act No. (RA) 7730.

However, the SC found that there was no employer-employee relationship between PBS and and private respo.
It was held that while the DOLE may make a determination of the existence of an employer-employee
relationship, this function could not be co-extensive with the visitorial and enforcement power provided in Art.
128(b) of the Labor Code, as amended by RA 7730. The National Labor Relations Commission (NLRC) was
held to be the primary agency in determining the existence of an employer-employee relationship. This was the
interpretation of the Court of the clause in cases where the relationship of employer-employee still exists in Art.
128(b).

From this Decision, the Public Attorneys Office (PAO) filed a Motion for Clarification of Decision. The PAO
sought to clarify as to when the visitorial and enforcement power of the DOLE be not considered as co-
extensive with the power to determine the existence of an employer-employee relationship. The DOLE also
sought the same clarification.

ISSUE: Is the NLRC the sole body with jurisdiction to determine the existence of an employer-employee
relationship?

SC RULING:
NO. No procedure was laid down where the DOLE would only make a preliminary finding, that the power was
primarily held by the NLRC. The law did not say that the DOLE would first seek the NLRCs determination of the
existence of an employer-employee relationship, or that should the existence of the employer-employee
relationship be disputed, the DOLE would refer the matter to the NLRC. The DOLE must have the power to
determine whether or not an employer-employee relationship exists, and from there to decide whether or not to
issue compliance orders in accordance with Art. 128(b) of the Labor Code, as amended by RA 7730.

San Beda College of Law 237


4S: 2015 - 2016
LABOR LAW REVIEW Atty. Joyrich Golangco

The determination of the existence of an employer-employee relationship by the DOLE must be respected. The
expanded visitorial and enforcement power of the DOLE granted by RA 7730 would be rendered nugatory if the
alleged employer could, by the simple expedient of disputing the employer-employee relationship, force the
referral of the matter to the NLRC.
If the DOLE makes a finding that there is an existing employer-employee relationship, it takes cognizance of the
matter, to the exclusion of the NLRC. The DOLE would have no jurisdiction only if the employer-employee
relationship has already been terminated, or it appears, upon review, that no employer-employee relationship
existed in the first place.

It must also be remembered that the power of the DOLE to determine the existence of an employer-employee
relationship need not necessarily result in an affirmative finding. The DOLE may well make the determination
that no employer-employee relationship exists, thus divesting itself of jurisdiction over the case. It must not be
precluded from being able to reach its own conclusions, not by the parties, and certainly not by the SC.

To recapitulate, if a complaint is brought before the DOLE to give effect to the labor standards provisions of the
Labor Code or other labor legislation, and there is a finding by the DOLE that there is an existing employer-
employee relationship, the DOLE exercises jurisdiction to the exclusion of the NLRC. If the DOLE finds that
there is no employer-employee relationship, the jurisdiction is properly with the NLRC. If a complaint is filed with
the DOLE, and it is accompanied by a claim for reinstatement, the jurisdiction is properly with the Labor Arbiter,
under Art. 217(3) of the Labor Code, which provides that the Labor Arbiter has original and exclusive jurisdiction
over those cases involving wages, rates of pay, hours of work, and other terms and conditions of employment, if
accompanied by a claim for reinstatement. If a complaint is filed with the NLRC, and there is still an existing
employer-employee relationship, the jurisdiction is properly with the DOLE. The findings of the DOLE, however,
may still be questioned through a petition for certiorari under Rule 65 of the Rules of Court.

San Beda College of Law 238


4S: 2015 - 2016
LABOR LAW REVIEW Atty. Joyrich Golangco

113. EX-BATAAN VETERANS SECURITY AGENCY, INC., (EBVSAI) v. THE SECRETARY OF LABOR
BIENVENIDO E. LAGUESMA
G.R. No. 152396 November 20, 2007
CARPIO, J.:

THE VISITORIAL AND ENFORCEMENT POWERS OF THE DOLE REGIONAL DIRECTOR CAN BE
EXERCISED EVEN WHERE THE INDIVIDUAL CLAIM EXCEEDS P5,000

DOCTRINE:
While it is true that under Articles 129 and 217 of the Labor Code, the LA has jurisdiction to hear and decide
cases where the aggregate money claims of each employee exceeds P5,000.00, said provisions of law do not
contemplate nor cover the visitorial and enforcement powers of the Secretary of Labor or his duly authorized
representatives. Rather, said powers are defined and set forth in Article 128 of the Labor Code.

FACTS:
Private respondents are EBVSAI's employees who instituted a complaint for underpayment of wages against
EBVSAI before the Regional Office (RO) of DOLE. Consequently, RO conducted a complaint inspection of
EBVSAIs Plant where several labor law violations were noted. On the same day, the RO issued a notice of
hearing requiring EBVSAI and private respondents to attend. After the hearing, the Regional Director (RD)
ordered EBVSAI to pay Php 763,927.85 to the affected employees.

EBVSAI filed a motion for reconsideration and alleged that under Articles 129 and 217(6) of the Labor Code, the
Labor Arbiter, not the Regional Director, has exclusive and original jurisdiction over the case because the
individual monetary claim of private respondents exceeds P5,000. RD denied the motion stating that, pursuant
to RA 7730, the limitations under Articles 129 and 217(6) of the Labor Code no longer apply to the Secretary of
Labor's visitorial and enforcement powers under Article 128(b). The Secretary of Labor or his duly authorized
representatives are now empowered to hear and decide, in a summary proceeding, any matter involving the
recovery of any amount of wages and other monetary claims arising out of employer-employee relations at the
time of the inspection.

DOLE SECRETARY RULING: It affirmed the Directors decision on the ground that pursuant to RA 7730, the
Court's decision in the Servando case is no longer controlling insofar as the restrictive effect of Article 129 on
the visitorial and enforcement power of the Secretary of Labor is concerned.

CA RULING: affirmed DOLE Secretary ruling

ISSUE: Whether the Secretary of Labor or his duly authorized representatives have jurisdiction over the money
claims of private respondents which exceed P5,000?

SC RULING:
YES. In Allied Investigation Bureau, Inc. v. Sec. of Labor, SC ruled that while it is true that under Articles 129
and 217 of the Labor Code, the LA has jurisdiction to hear and decide cases where the aggregate money claims
of each employee exceeds P5,000.00, said provisions of law do not contemplate nor cover the visitorial and
enforcement powers of the Secretary of Labor or his duly authorized representatives. Rather, said powers are
defined and set forth in Article 128 of the Labor Code (as amended by R.A. No. 7730) thus: (b) Notwithstanding
the provisions of Article[s] 129 and 217 of this Code to the contrary, and in cases where the relationship of
employer-employee still exists, the Secretary of Labor and Employment or his duly authorized representatives
shall have the power to issue compliance orders to give effect to [the labor standards provisions of this Code
and other] labor legislation based on the findings of labor employment and enforcement officers or industrial
safety engineers made in the course of inspection.

However, if the labor standards case is covered by the exception clause in Article 128(b) of the Labor Code,
then the RD will have to endorse the case to the appropriate Arbitration Branch of the NLRC. In order to divest
the RD or his representatives of jurisdiction, the following elements must be present: (a) that the employer
contests the findings of the labor regulations officer and raises issues thereon; (b) that in order to resolve such
issues, there is a need to examine evidentiary matters; and (c) that such matters are not verifiable in the normal
course of inspection. The rules also provide that the employer shall raise such objections during the hearing of
the case or at any time after receipt of the notice of inspection results.

San Beda College of Law 239


4S: 2015 - 2016
LABOR LAW REVIEW Atty. Joyrich Golangco

In this case, the RD validly assumed jurisdiction over the money claims of private respondents even if the claims
exceeded P5,000 because such jurisdiction was exercised in accordance with Article 128(b) of the Labor Code
and the case does not fall under the exception clause. EBVSAI did not contest the findings of the labor
regulations officer during the hearing or after receipt of the notice of inspection results. It was only in its
supplemental motion for reconsideration before the RD that EBVSAI questioned the findings of the labor
regulations officer and presented documentary evidence to controvert the claims of private respondents. But
even if this was the case, the RD and the Secretary of Labor still looked into and considered EBVSAI's
documentary evidence and found that such did not warrant the reversal of the order.

San Beda College of Law 240


4S: 2015 - 2016
LABOR LAW REVIEW Atty. Joyrich Golangco

114. ARSENIO LOCSIN v. NISSAN CAR LEASE PHILS., INC. (NCLPI) and LUIS BANSON
G.R. No. 185567 October 20, 2010
BRION, J.:

LA HAS NO JURISDICTION OVER INTRA-CORPORATE CONTROVERSY

DOCTRINE:
Given Locsins status as a corporate officer, the RTC, not the Labor Arbiter or the NLRC, has jurisdiction to hear
the legality of the termination of his relationship with Nissan. A corporate officers dismissal is always a corporate
act, or an intra-corporate controversy which arises between a stockholder and a corporation so that RTC should
exercise jurisdiction based on Section 5(c) of PD 902-A.

FACTS:
Locsin was elected Executive Vice President and Treasurer (EVP/Treasurer) of NCLPI. Locsin held this position
for 13 years until he was nominated and elected Chairman. A few months thereafter, an election was held and
Locsin was neither re-elected Chairman nor reinstated to his previous position as EVP/Treasurer. Locsin filed a
complaint for illegal dismissal before the Labor Arbiter against NCLPI. NCLPI filed a Motion to Dismiss on the
ground that the Labor Arbiter did not have jurisdiction over the case since the issue of Locsins removal as
EVP/Treasurer involves an intra-corporate dispute. Locsin maintained that he is an employee of NCPI.

LA RULING: LA denied the Motion to Dismiss, holding that its office-acquired jurisdiction to arbitrate and/or
decide the instant complaint finding extant in the case an employer-employee relationship. Article 280 of the
Labor Code, the receipt of salaries by Locsin, SSS deductions on that salary, and the element of control in the
performance of work duties were used by LA to conclude that Locsin was a regular employee.

CA RULING: NCLPI elevated the case to the CA through a Petition for Certiorari under Rule 65 of the Rules of
Court. CA ruled that Locsin was a corporate officer; hence the issue of his removal as EVP/Treasurer is an intra-
corporate dispute under the RTCs jurisdiction. The fact that the position of EVP/Treasurer is specifically
enumerated as an office in the corporations by-laws makes him a corporate officer.

ISSUE: Whether Locsins position as EVP/Treasurer makes him a corporate officer thereby excluding him from
the coverage of the Labor Code?

SC RULING:
YES. Locsin was undeniably Chairman and President, and was elected to these positions by the Nissan board
pursuant to its By-laws. As such, he was a corporate officer, not an employee. Section 25 of the Corporation
Code provides that corporate officers are the president, secretary, treasurer and such other officers as may
be provided for in the by-laws.

Even as EVP/Treasurer, Locsin already acted as a corporate officer because such position is provided for in
Nissans By-Laws. An office is created by the charter of the corporation and the officer is elected by the directors
or stockholders. On the other hand, an employee usually occupies no office and generally is employed by the
managing officer of the corporation who also determines the compensation to be paid to such employee. Locsin
was elected by the NCLPI Board, in accordance with the Amended By-Laws of the corporation.
Given Locsins status as a corporate officer, the RTC, not the Labor Arbiter or the NLRC, has jurisdiction to hear
the legality of the termination of his relationship with Nissan. A corporate officers dismissal is always a corporate
act, or an intra-corporate controversy which arises between a stockholder and a corporation so that RTC should
exercise jurisdiction based on Section 5(c) of PD 902-A.

San Beda College of Law 241


4S: 2015 - 2016
LABOR LAW REVIEW Atty. Joyrich Golangco

115. OSCAR C. REYES vs. HON. REGIONAL TRIAL COURT OF MAKATI, Branch 142, ZENITH
INSURANCE CORPORATION, and RODRIGO C. REYES
G.R. No. 165744 August 11, 2008
BRION, J.:

JURISDICTION OF SPECIAL COMMERCIAL COURTS

DOCTRINE:
Without the settlement of Anastacias estate, there can be no definite partition and distribution of the estate to
the heirs. Without the partition and distribution, there can be no registration of the transfer. And without the
registration, we cannot consider the transferee-heir a stockholder who may invoke the existence of an intra-
corporate relationship as premise for an intra-corporate controversy within the jurisdiction of a special
commercial court.

FACTS:
Oscar and private respondent Rodrigo C. Reyes (Rodrigo) are the children of the spouses Pedro and Anastacia
Reyes. Pedro, Anastacia, Oscar, and Rodrigo each owned shares of stock of Zenith Insurance Corporation
(Zenith). Pedro died in 1964, while Anastacia died in 1993. Although Pedros estate was judicially partitioned
among his heirs sometime in the 1970s, no similar settlement and partition appear to have been made with
Anastacias estate, which included her shareholdings in Zenith. Zenith and Rodrigo filed a derivative suit with
SEC (now RTC) against Oscar in order to obtain an accounting of the funds and assets of Zenith which are now
in the possession of Oscar and to determine the shares of stock of deceased spouses that were arbitrarily and
fraudulently appropriated by Oscar for himself and which were not collated and taken into account in the
partition, distribution, and/or settlement of the estate.

Oscar filed a Motion to Declare Complaint as Nuisance or Harassment Suit. He claimed that the complaint is a
mere nuisance or harassment suit and should be dismissed; and that it is not a bona fide derivative suit as it
partakes of the nature of a petition for the settlement of estate of the Anastacia that is outside the jurisdiction of
a RTC.

RTC RULING: RTC denied the motion as to the action for determination of the shares of stock of deceased
allegedly taken by Oscar, its accounting and the corresponding delivery of these shares since it is not a
derivative suit and should properly be threshed out in a petition for settlement of estate. However, the action
with respect to the derivative suit for accounting of the funds and assets of the corporation which are in the
control, custody, and/or possession of the Oscar was not dismissed and was taken cognizance of by RTC.

CA RULING: affirmed the RTC order

ISSUE: Whether the special commercial court (RTC) have jurisdiction over the subject matter of Rodrigos
complaint?

SC RULING:
NO. While Rodrigo holds shares of stock in Zenith, he holds them in two capacities: in his own right with respect
to the 4,250 shares registered in his name, and as one of the heirs of Anastacia Reyes with respect to the
136,598 shares registered in her name. What is material in resolving the issues of this case under the
allegations of the complaint is Rodrigos interest as an heir since the subject matter of the present controversy
centers on the shares of stocks belonging to Anastacia, not on Rodrigos personally-owned shares nor on his
personality as shareholder owning these shares.

Hence, Rodrigo must first prove that there are shareholdings that will be left to him and his co-heirs, and this
can be determined only in a settlement of the decedents estate. No such proceeding has been commenced to
date. Without the settlement of Anastacias estate, there can be no definite partition and distribution of the estate
to the heirs. Without the partition and distribution, there can be no registration of the transfer. And without the
registration, we cannot consider the transferee-heir a stockholder who may invoke the existence of an intra-
corporate relationship as premise for an intra-corporate controversy within the jurisdiction of a special
commercial court.

San Beda College of Law 242


4S: 2015 - 2016
LABOR LAW REVIEW Atty. Joyrich Golangco

116. LESLIE OKOL v. SLIMMERS WORLD INTERNATIONAL, BEHAVIOR MODIFICATIONS, INC., and
RONALD JOSEPH MOY
G.R. No. 160146 DECEMBER 11, 2009
CARPIO, J.:

LA HAS NO JURISDICTION OVER INTRA-CORPORATE CONTROVERSY

DOCTRINE: In a number of cases, SC held that a corporate officers dismissal is always a corporate act, or an
intra-corporate controversy which arises between a stockholder and a corporation. The question of
remuneration involving a stockholder and officer, not a mere employee, is not a simple labor problem but a
matter that comes within the area of corporate affairs and management and is a corporate controversy in
contemplation of the Corporation Code.

FACTS:
Respondent Slimmers World International operating under the name Behavior Modifications, Inc. (Slimmers
World) employed petitioner Leslie Okol (Okol) as a management trainee. Okol was promoted as Head Office
Manager and then Director and Vice President. Okols services was terminated by Slimmers World due to the
seizure by the Bureau of Customs of machines and treadmills to or consigned to Slimmers World but the
shipment of the equipment was placed under the name of Okol.

Okol filed an illegal dismissal complaint with the LA. Respondents filed a Motion to Dismiss asserting that the
NLRC had no jurisdiction over the subject matter of the complaint. Okol argued that even as vice-president, the
work that she performed conforms to that of an employee rather than a corporate officer. Mere title or
designation in a corporation will not, by itself, determine the existence of an employer-employee relationship.

LA RULING: LA granted the motion to dismiss ruling that Okol was the vice-president of Slimmers World at the
time of her dismissal. Since it involved a corporate officer, the dispute was an intra-corporate controversy falling
outside the jurisdiction of the Arbitration branch.

NLRC RULING: It reversed the LA decision

CA RULING: It affirmed LAs ruling holding that being an intra-corporate dispute, the case falls within the
jurisdiction of the regular courts pursuant to Republic Act No. 8799.

ISSUE: Does NLRC have jurisdiction over the illegal dismissal case filed by petitioner?

SC RULING:
NO. Section 25 of the Corporation Code enumerates corporate officers as the president, secretary, treasurer
and such other officers as may be provided for in the by-laws. An office is created by the charter of the
corporation and the officer is elected by the directors or stockholders. On the other hand, an employee usually
occupies no office and generally is employed not by action of the directors or stockholders but by the managing
officer of the corporation who also determines the compensation to be paid to such employee.

The Amended By-Laws of Slimmers World which enumerate the power of the board of directors as well as the
officers of the corporation clearly shows that Okol was a director and officer of Slimmers World. In a number of
cases, SC held that a corporate officers dismissal is always a corporate act, or an intra-corporate controversy
which arises between a stockholder and a corporation. The question of remuneration involving a stockholder
and officer, not a mere employee, is not a simple labor problem but a matter that comes within the area of
corporate affairs and management and is a corporate controversy in contemplation of the Corporation Code.

San Beda College of Law 243


4S: 2015 - 2016
LABOR LAW REVIEW Atty. Joyrich Golangco

117. RURAL BANK OF CORON (PALAWAN), INC., EMPIRE COLD STORAGE AND DEVELOPMENT
CORPORATION, CITIZENS DEVELOPMENT INCOPRORATED (CDI), CARIDAD B. GARCIA, SANDRA G.
ESCAT, LORNA GARCIA, and OLGA G. ESCAT v. ANNALISA CORTES
G.R. No. 164888 December 6, 2006
CARPIO MORALES, J.:

JURISDICTION OF LA; POSTING A BOND IS A REQUIREMENT FOR PERFECTION OF APPEAL TO NLRC

DOCTRINES:
1. While respondent was the Corporate Secretary of the Rural Bank of Coron, she was also its Financial
Assistant and the Personnel Officer of the two other petitioner corporations. A corporation can engage
its corporate officers to perform services under a circumstance which would make them employees. The
Labor Arbiter has thus jurisdiction over respondents complaint.
2. All that is required to perfect the appeal is the posting of a bond to ensure that the award is eventually
paid should the appeal be dismissed. Petitioners should thus have posted a bond, even if it were only
partial, but they did not.

FACTS:
Respondent was the Financial Assistant, Personnel Officer and Corporate Secretary of The Rural Bank of
Coron, Personnel Officer of CDI, and also Personnel Officer and Disbursing Officer of The Empire Cold Storage
Development Corporation (ECSDC). She simultaneously received salaries from these corporations.
On examination of the financial books of the corporations, it was discovered that respondent was involved in
5
several anomalies, drawing petitioners to terminate respondents services. Respondent filed a complaint for
illegal dismissal and non-payment of salaries and other benefits before the LA. Petitioners moved for the
dismissal of the complaint on the ground of lack of jurisdiction, contending that the case was an intra-corporate
controversy involving the removal of a corporate officer, respondent being the Corporate Secretary of the Rural
Bank of Coron, Inc., hence, cognizable by the Securities and Exchange Commission (SEC) (now RTC) pursuant
to Section 5 of PD 902-A.

LA RULING: LA assumed jurisdiction ruling that aside from her being Corporate Secretary of Rural Bank of
Coron, complainant was likewise appointed as Financial Assistant & Personnel Officer, which is not a corporate
officer of petitioners. LA ordered petitioners to pay respondent P1,168,090.00.

NLRC RULING: On the tenth or last day of the period of appeal, petitioners filed a Notice of Appeal and Motion
for Reduction of Bond to which they attached a Memorandum on Appeal. In their Motion for Reduction of Bond,
petitioners alleged that the corporations were under financial distress and the Rural Bank of Coron was under
receivership. NLRC, while noting that petitioners timely filed the appeal, held that the same was not
accompanied by an appeal bond, a mandatory requirement under Article 223 of the Labor Code and Section 6,
Rule VI of the NLRC New Rules of Procedure. It also noted that the Motion for Reduction of Bond was
"premised on self-serving allegations." It accordingly dismissed the appeal.

ISSUES:
1. Whether LA has jurisdiction over the case?
2. Whether petitioners appeal before NLRC was perfected?

SC RULING:
1. YES. While respondent was the Corporate Secretary of the Rural Bank of Coron, she was also its
Financial Assistant and the Personnel Officer of the two other petitioner corporations. Mainland
Construction Co., Inc. v. Movilla instructs that a corporation can engage its corporate officers to perform
services under a circumstance which would make them employees. The Labor Arbiter has thus
jurisdiction over respondents complaint.
2. NO. All that is required to perfect the appeal is the posting of a bond to ensure that the award is
eventually paid should the appeal be dismissed. Petitioners should thus have posted a bond, even if it
were only partial, but they did not. In the case at bar, petitioner did not post a full or partial appeal bond
within the prescribed period, thus, no appeal was perfected from the decision of the LA. For this reason,
the decision sought to be appealed to the NLRC had become final and executory and therefore
immutable. No relaxation of the Rule may thus be considered. Clearly then, the NLRC has no authority
to entertain the appeal, much less to reverse the decision of the LA.
San Beda College of Law 244
4S: 2015 - 2016
LABOR LAW REVIEW Atty. Joyrich Golangco

San Beda College of Law 245


4S: 2015 - 2016
LABOR LAW REVIEW Atty. Joyrich Golangco

118. HALGUENA v. PAL
G.R. No. 172013 October 2, 2009
PERALTA, J.:

JURISDICTION OF LABOR ARBITER

DOCTRINE:
Not every controversy or money claim by an employee against the employer or vice-versa is within the exclusive
jurisdiction of the labor arbiter. Actions between employees and employer where the employer-employee
relationship is merely incidental and the cause of action precedes from a different source of obligation is within
the exclusive jurisdiction of the regular court.

FACTS:
Petitioners were employed as female flight attendants of PAL. They are members of the Flight Attendants and
Stewards Association of the Philippines (FASAP), the exclusive exclusive bargaining representative of the flight
attendants.Section 144, Part A of the PAL-FASAP CBA, provides that: 3. Compulsory Retirement. Subject to
the grooming standards provisions of this Agreement, compulsory retirement shall be fifty-five (55) for females
and sixty (60) for males. x x x. petitioners and several female cabin crews manifested that the aforementioned
CBA provision on compulsory retirement is discriminatory, and demanded for an equal treatment with their male
counterparts. This demand was reiterated in a letter. On July 12, 2004, Robert D. Anduiza, President of FASAP
submitted their 2004-2005 CBA proposals[6] and manifested their willingness to commence the collective
bargaining negotiations between the management and the association, at the soonest possible time.

In 2004, petitioners filed a Special Civil Action for Declaratory Relief with Prayer for the Issuance of TRO and
Writ of Preliminary Injunction with the Regional Trial Court (RTC) of Makati Cityagainst respondent for the
invalidity of Section 144, Part A of the PAL-FASAP CBA.

RTC RULING: The RTC issued an Order upholding its jurisdiction over the present case. The RTC reasoned
that: The allegations in the Petition do not make out a labor dispute arising from employer-employee relationship
as none is shown to exist. This case is not directed specifically against respondent arising from any act of the
latter, nor does it involve a claim against the respondent. Rather, this case seeks a declaration of the nullity of
the questioned provision of the CBA, which is within the Court's competence, with the allegations in the Petition
constituting the bases for such relief sought.

The RTC issued a TRO on August 10, 2004, enjoining the respondent for implementing Section 144, Part A of
the PAL-FASAP CBA.

CA RULING: declared RTC to have NO JURISDICTION OVER THE CASE

ISSUE: Does the RTC have jurisdiction over the petitioners' action challenging the legality or constitutionality of
the provisions on the compulsory retirement age contained in the CBA between respondent PAL and FASAP?

SC RULING:
YES. The subject of litigation is incapable of pecuniary estimation, exclusively cognizable by the RTC, pursuant
to Section 19 (1) of Batas Pambansa Blg. 129, as amended. Being an ordinary civil action, the same is beyond
the jurisdiction of labor tribunals.

The said issue cannot be resolved solely by applying the Labor Code. Rather, it requires the application of the
Constitution, labor statutes, law on contracts and the Convention on the Elimination of All Forms of
Discrimination Against Women, and the power to apply and interpret the constitution and CEDAW is within the
jurisdiction of trial courts, a court of general jurisdiction. In Georg Grotjahn GMBH & Co. v. Isnani, this Court
held that not every dispute between an employer and employee involves matters that only labor arbiters and the
NLRC can resolve in the exercise of their adjudicatory or quasi-judicial powers. The jurisdiction of labor arbiters
and the NLRC under Article 217 of the Labor Code is limited to disputes arising from an employer-employee
relationship which can only be resolved by reference to the Labor Code, other labor statutes, or their collective
bargaining agreement.

Not every controversy or money claim by an employee against the employer or vice-versa is within the exclusive
jurisdiction of the labor arbiter. Actions between employees and employer where the employer-employee
San Beda College of Law 246
4S: 2015 - 2016
LABOR LAW REVIEW Atty. Joyrich Golangco

relationship is merely incidental and the cause of action precedes from a different source of obligation is within
the exclusive jurisdiction of the regular court. Here, the employer-employee relationship between the parties is
merely incidental and the cause of action ultimately arose from different sources of obligation, i.e., the
Constitution and CEDAW.

Thus, where the principal relief sought is to be resolved not by reference to the Labor Code or other labor
relations statute or a collective bargaining agreement but by the general civil law, the jurisdiction over the
dispute belongs to the regular courts of justice and not to the labor arbiter and the NLRC. In such situations,
resolution of the dispute requires expertise, not in labor management relations nor in wage structures and other
terms and conditions of employment, but rather in the application of the general civil law. Clearly, such claims
fall outside the area of competence or expertise ordinarily ascribed to labor arbiters and the NLRC and the
rationale for granting jurisdiction over such claims to these agencies disappears.

San Beda College of Law 247


4S: 2015 - 2016
LABOR LAW REVIEW Atty. Joyrich Golangco

119. SANTIAGO v. CF SHARP CREW MANAGEMENT
G.R. No. 162419 July 10, 2007
TINGA, J.:

JURISDICTION OF LABOR ARBITER

DOCTRINE:
The jurisdiction of labor arbiters is not limited to claims arising from employer-employee relationships.

FACTS:
In 1998, Paul Santiago signed a new contract of employment with CF Sharp Crew Mgmt., Inc., with the duration
of nine (9) months. He was assured of a monthly salary of US$515.00, overtime pay and other benefits.
Santiago was to be deployed on board the "MSV Seaspread". A week before the scheduled date of departure,
Capt. Pacifico Fernandez, CF Sharps Vice President, sent a fax to the captain of "MSV Seaspread telling the
latter that he received calls from various individuals about the possibility that Santiago may jump ship in Canada
like his brother did before him. Santiago was thus told that he would not be leaving for Canada anymore, but he
was reassured that he might be considered for deployment at some future date.

Consequently, Santiago filed a complaint for illegal dismissal, damages, and attorney's fees against CF Sharp
and its foreign principal. In defense, CF Sharp contends that there is no employer-employee relationship
between petitioner and respondent because under the POEA Standard Contract, the employment contract shall
commence upon actual departure of the seafarer from the airport or seaport at the point of hire. In the absence
of an employer-employee relationship between the parties, the claims for illegal dismissal, actual damages, and
attorneys fees should be dismissed as the NLRC does not have jurisdiction over the same.

LA RULING: The labor arbiter held respondent liable

NLRC RULING: (NLRC) ruled that there is no employer-employee relationship between petitioner and
respondent because under the Standard Terms and Conditions Governing the Employment of Filipino Seafarers
on Board Ocean Going Vessels (POEA Standard Contract), the employment contract shall commence upon
actual departure of the seafarer from the airport or seaport at the point of hire and with a POEA-approved
contract. In the absence of an employer-employee relationship between the parties, the claims for illegal
dismissal, actual damages, and attorneys fees should be dismissed.

CA RULING: It agreed with the NLRCs finding that petitioners non-deployment was a valid exercise of
respondents management prerogative.

ISSUE: Does the NLRC have jurisdiction over the case?

SC RULING:
YES. The jurisdiction of labor arbiters is not limited to claims arising from employer-employee relationships.
Section 10 of R.A. No. 8042 (Migrant Workers Act), provides that:

Sec. 10. Money Claims. Notwithstanding any provision of law to the contrary, the Labor Arbiters of the National
Labor Relations Commission (NLRC) shall have the original and exclusive jurisdiction to hear and decide, within
ninety (90) calendar days after the filing of the complaint, the claims arising out of an employer-employee
relationship or by virtue of any law or contract involving Filipino workers for overseas deployment including
claims for actual, moral, exemplary and other forms of damages. x x x

Since the present petition involves the employment contract entered into by petitioner for overseas employment,
his claims are cognizable by the labor arbiters of the NLRC.

San Beda College of Law 248


4S: 2015 - 2016
LABOR LAW REVIEW Atty. Joyrich Golangco

120. ATLAS FARMS, INC. v. NLRC
G.R. No. 142244 November 18, 2002
QUISUMBING, J.:

JURISDICTION OF LABOR ARBITER

DOCTRINE:
Where the dispute is just in the interpretation, implementation or enforcement stage, it may be referred to the
grievance machinery set up in the CBA, or brought to voluntary arbitration. But, where there was already actual
termination, with alleged violation of the employees rights, it is already cognizable by the labor arbiter.

FACTS:
Private respondent Jaime O. dela Pea was employed as a veterinary aide by petitioner. He was among several
employees terminated in July 1989. On July 8, 1989, he was re-hired by petitioner and given the additional job
of feedmill operator. He was instructed to train selected workers to operate the feedmill.

In 1993, Pea was allegedly caught urinating and defecating on company premises not intended for the purpose.
The farm manager of petitioner issued a formal notice directing him to explain within 24 hours why disciplinary
action should not be taken against him. Pea refused, however, to receive the formal notice. He never bothered
to explain. Thus, a notice of termination with payment of his monetary benefits was sent to him.

Co-respondent Marcial I. Abion was a carpenter/mason and a maintenance man whose employment by
petitioner. Allegedly, he caused the clogging of the fishpond drainage resulting in damages worth several
hundred thousand pesos when he improperly disposed of the cut grass and other waste materials into the
ponds drainage system. Petitioner sent a written notice to Abion, requiring him to explain what happened,
otherwise, disciplinary action would be taken against him. He refused to receive the notice and give an
explanation, according to petitioner. Consequently, the company terminated his services. He acknowledged
receipt of a written notice of dismissal, with his separation pay.

Pea and Abion filed separate complaints for illegal dismissal that were later consolidated. Both claimed that their
termination from service was due to petitioners suspicion that they were the leaders in a plan to form a union to
compete and replace the existing management-dominated union.

LA RULING: The labor arbiter dismissed their complaints on the ground that the grievance machinery in the
collective bargaining agreement (CBA) had not yet been exhausted. Private respondents availed of the
grievance process, but later on refiled the case before the NLRC in Region IV. They alleged lack of sympathy on
petitioners part to engage in conciliation proceedings.

NLRC RULING: NLRC reversed the labor arbiters decision.

CA RULING: The appellate court denied the petition and affirmed the NLRC resolution with some modifications,
thus: 1) The private respondents can not be reinstated, due to their acceptance of the separation pay offered by
the petitioner; 2) The private respondents are entitled to their full back wages; and, 3) The amount of the
separation pay received by private respondents from petitioner shall not be deducted from their full back wages.

ISSUE: Does the LA and NLRC have jurisdiction over the case?

SC RULING:
YES. Coming to the merits of the petition, the NLRC found that petitioner did not comply with the requirements
of a valid dismissal. For a dismissal to be valid, the employer must show that: (1) the employee was accorded
due process, and (2) the dismissal must be for any of the valid causes provided for by law. No evidence was
shown that private respondents refused, as alleged, to receive the notices requiring them to show cause why no
disciplinary action should be taken against them. Without proof of notice, private respondents who were
subsequently dismissed without hearing were also deprived of a chance to air their side at the level of the
grievance machinery. Given the fact of dismissal, it can be said that the cases were effectively removed from
the jurisdiction of the voluntary arbitrator, thus placing them within the jurisdiction of the labor arbiter. Where the
dispute is just in the interpretation, implementation or enforcement stage, it may be referred to the grievance
machinery set up in the CBA, or brought to voluntary arbitration. But, where there was already actual
termination, with alleged violation of the employees rights, it is already cognizable by the labor arbiter.
San Beda College of Law 249
4S: 2015 - 2016
LABOR LAW REVIEW Atty. Joyrich Golangco

121. PERPETUAL HELP CREDIT COOPERATIVE, INC. (PHCCI) v. BENEDICTO FABURADA
G.R. No. 121948. October 8, 2001
SANDOVAL-GUTIERREZ, J.:

JURISDICTION OF LABOR ARBITER

DOCTRINE:
The dispute is about payment of wages, overtime pay, rest day and termination of employment. Under Art. 217
of the Labor Code, these disputes are within the original and exclusive jurisdiction of the Labor Arbiter.

FACTS:
Benedicto Faburada, Sisinita Vilar, Imelda Tamayo and Harold Catipay, private respondents, filed a complaint
against petitioner, with the Arbitration Branch, DOLE for illegal dismissal, premium pay on holidays and rest
days, separation pay, wage differential, moral damages, and attorneys fees.

Petitioner PHCCI filed a motion to dismiss the complaint on the ground that there is no employer-employee
relationship between them as private respondents are all members and co-owners of the cooperative and they
have not exhausted the remedies provided in the cooperative by-laws. Petitioner filed a supplemental motion to
dismiss alleging that Article 121 of R.A. No. 6939 or the Cooperative Development Authority Law which took
effect on March 26, 1990, requires conciliation or mediation within the cooperative before a resort to judicial
proceeding.

LA RULING: The Labor Arbiter denied petitioner's motion to dismiss, holding that the case is impressed with
employer-employee relationship and that the law on cooperatives is subservient to the Labor Code.

NLRC RULING: NLRC affirmed the Labor Arbiter's decision

CA RULING: The appellate court denied the petition and affirmed the NLRC resolution with some modifications,
thus: 1) The private respondents cannot be reinstated, due to their acceptance of the separation pay offered by
the petitioner; 2) The private respondents are entitled to their full back wages; and, 3) The amount of the
separation pay received by private respondents from petitioner shall not be deducted from their full back wages.

ISSUE: Does the LA have jurisdiction over the case?

SC RULING:
YES. As aptly stated by the Solicitor General in his comment, P.D. 175 (strengthening the Cooperative
Movement) does not provide for a grievance machinery where a dispute or claim may first be submitted. LOI 23
refers to instructions to the Secretary of Public Works and Communications to implement immediately the
recommendation of the Postmaster General for the dismissal of some employees of the Bureau of Post.
Obviously, this LOI has no relevance to the instant case.

Article 121 of Republic Act No. 6938 (Cooperative Code of the Philippines) provides the procedure how
cooperative disputes are to be resolved, thus:

ART. 121. Settlement of Disputes.- Disputes among members, officers, directors, and
committee members, and intra-cooperative disputes shall, as far as practicable, be settled
amicably in accordance with the conciliation or mediation mechanisms embodied in the bylaws
of the cooperative, and in applicable laws.

Should such a conciliation/mediation proceeding fail, the matter shall be settled in a court of competent
jurisdiction.

Complementing this Article is Section 8 of R.A. No. 6939 (Cooperative Development Authority Law) which
reads:

SEC. 8 Mediation and Conciliation.- Upon request of either or both parties, the Authority shall
mediate and conciliate disputes within a cooperative or between cooperatives: Provided, That if
no mediation or conciliation succeeds within three (3) months from request thereof, a certificate
of non-resolution shall be issued by the Commission prior to the filing of appropriate action
San Beda College of Law 250
4S: 2015 - 2016
LABOR LAW REVIEW Atty. Joyrich Golangco

before the proper courts.

The above provisions apply to members, officers and directors of the cooperative involved in disputes within a
cooperative or between cooperatives.

There is no evidence that private respondents are members of petitioner PHCCI and even if they are, the
dispute is about payment of wages, overtime pay, rest day and termination of employment. Under Art. 217 of the
Labor Code, these disputes are within the original and exclusive jurisdiction of the Labor Arbiter.

San Beda College of Law 251


4S: 2015 - 2016
LABOR LAW REVIEW Atty. Joyrich Golangco

122. AUSTRIA v. NLRC
G.R. No. 124382 August 16, 1999
KAPUNAN, J.:

JURISDICTION OF LABOR ARBITER

DOCTRINE:
Under the Labor Code, the provision which governs the dismissal of employees, is comprehensive enough to
include religious corporations, such as the SDA, in its coverage.

The active participation of a party against whom the action was brought, coupled with his failure to object to the
jurisdiction of the court or quasi-judicial body where the action is pending, is tantamount to an invocation of that
jurisdiction and a willingness to abide by the resolution of the case and will bar said party from later on
impugning the court or bodys jurisdiction.

FACTS:
Private Respondent Central Philippine Union Mission Corporation of the Seventh-Day Adventists (SDA) is a
religious corporation. Petitioner, on the other hand, was a Pastor of the SDA until 31 October 1991, when his
services were terminated.

petitioner received several communications from Mr. Eufronio Ibesate, the treasurer of the Negros Mission
asking him to admit accountability and responsibility for the church tithes and offerings collected by his wife,
Mrs. Thelma Austria, in his district which amounted to P15,078.10, and to remit the same to the Negros Mission.
Petitioner reasoned out that he should not be made accountable since it was private respondents Pastor Gideon
Buhat and Mr. Eufronio Ibesate who authorized his wife to collect the tithes and offerings since he was very sick
to do the collecting at that time.

On 16 October 1991,Petitioner went to the office of Pastor Buhat, the president of the Negros Mission. During
said call, petitioner tried to persuade Pastor Buhat to convene the Executive Committee for the purpose of
settling the dispute between him and the private respondent, Pastor David Rodrigo. The dispute between Pastor
Rodrigo and petitioner arose from an incident in which petitioner assisted his friend, Danny Diamada, to collect
from Pastor Rodrigo the unpaid balance for the repair of the latters motor vehicle which he failed to pay to
Diamada. Due to the assistance of petitioner in collecting Pastor Rodrigos debt, the latter harbored ill-feelings
against petitioner. When news reached petitioner that Pastor Rodrigo was about to file a complaint against him
with the Negros Mission, he immediately proceeded to the office of Pastor Buhat on the date abovementioned
and asked the latter to convene the Executive Committee. Pastor Buhat denied the request of petitioner since
some committee members were out of town and there was no quorum. Thereafter, the two exchanged heated
arguments.

A fact-finding committee was created to investigate petitioner. Subsequently, petitioner received a letter of
dismissal citing misappropriation of denominational funds, willful breach of trust, serious misconduct, gross and
habitual neglect of duties, and commission of an offense against the person of employers duly authorized
representative, as grounds for the termination of his services.

Reacting against the adverse decision of the SDA, petitioner filed a complaint before the Labor Arbiter for illegal
dismissal against the SDA and its officers and prayed for reinstatement with backwages and benefits, moral and
exemplary damages and other labor law benefits.

Private respondents contend that by virtue of the doctrine of separation of church and state, the Labor Arbiter
and the NLRC have no jurisdiction to entertain the complaint filed by petitioner. Since the matter at bar allegedly
involves the discipline of a religious minister, it is to be considered a purely ecclesiastical affair to which the
State has no right to interfere.

LA RULING: The Labor Arbiter RENDERED DECISION IN FAVOR OF PETITIONER.

NLRC RULING: sustained the argument posed by private respondents and, accordingly, dismissed the
complaint of petitioner.

ISSUE: Does the LA have jurisdiction over the case?


San Beda College of Law 252
4S: 2015 - 2016
LABOR LAW REVIEW Atty. Joyrich Golangco

SC RULING:
YES. Under the Labor Code, the provision which governs the dismissal of employees, is comprehensive enough
to include religious corporations, such as the SDA, in its coverage. Article 278 of the Labor Code on post-
employment states that the provisions of this Title shall apply to all establishments or undertakings, whether for
profit or not. Obviously, the cited article does not make any exception in favor of a religious corporation. This is
made more evident by the fact that the Rules Implementing the Labor Code, particularly, Section 1, Rule 1,
Book VI on the Termination of Employment and Retirement, categorically includes religious institutions in the
coverage of the law, to wit:

Section 1. Coverage. This Rule shall apply to all establishments and undertakings, whether operated for profit or
not, including educational, medical, charitable and religious institutions and organizations, in cases of regular
employment with the exception of the Government and its political subdivisions including government-owned or
controlled corporations.

With this clear mandate, the SDA cannot hide behind the mantle of protection of the doctrine of separation of
church and state to avoid its responsibilities as an employer under the Labor Code.

Finally, as correctly pointed out by petitioner, private respondents are estopped from raising the issue of lack of
jurisdiction for the first time on appeal. It is already too late in the day for private respondents to question the
jurisdiction of the NLRC and the Labor Arbiter since the SDA had fully participated in the trials and hearings of
the case from start to finish. The Court has already ruled that the active participation of a party against whom
the action was brought, coupled with his failure to object to the jurisdiction of the court or quasi-judicial body
where the action is pending, is tantamount to an invocation of that jurisdiction and a willingness to abide by the
resolution of the case and will bar said party from later on impugning the court or bodys jurisdiction. Thus, the
active participation of private respondents in the proceedings before the Labor Arbiter and the NLRC mooted the
question on jurisdiction.

San Beda College of Law 253


4S: 2015 - 2016
LABOR LAW REVIEW Atty. Joyrich Golangco

123. DEPARTMENT OF FOREIGN AFFAIRS v. NATIONAL LABOR RELATIONS COMMISSION, HON.
LABOR ARBITER NIEVES V. DE CASTRO and JOSE C. MAGNAYI
G.R. No. 113191 September 18, 1996
VITUG, J.:

ART. 217

DOCTRINE:
The stipulations of both the Charter and Headquarters Agreement should be able, may well enough, to establish
that, except in the specified cases of borrowing and guarantee operations, as well as the purchase, sale and
underwriting of securities, the ADB enjoys immunity from legal process of every form. Thus, the decision of the
Labor Arbiter is rendered vacant for being null and void.

FACTS:
Jose Magnayi initiated case for his alleged illegal dismissal by ADB and the latter's violation of the "labor-only"
contracting law. Two summonses were served, one sent directly to the ADB and the other through DFA, both
with a copy of the complaint. Forthwith, the ADB and the DFA notified respondent Labor Arbiter that the ADB, as
well as its President and Officers, were covered by an immunity from legal process except for borrowings,
guaranties or the sale of securities pursuant to its Charter in relation to Headquarters Agreement of ADB and
the Government.

LA RULING: The Labor Arbiter took cognizance of the complaint on the impression that the ADB had waived its
diplomatic immunity from suit. Labor Arbiter concluded (that there Magnayi is illegally dismissed): The ADB did
not appeal. Instead, the DFA sought a "formal vacation of the void judgment from NLRC.

NLRC CHAIRMAN: The defense of immunity could have been raised before the Labor Arbiter by a special
appearance which, naturally, may NOT be considered as a waiver of the very defense being raised. Except
where an appeal is seasonably and properly made, neither the Commission nor the NLRC Chairman may
review, or even question, the propriety of any decision by a Labor Arbiter. Incidentally, the Commission sits en
banc (all fifteen Commissioners) only to promulgate rules of procedure or to formulate policies (Art. 213, Labor
Code).

"If the Department of Foreign Affairs feels that the action of Labor Arbiter Nieves de Castro constitutes
misconduct, malfeasance or misfeasance, it is suggested that an appropriate complaint be lodged with the
Office of the Ombudsman.

Dissatisfied, the DFA lodged the instant petition for certiorari.

OSG in its comment initially assailed the claim of immunity by the ADB. Subsequently, however, it submitted a
Manifestation stating, that ADB, indeed, was correct in invoking its immunity from suit under the Charter and the
Headquarters Agreement.

ISSUE: Is ADB covered by immunity rendering NLRC without jurisdiction?

SC RULING:
YES. The stipulations of both the Charter and Headquarters Agreement should be able, may well enough, to
establish that, except in the specified cases of borrowing and guarantee operations, as well as the purchase,
sale and underwriting of securities, the ADB enjoys immunity from legal process of every form. The Banks
officers, on their part, enjoy immunity in respect of all acts performed by them in their official capacity.

Diplomatic immunity is essentially a political question and courts should refuse to look beyond a determination
by the executive branch of the government, and where the plea of diplomatic immunity is recognized and
affirmed by the executive branch of the government x x x it is then the duty of the courts to accept the claim of
immunity upon appropriate suggestion by the principal law officer of the government, x x x or other officer acting
under his direction.
Being an international organization that has been extended a diplomatic status, the ADB is independent of the
municipal law.
The Office of the President, likewise, has issued a letter to the Secretary of Labor

San Beda College of Law 254


4S: 2015 - 2016
LABOR LAW REVIEW Atty. Joyrich Golangco

"Despite information from DFA, the labor arbiter in question persisted to send summons. Courts should respect
diplomatic immunities of foreign officials recognized by the Philippine government.
There are two conflicting concepts of sovereign immunity, each widely held and firmly established. According to
the classical or absolute theory, a sovereign cannot, without its consent, be made a respondent in the Courts of
another sovereign. According to the newer or restrictive theory, the immunity of the sovereign is recognized only
with regard to public acts or acts jure imperii of a state, but not with regard to private act or acts jure gestionis.

The service contracts referred to by private respondent have not been intended by the ADB for profit or gain but
are official acts over which a waiver of immunity would not attach.

The DFA must be allowed to plead its case whenever necessary or advisable to enable it to help keep the
credibility of the Philippine government before the international community.

"In the United States, the procedure followed is the process of 'suggestion,' where the foreign state or the
international organization sued in an American court requests the Secretary of State to make a determination as
to whether it is entitled to immunity.

"In the Philippines, the practice is for the foreign government or the international organization to first secure an
executive endorsement of its claim of sovereign or diplomatic immunity.
Decision of the Labor Arbiter is VACATED for being NULL AND VOID.

San Beda College of Law 255


4S: 2015 - 2016
LABOR LAW REVIEW Atty. Joyrich Golangco

124. PHILIPPINE NATIONAL BANK v. FLORENCE O. CABANSAG
G.R. No. 157010 June 21, 2005
PANGANIBAN, J.:
DOCTRINE:
Philippine government requires non-Filipinos working in the country to first obtain a local work permit in order to
be legally employed here. That permit, however, does not automatically mean that the non-citizen is thereby
bound by local laws only, as averred by petitioner. It does not at all imply a waiver of ones national laws on
labor. Absent any clear and convincing evidence to the contrary, such permit simply means that its holder has a
legal status as a worker in the issuing country.
All Filipino workers, whether employed locally or overseas, enjoy the protective mantle of Philippine labor and
social legislations. Our labor statutes may not be rendered ineffective by laws or judgments promulgated, or
stipulations agreed upon, in a foreign country.

FACTS:
Florence Cabansag] arrived in Singapore as a tourist. She applied for employment, with the Singapore Branch of
the Philippine National Bank. At the time, too, the Branch Office had two (2) types of employees: (a) expatriates
or the regular employees, hired in Manila and assigned abroad including Singapore, and (b) locally (direct)
hired. Tobias, General Manager found her eminently qualified recommending the appointment of Florence O.
Cabansag, for the position which was approved.

She then filed an Application, with the Ministry of Manpower of the Government of Singapore, for the issuance
of an Employment Pass as an employee of the Singapore PNB Branch. Her application was approved for a
period of two (2) years.

Cabansag submitted to Ruben C. Tobias, her initial Performance Report. Ruben C. Tobias was so impressed
with the Report that he made a notation and, on said Report: GOOD WORK. However, in the evening, she was
told by two (2) co-employees that Ruben C. Tobias has asked them to tell Florence O. Cabansag to resign from
her job. Tobias confirmed the veracity of the information, with the explanation that her resignation was imperative
as a cost-cutting measure of the Bank. She then asked Ruben C. Tobias that she be furnished with a Formal
Advice from the PNB Head Office in Manila. However, Tobias flatly refused. Florence O. Cabansag did not
submit any letter of resignation.

Tobias again summoned Florence O. Cabansag to his office and demanded that she submit her letter of
resignation. For failure thereof, she received a letter from Ruben C. Tobias terminating her employment with the
Bank.

LA RULING: rendered finding respondents guilty of Illegal dismissal.


NLRC RULING: the NLRC affirmed that Decision.

CA RULING: CA noted that petitioner bank had failed to adduce in evidence the Singaporean law supposedly
governing the latters employment Contract with respondent. CA found that the Contract had actually been
processed by the Philippine Embassy in Singapore and approved by POEA, which then used that Contract as a
basis for issuing an Overseas Employment Certificate in favor of respondent.

Even though respondent secured an employment pass from the Singapore Ministry of Employment, she did not
thereby waive Philippine labor laws, or the jurisdiction of the labor arbiter or the NLRC over her Complaint for
illegal dismissal. Finally, the CA held that PNB had failed to establish a just cause for the dismissal of
respondent.

ISSUE:Whether or not the arbitration branch of the NLRC in the National Capital Region has jurisdiction over
the instant controversy;

SC RULING:
YES. The jurisdiction of labor arbiters and the NLRC is specified in Article 217.

More specifically, Section 10 of RA 8042 reads in part:


SECTION 10. Money Claims. Notwithstanding any provision of law to the contrary, the Labor
Arbiters of the National Labor Relations Commission (NLRC) shall have the original and
exclusive jurisdiction to hear and decide, within ninety (90) calendar days after the filing of the
San Beda College of Law 256
4S: 2015 - 2016
LABOR LAW REVIEW Atty. Joyrich Golangco

complaint, the claims arising out of an employer-employee relationship or by virtue of any law
or contract involving Filipino workers for overseas deployment including claims for actual,
moral, exemplary and other forms of damages.
Based on the foregoing provisions, labor arbiters clearly have original and exclusive jurisdiction over claims
arising from employer-employee relations, including termination disputes involving all workers, among whom are
overseas Filipino workers (OFW).

Prior to employing respondent, petitioner had to obtain an employment pass for her from the Singapore Ministry
of Manpower.

Similarly, the Philippine government requires non-Filipinos working in the country to first obtain a local work
permit in order to be legally employed here. That permit, however, does not automatically mean that the non-
citizen is thereby bound by local laws only, as averred by petitioner. It does not at all imply a waiver of ones
national laws on labor. Absent any clear and convincing evidence to the contrary, such permit simply means that
its holder has a legal status as a worker in the issuing country.

Under Philippine law, this document authorized her working status in a foreign country and entitled her to all
benefits and processes under our statutes. Thus, even assuming arguendo that she was considered at the start
of her employment as a direct hire governed by and subject to the laws, common practices and customs
[17]
prevailing in Singapore she subsequently became a contract worker or an OFW who was covered by
Philippine labor laws and policies upon certification by the POEA.

Undeniably, respondent was employed by petitioner in its branch office in Singapore. Admittedly, she is a
Filipino and not a legal resident of that state. She thus falls within the category of migrant worker or overseas
Filipino worker.

As such, it is her option to choose the venue of her Complaint against petitioner for illegal dismissal. The law
gives her two choices: (1) at the Regional Arbitration Branch (RAB) where she resides or (2) at the RAB where
the principal office of her employer is situated. Since her dismissal by petitioner, respondent has returned to the
Philippines -- specifically to her residence at Filinvest II, Quezon City. Thus, in filing her Complaint before the
RAB office in Quezon City, she has made a valid choice of proper venue.

Notice and Hearing Not Complied With; No Valid Cause for Dismissal. Cabansag was Illegally Dismissed.

San Beda College of Law 257


4S: 2015 - 2016
LABOR LAW REVIEW Atty. Joyrich Golangco

125. BEBIANO M. BAEZ v. HON. DOWNEY C. VALDEVILLA and ORO MARKETING, INC.
G.R. No. 128024 May 9, 2000
GONZAGA-REYES, J.:

DOCTRINE:
By the designating clause "arising from the employer-employee relations" Article 217 should apply with equal
force to the claim of an employer for actual damages against its dismissed employee, where the basis for the
claim arises from or is necessarily connected with the fact of termination, and should be entered as a
counterclaim in the illegal dismissal case.

This is, of course, to distinguish from cases of actions for damages where the employer-employee relationship is
merely incidental and the cause of action proceeds from a different source of obligation. Thus, the jurisdiction of
regular courts was upheld where the damages, claimed for were based on tort, malicious prosecution, or breach
of contract, as when the claimant seeks to recover a debt from a former employee or seeks liquidated damages
in enforcement of a prior employment contract.

FACTS:
Bebiano Baez was the sales operations manager of Oro Marketing in its branch in Iligan City Oro "indefinitely
suspended" petitioner and the latter filed a complaint for illegal dismissal with NLRC.

Baez alleged a modus operandi used by Oro Marketing. herein: Defendant canvassed customers personally or
through salesmen of plaintiff which were hired or recruited by him. If said customer decided to buy items from
plaintiff on installment basis, defendant, without the knowledge of said customer and plaintiff, would buy the
items on cash basis at ex-factory price, a privilege not given to customers, and thereafter required the customer
to sign promissory notes and other documents using the name and property of plaintiff, purporting that said
customer purchased the items from plaintiff on installment basis. Thereafter, defendant collected the installment
payments either personally or through Venus Lozano, a Group Sales Manager of plaintiff but also utilized by him
as secretary in his own business for collecting and receiving of installments, purportedly for the plaintiff but in
reality on his own account or business. The collection and receipt of payments were made inside the Iligan City
branch using plaintiffs facilities, property and manpower. That accordingly plaintiffs sales decreased and
reduced to a considerable extent the profits which it would have earned.

LA RULING: Labor Arbiter found petitioner to have been illegally dismissed.

NLRC RULING: dismissed the same for having been filed out of time.
[3]
Elevated by petition for certiorari before the Supreme Court, the case was dismissed on technical grounds ;
and that even if all the procedural requirements for the filing of the petition were met, it would still be dismissed
for failure to show grave abuse of discretion on the part of the NLRC.

Oro filed a complaint for damages before RTC Misamis Oriental which prayed for the payment of loss of profit
and/or unearned income and expenses of litigation.

Baez filed a motion to dismiss the above complaint. He interposed in the court below that the action for
damages, having arisen from an employer-employee relationship, was squarely under the exclusive original
jurisdiction of the NLRC. He accused Oro Marketing of splitting causes of action, stating that the latter could
very well have included the instant claim for damages in its counterclaim before the Labor Arbiter. He also
pointed out that the civil action of private respondent is an act of forum-shopping.

RTC RULING: A perusal of the complaint which is for damages does not ask for any relief under the Labor
Code. The Court believes such cause of action is within the realm of civil law, and jurisdiction over the
controversy belongs to the regular courts.

ISSUE: Whether RTC has jurisdiction over the case.

SC RULING:
NO. Article 217(a), paragraph 4 of the Labor Code,

ART. 217. Jurisdiction of Labor Arbiters and the Commission.


San Beda College of Law 258
4S: 2015 - 2016
LABOR LAW REVIEW Atty. Joyrich Golangco

4. Claims for actual, moral, exemplary and other forms of damages arising from the employer-employee
relations;

The above provisions are a result of the amendment by Section 9 of R.A. No. 6715, which put to rest the earlier
confusion as to who between Labor Arbiters and regular courts had jurisdiction over claims for damages as
between employers and employees.

By the designating clause "arising from the employer-employee relations" Article 217 should apply with equal
force to the claim of an employer for actual damages against its dismissed employee, where the basis for the
claim arises from or is necessarily connected with the fact of termination, and should be entered as a
counterclaim in the illegal dismissal case.

In the case before us, private respondent's claim against petitioner for actual damages arose from a prior
employer-employee relationship. In the first place, private respondent would not have taken issue with
petitioner's "doing business of his own" had the latter not been concurrently its employee.

Second, and more importantly, to allow respondent court to proceed with the instant action for damages would
be to open anew the factual issue of whether petitioner's installment sale scheme resulted in business losses
and the dissipation of private respondent's property. This issue has been duly raised and ruled upon in the
illegal dismissal case. The Labor Arbiter, however, found to the contrary ---that no business losses may be
attributed to petitioner as in fact, it was by reason of petitioner's installment plan that the sales of the Iligan
branch reached its highest record level.

Evidently, the lawmaking authority had second thoughts about depriving the Labor Arbiters and the NLRC of the
jurisdiction to award damages in labor cases because that setup would mean duplicity of suits, splitting the
cause of action and possible conflicting findings and conclusions by two tribunals on one and the same claim.

This is, of course, to distinguish from cases of actions for damages where the employer-employee relationship is
merely incidental and the cause of action proceeds from a different source of obligation. Thus, the jurisdiction of
regular courts was upheld where the damages, claimed for were based on tort, malicious prosecution, or breach
of contract, as when the claimant seeks to recover a debt from a former employee or seeks liquidated damages
in enforcement of a prior employment contract.

Furthermore, the Labor Arbiter has jurisdiction to award not only the reliefs provided by labor laws, but also
damages governed by the Civil Code.

San Beda College of Law 259


4S: 2015 - 2016
LABOR LAW REVIEW Atty. Joyrich Golangco

126. MA. ISABEL T. SANTOS, represented by ANTONIO P. SANTOS, v. SERVIER PHILIPPINES, INC. and
NATIONAL LABOR RELATIONS COMMISSION
G.R. No. 166377 November 28, 2008
NACHURA, J.:

DOCTRINE:
The issue of deduction for tax purposes is intertwined with the main issue of whether or not petitioners benefits
have been fully given her. It is, therefore, a money claim arising from the employer-employee relationship, which
clearly falls within the jurisdictionhttp://sc.judiciary.gov.ph/jurisprudence/2008/november2008/166377.htm -
_ftn41 of the Labor Arbiter and the NLRC.

FACTS:
Ma. Isabel T. Santos was the Human Resource Manager of respondent Servier Philippines, Inc., Isabel
attended a meeting of all human resource managers of respondent, held in Paris, France. Since the last day of
the meeting coincided with the graduation of Santos only child, she arranged for a European vacation with her
family right after the meeting.

Isabel together with her husband Antonio P. Santos, her son, and some friends, had dinner at Leon des
Bruxelles, a Paris restaurant known for mussels as their specialty. While having dinner, petitioner complained of
stomach pain, then vomited. Eventually, she was brought to the hospital where she fell into coma for 21 days;
and later stayed at the Intensive Care Unit (ICU) for 52 days.

During the time that petitioner was confined at the hospital, her husband and son stayed with her
in Paris. Petitioners hospitalization expenses, as well as those of her husband and son, were paid by
respondent.

She went back to the Philippines and was then confined at the St. Lukes Medical Center for
rehabilitation. During the period of petitioners rehabilitation, respondent continued to pay the formers salaries;
and to assist her in paying her hospital bills.

Petitioners physician concluded that the Santos had not fully recovered mentally and physically. Hence,
respondent was constrained to terminate petitioners services.

Respondent offered a retirement package.

Of the promised retirement benefits amounting to P1,063,841.76, only P701,454.89 was released to
petitioners husband, the balance thereof was withheld allegedly for taxation purposes. Respondent also
failed to give the other benefits. Petitioner, represented by her husband, instituted the instant case for
unpaid amounts.

LA RULING: Labor Arbiter dismissed petitioners complaint. The Labor Arbiter stressed that respondent had
been generous in giving financial assistance to the petitioner. The arbiter refused to rule on the legality of the
deductions made by respondent from petitioners total retirement benefits for taxation purposes, as the issue
was beyond the jurisdiction of the NLRC.

NLRC RULING: NLRC set aside the Labor Arbiters decision.

The NLRC emphasized that petitioner was not retired from the service pursuant to law, collective bargaining
agreement (CBA) or other employment contract; rather, she was dismissed from employment due to a
disease/disability under Article 284. The NLRC therefore ordered the payment of the other benefits promised by
the respondent.

CA RULING: affirmed the NLRC decision.

ISSUE: Whether the benefits are taxable and thus, it was proper for Servier to deduct P362,386.87 for taxation
benefits. (Court ruled that petitioners belatedly claimed entitlement to retirement benefits which issues are not
raised in the pleading, thus deemed abandoned.)

San Beda College of Law 260


4S: 2015 - 2016
LABOR LAW REVIEW Atty. Joyrich Golangco

SC RULING:
YES. As she was dismissed on the ground of Disease, the law gives the petitioner the right to demand
separation pay. However, respondent established a retirement plan in favor of all its employees.The receipt of
retirement benefits does not bar the retiree from receiving separation pay. Separation pay is a statutory right
designed to provide the employee with the wherewithal during the period that he/she is looking for another
employment. On the other hand, retirement benefits are intended to help the employee enjoy the remaining
years of his life, lessening the burden of worrying about his financial support, and are a form of reward for his
[34]
loyalty and service to the employer. Hence, they are not mutually exclusive. However, this is only true if there
is no specific prohibition against the payment of both benefits in the retirement plan and/or in the Collective
[35]
Bargaining Agreement (CBA).

In the instant case, the Retirement Plan bars the petitioner from claiming additional benefits on top of that
provided. Section 2, Article XII of the Retirement Plan provides:

Section 2. NO DUPLICATION OF BENEFITS

Petitioners claim for illegal deduction (for tax purposes) falls within the tribunals jurisdiction. It is noteworthy that
petitioner demanded the completion of her retirement benefits, including the amount withheld by respondent for
taxation purposes. The issue of deduction for tax purposes is intertwined with the main issue of whether or not
petitioners benefits have been fully given her. It is, therefore, a money claim arising from the employer-
[41]
employee relationship, which clearly falls within the jurisdiction of the Labor Arbiter and the NLRC.

Section 32 (B) (6) (a) of the New National Internal Revenue Code (NIRC) provides for the exclusion of
retirement benefits from gross income.

Thus, for the retirement benefits to be exempt from the withholding tax, the taxpayer is burdened to prove the
concurrence of the following elements: (1) a reasonable private benefit plan is maintained by the employer; (2)
the retiring official or employee has been in the service of the same employer for at least ten (10) years; (3) the
retiring official or employee is not less than fifty (50) years of age at the time of his retirement; and (4) the benefit
[43]
had been availed of only once.

Petitioner was qualified for disability retirement. At the time of such retirement, petitioner was only 41 years of
age; and had been in the service for more or less eight (8) years. As such, the above provision is not applicable
for failure to comply with the age and length of service requirements. Therefore, respondent cannot be faulted
for deducting from petitioners total retirement benefits the amount of P362,386.87, for taxation purposes.

San Beda College of Law 261


4S: 2015 - 2016
LABOR LAW REVIEW Atty. Joyrich Golangco

127. PEPSI COLA DISTRIBUTORS OF THE PHILIPPINES, INC., represented by its Plant General Manager
ANTHONY B. SIAN, ELEAZAR LIMBAB, IRENEO BALTAZAR & JORGE HERAYA v.
HON. LOLITA O. GAL-LANG, SALVADOR NOVILLA, ALEJANDRO OLIVA, WILFREDO CABAAS &
FULGENCIO LEGO
G.R. No. 89621 September 24, 1991
CRUZ, J.:

DOCTRINE: Not every controversy involving workers and their employers can be resolved only by the labor
arbiters. This will be so only if there is a "reasonable causal connection" between the claim asserted and
employee-employer relations to put the case under the provisions of Article 217. Absent such a link, the
complaint will be cognizable by the regular courts of justice in the exercise of their civil and criminal jurisdiction.

FACTS: The private respondents were employees of the Pepsi who were suspected of complicity in the
irregular disposition of empty Pepsi Cola bottles. Pepsi filed a criminal complaint for theft against them but this
was later withdrawn and substituted with a criminal complaint for falsification of private documents. After a
preliminary investigation, the complaint was dismissed. The dismissal was affirmed by the Office of the
Provincial Prosecutor.
Meantime, allegedly after an administrative investigation, the private respondents were dismissed by the
petitioner company As a result, they lodged a complaint for illegal dismissal with NLRC in Tacloban City.

NLRC RULING: mandated reinstatement with damages.

In addition, they instituted in the Regional Trial Court of Leyte, a separate civil complaint against the petitioners
for damages arising from what they claimed to be their malicious prosecution.
Pepsi moved to dismiss the civil complaint on the ground that the trial court had no jurisdiction over the case
because it involved employee-employer relations.

RTC RULING: the respondent judge, acting on the motion for reconsideration, reinstated the complaint, saying
it was "distinct from the labor case for damages now pending before the labor courts.

Pepsi invoke Article 217 of the Labor Code and a number of decisions of this Court to support their position that
the private respondents civil complaint for damages falls under the jurisdiction of the labor arbiter.

ISSUE: Whether the RTC has jurisdiction over the case?

SC RULING:
YES. Not every controversy involving workers and their employers can be resolved only by the labor arbiters.
This will be so only if there is a "reasonable causal connection" between the claim asserted and employee-
employer relations to put the case under the provisions of Article 217. Absent such a link, the complaint will be
cognizable by the regular courts of justice in the exercise of their civil and criminal jurisdiction.

EXAMPLES OF CASES:
3
1.) In Medina v. Castro-Bartolome, two employees filed in the Court of First Instance of Rizal a civil
complaint for damages against their employer for slanderous remarks made against them by the
company president. Theirs is a simple action for damages for tortious acts allegedly committed by the
defendants. Such being the case, the governing statute is the Civil Code and not the Labor Code. It
results that the orders under review are based on a wrong premise.
4
2.) In Singapore Airlines Ltd. v. Pao, where the plaintiff was suing for damages for alleged violation by
the defendant of an "Agreement for a Course of Conversion Training at the Expense of Singapore
Airlines Limited. Petitioner seeks protection under the civil laws and claims no benefits under the Labor
Code. The primary relief sought is for liquidated damages for breach of a contractual obligation.
6
3.) In Molave Sales, Inc. v. Laron, the same Justice held for the Court that the claim of the plaintiff against
its sales manager for payment of certain accounts pertaining to his purchase of vehicles and automotive
parts, repairs of such vehicles, and cash advances from the corporation was properly cognizable by the
Regional Trial Court because "although a controversy is between an employer and an employee, the
Labor Arbiters have no jurisdiction if the Labor Code is not involved."

San Beda College of Law 262


4S: 2015 - 2016
LABOR LAW REVIEW Atty. Joyrich Golangco

4.) The latest ruling on this issue is found in San Miguel Corporation v. NLRC. That case involved a claim
of an employee for a P60,000.00 prize for a proposal made by him which he alleged had been accepted
and implemented by the defendant corporation.

Where the claim to the principal relief sought is to be resolved not by reference to the Labor Code or other labor
relations statute or a collective bargaining agreement but by the general civil law, the jurisdiction over the
dispute belongs to the regular courts of justice and not to the Labor Arbiter and the NLRC. While paragraph 3
above refers to "all money claims of workers," it is not necessary to suppose that the entire universe of money
claims that might be asserted by workers against their employers has been absorbed into the original and
exclusive jurisdiction of Labor Arbiters.

The case now before the Court involves a complaint for damages for malicious prosecution which was filed with
the Regional Trial Court of Leyte by the employees of the defendant company. It does not appear that there is a
"reasonable causal connection" between the complaint and the relations of the parties as employer and
employees. The complaint did not arise from such relations and in fact could have arisen independently of an
employment relationship between the parties. No such relationship or any unfair labor practice is asserted. What
the employees are alleging is that the petitioners acted with bad faith when they filed the criminal complaint
which the Municipal Trial Court said was intended "to harass the poor employees" and the dismissal of which
was affirmed by the Provincial Prosecutor "for lack of evidence to establish even a slightest probability that all
the respondents herein have committed the crime imputed against them." This is a matter which the labor arbiter
has no competence to resolve as the applicable law is not the Labor Code but the Revised Penal Code.

San Beda College of Law 263


4S: 2015 - 2016
LABOR LAW REVIEW Atty. Joyrich Golangco

128. 7K CORPORATION v. EDDIE ALBARICO
G.R. No. 182295 June 26, 2013
SERENO, C.J.:

JURISDICTION OF THE VOLUNTARY ARBITRATOR

DOCTRINES:
A voluntary arbitrator may, by agreement of the parties, assume jurisdiction over any of the labor disputes
enumerated under Article 223 of the Labor Code or those which could fall under the jurisdiction of the Labor
Arbiter. He has plenary jurisdiction and authority to interpret an agreement to arbitrate and to determine the
scope of his own authority when the said agreement is vague subject only, in a proper case, to the certiorari
jurisdiction of this Court.

In deciding a case, the voluntary arbitrator may award backwages upon a finding of illegal dismissal, even
though the issue of entitlement thereto is not explicitly claimed in the Submission Agreement. Backwages, in
general, are awarded on the ground of equity as a form of relief that restores the income lost by the terminated
employee by reason of his illegal dismissal.

Aside from illegal dismissal cases, separation pay may also be awarded in the following instances:
a. when employees have been terminated for authorized causes, such as redundancy, retrenchment or
installation of labor-saving devices;
b. when employees have been terminated for a just cause other than serious misconduct or an act
reflecting on moral character and social justice calls for the awarding of separation pay;
c. when it has become an established practice of the company to pay the said benefit to voluntarily
resigning employees; or
d. when an employee has been validly dismissed for non-membership in a union as required in a closed-
shop agreement
FACTS:
When he was dismissed on 5 April 1993, Albarico was a regular employee of 7K Corporation, a company selling
water purifiers. He started working for the company in 1990 as a salesman. Because of his good performance,
his employment was regularized. He was also promoted several times: from salesman, he was promoted to
senior sales representative and then to acting team field supervisor. In 1992, he was awarded the Presidents
Trophy for being one of the companys top water purifier specialist distributors.

In April of 1993, the chief operating officer of 7K Corporation terminated Albaricos employment allegedly for his
poor sales performance. Albarico had to stop reporting for work, and he subsequently submitted his money
claims against 7K Corporation for arbitration before the National Conciliation and Mediation Board (NCMB). The
issue for voluntary arbitration before the NCMB, according to the parties Submission Agreement was whether
Albarico was entitled to the payment of separation pay and the sales commission reserved for him by the
corporation.

As for its defense, 7K Corporation claimed Albarico had voluntarily stopped reporting for work after receiving a
verbal reprimand for his sales performance; hence, it was he who was guilty of abandonment of employment

While the case was pending before the NCMB, Albarico filed a complaint for illegal dismissal before the LA. The
latter ruled in favor of Albarico. However, the NLRC, on appeal, vacated the decision of the LA on the ground of
forum-shopping, without prejudice to the pending NCMB arbitration case. The decision of the NLRC became
final.

NCMB RULING: Albarico was ILLEGALLY DISMISSED


The arbitrator explained that the promotions, increases in salary, and awards received by respondent belied the
claim that the latter was performing poorly. It was also found that Albarico could not have abandoned his job, as
the abandonment should have been clearly shown. The VA also found that Albarico was dismissed from his
work without due process.

However, it was found that reinstatement was no longer possible because of the strained relationship of the
parties. Thus, in lieu of reinstatement, the VA ordered 7K Corporation to pay separation pay for two years at
P4,456 for each year, or a total amount of P8,912. The VA also ordered 7K Corporation to pay backwages in the

San Beda College of Law 264


4S: 2015 - 2016
LABOR LAW REVIEW Atty. Joyrich Golangco

amount of P90,804.19, plus attorneys fees since Albarico had been compelled to file an action for illegal
dismissal.

7K Corporation appealed to the CA, imputing grave abuse of discretion on the part of VA for ruling on the issue
of illegal dismissal and for awarding payment of backwages and attorneys fees. 7K Corporation contended that
the issue of the legality of dismissal was not explicitly included in the Submission Agreement.

CA RULING: AFFIRMED VA; Deleted Attorneys Fees for lack of factual basis.

ISSUE: Did the VA properly assume jurisdiction to decide the issue of the legality of the dismissal of Albarico as
well as the latters entitlement to backwages?

SC RULING:
YES. The circumstances of the instant case lead to no other conclusion than that the claim of Albarico for
separation pay was premised on his allegation of illegal dismissal. Thus, the VA properly assumed jurisdiction
over the issue of the legality of his dismissal

Moreover, it should be noted that even the NLRC was of the understanding that the NCMB arbitration case
sought to resolve the issue of the legality of the dismissal of the Albarico. In fact, the identity of the issue of the
legality of his dismissal, which was previously submitted to the NCMB, and later submitted to the NLRC, was the
basis of the latters finding of forum shopping and the consequent dismissal of the case before it. In fact, 7K
Corporation also implicitly acknowledged this when it filed before the NLRC its Motion to Dismiss Albaricos
Complaint on the ground of forum shopping. Thus, it is now estopped from claiming that the issue before the
NCMB does not include the issue of the legality of the dismissal of respondent. Besides, there has to be a
reason for deciding the issue of respondents entitlement to separation pay. To think otherwise would lead to
absurdity, because the voluntary arbitrator would then be deciding that issue in a vacuum. The arbitrator would
have no basis whatsoever for saying that Albarico was entitled to separation pay or not if the issue of the legality
of Albaricos dismissal was not resolve first.

San Beda College of Law 265


4S: 2015 - 2016
LABOR LAW REVIEW Atty. Joyrich Golangco

129. VIRGILIO KAWACHI, et al. v. DOMINIE DEL QUERO
GR No. 163768 March 27, 2007
TINGA, J.:

LA STILL HAS JURISDICTION OVER CLAIMS FOR DAMAGES ARISING FROM INCIDENTS WITH
REASONABLE CAUSAL CONNECTION WITH EMPLOYEE-EMPLOYER RELATIONSHIP

FACTS:
Kawachi hired Del Quero as a clerk of A/J Raymundo Pawnshop, Inc. On August 10, 2002, Kawachi scolded
Del Quero in front of many people about the way she treated the customers of the pawnshop and afterwards
terminated Del Quero from employment without affording her due process. Del Quero charged Virgilio Kawachi,
Julius Kawachi and A/J Raymundo Pawnshop, Inc., with illegal dismissal, non-execution of a contract of
employment, violation of minimum wage law, and non-payment of overtime pay. A few months after, Del Quero
filed an action for damages against Virgilio and Julius Kawachi before the MeTC of Quezon City. Del Quero
claimed that the August 10, 2002 incident had caused her to suffer serious embarrassment and shame so that
she could not do anything but cry because of the shameless way by which she was terminated from the service.
The Kawachis then moved for the dismissal of the complaint on the grounds of lack of jurisdiction and forum-
shopping or splitting causes of action.

MeTC RULING: DENIED the Motion for Dismissal


It ruled that no causal connection appeared between Del Queros cause of action and the employer-employee
relations between the parties.

The Kawachis filed a petition for certiorari.

RTC RULING: AFFIRMED the MeTC


It upheld the jurisdiction of the MeTC over Del Queros complaint for damages. The employees action for
damages based on slanderous remarks uttered by the employer was within the regular courts jurisdiction since
the complaint did not allege any unfair labor practice on the part of the employer.

ISSUE: Do the regular courts have jurisdiction over the claim for damages?

SC RULING:
NO. The NLRC has jurisdiction over Del Queros complaint for illegal dismissal and damages arising therefrom.
She cannot be allowed to file separate or independent civil action for damages where the alleged injury has a
reasonable connection to her termination from employment. Consequently, the action for damages filed before
the MeTC must be dismissed.

Jurisprudence has developed the reasonable causal connection rule. Under this rule, if there is a reasonable
causal connection between the claim asserted and the employer-employee relations, then the case is within the
jurisdiction of the labor courts; in the absence of such nexus, it is the regular courts that have jurisdiction. In the
instant case, the allegations of Del Quero in her complaint for damages show that her injury was the offshoot of
Kawachis immediate harsh reaction as her administrative superior to the supposedly sloppy manner by which
she had discharged her duties. The allegations in Del Queros complaint unmistakably relate to the manner of
her alleged illegal dismissal.

The Court further notes that for a single cause of action, the dismissed employee cannot be allowed to sue in
two forums: one, before the labor arbiter for reinstatement and recovery of back wages; and two, before a court
of justice for recovery of damages. Suing in the manner described is known as splitting a cause of action, a
practice engendering multiplicity of actions.

San Beda College of Law 266


4S: 2015 - 2016
LABOR LAW REVIEW Atty. Joyrich Golangco

130. GILDA G. LUNZAGA v. ALBAR SHIPPING AND TRADING CORP. AND/OR AKIRA KATO, AND
DARWIN, VENUS, ROMEO ULYSSES, MARIKIT ODESSA, ALL SURNAMED LUNZAGA (Lunzaga Siblings)
G.R. No. 200476 April 18, 2012

RELAXATION OF THE TECHNICAL RULES (1-DAY LATE IN FILING AN APPEAL)

DOCTRINE:
It has been said this time and again that the perfection of an appeal within the period fixed by the rules is
mandatory and jurisdictional. But, it is always in the power of this Court to suspend its own rules, or to except a
particular case from its operation, whenever the purposes of justice require it. Strong compelling reasons such
as serving the ends of justice and preventing a grave miscarriage thereof warrant the suspension of the rules.

FACTS:
Romeo Lunzaga was a seaman working for Albar Shipping. On June 11, 2008, Romeo was assigned as Chief
Engineer on board Albar's Philippine vessel MV Lake Aru. One month later, Romeo suffered a heart attack and
was repatriated to the Philippines only to die on September 5, 2008.

Sometime in early 2009, Gilda, claiming to be the surviving spouse of Romeo, filed with the NLRC a complaint
against Albar Shipping for payment of death benefits, damages and attorney's fees. It should be noted that Gilda
was the designated heir in Romeo's Overseas Filipino Worker Verification Sheet and PhilHealth Information
Sheet. The Lunzaga sibling, children of Romeo from his first marriage that was judicially declared null and void,
opposed the complaint through a complaint-in-intervention. The Lunzaga siblings claimed that Gilda is not
entitled to the death benefits of Romeo, as she had a subsisting marriage when she married him. They claim
that her marriage with Romeo was, therefore, bigamous. . During the mandatory conferences of the parties
before the Labor Arbiter, Albar Shipping signified its willingness to pay Romeo's death benefits in the amount of
USD 55,547.44. However, Gilda and the Lunzaga siblings could not agree as to the sharing of the benefits.

LA RULING:
The Labor Arbiter issued an Order temporarily dismissing the complaint and directing the parties to file their
case with the regular courts.

Gilda appealed to the NLRC, however, the same was made one day past the 10-day period for filing an appeal
from the decision of the Labor Arbiter

NLRC RULING: DISMISSED for filing beyond the regalamentary period.

CA RULING: AFFIRMED the decision of the NLRC


The CA ruled that despite the fact that the appeal to the NLRC was filed only one day beyond the reglementary
period, Gilda failed to present any reason for the liberal application of the rule on filing of appeals.

ISSUE: Did the NLRC and the CA err in not giving due course to the appeal due to a one (1)-day delay of its
filing?

SC RULING:
YES. Considering that the issue on whether the heirs of Romeo are entitled to receive his death benefits from
Albar Shipping properly falls under the jurisdiction of the LA, the NLRC and the CA should have had relaxed the
rigid application of the rules of procedure to afford the parties the opportunity to fully ventilate their cases on the
merits. This is in line with the time honored principle that cases should be decided only after giving all parties the
chance to argue their causes and defenses. Technicality and procedural imperfections should thus not serve as
bases of decisions. In that way, the ends of justice would be better served. For indeed, the general objective of
procedure is to facilitate the application of justice to the rival claims of contending parties, bearing always in
mind that procedure is not to hinder but to promote the administration of justice.

Verily, Albar Shipping is liable to the heirs of Romeo for the amount of USD 55,547.44. Albar hereby is ordered
to deposit this amount in an escrow account under the control of the NLRC in order to protect the interests of
Romeo's heirs. The parties claiming to be the beneficiaries of Romeo are directed to file the appropriate action
with a trial court.

San Beda College of Law 267


4S: 2015 - 2016
LABOR LAW REVIEW Atty. Joyrich Golangco

131. AMECOS INNOVATIONS, INC. and ANTONIO F. MATEO v. ELIZA R. LOPEZ
G.R. No.178055 July 2, 2014

LA HAS JURISDICTION OVER CASES INVOLVING REIMBURSEMENT OF SSS CONTRIBUTION

FACTS:
Amecos is a corporation engaged in the business of selling assorted products. In 2003, a complaint was filed by
the SSS against Amecos for an alleged delinquency in the remittance of SSS contributions and penalty liabilities
in violation of Section 22(a) and 22(d) in relation to Section 28(e) of the SSS law, as amended.

By way of explanation, Amecos claimed that it hired Lopez as Marketing Assistant to promote its products; that
upon hiring, Lopez refused to provide Amecos with her SSS Number and to be deducted her contributions; that
on the basis of the foregoing, Amecos no longer enrolled Lopez with the SSS and did not deduct her
corresponding contributions up to the time of her termination in February 2002.

Amecos eventually settled its obligations with the SSS; consequently, SSS filed a Motion to Withdraw
Complaint, which was approved by the Office of the City Prosecutor.

Thereafter, Amecos sent a demand letter to Lopez for P27,791.65 representing her share in the SSS
contributions and expenses for processing, but to no avail. Thus, Amecos filed a complaint for sum of money
and damages against Lopez before the MeTC.

Lopez filed her Answer with Motion to Dismiss claiming, among others, that the regular courts do not have
jurisdiction over the instant case as it arose out of their employer-employee relationship.

MeTC RULING: DISMISSED for lack of jurisdiction

RTC RULING: AFFIRMED the MeTC

CA RULING: AFFIRMED the RTC

ISSUE: Does the LA have jurisdiction over cases involving the reimbursement of SSS contribution paid by the
Amecos in behalf of Lopez?

SC RULING:
YES. The LA has original and exclusive jurisdiction over the matter, since the same necessarily flowed from the
employer-employee relationship between Amecos and Lopez. In this connection, it is noteworthy to state that
"the Labor Arbiter has jurisdiction to award not only the reliefs provided by labor laws, but also damages
governed by the Civil Code."

At the same time, it cannot be assumed that since the dispute concerns the payment of SSS premiums,
Amecos claim should be referred to the Social Security Commission (SSC). As far as SSS is concerned, there
is no longer a dispute with respect to Amecos accountability to the System; Amecos already settled their
pecuniary obligations to it. Since there is no longer any dispute regarding coverage, benefits, contributions and
penalties to speak of, the SSC need not be unnecessarily dragged into the picture. Besides, it cannot be made
to act as a collecting agency for petitioners claims against the respondent; the Social Security Law should not
be so interpreted, lest the SSC be swamped with cases of this sort.

At any rate, the complaint shall be dismissed for lack of cause of action. Since Amecos did not remit the full SSS
contributions of Lopez, the latter was never covered by and protected under the System. If she was never
covered by the System, certainly there is no sense in making her answerable for the required contributions
during the period of her employment. And it follows as a matter of consequence that claims for other damages
founded on the foregoing non-existent cause of action should likewise fail.

San Beda College of Law 268


4S: 2015 - 2016
LABOR LAW REVIEW Atty. Joyrich Golangco

ARTICLE 218 (now Art. 225) Powers of the Commission

132. PHILIPPINE AIRLINES, INC. v. NLRC, FERDINAND PINEDA and GOGFREDO CABLING
G.R. No. 120567 March 20, 1998
MARTINEZ, J.:

INJUNCTION CAN ONLY BE AN ANCILLARY WRIT IN ORDINARY LABOR DISPUTES.

DOCTRINE:
The power of the NLRC to issue an injunctive writ originates from any labor dispute upon the application by a
party thereof, which application if not granted may cause grave and irreparable damage to any party or render
ineffectual any decision in favor of such party. The term labor dispute is defined as any controversy or matter
concerning terms and conditions of employment x x x x. The term controversy is likewise defined as a
litigated question or a justiciable controversy. A justiciable controversy is one involving an active antagonistic
assertion of a legal right on one side and a denial thereof on the other concerning a real, and not a mere
theoretical question or issue. Given the definitions, it is thus essential that there must be a labor dispute
between the contending parties before the LA to enable the NLRC issue a injunction writ.

FACTS:
Pineda and Cabling were flight stewards of PAL. Both were dismissed from service for their alleged smuggling
in Hong Kong of a bag said to contain some PHP2.5 Million in cash. Instead of filing a case for illegal dismissal,
Pineda and Cabling filed a Petition for Injunction, with a prayer for the issuance of TRO, against PAL before the
NLRC, seeking to prohibit PAL from enforcing their Order of Dismissal against them and to ultimately reinstate
them upon a favorable decision.

NLRC RULING: TRO GRANTED


The NLRC adopted the view that Pineda and Cabling have been illegally dismissed, for the reason that PALs
Code of Discipline was formulated without the participation of its employees. The baseless dismissal has
caused Pineda and Cabling grave and irreparable injury with no speedy and adequate remedy at law.

PAL filed the present petition for certiorari.

ISSUE: Can the NLRC issue an injunctive writ even without a complaint for illegal dismissal before the LA?

SC RULING:
NO. The power of the NLRC to issue an injunctive writ originates from any labor dispute, which means that
there must be an existing controversy or a litigated question before it can issue the same. Since there is no
labor dispute between the parties as there has yet been no complaint for illegal dismissal filed before the labor
arbiter by Pineda and Cabling against PAL, the NLRC cannot, therefore, issue the assailed Order.

Contrary to the findings of the NLRC, there is no grave and irreparable damage in this case because Pineda
and Cabling can be adequately compensated if they are indeed illegally dismissed. It cannot be also said that
there is no adequate remedy because Pineda and Cabling can still file a complaint for illegal dismissal with the
LA.

It should also be noted that the Petition for Injunction filed before the NLRC is really in the nature of an action for
illegal dismissal. As such, it falls under the original and exclusive jurisdiction of the LA. The NLRC cannot
therefore entertain the petition since it only exercises appellate jurisdiction over illegal dismissal cases.

San Beda College of Law 269


4S: 2015 - 2016
LABOR LAW REVIEW Atty. Joyrich Golangco

133. LAND BANK OF THE PHILIPPINES v. SEVERINO LISTANA, SR.
G.R. No. 152611 August 5, 2003
YNARES-SANTIAGO, J.:

CONTEMPT POWERS OF QUASI-JUDICIAL AGENCIES

DOCTRINE:
Evidently, quasi-judicial agencies that have the power to cite persons for indirect contempt pursuant to Rule 71
of the Rules of Court can only do so by initiating them in the proper Regional Trial Court. It is not within their
jurisdiction and competence to decide the indirect contempt cases. These matters are still within the province of
the Regional Trial Courts.

FACTS:
Respondent Severino Listana is the owner of a parcel of land containing an area of 246.0561 hectares, located
in Inlagadian, Casiguran, Sorsogon. He voluntarily offered to sell the said land to the government, through the
Department of Agrarian Reform (DAR), under Section 20 of R.A. 6657, also known as the Comprehensive
Agrarian Reform Law of 1988 (CARL). The DAR valued the property at P5,871,689.03, which was however
rejected by the respondent. Hence, the Department of Agrarian Reform Adjudication Board (DARAB) of
Sorsogon commenced summary administrative proceedings to determine the just compensation of the land.

The DARAB rendered a Decision, setting aside the prior valuation made by the Land Bank and made a new
valuation in the amount of P10,956,963.25 for the acquired area of 240.9066 hectares. A Writ of Execution was
issued by the PARAD directing the manager of Land Bank to pay the respondent the aforesaid amount as just
compensation in the manner provided by law.
Respondent filed a Motion for Contempt with the PARAD, alleging that petitioner Land Bank failed to comply
with the Writ of Execution. He argued that such failure of the petitioner to comply with the writ of execution
constitutes contempt of the DARAB. The PARAD issued an Order granting the Motion for Contempt and issued
an arrest order against petitioners Manager Alex A. Lorayes.

ISSUE: Is the order for the arrest of petitioners manager, Mr. Alex Lorayes, by the PARAD, valid?

SC RULING:
NO. Rule 71, Section 12 of the 1997 Rules of Civil Procedure, referring to indirect contempt against quasi-
judicial entities, provides:
Sec. 12. Contempt against quasi-judicial entities. Unless otherwise provided by law, this Rule shall apply to
contempt committed against persons, entities, bodies or agencies exercising quasi-judicial functions, or shall
have suppletory effect to such rules as they may have adopted pursuant to authority granted to them by law to
punish for contempt. The Regional Trial Court of the place wherein the contempt has been committed shall
have jurisdiction over such charges as may be filed therefore.

Evidently, quasi-judicial agencies that have the power to cite persons for indirect contempt pursuant to Rule 71
of the Rules of Court can only do so by initiating them in the proper Regional Trial Court. It is not within their
jurisdiction and competence to decide the indirect contempt cases. These matters are still within the province of
the Regional Trial Courts. In the present case, the indirect contempt charge was filed, not with the Regional Trial
Court, but with the PARAD, and it was the PARAD that cited Mr. Lorayes with indirect contempt.

Hence, the contempt proceedings initiated through an unverified Motion for Contempt filed by the respondent
with the PARAD were invalid for the following reasons: First, the Rules of Court clearly require the filing of a
verified petition with the Regional Trial Court, which was not complied with in this case. The charge was not
initiated by the PARAD motu proprio; rather, it was by a motion filed by respondent. Second, neither the PARAD
nor the DARAB have jurisdiction to decide the contempt charge filed by the respondent. The issuance of a
warrant of arrest was beyond the power of the PARAD and the DARAB. Consequently, all the proceedings that
stemmed from respondents Motion for Contempt, specifically the Orders of the PARAD dated August 20, 2000
and January 3, 2001 for the arrest of Alex A. Lorayes, are null and void.

San Beda College of Law 270


4S: 2015 - 2016
LABOR LAW REVIEW Atty. Joyrich Golangco

134. FEDERICO S. ROBOSA, et. al. v. NATIONAL LABOR RELATIONS COMMISSION
G.R. No. 176085 February 8, 2012
BRION, J.:

CONTEMPT POWERS OF THE NLRC

DOCTRINE:
Under Article 218(d) of the Labor Code, the labor arbiter or the Commission is empowered or has jurisdiction to
hold the offending party or parties in direct or indirect contempt.

FACTS:
Petitioners were rank-and-file employees of respondent Chemo-Technische Manufacturing, Inc. (CTMI), the
manufacturer and distributor of Wella products. They were officers and members of the CTMI Employees Union-
DFA (union). Sometime in the first semester of 1991, the union filed a petition for certification election at CTMI.

On July 15, 1991, CTMI issued two memoranda, which were considered as union busting acts constituting
unfair labor practice by the union. Thus, the union asked for the withdrawal and deferment of CTMIs directives.
CTMI ignored the request. Instead, it issued on July 23, 1991 a notice of termination of employment to the sales
drivers, due to the abolition of the sales driver positions.

The union and its affected members filed a complaint for illegal dismissal and unfair labor practice, with a claim
for damages, against private respondents CTMI, De Luzuriaga and other CTMI officers. The union also moved
for the issuance of a writ of preliminary injunction and/or temporary restraining order.

The NLRC issued a TRO, directing CTMI, De Luzuriaga and other company executives to cease and desist
from dismissing any member of the union and from implementing the July 23, 1991 memorandum terminating
the services of the sales drivers, and to immediately reinstate them if the dismissals have been effected.

Allegedly, the respondents did not comply with the NLRCs August 23, 1991 resolution. They instead moved to
dissolve the TRO and opposed the unions petition for preliminary injunction. The NLRC upgraded the TRO to a
writ of preliminary injunction. The respondents moved for reconsideration. The union opposed the motion and
urgently moved to cite the responsible CTMI officers in contempt of court.

Private respondent De Luzuriaga argued that they were charged with indirect contempt which may be initiated
only in the appropriate regional trial court, pursuant to Section 12, Rule 71 of the Rules of Court. He posits that
the NLRC has no jurisdiction over an indirect contempt charge. He thus argues that the petitioners improperly
brought the contempt charge before the NLRC.

ISSUE: Does the NLRC (and labor arbiters) have contempt powers?

SC RULING:
YES. Under Article 218 of the Labor Code, the NLRC (and the labor arbiters) may hold any offending party in
contempt, directly or indirectly, and impose appropriate penalties in accordance with law. The penalty for direct
contempt consists of either imprisonment or fine, the degree or amount depends on whether the contempt is
against the Commission or the labor arbiter. The Labor Code, however, requires the labor arbiter or the
Commission to deal with indirect contempt in the manner prescribed under Rule 71 of the Rules of Court.

Rule 71 of the Rules of Court does not require the labor arbiter or the NLRC to initiate indirect contempt
proceedings before the trial court. This mode is to be observed only when there is no law granting them
contempt powers. As is clear under Article 218(d) of the Labor Code, the labor arbiter or the Commission is
empowered or has jurisdiction to hold the offending party or parties in direct or indirect contempt. The
petitioners, therefore, have not improperly brought the indirect contempt charges against the respondents before
the NLRC.

San Beda College of Law 271


4S: 2015 - 2016
LABOR LAW REVIEW Atty. Joyrich Golangco

ARTICLE 221 (now Art. 227) Technical Rules Not Binding

135. MANILA ELECTRIC COMPANY v. JAN CARLO GALA,


G.R. Nos. 191288 & 191304 March 7, 2012
BRION, J.:

TECHNICAL RULES NOT BINDING IN LABOR CASES

DOCTRINE:
It is the spirit and intention of labor legislation that the NLRC and the labor arbiters shall use every
reasonable means to ascertain the facts in each case speedily and objectively, without regard to
technicalities of law or procedure, provided due process is duly observed.

FACTS:
On March 2, 2006, respondent Jan Carlo Gala commenced employment with the petitioner Meralco
Electric Company (Meralco) as a probationary lineman.

On July 27, 2006, barely four months on the job, Gala was dismissed for alleged complicity in pilferages of
Meralcos electrical supplies, particularly, for the incident which took place on May 25, 2006. On that day, Gala
and other Meralco workers were instructed to replace a worn-out electrical pole at the Pacheco Subdivision in
Valenzuela City. While the Meralco crew was at work, one Noberto Bing Llanes, a non-Meralco employee,
arrived. He appeared to be known to the Meralco foremen as they were seen conversing with him. Llanes
boarded the trucks, without being stopped, and took out what were later found as electrical supplies. Aside from
Gala, the foremen and the other linemen who were at the worksite when the pilferage happened were later
charged with misconduct and dishonesty for their involvement in the incident. Unknown to Gala and the rest of
the crew, a Meralco surveillance task force was monitoring their activities and recording everything with a Sony
video camera.

Meralco called for an investigation of the incident and asked Gala to explain. Gala denied involvement in the
pilferage, contending that even if his superiors might have committed a wrongdoing, he had no participation in
what they did. Despite Galas explanation, Meralco proceeded with the investigation and eventually terminated
his employment on July 27, 2006. Gala responded by filing an illegal dismissal complaint against Meralco.

LA RULING: dismissed the complaint for lack of merit.

NLRC RULING: reversed the labor arbiters ruling. It found that Gala had been illegally dismissed.

CA RULING: concurred with the NLRC that Gala had been illegally dismissed.

ISSUES:
1. Should the Court dismiss the petition outright based on procedural grounds?
2. Was Gala illegally dismissed by petitioner Meralco?

SC RULING:
1. NO. Gala would want the petition to be dismissed outright on procedural grounds, claiming that the
Verification and Certification, Secretarys Certificate and Affidavit of Service accompanying the petition do not
contain the details of the Community Tax Certificates of the affiants, and that the lawyers who signed the
petition failed to indicate their updated MCLE certificate numbers, in violation of existing rules.

We stress at this point that it is the spirit and intention of labor legislation that the NLRC and the labor arbiters
shall use every reasonable means to ascertain the facts in each case speedily and objectively, without regard
to technicalities of law or procedure, provided due process is duly observed. In keeping with this policy and in
the interest of substantial justice, we deem it proper to give due course to the petition, especially in view of
the conflict between the findings of the labor arbiter, on the one hand, and the NLRC and the CA, on the
other. As we said in S.S. Ventures International, Inc. v. S.S. Ventures Labor Union, the application of
technical rules of procedure in labor cases may be relaxed to serve the demands of substantial justice.

2. NO. Contrary to the conclusions of the CA and the NLRC, there is substantial evidence supporting Meralcos
position that Gala had become unfit to continue his employment with the company. Gala was found, after an
San Beda College of Law 272
4S: 2015 - 2016
LABOR LAW REVIEW Atty. Joyrich Golangco

administrative investigation, to have failed to meet the standards expected of him to become a regular
employee and this failure was mainly due to his undeniable knowledge, if not participation, in the pilferage
activities done by their group, all to the prejudice of the Companys interests.

San Beda College of Law 273


4S: 2015 - 2016
LABOR LAW REVIEW Atty. Joyrich Golangco

136. NATIONWIDE SECURITY ANDALLIED SERVICES, INC. v. COURT OF APPEALS
G.R. No. 155844 July 14, 2008
QUISUMBING, J.:

REGLEMENTARY PERIOD FOR APPEAL MUST BE STRICTLY FOLLOWED

DOCTRINE:
The right to appeal is a statutory right and one who seeks to avail of the right must comply with the statute or the
rules. The rules, particularly the requirements for perfecting an appeal within the reglementary period specified
in the law, must be strictly followed as they are considered indispensable interdictions against needless delays
and for the orderly discharge of judicial business. It is only in highly meritorious cases that this Court will opt not
to strictly apply the rules and thus prevent a grave injustice from being done.

FACTS:
Labor Arbiter Manuel M. Manansala found petitioner Nationwide Security and Allied Services, Inc., a security
agency, not liable for illegal dismissal in NLRC NCR 00-01-00833-96 and 00-02-01129-96 involving eight
security guards who were employees of the petitioner. However, the Labor Arbiter directed the petitioner to pay
the aforementioned security guards P81,750.00 in separation pay, P8,700.00 in unpaid salaries, P93,795.68 for
underpayment and 10% attorneys fees based on the total monetary award.

Dissatisfied with the decision, petitioner appealed to the NLRC which dismissed its appeal for two reasons first,
for having been filed beyond the reglementary period within which to perfect the appeal and second, for filing an
insufficient appeal bond.

Petitioner then appealed to the Court of Appeals to have the appeal resolved on the merits rather than on pure
technicalities in the interest of due process. The Court of Appeals dismissed the case, holding that in a special
action for certiorari, the burden is on petitioner to prove not merely reversible error, but grave abuse of discretion
amounting to lack of or excess of jurisdiction on the part of public respondent NLRC.

ISSUE: Should technicalities in labor cases prevail over the spirit and intention of the Labor Code?

SC RULING:
After considering all the circumstances in this case and the submission by the parties, we are in agreement that
the petition lacks merit.

There being a remedy of appeal via petition for review under Rule 45 of the Rules of Court available to the
petitioner, the filing of a petition for certiorari under Rule 65 is improper. But even if we bend our Rules to allow
the present petition for certiorari, still it will not prosper because we do not find any grave abuse of discretion
amounting to lack of or excess of jurisdiction on the part of the Court of Appeals when it dismissed the petition of
the security agency. The assailed decision by the Court of Appeals was certainly not capricious nor arbitrary,
nor was it a whimsical exercise of judgment amounting to a lack of jurisdiction.

The appeal to the NLRC should have been perfected, as provided by its Rules, within a period of 10 days from
receipt by petitioner of the decision on July 16, 1999. Clearly, the filing of the appeal--three days after July 26,
1999--was already beyond the reglementary period and in violation of the NLRC Rules and the pertinent Article
on Appeal in the Labor Code.

Failure to perfect an appeal renders the decision final and executory. The right to appeal is a statutory right and
one who seeks to avail of the right must comply with the statute or the rules. The rules, particularly the
requirements for perfecting an appeal within the reglementary period specified in the law, must be strictly
followed as they are considered indispensable interdictions against needless delays and for the orderly
discharge of judicial business. It is only in highly meritorious cases that this Court will opt not to strictly apply the
rules and thus prevent a grave injustice from being done. The exception does not obtain here. Thus, we are in
agreement that the decision of the Labor Arbiter already became final and executory because petitioner failed to
file the appeal within 10 calendar days from receipt of the decision.

Clearly, the NLRC committed no grave abuse of discretion in dismissing the appeal before it. It follows that the
Court of Appeals, too, did not err, nor gravely abuse its discretion, in sustaining the NLRC Order, by dismissing the
petition for certiorari before it.
San Beda College of Law 274
4S: 2015 - 2016
LABOR LAW REVIEW Atty. Joyrich Golangco

San Beda College of Law 275


4S: 2015 - 2016
LABOR LAW REVIEW Atty. Joyrich Golangco

137. DIAMOND TAXI and/or BRYAN ONG v. FELIPE LLAMAS, JR.
G.R. No. 190724 March 12, 2014
BRION, J.:

EMPLOYEES APPEAL MUST NOT BE DISMISSED ON PURELY TECHINCAL GROUNDS

DOCTRINE:
The dismissal of an employees appeal on purely technical ground is inconsistent with the constitutional
mandate on protection to labor. Under the Constitution and the Labor Code, the State is bound to protect labor
and assure the rights of workers to security of tenure tenurial security being a preferred constitutional right
that, under these fundamental guidelines, technical infirmities in labor pleadings cannot defeat.

FACTS:
Felipe Llamas worked as a taxi driver for petitioner Diamond Taxi, owned and operated by petitioner Bryan Ong.
Llamas filed before the Labor Arbiter (LA) a complaint for illegal dismissal against the petitioners. In their
position paper, the petitioners denied dismissing Llamas. They claimed that Llamas had been absent without
official leave for several days, and submitted a copy of the attendance logbook as proof. They also pointed out
that Llamas committed several traffic violations amounting and several acts of insubordination and refusal to
heed management instructions. They argued that these acts traffic violations, insubordination and refusal to
heed management instructions constitute grounds for the termination of Llamas employment.

Llamas failed to seasonably file his position paper. On November 29, 2005, the LA rendered a
decision dismissing Llamas complaint for lack of merit. Llamas received a copy of this LA decision on January
5, 2006. Meanwhile, he filed his position paper on December 20, 2005. Llamas claimed that his failure to file his
position paper was due to the refusal of his previous counsel to comply.

He also alleged that he had a misunderstanding with Aljuver Ong, Bryans brother and operations manager of
Diamond Taxi, and the incident led to his forced resignation. Llamas filed a motion for reconsideration before the
LA. The LA treated Llamas motion as an appeal per Section 15, Rule V of the 2005 Revised Rules of
Procedure of the NLRC (2005 NLRC Rules) (the governing NLRC Rules of Procedure at the time Llamas filed
his complaint before the LA).

NLRC RULING: The NLRC dismissed for non-perfection Llamas motion for reconsideration treated as an
appeal. The NLRC pointed out that Llamas failed to attach the required certification of non-forum shopping per
Section 4, Rule VI of the 2005 NLRC Rules.

CA RULING: The CA reversed and set aside the assailed NLRC resolution. Citing jurisprudence, the CA pointed
out that non-compliance with the requirement on the filing of a certificate of non-forum shopping, while
mandatory, may nonetheless be excused upon showing of manifest equitable grounds proving substantial
compliance.

ISSUE: Did the NLRC committed grave abuse of discretion in dismissing Llamas appeal on mere technicality?

SC RULING:
YES. Article 223 (now Article 229) of the Labor Code states that decisions (or awards or orders) of the LA shall
become final and executory unless appealed to the NLRC within ten (10) calendar days from receipt of the
decision. Consistent with Article 223, Section 1, Rule VI of the 2005 NLRC Rules also provides for a ten (10)-
day period for appealing the LAs decision. Under Section 4(a), Rule VI of the 2005 NLRC Rules, the appeal
shall be in the form of a verified memorandum of appeal and accompanied by proof of payment of the appeal
fee, posting of cash or surety bond (when necessary), certificate of non-forum shopping, and proof of service
upon the other parties. Failure of the appealing party to comply with any or all of these requisites within the
reglementary period will render the LAs decision final and executory.

Indisputably, Llamas did not file a memorandum of appeal from the LAs decision. Instead, he filed, within the
ten (10)-day appeal period, a motion for reconsideration. Under Section 15, Rule V of the 2005 NLRC Rules,
motions for reconsideration from the LAs decision are not allowed; they may, however, be treated as an appeal
provided they comply with the requirements for perfecting an appeal. The NLRC dismissed Llamas motion for
reconsideration treated as an appeal for failure to attach the required certificate of non-forum shopping per
Section 4(a), Rule VI of the 2005 NLRC Rules.
San Beda College of Law 276
4S: 2015 - 2016
LABOR LAW REVIEW Atty. Joyrich Golangco

The requirement for a sworn certification of non-forum shopping was prescribed by the Court under Revised
Circular 28-91, as amended by Administrative Circular No. 04-
94,http://www.lawphil.net/judjuris/juri2014/mar2014/gr_190724_2014.html - fnt22 to prohibit and penalize the
evils of forum shopping. Revised Circular 28-91, as amended by Administrative Circular No. 04-94, requires a
sworn certificate of non-forum shopping to be filed with every petition, complaint, application or other initiatory
pleading filed before the Court, the CA, or the different divisions thereof, or any other court, tribunal or agency.

Ordinarily, the infirmity in Llamas appeal would have been fatal and would have justified an end to the case. A
careful consideration of the circumstances of the case, however, convinces us that the NLRC should, indeed,
have given due course to Llamas appeal despite the initial absence of the required certificate. We note that in
his motion for reconsideration of the NLRCs May 30, 2006 resolution, Llamas attached the required certificate
of non-forum shopping.

Under Article 221 (now Article 227) of the Labor Code, "the Commission and its members and the Labor
Arbiters shall use every and all reasonable means to ascertain the facts in each case speedily and objectively
and without regard to technicalities of law or procedure, all in the interest of due process." Consistently, we have
emphasized that "rules of procedure are mere tools designed to facilitate the attainment of justice. A strict and
rigid application which would result in technicalities that tend to frustrate rather than promote substantial justice
should not be allowed x x x. No procedural rule is sacrosanct if such shall result in subverting
justice." Ultimately, what should guide judicial action is that a party is given the fullest opportunity to establish
the merits of his action or defense rather than for him to lose life, honor, or property on mere technicalities.

San Beda College of Law 277


4S: 2015 - 2016
LABOR LAW REVIEW Atty. Joyrich Golangco

138. SARA LEE PHILIPPINES, INC. v. EMILINDA D. MACATLANG, ET AL.
G.R. No. 180147 January 14, 2015
PEREZ, J.:

APPEAL BOND; JUDICIAL COURTESY; COMPROMISE AGREEMENT

DOCTRINE:
The NLRC retains its authority and duty to resolve the motion and determine the final amount of bond that shall
be posted by the appellant, still in accordance with the standards of "meritorious grounds" and "reasonable
amount." Should the NLRC, after considering the motions merit, determine that a greater amount or the full
amount of the bond needs to be posted by the appellant, then the party shall comply accordingly. The appellant
shall be given a period of 10 days from notice of the NLRC order within which to perfect the appeal by posting
the required appeal bond.

FACTS:
On Oct. 9, 1995, Aris Philippines, Inc. permanently ceased operations displacing 5,984 rank-and-file employees.
On Oct. 26, 1996, Fashion Accessories Phils., Inc. (FAPI) was incorporated prompting former Aris employees to
file a case for illegal dismissal alleging that FAPI was a continuing business of Aris. Sara Lee Corporation
(SLC), Sara Lee Philippines (SLP), and Atty. Cesar C. Cruz were impleaded as defendants being major
stockholders of FAPI and officers of Aris, respectively.

LA RULING: On Oct. 30, 2004, the LA found the dismissal of 5,984 Aris employees illegal and awarded them
monetary benefits amounting to P3,453,664,710.86 as separation pay, backwages, moral and exemplary
damages, and attorneys fees.

The Corporations filed a Notice of Appeal with Motion to Reduce Appeal Bond posting a P4.5 million bond.

NLRC RULING: The NLRC granted the reduction of the appeal bond and ordered the Corporations to post an
additional P4.5 million bond.

The Aris employees, represented by Emilinda Macatlang, filed a petition for review before the CA insisting that
the appeal was not perfected due to failure of the Corporations to post the correct amount of the bond which is
equivalent to the judgment award.

Nonetheless, the NLRC prematurely issued an order setting aside the LAs decision for being procedurally
infirmed.

CA RULING: On March 26, 2007, the CA ordered the Corporations to post an additional appeal bond of P1
billion.

SC RULING: On June 4, 2014, the SC ordered the Corporations to post P725 million, in case or surety bond
and vacated the NLRC decision for being premature and directed the same to act with dispatch to resolve the
merits of the case upon perfection of the appeal.

Hence, this MR where the Corporations, relying on McBurnie v. Ganzon, argued that only 10% of the monetary
award is required to be posted as bond.

Furthermore, the Corporations filed a Motion to Admit Confession of Judgment claiming that the Corporations
entered into as compromise with some of the former Aris employees.

ISSUES:
1. Is the 10% bond requirement applicable in this case?
2. Was the ruling of the NLRC premature?
3. Should the motion to admit confession of judgment be approved?

SC RULING:
(1) No. The 10% requirement pertains to the reasonable amount which the NLRC would accept as the minimum
of the bond that should accompany the motion to reduce bond in order to suspend the period to perfect an

San Beda College of Law 278


4S: 2015 - 2016
LABOR LAW REVIEW Atty. Joyrich Golangco

appeal under the NLRC rules. The 10% is based on the judgment award and should in no case be
construed as the minimum amount of bond to be posted in order to perfect appeal.

The NLRC retains its authority and duty to resolve the motion and determine the final amount of bond that
shall be posted by the appellant, still in accordance with the standards of "meritorious grounds" and
"reasonable amount." Should the NLRC, after considering the motions merit, determine that a greater
amount or the full amount of the bond needs to be posted by the appellant, then the party shall comply
accordingly. The appellant shall be given a period of 10 days from notice of the NLRC order within which to
perfect the appeal by posting the required appeal bond.

The appeal bond was set at P725 million after taking into consideration the interests of all parties. The
underlying purpose of the appeal bond is to ensure that the employer has properties on which he or she can
execute upon in the event of a final, providential award. Thus, non-payment or woefully insufficient payment
of the appeal bond by the employer frustrates these ends.

(2) Yes. There was a legal impediment for NLRC to issue the resolution vacating the LAs decision. The
principle of judicial courtesy applies if there is a strong probability that the issues before the higher court
would be renedered moot as a result of the continuation of the proceedings in the lower court.

The NLRCs ruling would moot the appeal filed before the higher courts because the issue involves the
appeal bond which is an indispensable requirement to the perfection of the appeal before the NLRC. Thus,
unless this issue is resolved, the NLRC should be precluded from ruling on the merits of the case.

Thus, the stage that has been passed in this case is the proceedings before the LA. Without the NLRC
stage, the LAs decision is final and executory.

(3) No. A confession of judgement is an acknowledgement that a debt is justly due and cuts off all defenses and
right of appeal. It is used as a shortcut to a judgment in a case where the defendant concedes liability. It is
seen as the written authority of the debtor and a director for entry of judgment against the debtor.

On the other hand, a compromise agreement is a contract whereby the parties, by making reciprocal
concessions, avoid a litigation or put an end to one already commenced. It is an agreement between two or
more persons, who, for preventing or putting an end to a lawsuit, adjust their difficulties by mutual consent in
the manner which they agree on, and which everyone of them prefers to the hope of gaining, balanced by
the danger of losing. It must not be contrary to law, morals, good customs and public policy; and must have
been freely and intelligently executed by and between the parties.

Article 227 of the Labor Code of the Philippines authorizes compromise agreements voluntarily agreed upon
by the parties, in conformity with the basic policy of the State to promote and emphasize the primacy of free
collective bargaining and negotiations, including voluntary arbitration, mediation and conciliation, as modes
of settling labor or industrial disputes.

In this case, a review of the compromise agreement shows a gross disparity between the amount offered by
the Corporations compared to the judgment award. The judgment award is P3,453,664,710.86 or each
employee is slated to receive P577,149.85. On the other hand, the P342,284,800.00 compromise is to be
distributed among 5,984 employees which would translate to only P57,200.00 per employee. From this
amount, P8,580.00 as attorneys fees will be deducted, leaving each employee with a measly P48,620.00.
In fact, the compromised amount roughly comprises only 10% of the judgment award.

The SC had already directed the NLRC to act with dispatch in resolving the merits of the case upon receipt
of the bond. If indeed the parties want an immediate and expeditious resolution of the case, then the NLRC
should be unhindered with technicalities to dispose of the case.

The petition was denied.

San Beda College of Law 279


4S: 2015 - 2016
LABOR LAW REVIEW Atty. Joyrich Golangco

ARTICLE 223 (now Art. 229) Appeal

139. ISLRIZ TRADING / VICTOR HUGO LU v. EFREN CAPADA, LAURO LICUP, NORBERTO NIGOS,
RONNIE ABEL, GODOFREDO MAGNAYE, ARNEL SIBERRE, EDMUNDO CAPADA, NOMERLITO
MAGNAYE, and ALBERTO DELA VEGA
G.R. No. 168501 January 31, 2011
DEL CASTILLO, J.:

EXTENT OF APPLICABILITY OF ART. 223

DOCTRINE:
Employees are entitled to their accrued salaries during the period between the Labor Arbiters order of
reinstatement pending appeal and the resolution of the National Labor Relations Commission (NLRC)
overturning that of the Labor Arbiter. Otherwise stated, even if the order of reinstatement of the Labor Arbiter is
reversed on appeal, the employer is still obliged to reinstate and pay the wages of the employee during the
period of appeal until reversal by a higher court or tribunal.

FACTS:
Efren Capada, Lauro Licup, Norberto Nigos, and Godofredo Magnaye were drivers while Ronnie Abel, Arnel
Siberre, Edmundo Capada, Nomerlito Magnaye, and Alberto Dela Vega were helpers of Islriz Trading, a gravel
and sand business owned and operated by Victor Hugo Lu.

Claiming that they were illegally dismissed, Capada, et al. filed a complaint for illegal dismissal and non-
payment of overtime pay,holiday pay, rest day pay, allowances, and separation pay against Islriz on August 9,
2000 before the LA. For his part, Lu imputed abandonment of work against Capada, et al.

LA & NLRC RULING: On Dec. 21, 2001, LA Waldo Emerson Gan declared Islriz Trading guilty of illegal
dismissal and ordered the reinstatement of complainants to the ir former positions and the payment of full
backwages plus 10% attorneys fees.

On Sept. 5, 2002, the NLRC set aside the LAs decision after finding that Capada, et al.s failure to continue
working for Islriz Trading was neither caused by termination nor abandonment of work and ordered
reinstatement of Capada, et al. but without backwages. On Nov. 18, 2002, MR was denied. On Dec. 7, 2002,
this became final and executory.

On Dec. 9, 2003, however, Capada, et al. filed with the LA an Ex-Parte Motion to Set Case for Conference with
Motion averring that since the LA decision ordered their reinstatement, a writ of execution dated April 22, 2002
was already issued for the enforcement of its reinstatement aspect as the same is immediately executory even
pending appeal, but Islriz still refused to reinstate them, thereby, praying that, in view of the orders of
reinstatement, a computation of the award of backwages be made and that an alias writ of execution for its
enforcement be issued.

On Jan. 29, Feb. 24, and March 5, 2004, pre-execution conferences were held but the parties failed to come to
terms on the issue of the monetary award.

The LA thus issued an undated computation of Capada, et al.s accrued salaries amounting to P1,110,665.60.
LA Danna Castillon, despite Islrizs questioning of the computation since the same was without factual or legal
basis since LA Ganss decision had already been reversed and set aside by the NLRC, issued a writ of
execution dated March 9, 2004 to enforce the monetary award. Hence, the personal property of Islriz was
levied, despite Islrizs protest, in favor of Capada, et al.

Later, Capada, et al. claimed that they could not take full control, ownership, and possession of the same
because Lu had allegedly padlocked the premises where the properties were situated. Hence, they asked LA
Castillo to issue a break-open order.

For his part, Lu filed a Motion to Quash Writ of Execution, Notice of Sale/Levy on Execution of Personal
Property and Auction Sale on Additional Grounds reiterating that since the NLRC reversed the LAs decision,
only the execution of reinstatement sans any backwages or other monetary award should be enforced.

San Beda College of Law 280


4S: 2015 - 2016
LABOR LAW REVIEW Atty. Joyrich Golangco

On June 3, 2004, LA Castillon explained that the monetary award subject of the writ refers to Capada, et al.s
accrued salaries by reason of the reinstatement order of LA Gan which is self-executory pursuant to Art. 223 of
the Labor Code.

CA RULING: On March 18, 2005, the CA agreed with the LAs ratiocination that pursuant to Art. 223 of the
Labor Code, what is sought to be enforced by the writ of execution is the accrued salaries owing to Capada, et
al. by reason of LA Gans reinstatement order. MR was also denied on June 16, 2005.

Hence, this petition for review where Lu posits that Art. 223 of the Labor Code only applies when an employee
has been illegally dismissed from work and since, Capada, et al.s failure to continue working for Islriz was not
occasioned by termination, there is no illegal dismissal to speak of, hence, Art. 223 does not apply in this case.

ISSUE: Is Art. 223 of the Labor Code applicable to this case? May Capada, et al. collect their wages during the
period between the LAs order of reinstatement pending appeal and the NLRC Resolution overturining that of
the LA?

SC RULING:
Yes. Even if the order of reinstatement of the Labor Arbiter is reversed on appeal, it is obligatory on the part of
the employer to reinstate and pay the wages of the dismissed employee during the period of appeal until
reversal by the higher court or tribunal. It likewise settled the view that the Labor Arbiters order of reinstatement
is immediately executory and the employer has to either re-admit them to work under the same terms and
conditions prevailing prior to their dismissal, or to reinstate them in the payroll, and that failing to exercise the
options in the alternative, employer must pay the employees salaries.

After the Labor Arbiters decision is reversed by a higher tribunal, the employee may be barred from collecting
the accrued wages, if it is shown that the delay in enforcing the reinstatement pending appeal was without fault
on the part of the employer. It then provided for the two-fold test in determining whether an employee is barred
from recovering his accrued wages, to wit: (1) there must be actual delay or that the order of reinstatement
pending appeal was not executed prior to its reversal; and (2) the delay must not be due to the employers
unjustified act or omission. If the delay is due to the employers unjustified refusal, the employer may still be
required to pay the salaries notwithstanding the reversal of the Labor Arbiters Decision.

In this case, (1) there was an actual delay in the execution of the reinstatement aspect of LA Gans decision
prior to the issuance of the NLRC resolution overturning the same; and (2) the delay in the execution of Capada,
et al.s reinstatement was due to Islrizs unjustified refusal to effect the same.

Therefore, Capada, et al. have the right to collect their accrued salaries during the period between the LAs
decision ordering their reinstatement pending appeal and the NLRC resolution overturning thr same.

The petition was denied.

San Beda College of Law 281


4S: 2015 - 2016
LABOR LAW REVIEW Atty. Joyrich Golangco

140. CESAR V. GARCIA, CARLOS RAZON, ALBERTO DE GUZMAN, TOMAS RAZON, OMER PALO,
RIZALDE VALENCIA, ALLAN BBASA, JESSIE GACIA, JUANITO PARAS, ALEJANDRO ORAG, ROMMEL
PANGAN, RUEL SOLIMAN, AND CENEN CANLAPAN, represented by CESAR V. GARCIA v. KJ
COMMERCIAL and REYNALDO QUE
G.R. No. 196830 February 29, 2012
CARPIO, J.:

PERFECTION OF APPEAL; APPEAL BOND

DOCTRINE:
Section 2, Article I of the Rules of Procedure of the NLRC states: These Rules shall be liberally construed to
carry out the objectives of the Constitution, the Labor Code of the Philippines and other relevant legislations,
and to assist the parties in obtaining just, expeditious and inexpensive resolution and settlement of labor
disputes. In order to give full effect to the provisions on motion to reduce bond, the appellant must be allowed to
wait for the ruling of the NLRC on the motion even beyond the 10-day period to perfect an appeal. If the NLRC
grants the motion and rules that there is indeed meritorious ground and that the amount of the bond posted is
reasonable, then the appeal is perfected. If the NLRC denies the motion, the appellant may still file a motion for
reconsideration as provided under Section 15, Rule VII of the Rules. If the NLRC grants the motion for
reconsideration and rules that there is indeed meritorious ground and that the amount of the bond posted is
reasonable, then the appeal is perfected. If the NLRC denies the motion, then the decision of the labor arbiter
becomes final and executory.

FACTS:
KJ Commercial (KJC) is a sole proprietorship which owns trucks and engages in the business of distributing
cement products. On different dates, KJC employed as truck drivers and truck helpers herein petitioners Cesar
Garcia, Carlos Razon, Alberto De Guzman, Tomas Razon, Omer Palo, Rizalde Valencia, Allan Bbasa, Jessie
Gacia, Juanito Paras, Alejandro Orag, Rommel Pangan, Ruel Soliman, and Cenen Canlapan.

On Jan. 2, 2006, Garcia, et al. demanded for a P40 daily salary increase. To pressure KJC to grant their
demand, they stopped working and abandoned their trucks at Northern Cement Plan Station in Sison,
Pangasinan.

On Feb. 3, 2006, Garcia, et al. filed with the LA a complaint for illegal dismissal, underpayment of salary, and
th
non-payment of SIL and 13 month pay.

LA RULING: On Oct. 30, 2008, the LA held that KJC illegally dismissed Garcia, et al.

KJC appealed to the NLRC. It filed a motion to reduce bond and posted a P50,000 cash bond.

NLRC RULING: On March 9, 2009, the NLRC dismissed the appeal.

KJC filed an MR and posted a P2,562,930 surety bond.

On Feb. 8, 2010, the NLRC granted the motion and set aside the LAs decision. Garcia, et al. filed an MR which
was denied on June 25, 2010.

CA RULING: On April 21, 2011, the CA affirmed the NLRCs decision. Hence, this petition for review on
certiorari.

ISSUE: Did the motion to reduce bond stop the running of the period to appeal?

SC RULING:
Yes. KJCs filing of a motion to reduce bond and delayed posting of the P2,562,930 surety bond did not render
the LAs decision final and executory.

The Rules of Procedure of the NLRC allows the filing of a motion to reduce bond subject to two conditions:
1. There is meritorious ground; and
2. A bond in a reasonable amount is posted.

San Beda College of Law 282


4S: 2015 - 2016
LABOR LAW REVIEW Atty. Joyrich Golangco

The filing of a motion to reduce bond and compliance with the two conditions stop the running of the period to
perfect an appeal. The NLRC has full discretion to grant or deny the motion to reduce bond,21 and it may rule
on the motion beyond the 10-day period within which to perfect an appeal. Obviously, at the time of the filing of
the motion to reduce bond and posting of a bond in a reasonable amount, there is no assurance whether the
appellants motion is indeed based on meritorious ground and whether the bond he or she posted is of a
reasonable amount. Thus, the appellant always runs the risk of failing to perfect an appeal.

Section 2, Article I of the Rules of Procedure of the NLRC states: These Rules shall be liberally construed to
carry out the objectives of the Constitution, the Labor Code of the Philippines and other relevant legislations,
and to assist the parties in obtaining just, expeditious and inexpensive resolution and settlement of labor
disputes. In order to give full effect to the provisions on motion to reduce bond, the appellant must be allowed to
wait for the ruling of the NLRC on the motion even beyond the 10-day period to perfect an appeal. If the NLRC
grants the motion and rules that there is indeed meritorious ground and that the amount of the bond posted is
reasonable, then the appeal is perfected. If the NLRC denies the motion, the appellant may still file a motion for
reconsideration as provided under Section 15, Rule VII of the Rules. If the NLRC grants the motion for
reconsideration and rules that there is indeed meritorious ground and that the amount of the bond posted is
reasonable, then the appeal is perfected. If the NLRC denies the motion, then the decision of the labor arbiter
becomes final and executory.

KJ Commercial filed a motion to reduce bond and posted a P50,000 cash bond. When the NLRC denied its
motion, KJ Commercial filed a motion for reconsideration and posted the full P2,562,930 surety bond. The
NLRC then granted the motion for reconsideration.

Furthermore, KJ Commercial showed willingness to post a partial bond. In fact, it posted a P50,000 cash bond.
Also, KJ Commercial immediately posted the full amount of the bond when it filed its motion for reconsideration
of the NLRCs decision.

The petition was denied. CA decision was affirmed.

San Beda College of Law 283


4S: 2015 - 2016
LABOR LAW REVIEW Atty. Joyrich Golangco

141. MARIANO ONG, doing business under the name and style MILESTONE METAL MANUFACTURING
v. THE COURT OF APPEALS, CONRADO DABAC, BERNABE TAYACTAC, MANUEL ABEJUELLA,
LOLITO ABELONG, RONNIE HERRERO, APOLLO PAMIAS, JAIME ONGUTAN, NOEL ATENDIDO,
CARLOS TABBAL, JOEL ATENDIDO, BIENVENIDO EBBER, RENATO ABEJUELLA, LEONILO ATENDIDO,
JR., LODULADO FAA and JAIME LOZADA
G.R. No. 152494 September 22, 2004
YNARES-SANTIAGO, J.:

PERFECTION OF APPEAL; APPEAL BOND

DOCTRINE:
The NLRC Rules clearly provide that the filing of the motion to reduce bond shall not stop the running of the
period to perfect appeal. Ong should have seasonably filed the appeal bond within the ten-day reglementary
period following the receipt of the order, resolution or decision of the NLRC to forestall the finality of such order,
resolution or decision. In the alternative, he should have paid only a moderate and reasonable sum for the
premium. The law does not require its outright payment, but only the posting of a bond to ensure that the award
will be eventually paid should the appeal fail. What petitioners have to pay is a moderate and reasonable sum
for the premium for such bond.

FACTS:
Mariano Ong is the sole proprietor of Milestone Metal Manufacturing, which manufactures, among others,
wearing apparels, belts, and umbrellas. Sometime in May 1998, the business suffered very low sales and
productivity because of the economic crisis in the country. Hence, it adopted a rotation scheme by reducing the
workdays of its employees to 3 days a week or less for an indefinite period.

On separate dates, the 15 respondents filed before the NLRC complaints for illegal dismissal, underpayment of
th
wages, non-payment of overtime pay, holiday pay, SIL pay, 13 month pay, damages, and attorneys fees
against Ong.

Ong claimed that:


9 of the 15 respondents were not employees of Milestone but of Proton Industrial Corporation which,
however, stopped its operation due to business losses.
Abuela, Abelong, Herrero, Tabbal, Dabac, and Faa were not dismissed from employment; rather, they
refused to work after the rotation scheme was adopted.
Anent their monetary claims, Ong presented documents showing that he paid respondents minimum wage,
th
13 month pay, holiday pay, and contributions to the SSS, Medicare, and Pag-ibig Funds.

LA RULING: On Nov. 25, 1999, LA awarded respondents P1,111,200.40 representing their wage differential,
th
holiday pay, SIL pay, and 13 month pay, plus 10% attorneys fees and ordered Ong to pay respondents
separation pay due to the indefiniteness of the rotation scheme and strained relations caused by the filing of the
complaints.

Ong filed with the NLRC a notice of appeal with a memorandum of appeal and paid the docket fees but instead
of posting the required cash or surety bond, he filed a motion to reduce the appeal bond.

NLRC RULING: On April 28, 2000, the NLRC denied the motion to reduce bond and dismissed the appeal for
failure to post cash or surety bond. MR was also denied.

CA RULING: Petition for certiorari was dismissed. MR was denied.

Hence, this petition for review where Ong contends that he was deprived of the chance to post bond because
the NLRC took 102 days to decide his motion.

ISSUE: Is the mere filing of the motion to reduce the appeal bond, without posting the required surety or cash
bond, sufficient to perfect an appeal?

SC RULING:
No. The NLRC did not act with GAD when it denied Ongs motion for the same failed to either elucidate why the
amount of the bond was unjustified and prohibitive or to indicate what would be a reasonable level. A
San Beda College of Law 284
4S: 2015 - 2016
LABOR LAW REVIEW Atty. Joyrich Golangco

substantial monetary award, even if it runs into millions, does not necessarily give the employer-appellant a
meritorious case and does not automatically warrant a reduction of the appeal bond.

Even granting arguendo that Ong has meritorious grounds to reduce the appeal bond, the result would have
been the same since he failed to post cash or surety bond within the prescribed period.

An appeal from the Labor Arbiter to the NLRC must be perfected within ten calendar days from receipt of such
decisions, awards or orders of the Labor Arbiter. In a judgment involving a monetary award, the appeal shall be
perfected only upon (1) proof of payment of the required appeal fee; (2) posting of a cash or surety bond issued
by a reputable bonding company; and (3) filing of a memorandum of appeal. A mere notice of appeal without
complying with the other requisites mentioned shall not stop the running of the period for perfection of appeal.
The posting of cash or surety bond is not only mandatory but jurisdictional as well, and non-compliance
therewith is fatal and has the effect of rendering the judgment final and executory. This requirement is intended
to discourage employers from using the appeal to delay, or even evade, their obligation to satisfy their
employees just and lawful claims.

The intention of the lawmakers to make the bond an indispensable requisite for the perfection of an appeal by
the employer is underscored by the provision that an appeal by the employer may be perfected only upon the
posting of a cash or surety bond. The word only makes it perfectly clear that the lawmakers intended the posting
of a cash or surety bond by the employer to be the exclusive means by which an employers appeal may be
perfected.

The fact that the NLRC took 102 days to resolve the motion will not help Ongs case. The NLRC Rules clearly
provide that the filing of the motion to reduce bond shall not stop the running of the period to perfect appeal.
Ong should have seasonably filed the appeal bond within the ten-day reglementary period following the receipt
of the order, resolution or decision of the NLRC to forestall the finality of such order, resolution or decision. In
the alternative, he should have paid only a moderate and reasonable sum for the premium. The law does not
require its outright payment, but only the posting of a bond to ensure that the award will be eventually paid
should the appeal fail. What petitioners have to pay is a moderate and reasonable sum for the premium for such
bond.

While the bond requirement on appeals involving monetary awards has been relaxed in certain cases, this can
only be done where there was substantial compliance of the Rules or where the appellants, at the very least,
exhibited willingness to pay by posting a partial bond.

Ong did not post a full or partial appeal bond within the prescribed period, thus, no appeal was perfected from
the decision of the Labor Arbiter. For this reason, the decision sought to be appealed to the NLRC had become
final and executory and therefore immutable. Clearly, then, the NLRC has no authority to entertain the appeal,
much less to reverse the decision of the Labor Arbiter. Any amendment or alteration made which substantially
affects the final and executory judgment is null and void for lack of jurisdiction, including the entire proceeding
held for that purpose.

The petition was denied. The CA decision was affirmed.

San Beda College of Law 285


4S: 2015 - 2016
LABOR LAW REVIEW Atty. Joyrich Golangco

142. ROSEWOOD PROCESSING, INC. v. NATIONAL LABOR RELATIONS COMMISSION, NAPOLEON C.
MAMON, ARSENIO GAZZINGAN, ROMEO C. VELASCO, ARMANDO L. BALLON, VICTOR E. ALDEZA,
JOSE L. CABRERA, VETERANS PHILIPPINE SCOUT SECURITY AGENCY, and/or ENGR. SERGIO
JAMILA IV
G.R. Nos. 116476-84 May 21, 1998
PANGANIBAN, J.:

PERFECTION OF APPEAL; APPEAL BOND; SOLIDARY LIABILITY OF INDIRECT EMPLOYER

DOCTRINES:
In a number of cases, the SC has relaxed the bond requirement in order to bring about the immediate and
appropriate resolution of controversies on the merits. Some of these cases include: (a) counsels reliance on
the footnote of the notice of the decision of the labor arbiter that the aggrieved party may appeal xxx within
ten (10) working days; (b) fundamental consideration of substantial justice; (c) prevention of miscarriage of
justice or of unjust enrichment, as where the tardy appeal is from a decision granting separation pay which
was already granted in an earlier final decision; and (d) special circumstances of the case combined with its
legal merits or the amount and the issue involved.
Under the Labor Code, an employer is solidarily liable for legal wages due security guards for the period of
time they were assigned to it by its contracted security agency. However, in the absence of proof that the
employer itself committed the acts constitutive of illegal dismissal or conspired with the security agency in
the performance of such acts, the employer shall not be liable for back wages and/or separation pay arising
as a consequence of such unlawful termination.

FACTS:
On May 13, 1991, a complaint for illegal dismissal, underpayment of wages, and non-payment of overtime pay,
th
holiday pay, premium pay for holiday and rest day, and 13 month pay, cash bond deposit, unpaid wages, and
damages was filed aganst Veterans Philippine Scout Security Agency and/or Sergio Jamila IV. Thereafter,
Rosewood Processing, Inc. (RPI) was impleaded as a third-party respondent by the security agency.

LA RULING: On March 26, 1993, the security agency, Jamila, and RPI were ordered to pay jointly and severally
the complainants the aggregate amount of P789,154.39 plus attorneys fees on the basis that the security
agency and RPI offered no evidence refuting or rebutting the complainants computation of monetary claims.

NLRC RULING: On April 28, 1994, the appeal was dismissed for failure of RPI to file the required appeal bond
within the reglementary period, thereby rendering the LAs decision final and executory as of April 23, 1993. MR
was likewise denied.

Hence, this petition.

ISSUE:
1. Was the appeal from the LA to the NLRC perfected on time?
2. Is RPI solidarily liable with the security agency for the payment of back wages, wage differential, and
separation pay.

SC RULING:
(1) Yes. The perfection of an appeal within the reglementary period and in the manner prescribed by law is
jurisdictional, and non-compliance with such legal requirement is fatal and effectively renders the judgment final
and executory. The appeal of a decision involving a monetary award in labor cases may be perfected only upon
the posting of a cash or surety bond. The lawmakers intended the posting of the bond to be an indispensable
requirement to perfect an employers appeal.

However, in a number of cases, the SC has relaxed this requirement in order to bring about the immediate and
appropriate resolution of controversies on the merits. Some of these cases include: (a) counsels reliance on the
footnote of the notice of the decision of the labor arbiter that the aggrieved party may appeal xxx within ten (10)
working days; (b) fundamental consideration of substantial justice; (c) prevention of miscarriage of justice or of
unjust enrichment, as where the tardy appeal is from a decision granting separation pay which was already
granted in an earlier final decision; and (d) special circumstances of the case combined with its legal merits or
the amount and the issue involved.

San Beda College of Law 286


4S: 2015 - 2016
LABOR LAW REVIEW Atty. Joyrich Golangco

In this case, RPI claims to have received a copy of the LAs decision only on April 6, 1993, and that it filed on
April 16, 1993, within the prescribed time, a Notice of Appeal with a Memorandum on Appeal, a Motion to
Reduce Appeal Bond, and a surety bond issued by Prudential Guarantee and Assurance Inc. in the amount of
P50,000. Ignoring RPIs motion to reduce bond, the NLRC rendered its assailed resolution dismissing the
appeal due to the late filing of the appeal bond.

RPIs motion to reduce the bond is a substantial compliance with the Labor Code. Letter-perfect rules must yield
to the broader interest of substantial justice.

(2) Yes. Notwithstanding the service contract between the petitioner and the security agency, the former is still
solidarily liable to the employees, who were not privy to said contract, pursuant to the aforecited provisions of
the Code. Labor standard legislations are enacted to alleviate the plight of workers whose wages barely meet
the spiraling costs of their basic needs.

They are considered written in every contract, and stipulations in violation thereof are considered not written.
Similarly, legislated wage increases are deemed amendments to the contract. Thus, employers cannot hide
behind their contracts in order to evade their or their contractors or subcontractors liability for noncompliance
with the statutory minimum wage.

The joint and several liability of the employer or principal was enacted to ensure compliance with the provisions
of the Code, principally those on statutory minimum wage. The contractor or subcontractor is made liable by
virtue of his or her status as a direct employer, and the principal as the indirect employer of the contractors
employees. This liability facilitates, if not guarantees, payment of the workers compensation, thus, giving the
workers ample protection as mandated by the 1987 Constitution. This is not unduly burdensome to the
employer. Should the indirect employer be constrained to pay the workers, it can recover whatever amount it
had paid in accordance with the terms of the service contract between itself and the contractor.

Withal, fairness dictates that RPI should not, however, be held liable for wage differentials incurred while the
complainants were assigned to other companies. The indirect employers liability to the contractors employees
extends only to the period during which they were working for the petitioner, and the fact that they were
reassigned to another principal necessarily ends such responsibility. The principal is made liable to his indirect
employees, because it can protect itself from irresponsible contractors by withholding such sums and paying
them directly to the employees or by requiring a bond from the contractor or subcontractor for this purpose.

Similarly, the solidary liability for payment of back wages and separation pay is limited, under Article 106, to the
extent of the work performed under the contract; under Article 107, to the performance of any work, task, job or
project; and under Article 109, to the extent of their civil liability under this Chapter [on payment of wages].

The liability arising from an illegal dismissal is unlike an order to pay the statutory minimum wage, because the
workers right to such wage is derived from law. The proposition that payment of back wages and separation pay
should be covered by Article 109, which holds an indirect employer solidarily responsible with his contractor or
subcontractor for any violation of any provision of this Code, would have been tenable if there were proof --
there was none in this case -- that the principal/employer had conspired with the contractor in the acts giving
rise to the illegal dismissal.

The petition was partially granted.

San Beda College of Law 287


4S: 2015 - 2016
LABOR LAW REVIEW Atty. Joyrich Golangco

San Beda College of Law 288


4S: 2015 - 2016
LABOR LAW REVIEW Atty. Joyrich Golangco

143. FLORENCIO M. DE LA CRUZ, JR. v. NATIONAL LABOR RELATIONS COMMISSION (4th Division)
SHEMBERG MARKETING CORPORATION and ERNESTO U. DACAY, JR.
G.R. No. 145417 December 11, 2003
CORONA, J.:

ART. 223 JURISDICTION OF THE LABOR ARBITERS AND THE COMMISSION

DOCTRINE: A probationary employee is one who, for a given period of time, is under observation and
evaluation to determine whether or not he is qualified for permanent employment. During the probationary
period, the employer is given the opportunity to observe the skill, competence and attitude of the employee
while the latter seeks to prove to the employer that he has the qualifications to meet the reasonable standards
for permanent employment. The length of time is immaterial in determining the correlative rights of both the
employer and the employee in dealing with each other during said period.

FACTS:
On May 27, 1996, petitioner Florencio M. de la Cruz, Jr. was hired by private respondent Shemberg Marketing
Corporation (Shemberg) as senior sales manager with a monthly salary of P40,500. Shemberg was engaged in
the business of manufacturing, trading, distributing and importing various consumer products. The position of
senior sales manager was then newly created; its duties included, among others, the supervision and control of
the sales force of the company. The senior sales manager was also vested with some discretion to decide on
matters within the scope of his functions, including the appointment of district sales representatives and the
reshuffling of salesmen to achieve sales targets.

On September 14, 1996, Shembergs human resource department manager, Ms. Lilybeth Y. Llanto, summoned
petitioner and informed him of the managements decision to terminate his services. He was merely informed
that it had something to do with the drop in the companys sales. His request to be furnished a 30-day written
notice was also denied by the management. Hence, petitioner filed a complaint for illegal dismissal, non-
payment of salary, backwages, 13th month pay and damages.

Respondents answered that petitioners dismissal for his failure to meet the required company standards and for
loss of trust and confidence: (1) his poor performance as evidenced by the steady and substantial drop in
company sales since his assumption as senior sales manager; (2) the dissatisfaction of his subordinates over
his management style and dealings with the companys distributors which resulted in the low morale of
Shembergs sales force; (3) his unauthorized use of company cellular phone for overseas personal calls and (4)
the unauthorized reimbursement of the plane tickets of his wife and child.

LA RULING: Found that petitioner Florencio de la Cruz was illegally dismissed and granted his claim for
separation pay, backwages and unpaid wages.

NLRC RULING: Dismissed the appeal. Respondents moved for reconsideration and the NLRC partially granted
the MR, abandoning its previous decision. A new decision was rendered ordering respondent Shemberg to pay
unpaid wages of P18,900.00 and indemnity of P5,000.00.

CA RULING: De La Cruzs petition for certiorari was dismissed for lack of merit. His subsequent MR was
likewise denied.

ISSUES:
(1) Did the submission of the familys plane tickets for reimbursement tantamount to fraud and deceit which
justified the employers loss of trust and confidence in him?
(2) Is the petitioner a probationary employee?

SC DECISION:
(1) Petitioner was holding a managerial position which required the full trust and confidence of his employer.
While petitioner could exercise some discretion, this obviously did not cover acts for his own personal benefit.
The petitioners denial cannot prevail over the actual presentation of the plane ticket in the name of petitioner
and his family and terminal fee stubs bearing three (3) different serial numbers but similarly dated. The
possession by respondent corporation of the plane tickets of petitioners wife and child clearly shows that the
same were submitted to management for reimbursement along with the other transportation expenses of
petitioner. Otherwise, there is no way respondent corporation could have gotten hold of the same. Petitioner
San Beda College of Law 289
4S: 2015 - 2016
LABOR LAW REVIEW Atty. Joyrich Golangco

opted not to explain why these plane tickets were in the possession of respondent corporation. His denials
without accompanying proof coupled with his silence on this matter cannot but be taken against him.

(2) YES. Article 281 of the Labor Code provides: Probationary employment Probationary employment shall not
exceed six (6) months from the date the employee started working, unless it is covered by an apprenticeship
agreement stipulating a longer period. The services of an employee who has been engaged on a probationary
basis may be terminated for a just cause or when he fails to qualify as a regular employee in accordance with
reasonable standards, made known by the employer to the employee at the time of his engagement. An
employee who is allowed to work after a probationary period shall be considered a regular employee.

This Court notes the evidence on record clearly showing that petitioner was well informed of the standards to be
met before he could qualify as a regular employee. Attached to his appointment paper was the job description of
sales manager which read:

xxx
5. Performance subject to evaluation and trial period for six (6) months or more.

A probationary employee is one who, for a given period of time, is under observation and evaluation to
determine whether or not he is qualified for permanent employment. During the probationary period, the
employer is given the opportunity to observe the skill, competence and attitude of the employee while the latter
seeks to prove to the employer that he has the qualifications to meet the reasonable standards for permanent
employment. The length of time is immaterial in determining the correlative rights of both the employer and the
employee in dealing with each other during said period.

There is no dispute that petitioner, as a probationary employee, enjoyed only temporary employment status. In
general terms, this meant that he was terminable anytime, permanent employment not having been attained in
the meantime. The employer could well decide he no longer needed the probationary employees services or his
performance fell short of expectations, etc. As long as the termination was made before the expiration of the six-
month probationary period, the employer was well within his rights to sever the employer-employee relationship.
A contrary interpretation would defect the clear meaning of the term "probationary." In this case, respondent
Shemberg had good reason to terminate petitioners employment and that was his dishonesty.

San Beda College of Law 290


4S: 2015 - 2016
LABOR LAW REVIEW Atty. Joyrich Golangco

144. LYDIA BUENAOBRA, et al. v. LIM KING GUAN, et al. as corporate officers of UNIX INTERNATIONAL
EXPORT CORPORATION, and CHEN HSIU TSUNG, et al. as stockholders of record of UNIX
INTERNATIONAL EXPORT CORPORATION, and FUJI ZIPPER MANUFACTURING CORPORATION
G.R. No. 150147 January 20, 2004
CORONA, J.:

ART. 223 JURISDICTION OF THE LABOR ARBITERS AND THE COMMISSION

DOCTRINE:
It is true that the perfection of an appeal in the manner and within the period prescribed by law is not only
mandatory but jurisdictional, and failure to perfect an appeal has the effect of making the judgment final and
executory. However, technicality should not be allowed to stand in the way of equitably and completely resolving
the rights and obligations of the parties.

FACTS:
Petitioners were employees of UNIX, a corporation engaged in the business of manufacturing bags, wallets and
the like.

Sometime in 1991 and 1992, petitioners filed several cases against UNIX and its incorporators and officers for
unfair labor practice, illegal lockout/dismissal, underpayment of wages, holiday pay, proportionate 13th month
pay, unpaid wages, interest, moral and exemplary damages and attorneys fees.

LA RULING:
In favor of the petitioners ordering respondent Unix as follows:
1. P5,821,838.40 as backwages;
2. P1,484,912.00 as separation pay;
3. P527,748.00 as wage differentials;
4. P33,830.00 as regular holiday pay differentials; and
5. P365,551.95 as proportionate 13th month pay for 1990.

All other claims were dismissed for lack of merit. However, petitioners complained that the decision could not be
executed because UNIX allegedly diverted, invested and transferred all its money, assets and properties to FUJI
whose stockholders and officers were also those of UNIX.

Thus, petitioners filed another complaint against respondents UNIX, its corporate officers and stockholders of
record, and FUJI. Petitioners mainly prayed that respondents UNIX and FUJI be held jointly and severally held
liable for the payment of the monetary awards.

LA Pati rendered a decision on the second complaint piercing the veil of corporate fiction of the two respondent
sister corporations which were considered as mere associations of persons jointly and severally pay the subject
amount of P8,233,880.30 out of the properties and unpaid subscription on subscribed Capital Stock of the Board
of Directors, Corporate Officers, Incorporators and Stockholders of said respondent corporations, plus the
amount of P3,000,000.00 and P1,000,000.00 in the form of moral and exemplary damages, respectively, as well
as 10% attorneys fees from any recoverable amounts.

FUJI, its officers and stockholders filed a memorandum on appeal and a motion to dispense with the posting of
a cash or surety appeal bond on the ground that they were not the employers of petitioners.

NLRC RULING: Motion to exempt from filing appeal bond was DENIED for lack of merit. Respondents were
directed to post cash or surety bond.

Petitioners moved for reconsideration of the said order, arguing that the timely posting of an appeal bond is
mandatory for the perfection of an appeal and should be complied with. NLRC rendered an order dismissing the
MR.

Petitioners filed a petition in the Court of Appeals imputing grave abuse of discretion to the NLRC, Third Division
when it allowed private respondents to post the mandated cash or surety bond four months after the filing of
their memorandum on appeal.

San Beda College of Law 291


4S: 2015 - 2016
LABOR LAW REVIEW Atty. Joyrich Golangco

CA RULING: CA dismissed the petition for lack of merit.

ISSUE: Is the posting of bond a four months after the filing of the memorandum of appeal violative of the Labor
Code?

SC DECISION:
NO. The provision of Article 223 of the Labor Code requiring the posting of bond on appeals involving monetary
awards must be given liberal interpretation in line with the desired objective of resolving controversies on the
3
merits. If only to achieve substantial justice, strict observance of the reglementary periods may be relaxed if
warranted. The NLRC, Third Division could not be said to have abused its discretion in requiring the posting of
bond after it denied private respondents motion to be exempted therefrom.

It is true that the perfection of an appeal in the manner and within the period prescribed by law is not only
mandatory but jurisdictional, and failure to perfect an appeal has the effect of making the judgment final and
executory. However, technicality should not be allowed to stand in the way of equitably and completely resolving
4
the rights and obligations of the parties. We have allowed appeals from the decisions of the labor arbiter to the
NLRC, even if filed beyond the reglementary period, in the interest of justice.

It is only fair and just that respondent FUJI be afforded the opportunity to be heard on appeal before the NLRC,
specially in the light of labor arbiter Patis later decision holding FUJI jointly and severally liable with UNIX in the
payment of the monetary awards adjudged by labor arbiter de Vera against UNIX.

San Beda College of Law 292


4S: 2015 - 2016
LABOR LAW REVIEW Atty. Joyrich Golangco

145. LEPANTO CONSOLIDATED MINING CORPORATION v. BELIO ICAO
G.R. No. 196047 January 15, 2014
SERENO, CJ:

ART. 223 JURISDICTION OF THE LABOR ARBITERS AND THE COMMISSION

DOCTRINE:
The intention of the lawmakers to make the bond an indispensable requisite for the perfection of an appeal by
the employer, is clearly limned in the provision that an appeal by the employer may be perfected "only upon the
posting of a cash or surety bond." The word "only" makes it perfectly clear, that the lawmakers intended the
posting of a cash or surety bond by the employer to be the exclusive means by which an employer's appeal may
be perfected.

FACTS:
A complaint for illegal dismissal and damages was filed by private respondent Icao against petitioners Lepanto
Consolidated Mining Company (LCMC) and its CEO Felipe U. Yap before the Arbitration Branch of the NLRC.

Icao essentially alleged in his complaint that he was an employee of petitioner LCMC assigned as a lead miner
in its underground mine in Paco, Mankayan, Benguet. On January 4, 2008, private respondent reported for
work. While waiting for the time to ignite their round, one of his co-workers shouted to prepare the explosives for
blasting, prompting private respondent to run to the adjacent panels and warn the other miners. Thereafter, he
decided to take a bath and proceeded to the bathing station where 4 of his co-workers were also present.
Before he could join them, he heard a voice at his back and saw Security Guard (SG) Larry Bulwayan
instructing his companion SG Dale Papsa-ao to frisk him. As private respondent was removing his boots, SG
Bulwayan forcibly pulled his skullguard from his head causing it to fall to the ground including its harness and his
detergent soap which was inserted in the skullguard harness. A few minutes later, private respondent saw SG
Bulwayan pick up a wrapped object at the bathing station and gave it to his companion. SGs Bulwayan and
Papsa-ao invited the private respondent to go with them at the investigation office to answer questions
regarding the wrapped object. He was then charged with "highgrading" or the act of concealing, possessing or
unauthorized extraction of highgrade material/ore without proper authority. Private respondent vehemently
denied the charge. Consequently, he was dismissed from his work.

LA RULING: That petitioner and its CEO are liable for illegal dismissal and ordered them to pay respondent
Icao P345,879.45, representing his full backwages and separation pay. The alleged highgrading attributed by
LCMCs security guards was found to have been fabricated; consequently, there was no just cause for the
dismissal of respondent.

On 8 December 2008, petitioner and its CEO filed an Appearance with Memorandum of Appeal before the
NLRC. Instead of posting the required appeal bond in the form of a cash bond or a surety bond in an amount
equivalent to the monetary award adjudged in favor of Icao, they filed a Consolidated Motion For Release Of
Cash Bond And To Apply Bond Subject For Release As Payment For Appeal Bond (Consolidated Motion).

NLRC RULING: NLRC dismissed the appeal of petitioner and the latters CEO for non-perfection. It found that
they had failed to post the required appeal bond. equivalent to the monetary award of P345,879.45. It explained
that their Consolidated Motion for the release of the cash bond in another case (Dangiw Siggaao), for the
purpose of applying the same bond to the appealed case before it, could not be considered as compliance with
the requirement to post the required appeal bond. Consequently, it declared the labor arbiters Decision to be
final and executory.

Petitioner and its CEO filed a Motion for Reconsideration. They emphasized therein that they had tried to
comply in good faith with the requisite appeal bond by trying to produce a cash bond anew and also to procure a
new surety bond. However, after canvassing several bonding companies, the costs have proved to be
prohibitive. Hence, they resorted to using the cash bond they posted in Dangiw Siggaao because the bond was
now free, unencumbered and could rightfully be withdrawn and used by them. Their motion was denied. Hence,
they filed a Petition for Certiorari with the CA.

CA RULING: CA affirmed the Order of the NLRC, which had dismissed the appeal of petitioner and the latters
CEO for failure to comply with the requirements of law and consequently lost the right to appeal. CA said that
since the payment of appeal fees and the posting of an appeal bond are indispensable jurisdictional
San Beda College of Law 293
4S: 2015 - 2016
LABOR LAW REVIEW Atty. Joyrich Golangco

requirements, noncompliance with them resulted in petitioners failure to perfect its appeal. Consequently, the
labor arbiters Decision became final and executory and, hence, binding upon the appellate court.

ISSUE: Did the petitioners Consolidated Motion to release the cash bond it posted in a previous case, for
application to the present case, constitute compliance with the appeal bond requirement under the Labor Code?

SC RULING:
YES. The Court finds that petitioner substantially complied with the appeal bond requirement. In appeals from
any decision or order of the labor arbiter, the posting of an appeal bond is required under Article 223 of the
Labor Code, which reads:
Article 223. APPEAL. Decisions, awards, or orders of the Labor Arbiter are final and executory unless
appealed to the Commission by any or both parties within ten (10) calendar days from receipt of such decisions,
awards, or orders. Such appeal may be entertained only on any of the following grounds:
xxxx
In case of a judgment involving a monetary award, an appeal by the employer may be perfected only upon
the posting of a cash or surety bond issued by a reputable bonding company duly accredited by the
Commission in the amount equivalent to the monetary award in the judgment appealed from.

The intention of the lawmakers to make the bond an indispensable requisite for the perfection of an appeal by
the employer, is clearly limned in the provision that an appeal by the employer may be perfected "only upon the
posting of a cash or surety bond." The word "only" makes it perfectly clear, that the lawmakers intended the
posting of a cash or surety bond by the employer to be the exclusive means by which an employer's appeal may
be perfected.

First, there is no question that the appeal was filed within the 10-day reglementary period. Except for the alleged
failure to post an appeal bond, the appeal to the NLRC was therefore in order.

Second, it is also undisputed that petitioner has an unencumbered amount of money in the form of cash in the
custody of the NLRC. To reiterate, petitioner had posted a cash bond of P401,610.84 in the separate case
Dangiw Siggaao, which was earlier decided in its favor. As claimed by petitioner and confirmed by the Judgment
Division of the Judicial Records Office of this Court, the Decision in Dangiw Siggaao had become final and
executory as of 28 April 2008, or more than seven months before petitioner had to file its appeal in the present
case. This fact is shown by the Entry of Judgment on file with the aforementioned office. Hence, the cash bond
in that case ought to have been released to petitioner then.

Third, the cash bond in the amount of P401,610.84 posted in Dangiw Siggaao is more than enough to cover the
appeal bond in the amount of P345,879.45 required in the present case.

Fourth, this ruling remains faithful to the spirit behind the appeal bond requirement which is to ensure that
workers will receive the money awarded in their favor when the employers appeal eventually fails. There was no
showing at all of any attempt on the part of petitioner to evade the posting of the appeal bond. On the contrary,
petitioners move showed a willingness to comply with the requirement. Hence, the welfare of Icao is adequately
protected.

The Court found exceptional circumstances that warranted an extraordinary exercise of its power to exempt a
party from the rules on appeal bond, there is all the more reason in the present case to find that petitioner
substantially complied with the requirement. Having complied with the appeal bond requirement, petitioner s
appeal before the NLRC must therefore be reinstated.

San Beda College of Law 294


4S: 2015 - 2016
LABOR LAW REVIEW Atty. Joyrich Golangco

146. FROILAN M. BERGONIO, et al. v. SOUTH EAST ASIAN AIRLINES and IRENE DORNIER
G.R. No. 195227 April 21, 2014
BRION, J.:

ART. 223 JURISDICTION OF THE LABOR ARBITERS AND THE COMMISSION

DOCTRINE:
Under paragraph 3, Article 223 of the Labor Code, the LAs order for the reinstatement of an employee found
illegally dismissed is immediately executory even during pendency of the employers appeal from the decision.
Under this provision, the employer must reinstate the employee either by physically admitting him under the
conditions prevailing prior to his dismissal, and paying his wages; or, at the employers option, merely reinstating
the employee in the payroll until the decision is reversed by the higher court. Failure of the employer to comply
with the reinstatement order, by exercising the options in the alternative, renders him liable to pay the
employees salaries.

FACTS:
Petitioners filed before the LA a complaint for illegal dismissal and illegal suspension with prayer for
reinstatement against respondents South East Asian Airlines (SEAIR) and Irene Dornier as SEAIRs President.

LA RULING: LA found the petitioners illegally dismissed and ordered the respondents, among others, to
immediately reinstate the petitioners with full backwages. The respondents received their copy of this decision
6
on July 8, 2005.

On August 20, 2005, the petitioners filed before the LA a Motion for issuance of Writ of Execution for their
immediate reinstatement.

During the scheduled pre-execution conference, the respondents manifested their option to reinstate the
petitioners in the payroll. The payroll reinstatement, however, did not materialize. Thus, on September 22, 2005,
the petitioners filed before the LA a manifestation for their immediate reinstatement. The respondents filed an
opposition claiming that the relationship between them and the petitioners had already been strained because of
the petitioners threatening text messages, thus precluding the latters reinstatement.

LA granted the petitioners motion and issued a writ of execution but was returned unsatisfied. In response, the
petitioners filed a motion for re-computation of accrued wages and a motion for execution of the re-computed
amount which was granted. The respondents appealed with the NLRC.

NLRC RULING: NLRC dismissed the respondents appeal for non-perfection. The NLRC likewise denied the
respondents MR, prompting the respondents to file before the CA a petition for certiorari.

CA RULING: CA rendered its decision (on the illegal dismissal ruling of the LA) partly granting the respondents
petition, declaring the dismissal valid and awarded the petitioners P30,000.00 as nominal damages for the
respondents failure to observe due process.

The CA agreed that the reinstatement aspect of the LAs decision is immediately executory even pending
appeal, such that the employer is obliged to reinstate and pay the wages of the dismissed employee during the
period of appeal until the decision (finding the employee illegally dismissed including the reinstatement order) is
reversed by a higher court. Applying this principle, the CA noted that the petitioners accrued wages could have
been properly computed until December 18, 2007, the date of the CAs decision finding the petitioners validly
dismissed.

Thus, the CA declared that, given this peculiar circumstance (of the petitioners failure to report for work), the
petitioners accrued wages should only be computed until February 24, 2006 when they were supposed to
report for work per the return-to-work Memorandum. Accordingly, the CA reversed, for grave abuse of
discretion, the NLRCs July 16, 2008 decision that affirmed the LAs order to release the garnished amount.

ISSUES:
(1) Is the LAs order for reinstatement of an illegally dismissed employee immediately executory even during
pendency of the employers appeal from the decision?

San Beda College of Law 295


4S: 2015 - 2016
LABOR LAW REVIEW Atty. Joyrich Golangco

(2) Should the accrued wages be computed until December 17, 2008, when the CA reversed the illegal
dismissal findings of the LA or only until February 24, 2006, when the petitioners were supposed to report
for work?

SC RULING:
(1) YES. Under paragraph 3, Article 223 of the Labor Code, the LAs order for the reinstatement of an employee
found illegally dismissed is immediately executory even during pendency of the employers appeal from the
decision. Under this provision, the employer must reinstate the employee either by physically admitting him
under the conditions prevailing prior to his dismissal, and paying his wages; or, at the employers option, merely
reinstating the employee in the payroll until the decision is reversed by the higher court. Failure of the employer
to comply with the reinstatement order, by exercising the options in the alternative, renders him liable to pay the
employees salaries.

Otherwise stated, a dismissed employee whose case was favorably decided by the LA is entitled to receive
wages pending appeal upon reinstatement, which reinstatement is immediately executory. Unless the appellate
tribunal issues a restraining order, the LA is duty bound to implement the order of reinstatement and the
employer has no option but to comply with it.

Moreover, and equally worth emphasizing, is that an order of reinstatement issued by the LA is self-executory,
i.e., the dismissed employee need not even apply for and the LA need not even issue a writ of execution to
trigger the employers duty to reinstate the dismissed employee.

After the LAs decision is reversed by a higher tribunal, the employers duty to reinstate the dismissed employee
is effectively terminated. This means that an employer is no longer obliged to keep the employee in the actual
service or in the payroll. The employee, in turn, is not required to return the wages that he had received prior to
the reversal of the LAs decision.

The reversal by a higher tribunal of the LAs finding (of illegal dismissal), notwithstanding, an employer, who,
despite the LAs order of reinstatement, did not reinstate the employee during the pendency of the appeal up to
the reversal by a higher tribunal may still be held liable for the accrued wages of the employee, i.e., the unpaid
32
salary accruing up to the time the higher tribunal reverses the decision. The rule, therefore, is that an
employee may still recover the accrued wages up to and despite the reversal by the higher tribunal. This
entitlement of the employee to the accrued wages proceeds from the immediate and self-executory nature of the
reinstatement aspect of the LAs decision.

By way of exception to the above rule, an employee may be barred from collecting the accrued wages if shown
that the delay in enforcing the reinstatement pending appeal was without fault on the part of the employer.

(2) To determine whether an employee is thus barred, two tests must be satisfied: (1) actual delay or the fact
that the order of reinstatement pending appeal was not executed prior to its reversal; and (2) the delay must not
be due to the employers unjustified act or omission. Note that under the second test, the delay must be without
the employers fault. If the delay is due to the employers unjustified refusal, the employer may still be required
to pay the salaries notwithstanding the reversal of the LAs decision.

First, the existence of delay - whether there was actual delay or whether the order of reinstatement pending
appeal was not executed prior to its reversal? We answer this test in the affirmative.

To recall, on May 31, 2005, the LA rendered the decision finding the petitioners illegally dismissed and ordering
their immediate reinstatement. Per the records, the respondents received copy of this decision on July 8, 2005.
On August 20, 2005, the petitioners filed before the LA a Motion for Issuance of Writ of Execution for their
immediate reinstatement. The LA issued the Writ of Execution on October 7, 2005. From the time the
respondents received copy of the LAs decision, and the issuance of the writ of execution, until the CA reversed
this decision on December 17, 2008, the respondents had not reinstated the petitioners, either by actual
reinstatement or in the payroll. This continued non-execution of the reinstatement order in fact moved the LA to
issue an alias writ of execution on February 16, 2006 and another writ of execution on April 24, 2007.

From these facts and without doubt, there was actual delay in the execution of the reinstatement aspect of the
LAs May 31, 2005 decision before it was reversed in the CAs decision.

San Beda College of Law 296


4S: 2015 - 2016
LABOR LAW REVIEW Atty. Joyrich Golangco

Second, the cause of the delay whether the delay was not due to the employers unjustified act or omission.
We answer this test in the negative; we find that the delay in the execution of the reinstatement pending appeal
was due to the respondents unjustified acts. For one, the respondents filed several pleadings to suspend the
execution of the LAs reinstatement order. These pleadings, to our mind, show a determined effort on the
respondents part to prevent or suspend the execution of the reinstatement pending appeal.

The respondents did not sufficiently notify the petitioners of their intent to actually reinstate them; neither did the
respondents give them ample opportunity to comply with the return-to-work directive.

Lastly, the petitioners continuously and actively pursued the execution of the reinstatement aspect of the LAs
decision, i.e., by filing several motions for execution of the reinstatement order, and motion to cite the
respondents in contempt and re-computation of the accrued wages for the respondents continued failure to
reinstate them.

These facts altogether show that the respondents were not at all sincere in reinstating the petitioners. These
facts when taken together with the fact of delay reveal the respondents obstinate resolve and willful
disregard of the immediate and self-executory nature of the reinstatement aspect of the LAs decision.

San Beda College of Law 297


4S: 2015 - 2016
LABOR LAW REVIEW Atty. Joyrich Golangco

147. WILGEN LOON, et al. v. POWER MASTER, INC., TRI-C GENERAL SERVICES, and SPOUSES HOMER
and CARINA ALUMISIN
G.R. No. 189404 December 11, 2013
BRION, J.:

ART. 223 JURISDICTION OF THE LABOR ARBITERS AND THE COMMISSION

DOCTRINE:
The issue of the appeal bonds validity may be raised for the first time on appeal since its proper filing is a
jurisdictional requirement. The requirement that the appeal bond should be issued by an accredited bonding
company is mandatory and jurisdictional. The rationale of requiring an appeal bond is to discourage the
employers from using an appeal to delay or evade the employees' just and lawful claims. It is intended to assure
the workers that they will receive the money judgment in their favor upon the dismissal of the employers appeal.

FACTS:
Respondents Power Master, Inc. and Tri-C General Services employed and assigned the petitioners as janitors
and leadsmen in various PLDT offices in Metro Manila area. Subsequently, the petitioners filed a complaint for
money claims against the respondents alleging that they were not paid minimum wages, overtime, holiday,
premium, service incentive leave, and thirteenth month pays. They further averred that the respondents made
them sign blank payroll sheets. The petitioners amended their complaint and included illegal dismissal as their
cause of action.

Notably, the respondents did not participate in the proceedings before the Labor Arbiter except on April 19,
5
2001 and May 21, 2001 when Mr. Romulo Pacia, Jr. appeared on the respondents behalf. The
6
respondents counsel also appeared in a preliminary mandatory conference on July 5, 2001. However,
the respondents neither filed any position paper nor proffered pieces of evidence in their defense despite their
knowledge of the pendency of the case.

LA RULING: LA Elias H. Salinas partially ruled in favor of the petitioners. The LA awarded the petitioners salary
differential, service incentive leave, and thirteenth month pays. However, the LA denied the petitioners
claims for backwages, overtime, holiday, and premium pays. The LA observed that the petitioners failed to
show that they rendered overtime work and worked on holidays and rest days without compensation. The LA
further concluded that the petitioners cannot be declared to have been dismissed from employment because
they did not show any notice of termination of employment. They were also not barred from entering the
respondents premises.

Both parties appealed to the NLRC. The respondents insisted that they were not personally served with
summons and other processes. They also claimed that they paid the petitioners minimum wages, service
incentive leave and thirteenth month pays. As proof, they attached photocopied and computerized copies of
payroll sheets to their memorandum on appeal.

On January 3, 2003, the respondents filed an unverified supplemental appeal. They attached photocopied and
computerized copies of list of employees with ATM cards to the supplemental appeal. This list also showed the
11
amounts allegedly deposited in the employees ATM cards. They also attached documentary evidence
showing that the petitioners were dismissed for cause and had been accorded due process.
16
NLRC RULING: NLRC partially ruled in favor of the respondents. The NLRC affirmed the LAs awards of
holiday pay and attorneys fees. It also maintained that the LA acquired jurisdiction over the persons of the
respondents through their voluntary appearance. However, it allowed the respondents to submit pieces of
evidence for the first time on appeal on the ground that they had been deprived of due process. NLRC also
vacated the LAs awards of salary differential, thirteenth month and service incentive leave pays.

The NLRC further ruled that the petitioners were lawfully dismissed on grounds of serious misconduct and willful
disobedience. It found that the petitioners failed to comply with various memoranda directing them to transfer to
other workplaces and to attend training seminars for the intended reorganization and reshuffling.

CA RULING: The CA affirmed the NLRCs ruling. The CA held that the petitioners were afforded substantive
and procedural due process. It also upheld the NLRCs findings on the petitioners monetary claims. The CA
denied the petitioners MR.
San Beda College of Law 298
4S: 2015 - 2016
LABOR LAW REVIEW Atty. Joyrich Golangco

ISSUES:
(1) Did the respondents perfect their appeal before the NLRC?
(2) Were the petitioners illegally dismissed and are thus entitled to backwages, salary differential, holiday,
service incentive leave, and thirteenth month pays?
(3) Are the petitioners entitled to overtime and premium pay?

SC RULING:
(1) YES. Paragraph 2, Article 223 of the Labor Code provides that "[i]n case of a judgment involving a monetary
award, an appeal by the employer may be perfected only upon the posting of a cash or surety bond issued by a
reputable bonding company duly accredited by the Commission in the amount equivalent to the monetary award
in the judgment appealed from."

The issue of the appeal bonds validity may be raised for the first time on appeal since its proper filing is a
jurisdictional requirement. The requirement that the appeal bond should be issued by an accredited bonding
company is mandatory and jurisdictional. The rationale of requiring an appeal bond is to discourage the
employers from using an appeal to delay or evade the employees' just and lawful claims. It is intended to assure
the workers that they will receive the money judgment in their favor upon the dismissal of the employers appeal.

In the present case, the respondents filed a surety bond issued by Security Pacific. At that time, Security Pacific
24
was still an accredited bonding company. However, the NLRC revoked its accreditation on February 16, 2003.
Nonetheless, this subsequent revocation should not prejudice the respondents who relied on its then subsisting
accreditation in good faith.

The CA also correctly ruled that the NLRC properly gave due course to the respondents supplemental appeal.
Neither the laws nor the rules require the verification of the supplemental appeal. Furthermore, verification is a
formal, not a jurisdictional, requirement. It is mainly intended for the assurance that the matters alleged in the
pleading are true and correct and not of mere speculation. Also, a supplemental appeal is merely an addendum
to the verified memorandum on appeal that was earlier filed in the present case; hence, the requirement for
verification has substantially been complied with.

In labor cases, strict adherence to the technical rules of procedure is not required. Time and again, we have
allowed evidence to be submitted for the first time on appeal with the NLRC in the interest of substantial justice.
Thus, we have consistently supported the rule that labor officials should use all reasonable means to ascertain
the facts in each case speedily and objectively, without regard to technicalities of law or procedure, in the
interest of due process.

The respondents failed to sufficiently prove the allegations sought to be proven. Why the respondents
photocopied and computerized copies of documentary evidence were not presented at the earliest opportunity is
a serious question that lends credence to the petitioners claim that the respondents fabricated the evidence for
purposes of appeal. While we generally admit in evidence and give probative value to photocopied
documents in administrative proceedings, allegations of forgery and fabrication should prompt the
adverse party to present the original documents for inspection.

It was also gross error for the CA to affirm the NLRCs proposition that "[i]t is of common knowledge that there
are many people who use at least two or more different signatures." The NLRC cannot take judicial notice that
many people use at least two signatures, especially in this case where the petitioners themselves disown the
signatures in the respondents assailed documentary evidence. The NLRCs position is unwarranted and is
patently unsupported by the law and jurisprudence.

(2) YES. In termination cases, the burden of proving just and valid cause for dismissing an employee from his
employment rests upon the employer. The employers failure to discharge this burden results in the finding that
the dismissal is unjustified. This is exactly what happened in the present case.

As in illegal dismissal cases, the general rule is that the burden rests on the defendant to prove payment rather
than on the plaintiff to prove non-payment of these money claims. The rationale for this rule is that the pertinent
personnel files, payrolls, records, remittances and other similar documents which will show that differentials,

San Beda College of Law 299


4S: 2015 - 2016
LABOR LAW REVIEW Atty. Joyrich Golangco

service incentive leave and other claims of workers have been paid are not in the possession of the worker but
are in the custody and control of the employer.

(3) NO. CA was correct in its finding that the petitioners failed to provide sufficient factual basis for the award of
overtime, and premium pays for holidays and rest days. The burden of proving entitlement to overtime pay and
premium pay for holidays and rest days rests on the employee because these are not incurred in the normal
course of business. In the present case, the petitioners failed to adduce any evidence that would show that they
actually rendered service in excess of the regular eight working hours a day, and that they in fact worked on
holidays and rest days.

San Beda College of Law 300


4S: 2015 - 2016
LABOR LAW REVIEW Atty. Joyrich Golangco

148. ANDREW JAMES MCBURNIE v. EULALIO GANZON, EGI-MANAGERS, INC. and E. GANZON, INC.
G.R. Nos. 178034 & 178117 G R. Nos. 186984-85 October 17, 2013
REYES, J.:

APPEAL BOND

DOCTRINE:
On the matter of the filing and acceptance of motions to reduce appeal bond, as provided in Section 6, Rule VI
of the 2011 NLRC Rules of Procedure, the following guidelines shall be observed:
(a) The filing of a motion to reduce appeal bond shall be entertained by the NLRC subject to the following
conditions: (1) there is meritorious ground; and (2) a bond in a reasonable amount is posted;
(b) For purposes of compliance with condition no. (2), a motion shall be accompanied by the posting of a
provisional cash or surety bond equivalent to ten percent (10%) of the monetary award subject of the
appeal, exclusive of damages and attorney's fees;
(c) Compliance with the foregoing conditions shall suffice to suspend the running of the 10-day
reglementary period to perfect an appeal from the labor arbiter's decision to the NLRC;
(d) The NLRC retains its authority and duty to resolve the motion to reduce bond and determine the final
amount of bond that shall be posted by the appellant, still in accordance with the standards of
meritorious grounds and reasonable amount; and
(e) In the event that the NLRC denies the motion to reduce bond, or requires a bond that exceeds the
amount of the provisional bond, the appellant shall be given a fresh period of ten 10 days from notice of
the NLRC order within which to perfect the appeal by posting the required appeal bond.

FACTS:
McBurnie, an Australian national, instituted a complaint for illegal dismissal and other monetary claims against
the respondents. McBurnie claimed that he signed an employment agreement with the company EGI as an
Executive Vice-President. On the other hand, the respondents opposed the complaint, contending that their
agreement with McBurnie was to jointly invest in and establish a company for the management of hotels and did
not intend to create an employer-employee relationship.

LA RULING: McBurnie was illegally dismissed from employment.

The respondents appealed the LAs Decision to the NLRC. They filed their Memorandum of Appeal and Motion
to Reduce Bond, and posted an appeal bond in the amount of P100,000.00. The respondents contended in their
Motion to Reduce Bond, inter alia, that the monetary awards of the LA were null and excessive, allegedly with
the intention of rendering them incapable of posting the necessary appeal bond. They claimed that an award of
"more than P60 Million Pesos to a single foreigner who had no work permit and who left the country for good
one month after the purported commencement of his employment" was a patent nullity. Furthermore, they
claimed that because of their business losses that may be attributed to an economic crisis, they lacked the
capacity to pay the bond of almost P60 Million, or even the millions of pesos in premium required for such bond.

NLRC RULING: The NLRC denied the motion to reduce bond, explaining that "in cases involving monetary
award, an employer seeking to appeal the LAs decision to the Commission is unconditionally required by Art.
223, Labor Code to post bond in the amount equivalent to the monetary award. Thus, the NLRC required from
the respondents the posting of an additional bond in the amount of P54,083,910.00.

CA RULING: The CA allowed the respondents motion to reduce appeal bond to P10,000,000.00 and directing
the NLRC to give due course to their appeal. The CA explained that "while Art. 223 of the Labor Code requiring
bond equivalent to the monetary award is explicit, Section 6, Rule VI of the NLRC Rules of Procedure, as
amended, recognized as exception a motion to reduce bond upon meritorious grounds and upon posting of a
bond in a reasonable amount in relation to the monetary award."

ISSUE: Is the 10 million pesos bond substantial and special meritorious circumstance to merit reconsideration of
the appeal?

SC RULING:
YES. The 10 million pesos bond is substantial and a special meritorious circumstance to merit reconsideration
of the appeal.

San Beda College of Law 301


4S: 2015 - 2016
LABOR LAW REVIEW Atty. Joyrich Golangco

The present rule on the matter is Section 6, Rule VI of the 2011 NLRC Rules of Procedure, which was
substantially the same provision in effect at the time of the respondents appeal to the NLRC, and which reads:

No motion to reduce bond shall be entertained except on meritorious grounds and upon the
posting of a bond in a reasonable amount in relation to the monetary award.

Prevailing rules and jurisprudence allow the reduction of appeal bonds.

While the bond may be reduced upon motion by the employer, this is subject to the conditions that (1) the
motion to reduce the bond shall be based on meritorious grounds; and (2) a reasonable amount in relation to the
monetary award is posted by the appellant, otherwise the filing of the motion to reduce bond shall not stop the
running of the period to perfect an appeal. The qualification effectively requires that unless the NLRC grants the
reduction of the cash bond within the 10 day reglementary period, the employer is still expected to post the cash
or surety bond securing the full amount within the said 10-day period. If the NLRC does eventually grant the
motion for reduction after the reglementary period has elapsed, the correct relief would be to reduce the cash or
surety bond already posted by the employer within the 10-day period.

San Beda College of Law 302


4S: 2015 - 2016
LABOR LAW REVIEW Atty. Joyrich Golangco

149. WATERFRONT CEBU CITY CASINO HOTEL, INC. AND MARCO PROTACIO v. ILDEBRANDO
LEDESMA
G.R. No. 197556 March 25, 2015
VILLARAMA, JR., J.:

APPEAL

DOCTRINE:
As to the 60-period reglementary period of filing an appeal, the relaxation of procedural rules may be allowed
only when there are exceptional circumstances to justify the same.

FACTS:
Respondent was employed as a House Detective at Waterfront. On the basis of the complaints filed before
Waterfront by Christe Mandal, a supplier of a concessionaire of Waterfront, and Rosanna Lofranco, who was
seeking a job at the same hotel, Ledesma was dismissed from employment. From the affidavits and testimonies
of Christe Mandal and Rosanna Lofranco during the administrative hearings conducted by Waterfront, the latter
found, among others, that Ledesma kissed and mashed the breasts of Christe Mandal inside the hotels
elevator, and exhibited his penis and asked Rosanna Lofranco to masturbate him at the conference room of the
hotel. Ledesma filed a complaint for illegal dismissal.

LA RULING: The LA found that the allegations leveled against Ledesma are mere concoctions, and concluded
that Ledesma was illegally dismissed, and ordered the petitioner among others to reinstate Ledesma.

NLRC RULING: The NLRC reversed the decision of the LA. The NLRC denied Ledesmas motion for
reconsideration in a Resolution dated February 22, 2010. A copy of the said Resolution was received by Atty.
Gines Abellana, Ledesmas counsel of record, on March 15, 2010. On May 17, 2010, or sixty-three (63) days
after Atty. Abellana received a copy of the NLRCs Resolution denying the motion for reconsideration, said
counsel filed before the CA a petition for certiorari under Rule 65 of the Rules of Court. In its Comment,
Waterfront prayed for the outright dismissal of the petition on the ground that it was belatedly filed.

CA RULING: The CA entertained the petition and reinstated the decision of the LA.

ISSUE: Is the unjustified failure of Ledesma to file his petition for certiorari before the CA within the 60-day
period a ground for the outright dismissal of said petition?

SC RULING:
YES. The unjustified failure of Ledesma to file his petition for certiorari before the CA within the 60-day period a
ground for the outright dismissal of said petition.

A reading of the rulings leads to the simple conclusion that the case of Laguna Metts Corporation involves a
strict application of the general rule that petitions for certiorari must be filed strictly within sixty (60) days from
notice of judgment or from the order denying a motion for reconsideration. Domdom case, on the other hand,
relaxed the rule and allowed an extension of the sixty (60)-day period subject to the Courts sound discretion. In
relaxing the rules and allowing an extension, Thenamaris Philippines, Inc. v. Court of Appeals reiterated the
necessity for the party invoking liberality to advance a reasonable or meritorious explanation for the failure to file
the petition for certiorari within the 60-day period.

The relaxation of procedural rules may be allowed only when there are exceptional circumstances to justify the
same. There should be an effort on the part of the party invoking liberality to advance a reasonable or
meritorious explanation for his/her failure to comply with the rules. Moreover, those who seek exemption from
the application of a procedural rule have the burden of proving the existence of exceptionally meritorious reason
warranting such departure.

Both in his petition and amended petition, Ledesma never invoked the liberality of the CA nor endeavored to
justify the belated filing of his petition. On the contrary, Ledesma remained firm that his petition was filed with
the CA within the reglementary period. Absent valid and compelling reasons for the procedural lapse, the
desired leniency cannot be accorded to Ledesma.

San Beda College of Law 303


4S: 2015 - 2016
LABOR LAW REVIEW Atty. Joyrich Golangco

In sum, the late filing by Ledesma of his petition for certiorari, and his failure to justify his procedural lapse to
merit a lenient application of the rules divested the CA of jurisdiction to entertain the petition.

San Beda College of Law 304


4S: 2015 - 2016
LABOR LAW REVIEW Atty. Joyrich Golangco

Reinstatement Aspect of LAs Decision

150. PIONEER TEXTURIZING CORP. and/or JULIANO LIM v. NATIONAL LABOR RELATIONS
COMMISSION, PIONEER TEXTURIZING WORKERS UNION and LOURDES A. DE JESUS
G.R. No. 118651 October 16, 1997
FRANCISCO, J.:

REINSTATEMENT ASPECT OF LAs DECISION

DOCTRINE:
An award or order for reinstatement is self-executory, and does not require a writ of execution, much less a
motion for its issuance.

FACTS:
Private respondent Lourdes A. de Jesus is petitioners reviser/trimmer since 1980. As reviser/trimmer, de Jesus
based her assigned work on a paper note posted by petitioners. The petitioners terminated her employment for
dishonesty and tampering of official records and documents with the intention of cheating. De Jesus maintained
that she merely committed a mistake.

LA RULING: Petitioners are guilty of illegal dismissal. Petitioners were accordingly ordered to reinstate de
Jesus to her previous position without loss of seniority rights and with full backwages from the time of her
suspension.

NLRC RULING: The NLRC declared that the status quo between them should be maintained and affirmed the
Labor Arbiters order of reinstatement, but without backwages. The NLRC further directed petitioner to pay de
Jesus her back salaries from the date she filed her motion for execution on September 21, 1993 up to the date
of the promulgation of the decision.

Petitioners Argument: An order for reinstatement is not self-executory. They maintain that even if a writ of
execution was issued, a timely appeal coupled by the posting of appropriate supersedeas bond, which they did
in this case, effectively forestalled and stayed execution of the reinstatement order of the Labor Arbiter.

ISSUE: Is a writ of execution required in an order for reinstatement?

SC RULING:
NO. A writ of execution is not required in an order for reinstatement.

ART. 223. Appeal. --Decisions, awards, or orders of the Labor Arbiter are final and executory
unless appealed to the Commission by any or both parties within ten (10) calendar days from
receipt of such decisions, awards, or orders.
xxx xxx xxx
In an event, the decision of the Labor Arbiter reinstating a dismissed or separated employee,
insofar as the reinstatement aspect is concerned, shall immediately be executory, even pending
appeal. The employee shall either be admitted back to work under the same terms and conditions
prevailing prior to his dismissal or separation or, at the option of the employer, merely reinstated
in the payroll. The posting of a bond by the employer shall not stay the execution for
reinstatement provided herein.
xxx xxx xxx

Under the said provision of law, the decision of the Labor Arbiter reinstating a dismissed or separated employee
insofar as the reinstatement aspect is concerned, shall be immediately executory, even pending appeal. The
employer shall reinstate the employee concerned either by: (a) actually admitting him back to work under the
same terms and conditions prevailing prior to his dismissal or separation; or (b) at the option of the employer,
merely reinstating him in the payroll. Immediate reinstatement is mandated and is not stayed by the fact that the
employer has appealed, or has posted a cash or surety bond pending appeal.

San Beda College of Law 305


4S: 2015 - 2016
LABOR LAW REVIEW Atty. Joyrich Golangco

151. ALEJANDRO ROQUERO vs. PHILIPPINE AIRLINES, INC.
G.R. No. 152329 April 22, 2003
PUNO, J.:

REINSTATEMENT ASPECT OF LAs DECISION

DOCTRINE:
The reinstatement aspect of a labor tribunals order is immediately executory unless there is a restraining order
or preliminary injunction.

FACTS:
Roquero, along with Rene Pabayo, were ground equipment mechanics of respondent PAL. From the evidence
on record, it appears that Roquero and Pabayo were caught red-handed possessing and using
Methampethamine Hydrochloride or shabu in a raid conducted by PAL security officers and NARCOM
personnel. Roquero and Pabayo were dismissed by PAL. Thus, they filed a case of illegal dismissal.

LA RULING: The dismissal of Roquero and Pabayo was upheld. The LA found both parties at fault PAL for
applying means to entice the complainants into committing the infraction and the complainants for giving in to
the temptation and eventually indulging in the prohibited activity. Nonetheless, separation pay and attorneys
fees are awarded.

NLRC RULING: It ruled in favor of complainants as it likewise found PAL guilty of instigation. It ordered
reinstatement to their former positions but without backwages.

Complainants did not appeal from the decision but filed a motion for a writ of execution of the order of
reinstatement. The LA granted the motion but PAL refused to execute the said order on the ground that they
have filed a Petition for Review before the SC which was reffered to CA.

CA RULING: It reversed the decision of the NLRC and reinstated the decision of the LA insofar as it upheld the
dismissal of Roquero. However, it denied the award of separation pay and attorneys fees to Roquero on the
ground that one who has been validly dismissed is not entitled to those benefits.

ISSUE: Can the executory nature of the decision, more so the reinstatement aspect of a labor tribunals order be
halted by a petition having been filed in higher courts without any restraining order or preliminary injunction
having been ordered in the meantime?

SC RULING:
NO. The executory nature of the decision, more so the reinstatement aspect of a labor tribunals order cannot be
halted by a petition having been filed in higher courts without any restraining order or preliminary injunction
having been ordered in the meantime.

The order of reinstatement is immediately executory. The unjustified refusal of the employer to reinstate a
dismissed employee entitles him to payment of his salaries effective from the time the employer failed to
reinstate him despite the issuance of a writ of execution. Unless there is a restraining order issued, it is
ministerial upon the Labor Arbiter to implement the order of reinstatement. In the case at bar, no restraining
order was granted. Thus, it was mandatory on PAL to actually reinstate Roquero or reinstate him in the payroll.
Having failed to do so, PAL must pay Roquero the salary he is entitled to, as if he was reinstated, from the time
of the decision of the NLRC until the finality of the decision of this Court.

We reiterate the rule that technicalities have no room in labor cases where the Rules of Court are applied only in
a suppletory manner and only to effectuate the objectives of the Labor Code and not to defeat them. Hence,
even if the order of reinstatement of the Labor Arbiter is reversed on appeal, it is obligatory on the part of the
employer to reinstate and pay the wages of the dismissed employee during the period of appeal until reversal by
the higher court. On the other hand, if the employee has been reinstated during the appeal period and such
reinstatement order is reversed with finality, the employee is not required to reimburse whatever salary he
received for he is entitled to such, more so if he actually rendered services during the period.

San Beda College of Law 306


4S: 2015 - 2016
LABOR LAW REVIEW Atty. Joyrich Golangco

152. AIR PHILIPPINES CORPORATION vs. ENRICO E. ZAMORA
G.R. NO. 148247 August 7, 2006
AUSTRIA-MARTINEZ, J.:

REINSTATEMENT ASPECT OF LAs DECISION

DOCTRINE:
Even if the order of reinstatement of the Labor Arbiter is reversed on appeal, it is obligatory on the part of the
employer to reinstate and pay the wages of the dismissed employee during the period of appeal until reversal by
the higher court.

FACTS:
Enrico Zamora was employed with Air Philippines Corporation (APC) as a Flight Deck Crew. He applied for
promotion to the position of airplane captain and underwent the requisite training program. After completing
training, he inquired about his promotion but APC did not act on it; instead, it continued to give him assignments
as flight deck crew. Thus, Zamora filed a Complaint with the LA. He argued that the act of APC of withholding
his promotion rendered his continued employment with it oppressive and unjust. He therefore asked that APC
be held liable for constructive dismissal.

LA RULING: Respondent was liable for illegal dismissal and ordered the respondent, among others, to reinstate
complainant to his position as Captain without loss of seniority right immediately upon receipt the decision.

NLRC RULING: It held that no dismissal, constructive or otherwise, took place for it was Zamora himself who
voluntarily terminated his employment by not reporting for work and by joining a competitor Grand Air. However,
it ordered APC to pay salaries and allowances to complainant arose from the order of his reinstatement which is
executory even pending appeal of respondent questioning the same, pursuant to Article 223 of the Labor Code.

CA RULING: It dismissed the petition of APC for failure of petitioner to attach copies of all pleadings (such
complaint, answer, position paper) and other material portions of the record as would support the allegations
therein.

ISSUE: Is the NLRC correct in ordering the APC to pay Zamora the salaries and allowances that arose from the
order of his reinstatement of the LA?

SC RULING:
YES. The NLRC is correct in ordering the APC to pay Zamora the salaries and allowances that arose from the
order of his reinstatement of the LA

The premise of the award of unpaid salary to respondent is that prior to the reversal by the NLRC of the decision
of the LA, the order of reinstatement embodied therein was already the subject of an alias writ of execution even
pending appeal. Although petitioner did not comply with this writ of execution, its intransigence made it liable
nonetheless to the salaries of respondent pending appeal. There is logic in this reasoning of the NLRC.

In Aris (Phil.) Inc. v. National Labor Relations Commission, we held: Then, by and pursuant to the same power
(police power), the State may authorize an immediate implementation, pending appeal, of a decision reinstating
a dismissed or separated employee since that saving act is designed to stop, although temporarily since the
appeal may be decided in favor of the appellant, a continuing threat or danger to the survival or even the life of
the dismissed or separated employee and his family.

San Beda College of Law 307


4S: 2015 - 2016
LABOR LAW REVIEW Atty. Joyrich Golangco

153. LUNESA O. LANSANGAN AND ROCITA CENDAA v. AMKOR TECHNOLOGY PHILIPPINES, INC.,
G.R. NO. 177026 January 30, 2009
CARPIO MORALES, J.:

DOCTRINE:
In cases of regular employment, the employer shall not terminate the services of an employee except for a just
cause or when authorized by this Title. An employee who is unjustly dismissed from work shall be entitled to
reinstatement without loss of seniority rights and other privileges and to his full backwages, inclusive of
allowances, and to his other benefits or their monetary equivalent computed from the time his compensation
was withheld from him up to the time of his actual reinstatement.

FACTS:
An anonymous e-mail was sent to the General Manager of Amkor Technology Philippines (respondent) detailing
allegations of malfeasance on the part of its supervisory employees Lunesa Lansangan and Rosita Cendaa
(petitioners) for "stealing company time. Respondent thus investigated the matter, requiring petitioners to
submit their written explanation. In handwritten letters, petitioners admitted their wrongdoing. Respondent
thereupon terminated petitioners for "extremely serious offenses" as defined in its Code of Discipline.
Petitioners filed a complaint for illegal dismissal.

LA RULING: The Labor Arboter dismissed the petitioners complaint. Dismissal was for a valid cause. The
Arbiter, however, ordered the reinstatement of petitioners to their former positions without backwages
"as a measure of equitable and compassionate relief" owing mainly to petitioners prior unblemished
employment records, show of remorse, harshness of the penalty and defective attendance monitoring system of
respondent. Respondent appealed. Meanwhile, the petitioners moved to the issuance of writ of reinstatement.
The Arbiter issued an alias writ of execution following which respondents bank account at Equitable-PCI Bank
was garnished. Respondent thereupon moved for the quashal of the alias writ of execution and lifting of the
notice of garnishment, which the Arbiter, Respondent appealed to the NLRC.

NLRC RULING: NLRC, granted respondents appeals by deleting the reinstatement aspect of the Arbiters
decision and setting aside the Arbiters Alias Writ of Execution and Notice of Garnishment. Petitioners file a
motion for reconsideration which was denied. They subsequently appealed to CA.

CA RULING: Affirming the finding of LA and NLRC that there was a valid dismissal. Respondent were ordered
to "pay petitioners their corresponding backwages without qualification and deduction for the period
covering October 20, 2004 (date of the Arbiters decision) up to June 30, 2005 (date of the NLRC
Decision)," citing Article 223 of the Labor Code and Roquero v. Philippine Airlines.

Both parties filed their respective motions for partial reconsideration which were denied. Only petitioners
appealed to the SC. Petitioners highlight the Courts ruling in Roquero v. Philippine Airlines where the therein
employer was ordered to pay the wages to which the therein employee was entitled from the time the
reinstatement order was issued until the FINALITY of this Courts decision.

ISSUE: WON petitioners (Lansangan et al) are entitled to full backwages from time the reinstatement order was
issued until the FINALITY of SCs decision.

SC RULING:
NO. The decision of the Arbiter finding that petitioners committed "dishonesty as a form of serious misconduct
and fraud, or breach of trust" had become final, petitioners not having appealed the same before the NLRC as in
fact they even moved for the execution of the reinstatement aspect of the decision. It bears recalling that it was
only respondent which assailed the Arbiter's decision to the NLRC - to solely question the propriety of the order
for reinstatement, and it succeeded.

Roquero, as well as Article 22318 of the Labor Code on which the appellate court also relied, finds no
application in the present case. Article 223 concerns itself with an interim relief, granted to a dismissed or
separated employee while the case for illegal dismissal is pending appeal, as what happened in Roquero. It
does not apply where there is no finding of illegal dismissal, as in the present case.

The Arbiter found petitioners' dismissal to be valid. Such finding had, as stated earlier, become final, petitioners
not having appealed it. Following Article 279 which provides:
San Beda College of Law 308
4S: 2015 - 2016
LABOR LAW REVIEW Atty. Joyrich Golangco

In cases of regular employment, the employer shall not terminate the services of an employee
except for a just cause or when authorized by this Title. An employee who is unjustly dismissed
from work shall be entitled to reinstatement without loss of seniority rights and other privileges and
to his full backwages, inclusive of allowances, and to his other benefits or their monetary
equivalent computed from the time his compensation was withheld from him up to the time of his
actual reinstatement (Emphasis, underscoring and italics supplied), petitioners are not entitled to
full backwages as their dismissal was not found to be illegal. Agabon v. NLRC19 so states ''
payment of backwages and other benefits is justified only if the employee was unjustly
dismissed.

WHEREFORE, the petition is DENIED.

San Beda College of Law 309


4S: 2015 - 2016
LABOR LAW REVIEW Atty. Joyrich Golangco

154. MARILOU S. GENUINO v. NATIONAL LABOR RELATIONS COMMISSION, CITIBANK, N.A., WILLIAM
FERGUSON, and AZIZ RAJKOTWALA
G.R. NOS. 142732-33 December 4, 2007

DOCTRINE:
If the decision of the labor arbiter is later reversed on appeal upon the finding that the ground for dismissal is
valid, then the employer has the right to require the dismissed employee on payroll reinstatement to refund the
salaries s/he received while the case was pending appeal, or it can be deducted from the accrued benefits that
the dismissed employee was entitled to receive from his/her employer under existing laws, collective bargaining
agreement provisions, and company practices. However, if the employee was reinstated to work during the
pendency of the appeal, then the employee is entitled to the compensation received for actual services rendered
without need of refund.

FACTS:
Genuino was employed by Citibank, an American banking corporation duly licensed to do business in the
Philippines, sometime in January 1992 as Treasury Sales Division Head with the rank of Assistant Vice-
President. Genuino's employment was terminated by Citibank on grounds of (1) serious misconduct, (2) willful
breach of the trust reposed upon her by the bank, and (3) commission of a crime against the bank.

On October 15, 1993, Genuino filed before the Labor Arbiter a Complaint against Citibank for illegal suspension
and illegal dismissal with damages and prayer for temporary restraining order and/or writ of preliminary
injunction.

LA RULING: Labor Arbiter found Marilous dismissal to be without just cause and in violation of the latters right
to due process. LA rendered ordered for her reinstatement immediately to her former position, with backwages,
moral and exemplary damages plus 10% of the total monetary award as attorney's fees.

NLRC RULING: The NLRC reversed the Labor Arbiter's decision, declaring the dismissal of the complainant
valid and legal on the ground of serious misconduct and breach of trust and confidence and consequently
dismissing the complaint a quo; but (3) ORDERING the respondent bank to pay the salaries due to the
complainant from the date it reinstated complainant in the payroll (computed at P60,000.00 a month, as found
by the Labor Arbiter) up to and until the date of this decision.

Genuino prayed for the reversal of the NLRC's decision insofar as it declared her dismissal valid and legal.
Meanwhile, Citibank questioned the NLRC's order to pay Genuino's salaries from the date of reinstatement until
the date of the NLRC's decision.

CA RULING: The Court found that the dismissal of Genuino is for a legal and valid ground. It affirmed the ruling
of the NLRC but ordered Citibank to pay Marilou P5,000 as indemnity for non-observance of due process.

ISSUES:

1. Whether or not the dismissal of genuino is for a just cause? and in accordance with due process?
2. WON Citibank needs to "to pay the salaries due to the complainant from the date it reinstated
complainant in the payroll (computed at P60,000.00 a month, as found by the Labor Arbiter) up to and
until the date of this decision,"
SC RULING:

1. YES, dismissal was for just cause but LACKED DUE PROCESS.

The Implementing Rules and Regulations of the Labor Code provide that any employer seeking to dismiss a
worker shall furnish the latter a written notice stating the particular acts or omissions constituting the grounds for
dismissal.25 The purpose of this notice is to sufficiently apprise the employee of the acts complained of and
enable him/her to prepare his/her defense.

In this case, the letters dated August 23, September 13 and 20, 1993 sent by Citibank did not identify the
particular acts or omissions allegedly committed by Genuino. The August 23, 1993 letter charged Genuino with
having "some knowledge and/or involvement" in some transactions "which have the appearance of being
irregular at the least and may even be fraudulent." The September 13, 1993 letter, on the other hand, mentioned
San Beda College of Law 310
4S: 2015 - 2016
LABOR LAW REVIEW Atty. Joyrich Golangco

"irregular transactions" involving Global Pacific and/or Citibank and 12 bank clients. Lastly, the September 20,
1993 letter stated that Genuino and "Mr. Dante Santos, using the facilities of their family corporations (Torrance
and Global) appear to have participated in the diversion of bank clients' funds from Citibank to, and investment
thereof in, other companies and that they made money in the process, in violation of the conflict of law rule
[sic]." The extent of Genuino's alleged knowledge and participation in the diversion of bank's clients' funds,
manner of diversion, and amounts involved; the acts attributed to Genuino that conflicted with the bank's
interests; and the circumstances surrounding the alleged irregular transactions, were not specified in the
notices/letters.

While the bank gave Genuino an opportunity to deny the truth of the allegations in writing and participate in the
administrative investigation, the fact remains that the charges were too general to enable Genuino to intelligently
and adequately prepare her defense.

The two-notice requirement of the Labor Code is an essential part of due process. The first notice informing the
employee of the charges should neither be pro-forma nor vague. It should set out clearly what the employee is
being held liable for. The employee should be afforded ample opportunity to be heard and not mere opportunity.
As explained in King of Kings Transport, Inc., ample opportunity to be heard is especially accorded the
employees sought to be dismissed after they are specifically informed of the charges in order to give them an
opportunity to refute such accusations leveled against them. Since the notice of charges given to Genuino is
inadequate, the dismissal could not be in accordance with due process.

While we hold that Citibank failed to observe procedural due process, we nevertheless find Genuino's dismissal
justified.

2. NO.
Anent the directive of the NLRC in its September 3, 1994 Decision ordering Citibank "to pay the salaries due to
the complainant fromthe date it reinstated complainant in the payroll (computed at P60,000.00 a month, as
found by the Labor Arbiter) up to and until the date of this decision," the Court hereby cancels said award in
view of its finding that the dismissal of Genuino is for a legal and valid ground.

Ordinarily, the employer is required to reinstate the employee during the pendency of the appeal pursuant to Art.
223, paragraph 3 of the Labor Code, which states:

In any event, the decision of the Labor Arbiter reinstating a dismissed or separated employee, insofar as the
reinstatement aspect is concerned, shall immediately be executory, even pending appeal. The employee shall
either be admitted back to work under the same terms and conditions prevailing prior to his dismissal or
separation or, at the option of the employer, merely reinstated in the payroll. The posting of a bond by the
employer shall not stay the execution for reinstatement provided herein.

If the decision of the labor arbiter is later reversed on appeal upon the finding that the ground for dismissal is
valid, then the employer has the right to require the dismissed employee on payroll reinstatement to refund the
salaries s/he received while the case was pending appeal, or it can be deducted from the accrued benefits that
the dismissed employee was entitled to receive from his/her employer under existing laws, collective bargaining
agreement provisions, and company practices. However, if the employee was reinstated to work during the
pendency of the appeal, then the employee is entitled to the compensation received for actual services rendered
without need of refund.

Considering that Genuino was not reinstated to work or placed on payroll reinstatement, and her dismissal is
based on a just cause, then she is not entitled to be paid the salaries stated in item no. 3 of the fallo of the
September 3, 1994 NLRC Decision.

San Beda College of Law 311


4S: 2015 - 2016
LABOR LAW REVIEW Atty. Joyrich Golangco

155. JUANITO A. GARCIA and ALBERTO J. DUMAGO v. PHILIPPINE AIRLINES
G.R. No. 164856 January 20, 2009
CARPIO MORALES, J.:

DOCTRINE:
While reinstatement pending appeal aims to avert the continuing threat or danger to the survival or even the life
of the dismissed employee and his family, it does not contemplate the period when the employer-corporation
itself is similarly in a judicially monitored state of being resuscitated in order to survive.

FACTS:
On July 24, 1995, an administrative charge was filed by PAL against its employees-herein petitioners after they
were allegedly caught in the act of sniffing shabu when a team of company security personnel and law enforcers
raided the PAL Technical Centers Toolroom Section. On October 9, 1995, after due notice, PAL dismissed
petitioners for transgressing the PAL Code of Discipline. Petitioners filed a complaint for illegal dismissal and
damages. Prior to the promulgation of the Labor Arbiters decision, the Securities and Exchange Commission
(SEC) placed PAL, which was suffering from severe financial losses, under an Interim Rehabilitation Receiver.
January 1999-The Interim Rehabilitation Receiver was subsequently replaced by a Permanent Rehabilitation
Receiver.

LA RULING: The Labor Arbiter found that there was illegal dismissal, ordering PAL to, inter alia, immediately
comply with the reinstatement aspect of the decision.

On October 5, 2000 (note: after NLRC reversed LAs decision), the Labor Arbiter issued a Writ of Execution
(Writ) respecting the reinstatement aspect of his January 11, 1999 Decision, and on October 25, 2000, he
issued a Notice of Garnishment. PAL thereupon moved to quash the Writ and to lift the Notice while petitioners
moved to release the garnished amount.

NLRC RULING: The NLRC reversed said decision and dismissed petitioners complaint for lack of merit.
However later on it affirmed the validity of the Writ and the Notice issued by the Labor Arbiter but suspended
and referred the action to the Rehabilitation Receiver for appropriate action.

CA RULING: The CA nullified the NLRC Resolutions on two grounds: (1) a subsequent finding of a valid
dismissal removes the basis for implementing the reinstatement aspect of a labor arbiters decision (the first
ground), and (2) the impossibility to comply with the reinstatement order due to corporate rehabilitation provides
a reasonable justification for the failure to exercise the options under Article 223 of the Labor Code.

ISSUE: Whether petitioners may collect their wages during the period between the Labor Arbiters order of
reinstatement pending appeal and the NLRC decision overturning that of the Labor Arbiter, now that PAL has
exited from rehabilitation proceedings.

SC RULING: NO.

Respondents failure to exercise the alternative options of actual reinstatement and payroll
reinstatement was JUSTIFIED.

While reinstatement pending appeal aims to avert the continuing threat or danger to the survival or even the life
of the dismissed employee and his family, it does not contemplate the period when the employer-corporation
itself is similarly in a judicially monitored state of being resuscitated in order to survive.

PAL, during the period material to the case, was effectively deprived of the alternative choices under Article 223
of the Labor Code, not only by virtue of the statutory injunction but also in view of the interim relinquishment of
management control to give way to the full exercise of the powers of the rehabilitation receiver. Had there been
no need to rehabilitate, respondent may have opted for actual physical reinstatement pending appeal to
optimize the utilization of resources. Then again, though the management may think this wise, the rehabilitation
receiver may decide otherwise, not to mention the subsistence of the injunction on claims.

In sum, the obligation to pay the employees salaries upon the employers failure to exercise the alternative
options under Article 223 of the Labor Code is not a hard and fast rule, considering the inherent constraints of
corporate rehabilitation.
San Beda College of Law 312
4S: 2015 - 2016
LABOR LAW REVIEW Atty. Joyrich Golangco

156. MT. CARMEL COLLEGE v. JOCELYN RESUENA, et. al.
G.R. No. 173076 October 10, 2007
CHICO-NAZARIO, J.:

DOCTRINE:
An illegally dismissed employee is entitled to two reliefs: backwages and reinstatement. The two reliefs
provided are separate and distinct. In instances where reinstatement is no longer feasible because of strained
relations between the employee and the employer, separation pay is granted. In effect, an illegally dismissed
employee is entitled to either reinstatement, if viable, or separation pay if reinstatement is no longer viable, and
backwages.

FACTS:
Petitioner is a private educational institution administered by the Carmelite Fathers at New Escalante, Negros
Occidental while respondents were the employees of petitioner. On November 1997 respondents, together with
several faculty members, non-academic personnel, and other students, participated in a protest action against
petitioner. Because of this, respondents were terminated by petitioner on 15 May 1998. Thus, petitioner filed
separate complaints before Regional Arbitration Branch VI of the NLRC in Bacolod City, charging petitioner with
illegal dismissal and claimed 13th month pay, separation pay, damages and attorney's fees

ISSUES:
1. WON reinstatement in the instant case is self-executory and does not need a writ of execution for its
enforcement
2. WON the continuing award of backwages is proper

LA RULING: Labor Arbiter Drilon found that they were not illegally dismissed but ordered that they be awarded
13th month pay, separation pay and attorneys fees in the amount of P334,875.47.

NLRC RULING: the NLRC reversed the findings of the Labor Arbiter ruling that the termination of respondents
was illegal and ordering the payment of back wages of respondents from 15 May 1998 up to 25 May 1999. It
further directed the reinstatement of respondents or payment of separation pay, with back wages.

CA RULING: The CA affirmed NLRCs decision.

SC RULING:
1. NO. An order for reinstatement must be specifically declared and cannot bepresumed; like back wages,
it is a separate and distinct relief given to an illegally dismissed employee. There being no specific
order for reinstatement and the order being for complainants separation, there can be no basis for the
award of salaries/back wages during the pendency of appeal. This Court had declared in the aforesaid
case that reinstatement during appeal iswarranted only when the Labor Arbiter himself rules that the
dismissed employee should be reinstated. But this was precisely because on appeal to the NLRC, it
found that there was no illegal dismissal; thus, neither reinstatement nor back wages may be awarded.
2. YES. An illegally dismissed employee is entitled to two reliefs: back wages and reinstatement. The two
reliefs provided are separate and distinct. In instances where reinstatement is no longer feasible
because of strained relations between the employee and theemployer, separation pay is granted. In
effect, an illegally dismissed employee is entitled to either reinstatement, if viable, or separation pay if
reinstatement is no longer viable, and back wages.
The normal consequences of respondents illegal dismissal, then, are reinstatement without loss
of seniority rights, and payment of back wages computed from the time compensation was withheld up
to the date of actual reinstatement. Where reinstatement is no longer viable as an option, separation
pay equivalent to one (1)month salary for every year of service should be awarded as an alternative.
Thepayment of separation pay is in addition to payment of back wages.
Concomitantly, it is evident that respondents backwages should not be limited to the period from
15 May 1998 to 25 May 1999. The backwages due respondents must be computed from the time they
were unjustly dismissed until their actual reinstatement to their former position or upon petitioners
payment of separation pay to them if reinstatement is no longer feasible. Thus, until petitioner actually
implements the reinstatement aspect of the NLRC Decision dated 30 October 2001, as affirmed in the
Court of Appeals Decision dated 17 March 2004 in CA-G.R. SP No. 80639, its obligation to
respondents, insofar as accrued backwages and other benefits are concerned, continues to
accumulate.
San Beda College of Law 313
4S: 2015 - 2016
LABOR LAW REVIEW Atty. Joyrich Golangco

157. NERISSA BUENVIAJE et al v. THE HONORABLE COURT OF APPEALS (SPECIAL FORMER
SEVENTH DIVISION), HONORABLE ARBITER ROMULUS PROTASIO, COTTONWAY MARKETING
CORPORATION and MICHAEL G. TONG, President and General Manager

DOCTRINE:
In any event, the decision of the Labor Arbiter reinstating a dismissed or separated employee, insofar as the
reinstatement aspect is concerned, shall immediately be executory, even pending appeal. The employee shall
either be admitted back to work under the same terms and conditions prevailing prior to his dismissal or
separation or, at the option of the employer, merely reinstated in the payroll. The posting of a bond by the
employer shall not stay the execution for reinstatement provided herein

FACTS:
Petitioners were former employees of Cottonway Marketing Corp. (Cottonway), hired as promo girls for their
garment products. In October, 1994, after their services were terminated as the company was allegedly
suffering business losses, petitioners filed with the National Labor Relations Commission (NLRC) a complaint
for illegal dismissal, underpayment of salary, and non-payment of premium pay for rest day, service incentive
leave pay and thirteenth month pay against Cottonway Marketing Corp. and Network Fashion Inc./JCT
International Trading.

LA RULING: The Labor Arbiter issued a Decision finding petitioners' retrenchment valid and ordering Cottonway
to pay petitioners' separation pay and their proportionate thirteenth month pay.

NLRC RULING: The NLRC, in its Decision reversed the Decision of the Labor Arbiter and ordered the
reinstatement of petitioners without loss of seniority rights and other privileges. It also ordered Cottonway to pay
petitioners their proportionate thirteenth month pay and their full backwages inclusive of allowances and other
benefits, or their monetary equivalent computed from the time their salaries were withheld from them up to the
date of their actual reinstatement.

Cottonway filed with the NLRC a manifestation stating that they have complied with the order of reinstatement
by sending notices dated June 5, 1996 requiring the petitioners to return to work, but to no avail; and
consequently, they sent letters to petitioners dated August 1, 1996 informing them that they have lost their
employment for failure to comply with the return to work order.

On November 6, 1997, petitioners filed with the NLRC a motion for execution of its Decision on the ground that it
had become final and executory. Nonetheless, Labor Arbiter Romulus S. Protasio issued an Order declaring
that the award of backwages and proportionate thirteenth month pay to petitioners should be limited from the
time of their illegal dismissal up to the time they received the notice of termination sent by the company upon
their refusal to report for work despite the order of reinstatement.

ISSUE: Whether or not petitioners failure to immediately comply with an order to return for work constitutes
abandonment which justifies their dismissal

SC RULING:
NO. The facts of this case do not support the claim of Cottonway that petitioners have abandoned their desire to
return to their previous work at said company. It appears that three months after the NLRC had rendered its
decision ordering petitioners reinstatement to their former positions, Cottonway sent individual notices to
petitioners mandating them to immediately report to work.

The petitioners, however, were not able to promptly comply with the order. Instead, their counsel, Atty. Roberto
LL. Peralta, sent a reply letter to Atty. De Luna stating that his clients were not in a position to comply with said
order since the NLRC has not yet finally disposed of the case. Consequently, Cottonway sent the petitioners
individual notices of termination.

We note that Cottonway, before finally deciding to dispense with their services, did not give the petitioners the
opportunity to explain why they were not able to report to work. The records also do not bear any proof that all
the petitioners received a copy of the letters. Cottonway merely claimed that some of them have left the country
and some have found other employment. This, however, does not necessarily mean that petitioners were no
longer interested in resuming their employment at Cottonway as it has not been shown that their employment in
the other companies was permanent. It should be expected that petitioners would seek other means of income
San Beda College of Law 314
4S: 2015 - 2016
LABOR LAW REVIEW Atty. Joyrich Golangco

to tide them over during the time that the legality of their termination is under litigation. Furthermore, petitioners
never abandoned their suit against Cottonway. While the case was pending appeal before the NLRC, the Court
of Appeals and this Court, petitioners continued to file pleadings to ensure that the company would comply with
the directive of the NLRC to reinstate them and to pay them full backwages in case said decision is upheld.
Moreover, in his reply to the companys first letter, petitioners counsel expressed willingness to meet with the
companys representative regarding the satisfaction of the NLRC decision.

It appears that the supposed notice sent by Cottonway to the petitioners demanding that they report back to
work immediately was only a scheme to remove the petitioners for good. Petitioners failure to instantaneously
abide by the directive gave them a convenient reason to dispense with their services.

The law mandates the employer to either admit the dismissed employee back to work under the same terms
and conditions prevailing prior to his dismissal or to reinstate him in the payroll to abate further loss of income
on the part of the employee during the pendency of the appeal. But we cannot stretch the language of the law
as to give the employer the right to remove an employee who fails to immediately comply with the reinstatement
order, especially when there is reasonable explanation for the failure.

San Beda College of Law 315


4S: 2015 - 2016
LABOR LAW REVIEW Atty. Joyrich Golangco

158. PFIZER INC v. VELASCO
G.R. No. 177467 March 9, 2011
Leonardo-De Castro, J.:

DOCTRINE:
a. Even if the order of reinstatement of the Labor Arbiter is reversed on appeal, it is obligatory on the part of
the employer to reinstate and pay the wages of the dismissed employee during the period of appeal until
reversal by the higher court.
b. An award by the Labor Arbiter for reinstatement shall be immediately executory even pending appeal and
the posting of a bond by the employer shall not stay the execution for reinstatement. To require the
application for and issuance of a writ of execution as prerequisites for the execution of a reinstatement
award would certainly betray the executory nature of a reinstatement order or award.
c. An employee entitled to reinstatement shall either be admitted back to work under the same terms and
conditions prevailing prior to his dismissal or separation or, at the option of the employer, merely
reinstated in the payroll. It is established in jurisprudence that reinstatement means restoration to a state
or condition from which one had been removed or separated. It presupposes that the previous position
from which one had been removed still exists, or that there is an unfilled position which is substantially
equivalent or of similar nature as the one previously occupied by the employee

FACTS:
Private respondent Geraldine L. Velasco was employed with petitioner PFIZER, INC. as Professional Health
Care Representative. Sometime in April 2003, Velasco had a medical work up for her high-risk pregnancy and was
subsequently advised bed rest which resulted in her extending her leave of absence. While Velasco was still on leave,
PFIZER through its Area Sales Manager personally served Velasco a "Show-cause Notice." Aside from
mentioning about an investigation on her possible violations of company work rules regarding "unauthorized
deals and/or discounts in money or samples and unauthorized withdrawal and/or pull-out of stocks" and
instructing her to submit her explanation on the matter within 48 hours from receipt of the same, the notice also
advised her that she was being placed under "preventive suspension" for 30 days and consequently ordered to
surrender the some accountabilities. In response, Velasco sent a letter addressed to Cortez denying the
charges. Later on, Velasco received a "Second Show-cause Notice" informing her of additional developments in
their investigation Velasco sent a letter to PFIZER asking for additional time to answer the second Show-cause
Notice. That same day, Velasco filed a complaint for illegal suspension with money claims before the Regional
Arbitration Branch. The following day, PFIZER sent her a letter inviting her to a disciplinary hearing. Velasco
received it under protest and informed PFIZER via the receiving copy of the said letter that she had lodged
a complaint against the latter and that the issues that may be raised in the hearing "can be tackled during the
hearing of her case." She likewise opted to withhold answering the second Show-Cause Notice. Thereafter, she
received a third Show-Cause Notice. Finally, she received a termination letter.

LA RULING: The Labor Arbiter held that petitioner employer illegally dismissed the respondent employee,
ordering her reinstatement with backwages. Due to the order of reinstatement issued by the Labor Arbiter,
petitioner employer sent a letter to the respondent employee to report back to work and assigned her to a new
location (from Baguio branch where her residence was to Makati branch)

NLRC RULING: Upheld LA. Pending its appeal, petitioner employer failed to immediately admit respondent
employee back to work despite of an order of reinstatement.

CA RULING: Reversed the decision and ruled that the dismissal was valid. However, it ordered petitioner
employer to pay respondent employee her salary from the date of the Labor Arbiters decision ordering her
reinstatement until the Court of Appeals rendered its decision declaring the dismissal valid. Petitioner employer
questioned the order and refused to pay.

ISSUES:
a. Is it obligatory for the employer to reinstate and pay the wages of the dismissed employee during the
period of appeal even if later on reversal by the higher court?
b. Is the award by the Labor Arbiter for reinstatement immediately executory even pending appeal?
c. Was Pfizer correct in requiring the respondent employee to assign her to a new location in compliance
with the LAs reinstatement order?
SC RULING:

San Beda College of Law 316


4S: 2015 - 2016
LABOR LAW REVIEW Atty. Joyrich Golangco

A. Yes. The Court held that even if the order of reinstatement of the Labor Arbiter is reversed on appeal, it is
obligatory on the part of the employer to reinstate and pay the wages of the dismissed employee during
the period of appeal until reversal by the higher court. On the other hand, if the employee has been
reinstated during the appeal period and such reinstatement order is reversed with finality, the employee is
not required to reimburse whatever salary he received, more so, if he actually rendered services during
the period. The payment of such wages cannot be deemed as unjust enrichment on respondents part.

B. Yes. The Court held that that the provision of Article 223 is clear that an award by the Labor Arbiter for
reinstatement shall be immediately executory even pending appeal and the posting of a bond by the
employer shall not stay the execution for reinstatement. The legislative intent is to make an award of
reinstatement immediately enforceable, even pending appeal. To require the application for and issuance
of a writ of execution as prerequisites for the execution of a reinstatement award would certainly betray
the executory nature of a reinstatement order or award. In the case at bar, petitioner employer did not
immediately admit respondent employee back to work which, according to the law, should have been
done as soon as an order or award of reinstatement is handed down by the Labor Arbiter without need for
the issuance of a writ of execution.

C. No. The Court held that such is not a bona fide reinstatement. Under Article 223 of the Labor Code, an
employee entitled to reinstatement shall either be admitted back to work under the same terms and
conditions prevailing prior to his dismissal or separation or, at the option of the employer, merely
reinstated in the payroll. It is established in jurisprudence that reinstatement means restoration to a state
or condition from which one had been removed or separated. The person reinstated assumes the position
he had occupied prior to his dismissal. Reinstatement presupposes that the previous position from which
one had been removed still exists, or that there is an unfilled position which is substantially equivalent or
of similar nature as the one previously occupied by the employee. Applying the foregoing principle, it
cannot be said that petitioner employer has a clear intent to reinstate respondent employee to her former
position under the same terms and conditions nor to a substantially equivalent position. To begin with, the
return-to-work order petitioner sent to respondent employee is silent with regard to the position it wanted
the respondent employee to assume. Moreover, a transfer of work assignment without any justification
therefor, even if respondent employee would be presumably doing the same job with the same pay,
cannot be deemed as faithful compliance with the reinstatement order.

San Beda College of Law 317


4S: 2015 - 2016
LABOR LAW REVIEW Atty. Joyrich Golangco

159. WENPHIL v. ABING
G.R. No. 207983 April 7, 2014
Brion, J.:

DOCTRINE:
The period for computing the backwages due to the respondents during the period of appeal should end on the
date that a higher court reversed the labor arbitration ruling of illegal dismissal.

FACTS:
This case stemmed from a complaint for illegal dismissal filed by the respondents against Wenphil, docketed as
NLRC NCR Case No. 30-03-00993-00.
LA Bartolabac ruled that the respondents had been illegally dismissed by Wenphil. According to the LA, the
allegation of serious misconduct against the respondents had no factual and legal basis. Consequently, LA
Bartolabac ordered Wenphil to immediately reinstate the respondents to their respective positions or to
equivalent ones, whether actuall or in the payroll. Also, the LA ordered Wenphil to pay the respondents their
backwages until the date of their actual reinstatement.
Wenphil appealed to the NLRC. In the meantime, the respondents moved for the immediate execution of the
LAs decision.

NLRC issued a resolution affirming LA Bartolabacs decision with modifications. Instead of ordering the
respondents reinstatement, the NLRC directed Wenphil to pay the respondents their respective separation pay
at the rate of one (1) month salary for every year of service. Also, the NLRC found that while the respondents
had been illegally dismissed, they had not been illegally suspended. Thus, the period from February 3 to
February 28, 2000 during which the respondents were on preventive suspension was excluded by the NLRC
in the computation of the respondents backwages.

CA rendered its decision reversing the NLRCs finding that the respondents had been illegally dismissed.
According to the CA, there was enough evidence to show that the respondents had been guilty of serious
misconduct; thus, their dismissal was for a valid cause.

SC, in G.R. No. 162447, denied the respondents petition for review on certiorari and affirmed the CAs decision
and resolution. The respondents did not file any motion for reconsideration to question the SCs decision; thus,
the decision became final and executory.

Sometime after the SCs decision in G.R. No. 162447 became final and executory, the respondents filed with LA
Bartolabac a motion for computation and issuance of writ of execution. The respondents asserted in this motion
that although the CAs ruling on the absence of illegal dismissal (as affirmed by the SC) was adverse to them,
under the law and settled jurisprudence, they were still entitled to backwages from the time of their dismissal
until the NLRCs decision finding them to be illegally dismissed was reversed with finality.

LA RULING: LA Bartolabac granted the respondents motion and, in an order, directed Wenphil to pay each
complainant their salaries on reinstatement covering the period from Feb 15, 2002, the date Wenphil last paid
the respondents respective salaries, until Nov. 8, 2002 when the NLRCs decision finding the respondents
illegally dismissed became final and executory.

NLRC RULING: Affirmed LA

CA RULING: CA, in setting aside the NLRCs rulings, relied on the case of Pfizer v. Velasco (G.R. No. 177467,
March 9, 2011, 645 SCRA 135) where the Supreme Court ruled that the backwages of the dismissed employee
should be granted during the period of appeal until reversal by a higher court. Since the first CA decision that
found the respondents had not been illegally dismissed was promulgated on Aug. 27, 2003, then the reversal by
the higher court was effectively made on Aug. 27, 2003

ISSUE: Which computation is correct, the LAs or the CAs?


SC RULING: That of CA.
Among these views, the commanding one is the rule in Pfizer, which merely echoes the rulings the Supreme
Court (SC) made in the cases of Roquero v. Philippine Airlines (G.R. No. 152329, 449 Phil. 437 (2003)) and
Garcia v. Philippine Airlines (G.R. No. 164856, January 20, 2009, 576 SCRA 479) that the period for computing
the backwages due to the respondents during the period of appeal should end on the date that a higher court
San Beda College of Law 318
4S: 2015 - 2016
LABOR LAW REVIEW Atty. Joyrich Golangco

reversed the labor arbitration ruling of illegal dismissal. In this case, the higher court that first reversed the
NLRCs ruling was not the SC but rather the CA. In this light, the CA was correct when it found that that the
period of computation should end on Aug. 27, 2003. The date when the SCs decision became final and
executory need not matter as the rule in Roquero, Garcia and Pfizer merely referred to the date of reversal, not
the date of the ultimate finality of such reversal.

As a last minor detail, we do not agree with the CA that the date of computation should start on Feb. 15, 2002.
Rather, it should be on Feb. 16, 2002. The respondents themselves admitted in their motion for computation
and issuance of writ of execution that the last date when they were paid their backwages was on Feb. 15, 2002.
To start the computation on the same date would result to a duplication of wages for this day; thus, computation
should start on the following date Feb. 16, 2002.

San Beda College of Law 319


4S: 2015 - 2016
LABOR LAW REVIEW Atty. Joyrich Golangco

160. SMART COMMUNICATIONS, INC. v. SOLIDUM
G.R. No. 204646 April 15, 2015
Carpio, J.:

DOCTRINE:
Even if the order of reinstatement of the Labor Arbiter is reversed on appeal, it is obligatory on the part of
the employer to reinstate and pay the wages of the dismissed employee during the period of appeal until
reversal by the higher court.

FACTS:
Smart hired respondent Solidum as Department Head for Smart Buddy Activation. Smart Buddy Activation is
under the Product Marketing Group which is headed by Isla.

Isla gave Solidum a memorandum informing him of alleged acts of dishonesty, directing him to explain why his
employment should not be terminated, and placing him under preventive suspension without pay for 30 days.
On Solidum submitted his written explanation in response to the notice.

Isla gave Solidum a memorandum informing him of a modified set of alleged acts of dishonesty, directing him to
explain why his employment should not be terminated, extending his preventive suspension by 10 days, and
inviting him to an administrative investigation.

Isla gave Solidum a memorandum terminating his employment for fraud or willful breach of trust, falsification,
misrepresentation, conflict of interest, serious misconduct and dishonesty-related offenses.
Solidum filed against Smart a complaint for illegal dismissal, illegal suspension, non-payment of salaries, actual,
moral and exemplary damages, and attorneys fees.

The Labor Arbiter found that Solidums preventive suspension and dismissal were illegal and that he was
entitled to full back wages, moral and exemplary damages, and attorneys fees. LA ordered Solidums
reinstatement. The Labor Arbiter issued a writ of execution ordering the sheriff to collect from petitioners
Solidums accrued salaries, allowances, benefits, incentives and bonuses. Said Labor Arbiter issued seven
other alias writs of execution ordering the sheriff to collect from petitioners Solidums accrued salaries,
allowances, benefits, incentives and bonuses.

On January 26, 2009, the NLRC reversed the Labor Arbiters Decision and dismissed for lack of merit
Solidums complaint. Solidum filed a motion for reconsideration but was denied on May 29, 2009.
Solidum filed with the Labor Arbiter an ex-parte motion praying that an alias writ of execution be issued
directing the sheriff to collect from petitioners Solidums accrued salaries, allowances, benefits, incentives
and bonuses.

LA RULING: LA denied the ex-parte motion because the recent decision of the NLRC reversing the Decision of
this Office prevents any future issuance of any writ of execution on the reinstatement aspect

NLRC RULING: Reversed the LA ruling. Since Smart failed to reinstate Solidum, it held that pursuant to Article
223 of the Labor Code, as amended, relative to the reinstatement aspect of the Labor Arbiters Decision,
respondents are obligated to pay complainants salaries and benefits, computed from the date when
respondents received a copy of the Labor Arbiters Decision which, among others, ordered the
reinstatement of complainant, up to the date of finality of the Commissions resolution reversing the Labor
Arbiters Decision.

CA RULING: Upheld NLRC. Reaffirmed the prevailing principle that even if the order of reinstatement of the
Labor Arbiter is reversed on appeal, it is obligatory on the part of the employer to reinstate and pay the
wages of the dismissed employee during the period of appeal until reversal by the higher court.

ISSUES:
a) Is Smart obligated to pay Solidums salaries and benefits, computed from the date when respondents
received a copy of the Labor Arbiters Decision which ordered the reinstatement of complainant, up
to the date of finality of the Commissions resolution reversing the Labor Arbiters Decision?
b) When did the May 29, 2009 NLRC decision become Final and Executory?
San Beda College of Law 320
4S: 2015 - 2016
LABOR LAW REVIEW Atty. Joyrich Golangco

SC RULING:

a) Yes. In Bago v. NLRC, the Court held that employees are entitled to their accrued salaries, allowances,
benefits, incentives and bonuses until the NLRCs reversal of the labor arbiters order of reinstatement
becomes final and executory, as shown on the entry of judgment.

b) August 10, 2009. Rule VII, Sec. 14 of the 2005 Revised Rules of Procedure of the NLRC provides: The
executive clerk or deputy executive clerk shall consider the decision , resolution or order as final and
executory after sixty (60) days from date of mailing in the absence of return cards, certifications from the
post office, or other proof of service to parties. Since the May 29, 2009 Decision was mailed on 11 June
2009 and in the absence of return cards, the decision became final and executory on 10 August 2009.

San Beda College of Law 321


4S: 2015 - 2016
LABOR LAW REVIEW Atty. Joyrich Golangco

ARTICLE 224 (now Art. 230) Execution of Decisions, Orders, or Awards

161. SY ET.AL. v. FAIRLAND KNITCRAFT CO., INC.


G.R. No. 182915 December 12, 2011
Del Castillo, J.:

DOCTRINE:
a) The Labor Arbiter can acquire jurisdiction over the person of the respondent, even without the latter being
served with summons, through the latters voluntary appearance;

b) Article 224 contemplates the furnishing of copies of final decisions, orders or awards both to the parties
and their counsel in connection with the execution of such final decisions, orders or awards.

FACTS:
Workers Marialy O. Sy et.al. filed with the Arbitration Branch of the NLRC a Complaint for underpayment and/or
non-payment of wages, overtime pay, premium pay for holidays, 13th month pay and other monetary benefits
against Weesan and its owner Susan.
Weesan filed before the DOLE-NCR a report on its temporary closure for a period of not less than six months.
As the workers were not anymore allowed to work on that same day, they filed Amended Complaint and another
pleading entitled Amended Complaints and Position Paper for Complainants, to include the charge of illegal
dismissal and impleaded Fairland and its manager, Debbie, as additional respondents.
A Notice of Hearing was thereafter sent to Weesan requesting it to appear before Labor Arbiter Reyes. On said
date and time, Atty. Antonio A. Geronimo (Atty. Geronimo) appeared as counsel for Weesan and requested for
an extension of time to file his clients position paper. On the next hearing, Atty. Geronimo also entered his
appearance for Fairland and again requested for an extension of time to file position paper.
Atty. Geronimo filed two separate position papers one for Fairland and another for Susan/Weesan. The
Position Paper for Fairland was verified by Debbie while the one for Susan/Weesan was verified by Susan. To
these pleadings, the workers filed a Reply. Atty. Geronimo then filed a Consolidated Reply verified both by
Susan and Debbie.

LA RULING: Dismissed the Complaint for lack of merit.


NLRC RULING: There was Illegal dismissal. Susan/Weesan and Fairland are solidarily liable to the workers

Atty. Geronimo filed a Motion for Reconsideration. However, Fairland filed another Motion for
Reconsideration through Atty. Tecson assailing the jurisdiction of the Labor Arbiter and the NLRC over it,
claiming that it was never summoned to appear, attend or participate in all the proceedings conducted therein. It
also denied that it engaged the services of Atty. Geronimo. The NLRC however, denied both motions for lack of
merit. The NLRC resolution denying the said motions, however, were not served upon Fairland.

CA RULING: It held that the labor tribunals did not acquire jurisdiction over the person of Fairland. Furthermore,
the CA concluded that since Fairland and its counsel were not separately furnished with a copy of the NLRC
Resolution denying the motions for reconsideration of its Decision, said Decision cannot be enforced against
Fairland. The CA likewise concluded that because of this, said Decision which held Susan/Weesan and Fairland
solidarily liable to the workers, has not attained finality.
ISSUES:
a. Did the Labor tribunals acquire jurisdiction over the [person of the] respondent?
b. Has the NLRC Decision holding Susan/Weesan and Fairland solidarily liable to the workers attained
finality?

SC RULING:
a. Yes.
It is basic that the Labor Arbiter cannot acquire jurisdiction over the person of the respondent without the latter
being served with summons.

It must be noted that for the initial complaints, the Labor Arbiter issued summons to Susan/Weesan which was
received by the latter. The workers thereafter amended their then already consolidated complaints to include
illegal dismissal as an additional cause of action as well as Fairland and Debbie as additional respondents. We
have, however, scanned the records but found nothing to indicate that summons with respect to the said

San Beda College of Law 322


4S: 2015 - 2016
LABOR LAW REVIEW Atty. Joyrich Golangco

amended complaints was ever served upon Weesan, Susan, or Fairland. True to their claim, Fairland and
Debbie were indeed never summoned by the Labor Arbiter.
Although not served with summons, jurisdiction over Fairland and Debbie was acquired through their voluntary
appearance.
The fact that Atty. Geronimo entered his appearance for Fairland and Debbie and that he actively defended
them before the Labor Arbiter raised the presumption that he is authorized to appear for them. It is unlikely that
Atty. Geronimo would have been so irresponsible as to represent Fairland and Debbie if he were not in fact
authorized. As an officer of the Court, Atty. Geronimo is presumed to have acted with due propriety. Moreover,
"[i]t strains credulity that a counsel who has no personal interest in the case would fight for and defend a case
with persistence and vigor if he has not been authorized or employed by the party concerned."

b. Yes
Citing PNOC Dockyard and Engineering Corporation v. National Labor Relations Commission, the CA likewise
emphasized that in labor cases, both the party and his counsel must be duly served their separate copies
of the order, decision or resolution unlike in ordinary proceedings where notice to counsel is deemed
notice to the party. It then quoted Article 224 of the Labor Code as follows:

ARTICLE 224. Execution of decisions, orders or awards. (a) the Secretary of Labor and
Employment or any Regional Director, the Commission or any Labor Arbiter, or med-arbiter or
voluntary arbitrator may, motu proprio or on motion of any interested party, issue a writ of
execution on a judgment within five (5) years from the date it becomes final and executory,
requiring a sheriff or a duly deputized officer to execute or enforce final decisions, orders or
awards of the Secretary of Labor and Employment or [R]egional Director, the Commission, the
Labor Arbiter or Med-Arbiter, or Voluntary Arbitrators. In any case, it shall be the duty of the
responsible officer to separately furnish immediately the counsels of record and the
parties with copies of said decision, orders or awards. Failure to comply with the duty
prescribed herein shall subject such responsible officer to appropriate administrative sanctions
x x x (Emphasis in the original).

The CA then concluded that since Fairland and its counsel were not separately furnished with a copy of the
NLRC Resolution denying the motions for reconsideration of its 2004 Decision, said Decision cannot be
enforced against Fairland.

We cannot agree.

To stress, Article 224 contemplates the furnishing of copies of final decisions, orders or awards both to the
parties and their counsel in connection with the execution of such final decisions, orders or awards.
However, for the purpose of computing the period for filing an appeal from the NLRC to the CA, same
shall be counted from receipt of the decision, order or award by the counsel of record pursuant to the
established rule that notice to counsel is notice to party. And since the period for filing of an appeal is
reckoned from the counsels receipt of the decision, order or award, it necessarily follows that the reckoning
period for their finality is likewise the counsels date of receipt thereof, if a party is represented by counsel.
Hence, the date of receipt referred to in Sec. 14, Rule VII of the then in force New Rules of Procedure of the
NLRC which provides that decisions, resolutions or orders of the NLRC shall become executory after 10
calendar days from receipt of the same, refers to the date of receipt by counsel. Thus contrary to the CAs
conclusion, the said NLRC Decision became final, as to Fairland, 10 calendar days after Atty. Tecsons
receipt thereof.

San Beda College of Law 323


4S: 2015 - 2016
LABOR LAW REVIEW Atty. Joyrich Golangco

162. YUPANGCO COTTON MILLS v. CA
G.R. No. 126322 January 16, 2002
Pardo, J.:

DOCTRINE:
A third party whose property has been levied upon by a sheriff to enforce a decision against a judgment debtor
is afforded with several alternative remedies to protect its interests. The third party may avail himself of
alternative remedies cumulatively, and one will not preclude the third party from availing himself of the other
alternative remedies in the event he failed in the remedy first availed of.

FACTS:
From the records before us and by petitioners own allegations and admission, it has taken the following actions
in connection with its claim that a sheriff of the National Labor Relations Commission erroneously and
unlawfully levied upon certain properties which it claims as its own.

1. It filed a notice of third-party claim with the Labor Arbiter on May 4, 1995.
2. It filed an Affidavit of Adverse Claim with the National Labor Relations Commission (NLRC) on July 4, 1995,
which was dismissed on August 30, 1995, by the Labor Arbiter.
3. It filed a petition for certiorari and prohibition with the Regional Trial Court of Manila, Branch 49, docketed as
Civil Case No. 95-75628 on October 6, 1995. The Regional Trial Court dismissed the case on October 11,
1995 for lack of merit.
4. It appealed to the NLRC the order of the Labor Arbiter dated August 13, 1995 which dismissed the appeal
for lack of merit on December 8, 1995.
5. It filed an original petition for mandatory injunction with the NLRC on November 16, 1995. This was
docketed as Case No. NLRC-NCR-IC. 0000602-95. This case is still pending with that Commission.
6. It filed a complaint in the Regional Trial Court in Manila which was docketed as Civil Case No. 95-76395.
The dismissal of this case by public respondent triggered the filing of the instant petition.

In all of the foregoing actions, petitioner raised a common issue, which is that it is the owner of the properties
located in the compound and buildings of Artex Development Corporation, which were erroneously levied upon
by the sheriff of the NLRC as a consequence of the decision rendered by the said Commission in a labor case
docketed as NLRC-NCR Case No. 00-05-02960-90.

CA RULING: Court of Appeals promulgated a decision dismissing the petition on the ground of forum shopping
and that petitioners remedy was to seek relief from this Court.
Petitioner filed with the Court of Appeals a motion for reconsideration of the decision. Petitioner argued that the
filing of a complaint for accion reinvindicatoria with the Regional Trial Court was proper because it is a remedy
specifically granted to an owner (whose properties were subjected to a writ of execution to enforce a decision
rendered in a labor dispute in which it was not a party) by Section 17 (now 16), Rule 39, Revised Rules of Court
and by several doctrines.
Court of Appeals denied petitioners motion for reconsideration

ISSUES:
a) Was there forum shopping in this case?
b) May a third party be precluded the from availing himself of the other alternative remedies in the event he
failed in the remedy first availed of?
SC RULING:
1) FORUM SHOPPING. There is no forum-shopping where two different orders were questioned, two distinct
causes of action and issues were raised, and two objectives were sought.

In the case at bar, there was no identity of parties, rights and causes of action and reliefs sought.

The case before the NLRC where Labor Arbiter Reyes issued a writ of execution on the property of petitioner
was a labor dispute between Artex and Samar-Anglo. Petitioner was not a party to the case. The only issue
petitioner raised before the NLRC was whether or not the writ of execution issued by the labor arbiter could be
satisfied against the property of petitioner, not a party to the labor case.

On the other hand, the accion reinvindicatoria filed by petitioner in the trial court was to recover the property
illegally levied upon and sold at auction. Hence, the causes of action in these cases were different.
San Beda College of Law 324
4S: 2015 - 2016
LABOR LAW REVIEW Atty. Joyrich Golangco

2) THIRD PARTY CLAIM. A third party whose property has been levied upon by a sheriff to enforce a decision
against a judgment debtor is afforded with several alternative remedies to protect its interests. The third party
may avail himself of alternative remedies cumulatively, and one will not preclude the third party from availing
himself of the other alternative remedies in the event he failed in the remedy first availed of.

Thus, a third party may avail himself of the following alternative remedies:
a) File a third party claim with the sheriff of the Labor Arbiter, and
b) If the third party claim is denied, the third party may appeal the denial to the NLRC.

The remedies above mentioned are cumulative and may be resorted to by a third-party claimant independent of
or separately from and without need of availing of the others. If a third-party claimant opted to file a proper
action to vindicate his claim of ownership, he must institute an action, distinct and separate from that in which
the judgment is being enforced, with the court of competent jurisdiction even before or without need of filing a
claim in the court which issued the writ, the latter not being a condition sine qua non for the former. In such
proper action, the validity and sufficiency of the title of the third-party claimant will be resolved and a writ of
preliminary injunction against the sheriff may be issued.

San Beda College of Law 325


4S: 2015 - 2016
LABOR LAW REVIEW Atty. Joyrich Golangco

163. ANDO v. CAMPO
G.R. No. 184007 February 16, 2011
NACHURA, J.

DOCTRINE:
Regular courts have no jurisdiction to hear and decide questions which arise from and are incidental to the
enforcement of decisions, orders, or awards rendered in labor cases by appropriate officers and tribunals of the
Department of Labor and Employment. Thus, it is, first and foremost, the NLRC Manual on the Execution of
Judgment that governs any question on the execution of a judgment of that body. However, the power of the
NLRC, or the courts, to execute its judgment extends only to properties unquestionably belonging to the
judgment debtor alone. A sheriff, therefore, has no authority to attach the property of any person except that of
the judgment debtor.

FACTS:
Petitioner was the president of Premier Allied and Contracting Services, Inc. (PACSI), an independent labor
contractor. Respondents were hired by PACSI as pilers or haulers tasked to manually carry bags of sugar from
the warehouse of Victorias Milling Company and load them on trucks. On June 1998, respondents were
dismissed from employment. They filed a case for illegal dismissal and with money
claims.http://www.lawphil.net/judjuris/juri2011/feb2011/gr_184007_2011.html - fnt5 On June 14, 2001, Labor
Arbiter Pura promulgated a decision, ruling in respondents favor. PACSI and petitioner were directed to pay a
total of P422, 702.28, representing respondents separation pay and the award of attorneys
fees.http://www.lawphil.net/judjuris/juri2011/feb2011/gr_184007_2011.html - fnt7 Petitioner and PACSI
appealed to the NLRC. The NLRC ruled that petitioner failed to perfect his appeal because he did not pay the
supersedeas bond. It also affirmed the Labor Arbiters decision with modification of the award for separation pay
to four other employees who were similarly situated. Upon finality of the decision, respondents moved for its
execution.http://www.lawphil.net/judjuris/juri2011/feb2011/gr_184007_2011.html - fnt9

To answer for the monetary award, the NLRC Acting Sheriff issued a Notice of Sale on Execution of Personal
Property over the property in the name of "Paquito V. Ando x x x married to Erlinda S. Ando." This prompted
petitioner to file an action for prohibition and damages with prayer for the issuance of a TRO before the Regional
Trial Court of Bacolod City. Petitioner claimed that the property belonged to him and his wife, not to the
corporation, and, hence, could not be subject of the execution sale. Since it is the corporation that was the
judgment debtor, execution should be made on the latters
properties.http://www.lawphil.net/judjuris/juri2011/feb2011/gr_184007_2011.html - fnt11

RTC RULING: The RTC issued an Order denying the prayer for a TRO, holding that the trial court had no
jurisdiction to try and decide the case. The RTC ruled that, pursuant to the NLRC Manual on the Execution of
Judgment, petitioners remedy was to file a third-party claim with the NLRC Sheriff. Despite lack of jurisdiction,
however, the RTC went on to decide the merits of the case. Petitioner did not file a motion for reconsideration of
the RTC Order. Instead, he filed a petition for certiorari under Rule 65 before the CA.

Petitioner argued that the writ of execution was issued improvidently or without authority since the property to be
levied belonged to him in his personal capacity and his wife. The RTC, respondent contended, could stay
the execution of a judgment if the same was unjust. He also contended that, pursuant to a ruling of this Court, a
third party who is not a judgment creditor may choose between filing a third-party claim with the NLRC sheriff or
filing a separate action with the courts. He maintains that this special civil action is purely civil in nature since it
involves the manner in which the writ of execution in a labor case will be implemented against the property of
petitioner which is not a corporate property of PACSI. What he is seeking to be restrained, petitioner maintains,
is not the Decision itself but the manner of its execution. Further, he claims that the property levied has been
constituted as a family home within the contemplation of the Family Code.

CA RULING: The CA affirmed the RTC Order in so far as it dismissed the complaint on the ground that it had
no jurisdiction over the case, and nullified all other pronouncements in the same Order. Petitioner moved for
reconsideration, but the motion was denied. Hence, this petition.

ISSUE: Do regular courts have jurisdiction over the enforcement of decisions, orders or awards rendered in
labor cases?

San Beda College of Law 326


4S: 2015 - 2016
LABOR LAW REVIEW Atty. Joyrich Golangco

SC RULING:
NO. The Court has long recognized that regular courts have no jurisdiction to hear and decide questions which
arise from and are incidental to the enforcement of decisions, orders, or awards rendered in labor cases by
appropriate officers and tribunals of the Department of Labor and Employment. To hold otherwise is to sanction
splitting of jurisdiction which is obnoxious to the orderly administration of justice. Thus, it is, first and foremost,
the NLRC Manual on the Execution of Judgment that governs any question on the execution of a judgment of
that body. Petitioner need not look further than that. The Rules of Court apply only by analogy or in a suppletory
character.

The NLRC Manual on the Execution of Judgment deals specifically with third-party claims in cases brought
before that body. It defines a third-party claim as one where a person, not a party to the case, asserts title to or
right to the possession of the property levied upon. It also sets out the procedure for the filing of a third-party
claim. There is no doubt in our mind that petitioners complaint is a third- party claim within the cognizance of the
NLRC. Petitioner may indeed be considered a "third party" in relation to the property subject of the execution
vis--vis the Labor Arbiters decision. There is no question that the property belongs to petitioner and his wife,
and not to the corporation. It can be said that the property belongs to the conjugal partnership, not to petitioner
alone. Thus, the property belongs to a third party, i.e., the conjugal partnership. At the very least, the Court can
consider that petitioners wife is a third party within contemplation of the law.

The broad powers granted to the Labor Arbiter and to the National Labor Relations Commission by Articles 217,
218 and 224 of the Labor Code can only be interpreted as vesting in them jurisdiction over incidents arising
from, in connection with or relating to labor disputes, as the controversy under consideration, to the exclusion of
the regular courts. There is no denying that the present controversy arose from the complaint for illegal
dismissal. The subject matter of petitioners complaint is the execution of the NLRC decision. Execution is an
essential part of the proceedings before the NLRC. Jurisdiction, once acquired, continues until the case is finally
terminated, and there can be no end to the controversy without the full and proper implementation of the
commissions directives.

That said, however, we resolve to put an end to the controversy right now, considering the length of time that
has passed since the levy on the property was made.

Petitioner claims that the property sought to be levied does not belong to PACSI, the judgment debtor, but to
him and his wife. Since he was sued in a representative capacity, and not in his personal capacity, the property
could not be made to answer for the judgment obligation of the corporation.

Moreover, the power of the NLRC, or the courts, to execute its judgment extends only to properties
unquestionably belonging to the judgment debtor alone. A sheriff, therefore, has no authority to attach the
property of any person except that of the judgment debtor. Likewise, there is no showing that the sheriff ever
tried to execute on the properties of the corporation. In sum, while petitioner availed himself of the wrong
remedy to vindicate his rights, nonetheless, justice demands that this Court look beyond his procedural missteps
and grant the petition.

San Beda College of Law 327


4S: 2015 - 2016
LABOR LAW REVIEW Atty. Joyrich Golangco

ARTICLE 226 (now Art. 232) Bureau of Labor Relations

164. EMPLOYEES UNION OF BAYER PHILS. v. BAYER PHILS.


G.R. No. 162943 December 6, 2010
VILLARAMA, JR., J.

DOCTRINE:
The registered CBA serves as the covenant between the parties and has the force and effect of law between
them during the period of its duration. If the employer grossly violates its CBA with the duly recognized union,
the former may be held administratively and criminally liable for unfair labor practice. The utter disregard of the
very existence of the CBA itself, is a gross violation of the CBA per se and is an act of ULP. When an employer
proceeds to negotiate with a splinter union despite the existence of its valid CBA with the duly certified and
exclusive bargaining agent, the former indubitably abandons its recognition of the latter and terminates the
entire CBA.

FACTS:
Petitioner Employees Union of Bayer Philippines (EUBP) is the exclusive bargaining agent of all rank-and-file
employees of Bayer Philippines (Bayer), and is an affiliate of the Federation of Free Workers (FFW). In 1997,
EUBP, headed by its president Juanito S. Facundo (Facundo), negotiated with Bayer for the signing of a
collective bargaining agreement (CBA). During the negotiations, EUBP rejected Bayers 9.9% wage-increase
proposal resulting in a bargaining deadlock. Subsequently, EUBP staged a strike, prompting the Secretary of
the DOLE to assume jurisdiction over the dispute.

Respondent Avelina Remigio (Remigio) and 27 other union members, without any authority from their union
leaders, accepted Bayers wage-increase proposal. EUBPs grievance committee questioned Remigios action
and reprimanded Remigio and her allies. The DOLE Secretary issued an arbitral award ordering EUBP and
Bayer to execute a CBA.

Barely six months from the signing of the new CBA, during a company-sponsored seminar, Remigio solicited
signatures from union members in support of a resolution containing the decision of the signatories to: (1)
disaffiliate from FFW, (2) rename the union as Reformed Employees Union of Bayer Philippines (REUBP), (3)
adopt a new constitution and by-laws for the union, (4) abolish all existing officer positions in the union and elect
a new set of interim officers, and (5) authorize REUBP to administer the CBA between EUBP and Bayer. The
said resolution was signed by 147 of the 257 local union members.

With both seeking recognition from Bayer and demanding remittance of the union dues collected from its rank-
and-file members. Remigios splinter group wrote Facundo, FFW and Bayer informing them of the decision of
the majority of the union members to disaffiliate from FFW. Bayer responded by deciding not to deal with either
of the two groups, and by placing the union dues collected in a trust account until the conflict between the two
groups is resolved.

EUBP filed a complaint for unfair labor practice (first ULP complaint) against Bayer for non-remittance of union
dues. EUBP later sent a letter to Bayer asking for a grievance conference. Apparently, the two groups failed to
settle their issues. Bayer decided to turn over the collected union dues amounting to P254,857.15 to respondent
Anastacia Villareal, Treasurer of REUBP. EUBP lodged a complaint against Remigios group before the
Industrial Relations Division of the DOLE praying for their expulsion from EUBP for commission of "acts that
threaten the life of the union." Labor Arbiter Jovencio Ll. Mayor, Jr. dismissed the first ULP complaint for lack of
jurisdiction. Petitioners then filed a second ULP complaint against herein respondents.

The Regional Director of the Industrial Relations Division of DOLE issued a decision dismissing the issue on
expulsion filed by EUBP against Remigio and her allies for failure to exhaust reliefs within the union and
ordering the conduct of a referendum to determine which of the two groups should be recognized as union
officers. The BLR reversed the Regional Directors ruling and ordered the management of Bayer to respect the
authority of the duly-elected officers of EUBP in the administration of the prevailing CBA.

LA RULING: Labor Arbiter Waldo Emerson R. Gan dismissed EUBPs second ULP complaint for lack of
jurisdiction.

NLRC RULING: Affirmed the decision.


San Beda College of Law 328
4S: 2015 - 2016
LABOR LAW REVIEW Atty. Joyrich Golangco

CA RULING: The CA sustained both the Labor Arbiter and the NLRCs rulings.

ISSUE: Can the act of the management of Bayer in dealing and negotiating with Remigios splinter group,
despite its validly existing CBA with EUBP, be considered unfair labor practice?

SC RULING:
YES. It must be remembered that a CBA is entered into in order to foster stability and mutual cooperation
between labor and capital. An employer should not be allowed to rescind unilaterally its CBA with the duly
certified bargaining agent it had previously contracted with, and decide to bargain anew with a different group if
there is no legitimate reason for doing so and without first following the proper procedure. If such behavior would
be tolerated, bargaining and negotiations between the employer and the union will never be truthful and
meaningful, and no CBA forged after arduous negotiations will ever be honored or be relied upon. Article 253 of
the Labor Code, as amended, plainly provides:

ART. 253. Duty to bargain collectively when there exists a collective bargaining agreement. Where
there is a collective bargaining agreement, the duty to bargain collectively shall also mean that neither
party shall terminate or modify such agreement during its lifetime. However, either party can serve a
written notice to terminate or modify the agreement at least sixty (60) days prior to its expiration date. It
shall be the duty of both parties to keep the status quo and to continue in full force and effect the terms
and conditions of the existing agreement during the 60-day period and/or until a new agreement is
reached by the parties. (Emphasis supplied.)

This is the reason why it is axiomatic in labor relations that a CBA entered into by a legitimate labor organization
that has been duly certified as the exclusive bargaining representative and the employer becomes the law
between them. Additionally, in the Certificate of Registration issued by the DOLE, it is specified that the
registered CBA serves as the covenant between the parties and has the force and effect of law between them
during the period of its duration. Compliance with the terms and conditions of the CBA is mandated by express
policy of the law primarily to afford protection to labor and to promote industrial peace. Thus, when a valid and
binding CBA had been entered into by the workers and the employer, the latter is behooved to observe the
terms and conditions thereof bearing on union dues and representation. If the employer grossly violates its CBA
with the duly recognized union, the former may be held administratively and criminally liable for unfair labor
practice.

Indeed, in Silva v. National Labor Relations Commission, we explained the correlations of Article 248 (1) and
Article 261 of the Labor Code to mean that for a ULP case to be cognizable by the Labor Arbiter, and for the
NLRC to exercise appellate jurisdiction thereon, the allegations in the complaint must show prima facie the
concurrence of two things, namely: (1) gross violation of the CBA; and (2) the violation pertains to the economic
provisions of the CBA.

This pronouncement in Silva, however, should not be construed to apply to violations of the CBA which can be
considered as gross violations per se, such as utter disregard of the very existence of the CBA itself, similar to
what happened in this case. When an employer proceeds to negotiate with a splinter union despite the
existence of its valid CBA with the duly certified and exclusive bargaining agent, the former indubitably
abandons its recognition of the latter and terminates the entire CBA.

San Beda College of Law 329


4S: 2015 - 2016
LABOR LAW REVIEW Atty. Joyrich Golangco

165. MONTAO v. VERCELES
G.R. No.168583 July 26, 2010
DEL CASTILLO, J.

DOCTRINE:
Section 226 of the Labor Code clearly provides that the BLR and the Regional Directors of DOLE have concurrent
jurisdiction over inter-union and intra-union disputes. Such disputes include the conduct or nullification of election of union
and workers association officers.

It is true that under the Implementing Rules, redress must first be sought within the organization itself in accordance with its
constitution and by-laws. However, this requirement is not absolute but yields to exception under varying circumstances.

FACTS:
Atty. Montao worked as legal assistant of FFW Legal Center. Subsequently, he joined the union of rank-and-file
employees, the FFW Staff Association, and eventually became the employees union president in July 1997. In
November 1998, he was likewise designated officer-in-charge of FFW Legal Center.

During the 21st National Convention and Election of National Officers of FFW, Atty. Montao was nominated for
the position of National Vice-President. In a letter dated May 25, 2001, however, the Commission on Election
(FFW COMELEC), informed him that he is not qualified for the position as his candidacy violates the 1998 FFW
Constitution and By-Laws. Atty. Montao thus filed an Urgent Motion for Reconsideration praying that his name
be included in the official list of candidates.

Election ensued on May 26-27, 2001 in the National Convention held at Subic International
Hotel, Olongapo City. Despite the pending motion for reconsideration with the FFW COMELEC, and strong
opposition and protest of respondent Atty. Ernesto C. Verceles (Atty. Verceles), a delegate to the convention
and president of University of the East Employees Association (UEEA-FFW) which is an affiliate union of FFW,
the convention delegates allowed Atty. Montaos candidacy. He emerged victorious and was proclaimed as the
National Vice-President.

Atty. Verceles, as President of UEEA-FFW and officer of the Governing Board of FFW, filed before the BLR a
petition for the nullification of the election of Atty. Montao as FFW National Vice-President. He alleged that, as
already ruled by the FFW COMELEC, Atty. Montao is not qualified to run for the position the FFW Constitution
and By-Laws prohibits federation employees from sitting in its Governing Board. Claiming that Atty. Montaos
premature assumption of duties and formal induction as vice-president will cause serious damage, Atty.
Verceles likewise prayed for injunctive relief.

Atty. Montao filed his Comment with Motion to Dismiss on the grounds that the Regional Director of the
Department of Labor and Employment (DOLE) and not the BLR has jurisdiction over the case; that the filing of
the petition was premature due to the pending and unresolved protest before the FFW COMELEC; and that,
Atty. Verceles has no legal standing to initiate the petition not being the real party in interest.

BLR RULING: The BLR, in its Order dated August 20, 2001, did not give due course to Atty. Montaos Motion to
Dismiss but ordered the latter to submit his answer to the petition pursuant to the rules. The parties thereafter
submitted their respective pleadings and position papers.

On May 8, 2002, the BLR rendered a Decision dismissing the petition for lack of merit. While it upheld its
jurisdiction over the intra-union dispute case and affirmed, as well, Atty. Verceles legal personality to institute
the action as president of an affiliate union of FFW, the BLR ruled that there were no grounds to hold Atty.
Montao unqualified to run for National Vice-President of FFW. It held that the applicable provision in the FFW
Constitution and By-Laws to determine whether one is qualified to run for office is not Section 76 of Article
XIX but Section 26 of Article VIII thereof. The BLR opined that there was sufficient compliance with the
requirements laid down by this applicable provision and, besides, the convention delegates unanimously
decided that Atty. Montao was qualified to run for the position of National Vice-President. Atty. Verceles filed a Motion
for Reconsideration but it was denied by the BLR.

CA RULING: CA set aside the BLRs Decision. While it agreed that jurisdiction was properly lodged with the
BLR, that Atty. Verceles has legal standing to institute the petition, and that the applicable provision of FFW
Constitution and By-Laws is Section 26 of Article VIII and not Section 76 of Article XIX, the CA however ruled
San Beda College of Law 330
4S: 2015 - 2016
LABOR LAW REVIEW Atty. Joyrich Golangco

that Atty. Montao did not possess the qualification requirement under paragraph (d) of Section 26 that
candidates must be an officer or member of a legitimate labor organization. According to the CA, since Atty.
Montao, as legal assistant employed by FFW, is considered as confidential employee, consequently, he is
ineligible to join FFW Staff Association, the rank-and-file union of FFW. The CA, thus, granted the petition and
nullified the election of Atty. Montao as FFW National Vice-President.

Montao raised before the SC the claim that the BLR has no jurisdiction over cases involving protests and
petitions for annulment of results of elections as such jurisdiction is expressly conferred by law to the Regional
Directors of the DOLE. He also reiterated that the petition was prematurely filed and thus must be dismissed for
failure to exhaust all remedies as mandated by the implementing rules of the Labor Code.

ISSUES:
(1) Does the BLR have jurisdiction to decide election contests despite express provision of law granting
said jurisdiction?
(2) Was the petition by respondent prematurely filed?

SC RULING:
(1) Yes. We find no merit in petitioners claim that under Section 6 of Rule XV in relation to Section 1 of Rule
XIV of Book V of the Omnibus Rules Implementing the Labor Code, it is the Regional Director of the DOLE and
not the BLR who has jurisdiction over election protests.

Section 226 of the Labor Code clearly provides that the BLR and the Regional Directors of DOLE have
concurrent jurisdiction over inter-union and intra-union disputes. Such disputes include the conduct or
nullification of election of union and workers association
officers.http://sc.judiciary.gov.ph/jurisprudence/2010/july2010/168583.htm - _ftn29 There is, thus, no doubt as to
the BLRs jurisdiction over the instant dispute involving member-unions of a federation arising from disagreement
over the provisions of the federations constitution and by-laws.

We agree with BLRs observation that:

Rule XVI lays down the decentralized intra-union dispute settlement mechanism. Section 1
states that any complaint in this regard shall be filed in the Regional Office where the union is
domiciled. The concept of domicile in labor relations regulation is equivalent to the place where
the union seeks to operate or has established a geographical presence for purposes of
collective bargaining or for dealing with employers concerning terms and conditions of
employment.

The matter of venue becomes problematic when the intra-union dispute involves a federation,
because the geographical presence of a federation may encompass more than one
administrative region. Pursuant to its authority under Article 226, this Bureau exercises original
jurisdiction over intra-union disputes involving federations. It is well-settled that FFW, having
local unions all over the country, operates in more than one administrative region. Therefore,
this Bureau maintains original and exclusive jurisdiction over disputes arising from any violation
of or disagreement over any provision of its constitution and by-laws.

(2) No. There is likewise no merit to petitioners argument that the petition should have been immediately
dismissed due to a pending and unresolved protest before the FFW COMELEC pursuant to Section 6, Rule XV,
Book V of the Omnibus Rules Implementing the Labor Code.

It is true that under the Implementing Rules, redress must first be sought within the organization itself in
accordance with its constitution and by-laws. However, this requirement is not absolute but yields to exception
under varying circumstances. In the case at bench, Atty. Verceles made his protest over Atty. Montaos
candidacy during the plenary session before the holding of the election proceedings. The FFW COMELEC,
notwithstanding its reservation and despite objections from certain convention delegates, allowed Atty. Montaos
candidacy and proclaimed him winner for the position. Under the rules, the committee on election shall
endeavour to settle or resolve all protests during or immediately after the close of election proceedings and any
protest left unresolved shall be resolved by the committee within five days after the close of the election
proceedings. A day or two after the election, Atty. Verceles made his written/formal protest over Atty. Montaos
candidacy/proclamation with the FFW COMELEC. He exhausted the remedies under the constitution and by-
San Beda College of Law 331
4S: 2015 - 2016
LABOR LAW REVIEW Atty. Joyrich Golangco

laws to have his protest acted upon by the proper forum and even asked for a formal hearing on the
matter. Still, the FFW COMELEC failed to timely act thereon. Thus, Atty. Verceles had no other recourse but to
take the next available remedy to protect the interest of the union he represents as well as the whole federation,
especially so that Atty. Montao, immediately after being proclaimed, already assumed and started to perform
the duties of the position. Consequently, Atty. Verceles properly sought redress from the BLR so that the right to
due process will not be violated. To insist on the contrary is to render the exhaustion of remedies within the
union as illusory and vain.

As regards the issue of whether Atty. Montao is qualified to run as FFW National President in view of the
prohibition established in Section 76, Article XIX of the 1998 FFW Constitution and By-Laws, the SC concurred
with the CA that Atty. Montao is not qualified to run for the position but not for failure to meet the requirement
specified under Section 26 (d) of Article VIII of FFW Constitution and By-Laws. We note that the CAs
declaration of the illegitimate status of FFW Staff Association is proscribed by law, owing to the preclusion of
collateral attack. We nonetheless resolve to affirm the CAs finding that Atty. Montao is disqualified to run for the
position of National Vice-President in view of the proscription in the FFW Constitution and By-Laws on
federation employees from sitting in its Governing Board. Accordingly, the election of Atty. Montao as FFW
Vice-President is null and void.

San Beda College of Law 332


4S: 2015 - 2016
LABOR LAW REVIEW Atty. Joyrich Golangco

166. DIOKNO v. CADAC
G.R. No.168475 July 4, 2007
CHICO-NAZARIO, J.

DOCTRINE:
BLR has original jurisdiction on all inter-union and intra-union conflicts. Since Art. 226 declared the BLR shall
have original and exclusive authority to act on all inter-union and intra-union conflicts, there should be no doubt
as to its jurisdiction. Intra-union conflict refers to a conflict within or inside a labor union.
(Exceptions to the Doctrine of Exhaustion of Administrative Remedies)

FACTS:
Petitioners and Respondents are members of First Line Association of Meralco Supervisory Employees
(FLAMES) which is the supervisory union of Meralco.

FLAMES COMELEC rejected the candidacy of Ong et al (Ong, Alvarez, Escall, Valeriano). Ong et al filed a
petition before the med-arbitration of DOLE to nullify the order of COMELEC. Consequently, DOLE personnel
were directed to observe the conduct of the FLAMES election.

Petitioners sought the disqualification of respondents Daya et al (Daya, lucas, tabilog, espiritu, vito, de luna,
yalung, layug, pabilona, reyes, escall, alcantara, cervitillo, morelos, ermine). Petitioners alleged that Daya, et
al., allowed themselves to be assisted by non-union members, and committed acts of disloyalty which are
inimical to the interest of FLAMES. In their campaign, they allegedly colluded with the officers of the Meralco
Savings and Loan Association (MESALA) and the Meralco Mutual Aid and Benefits Association (MEMABA) and
exerted undue influence on the members of FLAMES. As a result, the COMELEC disqualified Daya et al from
the election. On May 7, 2003 COMELEC proclaimed the winner of the elections which included Diokno as
president. Daya et al and Ong et al filed with the med-arbitration unit of DOLE a petition to nullify the order of
disqualification, election proceedings and counting of votes.

Another group, Jimenez et al (Jimenez Jr., Reyes, Gavino, Vidanes, Tayao, Cirujano, Cadavona, Caoc, Maclit,
Acorda, Ragasa, de Vera) filed a petition with the med-arbitration unit of dole to nullify the election on the
ground that it was not free, orderly and peaceful. Ong et al, Daya et al and Jimenez et al were eventually
consolidated.

MED ARBITER RULING: Med Arbiter reversed the disqualification imposed by the COMELEC. He said that the
COMELEC accepted all the allegations of petitioners against private respondents Daya, et al., sans evidence to
substantiate the same. Also, the COMELEC erred in putting forth the the basis for the disqualification of Daya et
al as the quoted provision in the CBL applies to dismissal/expulsion of members and not to disqualification of
candidates. Petitioners appealed to the BLR Director. The Med-Arbiter also defended his jurisdiction over the
case. He concluded that even as the election of union officers is an internal affair of the union, his office has the
right to inquire into the merits and conduct of the election when its jurisdiction is sought.

BLR RULING: BLR affirmed decision of Med-Arbiter. He also ruled that the case was an exception to the rule
on exhaustion of administrative remedy, they were left with no choice but to seek the intervention of the BLR
which was declared to have jurisdiction over intra union disputes even at its own initiative under Art. 226.

CA RULING: The CA upheld the decision of the BLR.


In filing the instant petition before the SC, petitioners question the jurisdiction of the BLR on the case at
bar because of the failure of private respondents Daya, et al.,to exhaust administrative remedies within the
union. It is the stance of petitioner that Article
226http://sc.judiciary.gov.ph/jurisprudence/2007/july2007/168475.htm - _ftn31 of the Labor Code which grants
power to the BLR to resolve inter-union and intra-union disputes is dead law, and has been amended by Section
14 of Republic Act No. 6715, whereby the conciliation, mediation and voluntary arbitration functions of the BLR
had been transferred to the National Conciliation and Mediation Board.

ISSUES:
1. Does the BLR have jurisdiction to resolve inter-union and intra-union disputes as provided under Article
226 of the Labor Code despite the supposed amendment by RA 6715?
2. Did the respondents prematurely seek the jurisdiction of the BLR?

San Beda College of Law 333


4S: 2015 - 2016
LABOR LAW REVIEW Atty. Joyrich Golangco

SC RULING:
(1) Yes. The amendment to Art 226 simply reads: The bureau sall have 15 days to act on labor cases before it,
subject to extension by agreement of the parties.

BLR has original jurisdiction on all inter-union and intra-union conflicts. Since Art. 226 declared the BLR shall
have original and exclusive authority to act on all inter-union and intra-union conflicts, there should be no doubt
as to its jurisdiction. Intra-union conflict refers to a conflict within or inside a labor union. An inter-union
controversy or dispute is one occurring or carried on between or among unions.

Rule 1 of Rules implementing book V: intra union dispute- refers to any conflict between and among union
members, and includes all disputes or grievances arising from any violation of or disagreement over any
provision of the constitution and by-laws of a union, including cases arising from chartering or affiliation of labor
organizations or from any vi0lation of the rights and conditions of union membership provided for in the code.

The controversy in the case at bar is an intra-union dispute. Even as the dispute involved allegations that Daya
et al sought the help of non-union members, the same does not detract from the real character of the
controversy. It remains as one which involves the grievance over the constitution and bylaws of a union and it is
a controversy involving members of the union. Moreover, the non-members of the union are not parties to the
case. BLR was properly within its cognizance, it being an intra-union dispute. Daya et al brought the case to the
BLR, it was an invocation of the power and authority of the BLR to act in an intra-union conflict.

(2) No. We affirm the findings of the Court of Appeals which upheld the application by the BLR Director of the
exception to the rule of exhaustion of administrative remedies. Before a party is allowed to seek the intervention
of the court, it is a pre-condition that he should have availed of all the means of administrative processes
afforded him. Hence, if a remedy within the administrative machinery can still be resorted to by giving the
administrative officer concerned every opportunity to decide on a matter that comes within his jurisdiction when
such remedy should be exhausted first before the courts judicial power can be sought. The premature
invocation of courts judicial intervention is fatal to ones cause of action.

Verily, there are exceptions to the applicability of the doctrine. Among the established exceptions are: 1) when
the question raised is purely legal; 2) when the administrative body is in estoppel; 3) when the act complained of
is patently illegal; 4) when there is urgent need for judicial intervention; 5) when the claim involved is small; 6)
when irreparable damage will be suffered; 7) when there is no other plain, speedy, and adequate remedy; 8)
when strong public interest is involved; 9) when the subject of the proceeding is private land; 10) in quo
warranto proceedings; and 11) where the facts show that there was a violation of due process. As aptly
determined by the BLR Director, private respondents Daya, et al., were prejudiced by the disqualification order
of the COMELEC. They endeavored to seek reconsideration, but the COMELEC failed to act thereon. The
COMELEC was also found to have refused to receive their written protest. The foregoing facts sustain the
finding that private respondents Daya, et al., were deprived of due process. Hence, it becomes incumbent upon
private respondents Daya, et al., to seek the aid of the BLR. To insist on the contrary is to render their
exhaustion of remedies within the union as illusory and vain. These antecedent circumstances convince this
Court that there was proper application by the Med-Arbiter of the exception to the rule of exhaustion of
administrative remedies, as affirmed by the BLR Director, and upheld by the Court of Appeals.

San Beda College of Law 334


4S: 2015 - 2016
LABOR LAW REVIEW Atty. Joyrich Golangco

ARTICLE 227 (now Art. 233) Compromise Agreements

167. MAGBANUA v. UY
G.R. No.161003 May 6, 2005
PANGANIBAN, J.

DOCTRINE:
The presence or the absence of counsel when a waiver is executed does not determine its validity. The test is
whether it was executed voluntarily, freely and intelligently; and whether the consideration for it was credible and
reasonable. The labor arbiters absence when the waivers were executed was remedied upon compliance with
the NLRC Rules of Procedure. The Court observes that the arbiter made searching questions during the pre-
execution conference to ascertain whether petitioners had voluntarily and freely executed the waivers.

FACTS:
In the case of Uy vs. NLRC the SC awarded Php. 1,487,312.69 to the 8 complainants therein as the amount of
wage differentials due them. Respondent Uy filed a manifestation requesting the case to be terminated stating
that the judgment award has been complied with to the satisfaction of petitioners. The manifestation was signed
by the 8 petitioners and was accompanied by a joint affidavit attesting to the receipt of payment and waiving all
other benefits due them in connection with their complaint.
Subsequently, petitioners filed an urgent motion for issuance of writ of execution, alleging that they
received only partial payments of the judgment award. Respondent claimed that the award was fully satisfied.
Six (6) of the eight (8) petitioners attested that they have no more collectible amount from respondent and if
there is any, they are abandoning and waiving it.

LA RULING: The LA denied the motion for issuance of write of execution.

NLRC RULING: The NLRC directed the issuance of a writ of execution holding that a final and executor
judgment can no longer be altered.

CA RULING: The CA held that compromise agreements may be entered into even after final judgment, thus
petitioners validly released respondent upon execution of the waiver pursuant to the compromise agreement.

ISSUE: Is the petitioners affidavit waiving the awards in the labor case executed without assistance of their
counsel and the labor arbiter valid?

SC RULING:
Yes. A compromise agreement is a contract whereby the parties make reciprocal concessions in order to
resolve their differences and thus avoid or put an end to a
lawsuit.http://sc.judiciary.gov.ph/jurisprudence/2005/may2005/161003.htm - _ftn11 They adjust their difficulties
in the manner they have agreed upon, disregarding the possible gain in litigation and keeping in mind that such
gain is balanced by the danger of losing. Verily, the compromise may be either extrajudicial (to prevent litigation)
or judicial (to end a litigation).

There is no justification to disallow a compromise agreement, solely because it was entered into after final
judgment. The validity of the agreement is determined by compliance with the requisites and principles of
contracts, not by when it was entered into. As provided by the law on contracts, a valid compromise must have
the following elements: (1) the consent of the parties to the compromise, (2) an object certain that is the subject
matter of the compromise, and (3) the cause of the obligation that is established.

In the present factual milieu, compliance with the elements of a valid contract is not in issue. Petitioners do not
challenge the factual finding that they entered into a compromise agreement with respondent. There are no
allegations of vitiated consent. Neither was there any proof that the agreement was defective or could be
characterized as rescissible, voidable, unenforceable, or void. Instead, petitioners base their argument on the
sole fact that the agreement was executed despite a final judgment, which the Court had previously ruled to be
allowed by law.

As regards the validity of the waiver, the presence or the absence of counsel when a waiver is executed does
not determine its validity. The test is whether it was executed voluntarily, freely and intelligently; and whether

San Beda College of Law 335


4S: 2015 - 2016
LABOR LAW REVIEW Atty. Joyrich Golangco

the consideration for it was credible and reasonable. Petitioners failed to present any evidence to show that their
consent had been vitiated.

The law is silent with regard to the procedure for approving a waiver after a case has been terminated.
Relevant, however, is this reference to the NLRCs New Rules of Procedure:

Should the parties arrive at any agreement as to the whole or any part of the dispute, the same shall be
reduced to writing and signed by the parties and their respective counsel, or authorized representative, if
any, before the Labor Arbiter.

The settlement shall be approved by the Labor Arbiter after being satisfied that it was voluntarily entered
into by the parties and after having explained to them the terms and consequences thereof.

A compromise agreement entered into by the parties not in the presence of the Labor Arbiter before whom
the case is pending shall be approved by him, if after confronting the parties, particularly the complainants,
he is satisfied that they understand the terms and conditions of the settlement and that it was entered into
freely and voluntarily by them and the agreement is not contrary to law, morals, and public policy.

This provision refers to proceedings in a mandatory/conciliation conference during the initial stage of the
litigation. Such provision should be made applicable to the proceedings in the pre-execution conference, for
which the procedure for approving a waiver after final judgment is not stated. There is no reason to make a
distinction between the proceedings in mandatory/conciliation and those in pre-execution conferences.

The labor arbiters absence when the waivers were executed was remedied upon compliance with the above
procedure. The Court observes that the arbiter made searching questions during the pre-execution conference
[52]
to ascertain whether petitioners had voluntarily and freely executed the waivers. Likewise, there was evidence
that they made an intelligent choice, considering that the contents of the written waivers had been explained to
them.http://sc.judiciary.gov.ph/jurisprudence/2005/may2005/161003.htm - _ftn53 The labor arbiters absence
when those waivers were executed does not, therefore, invalidate them.

San Beda College of Law 336


4S: 2015 - 2016
LABOR LAW REVIEW Atty. Joyrich Golangco

168. SOLOMON et al v. POWERTECH CORPORATION, WILLIE CABOBOS and COURT OF APPEALS
G.R. No. 150861 January 22, 2008
REYES, R.T., J.:

DOCTRINE:
Collusion is a species of fraud. Article 227, LC empowers the NLRC to void a compromise agreement for fraud.

FACTS:
A complaint for illegal dismissal was filed by Nagkakaisang Manggagawa Ng Powertech Corporation in behalf of
its 52 individual members and non-union members against their employer, Powertech. The Labor Arbiter
rendered a decision in favor of the employees awarding monetary claims in the total amount of P2,538,728.84.

Powertech appealed to the NLRC. During its pendency, Carlos Gestiada, for himself and on behalf of other
4
petitioners, executed a quitclaim, release and waiver in favor of Powertech in consideration of the amount of
P150,000.00. Earlier, Gestiada was appointed by his co-petitioners as their attorney-in-fact through a SPA.

Relying on the quitclaim and release, Powertech filed a motion for the withdrawal of the appeal and cash bond.
6
The NLRC granted the motion, dismissed the appeal and ordered the release of the cash bond. The
P150,000.00 check, however, bounced due to a stop payment order of Powertech. Aggrieved, petitioners
moved to nullify the release and quitclaim for lack of consideration. In a Resolution the NLRC declared the
quitclaim void for lack of consideration and reinstated the appeal.

Gestiada then terminated the services of their counsel, Atty. Evangelista and, instead, retained Atty. Manuel
Luis Felipe of the Public Attorneys Office. A day later, Powertech paid P150,000.00 to Gestiada purportedly as
compromise amount for all of petitioners. That same day, Gestiada, through Atty. Felipe, and Powertech filed a
10 11
joint motion to dismiss with the NLRC based on the compromise agreement. Atty. Evangelista opposed the
motion, alleging that the compromise agreement is unconscionable and that the P150,000.00 was received by
Gestiada as payment solely for his backwages and other monetary claims.

Petitioners assert that the P150,000.00 paid to Gestiada was payment solely for himself. As proof, they rely on
20
the letter written in Filipino by Gestiada to Atty. Evangelista dated March 23, 2000. Right at the opening
sentence, Gestiada stated that "ang kinuha kong pera sa Powertech Corporation na halagang P150,000.00 ay
bilang kabayaran sa aking backwages na iginawad sa desisyon ni Kagalang-galang Labor Arbiter Joseph
Rennel Dela Cruz."

Powertech, on the other hand, argues that the P150,000.00 was given to Gestiada as compromise amount for
all the petitioners. It relies on the release and quitclaim signed by Gestiada indicating that he signed "for himself
and attorney-in-fact of all complainants." It is pointed out that Gestiada was given a special power of attorney to
negotiate with Powertech on behalf of petitioners.

NLRC: denied the joint motion to dismiss

CA: reversed hence dismissed the case

ISSUE: Is the compromise agreement entered into by Gestiada on behalf petitioners valid?

SC RULING:
No, it is not valid. If reliance is placed solely on the quitclaim release and waiver executed by Gestiada and the
special power of attorney, it would be an inevitable conclusion that the P150,000.00 compromise covered the
claims of petitioners, not merely that of Gestiada. That is apparent from the waiver and the special power of
attorney. There is much to be said, however, of the circumstances in the execution and the payment of the
amount which lead Us to conclude that the P150,000.00 was given to Gestiada solely as payment for his
backwages and other monetary claims.

First, the P150,000 compromise is rather measly when taken in light of the more than P2.5 million judgment on
appeal to the NLRC. Petitioners already won on the arbiter level P2.5 million pesos. It is highly improbable that
they would suddenly agree to accept P150,000 as compromise for the P2.5 million. That translates to a paltry
sum of P6,000.00 each for petitioners. From this amount will still be deducted attorneys fees and other litigation
expenses. In effect, petitioners agreed to waive more than 94% of what they expect to receive from Powertech.
San Beda College of Law 337
4S: 2015 - 2016
LABOR LAW REVIEW Atty. Joyrich Golangco

Second, even granting for the mere sake of argument that the P150,000 was a fair and reasonable compromise
for all, petitioners failed to receive a single centavo from the compromise. This conclusively indicates that
Gestiada received the P150,000 in payment of his backwages and no other.

Third, We give credence to the admission of Gestiada that he received the P150,000.00 as payment for his own
backwages. In his letter to Atty. Evangelista, Gestiada said that he was pressured by Powertech to sign the
waiver and quitclaim for petitioners in order to receive his share in the P2.5 million judgment. Having no stable
job after his dismissal, Gestiada had no other choice but to breach his fiduciary obligation to petitioners. He
succumbed to the pressure of Powertech in signing the waiver, release and quitclaim in exchange for the
P150,000.00. In short, he colluded with Powertech to the detriment of petitioners.

Fourth, the events that led to the execution of the compromise agreement show that Powertech was negotiating
in bad faith. More importantly, they show that Powertech colluded with Gestiada to defraud petitioners of their
share of the P2.5 million Labor Arbiter judgment.

To give effect to the collusion, Gestiada had to get rid of Atty. Evangelista, who had previously succeeded in
nullifying the compromise agreement. He fired Atty. Evangelista without cause basing his dismissal on his
plenary authority as agent of petitioners. He then procured the services of another lawyer, Atty. Felipe. We find it
striking that Gestiada was not authorized under the special power of attorney to terminate or retain another
counsel for petitioners in the labor dispute. The special power of attorney merely authorized Gestiada to
negotiate with Powertech, nothing more.

In his letter, Gestiada admitted that the dismissal of Atty. Evangelista was upon the prodding of Virtue
Sarmiento, personnel manager of Powertech. Powertech imposed the dismissal of Atty. Evangelista as a
condition before Gestiada may receive the amount. A day after firing Atty. Evangelista, Gestiada received the
P150,000.00. That same day, Gestiada, represented by Atty. Felipe, and Powertech filed a joint motion to
dismiss with the NLRC.

All these circumstances indicate that the P150,000.00 was received by Gestiada solely as payment for his
backwages and not a whit of a settlement for the monetary claim of petitioners.
27
Collusion is a species of fraud. Article 227 of the Labor Code empowers the NLRC to void a compromise
agreement for fraud, thus:

Any compromise settlement, including those involving labor standard laws, voluntarily agreed upon by the
parties with the assistance of the Bureau or the regional office of the Department of Labor, shall be final and
binding upon the parties. The National Labor Relations Commission or any court shall not assume jurisdiction
over issues involved therein except in case of non-compliance thereof or if there is prima facie evidence that the
settlement was obtained through fraud, misrepresentation, or coercion.

San Beda College of Law 338


4S: 2015 - 2016
LABOR LAW REVIEW Atty. Joyrich Golangco

169. PHILIPPINE JOURNALISTS, INC., ET. AL. v. NATIONAL LABOR RELATIONS COMMISSION, HON.
COMMS. LOURDES JAVIER, TITO GENILO and ERNESTO VERCELES, JOURNAL EMPLOYEES UNION,
and THE COURT OF APPEALS
G.R. No. 166421 September 5, 2006
CALLEJO, SR., J.:

DOCTRINE:
When judgment is rendered based on a compromise agreement, the judgment becomes immediately executory,
there being an implied waiver of the parties' right to appeal from the decision. The judgment having become
final, the Court can no longer reverse, much less modify it.

FACTS:
Union filed a notice of strike before the National Conciliation and Mediation Board claiming that PJI was guilty of
unfair labor practice. PJI was then going to implement a retrenchment program due to "over-staffing or bloated
work force and continuing actual losses sustained by the company for the past three years resulting in negative
4
stockholders equity of P127.0 million." The Secretary of the Department of Labor and Employment certified the
labor dispute to the National Labor Relations Commission for compulsory arbitration. NLRC declared that the 31
complainants were illegally dismissed and that there was no basis for the implementation of petitioner's
retrenchment program. eclared that the retrenchment of 31 employees was illegal and ordered their
reinstatement "to their former position without loss of seniority rights and other benefits. The parties executed a
Compromise Agreement where PJI undertook to reinstate the 31 complainant-employees without loss of
seniority rights and benefits; 17 of them who were previously retrenched were agreed to be given full and
complete payment of their respective monetary claims, while 14 others would be paid their monetary claims
minus what they received by way of separation pay. The agreement stated that the parties entered the
agreement in a sincere effort at peace and reconciliation as well as to jointly establish a new era in labor
management relations. The compromise agreement was submitted to the NLRC for approval. In the meantime,
however, the Union filed another Notice of Strike. The Union claimed that 29 employees were illegally dismissed
14 15
from employment, and that the salaries and benefits of 50 others had been illegally reduced. After the
retrenchment program was implemented, 200 Union members-employees who continued working for petitioner
had been made to sign five-month contracts. The Union also alleged that the company, through its legal officer,
threatened to dismiss some 200 union members from employment if they refused to conform to a 40% to 50%
salary reduction; indeed, the 29 employees who refused to accede to these demands were dismissed.

NLRC RULING: It ruled that the complainants were not illegally dismissed. The May 31, 2001 Resolution
declaring the retrenchment program illegal did not attain finality as "it had been academically mooted by the
compromise agreement entered into between both parties. Pursuant to Article 223 of the Labor Code, this later
resolution attained finality upon the expiration of ten days from both parties' receipt thereof. Thus, the May 31,
2001 Resolution could not be made the basis to justify the alleged continued employment regularity of the 29
complainants subsequent to their retrenchment. Their separate acts of entering into fixed-term employment
contracts with petitioner after their separation from employment by virtue of retrenchment, they are deemed to
have admitted the validity of their separation from employment and are thus estopped from questioning it.
Moreover, there was no showing that the complainants were forced or pressured into signing the fixed-term
employment contracts which they entered into.

CA RULING: It further held that the act of respondent in hiring the retrenched employees as contractual workers
was a ploy to circumvent the latter's security of tenure. This is evidenced by the admission of PJI, that it hired
contractual employees (majority of whom were those retrenched) because of increased, albeit uncertain,
demand for its publications.

ISSUE: Does the NLRC Resolution, which includes a pronouncement that the members of a union had been
illegally dismissed, is abandoned or rendered "moot and academic" by a compromise agreement subsequently
entered into between the dismissed employees and the employer; this, in turn, raises the question of whether
such a compromise agreement constitutes res judicata to a new complaint later filed by other union members-
employees, not parties to the agreement, who likewise claim to have been illegally dismissed?

SC RULING:
The nature of a compromise is spelled out in Article 2028 of the New Civil Code: it is "a contract whereby the
parties, by making reciprocal concessions, avoid litigation or put an end to one already commenced." Parties to
26
a compromise are motivated by "the hope of gaining, balanced by the dangers of losing." It contemplates
San Beda College of Law 339
4S: 2015 - 2016
LABOR LAW REVIEW Atty. Joyrich Golangco

mutual concessions and mutual gains to avoid the expenses of litigation, or, when litigation has already begun,
27
to end it because of the uncertainty of the result. Article 227 of the Labor Code of the Philippines authorizes
compromise agreements voluntarily agreed upon by the parties, in conformity with the basic policy of the State
"to promote and emphasize the primacy of free collective bargaining and negotiations, including voluntary
arbitration, mediation and conciliation, as modes of settling labor or industrial disputes." Thus, contrary to the
allegation of petitioners, the execution and subsequent approval by the NLRC of the agreement forged between
it and the respondent Union did not render the NLRC resolution ineffectual, nor rendered it "moot and
academic." The agreement becomes part of the judgment of the court or tribunal, and as a logical consequence,
there is an implicit waiver of the right to appeal.

In any event, the compromise agreement cannot bind a party who did not voluntarily take part in the settlement
itself and gave specific individual consent. It must be remembered that a compromise agreement is also a
contract; it requires the consent of the parties, and it is only then that the agreement may be considered as
voluntarily entered into.

San Beda College of Law 340


4S: 2015 - 2016
LABOR LAW REVIEW Atty. Joyrich Golangco

ARTICLE 232 (now Art. 238) Prohibition on Certification Election

170. COLEGIO DE SAN JUAN DE LETRAN v. ASSOCIATION OF EMPLOYEES AND FACULTY OF


LETRAN and ELEONOR AMBAS
G.R. No. 141471 September 18, 2000
KAPUNAN, J.

DOCTRINE:
If a collective bargaining agreement has been duly registered in accordance with Article 231 of the Code, a
petition for certification election or a motion for intervention can only be entertained within sixty (60) days prior to
the expiry date of such agreement. No petition for certification election for any representation issue may be filed
after the lapse of the sixty-day freedom period. The old CBA is extended until a new one is signed. The rule is
that despite the lapse of the formal effectivity of the CBA the law still considers the same as continuing in force
and effect until a new CBA shall have been validly executed. Hence, the contract bar rule still applies.

Management has the prerogative to discipline its employees for insubordination. But when the exercise of such
management right tends to interfere with the employees' right to self-organization, it amounts to union-busting
and is therefore a prohibited act.

FACTS:
1992, Salvador Abtria, then President of respondent union, Association of Employees and Faculty of Letran,
initiated the renegotiation of its Collective Bargaining Agreement with petitioner Colegio de San Juan de Letran
for the last two (2) years of the CBA's five (5) year lifetime from 1989-1994. On the same year, the union elected
a new set of officers wherein private respondent Eleanor Ambas emerged as the newly elected President.
Ambas wanted to continue the renegotiation of the CBA but petitioner, through Fr. Edwin Lao, claimed that the
CBA was already prepared for signing by the parties. The parties submitted the disputed CBA to a referendum
by the union members, who eventually rejected the said CBA.

Petitioner accused the union officers of bargaining in bad faith before the National Labor Relations Commission
(NLRC). Labor Arbiter Edgardo M. Madriaga decided in favor of petitioner. However, the Labor Arbiter's decision
was reversed on appeal before the NLRC.

the union notified the National Conciliation and Mediation Board (NCMB) of its intention to strike on the of
refusal to bargain by petitioner.

On January 18, 1996, the parties agreed to disregard the unsigned CBA and to start negotiation on a new five-
year CBA starting 1994-1999. On February 7, 1996, the union submitted its proposals to petitioner, which
notified the union six days later or on February 13, 1996 that the same had been submitted to its Board of
Trustees. In the meantime, Ambas was informed through a letter dated February 15, 1996 from her superior that
her work schedule was being changed from Monday to Friday to Tuesday to Saturday. Ambas protested and
requested management to submit the issue to a grievance machinery under the old CBA.
Due to petitioner's inaction, the union filed a notice of strike on March 13, 1996. The parties met on March 27,
1996 before the NCMB to discuss the ground rules for the negotiation. On March 29, 1996, the union received
petitioner's letter dismissing Ambas for alleged insubordination. Hence, the union amended its notice of strike to
include Ambas' dismissal.

On April 20, 1996, both parties again discussed the ground rules for the CBA renegotiation. However, petitioner
stopped the negotiations after it purportedly received information that a new group of employees had filed a
petition for certification election.

ISSUES:
1. Is petitioner guilty of unfair labor practice by refusing to bargain with the union when it unilaterally
suspended the ongoing negotiations for a new Collective Bargaining Agreement (CBA) upon mere
information that a petition for certification has been filed by another legitimate labor organization?

2. Is the termination of the union president amounts to an interference of the employees' right to self-
organization?

SC RULING:
San Beda College of Law 341
4S: 2015 - 2016
LABOR LAW REVIEW Atty. Joyrich Golangco

1. YES. Art. 252. Meaning of duty to bargain collectively. - The duty to bargain collectively means the
performance of a mutual obligation to meet and convene promptly and expeditiously in good faith for the
purpose of negotiating an agreement with respect to wages, hours of work and all other terms and
conditions of employment including proposals for adjusting any grievances or questions arising under
such agreement and executing a contract incorporating such agreements if requested by either party but
such duty does not compel any party to agree to a proposal or to make any concession.

Noteworthy in the above definition is the requirement on both parties of the performance of the mutual
obligation to meet and convene promptly and expeditiously in good faith for the purpose of negotiating an
agreement. Undoubtedly, respondent Association of Employees and Faculty of Letran (AEFL)
(hereinafter, "union") lived up to this requisite when it presented its proposals for the CBA to petitioner on
February 7, 1996. On the other hand, petitioner devised ways and means in order to prevent the
negotiation.

Petitioner's utter lack of interest in bargaining with the union is obvious in its failure to make a timely reply
to the proposals presented by the latter. More than a month after the proposals were submitted by the
union, petitioner still had not made any counter-proposals. This inaction on the part of petitioner prompted
the union to file its second notice of strike on March 13, 1996. Petitioner could only offer a feeble
explanation that the Board of Trustees had not yet convened to discuss the matter as its excuse for failing
to file its reply.

The company's refusal to make counter-proposal to the union's proposed CBA is an indication of its bad
faith. Where the employer did not even bother to submit an answer to the bargaining proposals of the
6
union, there is a clear evasion of the duty to bargain collectively. In the case at bar, petitioner's actuation
show a lack of sincere desire to negotiate rendering it guilty of unfair labor practice.

In order to allow the employer to validly suspend the bargaining process there must be a valid petition for
certification election raising a legitimate representation issue. Hence, the mere filing of a petition for
certification election does not ipso facto justify the suspension of negotiation by the employer. The petition
must first comply with the provisions of the Labor Code and its Implementing Rules. Foremost is that a
petition for certification election must be filed during the sixty-day freedom period. The "Contract Bar Rule"
under Section 3, Rule XI, Book V, of the Omnibus Rules Implementing the Labor Code, provides that: "
. If a collective bargaining agreement has been duly registered in accordance with Article 231 of the
Code, a petition for certification election or a motion for intervention can only be entertained within sixty
8
(60) days prior to the expiry date of such agreement." The rule is based on Article 232, in relation to
Articles 253, 253-A and 256 of the Labor Code. No petition for certification election for any representation
issue may be filed after the lapse of the sixty-day freedom period. The old CBA is extended until a new
one is signed. The rule is that despite the lapse of the formal effectivity of the CBA the law still considers
9
the same as continuing in force and effect until a new CBA shall have been validly executed. Hence, the
10
contract bar rule still applies. The purpose is to ensure stability in the relationship of the workers and the
company by preventing frequent modifications of any CBA earlier entered into by them in good faith and
11
for the stipulated original period.
In the case at bar, the lifetime of the previous CBA was from 1989-1994.1wphi1 The petition for
certification election by ACEC, allegedly a legitimate labor organization, was filed with the Department of
Labor and Employment (DOLE) only on May 26, 1996. Clearly, the petition was filed outside the sixty-day
freedom period. Hence, the filing thereof was barred by the existence of a valid and existing collective
bargaining agreement. Consequently, there is no legitimate representation issue and, as such, the filing of
the petition for certification election did not constitute a bar to the ongoing negotiation.

2. The factual backdrop of the termination of Ms. Ambas leads us to no other conclusion that she was
dismissed in order to strip the union of a leader who would fight for the right of her co-workers at the
bargaining table. Ms. Ambas, at the time of her dismissal, had been working for the petitioner for ten (10)
years already. In fact, she was a recipient of a loyalty award. Moreover, for the past ten (10) years her
working schedule was from Monday to Friday. However, things began to change when she was elected as
union president and when she started negotiating for a new CBA. Thus, it was when she was the union
president and during the period of tense and difficult negotiations when her work schedule was altered
from Mondays to Fridays to Tuesdays to Saturdays. When she did not budge, although her schedule was
changed, she was outrightly dismissed for alleged insubordination.

San Beda College of Law 342


4S: 2015 - 2016
LABOR LAW REVIEW Atty. Joyrich Golangco

Admittedly, management has the prerogative to discipline its employees for insubordination. But when the
exercise of such management right tends to interfere with the employees' right to self-organization, it
amounts to union-busting and is therefore a prohibited act. The dismissal of Ms. Ambas was clearly
designed to frustrate the Union in its desire to forge a new CBA with the College that is reflective of the
true wishes and aspirations of the Union members. Her dismissal was merely a subterfuge to get rid of
her, which smacks of a pre-conceived plan to oust her from the premises of the College. It has the effect
of busting the Union, stripping it of its strong-willed leadership. When management refused to treat the
charge of insubordination as a grievance within the scope of the Grievance Machinery, the action of the
College in finally dismissing her from the service became arbitrary, capricious and whimsical, and
therefore violated Ms. Ambas' right to due process."

San Beda College of Law 343


4S: 2015 - 2016
LABOR LAW REVIEW Atty. Joyrich Golangco

ARTICLE 234 (now Art. 240) Requirements of Registration

171. MARIWASA SIAM CERAMICS v. SEC. OF LABOR


G.R. No. 183317 December 21, 2009
NACHURA, J.:

DOCTRINE:
In case the applicant is an independent union, the names of all its members comprising at least twenty percent
(20%) of all the employees in the bargaining unit where it seeks to operate is one of the requirements of
registration of a labor organization.

FACTS:
Samahan Ng Mga Manggagawa Sa Mariwasa Siam Ceramics, Inc was issued a Certificate of Registration as a
legitimate labor organization. Mariwasa Siam Ceramics, Inc. filed a Petition for Cancellation of Union
Registration against respondent, claiming that the latter violated Article 234 of the Labor Code for not complying
with the 20% requirement, and that it committed massive fraud and misrepresentation in violation of Article 239.
The petitioner insists that respondent failed to comply with the 20% union membership requirement for its
registration as a legitimate labor organization because of the disaffiliation from the total number of union
members of 102 employees who executed affidavits recanting their union membership. Respondent asserts that
it had a total of 173 union members at the time it applied for registration.

REGIONAL DIRECTOR: revoked the registration of respondent

BLE DIRECTOR: reversed and set aside the regional directors decision

CA: denied the petition for lack of merit

ISSUE: Are the affidavits of recantation valid?

SC RULING:
No. It is worthy to note, however, that the affidavit does not mention the identity of the people who allegedly
forced and deceived the affiant into joining the union, much less the circumstances that constituted such force
and deceit. Indeed, not only was this allegation couched in very general terms and sweeping in nature, but more
importantly, it was not supported by any evidence whatsoever. In the instant case, the affidavits of recantation
were executed after the identities of the union members became public, i.e., after the union filed a petition for
certification election on May 23, 2005, since the names of the members were attached to the petition. The
purported withdrawal of support for the registration of the union was made after the documents were submitted
to the DOLE, Region IV-A. The logical conclusion, therefore, following jurisprudence, is that the employees were
not totally free from the employers pressure, and so the voluntariness of the employees execution of the
affidavits becomes suspect.

San Beda College of Law 344


4S: 2015 - 2016
LABOR LAW REVIEW Atty. Joyrich Golangco

172. ELECTROMAT MANUFACTURING and RECORDING CORPORATION, v. HON. CIRIACO LAGUNZAD
G.R. No. 172699 July 27, 2011
BRION, J.:

REQUIREMENTS FOR REGISTRATION OF LABOR UNIONS AS SUPPLEMENTED BY D.O 40-03

FACTS:
Nagkakaisang Samahan ng Manggagawa ng Electromat-Wasto, a charter affiliate of the Workers Advocates for
Struggle, Transformation and Organization applied for registration with the Bureau of Labor Relations.
Supporting the application were the following documents: (1) copies of its ratified constitution and by-laws
(CBL); (2) minutes of the CBLs adoption and ratification; (3) minutes of the organizational meetings; (4) names
and addresses of the union officers; (5) list of union members; (6) list of rank-and-file employees in the
company; (7) certification of non-existence of a collective bargaining agreement (CBA) in the company; (8)
resolution of affiliation with WASTO, a labor federation; (9) WASTOs resolution of acceptance; (10) Charter
Certificate; and (11) Verification under oath. The BLR thereafter issued the union a Certification of Creation of
Local Chapter pursuant to DO 40-03. Petitioner Electromat Manufacturing and Recording Corporation
(company) filed a petition for cancellation of the unions registration certificate, for the unions failure to comply
with Article 234 of the Labor Code. It argued that D.O. 40-03 is an unconstitutional diminution of the Labor
Codes union registration requirements under Article 234.

DOLE: dismissed the petition

CA RULING: dismissed the petition and affirmed the assailed BLR ruling. It brushed aside the companys
objection to D.O. 40-03, and its submission that D.O. 40-03 removed the safety measures against the
commission of fraud in the registration of unions

ISSUE: Is the DO 40-03 a valid exercise of the rule-making power of the DOLE

SC RULING:
Yes. Undoubtedly, the intent of the law in imposing lesser requirements in the case of a branch or local of a
registered federation or national union is to encourage the affiliation of a local union with a federation or national
union in order to increase the local unions bargaining powers respecting terms and conditions of labor. D.O. 40-
03 represents an expression of the governments implementing policy on trade unionism. It builds upon the old
rules by further simplifying the requirements for the establishment of locals or chapters. As in D.O. 9, we see
nothing contrary to the law or the Constitution in the adoption by the Secretary of Labor and Employment of
D.O. 40-03 as this department order is consistent with the intent of the government to encourage the affiliation
of a local union with a federation or national union to enhance the locals bargaining power. If changes were
made at all, these were those made to recognize the distinctions made in the law itself between federations and
their local chapters, and independent unions; local chapters seemingly have lesser requirements because they
and their members are deemed to be direct members of the federation to which they are affiliated, which
federations are the ones subject to the strict registration requirements of the law.

San Beda College of Law 345


4S: 2015 - 2016
LABOR LAW REVIEW Atty. Joyrich Golangco

173. EAGLE RIDGE GOLF & COUNTRY CLUB vs. COURT OF APPEALS and EAGLE RIDGE EMPLOYEES
UNION (ER EU)
G.R No. 178989 March 18, 2010
VELASCO, JR., J:

GROUNDS FOR CANCELLATION OF UNION REGISTRATION

DOCTRINE:
We have in precedent cases said that the employees' withdrawal from a labor union made before the filing of the
petition for certification election is presumed voluntary, while withdrawal after the filing of such petition is
considered to be involuntary and does not affect the same. Now then, if a withdrawal from union membership
done after a petition for certification election has been filed does not vitiate such petition, is it not but logical to
assume that such withdrawal cannot work to nullify the registration of the union?

FACTS:
Petitioner Eagle Ridge is a corporation engaged in the business of maintaining golf courses. It had, at the end of
CY 2005, around 112 rank-and-file employees. On December 6, 2005, at least 20% of Eagle Ridges rank-and-
file employeesthe percentage threshold required under Article 234(c) of the Labor Code for union
registrationhad a meeting where they organized themselves into an independent labor union, named "Eagle
Ridge Employees Union" (EREU or Union), elected a set of officers, and ratified their constitution and by-laws.

On December 19, 2005, EREU formally applied for registration before the Department of Labor and
Employment (DOLE) Regional Office IV (RO IV). In time, DOLE RO IV granted the application. The EREU then
filed a petition for certification election in Eagle Ridge Golf & Country Club. Eagle Ridge opposed this petition,
followed by its filing of a petition for the cancellation of the application.

Eagle Ridges petition ascribed misrepresentation, false statement, or fraud to EREU in connection with the
adoption of its constitution and by-laws, the numerical composition of the Union, and the election of its officers.
Petitioner alleged that the EREU declared in its application for registration having 30 members, when the
minutes of its December 6, 2005 organizational meeting showed it only had 26 members. The misrepresentation
was exacerbated by the discrepancy between the certification issued by the Union secretary and president that
25 members actually ratified the constitution and by-laws on December 6, 2005 and the fact that 26 members
affixed their signatures on the documents, making one signature a forgery. Finally, petitioner contended that five
employees who attended the organizational meeting had manifested the desire to withdraw from the union. The
five executed individual affidavits or Sinumpaang Salaysay on February 15, 2006, attesting that they arrived late
at said meeting which they claimed to be drinking spree; that they did not know that the documents they signed
on that occasion pertained to the organization of a union; and that they now wanted to be excluded from the
Union. The withdrawal of the five, Eagle Ridge maintained, effectively reduced the union membership to 20 or
21, either of which is below the mandatory minimum 20% membership requirement under Art. 234(c) of the
Labor Code. Reckoned from
112 rank-and-file employees of Eagle Ridge, the required number would be 22 or 23 employees.

As a counterpoint, EREU alleged that discrepancies are not real for before filing of its application on December
19, 2005, four additional employees joined the union on December 8, 2005, thus raising the union membership
to 30 members as of December 19, 2005; that the understatement by one member who ratified the constitution
and by-laws was a typographical error, which does not make it either grave or malicious warranting the
cancellation of the unions registration; that the retraction of 5 union members should not be given any credence
for the reasons that:
(b) the sworn statements of the five retracting union members sans other affirmative evidence presented
hardly qualify as clear and credible evidence considering the joint affidavits of the other members
attesting to the orderly conduct of the organizational meeting;
(c) the retracting members did not deny signing the union documents;
(d) it can be presumed that "duress, coercion or valuable consideration" was brought to bear on the
retracting members; and
(e) once the required percentage requirement has been reached, the employees withdrawal from union
membership taking place after the filing of the petition for certification election will not affect the petition.

After due proceedings, the DOLE Regional Director, focusing on the question of misrepresentation, issued an
Order finding for Eagle Ridge. Aggrieved, the Union appealed to the BLR, which affirmed the appealed order of
San Beda College of Law 346
4S: 2015 - 2016
LABOR LAW REVIEW Atty. Joyrich Golangco

the DOLE Regional Director. Undeterred by successive set backs, EREU interposed a motion for
reconsideration which was granted. Eagle Ridge sought but was denied reconsideration. Eagle Ridge thereupon
went to the CA, which dismissed the petition for certiorari. The CA later denied Eagle Ridges motion for
reconsideration, hence the recourse with the SC.

ISSUE: Whether there was fraud in the application to merit the cancellation of the EREUs registration

SC RULING:
NO, a scrutiny of the records fails to show any misrepresentation, false statement, or fraud committed by
EREU to merit cancellation of its registration. The Supreme Court succinctly explained this decision in eight
points:

First. The Union submitted the required documents attesting to the facts of the organizational meeting on
December 6, 2005, the election of its officers, and the adoption of the Unions constitution and by-laws.

Second. The members of the EREU totaled 30 employees when it applied on December 19, 2005 for
registration. The Union thereby complied with the mandatory minimum 20% membership requirement under
Art. 234(c). Of note is the undisputed number of 112 rank-and-file employees in Eagle Ridge, as shown in the
Sworn Statement of the Union president and secretary and confirmed by Eagle Ridge in its petition for
cancellation.

Third. The Union has sufficiently explained the discrepancy between the number of those who attended the
organizational meeting showing 26 employees and the list of union members showing 30. The difference is
due to the additional four members admitted two days after the organizational meeting as attested to by their
duly accomplished Union Membership form.

Fourth. In its futile attempt to clutch at straws, Eagle Ridge assails the inclusion of the additional four
members allegedly for not complying with what it termed as "the sine qua non requirements" for union member
applications under the Unions constitution and by-laws, specifically Sec. 2 of Art. IV. We are not persuaded.
Any seeming infirmity in the application and admission of union membership, most especially in cases of
independent labor unions, must be viewed in favor of valid membership.

The right of employees to self-organization and membership in a union must not be trammeled by undue
difficulties. In this case, when the Union said that the four employee-applicants had been admitted as union
members, it is enough to establish the fact of admission of the four that they had duly signified such desire by
accomplishing the membership form. The fact, as pointed out by Eagle Ridge, that the Union, owing to its
scant membership, had not yet fully organized its different committees evidently shows the direct and valid
acceptance of the four employee applicants rather than deter their admission as erroneously asserted by
Eagle Ridge.

Fifth. The difference between the number of 26 members, who ratified the Unions constitution and by-laws,
and the 25 members shown in the certification of the Union secretary as having ratified it, is, as shown by the
factual antecedents, a typographical error. It was an insignificant mistake committed without malice or
prevarication. The list of those who attended the organizational meeting shows 26 members, as evidenced by
the signatures beside their handwritten names.

Sixth. In the more meaty issue of the affidavits of retraction executed by six union members, we hold that the
probative value of these affidavits cannot overcome those of the supporting affidavits of 12 union members
and their counsel as to the proceedings and the conduct of the organizational meeting on December 6, 2005.
The DOLE Regional Director and the BLR OIC Director obviously erred in giving credence to the affidavits of
retraction, but not according the same treatment to the supporting affidavits.

The six affiants of the affidavits of retraction were not presented in a hearing before the Hearing Officer (DOLE
Regional Director), as required under the Rules Implementing Book V of the Labor Code covering Labor
Relation. It is settled that affidavits partake the nature of hearsay evidence, since they are not generally
prepared by the affiant but by another who uses his own language in writing the affiants statement, which may
thus be either omitted or misunderstood by the one writing them. For their non-presentation and consonant to
the above-quoted rule, the six affidavits of retraction are inadmissible as evidence against the Union in the
instant case.
San Beda College of Law 347
4S: 2015 - 2016
LABOR LAW REVIEW Atty. Joyrich Golangco

Seventh. The fact that six union members, indeed, expressed the desire to withdraw their membership
through their affidavits of retraction will not cause the cancellation of registration on the ground of violation of
Art. 234(c) of the Labor Code requiring the mandatory minimum 20% membership of rank-and- file employees
in the employees union.

The six retracting union members clearly severed and withdrew their union membership. The query is whether
such separation from the Union can detrimentally affect the registration of the Union. We answer in the
negative.

Twenty percent (20%) of 112 rank-and-file employees in Eagle Ridge would require a union membership of at
least 22 employees (112 x 205 = 22.4). When the EREU filed its application for registration on December 19,
2005, there were clearly 30 union members. Thus, when the certificate of registration was granted, there is no
dispute that the Union complied with the mandatory 20% membership requirement. With the withdrawal of six
union members, there is still compliance with the mandatory membership requirement under Art. 234(c), for
the remaining 24 union members constitute more than the 20% membership requirement of 22 employees.

Eighth. Finally, it may not be amiss to note, given the factual antecedents of the instant case, that Eagle
Ridge has apparently resorted to filing the instant case for cancellation of the Unions certificate of registration
to bar the holding of a certification election. This can be gleaned from the fact that the grounds it raised in its
opposition to the petition for certification election are basically the same grounds it resorted to in the instant
case for cancellation of EREUs certificate of registration. This amounts to a clear circumvention of the law and
cannot be countenanced.

San Beda College of Law 348


4S: 2015 - 2016
LABOR LAW REVIEW Atty. Joyrich Golangco

Certificate of Registration of Labor Organization

174. TAGAYTAY HIGHLANDS INTERNATIONAL GOLF CLUB INCORPORATED v. TAGAYTAY


HIGHLANDS EMPLOYEES UNION-PGTWO
G.R. No. 142000 January 22, 2003
CARPIO MORALES, J:

GROUNDS FOR CANCELLATION OF UNION REGISTRATION

DOCTRINE:
After a certificate of registration is issued to a union, its legal personality cannot be subject to collateral attack. It
may be questioned only in an independent petition for cancellation in accordance with Section 5 of Rule V, Book
IV of the "Rules to Implement the Labor Code." The grounds for cancellation of union registration are provided
for under Article 239 of the Labor Code. The inclusion in a union of disqualified employees is not among the
grounds for cancellation, unless such inclusion is due to misrepresentation, false statement or fraud under the
circumstances enumerated in Sections (a) and (c) of Article 139 of above-quoted Article 239 of the Labor Code.
THEU, having been validly issued a certificate of registration, should be considered to have already acquired
juridical personality which may not be assailed collaterally. As for petitioner's allegation that some of the
signatures in the petition for certification election were obtained through fraud, false statement and
misrepresentation, the proper procedure is, as reflected above, for it to file a petition for cancellation of the
certificate of registration, and not to intervene in a petition for certification election.

FACTS:
On October 16, 1997, the Tagaytay Highlands Employees Union (THEU) Philippine Transport and General
Workers Organization (PTGWO), Local Chapter No. 776, a legitimate labor organization said to represent
majority of the rank- and-file employees of Tagaytay Highlands International Golf Club Incorporated
(THIGCI), filed a petition for certification election before the DOLE Mediation- Arbitration Unit, Regional
Branch No. IV.

THIGCI, in its Comment, opposed THEUs petition for certification election on the ground that the list of union
members submitted by it was defective and fatally flawed as it included the names and signatures of
supervisors, resigned, terminated and absent without leave (AWOL) employees, as well as employees of The
Country Club, Inc., a corporation distinct and separate from THIGCI; and that out of the 192 signatories to
the petition, only 71 were actual rank-and-file employees of THIGCI. THIGCI thus submitted a list of the
names of its 71 actual rank-and-file employees to the petition for certification election. And it therein
incorporated a tabulation showing the number of signatories to said petition whose membership in the union
was being questioned as disqualified and the reasons for disqualification.

THEU asserted that it complied with all the requirements for valid affiliation and inclusion in the roster of
legitimate labor organizations pursuant to DOLE Department Order No. 9, series of 1997, on account of
which it was duly granted a Certification of Affiliation by DOLE on October 10, 1997; and that Section 5, Rule
V of said Department Order provides that the legitimacy of its registration cannot be subject to collateral
attack, and for as long as there is no final order of cancellation, it continues to enjoy the rights accorded to a
legitimate organization. Therefore, the Med-Arbiter should, pursuant to Article
257 of the Labor Code and Section 11, Rule XI of DOLE Department Order No.
09, automatically order the conduct of a certification election.

On January 28, 1998, DOLE Med-Arbiter Anastacio Bactin ordered the holding of a certification election.

THIGCI appealed to the Office of the DOLE Secretary which, by Resolution of June 4, 1998, set aside the
said Med-Arbiters Order and accordingly dismissed the petition for certification election on the ground that
there is a "clear absence of community or mutuality of interests," it finding that THEU sought to represent two
separate bargaining units (supervisory employees and rank-and- file employees) as well as employees of
two separate and distinct corporate entities.

Upon Motion for Reconsideration by THEU, DOLE Undersecretary Rosalinda Dimalipis-Baldoz, by authority
of the DOLE Secretary, issued DOLE Resolution of November 12, 1998 setting aside the June 4, 1998
Resolution dismissing the petition for certification election. She held that since THEU is a local chapter, the
twenty percent (20%) membership requirement is not necessary for it to acquire legitimate status, hence,
San Beda College of Law 349
4S: 2015 - 2016
LABOR LAW REVIEW Atty. Joyrich Golangco

"the alleged retraction and withdrawal of support by 45 of the 70 remaining rank-and-file members . . . cannot
negate the legitimacy it has already acquired before the petition". THIGCIs Motion for Reconsideration was
denied by the DOLE Undersecretary hence it filed a petition for certiorari with the CA.

The CA denied THIGCIs Petition for Certiorari and affirmed the DOLE Resolution dated November 12, 1998.
It held that while a petition for certification election is an exception to the innocent bystander rule, hence, the
employer may pray for the dismissal of such petition on the basis of lack of mutuality of interests of the
members of the union as well as lack of employer-employee relationship and petitioner failed to adduce
substantial evidence to support its allegations.

ISSUE: Whether the unions legal personality can be subject to collateral attack after a certificate of
registration is issued

RULING:
NO, Petition is DENIED, and the records of the case are remanded to the office of origin.
While above-quoted Article 245 expressly prohibits supervisory employees from joining a rank-and-file union, it
does not provide what would be the effect if a rank-and-file union counts supervisory employees among its
members, or vice-versa.

Citing Toyota which held that "a labor organization composed of both rank-and-file and supervisory employees
is no labor organization at all," and the subsequent case of Progressive Development Corp. Pizza Hut v.
20
Ledesma which held that:

"The Labor Code requires that in organized and unorganized establishments, a petition for certification
election must be filed by a legitimate labor organization. The acquisition of rights by any union or labor
organization, particularly the right to file a petition for certification election, first and foremost,
depends onwhether or not the labor organization has attained the status of a legitimate labor
organization.

In the case before us, the Med-Arbiter summarily disregarded the petitioners prayer that the former look
into the legitimacy of the respondent Union by a sweeping declaration that the union was in the
possession of a charter certificate so that for all intents and purposes, Sumasaklaw sa Manggagawa sa
21
Pizza Hut (was) a legitimate organization," (Underscoring and emphasis supplied).

We also do not agree with the ruling of the respondent Secretary of Labor that the infirmity in the
membership of the respondent union ca n b e re m e d ie d in "the pr e - el ect i on con f er en ce thru
the exclusion-inclusion proceedings wherein those employees who are occupying rank-and-file positions will
be excluded from the list of eligible voters."
After a certificate of registration is issued to a union, its legal personality cannot be subject to collateral
attack. It may be questioned only in an independent petition for cancellation in accordance with Section 5 of
Rule V, Book IV of the "Rules to Implement the Labor Code" (Implementing Rules) which section reads:

Sec. 5. Effect of registration. The labor organization or workers association shall be deemed
registered and vested with legal personality on the date of issuance of its certificate of
registration. Such legal personality cannot thereafter be subject to collateral attack, but may be
questioned only in an independent petition for cancellation in accordance with these Rules.
(Emphasis supplied)
The inclusion in a union of disqualified employees is not among the grounds for cancellation, unless such
inclusion is due to misrepresentation, false statement or fraud under the circumstances enumerated in
Sections (a) and (c) of Article 239 of above-quoted Article 239 of the Labor Code.

THEU, having been validly issued a certificate of registration, should be considered to have already acquired
juridical personality which may not be assailed collaterally.

As for petitioners allegation that some of the signatures in the petition for certification election were obtained
through fraud, false statement and misrepresentation, the proper procedure is, as reflected above, for it to file a
petition for cancellation of the certificate of registration, and not to intervene in a petition for certification election.
Regarding the alleged withdrawal of union members from participating in the certification election, this Courts
following ruling is instructive:
San Beda College of Law 350
4S: 2015 - 2016
LABOR LAW REVIEW Atty. Joyrich Golangco

"[T]he best forum for determining whether there were indeed retractions from some of the laborers is in
thecertification election itself wherein the workers can freely express their choice in a secret ballot.
Suffice it to say that the will of the rank-and-file employees should in every possible instance be
determined by secret ballot rather than by administrative or quasi-judicial inquiry. Such representation
and certification election cases are not to be taken as contentious litigations for suits but as mere
investigations of a non-adversary, fact-finding character as to which of the competing unions represents
the genuine choice of the workers to be their sole and exclusive collective bargaining representative
with their employer."

As for the lack of mutuality of interest argument of petitioner, it, at all events, does not lie given, as found by the
court a quo, its failure to present substantial evidence that the assailed employees are actually occupying
supervisory positions.

While petitioner submitted a list of its employees with their corresponding job titles and ranks, there is nothing
mentioned about the supervisors respective duties, powers and prerogatives that would show that they can
effectively recommend managerial actions which require the use of independent judgment.

As this Court put it in Pepsi-Cola Products Philippines, Inc. v. Secretary of Labor:

Designation should be reconciled with the actual job description of subject employees x x x The mere
fact that an employee is designated manager does not necessarily make him one. Otherwise, there
would be an absurd situation where one can be given the title just to be deprived of the right to be a
member of a union. In the case of National Steel Corporation vs. Laguesma (G. R. No. 103743, January
29, 1996), it was stressed that:

What is essential is the nature of the employees function and not the nomenclature or
titlegiven to the job which determines whether the employee has rank-and-file or managerial
status or whether he is a supervisory employee. (Emphasis supplied).

San Beda College of Law 351


4S: 2015 - 2016
LABOR LAW REVIEW Atty. Joyrich Golangco

175. S.S. VENTURES INTERNATIONAL, INC. v. S.S. VENTURES LABOR UNION (SSVLU) and DIR. HANS
LEO CACDAC, in His capacity as Director of the Bureau of Labor Relations (BLR)
G.R. No. 161690 July 23, 2008
VELASCO, JR., J.:

REVOCATION OF CERTIFICATE OF REGISTRATION

DOCTRINE:
To decertify a union, it is not enough to show that the union includes ineligible employees in its membership.
It must also be shown that there was misrepresentation, false statement, or fraud in connection with the
application for registration and the supporting documents, such as the adoption or ratification of the
constitution and by-laws or amendments thereto and the minutes of ratification of the constitution or by-laws,
among other documents.

FACTS:
On March 21, 2000, the Union filed with DOLE-Region III a petition for certification election in behalf of the
rank-and-file employees of Ventures. 542 signatures, 82 of which belong to terminated Ventures employees,
appeared on the basic documents supporting the petition.

On August 21, 2000, Ventures filed a Petition to cancel the Unions certificate of registration invoking the
grounds set forth in Article 239(a) of the Labor Code. The petition alleged the following:
(1) The Union deliberately and maliciously included the names of more or less 82 former employees no
longer connected with Ventures in its list of members who attended the organizational meeting and in
the adoption/ratification of its constitution and by- laws held on January 9, 2000 in Mariveles, Bataan;
and the Union forged the signatures of these 82 former employees to make it appear they took part in
the organizational meeting and adoption and ratification of the constitution;
(2) The Union maliciously twice entered the signatures of three persons namely: Mara Santos,
Raymond Balangbang, and Karen Agunos;
(3) No organizational meeting and ratification actually took place; and
(4) The Unions application for registration was not supported by at least 20% of the rank-and-file
employees of Ventures, or 418 of the total 2,197-employee complement. Since more or less 82 of the
500 signatures were forged or invalid, then the remaining valid signatures would only be 418,
which is very much short of the 439 minimum (2197 total employees x 20% = 439.4) required by the
Labor Code.

In its Answer, the Union denied committing the imputed acts of fraud or forgery and alleged that: (1) the
organizational meeting actually took place on January 9, 2000 at the Shoe City basketball court in Mariveles;
(2) the 82 employees adverted to in Ventures petition were qualified Union members for, although they have
been ordered dismissed, the one-year prescriptive period to question their dismissal had not yet lapsed; (3) it
had complied with the 20%-member registration requirement since it had 542 members; and (4) the
"double" signatures were inadvertent human error.

In its supplemental reply memorandum Ventures cited other instances of fraud and misrepresentation,
claiming that the "affidavits" executed by 82 alleged Union members show that they were deceived into
signing paper minutes or were harassed to signing their attendance in the organizational meeting. Ventures
added that some employees signed the "affidavits" denying having attended such meeting.

ISSUE: Whether or not the Certification of Registration of the Union should be revoked

RULING:
The right to form, join, or assist a union is specifically protected by Art. XIII, Section 3 of the Constitution and
such right, according to Art. III, Sec. 8 of the Constitution and Art. 246 of the Labor Code, shall not be
abridged. Once registered with the DOLE, a union is considered a legitimate labor organization endowed
with the right and privileges granted by law to such organization. While a certificate of registration confers a
union with legitimacy with the concomitant right to participate in or ask for certification election in a
bargaining unit, the registration may be cancelled or the union may be decertified as the bargaining
unit, in which case the union is divested of the status of a legitimate labor organization. Among the grounds
for cancellation is the commission of any of the acts enumerated in Art. 239(a) of the Labor Code, such as
fraud and misrepresentation in connection with the adoption or ratification of the unions constitution and like
San Beda College of Law 352
4S: 2015 - 2016
LABOR LAW REVIEW Atty. Joyrich Golangco

documents. To decertify a union, it is not enough to show that the union includes ineligible employees in its
membership. It must also be shown that there was misrepresentation, false statement, or fraud in connection
with the application for registration and the supporting documents, such as the adoption or ratification of the
constitution and by-laws or amendments thereto and the minutes of ratification of the constitution or by-laws,
among other documents.

Essentially, Ventures faults both the BLR and the CA in finding that there was no fraud or misrepresentation
on the part of the Union sufficient to justify cancellation of its registration. In this regard, Ventures makes
much of, first, the separate hand-written statements of 82 employees who, in gist, alleged that they were
unwilling or harassed signatories to the attendance sheet of the organizational meeting.

However, as aptly noted by both the BLR and CA, these mostly undated written statements submitted by
Ventures on March 20, 2001, or seven months after it filed its petition for cancellation of registration,
partake of the nature of withdrawal of union membership executed after the Unions filing of a petition for
certification election on March 21, 2000. It was held that the employees withdrawal from a labor union made
before the filing of the petition for certification election is presumed voluntary, while withdrawal after the filing
of such petition is considered to be involuntary and does not affect the same. Now then, if a withdrawal from
union membership done after a petition for certification election has been filed does not vitiate such petition,
is it not but logical to assume that such withdrawal cannot work to nullify the registration of the union? Upon
this light, the Court is inclined to agree with the CA that the BLR did not abuse its discretion nor gravely err
when it concluded that the affidavits of retraction of the 82 members had no evidentiary weight.

It cannot be over-emphasized that the registration or the recognition of a labor union after it has submitted
the corresponding papers is not ministerial on the part of the BLR. After a labor organization has filed the
necessary registration documents, it becomes mandatory for the BLR to check if the requirements under Art.
234 of the Labor Code have been complied with. If the unions application is infected by falsification and like
serious irregularities, a union should be denied recognition as a legitimate labor organization. Prescinding
from these considerations, the issuance to the Union of the Certificate of Registration necessarily implies that
its application for registration and the supporting documents thereof are prima facie free from any vitiating
irregularities.

The cancellation of a unions registration doubtless has an impairing dimension on the right of labor to self-
organization. Accordingly, we can accord concurrence to the following apt observation of the BLR: "For fraud
and misrepresentation to be grounds for cancellation of union registration under Article 239 of the Labor
Code, the nature of the fraud and misrepresentation must be grave and compelling enough to vitiate the
consent of a majority of union members."

In its Comment, the Union points out that for almost seven (7) years following the filing of its petition, no
certification election has yet been conducted among the rank-and-file employees. If this be the case, the
delay has gone far enough and can no longer be allowed to continue. The CA is right when it said that
Ventures should not interfere in the certification election by actively and persistently opposing the
certification election of the Union. A certification election is exclusively the concern of employees and the
employer lacks the legal personality to challenge it. In fact, jurisprudence frowns on the employers
interference in a certification election for such interference unduly creates the impression that it intends to
establish a company union.

San Beda College of Law 353


4S: 2015 - 2016
LABOR LAW REVIEW Atty. Joyrich Golangco

ARTICLES 238-239 (now Arts. 245-247) Cancellation of Registration

176. THE HERITAGE HOTEL MANILA, acting through its owner, GRAND PLAZA HOTEL CORPORATION
v. NATIONAL UNION OF WORKERS IN THE HOTEL, RESTAURANT AND ALLIED INDUSTRIES-
HERITAGE HOTEL MANILA SUPERVISORS CHAPTER (NUWHRAIN-HHMSC)
G.R. No. 178296 January 12, 2011
NACHURA, J.:

GROUNDS FOR CANCELLATION OF UNION REGISTRATION

DOCTRINE:
It is undisputed that appellee failed to submit its annual financial reports and list of individual members in
accordance with Article 239 of the Labor Code. However, the existence of this ground should not necessarily
lead to the cancellation of union registration. Article 239 recognizes the regulatory authority of the State to
exact compliance with reporting requirements. Yet there is more at stake in this case than merely monitoring
union activities and requiring periodic documentation thereof. Failure to comply with the above requirements
shall not be a ground for cancellation of union registration but shall subject the erring officers or members to
suspension, expulsion from membership, or any appropriate penalty.

FACTS:
On October 11, 1995, respondent filed with the DOLE-NCR a petition for certification election. The Med-
Arbiter granted the petition on February 14, 1996 and ordered the holding of a certification election. On
appeal, the DOLE Secretary, in a Resolution dated August 15, 1996, affirmed the Med-Arbiters order and
remanded the case to the Med-Arbiter for the holding of a preelection conference on February 26, 1997.
Petitioner filed a motion for reconsideration, but it was denied on September 23, 1996.

The preelection conference was not held as initially scheduled; it was held a year later, or on February 20,
1998. Petitioner moved to archive or to dismiss the petition due to alleged repeated non-appearance of
respondent. The latter agreed to suspend proceedings until further notice. The preelection conference
resumed on January 29, 2000.

Subsequently, petitioner discovered that respondent had failed to submit to the Bureau of Labor Relations
(BLR) its annual financial report for several years and the list of its members since it filed its registration
papers in 1995. Consequently, on May 19, 2000, petitioner filed a Petition for Cancellation of Registration of
respondent, on the ground of the non-submission of the said documents. Petitioner prayed that respondents
Certificate of Creation of Local/Chapter be cancelled and its name be deleted from the list of legitimate labor
organizations. It further requested the suspension of the certification election proceedings.

On June 1, 2000, petitioner reiterated its request by filing a Motion to Dismiss or Suspend the Certification
Election Proceedings, arguing that the dismissal or suspension of the proceedings is warranted, considering
that the legitimacy of respondent is seriously being challenged in the petition for cancellation of registration.
Petitioner maintained that the resolution of the issue of whether respondent is a legitimate labor organization
is crucial to the issue of whether it may exercise rights of a legitimate labor organization, which include the
right to be certified as the bargaining agent of the covered employees.

Nevertheless, the certification election pushed through on June 23, 2000. Respondent emerged as the
winner.

On June 28, 2000, petitioner filed a Protest with Motion to Defer Certification of Election Results and
Winner,7 stating that the certification election held on June 23, 2000 was an exercise in futility because, once
respondents registration is cancelled, it would no longer be entitled to be certified as the exclusive
bargaining agent of the supervisory employees. Petitioner also claimed that some of respondents members
were not qualified to join the union because they were either confidential employees or managerial
employees. It then prayed that the certification of the election results and winner be deferred until the petition
for cancellation shall have been resolved, and that respondents members who held confidential or
managerial positions be excluded from the supervisors bargaining unit.

Meanwhile, respondent filed its Answer8 to the petition for the cancellation of its registration. It averred that
the petition was filed primarily to delay the conduct of the certification election, the respondents certification
San Beda College of Law 354
4S: 2015 - 2016
LABOR LAW REVIEW Atty. Joyrich Golangco

as the exclusive bargaining representative of the supervisory employees, and the commencement of
bargaining negotiations.
ISSUE: Whether or not the certificate of registration should be cancelled.

SC RULING:
Articles 238 and 239 of the Labor Code gives the Regional Director ample discretion in dealing with a petition
for cancellation of a unions registration, particularly, determining whether the union still meets the
requirements prescribed by law. It is sufficient to give the Regional Director license to treat the late filing of
required documents as sufficient compliance with the requirements of the law. After all, the law requires the
labor organization to submit the annual financial report and list of members in order to verify if it is still viable
and financially sustainable as an organization so as to protect the employer and employees from fraudulent
or fly-by-night unions. With the submission of the required documents by respondent, the purpose of the law
has been achieved, though belatedly.

We cannot ascribe abuse of discretion to the Regional Director and the DOLE Secretary in denying the
petition for cancellation of respondents registration. The union members and, in fact, all the employees
belonging to the appropriate bargaining unit should not be deprived of a bargaining agent, merely because of
the negligence of the union officers who were responsible for the submission of the documents to the BLR.

It is worth mentioning that the Labor Codes provisions on cancellation of union registration and on reportorial
requirements have been recently amended by R.A. No. 9481 which lapsed into law on May 25, 2007 and
became effective on June 14, 2007. The amendment sought to strengthen the workers right to self-
organization and enhance the Philippines compliance with its international obligations as embodied in the
International Labour Organization (ILO) Convention No. 87, pertaining to the non-dissolution of workers
organizations by administrative authority. Thus, R.A. No. 9481 amended Article 239 to read:

ART. 239. Grounds for Cancellation of Union Registration.The following may constitute
grounds for cancellation of union registration:

(a) Misrepresentation, false statement or fraud in connection with the adoption or ratification
of the constitution and by-laws or amendments thereto, the minutes of ratification, and
the list of members who took part in the ratification;
(b) Misrepresentation, false statements or fraud in connection with the election of officers,
minutes of the election of officers, and the list of voters;
(c) Voluntary dissolution by the members.

R.A. No. 9481 also inserted in the Labor Code Article 242-A, which provides: ART. 242-A. Reportorial
Requirements.The following are documents required to be submitted to the Bureau by the legitimate labor
organization concerned:
(a) Its constitution and by-laws, or amendments thereto, the minutes of ratification, and the list of
members who took part in the ratification of the constitution and by-laws within thirty (30)
days from adoption or ratification of the constitution and by-laws or amendments thereto;
(b) Its list of officers, minutes of the election of officers, and list of voters within thirty (30) days from
election;
(c) Its annual financial report within thirty (30) days after the close of every fiscal year; and
(d) Its list of members at least once a year or whenever required by the Bureau. Failure to comply
with the above requirements shall not be a ground for cancellation of union registration
but shall subject the erring officers or members to suspension, expulsion from
membership, or any appropriate penalty.

ILO Convention No. 87, which we have ratified in 1953, provides that "workers and employers organizations
shall not be liable to be dissolved or suspended by administrative authority." The ILO has expressed the
opinion that the cancellation of union registration by the registrar of labor unions, which in our case is the
BLR, is tantamount to dissolution of the organization by administrative authority when such measure would
give rise to the loss of legal personality of the union or loss of advantages necessary for it to carry out its
activities, which is true in our jurisdiction. Although the ILO has allowed such measure to be taken, provided
that judicial safeguards are in place, i.e., the right to appeal to a judicial body, it has nonetheless reminded its
members that dissolution of a union, and cancellation of registration for that matter, involve serious
consequences for occupational representation. It has, therefore, deemed it preferable if such actions were to
San Beda College of Law 355
4S: 2015 - 2016
LABOR LAW REVIEW Atty. Joyrich Golangco

be taken only as a last resort and after exhausting other possibilities with less serious effects on the
organization. It is undisputed that appellee failed to submit its annual financial reports and list of individual
members in accordance with Article 239 of the Labor Code. However, the existence of this ground should not
necessarily lead to the cancellation of union registration. Article 239 recognizes the regulatory authority of the
State to exact compliance with reporting requirements. Yet there is more at stake in this case than merely
monitoring union activities and requiring periodic documentation thereof.

The more substantive considerations involve the constitutionally guaranteed freedom of association and right
of workers to self-organization. Also involved is the public policy to promote free trade unionism and collective
bargaining as instruments of industrial peace and democracy. An overly stringent interpretation of the statute
governing cancellation of union registration without regard to surrounding circumstances cannot be allowed.
Otherwise, it would lead to an unconstitutional application of the statute and emasculation of public policy
objectives. Worse, it can render nugatory the protection to labor and social justice clauses that pervades the
Constitution and the Labor Code.

San Beda College of Law 356


4S: 2015 - 2016
LABOR LAW REVIEW Atty. Joyrich Golangco

177. REPUBLIC OF THE PHILIPPINES, represented by Department of Labor and Employment (DOLE) v.
KAWASHIMA TEXTILE MFG., PHILIPPINES, INC
G.R. No. 160352 July 23, 2008
AUSTRIA-MARTINEZ, J:

LEGITIMACY OF A DULY REGISTERED LABOR ORGANIZATION

FACTS:
Kawashima Free Workers Union (KFWU) filed a Petition for Certification Election to be conducted in the
bargaining unit composed of 145 rank-and-file employees of Kawashima Textile Mfg. Phils., Inc. Attached to
its petition are a Certificate of Creation of Local/Chapter issued on January 19, 2000 by DOLE Regional
Office No. IV, stating that it [KFWU] submitted to said office a Charter Certificate issued to it by the national
federation Phil. Transport & General Workers Organization (PTGWO), and a Report of Creation of
Local/Chapter.

Kawashima Textile Mfg. Phils., Inc. argues that KFWU did not acquire any legal personality because its
membership of mixed rank-and-file and supervisory employees violated Article 245 of the Labor Code, and
its failure to submit its books of account.

ISSUES:
The Republic of the Philippines filed the present petition to seek cl osur e on two issues:

1) WON a mixed membership of rank-and-file and supervisory employees in a union is a


ground for the dismissal of a petition for certification election in view of the amendment brought about
by D.O. 9, series of 1997, which deleted the phraseology in the old rule that "[t]he appropriate
bargaining unit of the rank-and-file employee shall not include the supervisory employees and/or
security guards; and
2) WON the legitimacy of a duly registered labor organization can be collaterally attacked in a petition
for a certification election through a motion to dismiss filed by an employer such as Kawashima
Textile Manufacturing Phils., Inc.

SC RULING:

1) No. In short: here, RA 9481 did not apply. If it did, the ruling would have been NO. Yet, even without
using RA 9481, the Court still ruled NO using 1997 Amended Omnibus Rules, as interpreted by the
Court in Tagaytay Highlands, San Miguel and Air Philippines.
The key to the closure that petitioner seeks could have been Republic Act (R.A.) No. 9481. Sections 8 and 9
thereof provide:

Section 8. Article 245 of the Labor Code is hereby amended to read as follows:

Art. 245. Ineligibility of Managerial Employees to Join any Labor Organization; Right of
Supervisory Employees. - Managerial employees are not eligible to join, assist or form any labor
organization. Supervisory employees shall not be eligible for membership in the collective
bargaining unit of the rank-and-file employees but may join, assist or form separate collective
bargaining units and/or legitimate labor organizations of their own. The rank and file union and
the supervisors' union operating within the same establishment may join the same federation or
national union.

Section 9. A new provision, Article 245-A is inserted into the Labor Code to read as follows:

Art. 245-A. Effect of Inclusion as Members of Employees Outside the Bargaining Unit. - The
inclusion as union members of employees outside the bargaining unit shall not be a
ground for the cancellation of the registration of the union. Said employees are
automatically deemed removed from the list of membership of said union." (Emphasis
supplied)

San Beda College of Law 357


4S: 2015 - 2016
LABOR LAW REVIEW Atty. Joyrich Golangco

Moreover, under Section 4, a pending petition for cancellation of registration will not hinder a legitimate labor
organization from initiating a certification election, viz:

Sec. 4. A new provision is hereby inserted into the Labor Code as Article 238-A to read as follows:

Art. 238-A. Effect of a Petition for Cancellation of Registration. - A petition for cancellation of
union registration shall not suspend the proceedings for certification election nor shall it
prevent the filing of a petition for certification election.

In case of cancellation, nothing herein shall restrict the right of the union to seek just and
equitable remedies in the appropriate courts." (Emphasis supplied)

Furthermore, under Section 12 of R.A. No. 9481, employers have no personality to interfere with or thwart a
petition for certification election filed by a legitimate labor organization, to wit:

Sec. 12. A new provision, Article 258-A is hereby inserted into the Labor Code to read as follows:

Art. 258-A. Employer as Bystander. - In all cases, whether the petition for certification election is
filed by an employer or a legitimate labor organization, the employer shall not be considered a
party thereto with a concomitant right to oppose a petition for certification election. The
employer's participation in such proceedings shall be limited to: (1) being notified or
informed of petitions of such nature; and (2) submitting the list of employees during the
pre-election conference should the Med-Arbiter act favorably on the petition." (Emphasis
supplied)

However, R.A. No. 9481 took effect only on June 14, 2007; hence, it applies only to labor representation cases
filed on or after said date. As the petition for certification election subject matter of the present petition was filed
by KFWU on January 24, 2000,http://www.lawphil.net/judjuris/juri2008/jul2008/gr_160352_2008.html - fnt28
29
R.A. No. 9481 cannot apply to it. There may have been curative labor legislations that were given
retrospective effect, but not the aforecited provisions of R.A. No. 9481, for otherwise, substantive rights and
interests already vested would be impaired in the process.

Instead, the law and rules in force at the time of the filing by KFWU of the petition for certification election on
33
January 24, 2000 are R.A. No. 6715, amending Book V of Presidential Decree (P.D.) No. 442 (Labor Code), as
34
amended, and the Rules and Regulations Implementing R.A. No. 6715, as amended by Department Order No.
9, series of 1997.
48
When the issue of the effect of mingling was brought to the fore in Toyota, the Court, citing Article 245 of the
Labor Code, as amended by R.A. No. 6715, held:

Clearly, based on this provision, a labor organization composed of both rank-and-file and
supervisory employees is no labor organization at all. It cannot, for any guise or purpose, be a
legitimate labor organization. Not being one, an organization which carries a mixture of
rank-and-file and supervisory employees cannot possess any of the rights of a
legitimate labor organization, including the right to file a petition for certification election
for the purpose of collective bargaining. It becomes necessary, therefore, anterior to the
granting of an order allowing a certification election, to inquire into the composition of any labor
organization whenever the status of the labor organization is challenged on the basis of Article
245 of the Labor Code.

xxxx

In the case at bar, as respondent union's membership list contains the names of at least
twenty-seven (27) supervisory employees in Level Five positions, the union could not, prior to
purging itself of its supervisory employee members, attain the status of a legitimate labor
organization. Not being one, it cannot possess the requisite personality to file a petition for
49
certification election. (Emphasis supplied)

San Beda College of Law 358


4S: 2015 - 2016
LABOR LAW REVIEW Atty. Joyrich Golangco

In Dunlop, in which the labor organization that filed a petition for certification election was one for supervisory
employees, but in which the membership included rank-and-file employees, the Court reiterated that such labor
organization had no legal right to file a certification election to represent a bargaining unit composed of
supervisors for as long as it counted rank-and-file employees among its members.

But then, on June 21, 1997, the 1989 Amended Omnibus Rules was further amended by Department Order
No. 9, series of 1997 (1997 Amended Omnibus Rules). Specifically, the requirement under Sec. 2(c) of the
1989 Amended Omnibus Rules - that the petition for certification election indicate that the bargaining unit of
rank-and-file employees has not been mingled with supervisory employees - was removed. Instead, what the
1997 Amended Omnibus Rules requires is a plain description of the bargaining unit, which does not require
that, for its creation and registration, a local or chapter submit a list of its members.
Then came Tagaytay Highlands Intl. Golf Club, Inc. v. Tagaytay Highlands Employees Union-PGTWO54 in
which the core issue was whether mingling affects the legitimacy of a labor organization and its right to file a
petition for certification election. This time, given the altered legal milieu, the Court abandoned the view in
Toyota and Dunlop and reverted to its pronouncement in Lopez that while there is a prohibition against the
mingling of supervisory and rank-and-file employees in one labor organization, the Labor Code does not
provide for the effects thereof. Thus, the Court held that after a labor organization has been registered, it may
exercise all the rights and privileges of a legitimate labor organization. Any mingling between supervisory and
rank-and-file employees in its membership cannot affect its legitimacy for that is not among the grounds for
cancellation of its registration, unless such mingling was brought about by misrepresentation, false statement
or fraud under Article 239 of the Labor Code.

In San Miguel Corp. (Mandaue Packaging Products Plants) v. Mandaue Packing Products Plants-San Miguel
Packaging Products-San Miguel Corp. Monthlies Rank-and-File Union-FFW, the Court explained that since
the 1997 Amended Omnibus Rules does not require a local or chapter to provide a list of its members, it
would be improper for the DOLE to deny recognition to said local or chapter on account of any question
pertaining to its individual members.

More to the point is Air Philippines Corporation v. Bureau of Labor Relations,59 which involved a petition for
cancellation of union registration filed by the employer in 1999 against a rank-and-file labor organization on
the ground of mixed membership:60 the Court therein reiterated its ruling in Tagaytay Highlands that the
inclusion in a union of disqualified employees is not among the grounds for cancellation, unless such
inclusion is due to misrepresentation, false statement or fraud under the circumstances enumerated in
Sections (a) and (c) of Article 239 of the Labor Code.

All said, while the latest issuance is R.A. No. 9481, the 1997 Amended Omnibus Rules, as interpreted by the
Court in Tagaytay Highlands, San Miguel and Air Philippines, had already set the tone for it. Toyota and Dunlop
no longer hold sway in the present altered state of the law and the rules.

FLOW: Toyota/Dunlop Omnibus Rules / Tagaytay/ San Miguel/ Air Phil

2) No. Except when it is requested to bargain collectively, an employer is a mere bystander to any
petition for certification election; such proceeding is non-adversarial and merely investigative, for the
purpose thereof is to determine which organization will represent the employees in their collective
bargaining with the employer. The choice of their representative is the exclusive concern of the
employees; the employer cannot have any partisan interest therein; it cannot interfere with, much less
oppose, the process by filing a motion to dismiss or an appeal from it; not even a mere allegation that
some employees participating in a petition for certification election are actually managerial employees
will lend an employer legal personality to block the certification election.65 The employer's only right in
the proceeding is to be notified or informed thereof.

San Beda College of Law 359


4S: 2015 - 2016
LABOR LAW REVIEW Atty. Joyrich Golangco

ARTICLE 241 (now Art. 250) Rights and Conditions of Membership in Labor Organization

178. DEL PILAR ACADEMY, EDUARDO ESPEJO and ELISEO OCAMPO, JR. v. DEL PILAR ACADEMY
EMPLOYEES UNION
G.R. No. 170112 April 30, 2008
NACHURA, J.;

RIGHT OF THE UNION TO COLLECT AGENCY FEES FROM NON-UNION MEMBERS (CHECK-OFF)

DOCTRINE:
Article 248(e) makes it explicit that Article 241, paragraph (o), requiring written authorization is inapplicable to
non-union members, especially in this case where the non-union employees receive several benefits under the
CBA.

FACTS:
Respondent Del Pilar Academy Employees Union (the UNION) is the certified collective bargaining
representative of teaching and non-teaching personnel of petitioner Del Pilar Academy (DEL PILAR), an
educational institution operating in Imus, Cavite. On September 15, 1994, the UNION and DEL PILAR entered
into a Collective Bargaining Agreement (CBA). The UNION then assessed agency fees from non-
union employees, and requested DEL PILAR to deduct said assessment from the employees salaries and
wages. DELPILAR, however, refused to effect deductions claiming that the non-union employees were not
amenable to it.

In September 1997, the UNION negotiated for the renewal of the CBA. DEL PILAR, however, refused to
re n e w t h e same unless the provision regarding entitlement to two (2) months summer vacation leave with
pay will be amended by limiting the same to teachers, who have rendered at least three (3) consecutive
academic years of satisfactory service. The UNION objected to the proposal claiming diminution of benefits.
DEL PILAR refused to sign the CBA, resulting in a deadlock. The UNION requested DEL PILAR to submit
the case for voluntary arbitration, but the latter allegedly refused, prompting the UNION to file a case for
unfair labor practice with the Labor Arbiter.

DEL PILAR denied committing unfair labor practices against the UNION. DEL PILAR
admitted its failure to deduct the agency fees from the salaries of non-union employees, but justifies the
non-deduction by the absence of individual written authorization. It posits that Article 248(e) is inapplicable
considering that its employees derived no benefits from the CBA. Besides, the non-union employees objected
to the deduction; hence, a written authorization is indispensable to affect a valid check off. As regards the
proposal to amend the provision on summer vacation leave with pay, DEL PILAR alleged that the proposal
cannot be considered unfair for it was done to make the provision of the CBA conformable to the DECS
Manual of Regulations for Private Schools.

LA RULING: There was an error on the part of DEL PILAR not to have collected agency fee due other
workers who are non-union members but are included in the bargaining unit being represented by the UNION.
As stated in Art. 248, to wit:

Employees of an appropriate collective bargaining unit who are not members of the
recognized collective bargaining agency may be assessed a reasonable fee equivalent
to the dues and other fees paid by members of the recognized collective
bargaining agreement: Provided, that the individual authorization required under Article
[241], paragraph (o) of this Code shall not apply to the non-members of the
recognized collective bargaining agent.

For receipt of CBA benefits brought about by the CBA negotiated with petitioners, non-union members
are duty bound to pay agency fees which may lawfully be deducted sans individual check-off
authorization. Being recipients of said benefits; they should share and be made to pay the same
considerations imposed upon the union members.

The proposal to decrease the coverage of the 11th and 12th month vacation with pay was not done in
bad faith but rather in an honest attempt to make perfect procession following the DECS Manuals. It is
of judicial notice that in the course of negotiation, almost all provisions are up for grabs, amendments or
San Beda College of Law 360
4S: 2015 - 2016
LABOR LAW REVIEW Atty. Joyrich Golangco

change. This is something normal in the course of a negotiation and does not necessarily connote
bad faith as each everyone has the right to negotiate reward or totally amend the provisions of the
contract/agreement. It must be noted that a CBA is a contract between labor and management and is
not simply a litany of benefits for labor. For unfair labor practice to prosper there must be a clear
showing of acts aimed at stifling the workers right to self- organization. Mere allegations and mistake
notions would not suffice.

NLRC RULING: Affirmed LAs ruling, upheld the UNIONs right to agency fee, but did not consider
DELPILARs failure to deduct the same an unfair labor practice

CA RULING: Private respondent Del Pilar Academy is ordered to deduct the agency fees from non-union
members. The agency fees shall be equivalent to the dues and other fees paid by the union members.

ISSUES: Can the Union collect agency fees from non-union members even without a written authorization
and thus constituting a valid check off?

SC RULING:
Yes, the grant of annual salary increase is not the only provision in the CBA that benefited the non-union
employees. The UNION negotiated for other benefits as well that surely benefited the non-union employees,
justifying the collection of, and the UNIONs entitlement to, agency fees.

Accordingly, no requirement of written authorization from the non-union employees is needed to effect a valid
check off. Article 248(e) makes it explicit that Article 241, paragraph (o), requiring written authorization is
inapplicable to non-union members, especially in this case where the non-union employees receive several
benefits under the CBA.

San Beda College of Law 361


4S: 2015 - 2016
LABOR LAW REVIEW Atty. Joyrich Golangco

179. EDUARDO J. MARINO, JR., ET. AL. v. GIL Y. GAMILLA, ET. AL.
G.R. No. 149763 July 7, 2009
CHICO-NAZARIO, J.:

CHECK OFF

DOCTRINE:
General rule is that attorneys fees, negotiation fees, and other similar charges may only be collected from
union funds, not from the amounts that pertain to individual union members

FACTS:
Petitioners were among the executive officers and directors (collectively called the Mario Group) of the
University of Sto. Tomas Faculty Union (USTFU), a labor union duly organized and registered under the laws
of the Republic of the Philippines and the bargaining representative of the faculty members of the University of
Santo Tomas (UST). Respondents were UST professors and USTFU members.

The 1986 CBA between UST and USTFU expired on 31 May 1988. Thereafter, bargaining negotiations ensued
between UST and the Mario Group, which represented USTFU. They were not able to reach an agreement
and a bargaining deadlock was declared.

On 1992, UST and USTFU executed a (MOA), whereby UST faculty members belonging to the collective
bargaining unit were granted additional economic benefits for the fourth and fifth years of the 1988-1993 CBA,
specifically, the period from 1 June 1992 up to 31 May 1993.

MEMORANDUM OF AGREEMENT

1.0. The University hereby grants additional benefits to Faculty Members


belonging to the collective bargaining unit xxx, which additional benefits shall amount in
the aggregate to P42,000,000.00[.]
7.0. It is clearly understood and agreed upon that the aggregate sum of P42
million is chargeable against the share of the faculty members in the incremental
proceeds of tuition fees collected and still to be collected; xxx and incremental
proceeds are, by law and pertinent (DECS) regulations, required to be allotted for the
payment of salaries, wages, allowances and other benefits of teaching and non-teaching
personnel for the UNIVERSITY.

On 12 September 1992, the majority of USTFU members signed individual instruments of ratification,
which purportedly signified their consent to the economic benefits granted under the MOA. USTFU, through its
President, petitioner Atty. Mario, wrote a letter to the UST Treasurer requesting the release to the union of the
sum ofP4.2 million, which was 10% of the P42 million economic benefits package granted by the MOA to faculty
members belonging to the collective bargaining unit. UST remitted the sum of P4.2 million to USTFU. After
deducting from the P42 million economic benefits package several expenses, a net amount of P6, 389,145.04
remained and was distributed to the faculty members.

On 15 December 1994, respondents filed with the Med-Arbiter, DOLE- National Capital Region (NCR), a
Complaint for the expulsion of the Mario Group as USTFU officers and directors. Alleged in their Complaint
that the Mario Group violated the rights and conditions of membership in USTFU, particularly by: 1) investing the
unspent balance of the P42 million economic benefits package given by UST without prior approval of the
general membership; 2) simultaneously holding elections viva voce; 3) ratifying the CBA involving the P42
million economic benefits package; and 4) approving the attorneys/agency fees worth P4.2 million in the
form of check-off.

On 16 December 1994, UST and USTFU, represented by the Mario Group, entered into a new CBA,
effective 1 June 1993 to 31 May 1998 (1993-1998 CBA).

24 September 1996, petitioner Collantes, as USTFU Secretary- General, posted notices in some faculty rooms at
UST, informing the union members of a general assembly. The agenda was the election of new On October 4
1996, the UST Secretary General headed a general faculty assembly. Respondents were among the elected
officers of USTFU (collectively referred to as the Gamilla Group). Med-Arbiter DOLE-NCR, nullified the election of
San Beda College of Law 362
4S: 2015 - 2016
LABOR LAW REVIEW Atty. Joyrich Golangco

the Gamilla Group as USTFU officers. Affirmed on appeal by the (BLR). Respondents filed before the Med-
Arbiter, DOLE-NCR, a complaint against the Mario Group, as well as the Philippine Foundation for the
Advancement of the Teaching Profession, Inc., Security Bank Corporation, and Bank of the Philippine Islands
claiming that they were the legitimate USTFU officers, having been elected on 4 October 1996. They prayed
for an order directing the Mario Group to cease and desist from using the name of USTFU and from performing
acts for and on behalf of the USTFU and the rest of the members of the collective bargaining unit.

DOLE-NCR Regional Director: Mario Group, as the executive officers of USTFU, guilty of violating the
provisions of the USTFU Constitution and By-laws. Also, they violated Article 241(c) and (l) of the Labor Code
when among others, they invested in a bank, without prior consent of USTFU members, the sum of
P9,766,570.01, which formed part of the P42 million economic benefits package. Additionally, the check-off
of P4.2 million collected by the Mario Group, as negotiation fees, was invalid. Under Republic Act No. 6728,
70% of the tuition fee increases should be allotted to academic and non-academic personnel. Given that the
records were silent as to how much of the P42 million economic benefits package was obtained through
negotiations and how much was from the statutory allotment of 70% of the tuition fee increases, the DOLE-
NCR Regional Director held that the entire amount was within the statutory allotment, which could not be the
subject of negotiation and, thus, could not be burdened by negotiation fees

BLR: Agreed with DOLE-NCR Regional Director

CA: Agreed with the BLR.

ISSUE: Was the 10% check off made by the Marino group valid?

SC RULING:
No. UST and USTFU stipulated in their 10 September 1992 MOA that the P42 million economic benefits
package granted by UST to the members of the collective bargaining unit represented by USTFU, was
chargeable against the 70% allotment from the proceeds of the tuition fee increases collected and still to be
collected by UST. As observed by the DOLE-NCR Regional Director, and affirmed by both the BLR and the
Court of Appeals, there is no showing that any portion of the P42 million economic benefits package was
derived from sources other than the 70% allotment from tuition fee increases of UST.

Given the lack of evidence to the contrary, it can be conclusively presumed that the entire P42 million economic
benefits package extended to USTFU came from the 70% allotment from tuition fee increases of
UST. Preceding from this presumption, any deduction from the P42 million economic benefits package,
such as the P4.2 million claimed by the Mario Group as attorneys/agency fees, should not be allowed,
because it would ultimately result in the reduction of the statutorily mandated 70% allotment from the
tuition fee increases of UST.

General rule is that attorneys fees, negotiation fees, and other similar charges may only be collected from
union funds, not from the amounts that pertain to individual union members. As an exception, special
assessments or other extraordinary fees may be levied upon or checked off from any amount due an employee
for as long as there is proper authorization by the employee.

A check-off is a process or device whereby the employer, on agreement with the Union, recognized as the
proper bargaining representative, or on prior authorization from the employees, deducts union dues or agency
fees from the latter's wages and remits them directly to the Union.

The Court finds that, in the instant case, the P42 million economic benefits package granted by UST did not
constitute union funds from whence the P4.2 million could have been validly deducted as attorneys fees. The
P42 million economic benefits package was not intended for the USTFU coffers, but for all the members of the
bargaining unit USTFU represented, whether members or non-members of the union. A close reading of the
terms of the MOA reveals that after the satisfaction of the outstanding obligations of UST under the 1986 CBA,
the balance of the P42 million was to be distributed to the covered faculty members of the collective bargaining
unit in the form of salary increases, returns on paycheck deductions; and increases in hospitalization,
educational, and retirement benefits, and other economic benefits. The deduction of the P4.2 million, as alleged
attorneys/agency fees, from the P42 million economic benefits package effectively decreased the share from
said package accruing to each member of the collective bargaining unit

San Beda College of Law 363


4S: 2015 - 2016
LABOR LAW REVIEW Atty. Joyrich Golangco

The Court further determines that the requisites for a valid levy and check-off of special assessments, laid
down by Article 241(n) and (o), respectively, of the Labor Code, as amended, have not been complied with
in the case at bar. To recall, these requisites are: (1) an authorization by a written resolution of the majority of
all the union members at the general membership meeting duly called for the purpose; (2) secretary's record of
the minutes of the meeting; and (3) individual written authorization for check-off duly signed by the employee
concerned.

In an attempt to comply with the foregoing requirements, the Mario Group caused the majority of the general
membership of USTFU to individually sign a document, which embodied the ratification of the MOA between
UST and USTFU, dated 10 September 1992, as well as the authorization for the check-off of P4.2 million, from
the P42 million economic benefits package, as payment for attorneys fees. As held by the Court of Appeals,
however, the said documents constitute unsatisfactory compliance with the requisites set forth in the Labor
Code, as amended, and in the USTFU Constitution and By-Laws, even though individually signed by a majority
of USTFU members.

The inclusion of the authorization for a check-off of union dues and special assessments for the Labor
Education Fund and attorneys fees, in the same document for the ratification of the 10 September 1992 MOA
granting the P42 million economic benefits package, necessarily vitiated the consent of USTFU members.

The failure of the Mario Group to strictly comply with the requirements set forth by the Labor Code, as
amended, and the USTFU Constitution and By- Laws, invalidates the questioned special assessment.
Substantial compliance is not enough in view of the fact that the special assessment will diminish the
compensation of the union members. Their express consent is required, and this consent must be obtained in
accordance with the steps outlined by law, which must be followed to the letter. No shortcuts are allowed.

San Beda College of Law 364


4S: 2015 - 2016
LABOR LAW REVIEW Atty. Joyrich Golangco

ARTICLE 242 (now Art. 251) Rights of Legitimate Labor Organizations

180. THE HERITAGE HOTEL MANILA, acting through its owner, GRAND PLAZA HOTEL CORPORATION
v. NATIONAL UNION OF WORKERS IN THE HOTEL, RESTAURANT AND ALLIED INDUSTRIES-
HERITAGE HOTEL MANILA SUPERVISORS CHAPTER (NUWHRAIN-HHMSC
G.R. No. 178296 January 12, 2011
NACHURA, J.:

DOCTRINE:
Non-filing of reportorial requirements is not a ground for cancellation of union registration.

FACTS:
Respondent filed with the Department of Labor and Employment- National Capital Region (DOLE-NCR) a
petition for certification election. The Med-Arbiter granted the petition and ordered the holding of a certification
election. On appeal, the DOLE Secretary affirmed the Med-Arbiters order and remanded the case to the Med-
Arbiter for the holding of a preelection conference. Petitioner filed a motion for reconsideration, but it was
denied.

Subsequently, petitioner discovered that respondent had failed to submit to the Bureau of Labor Relations (BLR)
its annual financial report for several years and the list of its members since it filed its registration papers in
1995. Consequently, petitioner filed a Petition for Cancellation of Registration of respondent, on the ground of
the non-submission of the said documents. Petitioner prayed that respondents Certificate of Creation of
Local/Chapter be cancelled and its name be deleted from the list of legitimate labor organizations. It further
requested the suspension of the certification election proceedings.

On June 1, 2000, petitioner reiterated its request by filing a Motion to dismiss or suspend the [Certification
Election] Proceedings. Nevertheless, the certification election pushed through on June 23, 2000. Respondent
emerged as the winner.

On June 28, 2000, petitioner filed a Protest with Motion to Defer Certification of Election Results and Winner,
stating that the certification election held on June 23, 2000 was an exercise in futility because, once
respondents registration is cancelled, it would no longer be entitled to be certified as the exclusive
bargaining agent of the supervisory employees.

Meanwhile, respondent filed its Answer to the petition for the cancellation of its registration. It averred
that the petition was filed primarily to delay the conduct of the certification election, the respondents certification
as the exclusive bargaining representative of the supervisory employees, and the commencement of bargaining
negotiations.

Respondent prayed for the dismissal of the petition for cancellation for the following reasons: (a) petitioner is
estopped from questioning respondents status as a legitimate labor organization as it had already recognized
respondent as such during the preelection conferences; (b) petitioner is not the party-in- interest, as the union
members are the ones who would be disadvantaged by the non-submission of financial reports; (c) it has
already complied with the reportorial requirements, having submitted its financial statements for 1996, 1997,
1998, and 1999, its updated list of officers, and its list of members for the years 1995, 1996, 1997, 1998,
and 1999; (d) the petition is already moot and academic, considering that the certification election had
already been held, and the members had manifested their will to be represented by respondent

MED-ARBITER: pendency of a petition for cancellation of registration is not a bar to the holding of a certification
election. Thus, in an Order dated January 26, 2001, the Med-Arbiter dismissed petitioners protest, and
certified respondent as the sole and exclusive bargaining agent of all supervisory employees.

In the meantime, Regional Director Alex E. Maraan (Regional Director Maraan) of DOLE-NCR finally resolved
the petition for cancellation of registration. While emphasizing that the non-compliance with the law is not
viewed with favorconsidered the belated submission of the annual financial reports and the list of members as
sufficient compliance thereof and considered them as having been submitted on time.

DOLE SEC: Denied petitioners MR (jurisdiction belongs to the BLR, but the BLR director in this case inhibited
himself, thus the DOLE Sec took cognizance of the appeal)
San Beda College of Law 365
4S: 2015 - 2016
LABOR LAW REVIEW Atty. Joyrich Golangco

CA RULING: DOLE sec did not commit grave abuse of discretion when she took cognizance of the case
because in the absence of the BLR Director, there is no person more competent to resolve the appeal than the
DOLE Secretary. Further, it affirmed DOLEs ruling requirements of registration of labor organizations are an
exercise of the overriding police power of the State, designed for the protection of workers against potential
abuse by the union that recruits them. These requirements should not be exploited to work against the workers
constitutionally protected right to self- organization.

ISSUE: Is the non-filing of reportorial requirements a ground for cancellation of union registration?

SC RULING:
No. Articles 238-239 give the Regional Director ample discretion in dealing with a petition for cancellation of a
unions registration, particularly, determining whether the union still meets the requirements prescribed by law. It
is sufficient to give the Regional Director license to treat the late filing of required documents as sufficient
compliance with the requirements of the law. After all, the law requires the labor organization to submit the
annual financial report and list of members in order to verify if it is still viable and financially sustainable as an
organization so as to protect the employer and employees from fraudulent or fly-by-night unions. With the
submission of the required documents by respondent, the purpose of the law has been achieved, though
belatedly.

We cannot ascribe abuse of discretion to the Regional Director and the DOLE Secretary in denying the petition
for cancellation of respondents registration. The union members and, in fact, all the employees belonging
to the appropriate bargaining unit should not be deprived of a bargaining agent, merely because of the
negligence of the union officers who were responsible for the submission of the documents to the
BLR.

It is worth mentioning that the Labor Codes provisions on cancellation of union registration and on
reportorial requirements have been recently amended by Republic Act (R.A.) No. 9481, An Act
Strengthening the Workers Constitutional Right to Self-Organization, Amending for the Purpose Presidential
Decree No. 442, As Amended, Otherwise Known as the Labor Code of the Philippines, which lapsed into law on
May 25, 2007 and became effective on June 14, 2007. Thus, R.A. No. 9481 amended Article 239 (now 245) to
read:

R.A. No. 9481 also inserted in the Labor Code Article 242-A (now 248-A), which provides:
ART. 242-A. Reportorial Requirements.The following are documents required to be s ubmitted to the
Bureau by the legitimate labor organization concerned:
xxxxxxx

Failure to comply with the above requirements shall not be a ground for cancellation of union
registration but shall subject the erring officers or members to suspension, expulsion from
membership, or any appropriate penalty.

San Beda College of Law 366


4S: 2015 - 2016
LABOR LAW REVIEW Atty. Joyrich Golangco

181. ABARIA vs. NATIONAL LABOR RELATIONS COMMISSION
G.R. No. 154113 December 7, 2011
VILLARAMA JR., J.

DOCTRINE:
Legality of mass termination of hospital employees who participated in strike and picketing activities.

FACTS:
The National Federation of Labor (NFL) is the exclusive bargaining representative of the rank-and-file
employees of Metro Cebu Community Hospital, Inc (MCCHI).
On December 6, 1995, Nava through NAMA-MCCH-NFL wrote Rev. Iyoy (Hospital admin) expressing
the unions desire to renew the CBA, attaching to her letter a statement of proposals signed/endorsed
by 153 union members. Nava subsequently requested that some employees to be allowed to avail of
one-day union leave with pay
Meanwhile, Atty. Alforque informed MCCHI that NFL has not authorized any person for collective
bargaining negotiations. By January 1996, the collection of union fees (check-off) was temporarily
suspended by MCCHI in view of the existing conflict between the federation and its local affiliate.
On February 26, 1996, upon the request of Atty. Alforque, MCCHI granted one-day union leave with pay
for 12 union members
The next day, several union members led by Nava and her group launched a series of mass actions
such as wearing black and red armbands/headbands, marching around the hospital premises and
putting up placards, posters and streamers.
MCCHI directed the union officers led by Nava to submit within 48 hours a written explanation why they
should not be terminated for having engaged in illegal concerted activities amounting to strike, and
placed them under immediate preventive suspension. Responding to this directive, Nava and her group
denied there was a temporary stoppage of work, explaining that employees wore their armbands only
as a sign of protest and reiterating their demand for MCCHI to comply with its duty to bargain
collectively.
Rev. Iyoy, having been informed that Nava and her group have also been suspended by NFL, directed
said officers to appear before his office for investigation in connection with the illegal strike. Said union
officers, however, invoked the grievance procedure provided in the CBA to settle the dispute between
management and the union.
On March 13 and 19, 1996, DOLE issued certifications stating that there is nothing in their records
which shows that NAMA-MCCH-NFL is a registered labor organization, and that said union submitted
only a copy of its Charter Certificate on January 31, 1995.
NAMA-MCCH-NFL filed a Notice of Strike but the same was deemed not filed for want of legal
personality on the part of the filer. Despite such rebuff, Nava and her group still conducted a strike vote,
which an overwhelming majority of union members approved of.
MCCHI again sent notices informing them that their refusal to submit to investigation is deemed a
waiver of their right to explain their side and management shall proceed to impose proper disciplinary
action under the circumstances. MCCHI then sent termination letters to union leaders and other
members who participated in the strike and picketing activities and also issued a cease-and-desist order
to the rest of the striking employees stressing that the wildcat concerted activities spearheaded by the
Nava group is illegal without a valid Notice of Strike and warning them that non-compliance will compel
management to impose disciplinary actions against them. For their continued picketing activities
despite the said warning, more than 100 striking employees were dismissed
Unfazed, the striking union members held more mass actions. The means of ingress to and egress
from the hospital were blocked. Employees and patients reported acts of intimidation and harassment
perpetrated by union leaders and members. With the intensified atmosphere of violence and animosity
within the hospital premises as a result of continued protest activities by union members, MCCHI
suffered heavy losses due to low patient admission rates.
MCCHI filed a petition for injunction in the NLRC and a TRO was issued.
MCCHI presented 12 witnesses (hospital employees and patients), including a security guard who was
stabbed by an identified sympathizer while in the company of Navas group. MCCHIs petition was
granted and a permanent injunction was issued enjoining the Nava group from committing illegal acts
mentioned in Art. 264 of the Labor Code
Thereafter, several complaints for illegal dismissal and unfair labor practice were filed by the terminated
employees against MCCHI, Rev. Iyoy, UCCP and members of the Board of Trustees of MCCHI.
San Beda College of Law 367
4S: 2015 - 2016
LABOR LAW REVIEW Atty. Joyrich Golangco

EXEC. LA: dismissed the complaints for unfair labor practice in NLRC for finding no basis for the charge of
unfair labor practice and declared the strike and picketing activities illegal having been conducted by NAMA-
MCCH-NFL which is not a legitimate labor organization;
NLRC: denied complainants motion for reconsideration
CA via a petition for certiorari: dismissed because only 47 out of 88 petitioners signed the certification against
forum shopping.

ISSUES:
(1) Did CA err in dismissing the certiorari?;
(2) Is MCCHI guilty of ULP?;
(3) Were employees illegally dismissed?; and
(4) If their termination was illegal, whether petitioning employees are entitled to separation pay, backwages,
damages and attorneys fees?

SC RULING:
1. Yes. The signatures of 47 out of 88 petitioning employees in the certification against forum shopping
constitute substantial compliance with the rule. When they appealed their case to the CA, they pursued the
same as a collective body, raising only one argument in support of their cause of action, i.e., the illegal dismissal
allegedly committed by MCCHI when union members resorted to strike and mass actions due to MCCHIs
refusal to bargain with officers of the local chapter. Clearly, the CA erred in dropping as parties-petitioners those
who did not sign the certification against forum shopping.

2. No. Art. 248 (g) of the Labor Code, as amended, makes it an unfair labor practice for an employer [t]o
violate the duty to bargain collectively as prescribed by the Code. The applicable provision in this case is
Art. 253 which provides:

ART. 253. Duty to bargain collectively when there exists a collective bargaining agreement.
When there is a collective bargaining agreement, the duty to bargain collectively shall also
mean that neither party shall terminate nor modify such agreement during its
lifetime. However, either party can serve a written notice to terminate or modify the
agreement at least sixty (60) days prior to its expiration date. It shall be the duty of both
parties to keep the status quo and to continue in full force and effect the terms and conditions
of the existing agreement during the 60-day period and/or until a new agreement is reached
by the parties.

Records of the NCMB and DOLE confirmed that NAMA-MCCH-NFL had not registered as a labor
organization, having submitted only its charter certificate as an affiliate or local chapter of NFL. Not
being a legitimate labor organization, NAMA-MCCH-NFL is not entitled to those rights granted to a
legitimate labor organization under Art. 242, specifically:
(a) To act as the representative of its members for the purpose of collective bargaining;
(b) To be certified as the exclusive representative of all the employees in an appropriate
collective bargaining unit for purposes of collective bargaining;

NAMA-MCCH-NFL is not the labor organization certified or designated by the majority of the rank-
and-file hospital employees to represent them in the CBA negotiations but the NFL, as evidenced by
CBAs concluded in 1987, 1991 and 1994. While it is true that a local union has the right to disaffiliate
from the national federation, NAMA-MCCH-NFL has not done so as there was no any effort on its
part to comply with the legal requisites for a valid disaffiliation, Nava and her group simply demanded
that MCCHI directly negotiate with the local union which has not even registered as one. In any case,
NAMA-MCCH-NFL at the time of submission of said proposals was not a duly registered labor
organization; hence it cannot legally represent MCCHIs rank-and-file employees for purposes of
collective bargaining. Hence, NAMA-MCCH-NFL cannot demand from MCCHI the right to bargain
collectively in their behalf. Hence, MCCHIs refusal to bargain then with NAMA-MCCH-NFL cannot be
considered an unfair labor practice to justify the staging of the strike

Art. 263 (b) of the Labor Code, as amended, provides:


ART. 263. Strikes, picketing and lockouts. x x x
(b) Workers shall have the right to engage in concerted activities for purposes of collective
bargaining or for their mutual benefit and protection. The right of legitimate labor
San Beda College of Law 368
4S: 2015 - 2016
LABOR LAW REVIEW Atty. Joyrich Golangco

organizations to strike and picket and of employers to lockout, consistent with the national
interest, shall continue to be recognized and respected. However, no labor union may strike
and no employer may declare a lockout on grounds involving inter-union and intra-union
disputes. x x x x (Emphasis supplied.)

Since NAMA-MCCH-NFL was not a duly registered or an independently registered union at the time it
filed the notice of strike and when it conducted the strike vote. Consequently, the mandatory notice of
strike and the conduct of the strike vote report were ineffective for having been filed and conducted
by NAMA-MCCH-NFL, which has no legal personality as a legitimate labor organization, in violation
of Art. 263 (c), (d) and (f) of the Labor Code and Rule XXII, Book V of the Omnibus Rules
Implementing the Labor Code. Furthermore, the strike was illegal due to the commission of the
following prohibited activities: (1) violence, coercion, intimidation and harassment against non-
participating employees; and (2) blocking of free ingress to and egress from the hospital, including
preventing patients and their vehicles from entering the hospital and other employees from
reporting to work, the putting up of placards with a statement advising incoming patients to proceed
to another hospital because MCCHI employees are on strike/protest. The prolonged work stoppage
and picketing activities of the striking employees severely disrupted hospital operations that MCCHI
suffered heavy financial losses.

3. Yes and No. The consequences of illegal strike to union officers and members are different.

Art. 264 (a) of the Labor Code, as amended, provides for the consequences of an illegal strike to the
participating workers:
x x x Any union officer who knowingly participates in illegal strike and any worker or union officer
who knowingly participates in the commission of illegal acts during a strike may be declared to
have lost his employment status: Provided, That mere participation of a worker in a lawful strike
shall not constitute sufficient ground for termination of his employment, even if a replacement had
been hired by the employer during such lawful strike.

The above provision makes a distinction between workers and union officers who participate in an illegal strike:
an ordinary striking worker cannot be terminated for mere participation in an illegal strike. There must be proof
that he or she committed illegal acts during a strike. A union officer, on the other hand, may be terminated from
work when he knowingly participates in an illegal strike, and like other workers, when he commits an illegal act
during a strike.

Considering their persistence in holding picketing activities despite the declaration by the NCMB that their union
was not duly registered as a legitimate labor organization and the letter from NFLs legal counsel informing that
their acts constitute disloyalty to the national federation, and their filing of the notice of strike and conducting a
strike vote notwithstanding that their union has no legal personality to negotiate with MCCHI for collective
bargaining purposes, there is no question that NAMA-MCCH-NFL officers knowingly participated in the illegal
strike.

With respect to the dismissed union members, although MCCHI submitted photographs taken at the picket line,
it did not individually name those striking employees and specify the illegal act committed by each of
them. Consequently, we find no error committed by the CA in CA-G.R. SP No. 66540 when it modified the
decision of the NLRC and ruled that the dismissal of union members who merely participated in the illegal strike
was illegal.

4. Separation pay only


Since there is no clear proof that union members actually participated in the commission of illegal acts during
the strike, they are not deemed to have lost their employment status as a consequence of a declaration of
illegality of the strike. Considering that 15 years had lapsed from the onset of this labor dispute, and in view of
strained relations that ensued, in addition to the reality of replacements already hired by the hospital which had
apparently recovered from its huge losses, and with many of the petitioners either employed elsewhere, already
old and sickly, or otherwise incapacitated, separation pay without back wages is the appropriate relief. We note
that during the pendency of the cases in this Court, some of the petitioners have entered into compromise
agreements with MCCHI, all of which were duly approved by this Court. Thus, there are some employees who
are excluded from the herein monetary awards are the following petitioners whose compromise agreements
have been approved by this Court.
San Beda College of Law 369
4S: 2015 - 2016
LABOR LAW REVIEW Atty. Joyrich Golangco

San Beda College of Law 370


4S: 2015 - 2016
LABOR LAW REVIEW Atty. Joyrich Golangco

Nature of Collective Bargaining Agreement (CBA)

182. WESLEYAN UNIVERSITY PHILIPPINES v. WESLEYAN UNIVERSITY- PHILIPPINES FACULTY AND


STAFF ASSOCIATION
G.R. No. 181806 March 12, 2014
DEL, CASTILLO, J.:

DOCTRINE:
A Collective Bargaining Agreement (CBA) is a contract entered into by an employer and a legitimate labor
organization concerning the terms and conditions of employment. Like any other contract, it has the force of law
between the parties and, thus, should be complied with in good faith. Unilateral changes or suspensions in the
implementation of the provisions of the CBA, therefore, cannot be allowed without the consent of both parties.

FACTS:
Petitioner Wesleyan University-Philippines is a non-stock, non-profit educational institution duly organized and
existing under the laws of the Philippines. Respondent Wesleyan University-Philippines Faculty and Staff
Association, on the other hand, is a duly registered labor organization acting as the sole and exclusive
bargaining agent of all rank-and-file faculty and staff employees of petitioner.

In December 2003, the parties signed a 5-year CBA effective June 1, 2003 until May 31, 2008.

On August 16, 2005, petitioner, through its President, Atty. Guillermo T. Maglaya (Atty. Maglaya), issued a
Memorandum providing guidelines on the implementation of vacation and sick leave credits as well as vacation
leave commutation.

On August 25, 2005, respondents President, Cynthia L. De Lara (De Lara) wrote a letter to Atty. Maglaya
informing him that respondent is not amenable to the unilateral changes made by petitioner. De Lara questioned
the guidelines for being violative of existing practices and the CBA.

On February 8, 2006, a Labor Management Committee (LMC) Meeting was held during which petitioner advised
respondent to file a grievance complaint on the implementation of the vacation and sick leave policy. In the
same meeting, petitioner announced its plan of implementing a one-retirement policy, which was unacceptable
to respondent.

VA: The assailed University guidelines on the availment of vacation and sick leave credits and vacation leave
commutation are contrary to law. The one retirement policy is contrary to law and is hereby revoked and
rescinded.

CA: The rulings of the VA are supported by substantial evidence and affirmed the decision thereof on that the
Memorandum and the one reirement policy unilaterally amended the CBA without the consent of respondent.

ISSUE: Were the acts of petitioner issuing the memorandum re vacation/sick leaves and announcing the one-
retirement policy valid?

SC RULING:
No. The Non-Diminution Rule found in Article 100 of the Labor Code explicitly prohibits employers from
eliminating or reducing the benefits received by their employees. This rule, however, applies only if the benefit
is based on an express policy, a written contract, or has ripened into a practice. To be considered a practice, it
must be consistently and deliberately made by the employer over a long period of time. Crallawlibrary An
exception to the rule is when the practice is due to error in the construction or application of a doubtful or
difficult question of law. The error, however, must be corrected immediately after its discovery; otherwise, the
rule on Non-Diminution of Benefits would still apply.

In this case, respondent was able to present substantial evidence in the form of affidavits to support its claim
that there are two retirement plans. Based on the affidavits, petitioner has been giving two retirement benefits as
early as 1997. Petitioner, on the other hand, failed to present any evidence to refute the veracity of these
affidavits. There is substantial evidence to prove that there is an existing practice of giving two retirement
benefits, one under the Private Education Retirement Annuity Association (PERAA) Plan and another under the
CBA Retirement Plan.
San Beda College of Law 371
4S: 2015 - 2016
LABOR LAW REVIEW Atty. Joyrich Golangco

As to the Memorandum, Sections 1 and 2 of Article XII of the CBA provide that all covered employees are
entitled to 15 days sick leave and 15 days vacation leave with pay every year and that after the second year of
service, all unused vacation leave shall be converted to cash and paid to the employee at the end of each
school year, not later than August 30 of each year.

The Memorandum, however, states that vacation and sick leave credits are not automatic as leave credits would
be earned on a month-to-month basis. This, in effect, limits the available leave credits of an employee at the
start of the school year. For example, for the first four months of the school year or from June to September, an
employee is only entitled to five days vacation leave and five days sick leave. Considering that the Memorandum
dated August 16, 2005 imposes a limitation not agreed upon by the parties nor stated in the CBA, we agree with
the CA that it must be struck down.

In closing, it may not be amiss to mention that when the provision of the CBA is clear, leaving no doubt on the
intention of the parties, the literal meaning of the stipulation shall govern. However, if there is doubt in its
interpretation, it should be resolved in favor of labor, as this is mandated by no less than the Constitution.

San Beda College of Law 372


4S: 2015 - 2016
LABOR LAW REVIEW Atty. Joyrich Golangco

ARTICLE 245 (now Art. 255) Ineligibility of Managerial Employees to
Join any Labor Organization; Right of Supervisory Employees

183. SMCC v. CHARTER CHEMICAL and COATING CO.


G.R. No. 169717 March 16, 2011
DEL CASTILLO, J.:

DOCTRINE:
The right to file a petition for certification election is accorded to a labor organization provided that it complies
with the requirements of law for proper registration. The inclusion of supervisory employees in a labor
organization seeking to represent the bargaining unit of rank-and-file employees does not divest it of its status
as a legitimate labor organization.

FACTS:
Samahang Manggagawa sa Charter Chemical Solidarity of Unions in the Philippines for Empowerment and
Reforms (SMCC) filed a petition for certification election among the regular rank-and-file employees of Charter
Chemical and Coating Corporation (CHARTER) with the Mediation Arbitration Unit.

CHARTER filed an Answer with Motion to Dismiss on the ground that petitioner union is not a legitimate labor
organization because of (1) failure to comply with the documentation requirements set by law, and (2) the
inclusion of supervisory employees within petitioner union.

MED ARBITERS RULING: Dismissed the petition for certification election. Petitioner union is not a legitimate
labor organization because the Charter Certificate were not executed under oath and certified by the union
secretary and attested to by the union president as required by Section 235 of the Labor Code. As a result, not
being a legitimate labor organization, petitioner union has no right to file a petition for certification election for the
purpose of collective bargaining.

DOLES RULING: Dismissed the SMCCs appeal on the ground that the latters petition for certification election
was filed out of time. Although, contrary to the findings of the Med-Arbiter, that the charter certificate need not
be verified and that there was no independent evidence presented to establish respondent companys claim that
some members of petitioner union were holding supervisory positions

DOLES RULING ON MR: DOLE reversed its earlier ruling. DOLE found that a review of the records indicates
that no certification election was previously conducted in respondent company. There was no obstacle to the
grant of petitioner unions petition for certification election.

CAS RULING: Nullified DOLEs ruling.


1. It gave credence to the findings of the Med-Arbiter that petitioner union failed to comply with the
documentation requirements under the Labor Code
2. Upheld the finding that petitioner union consisted of both rank-and-file and supervisory employee
3. Issues as to the legitimacy of petitioner union may be attacked collaterally in a petition for certification
election and the infirmity in the membership of petitioner union cannot be remedied through the
exclusion-inclusion proceedings in a pre-election conference
4. Considering that petitioner union is not a legitimate labor organization, it has no legal right to file a
petition for certification election.

ISSUES:
1. Is the mixture of rank-and-file and supervisory employees of SMCCs membership is a ground for the
cancellation of SMCCs personality and dismissal of petition for certification election?
2. Is failure to certify under oath the local charter certificate issued by its mother federation and list of the
union membership attending the organizational meeting is a ground for the cancellation of SMCCs legal
personality as a labor organization and for the dismissal of the petition for certification election?
3. May the legal personality of a union be collaterally attacked in the certification election proceedings?

SC RULING:
San Beda College of Law 373
4S: 2015 - 2016
LABOR LAW REVIEW Atty. Joyrich Golangco

1. NO. The mixture of rank-and-file and supervisory employees in petitioner union does not nullify its legal
personality as a legitimate labor organization.

The job descriptions indicate that the aforesaid employees exercise recommendatory managerial
actions which are not merely routinary but require the use of independent judgment, hence, falling within
the definition of supervisory employees under Article 212. The inclusion of the aforesaid supervisory
employees in petitioner union does not divest it of its status as a legitimate labor organization.

R.A. No. 6715 omitted specifying the exact effect any violation of the prohibition [on the co-mingling of
supervisory and rank-and-file employees] would bring about on the legitimacy of a labor organization.

It should be emphasized that the petitions for certification election involved in Toyota and Dunlop
(previous cases where the SC ruled that labor organization had no legal right to file a certification
election to represent a bargaining unit composed of supervisors for as long as it counted rank-and-file
employees among its members) were filed on November 26, 1992 and September 15, 1995,
respectively; hence, the 1989 Rules was applied in both cases.

The Court abandoned the view in Toyota and Dunlop and reverted to its pronouncement in Lopez that
while there is a prohibition against the mingling of supervisory and rank-and-file employees in one labor
organization, the Labor Code does not provide for the effects thereof. Thus, the Court held that after a
labor organization has been registered, it may exercise all the rights and privileges of a legitimate labor
organization. Any mingling between supervisory and rank-and-file employees in its membership cannot
affect its legitimacy for that is not among the grounds for cancellation of its registration, unless such
mingling was brought about by misrepresentation, false statement or fraud under Article 239 of the
Labor Code.

2. NO. The charter certificate need not be certified under oath by the local unions secretary or treasurer
and attested to by its president. [The operative facts in this case occurred in 1999, we shall decide the
issues under the pertinent legal provisions then in force (i.e., R.A. No. 6715)]

The Court ruled that it was not necessary for the charter certificate to be certified and attested by the
local/chapter officers. Id. While this ruling was based on the interpretation of the previous Implementing
Rules provisions which were supplanted by the 1997 amendments, we believe that the same doctrine
obtains in this case. Considering that the charter certificate is prepared and issued by the national union
and not the local/chapter, it does not make sense to have the local/chapters officers x x x certify or
attest to a document which they had no hand in the preparation of.

SMCCs charter certificate need not be executed under oath. Consequently, it validly acquired the
status of a legitimate labor organization upon submission of (1) its charter certificate, (2) the names of
its officers, their addresses, and its principal office, and (3) its constitution and by-laws the last two
requirements having been executed under oath by the proper union officials as borne out by the
records.

3. NO. The legal personality of petitioner union cannot be collaterally attacked by respondent company in
the certification election proceedings.

Except when it is requested to bargain collectively, an employer is a mere bystander to any petition for
certification election; such proceeding is non-adversarial and merely investigative, for the purpose
thereof is to determine which organization will represent the employees in their collective bargaining
with the employer.

San Beda College of Law 374


4S: 2015 - 2016
LABOR LAW REVIEW Atty. Joyrich Golangco

184. CATHAY PACIFIC STEEL CORPORATION v. CA
G.R. No. 164561 August 30, 2006
CHICO-NAZARIO, J.:

DOCTRINE:
Supervisory employees are those who, in the interest of the employer, effectively recommend such managerial
actions, if the exercise of such authority is not merely routinary or clerical in nature but requires the use of
independent judgment; whereas managerial employees are those who are vested with powers or prerogatives
to lay down and execute management policies and/or hire, transfer, suspend, lay off, recall, discharge, assign or
discipline employees.

In this case, being a supervisory employee of CAPASCO, Tamondong cannot be prohibited from joining or
participating in the union activities.

FACTS:
Cathay Pacific Steel Corporation (CAPASCO), hired Tamondong as Assistant to the Personnel Manager for its
Cainta Plant. He was promoted to the position of Personnel/Administrative Officer, and later to that of Personnel
Superintendent.

Supervisory personnel of CAPASCO launched a move to organize a union among their ranks, later known as
private respondent CUSE. Tamondong actively involved himself in the formation of the union and was even
elected as one of its officers after its creation. CAPASCO sent memo to Tamondong requiring him to explain
and to discontinue from his union activities, with a warning that a continuance thereof shall adversely affect his
employment in the company. Tamondong invoked his right as a supervisory employee to join and organize a
labor union. CAPASCO terminated the employment of Tamondong on the ground of loss of trust and
confidence, citing his union activities as acts constituting serious disloyalty to the company.

Tamondong challenged his dismissal for being illegal and as an act involving unfair labor practice.

CAPASCO contended that by virtue of private respondent Tamondongs position as Personnel Superintendent
and the functions actually performed by him in the company, he was considered as a managerial employee,
thus, under the law he was prohibited from joining a union as well as from being elected as one of its officers.

LA: Ruled in favor of Tamondong. Tamondong and CUSE filed a Petition for Certiorari under Rule 65 of the
Rules of Court before the Court of Appeals

CA: Granted the petition

PETITIONERS CONTENTION: Tamondong as Personnel Superintendent of CAPASCO was performing


functions of a managerial employee because he was the one laying down major management policies on
personnel relations such as: issuing memos on company rules and regulations, imposing disciplinary sanctions
such as warnings and suspensions, and executing the same with full power and discretion.

ISSUE: Is Tamondong a supervisory employee?

SC RULING:
YES. The findings of the Court of Appeals that Tamondong was indeed a supervisory employee and not a
managerial employee, thus, eligible to join or participate in the union activities of private respondent CUSE,
were supported by evidence on record.

CA made reference to the Memo which required private respondent Tamondong to observe fixed daily working
hours from 8:00 am to 12:00 noon and from 1:00 pm to 5:00 pm. This imposition upon private respondent
Tamondong, according to the Court of Appeals, is very uncharacteristic of a managerial employee. One of the
essential characteristics of an employee holding a managerial rank is that he is not subjected to the rigid
observance of regular office hours or maximum hours of work. Tamondong may have possessed enormous
powers and was performing important functions that goes with the position of Personnel Superintendent,
nevertheless, there was no clear showing that he is at liberty, by using his own discretion and disposition, to lay
down and execute major business and operational policies for and in behalf of CAPASCO. CAPASCO miserably
failed to establish that Tamondong was authorized to act in the interest of the company using his independent
San Beda College of Law 375
4S: 2015 - 2016
LABOR LAW REVIEW Atty. Joyrich Golangco

judgment. Tamondong may have been exercising certain important powers, such as control and supervision
over erring rank-and-file employees, however, x x x he does not possess the power to hire, transfer, terminate,
or discipline erring employees of the company. At the most, the record merely showed that [private respondent]
Tamondong informed and warned rank-and-file employees with respect to their violations of CAPASCOs rules
and regulations.

Supervisory v. Managerial

Article 212(m) of the Labor Code, as amended, differentiates supervisory employees from managerial
employees, to wit:
1. Supervisory employees are those who, in the interest of the employer, effectively recommend such
managerial actions, if the exercise of such authority is not merely routinary or clerical in nature but
requires the use of independent judgment;
2. Managerial employees are those who are vested with powers or prerogatives to lay down and execute
management policies and/or hire, transfer, suspend, lay off, recall, discharge, assign or discipline
employees.

San Beda College of Law 376


4S: 2015 - 2016
LABOR LAW REVIEW Atty. Joyrich Golangco

ARTICLE 247-249 (now Art. 258-260) Unfair Labor Practices

185. GENERAL SANTOS COCA-COLA PLANT FREE WORKERS UNION-TUPAS v. COCA-COLA


BOTTLERS PHILS. INC.
G.R. No. 178647 February 13, 2009
NACHURA, J.:

DOCTRINE:
Unfair labor practice refers to acts that violate the workers right to organize. The prohibited acts are related to
the workers right to self-organization and to the observance of a CBA. Without that element, the acts, even if
unfair, are not unfair labor practices.

FACTS:
Coca-Cola Bottlers Phil., Inc. (CCBPI) experienced a significant decline in profitability due to the Asian
economic crisis, decrease in sales, and tougher competition. To curb the negative effects on the company, it
implemented three (3) waves of an Early Retirement Program.

There was an inter-office memorandum mandating them to put on hold all requests for hiring to fill in vacancies
in both regular and temporary positions in the Head Office and in the Plants. Because several employees
availed of the early retirement program, vacancies were created in some departments, including the production
department of CCBPI Gen San, where members of petitioner Union worked.

Faced with the freeze hiring directive, CCBPI Gen San engaged the services of JLBP Services Corporation
(JLBP), a company in the business of providing labor and manpower services, including janitorial services,
messengers, and office workers to various private and government offices.

Petitioner filed with the National Conciliation and Mediation Board (NCMB) a Notice of Strike on the ground of
alleged unfair labor practice committed by CCBPI Gen San for contracting-out services regularly performed by
union members (union busting). Parties failed to come to an amicable settlement.

NLRCS RULING: CCBPI was not guilty of unfair labor practice for contracting out jobs to JLBP. The NLRC
anchored its ruling on the validity of the Going-to-the-Market (GTM) system implemented by the company, which
called for restructuring its selling and distribution system, leading to the closure of certain sales offices and the
elimination of conventional sales routes.

CAS RULING: Upheld the NLRCs finding that CCBPI was not guilty of unfair labor practice. It found that JLBP
was an independent contractor and that the decision to contract out jobs was a valid exercise of management
prerogative to meet exigent circumstances.

ISSUES:
1. Whether JLBP is an independent contractor
2. Whether CCBPIs contracting-out of jobs to JLBP amounted to unfair labor practice

SC RULING: The petition is bereft of merit.

1. Yes. It is true that the NLRC erroneously concluded that the contracting- out of jobs in CCBPI Gen San
was due to the GTM system. However, this does not diminish the NLRCs finding that JLBP was a
legitimate, independent contractor and that CCBPI Gen San engaged the services of JLBP to meet
business exigencies created by the freeze-hiring directive of the CCBPI Head Office.
2. No. The companys action to contract-out the services and functions performed by Union members did
not constitute unfair labor practice as this was not directed at the members right to self-organization.

ART. 248. UNFAIR LABOR PRACTICE OF EMPLOYERS. It shall be unlawful for an employer to
commit any of the following unfair labor practices:
(c) To contract out services or functions being performed by union members when such will interfere
with, restrain or coerce employees in the exercise of their right to self-organization;

San Beda College of Law 377


4S: 2015 - 2016
LABOR LAW REVIEW Atty. Joyrich Golangco

Unfair labor practice refers to acts that violate the workers right to organize. The prohibited acts are
related to the workers right to self-organization and to the observance of a CBA. Without that
element, the acts, even if unfair, are not unfair labor practices.

San Beda College of Law 378


4S: 2015 - 2016
LABOR LAW REVIEW Atty. Joyrich Golangco

186. UST FACULTY UNION v. UNIVERSITY OF SANTO TOMAS, ET AL
G.R. No. 180892 April 7, 2009
VELASCO, JR., J.:

DOCTRINE:
The onus probandi falls on the shoulders of petitioner to establish or substantiate such claims by the requisite
quantum of evidence. In labor cases as in other administrative proceedings, substantial evidence or such
relevant evidence as a reasonable mind might accept as sufficient to support a conclusion is required. In the
petition at bar, petitioner miserably failed to adduce substantial evidence as basis for the grant of relief.

FACTS:
University of Santo Tomas Faculty Union (USTFU) wrote a letter to all its members informing them of a General
Assembly (GA) that was to be held on October 5, 1996. The then incumbent president of the USTFU was Atty.
Eduardo J. Mario, Jr. The letter contained an agenda for the GA which included an election of officers.

Secretary General of the UST, issued a Memorandum allowing the request of the Faculty Clubs of the university
to hold a convocation. Members of the faculties of the university attended the convocation, including members
of the USTFU, without the participation of the members of the UST administration.

During the convocation, an election for the officers of the USTFU was conducted by a group called the
Reformist Alliance. Upon learning that the convocation was intended to be an election, members of the USTFU
walked out. Meanwhile, an election was conducted among those present.

Gil Gamilla and other faculty members (Gamilla Group) were elected as the president and officers, respectively,
of the union. Thus, there were two (2) groups claiming to be the USTFU: the Gamilla Group and the group led
by Atty. Mario, Jr. (Mario Group).

Mario Group filed a complaint for ULP against the UST with the Arbitration Branch. It also filed a complaint with
the Office of the Med-Arbiter of the Department of Labor and Employment (DOLE), praying for the nullification of
the election of the Gamilla Group as officers of the USTFU.

Collective Bargaining Agreement (CBA) was entered into by the Gamilla Group and the UST. The CBA
superseded an existing CBA entered into by the UST and USTFU

Gamilla, accompanied by the barangay captain in the area padlocked the office of the USTFU. Afterwards, an
armed security guard of the UST was posted in front of the USTFU office.

MED-ARBITER: election of the Gamilla group as null and void and ordering that this group cease and desist
from performing the duties and responsibilities of USTFU officers.

ARBITRATION BRANCH: dismissed the complaint for lack of merit.

The acts of UST which USTFU complained of as ULP were the following: (1) allegedly calling for a convocation
of faculty members which turned out to be an election of officers for the faculty union; (2) subsequently dealing
with the Gamilla Group in establishing a new CBA; and (3) the assistance to the Gamilla Group in padlocking
the USTFU office.

LABOR ARBITERS RULING: He explained that the alleged Memorandum dated October 2, 1996 merely
granted the request of faculty members to hold such convocation. By USTFUs own admission, no member of
the UST administration attended or participated in the convocation.

As to the CBA, the labor arbiter ruled that when the new CBA was entered into, (1) the Gamilla Group presented
more than sufficient evidence to establish that they had been duly elected as officers of the USTFU; and (2) the
ruling of the med-arbiter that the election of the Gamilla Group was null and void was not yet final and
executory.

ISSUE: Whether herein respondents are guilty of Unfair Labor Practice despite abundance of evidence showing
that Unfair Labor Practices were indeed committed.

San Beda College of Law 379


4S: 2015 - 2016
LABOR LAW REVIEW Atty. Joyrich Golangco

SC RULING:
No. UST is not guilty of ULP.

Petitioner claims that respondents violated paragraphs (a) and (d) of Art. 248 of the Code which provide:

Article 248. Unfair labor practices of employers.It shall be unlawful for an employer to commit any of the
following unfair labor practices:
(a) To interfere with, restrain or coerce employees in the exercise of their right to self-organization;
xxxx
(d) To initiate, dominate, assist or otherwise interfere with the formation or administration of any labor
organization, including the giving of financial or other support to it or its organizers or supporters.

The general principle is that one who makes an allegation has the burden of proving it. While there are
exceptions to this general rule, in the case of ULP, the alleging party has the burden of proving such ULP. Such
principle finds justification in the fact that ULP is punishable with both civil and/or criminal sanctions

In order to show that the employer committed ULP under the Labor Code, substantial evidence is required to
support the claim. Substantial evidence has been defined as such relevant evidence as a reasonable mind
might accept as adequate to support a conclusion.

In no way can the contents of the memorandum be interpreted to mean that faculty members were required to
attend the convocation. Not one coercive term was used in the memorandum to show that the faculty club
members were compelled to attend such convocation. And the phrase "we are allowing them to hold a
convocation" negates any idea that the UST would participate in the proceedings.

The Gamilla Group was not validly elected into office, there was no reason to believe that the members of the
Gamilla Group were not the validly elected officers and directors of USTFU.

As to the padlocking of the USTFU office, it must be emphasized that based on the Certification of Sibug,
Cardenas was merely present, with Brgy. Captain, at the padlocking of the USTFU office. The Certification also
stated that Sibug himself also padlocked the USTFU office and that he was neither harassed nor coerced by the
padlocking group. Clearly, Cardenas mere presence cannot be equated to a positive act of "aiding" the Gamilla
Group in securing the USTFU office.

Petitioner, however, fails to enumerate such objectionable actions of the UST. Again, petitioner fails to present
substantial evidence in support of its claim.

San Beda College of Law 380


4S: 2015 - 2016
LABOR LAW REVIEW Atty. Joyrich Golangco

187. PHILIPPINE SKYLANDERS, INC, ET AL v. NLRC, ET AL
G.R. No. 127374 January 31, 2002
BELLOSILLO, J.:

FACTS:
Philippine Skylanders Employees Association (PSEA), a local labor union affiliated with the Philippine
Association of Free Labor Unions (PAFLU) September (PAFLU), won in the certification election conducted
among the rank and file employees of Philippine Skylanders, Inc. (PSI).

Its rival union, Philippine Skylanders Employees Association-WATU (PSEA-WATU) immediately protested the
result of the election before the Secretary of Labor.

PSEA sent PAFLU a notice of disaffiliation citing as reason PAFLU's supposed deliberate and habitual
dereliction of duty toward its members. Attached to the notice was a copy of the resolution adopted and signed
by the officers and members of PSEA authorizing their local union to disaffiliate from its mother federation.

PSEA subsequently affiliated itself with the National Congress of Workers (NCW), changed its name to
Philippine Skylanders Employees Association - National Congress of Workers (PSEA-NCW), and to maintain
continuity within the organization, allowed the former officers of PSEA-PAFLU to continue occupying their
positions as elected officers in the newly-forged PSEA-NCW.

PSEA-NCW entered into a collective bargaining agreement with PSI which was immediately registered with the
Department of Labor and Employment.

PAFLU Secretary General Serafin Ayroso wrote Mariles C. Romulo requesting a copy of PSI's audited financial
statement. Ayroso explained that with the dismissal of PSEA-WATUs election protest the time was ripe for the
parties to enter into a collective bargaining agreement. PSI denied the request citing as reason PSEA's
disaffiliation from PAFLU and its subsequent affiliation with NCW.

PAFLU filed a complaint for unfair labor practice against PSI. PAFLU alleged that aside from PSIs refusal to
bargain collectively with its workers, the company through its president and personnel manager, was also liable
for interfering with its employees' union activities. In another complaint, PAFLU claimed that Dakila was present
in PSEA's organizational meeting thereby confirming his illicit participation in union activities. Ayroso added that
the members of the local union had unwittingly fallen into the manipulative machinations of PSI and were lured
into endorsing a collective bargaining agreement which was detrimental to their interests. These two were
consolidated.

PSEA-NCW took the cudgels for its officers who were being sued in their capacities as former officers of PSEA-
PAFLU and asserted that since PSEA was no longer affiliated with PAFLU, Ayroso or PAFLU for that matter had
no personality to file the instant complaint. In support of this assertion.

LABOR ARBITERS RULING: declared PSEA's disaffiliation from PAFLU invalid and held PSI, PSEA-PAFLU
and their respective officers guilty of unfair labor practice. According to the Labor Arbiter, was a classic case of
interference for which PSI could be held responsible. Its collective bargaining agreement with PSI was struck
down for being invalid.

NLRC: Upheld LA

NOTE: The issue of disaffiliation is an inter-union conflict the jurisdiction of which properly lies with the Bureau
of Labor Relations (BLR) and not with the Labor Arbiter. Nonetheless, with due recognition of this fact, we deem
it proper to settle the controversy at this instance since to remand the case to the BLR would only mean
intolerable delay for the parties.

ISSUE: May PSEA, which is an independent and separate local union, validly disaffiliate from PAFLU pending
the settlement of an election protest questioning its status as the sole and exclusive bargaining agent of PSI's
rank and file employees?

San Beda College of Law 381


4S: 2015 - 2016
LABOR LAW REVIEW Atty. Joyrich Golangco

SC RULING:
YES. The pendency of an election protest involving both the mother federation and the local union did not
constitute a bar to a valid disaffiliation. There is nothing shown in the records nor is it claimed by PAFLU that the
local union was expressly forbidden to disaffiliate from the federation nor were there any conditions imposed for
a valid breakaway.

We upheld the right of local unions to separate from their mother federation on the ground that as separate and
voluntary associations, local unions do not owe their creation and existence to the national federation to which
they are affiliated but, instead, to the will of their members. The sole essence of affiliation is to increase, by
collective action, the common bargaining power of local unions for the effective enhancement and protection of
their interests. Admittedly, there are times when without succor and support local unions may find it hard,
unaided by other support groups, to secure justice for themselves.

Yet the local unions remain the basic units of association, free to serve their own interests subject to the
restraints imposed by the constitution and by-laws of the national federation, and free also to renounce the
affiliation upon the terms laid down in the agreement which brought such affiliation into existence.

Neither was it disputed by PAFLU that 111 signatories out of the 120 members of the local union, or an
equivalent of 92.5% of the total union membership supported the claim of disaffiliation and had in fact
disauthorized PAFLU from instituting any complaint in their behalf. Surely, this is not a case where one (1) or
two (2) members of the local union decided to disaffiliate from the mother federation, but it is a case where
almost all local union members decided to disaffiliate.

It was entirely reasonable then for PSI to enter into a collective bargaining agreement with PSEA-NCW. As
PSEA had validly severed itself from PAFLU, there would be no restrictions which could validly hinder it from
subsequently affiliating with NCW and entering into a collective bargaining agreement in behalf of its members.

The mere act of disaffiliation did not divest PSEA of its own personality; neither did it give PAFLU the license to
act independently of the local union. Recreant to its mission, PAFLU cannot simply ignore the demands of the
local chapter and decide for its welfare. PAFLU might have forgotten that as an agent it could only act in
representation of and in accordance with the interests of the local union.

Petitions of Philippine Skylanders, Inc. and of Philippine Skylanders and Workers Association-NCW, together
with their respective officers, were GRANTED.

San Beda College of Law 382


4S: 2015 - 2016
LABOR LAW REVIEW Atty. Joyrich Golangco

Art. 263-264

EDEN GLADYS ABARIA, ET AL., vs. NLRC, ET AL.


G.R. No. 154113 December 7, 2011
VILLARAMA, JR., J.:

DOCTRINE:
ART. 263 provides that in cases of bargaining deadlocks, the duly certified or recognized bargaining
agent may file a notice of strike or the employer may file a notice of lockout with the Department at least 30
days before the intended date thereof. In cases of unfair labor practice, the period of notice shall be 15 days
and in the absence of a duly certified or recognized bargaining agent, the notice of strike may be filed by
any legitimate labor organization in behalf of its members.

Section 6, Rule XXII, Book V of the Omnibus Rules Implementing the Labor Code reads that any certified or
duly recognized bargaining representative may declare a strike in cases of bargaining deadlocks and unfair
labor practices. The employer may declare a lockout in the same cases. In the absence of a certified or duly
recognized bargaining representative, any legitimate labor organization in the establishment may declare a
strike but only on grounds of unfair labor practice.

FACTS:
Perla Nava, the President of NAMA-MCCH-NFL, wrote to the Hospital Administrator, expressing the unions
desire to renew the CBA. However, MCCHI returned the CBA proposal for Nava to secure first the endorsement
of the legal counsel of NFL as the official bargaining representative of MCCHI employees.

Atty. Alforque (legal counsel of NFL) informed MCCHI that the proposed CBA submitted by Nava was never
referred to NFL and that NFL has not authorized any other legal counsel or any person for collective bargaining
negotiations.

Thereafter, MCCHI attempted to take over the room being used as union office but was prevented to do so by
Nava and her group who protested these actions and insisted that management directly negotiate with them for
a new CBA. MCCHI referred the matter to Atty. Alforque, NFLs Regional Director, and advised Nava that their
group is not recognized by NFL.

In his letter addressed to Nava et al, Atty. Alforque suspended their union membership for serious violation of
the Constitution and By-Laws.

The next day, several union members led by Nava and her group launched a series of mass actions such as
wearing black and red armbands/headbands, marching around the hospital premises and putting up placards,
posters and streamers. Atty. Alforque immediately disowned the concerted activities being carried out by union
members which are not sanctioned by NFL. MCCHI directed the union officers led by Nava to submit within 48
hours a written explanation why they should not be terminated for having engaged in illegal concerted activities
amounting to strike, and placed them under immediate preventive suspension. Responding to this directive,
Nava and her group denied there was a temporary stoppage of work, explaining that employees wore their
armbands only as a sign of protest and reiterating their demand for MCCHI to comply with its duty to bargain
collectively. Rev. Iyoy, having been informed that Nava and her group have also been suspended by NFL,
directed said officers to appear before his office for investigation in connection with the illegal strike wherein they
reportedly uttered slanderous and scurrilous words against the officers of the hospital, threatening other workers
and forcing them to join the strike. Said union officers, however, invoked the grievance procedure provided in
the CBA to settle the dispute between management and the union.

The DOLE Regional Office No. 7 issued certifications stating that there is nothing in their records which shows
that NAMA-MCCH-NFL is a registered labor organization, and that said union submitted only a copy of its
Charter Certificate. MCCHI then sent individual notices to all union members asking them to submit within 72
hours a written explanation why they should not be terminated for having supported the illegal concerted
activities of NAMA-MCCH-NFL which has no legal personality as per DOLE records. In their collective
response/statement dated March 18, 1996, it was explained that the picketing employees wore armbands to
protest MCCHIs refusal to bargain; it was also contended that MCCHI cannot question the legal personality of
the union which had actively assisted in CBA negotiations and implementation.
San Beda College of Law 383
4S: 2015 - 2016
LABOR LAW REVIEW Atty. Joyrich Golangco

NAMA-MCCH-NFL filed a Notice of Strike but the same was deemed not filed for want of legal personality on
the part of the filer. Despite such rebuff, Nava and her group still conducted a strike vote during which an
overwhelming majority of union members approved the strike.

MCCHI issued a cease-and-desist order to the rest of the striking employees stressing that the wildcat
concerted activities spearheaded by the Nava group is illegal without a valid Notice of Strike and warning them
that non-compliance will compel management to impose disciplinary actions against them. For their continued
picketing activities despite the said warning, more than 100 striking employees were dismissed. Unfazed, the
striking union members held more mass actions. The means of ingress to and egress from the hospital were
blocked so that vehicles carrying patients and employees were barred from entering the premises.

With the volatile situation adversely affecting hospital operations and the condition of confined patients, MCCHI
filed a petition for injunction in the NLRC. A TRO was issued. MCCHIs petition was granted and a permanent
injunction was issued enjoining the Nava group from committing illegal acts mentioned in Article 264 of the
Labor Code.

The City Government of Cebu ordered the demolition of the structures and obstructions put up by the picketing
employees of MCCHI along the sidewalk, having determined the same as a public nuisance or nuisance per se.

Thereafter, several complaints for illegal dismissal and unfair labor practice were filed by the terminated
employees against MCCHI, Rev. Iyoy, UCCP and members of the Board of Trustees of MCCHI.

RULING OF THE LA:


Executive Labor Arbiter Belarmino dismissed the complaints for unfair labor practice filed by Nava and 90 other
complainants and found no basis for the charge of unfair labor practice and declared the strike and picketing
activities illegal having been conducted by NAMA-MCCH-NFL which is not a legitimate labor organization.

RULING OF THE NLRC:


The NLRC affirmed the decision of the LA.

ISSUE: Were the strike and picketing activities conducted by union officers and members illegal?

RULING: YES. Art. 263 (b) of the Labor Code, as amended, provides that workers shall have the right to
engage in concerted activities for purposes of collective bargaining or for their mutual benefit and protection.
However, no labor union may strike and no employer may declare a lockout on grounds involving inter-union
and intra-union disputes.

The same provision also provides that in cases of bargaining deadlocks, the duly certified or recognized
bargaining agent may file a notice of strike or the employer may file a notice of lockout with the Department at
least 30 days before the intended date thereof. In cases of unfair labor practice, the period of notice shall be 15
days and in the absence of a duly certified or recognized bargaining agent, the notice of strike may be
filed by any legitimate labor organization in behalf of its members.

Section 6, Rule XXII, Book V of the Omnibus Rules Implementing the Labor Code reads that any certified or
duly recognized bargaining representative may declare a strike in cases of bargaining deadlocks and unfair
labor practices. The employer may declare a lockout in the same cases. In the absence of a certified or duly
recognized bargaining representative, any legitimate labor organization in the establishment may declare a
strike but only on grounds of unfair labor practice.

As borne by the records, NAMA-MCCH-NFL was not a duly registered or an independently registered union at
the time it filed the notice of strike and when it conducted the strike vote. It could not then legally represent the
union members. Consequently, the mandatory notice of strike and the conduct of the strike vote report were
ineffective for having been filed and conducted by NAMA-MCCH-NFL which has no legal personality as a
legitimate labor organization, in violation of Art. 263 (c), (d) and (f) of the Labor Code and Rule XXII, Book V of
the Omnibus Rules Implementing the Labor Code. Furthermore, the strike was illegal due to the commission of
prohibited activities.

San Beda College of Law 384


4S: 2015 - 2016
LABOR LAW REVIEW Atty. Joyrich Golangco

TROPICAL HUT EMPLOYEES UNION- CGW, ET AL. v. TROPICAL HUT FOOD MARKET, INC.
G.R. No. L-43495
January 20, 1990

DISAFFILIATION FROM A LABOR FEDERATION

DOCTRINE: A local union, being a separate and voluntary association, is free to serve the interest of all its
members including the freedom to disaffiliate when circumstances warrant. this right is consistent with the
constitutional guarantee of freedom of association

FACTS: The rank and file workers of the Tropical Hut Food Market Incorporated organized a local union called
the Tropical Hut Employees Union (THEU), elected their officers, adopted their constitution and by-laws and
immediately sought affiliation with the National Association of Trade Unions (NATU). NATU accepted the THEU
application for affiliation. A Registration Certificate was issued by DOLE in the name of the Tropical Hut
Employees Union NATU. It appears, however, that NATU itself as a labor federation, was not registered.

A CBA was concluded between the parties which contained these clear and unequivocal terms:

Sec. 1. The COMPANY recognizes the UNION as the sole and exclusive collective bargaining agent for all its
workers and employees in all matters concerning wages, hours of work, and other terms and conditions of
employment.

Union Membership and Union Check-off


Sec. 1 . . . Employees who are already members of the UNION at the time of the signing of this Agreement or
who become so thereafter shall be required to maintain their membership therein as a condition of continued
employment.

Sec. 3Any employee who is expelled from the UNION for joining another federation or forming another union,
or who fails or refuses to maintain his membership therein as required, . . . shall, upon written request of the
UNION be discharged by the COMPANY.

NATU received a letter that THEU was disaffiliating from the NATU federation and its affiliation with the
Confederation of General Workers (CGW). The general membership of the so-called THEU-CGW held its
annual election of officers, with Jose Encinas elected as President. Encinas, in his capacity as THEU-CGW
President, informed the respondent company of the result of the elections. Pacifico Rosal, President of the
CGW, wrote a letter in behalf of complainant THEU-CGW to the respondent company demanding the remittance
of the union dues collected by the Tropical Hut to the THEU-CGW, but this was refused by the respondent
company.

NATU VP Lontok, Jr., informed Encinas in a letter, concerning the request made by the NATU federation to the
respondent company to dismiss Encinas in view of his violation of Section 3 of Article III of the CBA. Encinas
was also advised in the letter that NATU was returning the letter of disaffiliation.

In view of NATU's request, the respondent company suspended Encinas pending the application for clearance
with the DOLE to dismiss him. Members of the THEU-CGW passed a resolution protesting the suspension of
Encinas and reiterated their ratification and approval of their union's disaffiliation from NATU and their affiliation
with the CGW.

Upon the request of NATU, respondent company applied for clearance with the Secretary of Labor to dismiss
the other officers and members of THEU-CGW. The company also suspended them effective that day.
Petitioner THEU-CGW wrote a letter to Juan Ponce Enrile, Secretary of National Defense, complaining of the
unfair labor practices committed by respondent company against its members and requesting assistance on the
matter. The aforementioned letter contained the signatures of one hundred forty-three (143) members.

Secretary of THEU-NATU, notified the entire rank and file employees of the company that they will be given
forty-eight (48) hours upon receipt of the notice within which to answer and affirm their membership with THEU-
NATU. When the petitioner employees failed to reply, Arturo Dilag advised them thru letters that the THEU-
NATU shall enforce the union security clause set forth in the CBA, and that he had requested respondent
company to dismiss them.
San Beda College of Law 385
4S: 2015 - 2016
LABOR LAW REVIEW Atty. Joyrich Golangco

Respondent company, thereafter, wrote the petitioner employees demanding the latter's comment on Dilag's
charges before action was taken thereon. However, no comment or reply was received from petitioners. In view
of this, Estelita Que, President/General Manager of respondent company, upon Dilag's request, suspended
twenty four (24) workers on March 5, 1974, another thirty seven (37) on March 8, 1974 and two (2) more on
March 11, 1974, pending approval by the Secretary of Labor of the application for their dismissal.

As a consequence thereof, petitioners filed a case of ULP against Tropical Hut Food Market, Incorporated,
Estelita Que, Hernando Sarmiento and Arturo Dilag.

LA DECISION: LA issued an order holding that the issues raised by the parties became moot and academic
with the issuance of NLRC Order dated February 25, 1974, which directed the holding of a certification election
among the rank and file workers of the respondent company between the THEU-NATU and THEU-CGW. He
also ordered: a) the reinstatement of all complainants; b) for the respondent company to cease and desist from
committing further acts of dismissals without previous order from the NLRC and for the complainant THEU-
CGW to file representation cases on a case to case basis during the freedom period provided for by the existing
CBA between the parties.

NLRC DECISION: Ordered respondent company under pain of being cited for contempt for failure to do so, to
give the individual complainants a second chance by reemploying them upon their voluntary reaffirmation of
membership and loyalty to the THEU-NATU and the National Association of Trade Union in the event it hires
additional personnel.

ISSUE/S:
(3) Is the disaffiliation of the local union from the nation federation valid?
(4) Was the dismissal of the petitioner employees resulting from their unions disaffiliation from the mother union
illegal and constituted ULP?

RULING: The right of a local union to disaffiliate from its mother federation is well-settled. A local union, being a
separate and voluntary association, is free to serve the interest of all its members including the freedom to
disaffiliate when circumstances warrant. This right is consistent with the constitutional guarantee of freedom of
association (Volkschel Labor Union v. Bureau of Labor Relations, No. L-45824, June 19, 1985, 137 SCRA 42).

All employees enjoy the right to self organization and to form and join labor organizations of their own choosing
for the purpose of collective bargaining and to engage in concerted activities for their mutual aid or protection.
This is a fundamental right of labor that derives its existence from the Constitution. In interpreting the protection
to labor and social justice provisions of the Constitution and the labor laws or rules or regulations, We have
always adopted the liberal approach which favors the exercise of labor rights.

There is nothing in the constitution of the NATU or in the constitution of the THEU-NATU that the THEU was
expressly forbidden to disaffiliate from the federation (pp. 62, 281, Rollo), The alleged non-compliance of the
local union with the provision in the NATU Constitution requiring the service of three months notice of intention
to withdraw did not produce the effect of nullifying the disaffiliation for the following grounds: firstly, NATU was
not even a legitimate labor organization, it appearing that it was not registered at that time with the Department
of Labor, and therefore did not possess and acquire, in the first place, the legal personality to enforce its
constitution and laws, much less the right and privilege under the Labor Code to organize and affiliate chapters
or locals within its group, and secondly, the act of non-compliance with the procedure on withdrawal is premised
on purely technical grounds which cannot rise above the fundamental right of self-organization.

Further, there is no merit in the contention of the respondents that the act of disaffiliation violated the union
security clause of the CBA and that their dismissal as a consequence thereof is valid. A perusal of the collective
bargaining agreements shows that the THEU-NATU, and not the NATU federation, was recognized as the sole
and exclusive collective bargaining agent for all its workers and employees in all matters concerning wages,
hours of work and other terms and conditions of employment. Although NATU was designated as the sole
bargaining agent in the check-off authorization form attached to the CBA, this simply means it was acting only
for and in behalf of its affiliate. The NATU possessed the status of an agent while the local union remained the
basic principal union which entered into contract with the respondent company. When the THEU disaffiliated
from its mother federation, the former did not lose its legal personality as the bargaining union under the CBA.
Moreover, the union security clause embodied in the agreements cannot be used to justify the dismissals meted
San Beda College of Law 386
4S: 2015 - 2016
LABOR LAW REVIEW Atty. Joyrich Golangco

to petitioners since it is not applicable to the circumstances obtaining in this case. The CBA imposes dismissal
only in case an employee is expelled from the union for joining another federation or for forming another union
or who fails or refuses to maintain membership therein. The case at bar does not involve the withdrawal of
merely some employees from the union but of the whole THEU itself from its federation. Clearly, since there is
no violation of the union security provision in the CBA, there was no sufficient ground to terminate the
employment of petitioners.

Finally, with regard to the process by which the workers were suspended or dismissed, this Court finds that it
was hastily and summarily done without the necessary due process. The respondent company sent a letter to
petitioners herein, advising them of NATU/Dilag's recommendation of their dismissal and at the same time
giving them forty-eight (48) hours within which to comment. When petitioners failed to do so, respondent
company immediately suspended them and thereafter effected their dismissal. This is certainly not in fulfillment
of the mandate of due process, which is to afford the employee to be dismissed an opportunity to be heard.

San Beda College of Law 387


4S: 2015 - 2016
LABOR LAW REVIEW Atty. Joyrich Golangco

PUREFOODS CORPORATION vs. NAGKAKAISANG SAMAHANG MANGGAGAWA NG PUREFOODS
RANK-AND-FILE, ST. THOMAS FREE WORKERS UNION, PUREFOODS GRANDPARENT FARM
WORKERS UNION and PUREFOODS UNIFIED LABOR ORGANIZATIO

FACTS: Three labor organizations and a federation are respondents in this caseNagkakaisang Samahang
Manggagawa Ng Purefoods Rank-And-File (NAGSAMA-Purefoods), the exclusive bargaining agent of the rank-
and-file workers of Purefoods meat division throughout Luzon; St. Thomas Free Workers Union (STFWU), of
those in the farm in Sto. Tomas, Batangas; and Purefoods Grandparent Farm Workers Union (PGFWU), of
those in the poultry farm in Sta. Rosa, Laguna. These organizations were affiliates of the respondent federation,
1
Purefoods Unified Labor Organization (PULO).

On February 8, 1995, NAGSAMA-Purefoods manifested to petitioner corporation its desire to re-negotiate CBA
th
then due to expire on the 28 of the said month. Together with its demands and proposal, the organization
submitted to the company its January 28, 1995 General Membership Resolution approving and supporting the
unions affiliation with PULO, adopting the draft CBA proposals of the federation, and authorizing a negotiating
panel which included among others a PULO representative. While Purefoods formally acknowledged receipt of
the unions proposals, it refused to recognize PULO and its participation, even as a mere observer, in the
negotiation. Consequently, notwithstanding the PULO representatives non-involvement, the negotiation of the
terms of the CBA still resulted in a deadlock. A notice of strike was then filed by NAGSAMA-Purefoods. The
deadlock issues were settled except the matter of the companys recognition of the unions affiliation with
2
PULO.

In the meantime, STFWU and PGFWU also submitted their respective proposals for CBA renewal, and their
general membership resolutions which, among others, affirmed the two organizations affiliation with PULO.
Consistent with its stance, Purefoods refused to negotiate with the unions should a PULO representative be in
3
the panel. The parties then agreed to postpone the negotiations indefinitely.

On July 24, 1995, however, the petitioner company concluded a new CBA with another union in its farm in
Malvar, Batangas. Five days thereafter, or on July 29, 1995, four company employees facilitated the transfer of
around 23,000 chickens from the poultry farm in Sto. Tomas, Batangas (where STFWU was the exclusive
bargaining agent) to that in Malvar. The following day, the regular rank-and-file workers in the Sto. Tomas farm
were refused entry in the company premises; and on July 31, 1995, 22 STFWU members were terminated from
employment. The farm manager, supervisors and electrical workers of the Sto. Tomas farm, who were members
4
of another union, were nevertheless retained by the company in its employ.

Aggrieved by these developments, the four respondent labor organizations jointly instituted a complaint for
unfair labor practice (ULP), illegal lockout/dismissal and damages.

LA DECISION: Dismissed the complaint, and declaring that the company neither committed ULP nor illegally
dismissed the employees.

NLRC DECISION: Reversed the ruling of the LA, ordered the payment of P500,000.00 as moral and exemplary
damages and the reinstatement with full backwages of the STFWU members. It ruled that the petitioner
companys refusal to recognize the labor organizations affiliation with PULO was unjustified considering that the
latter had been granted the status of a federation by the Bureau of Labor Relations; and that this refusal
constituted undue interference in, and restraint on the exercise of the employees right to self-organization and
free collective bargaining.

CA DECISION: Petition for Certiorari was dismissed outright on the ground that the verification and certification
of non-forum shopping was defective since no proof of authority to act for and on behalf of the corporation was
submitted by the corporations senior vice-president who signed the same.

ISSUE/S: Are the employees entitled to award of damages?

RULING:
It is crystal clear that the closure of the Sto. Tomas farm was made in bad faith. Badges of bad faith are evident
from the following acts of the petitioner: it unjustifiably refused to recognize the STFWUs and the other unions
affiliation with PULO; it concluded a new CBA with another union in another farm during the agreed indefinite
suspension of the collective bargaining negotiations; it surreptitiously transferred and continued its business in a
San Beda College of Law 388
4S: 2015 - 2016
LABOR LAW REVIEW Atty. Joyrich Golangco

less hostile environment; and it suddenly terminated the STFWU members, but retained and brought the non-
members to the Malvar farm. Petitioner presented no evidence to support the contention that it was incurring
losses or that the subject farms lease agreement was pre-terminated. Ineluctably, the closure of the Sto. Tomas
farm circumvented the labor organizations right to collective bargaining and violated the members right to
19
security of tenure.

We deem as proper the award of moral and exemplary damages. We hold that the sudden termination of the
STFWU members is tainted with ULP because it was done to interfere with, restrain or coerce employees in the
exercise of their right to self-organization. Thus, the petitioner company is liable for the payment of the aforesaid
22
damages. Notable, though, is that this award, while stated in the body of the NLRC decision, was omitted in
the dispositive portion of the said ruling. To prevent any further confusion in the implementation of the said
decision, we correct the dispositive portion of the ruling to include the payment of P500,000.00 as moral and
exemplary damages to the illegally dismissed STFWU members.

As to the order of reinstatement, the Court modifies the same in that if it is no longer feasible considering the
length of time that the employees have been out of petitioners employ, the company is ordered to pay the
illegally dismissed STFWU members separation pay equivalent to one (1) month pay, or one-half (1/2) month
pay for every year of service, whichever is higher.
25
The releases and quitclaims, as well as the affidavits of desistance, signed by the concerned employees, who
were then necessitous men at the time of execution of the documents, are declared invalid and ineffective. They
will not bar the workers from claiming the full measure of benefits flowing from their legal rights.

San Beda College of Law 389


4S: 2015 - 2016
LABOR LAW REVIEW Atty. Joyrich Golangco

DLSU vs DLSUEA
G.R. No. 177283
April 7, 2009

FACTS: In 2001, a splinter group of respondent led by one Belen Aliazas (Aliazas group) filed a petition for
conduct of elections with the Department of Labor and Employment (DOLE), alleging that the then incumbent
officers of respondent had failed to call for a regular election since 1985.

Disputing the Aliazas groups allegation, respondent claimed that an election was conducted in 1987 but by
virtue of the enactment of Republic Act 6715 which amended the Labor Code, the term of office of its officers
was extended to five years or until 1992 during which a general assembly was held affirming their hold-over
tenure until the termination of collective bargaining negotiations; and that a collective bargaining agreement
(CBA) was executed only on March 30, 2000.

The DOLE-NCR ordered the conduct of elections to be placed under the control and supervision of its Labor
Relations Division and subject to pre-election conferences.

Without following the conditions set by DOLE-NCR, respondent called for a regular election to be held. This
prompted the Aliazas group to file a Motion for Intervention with the BLR. The BLR issued a cease and desist
order to the conduct of the elections pending compliance with the conditions set by DOLE-NCR.

The Aliazas group then wrote the DLSU President to put on escrow all union dues/agency fees and whatever
money considerations deducted from salaries of concerned co-academic personnel until such time that an
election of union officials has been scheduled and subsequent elections has been held. DLSU complied with the
request and established a separate account to place all the deductions from the salaries of employees. DLSU
also discontinued its relations with the Union and its officers in order to maintain neutrality.

DLSUEA filed a complaint against DLSU for Unfair Labor Practice claiming that petitioners unduly interfered with
its internal affairs and discriminated against its members.

ISSUE/S: Whether the acts of withholding union and agency dues and suspension of normal relations with
respondents incumbent set of officers pending the intra-union dispute did not constitute interference?

RULING: Yes. Pending the final resolution of the intra-union dispute, respondents officers remained duly
authorized to conduct union affairs.

It bears noting that at the time petitioners questioned moves were adopted, a valid and existing CBA had been
entered between the parties. It thus behooved petitioners to observe the terms and conditions thereof bearing
on union dues and representation. It is axiomatic in labor relations that a CBA entered into by a legitimate labor
organization and an employer becomes the law between the parties, compliance with which is mandated by
express policy of the law.

San Beda College of Law 390


4S: 2015 - 2016
LABOR LAW REVIEW Atty. Joyrich Golangco

MALAYANG SAMAHAN NG MGA MANGGAGAWA SA GREENFIELD (MSMG-UWP) v. RAMOS ET AL.
G.R. No. 113907. February 28, 2000
Purisima, J.

FACTS: MSMG is the local union/affiliate of respondent federation ULGWP. MSMG and the company
Greenfield entered into a CBA. The CBA contains a Union Security Clause providing that any employee who
fails to join or maintain membership in the Union shall be dismissed from the employment by the company upon
written recommendation to the company by the Union.

Due to certain disagreements between the MSMG and ULGWP regarding the collection of certain fees from
members, the MSMG declared general autonomy from ULGWP thru a Board Resolution which was later ratified
by the general membership.

Herein petitioners were later expelled from ULGWP allegedly because of their acts of disloyalty and/or acts
inimical to the interest and violative of the Constitution and By-laws of the federation. On the same day, the
federation advised respondent company of the expulsion of the 30 union officers and demanded their separation
from employment pursuant to the Union Security Clause in their collective bargaining agreement. The company
then dismissed the services of petitioners by bodily removing them from the premises of the company. This
provoked some of the members of the local union to demonstrate their protest for the dismissal of the said union
officers. Some union members left their work posts and walked out of the company premises. The employees
then staged a strike. The strike was attended with violence, force and intimidation on both sides resulting to
physical injuries to several employees, both striking and non-striking, and damage to company properties.

The employees who participated in the strike and allegedly figured in the violent incident were placed under
preventive suspension by respondent company. The company also sent return-to-work notices to the home
addresses of the striking employees thrice successively, however only a handful of employees returned to work.
The company then dismissed the employees who did not return to work for abandonment.

The petitioners herein filed a complaint charging private respondents of unfair labor practice which consists of
union busting, illegal dismissal, illegal suspension, interference in union activities, discrimination, threats,
intimidation, coercion, violence, and oppression.

Petitioners contend that their dismissal from work was effected in an arbitrary, hasty, capricious and illegal
manner because it was undertaken by the respondent company without any prior administrative investigation;
that, had respondent company conducted prior independent investigation it would have found that their
expulsion from the union was unlawful similarly for lack of prior administrative investigation; that the federation
cannot recommend the dismissal of the union officers because it was not a principal party to the collective
bargaining agreement between the company and the union.

Private respondents, on the other hand, maintain that the thirty dismissed employees who were former officers
of the federation have no cause of action against the company, the termination of their employment having been
made upon the demand of the federation pursuant to the union security clause of the CBA; the expelled officers
of the local union were accorded due process of law prior to their expulsion from their federation; that the strike
conducted by the petitioners was illegal for noncompliance with the requirements; that the employees who
participated in the illegal strike and in the commission of violence thereof were validly terminated from work; that
petitioners were deemed to have abandoned their employment when they did not respond to the three return to
work notices sent to them; that petitioner labor union has no legal personality to file and prosecute the case for
and on behalf of the individual employees as the right to do so is personal to the latter; and that, the officers of
respondent company cannot be liable because as mere corporate officers, they acted within the scope of their
authority.

ISSUE/S:
1. Can the employer dismiss an employee by virtue of the Union Security Clause without investigating
whether the employees were given due process by the Union?
2. Whether the act of disaffiliating from a federation can be considered as an act of disloyalty on the part of
the local union?
3. Whether the dismissal of an employee based on the Union Security Clause of the CBA constitutes
ULP?
4. Whether the employees in this case were validly dismissed?
San Beda College of Law 391
4S: 2015 - 2016
LABOR LAW REVIEW Atty. Joyrich Golangco

5. Whether the strike staged by the employees was illegal?

RULING:
1. No. As held the case of Cario v. National Labor Relations Commission, "the right of an employee to be
informed of the charges against him and to reasonable opportunity to present his side in a controversy
with either the company or his own union is not wiped away by a union security clause or a union shop
clause in a collective bargaining agreement. An employee is entitled to be protected not only from a
company which disregards his rights but also from his own union the leadership of which could yield to
the temptation of swift and arbitrary expulsion from membership and mere dismissal from his job."
While respondent company may validly dismiss the employees expelled by the union for disloyalty
under the union security clause of the collective bargaining agreement upon the recommendation by the
union, this dismissal should not be done hastily and summarily thereby eroding the employees right to
due process, self-organization and security of tenure. The enforcement of union security clauses is
authorized by law provided such enforcement is not characterized by arbitrariness, and always with due
process. Even on the assumption that the federation had valid grounds to expell the union officers, due
process requires that these union officers be accorded a separate hearing by respondent company.
2. No. A local union has the right to disaffiliate from its mother union or declare its autonomy. A local
union, being a separate and voluntary association, is free to serve the interests of all its members
including the freedom to disaffiliate or declare its autonomy. A local union which has affiliated itself with
a federation is free to sever such affiliation anytime and such disaffiliation cannot be considered
disloyalty. In the absence of specific provisions in the federations constitution prohibiting disaffiliation or
the declaration of autonomy of a local union, a local may dissociate with its parent union from the
federation to which it belongs when circumstances warrant, in accordance with the constitutional
guarantee of freedom of association
Here in the Constitution and By-Laws of the federation, there is no specific provision prohibiting
disaffiliation or declaration of autonomy by local unions.
3. No. Union security clauses in collective bargaining agreements, if freely and voluntarily entered into, are
valid and binding. Corrolarily, dismissals pursuant to union security clauses are valid and legal subject
only to the requirement of due process, that is, notice and hearing prior to dismissal. Thus, the dismissal
of an employee by the company pursuant to a labor unions demand in accordance with a union security
agreement does not constitute unfair labor practice.
4. No. the dismissal was invalidated in this case because of respondent companys failure to accord
petitioners with due process, that is, notice and hearing prior to their termination. Also, said dismissal
was invalidated because the reason relied upon by respondent Federation was not valid. Nonetheless,
the dismissal still does not constitute unfair labor practice.
5. No, when respondent company dismissed the union officers, the issue was transformed into a
termination dispute and brought respondent company into the picture. Petitioners believed in good faith
that in dismissing them upon request by the federation, respondent company was guilty of unfair labor
practice in that it violated the petitioners right to self-organization. The strike was staged to protest
respondent companys act of dismissing the union officers. Even if the allegations of unfair labor practice
are subsequently found out to be untrue, the presumption of legality of the strike prevails

San Beda College of Law 392


4S: 2015 - 2016
LABOR LAW REVIEW Atty. Joyrich Golangco

ALABANG COUNTRY CLUB, INC., petitioner, vs. NATIONAL LABOR RELATIONS COMMISSION,
ALABANG COUNTRY CLUB INDEPENDENT EMPLOYEES UNION, CHRISTOPHER PIZARRO, MICHAEL
BRAZA, and NOLASCO CASTUERAS, respondents.
G.R. No. 170287
February 14, 2008

FACTS: The Club and the Union entered into CBA, which provided for a Union shop and maintenance of
membership shop. The CBA includes that upon written demand of the UNION and after observing due process,
the Club shall dismiss a regular rank-and-file employee when the latter committed Malversation of union funds.
Respondents Christopher Pizarro, Michael Braza, and Nolasco Castueras were the Union President, Vice-
President, and Treasurer, respectively. Subsequently, an election was held and a new set of officers was
elected. Soon thereafter, the new officers conducted an audit of the Union funds. They discovered some
irregularly recorded entries, unaccounted expenses and disbursements, and uncollected loans from the Union
funds. The Union notified respondents Pizarro, Braza, and Castueras of the audit results and asked them to
explain the discrepancies in writing.

In a meeting called by the Union, respondents explained their side. Braza denied any wrongdoing and instead
asked that the investigation be addressed to Castueras, who was the Union Treasurer at that time. With regard
to his unpaid loans, Braza claimed he had been paying through monthly salary deductions and said the Union
could continue to deduct from his salary until full payment of his loans, provided he would be reimbursed should
the result of the initial audit be proven wrong by a licensed auditor. With regard to the Union expenses which
were without receipts, Braza explained that these were legitimate expenses for which receipts were not issued,
e.g. transportation fares, food purchases from small eateries, and food and transportation allowances given to
Union members with pending complaints with the DOLE, NLRC, and the fiscal's office. He explained that though
there were no receipts for these expenses, these were supported by vouchers and itemized as expenses.
Regarding his unpaid and unliquidated cash advances amounting to almost PhP 20,000, Braza explained that
these were not actual cash advances but payments to a certain Ricardo Ricafrente who had loaned PhP
200,000 to the Union. Pizarro, for his part, blamed Castueras for his unpaid and uncollected loan and cash
advances. He claimed his salaries were regularly deducted to pay his loan and he did not know why these
remained unpaid in the records. Nonetheless, he likewise agreed to continuous salary deductions until all his
accountabilities were paid. Castueras also denied any wrongdoing and claimed that the irregular entries in the
records were unintentional and were due to inadvertence because of his voluminous work load. He offered that
his unpaid personal loan of PhP 27,500 also be deducted from his salary until the loans were fully paid. Without
admitting any fault on his part, Castueras suggested that his salary be deducted until the unaccounted
difference between the loans and the amount collected amounting to a total of PhP 22,000 is paid.

Despite their explanations, respondents were expelled from the Union. The Union, invoking the Security Clause
of the CBA, demanded that the Club dismiss respondents in view of their expulsion from the Union. The Club
required the three respondents to show cause in writing within 48 hours from notice why they should not be
dismissed. They submitted their written explanations. The Club's general manager called respondents for an
informal conference inquiring about the charges against them. Said respondents gave their explanation and
asserted that the Union funds allegedly malversed by them were even over the total amount collected during
their tenure as Union officers-PhP 120,000 for Braza, PhP 57,000 for Castueras, and PhP 10,840 for Pizarro, as
against the total collection from April 1996 to December 2001 of only PhP 102,000. They claimed the charges
are baseless.

Nonetheless, after weighing the verbal and written explanations of the three respondents, the Club concluded
that said respondents failed to refute the validity of their expulsion from the Union. Thus, it was constrained to
terminate the employment of said respondents. Respondents challenged their dismissal from the Club in an
illegal dismissal complaint

ISSUE/S: Whether the three respondents were illegally dismissed and whether they were afforded due process.

RULING: One cause for termination is dismissal from employment due to the enforcement of the union security
clause in the CBA. Here, the CBA on Union security contains the provisions on the Union shop and
maintenance of membership shop. There is union shop when all new regular employees are required to join the
union within a certain period as a condition for their continued employment. There is maintenance of
membership shop when employees who are union members as of the effective date of the agreement, or who
thereafter become members, must maintain union membership as a condition for continued employment until
San Beda College of Law 393
4S: 2015 - 2016
LABOR LAW REVIEW Atty. Joyrich Golangco

18
they are promoted or transferred out of the bargaining unit or the agreement is terminated. Termination of
employment by virtue of a union security clause embodied in a CBA is recognized and accepted in our
19
jurisdiction. This practice strengthens the union and prevents disunity in the bargaining unit within the duration
of the CBA. By preventing member disaffiliation with the threat of expulsion from the union and the consequent
termination of employment, the authorized bargaining representative gains more numbers and strengthens its
position as against other unions which may want to claim majority representation.

In terminating the employment of an employee by enforcing the union security clause, the employer needs only
to determine and prove that: (1) the union security clause is applicable; (2) the union is requesting for the
enforcement of the union security provision in the CBA; and (3) there is sufficient evidence to support the
union's decision to expel the employee from the union. These requisites constitute just cause for terminating an
employee based on the CBA's union security provision.

The three respondents were expelled from and by the Union after due investigation for acts of dishonesty and
malversation of Union funds. In accordance with the CBA, the Union properly requested the Club, through the
October 18, 2001 letter signed by Mario Orense, the Union President, and addressed to Cynthia Figueroa, the
Club's HRD Manager, to enforce the Union security provision in their CBA and terminate said respondents.
Then, in compliance with the Union's request, the Club reviewed the documents submitted by the Union,
requested said respondents to submit written explanations, and thereafter afforded them reasonable opportunity
to present their side.

Were respondents Pizarro, Braza, and Castueras accorded due process before their employments were
terminated?
We rule that the Club substantially complied with the due process requirements before it dismissed the three
respondents.

The three respondents aver that the Club violated their rights to due process as enunciated in Malayang
23
Samahan, when it failed to conduct an independent and separate hearing before they were dismissed from
service.
Respondents errred in relying on Malayang Samahan, as its ruling has no application to this case. In Malayang
Samahan, the union members were expelled from the union and were immediately dismissed from the company
without any semblance of due process. Both the union and the company did not conduct administrative hearings
to give the employees a chance to explain themselves. In the present case, the Club has substantially complied
with due process. The three respondents were notified that their dismissal was being requested by the Union,
and their explanations were heard. Then, the Club, through its President, conferred with said respondents
during the last week of October 2001. The three respondents were dismissed only after the Club reviewed and
considered the documents submitted by the Union vis--vis the written explanations submitted by said
respondents. Under these circumstances, we find that the Club had afforded the three respondents a
reasonable opportunity to be heard and defend themselves.

On the applicability of Agabon, the Club points out that the CA ruled that the three respondents were illegally
dismissed primarily because they were not afforded due process. We are not unaware of the doctrine
enunciated in Agabon that when there is just cause for the dismissal of an employee, the lack of statutory due
process should not nullify the dismissal, or render it illegal or ineffectual, and the employer should indemnify the
27
employee for the violation of his statutory rights. However, we find that we could not apply Agabon to this case
as we have found that the three respondents were validly dismissed and were actually afforded due process.

San Beda College of Law 394


4S: 2015 - 2016
LABOR LAW REVIEW Atty. Joyrich Golangco

STANDARD CHARTERED BANK EMPLOYEES UNION (NUBE) vs. HONCONFESOR
G.R. No. 114974
June 16, 2004

FACTS: The Bank and the Union signed a five-year collective bargaining agreement (CBA) with a provision to
renegotiate the terms thereof on the third year. Before the commencement of the negotiation, the Union
suggested to the bank, that the bank lawyers should be excluded from the negotiating team. The Bank
acceded. Meanwhile, the Bank suggested to the Union, that Jose P. Umali, Jr., the President of the National
Union of Bank Employees (NUBE), the federation to which the Union was affiliated, be excluded from the
Unions negotiating panel. However, Umali was retained as a member thereof. Upon the Banks insistence, the
parties agreed to tackle the economic package item by item. Upon the Unions suggestion, the Bank indicated
which provisions it would accept, reject, retain and agree to discuss. Except for the provisions on signing bonus
and uniforms, the Union and the Bank failed to agree on the remaining economic provisions of the CBA. The
Union declared a deadlock and filed a Notice of Strike before the National Conciliation and Mediation Board
(NCMB).

On the other hand, the Bank filed a complaint for Unfair Labor Practice (ULP) and Damages before the NLRC
against the Union. The Bank alleged that the Union violated its duty to bargain, as it did not bargain in good
faith. It contended that the Union demanded "sky high economic demands," indicative ofblue-sky
bargaining. Further, the Union violated its no strike- no lockout clause by filing a notice of strike before the
NCMB.

The Secretary of Labor and Employment (SOLE) Nieves R. Confesor, issued an Order assuming jurisdiction
over the labor dispute at the Bank. The SOLE dismissed the charges of ULP of both the Union and the Bank,
explaining that both parties failed to substantiate their claims. Citing National Labor Union v. Insular-Yebana
Tobacco Corporation,the SOLE stated that ULP charges would prosper only if shown to have directly prejudiced
the public interest.

The Union alleges that the SOLE acted with grave abuse of discretion amounting to lack or excess of jurisdiction
when it found that the Bank did not commit unfair labor practice when it interfered with the Unions choice of
negotiator. It argued that, Dioknos suggestion that the negotiation be limited as a "family affair" was tantamount
to suggesting that Federation President Jose Umali, Jr. be excluded from the Unions negotiating panel. It
further argued that contrary to the ruling of the public respondent, damage or injury to the public interest need
not be present in order for unfair labor practice to prosper. The Union, likewise, pointed out that the public
respondent failed to rule on the ULP charges arising from the Banks surface bargaining. The Union contended
that the Bank merely went through the motions of collective bargaining without the intent to reach an agreement,
and made bad faith proposals when it announced that the parties should begin from a clean slate. It argued that
the Bank opened the political provisions "up for grabs," which had the effect of diminishing or obliterating the
gains that the Union had made.The Union also accused the Bank of refusing to disclose material and necessary
data, even after a request was made by the Union to validate its "guestimates."

The Bank contends that contrary to the Unions allegations, it was the Union that committed ULP when
negotiator Jose Umali, Jr. hurled invectives at the Banks head negotiator, Cielito Diokno, and demanded that
she be excluded from the Banks negotiating team. Moreover, the Union engaged in blue-sky bargaining and
isolated the no strike-no lockout clause of the existing CBA.

ISSUE/S: Whether or not the Union was able to substantiate its claim of unfair labor practice against the Bank
arising from the latters alleged "interference" with its choice of negotiator; surface bargaining; making bad faith
non-economic proposals; and refusal to furnish the Union with copies of the relevant data;

RULING: Article 248(a) of the Labor Code, considers it an unfair labor practice when an employer interferes,
restrains or coerces employees in the exercise of their right to self-organization or the right to form association.
The right to self-organization necessarily includes the right to collective bargaining.

Parenthetically, if an employer interferes in the selection of its negotiators or coerces the Union to exclude from
its panel of negotiators a representative of the Union, and if it can be inferred that the employer adopted the said
act to yield adverse effects on the free exercise to right to self-organization or on the right to collective
bargaining of the employees, ULP under Article 248(a) in connection with Article 243 of the Labor Code is
committed.
San Beda College of Law 395
4S: 2015 - 2016
LABOR LAW REVIEW Atty. Joyrich Golangco

In order to show that the employer committed ULP under the Labor Code, substantial evidence is required to
support the claim. Substantial evidence has been defined as such relevant evidence as a reasonable mind
48
might accept as adequate to support a conclusion. In the case at bar, the Union bases its claim of interference
on the alleged suggestions of Diokno to exclude Umali from the Unions negotiating panel.

The circumstances that occurred during the negotiation do not show that the suggestion made by Diokno to
Divinagracia is an anti-union conduct from which it can be inferred that the Bank consciously adopted such act
to yield adverse effects on the free exercise of the right to self-organization and collective bargaining of the
employees, especially considering that such was undertaken previous to the commencement of the negotiation
and simultaneously with Divinagracias suggestion that the bank lawyers be excluded from its negotiating panel.

The records show that after the initiation of the collective bargaining process, with the inclusion of Umali in the
Unions negotiating panel, the negotiations pushed through. The complaint was made only on August 16, 1993
after a deadlock was declared by the Union on June 15, 1993.

It is clear that such ULP charge was merely an afterthought. The accusation occurred after the arguments and
differences over the economic provisions became heated and the parties had become frustrated. It happened
after the parties started to involve personalities. As the public respondent noted, passions may rise, and as a
49
result, suggestions given under less adversarial situations may be colored with unintended meanings. Such is
what appears to have happened in this case.

The Duty to Bargain Collectively


If at all, the suggestion made by Diokno to Divinagracia should be construed as part of the normal relations and
innocent communications, which are all part of the friendly relations between the Union and Bank.

Surface bargaining is defined as "going through the motions of negotiating" without any legal intent to reach an
50
agreement. The resolution of surface bargaining allegations never presents an easy issue. The determination
of whether a party has engaged in unlawful surface bargaining is usually a difficult one because it involves, at
bottom, a question of the intent of the party in question, and usually such intent can only be inferred from the
51
totality of the challenged partys conduct both at and away from the bargaining table. It involves the question of
whether an employers conduct demonstrates an unwillingness to bargain in good faith or is merely hard
52
bargaining.

The minutes of meetings from March 12, 1993 to June 15, 1993 do not show that the Bank had any intention of
violating its duty to bargain with the Union. Records show that after the Union sent its proposal to the Bank on
February 17, 1993, the latter replied with a list of its counter-proposals on February 24, 1993. Thereafter,
meetings were set for the settlement of their differences. The minutes of the meetings show that both the Bank
and the Union exchanged economic and non-economic proposals and counter-proposals.

The Union has not been able to show that the Bank had done acts, both at and away from the bargaining table,
which tend to show that it did not want to reach an agreement with the Union or to settle the differences
between it and the Union. Admittedly, the parties were not able to agree and reached a deadlock. However, it is
herein emphasized that the duty to bargain "does not compel either party to agree to a proposal or require the
53
making of a concession." Hence, the parties failure to agree did not amount to ULP under Article 248(g) for
violation of the duty to bargain.

San Beda College of Law 396


4S: 2015 - 2016
LABOR LAW REVIEW Atty. Joyrich Golangco

GENERAL MILLING CORPORATION, petitioner, vs. HON. COURT OF APPEALS, GENERAL MILLING
CORPORATION INDEPENDENT LABOR UNION (GMC-ILU), and RITO MANGUBAT, respondents.

FACTS: In its two plants located at Cebu City and Lapu-Lapu City, petitioner General Milling Corporation (GMC)
employed 190 workers. They were all members of private respondent General Milling Corporation Independent
Labor Union (union, for brevity), a duly certified bargaining agent. A day before the expiration of the CBA, the
union sent GMC a proposed CBA, with a request that a counter-proposal be submitted within ten (10) days.
GMC had received collective and individual letters from workers who stated that they had withdrawn from their
union membership, on grounds of religious affiliation and personal differences. Believing that the union no
longer had standing to negotiate a CBA, GMC did not send any counter-proposal. Thus, the union filed, on July
2, 1992, a complaint against GMC with the NLRC, Arbitration Division, Cebu City. The complaint alleged unfair
labor practice on the part of GMC for: (1) refusal to bargain collectively; (2) interference with the right to self-
organization; and (3) discrimination. The labor arbiter dismissed the case with the recommendation that a
petition for certification election be held to determine if the union still enjoyed the support of the workers.

NLRC ordered GMC to abide by the CBA draft that the union proposed for a period of two (2) years. NLRC
pointed out that upon the effectivity of Rep. Act No. 6715, the duration of a CBA, insofar as the representation
aspect is concerned, is five (5) years which, in the case of GMC-Independent Labor Union was from December
1, 1988 to November 30, 1993. All other provisions of the CBA are to be renegotiated not later than three (3)
years after its execution. Thus, the NLRC held that respondent union remained as the exclusive bargaining
agent with the right to renegotiate the economic provisions of the CBA. Consequently, it was unfair labor
practice for GMC not to enter into negotiation with the union.

ISSUE/S: Did the company commit ULP?

RULING: Yes. The law mandates that the representation provision of a CBA should last for five years. The
relation between labor and management should be undisturbed until the last 60 days of the fifth year. Hence, it
is indisputable that when the union requested for a renegotiation of the economic terms of the CBA on
November 29, 1991, it was still the certified collective bargaining agent of the workers, because it was seeking
said renegotiation within five (5) years from the date of effectivity of the CBA on December 1, 1988. The unions
proposal was also submitted within the prescribed 3-year period from the date of effectivity of the CBA, albeit
just before the last day of said period. It was obvious that GMC had no valid reason to refuse to negotiate in
good faith with the union. For refusing to send a counter-proposal to the union and to bargain anew on the
economic terms of the CBA, the company committed an unfair labor practice under Article 248 of the Labor
Code

San Beda College of Law 397


4S: 2015 - 2016
LABOR LAW REVIEW Atty. Joyrich Golangco

HACIENDA FATIMA and/or PATRICIO VILLEGAS, ALFONSO VILLEGAS and CRISTINE SEGURA,
petitioners, vs. NATIONAL FEDERATION OF SUGARCANE WORKERS-FOOD AND GENERAL TRADE,
respondents.

FACTS: Contrary to the findings of the Labor Arbiter that complainants [herein respondents] refused to work
and/or were choosy in the kind of jobs they wanted to perform, the records is replete with complainants
persistence and dogged determination in going back to work. It would appear that respondents did not look with
favor workers having organized themselves into a union. Thus, when complainant union was certified as the
collective bargaining representative in the certification elections, respondents under the pretext that the result
was on appeal, refused to sit down with the union for the purpose of entering into a collective bargaining
agreement. Moreover, the workers including complainants herein were not given work for more than one month.
In protest, complainants staged a strike which was however settled upon the signing of a Memorandum of
Agreement which stipulated among others that: The management will provide fifteen (15) wagons for the
workers and that existing workforce prior to the actual strike will be given priority. However, in case the said
workforce would not be enough, the management can hire additional workers to supplement them. However,
alleging that complainants failed to load the fifteen wagons, respondents reneged on its commitment to sit down
and bargain collectively. Instead, respondent employed all means including the use of private armed guards to
prevent the organizers from entering the premises.

CA affirmed that while the work of respondents was seasonal in nature, they were considered to be merely on
leave during the off-season and were therefore still employed by petitioners. Moreover, the workers enjoyed
security of tenure. Any infringement upon this right was deemed by the CA to be tantamount to illegal dismissal.

ISSUE/S: Whether or not the Court of Appeals committed grave abuse of discretion in upholding the NLRCs
conclusion that private respondents were illegally dismissed, that petitioner[s were] guilty of unfair labor practice,
and that the union be awarded moral and exemplary damages

RULING: The appellate court found neither rhyme nor reason in petitioners argument that it was the workers
themselves who refused to or were choosy in their work. As found by the NLRC, the record of this case is
replete with complainants persistence and dogged determination in going back to work.

The CA likewise concurred with the NLRCs finding that petitioners were guilty of unfair labor practice. We
uphold the ruling of the CA.

San Beda College of Law 398


4S: 2015 - 2016
LABOR LAW REVIEW Atty. Joyrich Golangco

ST. JOHN COLLEGES, INC. v. ST. JOHN ACADEMY FACULTY AND EMPLOYEES UNION
GR No. 167892
October 27, 2006

DOCTRINE: Under Article 283 of the Labor Code, the following requisites must concur for a valid closure of the
business: (1) serving a written notice on the workers at least one (1) month before the intended date thereof; (2)
serving a notice with the DOLE one month before the taking effect of the closure; (3) payment of separation pay
equivalent to one (1) month or at least one half (1/2) month pay for every year of service, whichever is higher,
with a fraction of at least six (6) months to be considered as a whole year; and (4) cessation of the operation
must be bona fide.

FACTS: The Collective Bargaining Agreement between St. John Colleges and St. John Academy Faculty and
Employees Union was set to expire May 31, 1997. During the ensuing negotiations, SJCI rejected all the
proposals of the Union for an increase in workers benefits, resulting to a bargaining deadlock which led to the
holding of a valid strike. NCMB agreed to refer the labor dispute to SOLE for assumption of jurisdiction.

Pending resolution of the labor dispute before SOLE, the Board of Directors of the College approved for the
closure of the high school which was approved by the stockholders. SCJI informed DOLE, DECS, parents,
students and the union of the impending closure. SJCI informed SOLE that 51 employees had received
separation compensation packages while 25 refused the same.

The 25 employees filed a complaint for ULP, illegal dismissal and non-payment of monetary benefits against
SCJI, alleging that the closure of the high school was done in bad faith in order to get rid of the union and render
any decision of the SOLE on the CBA deadlocked issues.

LA Macam dismissed the Unions complaint for ULP and illegal dismissal while granting SJCIs petition to
declare the strike illegal coupled with a declaration of loss of employment status of 25 Union members involved
in the strike.

After the favorable decision of LA, SCJI resolved to reopen the high school for school year 1999-2000, however,
it did not restore the high school teaching and non-teaching employees it earlier terminated.

NLRC DECISION

On June 28, 2002, the NLRC rendered judgment reversing the decision of the Labor Arbiter. It found SJCI guilty
of ULP and illegal dismissal and ordered it to reinstate the 25 employees to their former positions without loss of
seniority rights and other benefits, and with full backwages. It also required SJCI to pay moral and exemplary
damages, attorneys fees, and two (2) months summer/vacation pay. Moreover, it ruled that the mass actions
conducted by the 25 employees on May 4, 1998 could not be considered as a strike since, by then, the
employer-employee relationship had already been terminated due to the closure of the high school.

COURT OF APPEALS

The Court of Appeals affirmed with modification the decision of the NLRC.

ISSUE/S: Whether SCJI is liable for ULP and illegal dismissal when it closed down the high school and
reopened the same at a later time.

RULING: Under Article 283 of the Labor Code, the following requisites must concur for a valid closure of the
business: (1) serving a written notice on the workers at least one (1) month before the intended date thereof; (2)
serving a notice with the DOLE one month before the taking effect of the closure; (3) payment of separation pay
equivalent to one (1) month or at least one half (1/2) month pay for every year of service, whichever is higher,
with a fraction of at least six (6) months to be considered as a whole year; and (4) cessation of the operation
must be bona fide. It is not disputed that the first two requisites were satisfied. The third requisite would have
been satisfied were it not for the refusal of the herein private respondents to accept the separation
compensation package. The instant case, thus, revolves around the fourth requisite, i.e., whether SJCI closed
the high school in good faith.

San Beda College of Law 399


4S: 2015 - 2016
LABOR LAW REVIEW Atty. Joyrich Golangco

The determination of whether SJCI acted in bad faith depends on the particular facts as established by the
evidence on record. Bad faith is, after all, an inference which must be drawn from the peculiar circumstances of
a case. The two decisive factors in determining whether SJCI acted in bad faith are (1) the timing of, and
reasons for the closure of the high school, and (2) the timing of, and the reasons for the subsequent opening of
a college and elementary department, and, ultimately, the reopening of the high school department by SJCI
after only one year from its closure.

Prior to the closure of the high school by SJCI, the parties agreed to refer the 1997 CBA deadlock to the SOLE
for assumption of jurisdiction under Article 263 of the Labor Code. As a result, the strike ended and classes
resumed. After the SOLE assumed jurisdiction, it required the parties to submit their respective position papers.
However, instead of filing its position paper, SJCI closed its high school, allegedly because of the "irreconcilable
differences between the school management and the Academys Union particularly the safety of our students
and the financial aspect of the ongoing CBA negotiations." Thereafter, SJCI moved to dismiss the pending labor
dispute with the SOLE contending that it had become moot because of the closure. Nevertheless, a year after
said closure, SJCI reopened its high school and did not rehire the previously terminated employees.

Under these circumstances, it is not difficult to discern that the closure was done to defeat the parties
agreement to refer the labor dispute to the SOLE; to unilaterally end the bargaining deadlock; to render nugatory
any decision of the SOLE; and to circumvent the Unions right to collective bargaining and its members right to
security of tenure. By admitting that the closure was due to irreconcilable differences between the Union and
school management, specifically, the financial aspect of the ongoing CBA negotiations, SJCI in effect admitted
that it wanted to end the bargaining deadlock and eliminate the problem of dealing with the demands of the
Union. This is precisely what the Labor Code abhors and punishes as unfair labor practice since the net
effect is to defeat the Unions right to collective bargaining.

San Beda College of Law 400


4S: 2015 - 2016
LABOR LAW REVIEW Atty. Joyrich Golangco

CENTRAL AZUCARERA DE BAIS EMPLOYEES UNION NFL
VERSUS CENTRALK AZUCARERA DE BAIS
GR No. 186605
November 17, 2010

DOCTRINE: For a charge of unfair labor practice to prosper, it must be shown that CAB was motivated by ill
will, bad faith, or fraud, or was oppressive to labor, or done in a manner contrary to morals, good customs, or
public policy, and, of course, that social humiliation, wounded feelings or grave anxiety resulted in suspending
negotiations with CABEU-NFL.

FACTS: Central Azucarera de Bais (CAB) NFL sent CAB a proposed CBA seeking increases in daily wage
th
and vacation and sick leave benefits of the monthly employees and the grant of leave benefits and 13 month
pay to seasonal workers. CAB sent a counter-proposal to the effect that the production bonus incentive and
special production bonus and incentives be maintained. CAB agreed to execute a pro-rated increase of wages
every time the government would mandate an increase in the minimum wage. CAB did not agree to grant the
additional and separate Christmas bonus. CAB received an amended union proposal by CABEU-NFL reducing
its previous demand regarding wages and bonuses. It resulted to a deadlock.

CABEU-NFL filed a notice of strike. The NCMB assumed conciliatory-mediation jurisdiction and summoned
parties. CABEU-NFL requested copies of CABs annual financial statements and asked for resumption of
conciliation meetings. CAB replied with the letter that it is moot and academic to continue for the Union has
already lost its majority status by reason of disauthorization and withdrawal of support by more than 90% of the
rank and file employees.

CABEU-NFL filed a complaint for ULP for refusal to bargain with.

LA DECISION

Record shows that respondent CAB replied to the complainant Unions CBA proposals with its own set of
counterproposals. Likewise, respondent CAB responded to the Unions subsequent counterproposals. Record
further shows that respondent CAB participated in a series of CBA negotiations conducted by the parties at the
plant level as well as in the conciliation/mediation proceedings conducted by the NCMB. Unfortunately, both
exercises resulted in a deadlock. At this juncture it cannot be said, therefore, that respondent CAB refused to
negotiate or that it violated its duty to bargain collectively in light of its active participation in the past CBA
negotiations at the plant level as well as in the NCMB.

NLRC DECISION

The important event to discuss in the instant case is respondents act of concluding a CBA with CABELA. As
gleaned from respondents letter to NCMB dated June 14, 2005, it concluded a CBA with CABELA because they
opined that complainant lost its majority status in January 2005 when 90% of the rank-and-file employees
disauthorized and withdrew their support to complainant. These rank-and-file employees who withdrew their
support, organized and formed CABELA. In fine, respondent believed that CABELA enjoyed the majority status
of CABELA since it was supported by 90% of all employees in the bargaining unit. Respondents failure to act on
the request of the complainant to resume negotiation for no valid reason constitutes unfair labor practice.
Consequently, the proposed CBA as amended should be imposed to respondent.

CA DECISION

In the case at bar, private respondent CABEU-NFL failed in its burden of proof to present substantial evidence
to support the allegation of unfair labor practice. The assailed Decision and Resolution of public respondent
referred merely to two (2) circumstances which allegedly support the conclusion that the presumption of good
faith had been rebutted and that bad faith was extant in petitioners actions. To recall, these circumstances are:
(a) the execution of a supposed collective bargaining agreement with another labor union, CABELA; and (b)
CABs sending of the letter dated June 14, 2005 to NCMB seeking to call off the collective bargaining
negotiations. These, however, are not enough to ascribe the very serious offense of unfair labor practice upon
petitioner.

ISSUE/S: Whether CAB is guilty of ULP on the ground of refusal to bargain collectively
San Beda College of Law 401
4S: 2015 - 2016
LABOR LAW REVIEW Atty. Joyrich Golangco

RULING:

For a charge of unfair labor practice to prosper, it must be shown that CAB was motivated by ill will, bad faith, or
fraud, or was oppressive to labor, or done in a manner contrary to morals, good customs, or public policy, and,
of course, that social humiliation, wounded feelings or grave anxiety resulted in suspending negotiations with
CABEU-NFL. Notably, CAB believed that CABEU-NFL was no longer the representative of the workers. It just
wanted to foster industrial peace by bowing to the wishes of the overwhelming majority of its rank and file
workers and by negotiating and concluding in good faith a CBA with CABELA. Such actions of CAB are
nowhere tantamount to anti-unionism, the evil sought to be punished in cases of unfair labor practices.

Furthermore, basic is the principle that good faith is presumed and he who alleges bad faith has the duty to
prove the same. By imputing bad faith to the actuations of CAB, CABEU-NFL has the burden of proof to present
substantial evidence to support the allegation of unfair labor practice. Apparently, CABEU-NFL refers only to the
circumstances mentioned in the letter-response, namely, the execution of the supposed CBA between CAB and
CABELA and the request to suspend the negotiations, to conclude that bad faith attended CABs actions. The
Court is of the view that CABEU-NFL, in simply relying on the said letter-response, failed to substantiate its
claim of unfair labor practice to rebut the presumption of good faith.

San Beda College of Law 402


4S: 2015 - 2016
LABOR LAW REVIEW Atty. Joyrich Golangco

UNION OF FILIPRO EMPLOYEES - DRUG, FOOD AND ALLIED INDUSTRIES UNIONS - KILUSANG MAYO
UNO (UFE-DFA-KMU) v. NESTL PHILIPPINES, INCORPORATED
G.R. Nos. 158930-31
March 3, 2008

NESTL PHILIPPINES, INCORPORATED v. UNION OF FILIPRO EMPLOYEES - DRUG, FOOD AND


ALLIED INDUSTRIES UNIONS - KILUSANG MAYO UNO (UFE-DFA-KMU)
G.R. Nos. 158944-45
March 3, 2008
CHICO-NAZARIO, J.

FACTS: UFE-DFA-KMU was the sole and exclusive bargaining agent of the rank-and-file employees of Nestle
belonging to the latters Alabang and Cabuyao plants. The Presidents of the Alabang and Cabuyao Divisions of
UFE-DFA-KMU informed Nestle of their intent to open a new CBA negotiations for the year 2001-2004. Nestle
on its part informed them it was preparing its own counter-proposal and proposed ground rules to govern the
impending conduct of CBA negotiations. Nestl reiterated its stance in a letter to UFE-DFA-KMU that "unilateral
grants, one-time company grants, company-initiated policies and programs, which include, but are not limited to
the Retirement Plan, Incidental Straight Duty Pay and Calling Pay Premium, are by their very nature not proper
subjects of CBA negotiations and therefore shall be excluded therefrom.

Nestle requested the NCMB, Regional Office, to conduct preventive mediation proceedings as result of an
impasse. However, the conciliation was ineffective and UFE-DFA-KMU filed a Notice of Strike on the ground of
a bargaining deadlock pertaining to economic issues. It filed another Notice of Strike this time on the ground of
unfair labor practice on the part of Nestle. It alleged that Nestle is bargaining in bad faith by setting pre-
conditions in the ground rules and/or refusing to include the issue of the Retirement Plan in the CBA
negotiations.

Secretary of DOLE Sto. Tomas assumed jurisdiction over the labor dispute upon Petition for Assumption of
Jurisdiction by Nestle. UFE-DFA-KMU filed a petition for Certiorari before the Court of Appeals after Sec. Sto.
Tomas denied their motion for reconsideration alleging that she committed grave abuse of discretion amounting
to lack or excess of jurisdiction when she issued the said orders. Acting Secretary of DOLE Brion issued an
order ruling in favor of Nestle. Another petition for Certiorari before the Court of Appeals was filed by UFE-DFA-
KMU. The Court of Appeals ruled in favor of UFE-DFA-KMU but absolved Nestle of any unfair labor practice.

ISSUE/S: Is Nestle liable for unfair labor practice?

RULING: No. For a charge of unfair labor practice to prosper, it must be shown that Nestl was motivated by ill
will, "bad faith, or fraud, or was oppressive to labor, or done in a manner contrary to morals, good customs, or
public policy, and, of course, that social humiliation, wounded feelings, or grave anxiety resulted x x x" in
disclaiming unilateral grants as proper subjects in their collective bargaining negotiations. While the law makes it
an obligation for the employer and the employees to bargain collectively with each other, such compulsion does
not include the commitment to precipitately accept or agree to the proposals of the other. All it contemplates is
that both parties should approach the negotiation with an open mind and make reasonable effort to reach a
common ground of agreement.

Nestle never refused to bargain collectively with UFE-DFA-KMU. The corporation simply wanted to exclude the
Retirement Plan from the issues to be taken up during CBA negotiations, on the postulation that such was in the
nature of a unilaterally granted benefit. An employers steadfast insistence to exclude a particular substantive
provision is no different from a bargaining representatives perseverance to include one that they deem of
absolute necessity. Indeed, an adamant insistence on a bargaining position to the point where the negotiations
reach an impasse does not establish bad faith. It is but natural that at negotiations, management and labor
adopt positions or make demands and offer proposals and counter-proposals. On account of the importance of
the economic issue proposed by UFE-DFA-KMU, Nestle could have refused to bargain with the former but it
did not. And the managements firm stand against the issue of the Retirement Plan did not mean that it was
bargaining in bad faith. It had a right to insist on its position to the point of stalemate.

San Beda College of Law 403


4S: 2015 - 2016
LABOR LAW REVIEW Atty. Joyrich Golangco

MALAYANG MANGGAGAWA NG STAYFAST PHILS., INC. vs. NATIONAL LABOR RELATIONS
COMMISSION, STAYFAST PHILIPPINES, INC./ MARIA ALMEIDA
G.R. No. 155306 August 28, 2013
LEONARDO-DE CASTRO, J.

FACTS: The Med-Arbiter issued an order certifying NLMS-Olalia as the sole and exclusive bargaining agent of
all rank and file employees of respondent company. Petitioner appealed to the Secretary of Labor and
Employment. The Secretary initially set aside the order of the Med-Arbiter and called for a run-off election
between petitioner and NLMS-Olalia but reconsidered his earlier decision and restored the decision of the Med-
Arbiter.

The respondent company rejected the petitioners demand to collectively bargain on the ground it will only
negotiate with whichever is finally certified as the sole and exclusive bargaining agent of the workers. Petitioner
filed a notice of strike in the NCMB. Respondent company filed a motion to dismiss on the ground petitioner was
not the certified bargaing agent. They were able to make concessions during the conciliation-mediation stage in
the NCMB which led to petitioner to withraw its notice of strike. However, petitioners members staged a sit-
down strike to dramatize their demand for a fair and equal treatment as respondent company allegedly
continued to discriminate against them. The respondent company terminated the services of the participants
after they failed to explain their actions. Petitioner then staged a strike and filed a complaint for unfair labor
practice, union busting and illegal lockout against respondent company and its General Manager in the NLRC.

In support of its complaint, petitioner alleged that respondents had repeatedly committed acts of discrimination,
such as the denial of the use of the company canteen for purposes of conducting a strike vote, the constant
denial of applications of petitioners members for leave to attend hearings in relation to certain labor cases while
similar applications of members of the other union were approved, and the suspension of petitioners president
for being absent due to attendance in hearings of labor cases involving petitioners members. Petitioner further
claimed that the termination of about 127 of its officers and members constituted union busting and unlawful
lockout. Respondent company, on the other hand, claimed that petitioner lacked legal authority to go on stike
snce it is a minority union.

The Labor Arbiter dismissed the complaint on the ground the petitioner failed to cite any instance of
discrimination or harassment when it filed its notice of strike. The NLRC upheld the decision of the Labor Arbiter
on the ground the actuations of the petitioner were patently illegal because the sit-down strike staged was made
barely a week after it withdrew its notice of strike and it failed to file a new notice of strike. The Court of Appeals
likewise affirmed the decision of the Labor Arbiter and the NLRC.

ISSUE/S: Is the respondent company liable for unfair labor practice?

RULING: No. Petitioners case rests on the alleged discriminatory acts of respondent company against
petitioners officers and members. However, both the Labor Arbiter and the NLRC held that there was no
sufficient proof of respondent companys alleged discriminatory acts. Thus, petitioners unfair labor practice,
union-busting and unlawful lockout claims do not hold water. Moreover, the established facts as found by the
NLRC are as follows: the "sit-down strike" made by petitioners officers and members on July 21, 1997 was in
violation of respondent companys rules, and petitioners officers and members ignored the opportunity given by
respondent company for them to explain their misconduct, which resulted in the termination of their employment.
The Court of Appeals ruled that the said findings were supported by substantial evidence. This Court finds that
such ruling of the appellate court is not grave abuse of discretion, nor could it be considered wrong.

San Beda College of Law 404


4S: 2015 - 2016
LABOR LAW REVIEW Atty. Joyrich Golangco

HOLY CHILD CATHOLIC SCHOOL vs. HON. PATRICIA STO. TOMAS, in her official capacity as Secretary
of the Department of Labor and Employment, and PINAG-ISANG TINIG AT LAKAS NG ANAKPAWIS
HOLY CHILD CATHOLIC SCHOOL TEACHERS AND EMPLOYEES LABOR UNION (HCCS-TELU-PIGLAS)
G.R. No. 179146
July 23, 2013

FACTS:
A petition for certification election was filed by private respondent alleging that it is a legitimate labor
organization. Its membership is quite easily be categorized into (2) general classes: one, the teaching staff; and
two, the non-teaching-staff. Now, the petition for certification election was assailed by the petitioner claiming that
the two classes of employees do not belong to the same collective bargaining unit.

ISSUE/S:
1. Whether or not petitioner can collaterally attack the legitimacy of private respondent by praying for the
dismissal of the petition for certification election?
2. Whether or not the petition for certification election should have been dismissed on the ground that
private respondent is not qualified to file such petition for its failure to qualify as a legitimate labor
organization?
3. Should the teaching and non-teaching personnel of petitioner school form separate bargaining units?

RULING:
1. NO. Following the doctrine laid down in Kawashima and SMCC-Super, it must be stressed that
petitioner cannot collaterally attack the legitimacy of private respondent by praying for the dismissal
of the petition for certification election:
Except when it is requested to bargain collectively, an employer is a mere bystander to any petition
for certification election; such proceeding is non-adversarial and merely investigative, for the
purpose thereof is to determine which organization will represent the employees in their collective
bargaining with the employer. The choice of their representative is the exclusive concern of the
employees; the employer cannot have any partisan interest therein; it cannot interfere with, much
less oppose, the process by filing a motion to dismiss or an appeal from it; not even a mere
allegation that some employees participating in a petition for certification election are actually
managerial employees will lend an employer legal personality to block the certification election. The
employer's only right in the proceeding is to be notified or informed thereof.
2. The court disagrees. Article 212(g) of the Labor Code defines a labor organization as "any union
or association of employees which exists in whole or in part for the purpose of collective bargaining
or of dealing with employers concerning terms and conditions of employment." Upon compliance
with all the documentary requirements, any applicant labor organization shall acquire legal
personality and shall be entitled to the rights and privileges granted by law to legitimate labor
organizations upon issuance of the certificate of registration.
In case of alleged inclusion of disqualified employees in a union, the proper procedure for an
employer like petitioner is to directly file a petition for cancellation of the unions certificate of
registration due to misrepresentation, false statement or fraud under the circumstances enumerated
in Article 239 of the Labor Code. To reiterate, private respondent, having been validly issued a
certificate of registration, should be considered as having acquired juridical personality which may
not be attacked collaterally.
On the other hand, a bargaining unit has been defined as a "group of employees of a given
employer, comprised of all or less than all of the entire body of employees, which the collective
interests of all the employees, consistent with equity to the employer, indicated to be best suited to
serve reciprocal rights and duties of the parties under the collective bargaining provisions of the
law."
3. Yes. The Supreme Court ordered the "non-academic rank-and-file employees of U.P. to constitute a
bargaining unit to the exclusion of the academic employees of the institution", but did not order them
to organize a separate labor organization. In the U.P. case, the Supreme Court did not dismiss the
petition and affirmed the order for the conduct of a certification election among the non-academic
personnel of U.P., without prejudice to the right of the academic personnel to constitute a separate
bargaining unit for themselves and for the All U.P. Workers Union to institute a petition for
certification election.
In the same manner, the teaching and non-teaching personnel of petitioner school must form
separate bargaining units. Thus, the order for the conduct of two separate certification elections,
San Beda College of Law 405
4S: 2015 - 2016
LABOR LAW REVIEW Atty. Joyrich Golangco

one involving teaching personnel and the other involving non-teaching personnel. It should be
stressed that in the subject petition, private respondent union sought the conduct of a certification
election among all the rank-and-file personnel of petitioner school. Since the decision of the
Supreme Court in the U.P. case prohibits us from commingling teaching and non-teaching
personnel in one bargaining unit, they have to be separated into two separate bargaining units
with two separate certification elections to determine whether the employees in the respective
bargaining units desired to be represented by private respondent. In the U.P. case, only one
certification election among the non-academic personnel was ordered, because ONAPUP sought to
represent that bargaining unit only. No petition for certification election among the academic
personnel was instituted by All U.P. Workers Union in the said case; thus, no certification election
pertaining to its intended bargaining unit was ordered by the Court.

San Beda College of Law 406


4S: 2015 - 2016
LABOR LAW REVIEW Atty. Joyrich Golangco

EMPLOYEES UNION OF BAYER PHILS., FFW and JUANITO S. FACUNDO vs. BAYER PHILIPPINES, INC.,
ET AL.
G.R. No. 162943
December 6, 2010
VILLARAMA, JR., J.:

DOCTRINE:
An intra-union dispute refers to any conflict between and among union members, including grievances arising
from any violation of the rights and conditions of membership, violation of or disagreement over any provision of
the unions constitution and by-laws, or disputes arising from chartering or disaffiliation of the union.

KEYWORDS:
INTER/INTRA-UNION DISPUTE, DEFINITION, JURISDICTION

FACTS:
Petitioner EUBP is the exclusive bargaining agent of all rank-and-file employees of Bayer Philippines, and is an
affiliate of the Federation of Free Workers. EUBP, headed by its president Juanito S. Facundo, negotiated with
Bayer for the signing of CBA.

Pending the resolution of the dispute, respondent Avelina Remigio and 27 other union members, without any
authority from their union leaders, accepted Bayers wage-increase proposal. EUBPs grievance committee
questioned Remigios action and reprimanded Remigio and her allies. Eventually, the DOLE Secretary issued
an arbitral award ordering EUBP and Bayer to execute a CBA retroactive to January 1, 1997 and to be made
effective until December 31, 2001.

Barely six months from the signing of the new CBA, during a company-sponsored seminar, Remigio solicited
signatures from union members in support of a resolution containing the decision of the signatories to: (1)
disaffiliate from FFW, (2) rename the union as Reformed Employees Union of Bayer Philippines (REUBP), (3)
adopt a new constitution and by-laws for the union, (4) abolish all existing officer positions in the union and elect
a new set of interim officers, and (5) authorize REUBP to administer the CBA between EUBP and Bayer. The
said resolution was signed by 147 of the 257 local union members. A subsequent resolution was also issued
affirming the first resolution.

A tug-of-war then ensued between the two rival groups, with both seeking recognition from Bayer and
demanding remittance of the union dues collected from its rank-and-file members. Bayer responded by deciding
not to deal with either of the two groups, and by placing the union dues collected in a trust account until the
conflict between the two groups is resolved.

EUBP filed a complaint for unfair labor practice (first ULP complaint) against Bayer for non-remittance of union
dues. While the first ULP case was still pending and despite EUBPs repeated request for a grievance
conference, Bayer decided to turn over the collected union dues amounting to P254,857.15 to the Treasurer of
REUBP.

Aggrieved by the said development, EUBP lodged a complaint against Remigios group before the Industrial
Relations Division of the DOLE praying for their expulsion from EUBP for commission of acts that threaten the
life of the union.

Labor Arbiter Jovencio Ll. Mayor, Jr. dismissed the first ULP complaint for lack of jurisdiction because the root
cause for Bayers failure to remit the collected union dues can be traced to the intra-union conflict between
EUBP and Remigios group and that the charges imputed against Bayer should have been submitted instead to
voluntary arbitration. EUBP did not appeal the said decision.

Petitioners filed a second ULP complaint against respondents. Three days later, petitioners amended the
complaint charging the respondents with unfair labor practice committed by organizing a company union, gross
violation of the CBA and violation of their duty to bargain. Petitioners complained that Bayer refused to remit the
collected union dues to EUBP despite several demands sent to the management. They also alleged that
notwithstanding the requests sent to Bayer for a renegotiation of the last two years of the 1997-2001 CBA
between EUBP and Bayer, the latter opted to negotiate instead with Remigios group.
San Beda College of Law 407
4S: 2015 - 2016
LABOR LAW REVIEW Atty. Joyrich Golangco

On even date, REUBP and Bayer agreed to sign a new CBA.

Later, petitioners filed a second amended complaint to include in its complaint the issue of gross violation of the
CBA for violation of the contract bar rule following Bayers decision to negotiate and sign a new CBA with
Remigios group.

Meanwhile, the Regional Director of the Industrial Relations Division of DOLE issued a decision dismissing the
issue on expulsion filed by EUBP against Remigio and her allies. EUBP seasonably appealed the said decision
to the Bureau of Labor Relations (BLR) which reversed the Regional Directors ruling and ordered the
management of Bayer to respect the authority of the duly-elected officers of EUBP in the administration of the
prevailing CBA.

Unfortunately, the said BLR ruling came late since Bayer had already signed a new CBA with REUBP. The said
CBA was eventually ratified by majority of the bargaining unit.

Labor Arbiter Waldo Emerson R. Gan dismissed EUBPs second ULP complaint for lack of jurisdiction and
observed that the case involves intra-union disputes and thus is bereft of any jurisdiction pursuant to Article 226
of the LC.

On June 28, 2000, the NLRC resolved to dismiss petitioners motion for a restraining order and/or injunction
stating that the subject matter involved an intra-union dispute, over which the said Commission has no
jurisdiction.

Aggrieved by the Labor Arbiters decision to dismiss the second ULP complaint, petitioners appealed the said
decision, but the NLRC denied the appeal. EUBPs motion for reconsideration was likewise denied.

ISSUE/S:
Did the LABOR ARBITER and THE NLRC correctly rule on the second ULP case that they are bereft of
jurisdiction for it is the BLR who should take cognizance of the case, it being an intra-union conflict?

RULING:
An intra-union dispute refers to any conflict between and among union members, including grievances arising
from any violation of the rights and conditions of membership, violation of or disagreement over any provision of
the unions constitution and by-laws, or disputes arising from chartering or disaffiliation of the union. Sections 1
and 2, Rule XI1 of Department Order No. 40-03, Series of 2003 of the DOLE enumerate the following
circumstances as inter/intra-union disputes.


1
INTER/INTRA-UNION
RULE XI DISPUTES AND
OTHER RELATED LABOR RELATIONS DISPUTES

SECTION 1. Coverage. - Inter/intra-union disputes shall include:

a. cancellation of registration of a labor organization filed by its members or by another labor organization;
b. conduct of election of union and workers association officers/nullification of election of union and workers association officers;
c. audit/accounts examination of union or workers association funds;
d. deregistration of collective bargaining agreements;
e. validity/invalidity of union affiliation or disaffiliation;
f. validity/invalidity of acceptance/non-acceptance for union membership;
g. validity/invalidity of impeachment/expulsion of union and workers association officers and members;
h. validity/invalidity of voluntary recognition;
i. opposition to application for union and CBA registration;
j. violations of or disagreements over any provision in a union or workers association constitution and by-laws;
k. disagreements over chartering or registration of labor organizations and collective bargaining agreements;
l. violations of the rights and conditions of union or workers association membership;
m. violations of the rights of legitimate labor organizations, except interpretation of collective bargaining agreements;
n. such other disputes or conflicts involving the rights to self-organization, union membership and collective bargaining
a. between and among legitimate labor organizations;
b. between and among members of a union or workers association.
San Beda College of Law 408
4S: 2015 - 2016
LABOR LAW REVIEW Atty. Joyrich Golangco

It is clear from the foregoing that the issues raised by petitioners do not fall under any of the aforementioned
circumstances constituting an intra-union dispute. More importantly, the petitioners do not seek a determination
of whether it is the Facundo group (EUBP) or the Remigio group (REUBP) which is the true set of union officers.
Instead, the issue raised pertained only to the validity of the acts of management in light of the fact that it still
has an existing CBA with EUBP. Thus as to Bayer, Lonishen and Amistoso the question was whether they
were liable for unfair labor practice, which issue was within the jurisdiction of the NLRC. The dismissal
of the second ULP complaint was therefore erroneous.

However, as to respondents Remigio and Villareal, we find that petitioners complaint was validly
dismissed.

Petitioners ULP complaint cannot prosper as against respondents Remigio and Villareal because the issue, as
against them, essentially involves an intra-union dispute based on Section 1 (n) of DOLE Department Order No.
40-03. To rule on the validity or illegality of their acts, the Labor Arbiter and the NLRC will necessarily touch on
the issues respecting the propriety of their disaffiliation and the legality of the establishment of REUBP issues
that are outside the scope of their jurisdiction. Accordingly, the dismissal of the complaint was validly made, but
only with respect to these two respondents.


SECTION 2. Coverage. Other related labor relations disputes shall include any conflict between a labor union and the employer
or any individual, entity or group that is not a labor organization or workers association. This includes: (1) cancellation of registration of
unions and workers associations; and (2) a petition for interpleader.

San Beda College of Law 409
4S: 2015 - 2016
LABOR LAW REVIEW Atty. Joyrich Golangco

ARTICLE 252

UNION OF FILIPRO EMPLOYEES - DRUG, FOOD AND ALLIED INDUSTRIES UNIONS - KILUSANG MAYO
UNO (UFE-DFA-KMU)
G.R. Nos. 158930-31
March 3, 2008

MEANING OF DUTY TO BARGAIN COLLECTIVELY (In good faith)

DOCTRINE: For a charge of unfair labor practice to prosper, it must be shown that Nestl was motivated by ill
will, "bad faith, or fraud, or was oppressive to labor, or done in a manner contrary to morals, good customs, or
public policy, and, of course, that social humiliation, wounded feelings, or grave anxiety resulted x x x" in
disclaiming unilateral grants as proper subjects in their collective bargaining negotiations. While the law makes it
an obligation for the employer and the employees to bargain collectively with each other, such compulsion does
not include the commitment to precipitately accept or agree to the proposals of the other. All it contemplates is
that both parties should approach the negotiation with an open mind and make reasonable effort to reach a
common ground of agreement.

CHICO-NAZARIO, J.:

FACTS: UFE-DFA-KMU was the sole and exclusive bargaining agent of the rank-and-file employees of Nestl
belonging to the latters Alabang and Cabuyao plants. (CBA) between Nestl and UFE-DFA-KMU was to end on
5 June 2001, the Presidents of the Alabang and Cabuyao Divisions of UFE-DFA-KMU informed Nestl of their
intent to "open [our] new Collective Bargaining Negotiation for the year 2001-2004 x x x as early as June 2001."
Nestl informed them that it was also preparing its own counter-proposal.

Dialogue between the company and the union thereafter ensued.

Nestl requested NCMB to conduct preventive mediation proceedings between it and UFE-DFA-KMU owing to
an alleged impasse in said dialogue.

Conciliation proceedings proved ineffective UFE-DFA-KMU filed a Notice of Strike with the NCMB, complaining,
in essence, of a bargaining deadlock pertaining to economic issues, i.e., "retirement (plan). Another Notice of
Strike was filed by the union, this time predicated on Nestls alleged unfair labor practices.

Prior to holding the strike, Nestl filed with the DOLE a Petition for Assumption of Jurisdiction, praying for the
Secretary of the DOLE, Hon. Patricia A. Sto. Tomas, to assume jurisdiction over the current labor dispute.
15
Sec. Sto. Tomas issued an Order assuming jurisdiction over the subject labor dispute.
Accordingly, any strike or lockout is hereby enjoined.
The parties are further directed to meet and convene for the discussion before the NCMB. If no
settlement of all the issues is reached, this Office shall thereafter define the outstanding issues and
order the filing of position papers for a ruling on the merits.

Despite the order enjoining the conduct of any strike or lockout and conciliation efforts by the NCMB, the
employee members of UFE-DFA-KMU at Nestls Cabuyao Plant went on strike.

Sec. Sto. Tomas directed them to Return-to-Work. Notwithstanding the Return-to-Work Order, the members of
UFE-DFA-KMU continued with their strike, thus, prompting Sec. Sto. Tomas to seek the assistance of the
Philippine National Police (PNP) for the enforcement of said order.

Nestl and UFE-DFA-KMU filed their respective position papers.

UFE-DFA-KMU filed several pleadings, one of which was a Manifestation with Motion for Reconsideration. The
union posited that Sec. Sto. Tomas "could only assume jurisdiction over the issues mentioned in the notice of
strike subject of the current dispute. Sec. Sto. Tomas denied the motion for reconsideration of UFE-DFA-KMU.

Thereafter, UFE-DFA-KMU filed a Petition for Certiorari before the Court of Appeals.

San Beda College of Law 410


4S: 2015 - 2016
LABOR LAW REVIEW Atty. Joyrich Golangco

In the interim, in an attempt to finally resolve the crippling labor dispute between the parties, then Acting
Secretary of the DOLE, Hon. Arturo D. Brion, came out with an Order that:
Retirement Plan not a mandatory subject for bargaining; ULP against the Company is hereby
dismissed; all union demands that are not covered by the provisions of the CBAs are hereby denied; the
parties shall execute their CBA within thirty (30) days from receipt of this Order.

UFE-DFA-KMU moved to reconsider the aforequoted ruling, but such was subsequently denied.

For the second time, UFE-DFA-KMU went to the Court of Appeals via another Petition for Certiorari seeking to
annul the Orders.

CA: promulgated its Decision on the twin petitions for certiorari, ruling entirely in favor of UFE-DFA-KMU.

Both parties appealed the ruling. UFE-DFA-KMU argues therein that Nestls "refusal to bargain on a very
important CBA economic provision constitutes unfair labor practice." It explains that Nestl set as a precondition
for the holding of collective bargaining negotiations the non-inclusion of the issue of Retirement Plan.

ISSUE/S:
1) WON Nestle is guilty of ULP.
2) WON the DOLE Secretarys jurisdiction was limited to the resolution of questions and matters pertaining
merely to the ground rules of the collective bargaining negotiations
3) WON the order of resumption of negotiations respecting the Retirement Plan directs the parties to
submit to a voluntary mode of dispute settlement:

RULING:
1.) NO.
The duty to bargain collectively is mandated by Labor Code
ART. 252. Meaning of duty to bargain collectively. The duty to bargain collectively means the
performance of a mutual obligation to meet and convene promptly and expeditiously in good faith for the
purpose of negotiating an agreement with respect to wages, hours, of work and all other terms and
conditions of employment including proposals for adjusting any grievances or questions arising under
such agreement and executing a contract incorporating such agreements if requested by either party
but such duty does not compel any party to agree to a proposal or to make any concession.
The statutes invite and contemplate a collective bargaining contract, but they do not compel one. The duty to
bargain does not include the obligation to reach an agreement.

For a charge of unfair labor practice to prosper, it must be shown that Nestl was motivated by ill will, "bad faith,
or fraud, or was oppressive to labor, or done in a manner contrary to morals, good customs, or public policy,
and, of course, that social humiliation, wounded feelings, or grave anxiety resulted x x x" In disclaiming unilateral
grants as proper subjects in their collective bargaining negotiations. While the law makes it an obligation for the
employer and the employees to bargain collectively with each other, such compulsion does not include the
commitment to precipitately accept or agree to the proposals of the other. All it contemplates is that both parties
should approach the negotiation with an open mind and make reasonable effort to reach a common ground of
agreement.

Herein, the union merely bases its claim of refusal to bargain on a letter by Nestl where the latter laid down its
position that "unilateral grants, one-time company grants, company-initiated policies and programs, which
include, but are not limited to the Retirement Plan, Incidental Straight Duty Pay and Calling Pay Premium, are
by their very nature not proper subjects of CBA negotiations and therefore shall be excluded therefrom." Said
letter is not tantamount to refusal to bargain.

The corporation simply wanted to exclude the Retirement Plan from the issues to be taken up during CBA
negotiations, on the postulation that such was in the nature of a unilaterally granted benefit. An employers
steadfast insistence to exclude a particular substantive provision is no different from a bargaining
representatives perseverance to include one that they deem of absolute necessity.

It is not enough that the union believed that the employer committed acts of unfair labor practice when the
circumstances clearly negate even a prima facie showing to warrant such a belief.

San Beda College of Law 411


4S: 2015 - 2016
LABOR LAW REVIEW Atty. Joyrich Golangco

Employers are accorded rights and privileges to assure their self-determination and independence and
reasonable return of capital.
There is no per se test of good faith in bargaining. Good faith or bad faith is an inference to be drawn from the
facts.

2.) NO. The Secretary of the DOLE simply relied on the Notices of Strike that were filed by UFE-DFA-KMU
alleging bargaining deadlock. Based on the Notices of Strike filed by UFE-DFA-KMU, the Secretary of the DOLE
rightly decided on matters of substance. That the union later on changed its mind is of no moment because to
give premium to such would make the legally mandated discretionary power of the Dole Secretary subservient
to the whims of the parties.

3.) NO.
Nowhere in the Courts previous Decision did we require parties to submit to negotiate by themselves the tenor
of the retirement benefits of the concerned employees of Nestl, precisely because the Secretary of the DOLE
had already assumed jurisdiction over the labor dispute subject of herein petitions.

The Secretary of the DOLE has been explicitly granted by Article 263(g) of the Labor Code the authority to
assume jurisdiction over a labor dispute causing or likely to cause a strike or lockout in an industry
indispensable to the national interest, and decide the same accordingly. And, as a matter of necessity, it
includes questions incidental to the labor dispute; that is, issues that are necessarily involved in the dispute
itself, and not just to that ascribed in the Notice of Strike or otherwise submitted to him for resolution.

The Secretary having already assumed jurisdiction over the labor dispute subject of these consolidated
petitions, the issue concerning the retirement benefits of the concerned employees must be remanded back to
him for proper disposition.

San Beda College of Law 412


4S: 2015 - 2016
LABOR LAW REVIEW Atty. Joyrich Golangco

UST FACULTY UNION VS. UNIVERSITY OF SANTO TOMAS,
April 7, 2009

ULP BY EMPLOYER; DUTY TO BARGAIN IN GOOD FAITH

DOCTRINE: It is not the duty or obligation of respondents to inquire into the validity of the election of the
Gamilla Group. Such issue is properly an intra-union controversy subject to the jurisdiction of the med-arbiter of
the DOLE.

FACTS: (USTFU) wrote a letter to all its members informing them of a General Assembly (GA) that was to be
held on October 5, 1996. The letter contained an agenda for the GA which included an election of officers. The
then incumbent president of the USTFU was Atty. Eduardo J. Mario, Jr. (Mario group/USTFU)

Secretary General of the UST, issued a Memorandum allowing the request of the Faculty Clubs of the university
to hold a convocation on October 4, 1996.

Members of the faculties of the university attended the convocation, including members of the USTFU, without
the participation of the members of the UST administration. Also during the convocation, an election for the
officers of the USTFU was conducted by a group called the Reformist Alliance. Upon learning that the
convocation was intended to be an election, members of the USTFU walked out. Meanwhile, an election was
conducted among those present, (Gamilla Group) were elected as the president and officers, respectively, of the
union. Such election was communicated to the UST administration.

Mario Group filed a complaint for ULP against the UST with the Arbitration Branch of the NLRC. It also filed a
complaint with the Office of the Med-Arbiter of (DOLE), praying for the nullification of the election of the Gamilla
Group as officers of the USTFU.

(CBA) was entered into by the Gamilla Group and the UST. The CBA superseded an existing CBA entered into
by the UST and USTFU.

Gamilla, accompanied by Chief Security Officer of the UST, padlocked the office of the USTFU. Afterwards, an
armed security guard of the UST was posted in front of the USTFU office.

MED-ARBITER: election of the Gamilla group as null and void and ordering that this group cease and desist
from performing the duties and responsibilities of USTFU officers.
(NOTE: THIS DECISION CAME ONLY AFTER THE SC HAS ALREADY DECIDED THIS CASE)

DIRECTOR OF BLR: Affirmed Med-Arbiter

NLRC: The complaint was dismissed on the ground that USTFU failed to establish with clear and convincing
evidence that indeed UST was guilty of ULP. UST merely granted the request of faculty members to hold such
convocation. Moreover, by USTFUs own admission, no member of the UST administration attended or
participated in the convocation.

As to the CBA, the labor arbiter ruled that when the new CBA was entered into, (1) the Gamilla Group presented
more than sufficient evidence to establish that they had been duly elected as officers of the USTFU; and (2) the
ruling of the med-arbiter that the election of the Gamilla Group was null and void was not yet final and
executory. Thus, UST was justified in dealing with and entering into a CBA with the Gamilla Group, including
helping the Gamilla Group in securing the USTFU office.

CA: affirmed NLRC.

Petitioner enumerates the acts constituting ULP as follows: (1) Atty. Domingo Legaspi, the legal counsel for the
UST, conducted a faculty meeting in his office, supplying derogatory information about the Mario Group; (2)
respondents provided the Gamilla Group with the facilities and forum to conduct elections, in the guise of a
convocation; and (3) respondents transacted business with the Gamilla Group such as the processing of
educational and hospital benefits, deducting USTFU dues from the faculty members without turning over the
dues to the Mario Group, and entering into a CBA with them.
San Beda College of Law 413
4S: 2015 - 2016
LABOR LAW REVIEW Atty. Joyrich Golangco

ISSUE/S: WON UST Is Guilty of ULP

RULING: NO (NOTE: At the time of these events, the legitimacy of the Gamilla Group as the valid officers and
directors of the USTFU was already submitted to the med-arbiter and no decision had yet been reached on the
matter.)

The general principle is that one who makes an allegation has the burden of proving it. In order to show that
the employer committed ULP under the Labor Code, substantial evidence is required to support the
claim. Substantial evidence has been defined as such relevant evidence as a reasonable mind might accept as
adequate to support a conclusion.

With regard to the alleged derogatory remarks of Atty. Legaspi, the three tribunals correctly ruled that there was
no evidence to support such allegation.

In no way can the contents of the memorandum be interpreted to mean that faculty members were required to
attend the convocation. Not one coercive term was used in the memorandum to show that the faculty club
members were compelled to attend such convocation. And the phrase we are allowing them to hold a
convocation negates any idea that the UST would participate in the proceedings.

ART. 252. Meaning of duty to bargain collectively.The duty to bargain collectively


means the performance of a mutual obligation to meet and convene promptly and
expeditiously in good faith for the purpose of negotiating an agreement with respect to
wages, hours of work and all other terms and conditions of employment including proposals for
adjusting any grievances or questions arising under such agreement and executing a contract
incorporating such agreements if requested by either party but such duty does not compel any
party to agree to a proposal or to make any concession.

In the instant case, until our Decision in G.R. No. 131235 that the Gamilla Group was not validly elected into
office, there was no reason to believe that the members of the Gamilla Group were not the validly elected
officers and directors of USTFU. To reiterate, the Gamilla Group submitted a Letter whereby it informed
Secretary General of UST that its members were the newly elected officers and directors of USTFU. In the
Letter, every officer allegedly elected was identified with the Letter signed by the alleged newly elected
Secretary General and President.

The records are bereft of any evidence to show that the Mario Group informed the UST of their objections to the
election of the Gamilla Group.

It is not the duty or obligation of respondents to inquire into the validity of the election of the Gamilla Group.
Such issue is properly an intra-union controversy subject to the jurisdiction of the med-arbiter of the DOLE.
Respondents could not have been expected to stop dealing with the Gamilla Group on the mere accusation of
the Mario Group that the former was not validly elected into office.

The subsequent ruling of this Court in G.R. No. 131235 that the Gamilla Group was not validly elected into office
cannot support petitioners allegation of ULP. Had respondents dealt with the Gamilla Group after our ruling in
G.R. No. 131235 had become final and executory, it would have been a different story. As the CA ruled
correctly, until the validity of the election of the Gamilla Group is resolved with finality, respondents could not be
faulted for negotiating with said group.

As to the padlocking of the USTFU office, it must be emphasized that based on the Certification of person
padlocking that the Commander of security agency was merely present which cannot be equated to a positive
act of aiding the Gamilla Group in securing the USTFU office.

Having been shown evidence to support the legitimacy of the Gamilla Group with no counter-evidence from the
Mario Group, UST had to recognize the Gamilla Group and negotiate with it. Thus, the acts of UST in support of
the USTFU as the legitimate representative of the bargaining unit, albeit through the Gamilla Group, cannot be
considered as ULP.

San Beda College of Law 414


4S: 2015 - 2016
LABOR LAW REVIEW Atty. Joyrich Golangco

GENERAL MILLING CORPORATION, petitioner, vs. HON. COURT OF APPEALS, GENERAL MILLING
CORPORATION INDEPENDENT LABOR UNION (GMC-ILU), and RITO MANGUBAT, respondents.

FACTS: In its two plants located at Cebu City and Lapu-Lapu City, petitioner General Milling Corporation (GMC)
employed 190 workers. They were all members of private respondent General Milling Corporation Independent
Labor Union (union, for brevity), a duly certified bargaining agent. A day before the expiration of the CBA, the
union sent GMC a proposed CBA, with a request that a counter-proposal be submitted within ten (10) days.
GMC had received collective and individual letters from workers who stated that they had withdrawn from their
union membership, on grounds of religious affiliation and personal differences. Believing that the union no
longer had standing to negotiate a CBA, GMC did not send any counter-proposal. Thus, the union filed, on July
2, 1992, a complaint against GMC with the NLRC, Arbitration Division, Cebu City. The complaint alleged unfair
labor practice on the part of GMC for: (1) refusal to bargain collectively; (2) interference with the right to self-
organization; and (3) discrimination. The labor arbiter dismissed the case with the recommendation that a
petition for certification election be held to determine if the union still enjoyed the support of the workers.

NLRC ordered GMC to abide by the CBA draft that the union proposed for a period of two (2) years. NLRC
pointed out that upon the effectivity of Rep. Act No. 6715, the duration of a CBA, insofar as the representation
aspect is concerned, is five (5) years which, in the case of GMC-Independent Labor Union was from December
1, 1988 to November 30, 1993. All other provisions of the CBA are to be renegotiated not later than three (3)
years after its execution. Thus, the NLRC held that respondent union remained as the exclusive bargaining
agent with the right to renegotiate the economic provisions of the CBA. Consequently, it was unfair labor
practice for GMC not to enter into negotiation with the union.

ISSUE/S: Did the company commit ULP?

RULING: Yes. The law mandates that the representation provision of a CBA should last for five years. The
relation between labor and management should be undisturbed until the last 60 days of the fifth year. Hence, it
is indisputable that when the union requested for a renegotiation of the economic terms of the CBA on
November 29, 1991, it was still the certified collective bargaining agent of the workers, because it was seeking
said renegotiation within five (5) years from the date of effectivity of the CBA on December 1, 1988. The unions
proposal was also submitted within the prescribed 3-year period from the date of effectivity of the CBA, albeit
just before the last day of said period. It was obvious that GMC had no valid reason to refuse to negotiate in
good faith with the union. For refusing to send a counter-proposal to the union and to bargain anew on the
economic terms of the CBA, the company committed an unfair labor practice under Article 248 of the Labor
Code

San Beda College of Law 415


4S: 2015 - 2016
LABOR LAW REVIEW Atty. Joyrich Golangco

KIOK LOY v. NLRC
G.R. No. L-54334
January 22, 1986

FACTS: Pambansang Kilusang Paggawa (Union for short) won and was subsequently certified in a resolution
by the BLR in a certification election as the sole and exclusive bargaining agent of the rank-and-file employees
of Sweden Ice Cream Plant (Company for short). The Company's motion for reconsideration of the said
resolution was denied.

The Company with two copies of its proposed collective bargaining agreement. At the same time, it requested
the Company for its counter proposals. Eliciting no response to the aforesaid request, the Union again wrote the
Company reiterating its request for collective bargaining negotiations and for the Company to furnish them with
its counter proposals. Both requests were ignored and remained unacted upon by the Company.

The Union filed a "Notice of Strike", with the BLR on ground of unresolved economic issues in collective
bargaining. Conciliation proceedings then followed during the thirty-day statutory cooling-off period. But all
attempts towards an amicable settlement failed, prompting the Bureau of Labor Relations to certify the case to
the NLRC for compulsory arbitration.

The Union submitted its position paper. The Company did not, and instead requested for a resetting which was
granted. The case was further reset to May 11, 1979 due to the withdrawal of the Company's counsel of record.
Atty. Fortunato Panganiban formally entered his appearance as counsel for the Company only to request for
another postponement allegedly for the purpose of acquainting himself with the case. Meanwhile, the Company
submitted its position paper on May 28, 1979. the Company's representative, Mr. Ching, who was supposed to
be examined, failed to appear. Atty. Panganiban then requested for another postponement which the labor
arbiter denied.

LA ruled that the Company has waived its right to present further evidence and, therefore, considered the case
submitted for resolution.

NLRC: Guilty of unjustified refusal to bargain

Petitioner Company now maintains that its right to procedural due process has been violated when it was
precluded from presenting further evidence in support of its stand and when its request for further postponement
was denied; that when the Union furnished them with a copy of the proposed Collective Bargaining Agreement
and it was only then that they came to know of the Union's demands; and finally, that the Collective Bargaining
Agreement approved and adopted by the NLRC is unreasonable and lacks legal basis.

RULING: Petition lacks merit. There can be no doubt that the Union has a valid cause to complain against its
(Company's) attitude, the totality of which is indicative of the latter's disregard of, and failure to live up to, what is
enjoined by the Labor Code to bargain in good faith.

Collective bargaining is one of the democratic frameworks under the New Labor Code, designed to stabilize the
relation between labor and management and to create a climate of sound and stable industrial peace. It is a
mutual responsibility of the employer and the Union and is characterized as a legal obligation.

While it is a mutual obligation of the parties to bargain, the employer, however, is not under any legal duty to
initiate contract negotiation. The mechanics of collective bargaining is set in motion only when the following
jurisdictional preconditions are present, namely, (1) possession of the status of majority representation of the
employees' representative in accordance with any of the means of selection or designation provided for by the
Labor Code; (2) proof of majority representation; and (3) a demand to bargain under Article 251, par. (a) of the
New Labor Code all of which preconditions are undisputedly present in the instant case.

We are in total conformity with respondent NLRC's pronouncement that petitioner Company is GUILTY of unfair
labor practice. It has been indubitably established that (1) respondent Union was a duly certified bargaining
agent; (2) it made a definite request to bargain, accompanied with a copy of the proposed Collective Bargaining
Agreement, to the Company not only once but twice which were left unanswered and unacted upon; and (3) the
Company made no counter proposal whatsoever all of which conclusively indicate lack of a sincere desire to
negotiate.
San Beda College of Law 416
4S: 2015 - 2016
LABOR LAW REVIEW Atty. Joyrich Golangco

A Company's refusal to make counter proposal if considered in relation to the entire bargaining process, may
indicate bad faith and this is specially true where the Union's request for a counter proposal is left unanswered.
Even during the period of compulsory arbitration before the NLRC, petitioner Company's approach and attitude-
stalling the negotiation by a series of postponements, non-appearance at the hearing conducted, and undue
delay in submitting its financial statements, lead to no other conclusion except that it is unwilling to negotiate
and reach an agreement with the Union. Petitioner has not at any instance, evinced good faith or willingness to
discuss freely and fully the claims and demands set forth by the Union much less justify its opposition thereto.

Certainly, the moves and overall behavior of petitioner-company were in total derogation of the policy enshrined
in the New Labor Code which is aimed towards expediting settlement of economic disputes. Hence, this Court is
not prepared to affix its imprimatur to such an illegal scheme and dubious maneuvers.

Where the intervention of the National Labor Relations Commission was properly sought for after conciliation
efforts undertaken by the BLR failed. The instant case being a certified one, it must be resolved by the NLRC
pursuant to the mandate of P.D. 873, as amended, which authorizes the said body to determine the
reasonableness of the terms and conditions of employment embodied in any Collective Bargaining Agreement.
To that extent, utmost deference to its findings of reasonableness of any Collective Bargaining Agreement as
the governing agreement by the employees and management must be accorded due respect by this Court.

San Beda College of Law 417


4S: 2015 - 2016
LABOR LAW REVIEW Atty. Joyrich Golangco

COLEGIO DE SAN JUAN DE LETRAN, petitioner, vs. ASSOCIATION OF EMPLOYEES AND FACULTY OF
LETRAN and ELEONOR AMBAS, respondents.

FACTS: The President of respondent union, Association of Employees and Faculty of Letran, initiated the
renegotiation of its Collective Bargaining Agreement with petitioner Colegio de San Juan de Letran for the last
two (2) years of the CBA's five (5) year lifetime from 1989-1994. The newly elected President wanted to
continue the renegotiation of the CBA but petitioner, through Fr. Edwin Lao, claimed that the CBA was already
prepared for signing by the parties. The parties submitted the disputed CBA to a referendum by the union
members, who eventually rejected the said CBA. Petitioner accused the union officers of bargaining in bad faith
before the NLRC. The union notified the (NCMB) of its intention to strike due to petitioner's: non-compliance with
the NLRC (1) order to delete the name of Atty. Federico Leynes as the union's legal counsel; and (2) refusal to
bargain. The union finally struck. On July 2, 1996, public respondent the Secretary of Labor and Employment
assumed jurisdiction and ordered all striking employees including the union president to return to work and for
petitioner to accept them back under the same terms and conditions before the actual strike. Petitioner
readmitted the striking members except Ambas, the new president.

Public respondent issued an order declaring petitioner guilty of unfair labor practice on two counts and directing
the reinstatement of private respondent Ambas with backwages

ISSUE/S: (1) whether petitioner is guilty of unfair labor practice by refusing to bargain with the union when it
unilaterally suspended the ongoing negotiations for a new Collective Bargaining Agreement (CBA) upon mere
information that a petition for certification has been filed by another legitimate labor organization? (2) whether
the termination of the union president amounts to an interference of the employees' right to self-organization?

RULING: Petitioner's utter lack of interest in bargaining with the union is obvious in its failure to make a timely
reply to the proposals presented by the latter. More than a month after the proposals were submitted by the
union, petitioner still had not made any counter-proposals. This inaction on the part of petitioner prompted the
union to file its second notice of strike on March 13, 1996. Petitioner could only offer a feeble explanation that
the Board of Trustees had not yet convened to discuss the matter as its excuse for failing to file its reply. In
order to allow the employer to validly suspend the bargaining process there must be a valid petition for
certification election raising a legitimate representation issue. Hence, the mere filing of a petition for certification
election does not ipso facto justify the suspension of negotiation by the employer. The petition must first comply
with the provisions of the Labor Code and its Implementing Rules. Foremost is that a petition for certification
election must be filed during the sixty-day freedom period. The "Contract Bar Rule" under Section 3, Rule XI,
Book V, of the Omnibus Rules Implementing the Labor Code, provides that: " . If a collective bargaining
agreement has been duly registered in accordance with Article 231 of the Code, a petition for certification
election or a motion for intervention can only be entertained within sixty (60) days prior to the expiry date of such
agreement." The rule is based on Article 232 in relation to Articles 253, 253-A and 256 of the Labor Code. No
petition for certification election for any representation issue may be filed after the lapse of the sixty-day freedom
period. The old CBA is extended until a new one is signed. The rule is that despite the lapse of the formal
effectivity of the CBA the law still considers the same as continuing in force and effect until a new CBA shall
have been validly executed. Hence, the contract bar rule still applies. The purpose is to ensure stability in the
relationship of the workers and the company by preventing frequent modifications of any CBA earlier entered
into by them in good faith and for the stipulated original period. Admittedly, management has the prerogative to
discipline its employees for insubordination. But when the exercise of such management right tends to interfere
with the employees' right to self-organization, it amounts to union-busting and is therefore a prohibited act. The
dismissal of Ms. Ambas was clearly designed to frustrate the Union in its desire to forge a new CBA with the
College that is reflective of the true wishes and aspirations of the Union members. Her dismissal was merely a
subterfuge to get rid of her, which smacks of a pre-conceived plan to oust her from the premises of the College.
It has the effect of busting the Union, stripping it of its strong-willed leadership. When management refused to
treat the charge of insubordination as a grievance within the scope of the Grievance Machinery, the action of the
College in finally dismissing her from the service became arbitrary, capricious and whimsical, and therefore
violated Ms. Ambas' right to due process."

San Beda College of Law 418


4S: 2015 - 2016
LABOR LAW REVIEW Atty. Joyrich Golangco

PHILIPPINE AIRLINES, INCORPORATED vs. PHILIPPINE AIRLINES EMPLOYEES ASSOCIATION (PALEA)
6
FACTS: The present petition arose from a labor complaint, filed by respondent PALEA against petitioners PAL
and one Mary Anne del Rosario, Director of Personnel of petitioner PAL, on 1 March 1989. The labor complaint
charged both petitioners with unfair labor practice for the alleged non-payment of the 13th month pay of
petitioner PALs employees who had not been regularized as of the 30 of April 1988, allegedly in contravention
of the Collective Bargaining Agreement (CBA) entered into by petitioner PAL and respondent PALEA. PAL
countered that those rank and file employees who were not regularized by 30 April of a particular year are, in
principle, not denied their 13th month pay considering they receive said mandatory bonus in the form of the
Christmas Bonus; that the Christmas Bonus given to all its employees is deemed a compliance with Presidential
Decree No. 851 and the latters implementing rules; and that the foregoing has been the practice formally
adopted in previous CBAs as early as 1970.

On 12 March 1990, the Labor Arbiter rendered a Decision dismissing the respondent PALEAs complaint for
lack of merit. The Labor Arbiter ruled that petitioner PAL was not guilty of unfair labor practice in withholding the
grant of the 13th Month Pay or Mid Year Bonus to the concerned employees. The giving of the particular bonus
was said to be merely an additional practice made in the past, "such being the case, it violated no agreement or
existing practice or committed unfair labor practice.

In refusing payment of the mid-year bonus, petitioner PAL argues that 1) the CBA does not apply to non-regular
employees such that any benefits arising from said agreement cannot be made to apply to them, including the
mid-year bonus; and 2) it has always been the company practice not to extend the mid-year bonus to those
employees who have not attained regular status prior to the month of May, when payment of the particular
bonus accrues.

ISSUE/S: whether or not the Court of Appeals committed reversible error in affirming the order of the NLRC for
the payment of the 13th month pay or mid-year bonus to its employees regularized after 30 April 1988.

RULING: We rule in the negative. It is a well-settled doctrine that the benefits of a CBA extend to the laborers
and employees in the collective bargaining unit, including those who do not belong to the chosen bargaining
32
labor organization. Otherwise, it would be a clear case of discrimination.

Hence, to be entitled to the benefits under the CBA, the employees must be members of the bargaining unit, but
not necessarily of the labor organization designated as the bargaining agent. A "bargaining unit" has been
defined as a group of employees of a given employer, comprised of all or less than all of the entire body of
employees, which the collective interest of all the employees, consistent with equity to the employer, indicates to
be the best suited to serve the reciprocal rights and duties of the parties under the collective bargaining
33
provisions of the law. At this point, the allegation of petitioner PAL that the non-regular employees do not
belong to the collective bargaining unit and are thus not covered by the CBA is unjustified and unsubstantiated.
It is apparent to us that petitioner PAL excludes certain employees from the benefits of the CBA only because
they have not yet achieved regular status by the cut-off date, 30 April 1988. There is no showing that the non-
regular status of the concerned employees by said cut-off date sufficiently distinguishes their interests from
those of the regular employees so as to exclude them from the collective bargaining unit and the benefits of the
CBA. Having ruled that the benefits provided by the subject CBA are applicable even to non-regular employees
who belong to the bargaining unit concerned, the next and crucial query to be addressed is whether the 13th
month pay or mid-year bonus can be equated to the Christmas bonus. it is crystal clear that petitioner PAL is
claiming an exemption from payment of the 13th month pay or mid-year bonus provided in the CBA under the
guise of paying the Christmas bonus which it claims to be the equivalent of the 13th month pay under
Presidential Decree No. 851.

Presidential Decree No. 851 mandates that all employers must pay all their employees receiving a basic salary
of not more than P1,000.00 a month, regardless of the nature of the employment, a 13th month pay not later
35
than 24 December of every year. Memorandum Order No. 28, dated 13 August 1986, removed the salary
ceiling, generally making all employees entitled to the 13th month pay regardless of the amount of their basic
salary, designation or employment status, and irrespective of the method by which their wages are paid,
provided that they have worked for at least one (1) month during a calendar year.

San Beda College of Law 419


4S: 2015 - 2016
LABOR LAW REVIEW Atty. Joyrich Golangco

ARTICLE 253-A

FVCLU-PTGWO vs SANAMA-FVC-SIGLO
G.R. No. 176249, November 27, 2009
Brion, J.

FACTS: On December 22, 1997, the petitioner FVCLU-PTGWO, the recognized bargaining agent of the rank-
and-file employees of the FVC Philippines, Inc., signed a five-year collective bargaining agreement with the
rd
company to cover the period from February 1, 1998 to January 30, 2003. At the end of the 3 year of the five-
year term and pursuant to the CBA, FVCLU-PTGWO and the company entered into the renegotiation of the
CBA and modified, among other provisions, the CBAs duration. The renegotiated CB provides that the
renegotiation agreement shall take effect beginning February 1, 2001 and until May 31, 2003 thus extending the
original five-year period of the CBA by four months. However, on January 21, 2003, nine days before the
January 30, 2003 expiration of the original five-year CBA, the responded SANAMA-FVC-SIGLO filed before the
Department of Labor and Employment a petition for certification election for the same rank-and-file unit covered
by the FVCLU-PTGWO CBA. Hence, FVCLU-PTGWO moved to dismiss the petition on the ground that the
certification election petition was filed outside the freedom period or outside of the sixty (60) days before the
expiration of the CBA on May 31, 2003.

The Med-Arbiter dismissed the petition on the ground that it was filed outside the 60-day period counted from
the May 31, 2003 expiry date of the amended CBA. SANAMA-SIGLO appealed the Med-Arbiters Order to the
DOLE Secretary, contending that the filing of the petition on January 21, 2003 was within 60-days from the
January 30, 2003 expiration of the original CBA term. The DOLE Secretary sustained SANAMA-SIGLOs
position, thereby setting aside the decision of the Med-Arbiter. However, the DOLE Acting Secretary later
reversed the decision upon FVCLU-PTGWOs motion for reconsideration and ruled that the amended CBA
(which extended the representation aspect of the original CBA by four [4] months) had been ratified by members
of the bargaining unit some of whom later organized themselves as SANAMA-SIGLO, the certification election
applicant. Since these SANAMA-SIGLO members fully accepted and in fact received the benefits arising from
the amendments, the Acting Secretary rationalized that they also accepted the extended term of the CBA and
cannot now file a petition for certification election based on the original CBA expiration date.

ISSUE/S: WON the certification election petition was filed within the freedom period of sixty (60) days prior to
the expiration of the CBA.

RULING:
Yes. Article 253-A of the Labor Code covers this situation and it provides:

Terms of a collective bargaining agreement. Any Collective Bargaining Agreement that the parties may
enter into, shall, insofar as the representation aspect is concerned, be for a term of five (5) years. No
petition questioning the majority status of the incumbent bargaining agent shall be entertained and no
certification election shall be conducted by the Department of Labor and Employment outside of the sixty
day period immediately before the date of expiry of such five-year term of the Collective Bargaining
Agreement. All other provisions of the Collective Bargaining Agreement shall be renegotiated not later
than three (3) years after its execution.

This Labor Code provision is implemented through Book V, Rule VIII of the Rules Implementing the Labor Code
which states:

Sec. 14. Denial of the petition; grounds. The Med-Arbiter may dismiss the petition on any of the following
grounds:

xxxx

(b) the petition was filed before or after the freedom period of a duly registered collective bargaining
agreement; provided that the sixty-day period based on the original collective bargaining agreement shall
not be affected by any amendment, extension or renewal of the collective bargaining
agreement (underscoring supplied).

San Beda College of Law 420


4S: 2015 - 2016
LABOR LAW REVIEW Atty. Joyrich Golangco

FVCLU-PTGWO has taken the view that its exclusive representation status should fully be in step with the
term of the CBA and that this status can be challenged only within 60 days before the expiration of this
term. Thus, when the term of the CBA was extended, its exclusive bargaining status was similarly extended so
that the freedom period for the filing of a petition for certification election should be counted back from the
expiration of the amended CBA term.

We hold this FVCLU-PTGWO position to be correct, but only with respect to the original five-year term of
the CBA which, by law, is also the effective period of the unions exclusive bargaining representation
status. While the parties may agree to extend the CBAs original five-year term together with all other CBA
provisions, any such amendment or term in excess of five years will not carry with it a change in the unions
exclusive collective bargaining status. By express provision of the above-quoted Article 253-A, the exclusive
bargaining status cannot go beyond five years and the representation status is a legal matter not for the
workplace parties to agree upon. In other words, despite an agreement for a CBA with a life of more than five
years, either as an original provision or by amendment, the bargaining unions exclusive bargaining status is
effective only for five years and can be challenged within sixty (60) days prior to the expiration of the CBAs first
five years.

The negotiated extension of the CBA term has no legal effect on the FVCLU-PTGWOs exclusive
bargaining representation status which remained effective only for five years ending on the original expiry date
of January 30, 2003. Thus, sixty days prior to this date, or starting December 2, 2002, SANAMA-SIGLO could
properly file a petition for certification election. Its petition, filed on January 21, 2003 or nine (9) days before the
expiration of the CBA and of FVCLU-PTGWOs exclusive bargaining status, was seasonably filed.

San Beda College of Law 421


4S: 2015 - 2016
LABOR LAW REVIEW Atty. Joyrich Golangco

SMCEU-PTGWO vs Hon. Confesor
G.R. No. 111262. September 19, 1996
Kapunan, J.

FACTS: Petitioner-union entered into a CBA with SMC to take effect upon the expiration of the previous CBA or
on June 30, 1989 until June 30, 1992. The CBA also stated that the term of the Agreement insofar as the
representation aspect is concerned, shall be for 5 years from July 1, 1989 to June 30, 1994. Hence, the freedom
period for purposes of such representation shall be sixty (60) days prior to June 30, 1994.

After June 30, 1992, the CBA was renegotiated in accordance with the terms of the CBA and Article 253-A of
the Labor Code. Negotiations started sometime in July, 1992 with the two parties submitting their respective
proposals and counterproposals. During the negotiations, the petitioner-union insisted that the bargaining unit of
SMC should still include the employees of the spun-off corporations: Magnolia and SMFI; and that the
renegotiated terms of the CBA shall be effective only for the remaining period of two years or until June 30,
1994.

SMC, on the other hand, contended that the members/employees who had moved to Magnolia and SMFI,
automatically ceased to be part of the bargaining unit at the SMC. Furthermore, the CBA should be effective for
three years in accordance with Art. 253-A of the Labor Code.

ISSUE/S:
1) WON the duration of the renegotiated terms of the CBA is to be effective for three years or for only two years;
and
2) WON the bargaining unit of SMC includes also the employees of the Magnolia and SMFI.

RULING: 1) The renegotiated terms of the CBA shall be for three (3) years.

Article 253-A states that the CBA has a term of five (5) years instead of three years, before the amendment of
the law as far as the representation aspect is concerned. All other provisions of the CBA shall be negotiated not
later than three (3) years after its execution. The "representation aspect" refers to the identity and majority
status of the union that negotiated the CBA as the exclusive bargaining representative of the appropriate
bargaining unit concerned. "All other provisions" simply refers to the rest of the CBA, economic as well as non-
economic provisions, except representation.

From the congressional discussions, the legislators were more inclined to have the period of effectivity for three
(3) years insofar as the economic as well as non-economic provisions are concerned, except representation.
Obviously, the framers of the law wanted to maintain industrial peace and stability by having both management
and labor work harmoniously together without any disturbance. Thus, no outside union can enter the
establishment within five (5) years and challenge the status of the incumbent union as the exclusive bargaining
agent. Likewise, the terms and conditions of employment (economic and non-economic) cannot be questioned
by the employers or employees during the period of effectivity of the CBA.

The CBA is a contract between the parties and the parties must respect the terms and conditions of the
agreement. Notably, the framers of the law did not give a fixed term as to the effectivity of the terms and
conditions of employment. It can be gleaned from their discussions that it was left to the parties to fix the period.
As a matter of policy the parties are encouraged to enter into a renegotiated CBA with a term which would
coincide with the aforesaid five (5) year term of the bargaining representative. In the event however, that the
parties, by mutual agreement, enter into a renegotiated contract with a term of three (3) years or one which does
not coincide with the said 5-year term, and said agreement is ratified by majority of the members in the
bargaining unit, the subject contract is valid and legal and therefore, binds the contracting parties. The same will
however not adversely affect the right of another union to challenge the majority status of the incumbent
bargaining agent within sixty (60) days before the lapse of the original five (5) year term of the CBA.

2) No. As a result of the spin-offs:

1. Each of the companies are run by, supervised and controlled by different management teams
including separate human resource/personnel managers.
2. Each Company enforces its own administrative and operational rules and policies and are not
dependent on each other in their operations.
San Beda College of Law 422
4S: 2015 - 2016
LABOR LAW REVIEW Atty. Joyrich Golangco

3. Each entity maintains separate financial statements and are audited separately from each other.

Indubitably, therefore, Magnolia and SMFI became distinct entities with separate juridical personalities. Thus,
they can not belong to a single bargaining unit. There are various factors which must be satisfied and
considered in determining the proper constituency of a bargaining unit. (1) will of the employees (Globe
Doctrine); (2) affinity and unit of employees' interest, such as substantial similarity of work and duties, or
similarity of compensation and working conditions; (3) prior collective bargaining history; and (4) employment
status, such as temporary, seasonal and probationary employees. Even assuming in gratia argumenti that at the
time of the election they were regular employees of San Miguel, nonetheless, these workers are no By RICKY
BOY CABATU/ 3B/ L-100355 longer connected with San Miguel Corporation in any manner because Magnolia
has ceased to be a division of San Miguel Corporation and has been formed into a separate corporation with a
personality of its own. This development, which was brought to our attention by private respondents, necessarily
renders moot and academic any further discourse on the propriety of the elections which petitioners impugn via
the recourse.

San Beda College of Law 423


4S: 2015 - 2016
LABOR LAW REVIEW Atty. Joyrich Golangco

ARTICLE 254

PAQUITO ANDO, et al. v. ANDRESITO CAMPO, et al.


G.R. No. 184007
February 16, 2011
NACHURA, J.:

DOCTRINE: Regular courts have no jurisdiction to hear and decide questions which arise from and are
incidental to the enforcement of decisions, orders, or awards rendered in labor cases by appropriate officers and
tribunals of the Department of Labor and Employment. This is further amplified by Article 254 of the Labor Code
which provides that no temporary or permanent injunction or restraining order in any case involving or growing
out of labor disputes shall be issued by any court or other entity, except as otherwise provided in Articles 218
and 264 of the Code.

FACTS: Pacquito Ando was president of Premier Allied and Contracting Services, Inc. (PACSI), an independent
labor contractor. PACSI hired Campo et al as its employees. They were, however, dismissed from employment.
They filed a case for illegal dismissal before the Labor Arbiter. The dismissed employees won the case and
moved for execution upon finality of the decision.

Consequently, the NLRC Acting Sheriff levied upon a real property covered by a TCT in the name of Paquito V.
Ando x x x married to Erlinda S. Ando. This prompted Ando to file an action for prohibition and damages with
prayer for the issuance of a TRO before the RTC. He claimed that the property belonged to him and his wife, not
to the corporation, and, hence, could not be subject of the execution sale.

The RTC denied the prayer for a TRO, holding that the trial court had no jurisdiction to try and decide the case.
The RTC ruled that, pursuant to the NLRC Manual on the Execution of Judgment, Andos remedy was to file a
third-party claim with the NLRC Sheriff. This ruling was affirmed by the CA.

Ando then went to the SC arguing that he was never sued in his personal capacity, but in his representative
capacity as president of PACSI. He concedes that the Labor Arbiters decision has become final. Hence, he is
not seeking to stop the execution of the judgment against the properties of PACSI.

ISSUE/S: Were the trial and appellate courts correct in dismissing Andos prayer for a TRO against the NLRC?

RULING: YES. The Court has long recognized that regular courts have no jurisdiction to hear and decide
questions which arise from and are incidental to the enforcement of decisions, orders, or awards rendered in
labor cases by appropriate officers and tribunals of the Department of Labor and Employment. This is further
amplified by Article 254 of the Labor Code which provides that no temporary or permanent injunction or
restraining order in any case involving or growing out of labor disputes shall be issued by any court or other
entity, except as otherwise provided in Articles 218 and 264 of the Code.

In this case, the CA did not err in upholding the RTCs lack of jurisdiction to restrain the implementation of the
writ of execution issued by the Labor Arbiter. It is, first and foremost, the NLRC Manual on the Execution of
Judgment that governs any question on the execution of a judgment of that body.

Clearly, Ando need not look further than that. The Rules of Court apply only by analogy or in a suppletory
character.

Note: The SC however granted Andos prayer to exclude the subject conjugal property from the execution
proceedings, in effect treating Andos petition as a third-party claim.

San Beda College of Law 424


4S: 2015 - 2016
LABOR LAW REVIEW Atty. Joyrich Golangco

ARTICLE 255

INTERNATIONAL SCHOOL ALLIANCE OF EDUCATORS (ISAE), petitioner, vs. HON. LEONARDO A.


QUISUMBING
G.R. No. 128845
June 1, 2000
KAPUNAN, J.:

DOCTRINE: A bargaining unit is "a group of employees of a given employer, comprised of all or less than all of
the entire body of employees, consistent with equity to the employer indicate to be the best suited to serve the
reciprocal rights and duties of the parties under the collective bargaining provisions of the law." The factors in
determining the appropriate collective bargaining unit are (1) the will of the employees (Globe Doctrine); (2)
affinity and unity of the employees' interest, such as substantial similarity of work and duties, or similarity of
compensation and working conditions (Substantial Mutual Interests Rule); (3) prior collective bargaining history;
and (4) similarity of employment status.

FACTS: In this well-known case centrally focusing on the discrimination of locally hired workers as against
foreign hired workers in an International School, one of the issues raised aside from the equal pay for equal
work concept was the propriety of including foreign hired teachers in the same bargaining unit as the local hires
of the school.

The collective bargaining history in the School shows that these groups were always treated separately.
Foreign-hires have limited tenure; local-hires enjoy security of tenure. Although foreign-hires perform similar
functions under the same working conditions as the local-hires, foreign-hires are accorded certain benefits not
granted to local-hires. These benefits, such as housing, transportation, shipping costs, taxes, and home leave
travel allowance, are reasonably related to their status as foreign-hires, and justify the exclusion of the former
from the latter.

ISSUE/S: Are the foreign hired teachers to be included under the same bargaining unit as the local hires?

RULING: NO. A bargaining unit is "a group of employees of a given employer, comprised of all or less than all of
the entire body of employees, consistent with equity to the employer indicate to be the best suited to serve the
reciprocal rights and duties of the parties under the collective bargaining provisions of the law." The factors in
determining the appropriate collective bargaining unit are (1) the will of the employees (Globe Doctrine); (2)
affinity and unity of the employees' interest, such as substantial similarity of work and duties, or similarity of
compensation and working conditions (Substantial Mutual Interests Rule); (3) prior collective bargaining history;
and (4) similarity of employment status.

The basic test of an asserted bargaining unit's acceptability is whether or not it is fundamentally the combination
which will best assure to all employees the exercise of their collective bargaining rights.

Here, it does not appear that foreign-hires have indicated their intention to be grouped together with local-hires
for purposes of collective bargaining. The collective bargaining history in the School also shows that these
groups were always treated separately. Foreign-hires have limited tenure; local-hires enjoy security of tenure.
Although foreign-hires perform similar functions under the same working conditions as the local-hires, foreign-
hires are accorded certain benefits not granted to local-hires. These benefits, such as housing, transportation,
shipping costs, taxes, and home leave travel allowance, are reasonably related to their status as foreign-hires,
and justify the exclusion of the former from the latter. To include foreign-hires in a bargaining unit with local-hires
would not assure either group the exercise of their respective collective bargaining rights.

San Beda College of Law 425


4S: 2015 - 2016
LABOR LAW REVIEW Atty. Joyrich Golangco

NATIONAL ASSOCIATION OF FREE TRADE UNIONS (NAFTU) vs. MAINIT LUMBER DEVELOPMENT
COMPANY WORKERS UNION-UNITED LUMBER AND GENERAL WORKERS OF THE PHILIPPINES.
(MALDECOWU-ULGWP)
G.R. No. 79526
December 21, 1990
192 SCRA 598

FACTS: Private respondent Mainit Lumber Development Company Workers Union-United Lumber and General
Workers of the Philippines, MALDECOWU-ULGWP (ULGWP, for short), a legitimate labor organization filed a
petition for certification election to determine the sole and exclusive collective bargaining representative among
the rank and file workers/employees of Mainit Lumber Development Company Inc. (MALDECO), employing
approximately 136 rank and file employees/workers. The Med-Arbiter granted the petition for certification
election composed of only one bargaining . NAFTU appealed the decision of the Med-Arbiter on the ground that
MALDECO was composed of two (2) bargaining units, the Sawmill Division and the Logging Division contending
that the employer MALDECO was composed of two bargaining units, the Sawmill Division in Butuan City and
the Logging Division, in Zapanta Valley, Kitcharao, Agusan Norte, about 80 kilometers distant from each other
and in fact, had then two separate CBA's, one for the Sawmill Division and another for the Logging Division,
both the petition and decision referred only to one bargaining unit; that from 1979 to 1985, the Ministry of Labor
and Employment recognized the existence of two (2) separate bargaining units at MALDECO, one for its
Logging Division and another for its Sawmill Division. But both the petition and decision treated these separate
and distinct units as one.

ISSUE/S: Whether or not it was right for the med-arbiter to change the employer from two separate bargaining
units to only one.

RULING: Yes. First, out of two hundred and one (201) employees of MALDECO, one hundred seventy five
(175) consented and supported the petition for certification election, thereby confirming their desire for one
bargaining representative.

Second, while the existence of a bargaining history is a factor that may be reckoned with in determining the
appropriate bargaining unit, the same is not decisive or conclusive. Other factors must be considered. The test
of grouping is community or mutuality of interests. This is so because "the basic test of an asserted bargaining
unit's acceptability is whether or not it is fundamentally the combination which will best assure to all employees
the exercise of their collective bargaining rights."

Certainly, there is a mutuality of interest among the employees of the Sawmill Division and the Logging Division.
Their functions mesh with one another. One group needs the other in the same way that the company needs
them both. There may be difference as to the nature of their individual assignments but the distinctions are not
enough to warrant the formation of a separate bargaining unit.

San Beda College of Law 426


4S: 2015 - 2016
LABOR LAW REVIEW Atty. Joyrich Golangco

ARTICLE 256

PICOP RESOURCES, INCORPORATED (PRI), vs. RICARDO DEQUILLA et.al.


G.R. No. 172666
December 7, 2011

FACTS: Ricardo Dequilla, Cesar Atienza and Aniceto Orbeta (private respondents), members of the NAMAPRI-
SPFL, a duly registered labor organization and existing bargaining agent of the PICOP rank-and-file employee,
were dismissed for allegedly campaigning, supporting and signing a petition for the certification of a rival union,
the Federation of Free Workers Union (FFW) before the 60-day freedom period and during the effectivity of the
CBA. Respondents were terminated from employment based on the alleged acts of disloyalty they committed
when they signed an authorization for the Federation of Free Workers (FFW) to file a Petition for Certification
Election among all rank-and-file employees. It contends that the acts of respondents are a violation of the Union
Security Clause, as provided in their Collective Bargaining Agreement. Such acts of disloyalty were construed to
be a valid cause for termination under the terms and conditions of the CBA. LA rendered a decision declaring
as illegal the termination of the private respondents and ordering reinstatement. PICOP elevated the LA
decision to the NLRC but its appeal was dismissed.

ISSUE/S:
(1) WON the act of signing of authorization letter in favor of FFW to file a petition for certification election
constitutes a violation of the Union Security Clause.
(2) WON the termination is valid.

RULING:

(1) No. There is no question that in the CBA entered into by the parties, there is a union security clause. The
clause imposes upon the workers the obligation to join and maintain membership in the companys recognized
union as a condition for employment.

The Court ruled that the acts of private respondents are not enough proof of a violation of the Union Security
Clause which would warrant their dismissal. PICOP failed to show in detail how private respondents
campaigned and supported FFW. Their mere act of signing an authorization for a petition for certification
election before the freedom period does not necessarily demonstrate union disloyalty. It is far from being within
the definition of acts of disloyalty as PICOP would want the Court to believe. The act of signing an authorization
for a petition for certification election is not disloyalty to the union per se considering that the petition for
certification election itself was filed during the freedom period which started on March 22, 2000.

There was no proof of any contemporaneous acts of resignation or withdrawal of union membership or non-
payment of union dues. Neither is there proof that private respondents joined FFW. The fact is, private
respondents remained in good standing with their union, NAMAPRI-SPFL.

"Union security" is a generic term, which is applied to and comprehends "closed shop," "union shop,"
"maintenance of membership," or any other form of agreement which imposes upon employees the obligation to
acquire or retain union membership as a condition affecting employment. There is union shop when all new
regular employees are required to join the union within a certain period as a condition for their continued
employment. There is maintenance of membership shop when employees, who are union members as of the
effective date of the agreement, or who thereafter become members, must maintain union membership as a
condition for continued employment until they are promoted or transferred out of the bargaining unit, or the
agreement is terminated. A closed shop, on the other hand, may be defined as an enterprise in which, by
agreement between the employer and his employees or their representatives, no person may be employed in
any or certain agreed departments of the enterprise unless he or she is, becomes, and, for the duration of the
agreement, remains a member in good standing of a union entirely comprised of or of which the employees in
interest are a part.

(2) Theres no sufficient evidence to support the termination of the employees.


The mere signing of the authorization in support of the Petition for Certification Election of FFW on March 19, 20
and 21, or before the "freedom period," is not sufficient ground to terminate the employment of respondents
inasmuch as the petition itself was actually filed during the freedom period. Nothing in the records would show
San Beda College of Law 427
4S: 2015 - 2016
LABOR LAW REVIEW Atty. Joyrich Golangco

that respondents failed to maintain their membership in good standing in the Union. Respondents did not resign
or withdraw their membership from the Union to which they belong. Respondents continued to pay their union
dues and never joined the FFW.

Significantly, petitioner's act of dismissing respondents stemmed from the latter's act of signing an authorization
letter to file a petition for certification election as they signed it outside the freedom period. However, we are
constrained to believe that an "authorization letter to file a petition for certification election" is different
from an actual "Petition for Certification Election." Strictly speaking, what is prohibited is the filing of a
petition for certification election outside the 60-day freedom period. This is not the situation in this case. If at all,
the signing of the authorization to file a certification election was merely preparatory to the filing of the petition
for certification election, or an exercise of respondents right to self-organization.

San Beda College of Law 428


4S: 2015 - 2016
LABOR LAW REVIEW Atty. Joyrich Golangco

NATIONAL UNION OF WORKERS IN HOTELS, RESTAURANTS AND ALLIED INDUSTRIES - MANILA
PAVILION HOTEL CHAPTER vs. SECRETARY OF LABOR AND EMPLOYMENT
G.R. No. 181531
July 31, 2009

CARPIO MORALES, J.:

ARTICLE 256: All rank and file employees in the appropriate bargaining unit, whether probationary or
permanent, are entitled to vote

FACTS:
A certification election was conducted on June 16, 2006 among the rank-and-file employees of Holiday Inn
Manila Pavilion Hotel with the following results: EMPLOYEES IN VOTERS LIST = 353; TOTAL VOTES CAST =
346; NUWHRAIN-MPHC = 151; HIMPHLU = 169; NO UNION = 1; SPOILED = 3; SEGREGATED = 22.

In view of the significant number of segregated votes, contending unions NUHWHRAIN-MPHC and HIMPHLU
referred the case back to Med-Arbiter Ma. Simonette Calabocal to decide which among those votes would be
opened and tallied. Calabocal ruled for the opening of 17 out of the 22 segregated votes, specially those cast by
the 11 dismissed employees and those cast by the 6 supposedly supervisory employees of the Hotel.

NUHWHRAIN appealed to the Secretary of Labor and Employment, arguing that the votes of the probationary
employees should have been opened and averred that HIMPHLU should not be immediately certified as the
bargaining agent.

Secretary of Labor: affirmed the Med-Arbiters Order. It held that pursuant to Section 5, Rule IX of the Omnibus
Rules Implementing the Labor Code on exclusion and inclusion of voters in a certification election, the
probationary employees cannot vote. The Secretary of Labor further ruled that even if the 17 votes of the
dismissed and supervisory employees were to be counted and presumed to be in favor of NUHWHRAIN, still,
the same would not suffice to overturn the 169 votes garnered by HIMPHLU. In fine, the Secretary of Labor
concluded that the certification of HIMPHLU as the exclusive bargaining agent was proper. MR denied.

Court of Appeals: affirmed the ruling of the Secretary of Labor.

ISSUE/S:
1.) Whether employees on probationary status at the time of the certification elections should be allowed to
vote. YES
2.) Whether HIMPHLU was able to obtain the required majority for it to be certified as the exclusive
bargaining agent. NO

RULING:
Probationary employees have the right to vote in a certification election. The votes of the six probationary
employees should have been counted. As Airtime Specialists, Inc. v. Ferrer-Calleja holds: In a certification
election, all rank and file employees in the appropriate bargaining unit, whether probationary or permanent, are
entitled to vote. This principle is clearly stated in Art. 255 of the Labor Code which states that the labor
organization designated or selected by the majority of the employees in an appropriate bargaining unit shall be
the exclusive representative of the employees in such unit for purposes of collective bargaining.

Collective bargaining covers all aspects of the employment relation and the resultant CBA negotiated by the
certified union binds all employees in the bargaining unit. Hence, all rank and file employees, probationary or
permanent, have a substantial interest in the selection of the bargaining representative. The Code makes no
distinction as to their employment status as basis for eligibility in supporting the petition for certification
election. The law refers to all the employees in the bargaining unit. All they need to be eligible to support the
petition is to belong to the bargaining unit.

All employees are, from the first day of their employment, eligible for membership in a labor organization. In the
present case, records show that the probationary employees were included in the list of employees in the
bargaining unit submitted by the Hotel. For purposes of self-organization, those employees are deemed eligible
to vote.

San Beda College of Law 429


4S: 2015 - 2016
LABOR LAW REVIEW Atty. Joyrich Golangco

As to whether HIMPHLU should be certified as the exclusive bargaining agent, the Court rules in the negative. It
is well-settled that under the so-called double majority rule, for there to be a valid certification election, majority
of the bargaining unit must have voted AND the winning union must have garnered majority of the valid votes
cast.

Prescinding from the Courts ruling that all the probationary employees votes should be deemed valid votes
while that of the supervisory employees should be excluded, it follows that the number of valid votes cast would
increase from 321 to 337. Under Art. 256 of the Labor Code, the union obtaining the majority of the valid votes
cast by the eligible voters shall be certified as the sole and exclusive bargaining agent of all the workers in the
appropriate bargaining unit. This majority is 50% + 1. Hence, 50% of 337 is 168.5 + 1 or at least 170.

HIMPHLU obtained 169. Clearly, HIMPHLU was not able to obtain a majority vote. Having declared that no
choice in the certification election conducted obtained the required majority, it follows that a run-off election must
be held to determine which between HIMPHLU and NUHWHRAIN should represent the rank-and-file
employees.

San Beda College of Law 430


4S: 2015 - 2016
LABOR LAW REVIEW Atty. Joyrich Golangco

ARTICLES 260-261

MIGUELA SANTUYO, et al., petitioners, vs. REMERCO GARMENTS MANUFACTURING, INC.


and/or VICTORIA REYES, respondents
G.R. No. 174420
March 22, 2010

CORONA, J.:

ARTICLE 260-261 in relation to ARTICLE 217; Cases arising from the interpretation or implementation of
collective bargaining agreements shall be disposed of by the Labor Arbiter by referring the same to the
grievance machinery and voluntary arbitration

FACTS: Due to a serious industrial dispute, the Kaisahan ng Manggagawa sa Remerco Garments
Manufacturing Inc. KMMKilusan staged a strike against Remerco Garments Manufacturing, Inc. (RGMI).
Because the strike was subsequently declared illegal, all union officers were dismissed. Employees who wanted
to sever their employment were paid separation pay while those who wanted to resume work were recalled on
the condition that they would no longer be paid a daily rate but on a piece-rate basis. Miguela Santuyo, et al.,
who had been employed as sewers, were among those recalled.

Without allowing RGMI to normalize its operations, the union filed a notice of strike in the NCMB. According to
the union, RGMI conducted a time and motion study and changed the salary scheme from a daily rate to piece-
rate basis without consulting it. RGMI therefore not only violated the existing CBA but also diminished the
salaries agreed upon. It therefore committed an unfair labor practice. RGMI filed a notice of lockout in the
NCMB.

RGMI transferred its factory site. The union went on strike and blocked the entry to RGMIs (new) premises. The
Secretary of Labor assumed jurisdiction pursuant to Article 263(g) of the Labor Code and ordered RGMIs
striking workers to return to work immediately. He likewise ordered the union and RGMI to submit their
respective position papers.

The Secretary of Labor held that RGMI did not lock out its employees inasmuch as it informed them of the
transfer of the worksite. However, he did not rule on the legality of the strike. Furthermore, based on the time
and motion study, the Secretary of Labor found that the employees would receive higher wages if they were
paid on a piece-rate rather than on a daily rate basis. Hence, the new salary scheme would be more
advantageous to the employees. For this reason, despite the provisions of the CBA, the change in salary
scheme was validated.

While the conciliation proceedings were pending, Santuyo, et al. filed a complaint for illegal dismissal against
RGMI and Victoria Reyes, accusing the latter of harassment.

Labor Arbiter: found that RGMI did not pay Santuyo, et al. their salaries and deprived them of the benefits they
were entitled to under the CBA.

NLRC: denied RGMIs appeal.


RGMI and Reyes filed a petition for certiorari in the CA claiming that the NLRC acted with grave abuse of
discretion in affirming the decision of the labor arbiter. They argued that since the complaint involved the
implementation of the CBA, the labor arbiter had no jurisdiction over it.

Court of Appeals: reversed and set aside the decision of the NLRC on the ground that the labor arbiter had no
jurisdiction over the complaint. MR denied.

ISSUE/S:
Whether the labor arbiter had jurisdiction as the complaint was for illegal dismissal. NO

RULING:
NO. Santuyo, et al. clearly and consistently questioned the legality of RGMIs adoption of the new salary
scheme (i.e., piece-rate basis), asserting that such action, among others, violated the existing CBA. Indeed, the

San Beda College of Law 431


4S: 2015 - 2016
LABOR LAW REVIEW Atty. Joyrich Golangco

controversy was not a simple case of illegal dismissal but a labor dispute involving the manner of ascertaining
employees salaries, a matter which was governed by the existing CBA.

With regard to the question of jurisdiction over the subject matter, Article 217(c) of the Labor Code provides:
Cases arising from the interpretation or implementation of collective bargaining agreements and those arising
from the interpretation or enforcement of company personnel policies shall be disposed of by the Labor Arbiter
by referring the same to the grievance machinery and voluntary arbitration as may be provided in said
agreements.

Moreover, Article 260 of the Labor Code clarifies that such disputes must be referred first to the grievance
machinery and, if unresolved within seven days, they shall automatically be referred to voluntary arbitration.

In this regard, Article 261 thereof states: The Voluntary Arbitrator or panel of Voluntary Arbitrators shall have
original and exclusive jurisdiction to hear and decide all unresolved grievances arising from the interpretation
or implementation of the Collective Bargaining Agreement and those arising from the interpretation or
enforcement of company personnel policies x x x.

Pursuant to Articles 217 in relation to Articles 260 and 261 of the Labor Code, the labor arbiter should have
referred the matter to the grievance machinery provided in the CBA. Because the labor arbiter clearly did not
have jurisdiction over the subject matter, his decision was void.

San Beda College of Law 432


4S: 2015 - 2016
LABOR LAW REVIEW Atty. Joyrich Golangco

ALBERT TENG, doing business under the firm name ALBERT TENG FISH TRADING, and EMILIA TENG-
CHUA, vs. ALFREDO S. PAHAGAC, EDDIE D. NIPA, ORLANDO P. LAYESE, HERNAN Y. BADILLES and
ROGER S. PAHAGAC
G.R. No. 169704
November 17, 2010
BRION, J.:

FACTS: Albert Teng Fish Trading is engaged in deep sea fishing and, for this purpose, owns boats (basnig),
equipment, and other fishing paraphernalia. As owner of the business, Teng claims that he customarily enters
into joint venture agreements with master fishermen (maestros) who are skilled and are experts in deep sea
fishing; they take charge of the management of each fishing venture, including the hiring of the members of its
complement. He avers that the maestros hired the respondent workers as checkers to determine the volume of
the fish caught in every fishing voyage

Respondent workers filed a complaint for illegal dismissal against Albert Teng Fish Trading, Teng, and Chua
before the NCMB alleging that Teng hired them, without any written employment contract, to serve as his "eyes
and ears" aboard the fishing boats; to classify the fish caught by baera; to report to Teng via radio
communication the classes and volume of each catch; to receive instructions from him as to where and when to
unload the catch; to prepare the list of the provisions requested by the maestro and the mechanic for his
approval; and, to procure the items as approved by him. They also claimed that they received regular monthly
salaries, 13th month pay, Christmas bonus, and incentives in the form of shares in the total volume of fish
caught.

VA: in Tengs favor; declared that no employer-employee relationship existed between Teng and the respondent
workers; all other claims are likewise dismissed for lack of merit

Respondent workers received filed a motion for reconsideration.

VA: MR denied; Section 6, Rule VII of the 1989 Procedural Guidelines in the Conduct of Voluntary Arbitration
Proceedings (1989 Procedural Guidelines) does not provide the remedy of a motion for reconsideration to the
party adversely affected by the VAs order or decision.

CA: reversed the VAs decision after finding sufficient evidence showing the existence of employer-employee
relationship

Teng moved to reconsider the CAs decision, but the CA denied.

ISSUE/S: Can respondent workers file an MR from the VA's decision?

RULING: We resolve to deny the petition for lack of merit.

Yes. Article 262-A of the Labor Code does not prohibit the filing of a motion for reconsideration.
As amended, Article 263 is now Article 262-A, which states:

Art. 262-A. x x x [T]he award or decision x x x shall contain the facts and the law on which it is based. It shall be
final and executory after ten (10) calendar days from receipt of the copy of the award or decision by the parties.

Notably, Article 262-A deleted the word "unappealable" from Article 263. The deliberate selection of the
language in the amendatory act differing from that of the original act indicates that the legislature intended a
change in the law, and the court should endeavor to give effect to such intent.

[U]nder Section 6, Rule VII of the same guidelines implementing Article 262-A of the Labor Code, this Decision,
as a matter of course, would become final and executory after ten (10) calendar days from receipt of copies of
the decision by the parties x x x unless, in the meantime, a motion for reconsideration or a petition for review to
the Court of Appeals under Rule 43 of the Rules of Court is filed within the same 10-day period. We
consequently rule that the respondent workers seasonably filed a motion for reconsideration of the VAs
judgment, and the VA erred in denying the motion because no motion for reconsideration is allowed.

San Beda College of Law 433


4S: 2015 - 2016
LABOR LAW REVIEW Atty. Joyrich Golangco

A contrary provision can be found in Section 7, Rule XIX of the Department of Labors Department Order (DO)
No. 40, series of 2003:
Rule XIX
Section 7. Finality of Award/Decision. The decision, order, resolution or award of the voluntary arbitrator or
panel of voluntary arbitrators shall be final and executory after ten (10) calendar days from receipt of the copy of
the award or decision by the parties and it shall not be subject of a motion for reconsideration.

Presumably on the basis of DO 40-03, the 1989 Procedural Guidelines was revised in 2005 (2005 Procedural
Guidelines),33 whose pertinent provisions provide that:

Rule VII DECISIONS


Section 6. Finality of Decisions. The decision of the Voluntary Arbitrator shall be final and executory after ten
(10) calendar days from receipt of the copy of the decision by the parties.
Section 7. Motions for Reconsideration. The decision of the Voluntary Arbitrator is not subject of a Motion for
Reconsideration.

In the exercise of its power to promulgate implementing rules and regulations, an implementing agency, such as
the Department of Labor is restricted from going beyond the terms of the law it seeks to implement.

By allowing a 10-day period, the obvious intent of Congress in amending Article 263 to Article 262-A is to
provide an opportunity for the party adversely affected by the VAs decision to seek recourse via a motion for
reconsideration or a petition for review under Rule 43 of the Rules of Court filed with the CA. Indeed, a motion
for reconsideration is the more appropriate remedy in line with the doctrine of exhaustion of administrative
remedies. Exhaustion of available remedies as a condition precedent to a petition under that Rule.

San Beda College of Law 434


4S: 2015 - 2016
LABOR LAW REVIEW Atty. Joyrich Golangco

SAMAHAN NG MGA MANGGAGAWA SA HYATT v. HON. VOLUNTARY ARBITRATOR BUENAVENTURA C.
MAGSALIN
G.R. No. 172303
June 6, 2011

FACTS: Petitioner Samahan ng mga Manggagawa sa Hyatt-NUWHRAIN-APL (Samahan) is a duly registered union
and the certified bargaining representative of the rank-and-file employees of Hyatt Regency owned and operated by
respondent Hotel Enterprises of the Philippines, Inc.

Angelito Caragdag, a waiter at the hotels Cafe Al Fresco restaurant and a director of the union on several occasions
violated Memorandums and of the Manager and the Code of Discipline of the company. Some of his acts are as
follows:
1. He refused to be frisked twice by the security personnel in violation of the memorandum issued by the
Manager.
2. When Moral, the manager of Hyatts Cafe Al Fresco and Caragdags immediate superior, was about to
counsel two staff members, Lacambacal and Alvaro, at the training room, Caragdag suddenly opened
the door and started yelling at the two with an enraged look.
3. Again, on one occasion, Caragdag left his work assignment during official hours without prior permission
from his Department Head.

The incidents were reported to the hotels Human Resources Department (HRD), which issued Memorandums to
Caragdag, requiring him to explain in writing why no disciplinary action should be taken against him, however the
HRD found Caragdags explanation insufficient and punished him in accordance with the Code of Discipline with
three separate suspensions.

An investigation board was formed after receipt of Caragdags written explanation, and the matter was set for hearing.
After due process, it resolved on the said date to dismiss Caragdag for violation of OSDA 4.32. Caragdag appealed
but the investigating board affirmed its resolution after hearing.

The hotel, through its attorney sent Caragdag a Notice of Dismissal. Caragdags dismissal was questioned by
petitioner, and the dispute was referred to voluntary arbitration.

VA Ruling: Three separate suspensions of Caragdag valid; union officers and members had no right to breach
company rules and regulations on security and employee discipline on the basis of certain suspicions against
management and an ongoing CBA negotiation standoff.

Petitioner sought reconsideration but the Voluntary Arbitrator denied his motions.

Petitioner assailed VAs decision in a petition for certiorari with CA.

CA Ruling: Petition is dismissed outright for being the wrong remedy. He should have filed a Petition for Review
under Rule 43, Section 5 of the ROC; Even if this Court treats the instant petition as a Petition for Review, still the
Court has to dismiss the same for having been filed out of time.
In the meantime respondent also filed a petition for review with the CA on the ground that the Voluntary Arbitrator
committed a grievous error in awarding financial assistance to Caragdag despite his finding that the dismissal due to
serious misconduct was valid.

CA Ruling: Deleted the award of financial assistance to Caragdag; CA agreed with the findings of the VA that
Caragdag was validly dismissed due to serious misconduct.

Petitioner sought reconsideration but the CA denied the motion for lack of merit. Hence, petitioner filed before us a
petition for review on certiorari docketed as G.R. No. 172303.

Considering that G.R. Nos. 164939 and 172303 have the same origin, involve the same parties, and raise interrelated
issues, the petitions were consolidated.

ISSUE/S:
1. Whether the CA erred in dismissing outright the petition for certiorari filed before it on the ground that the
same is an improper mode of appeal
San Beda College of Law 435
4S: 2015 - 2016
LABOR LAW REVIEW Atty. Joyrich Golangco

2. Whether the CA erred in deleting the award of financial assistance in the amount of P100,000.00 to
Caragdag.

RULING:

1. No. It is the well-settled rule that a decision or award of a voluntary arbitrator is appealable to the CA via
petition for review under Rule 43. We held that: In Luzon Development Bank v. Association of Luzon
Development Bank Employees, where the Court held that the decision or award of the voluntary arbitrator or
panel of arbitrators should likewise be appealable to the Court of Appeals, in line with the procedure outlined in
Revised Administrative Circular No. 1-95 (now embodied in Rule 43 of the 1997 Rules of Civil Procedure), just
like those of the quasi-judicial agencies, boards and commissions enumerated therein, and consistent with the
original purpose to provide a uniform procedure for the appellate review of adjudications of all quasi-judicial
entities.

Furthermore in Sections 1, 3 and 4, Rule 43 of the 1997 Rules of Civil Procedure, as amended, provide:
SECTION 1. Scope. - This Rule shall apply to appeals from judgments or final orders of the Court of Tax
Appeals and from awards, judgments, final orders or resolutions of or authorized by any quasi-judicial agency in
the exercise of its quasi-judicial functions. Among these agencies are the x x x, and voluntary arbitrators
authorized by law.
xxxx
SEC. 3. Where to appeal. - An appeal under this Rule may be taken to the Court of Appeals within the period
and in the manner therein provided, whether the appeal involves questions of fact, of law, or mixed questions of
fact and law.
SEC. 4. Period of appeal. - The appeal shall be taken within fifteen (15) days from notice of the award,
judgment, final order or resolution, or from the date of its last publication, if publication is required by law for its
effectivity, or of the denial of petitioners motion for new trial or reconsideration duly filed in accordance with the
governing law of the court or agency a quo. x x x.

Hence, upon receipt on May 26, 2003 of the Voluntary Arbitrators Resolution denying petitioners motion for
reconsideration, petitioner should have filed with the CA, within the fifteen (15)-day reglementary period, a
petition for review, not a petition for certiorari.

2. No. Separation pay shall be allowed as a measure of social justice only in those instances where the
employee is validly dismissed for causes other than serious misconduct or those reflecting on his moral
character. Where the reason for the valid dismissal is, for example, habitual intoxication or an offense involving
moral turpitude, like theft or illicit sexual relations with a fellow worker, the employer may not be required to give
the dismissed employee separation pay, or financial assistance, or whatever other name it is called, on the
ground of social justice. A contrary rule would, as the petitioner correctly argues, have the effect, of rewarding
rather than punishing the erring employee for his offense.

Here, Caragdags dismissal was due to several instances of willful disobedience to the reasonable rules and
regulations prescribed by his employer. The Voluntary Arbitrator pointed out that according to the hotels Code of
Discipline, an employee who commits three different acts of misconduct within a twelve (12)-month period
commits serious misconduct. He stressed that Caragdags infractions were not even spread in a period of twelve
(12) months, but rather in a period of a little over a month.

San Beda College of Law 436


4S: 2015 - 2016
LABOR LAW REVIEW Atty. Joyrich Golangco

263-264

EDEN GLADYS ABARIA, ET AL., vs. NLRC, ET AL.


G.R. No. 154113 December 7, 2011
VILLARAMA, JR., J.:

DOCTRINE:
ART. 263 provides that in cases of bargaining deadlocks, the duly certified or recognized bargaining
agent may file a notice of strike or the employer may file a notice of lockout with the Department at least 30
days before the intended date thereof. In cases of unfair labor practice, the period of notice shall be 15 days
and in the absence of a duly certified or recognized bargaining agent, the notice of strike may be filed by
any legitimate labor organization in behalf of its members.

Section 6, Rule XXII, Book V of the Omnibus Rules Implementing the Labor Code reads that any certified or
duly recognized bargaining representative may declare a strike in cases of bargaining deadlocks and unfair
labor practices. The employer may declare a lockout in the same cases. In the absence of a certified or duly
recognized bargaining representative, any legitimate labor organization in the establishment may declare a
strike but only on grounds of unfair labor practice.

FACTS:
Perla Nava, the President of NAMA-MCCH-NFL, wrote to the Hospital Administrator, expressing the unions
desire to renew the CBA. However, MCCHI returned the CBA proposal for Nava to secure first the endorsement
of the legal counsel of NFL as the official bargaining representative of MCCHI employees.

Atty. Alforque (legal counsel of NFL) informed MCCHI that the proposed CBA submitted by Nava was never
referred to NFL and that NFL has not authorized any other legal counsel or any person for collective bargaining
negotiations.

Thereafter, MCCHI attempted to take over the room being used as union office but was prevented to do so by
Nava and her group who protested these actions and insisted that management directly negotiate with them for
a new CBA. MCCHI referred the matter to Atty. Alforque, NFLs Regional Director, and advised Nava that their
group is not recognized by NFL.

In his letter addressed to Nava et al, Atty. Alforque suspended their union membership for serious violation of
the Constitution and By-Laws.

The next day, several union members led by Nava and her group launched a series of mass actions such as
wearing black and red armbands/headbands, marching around the hospital premises and putting up placards,
posters and streamers. Atty. Alforque immediately disowned the concerted activities being carried out by union
members which are not sanctioned by NFL. MCCHI directed the union officers led by Nava to submit within 48
hours a written explanation why they should not be terminated for having engaged in illegal concerted activities
amounting to strike, and placed them under immediate preventive suspension. Responding to this directive,
Nava and her group denied there was a temporary stoppage of work, explaining that employees wore their
armbands only as a sign of protest and reiterating their demand for MCCHI to comply with its duty to bargain
collectively. Rev. Iyoy, having been informed that Nava and her group have also been suspended by NFL,
directed said officers to appear before his office for investigation in connection with the illegal strike wherein they
reportedly uttered slanderous and scurrilous words against the officers of the hospital, threatening other workers
and forcing them to join the strike. Said union officers, however, invoked the grievance procedure provided in
the CBA to settle the dispute between management and the union.

The DOLE Regional Office No. 7 issued certifications stating that there is nothing in their records which shows
that NAMA-MCCH-NFL is a registered labor organization, and that said union submitted only a copy of its
Charter Certificate. MCCHI then sent individual notices to all union members asking them to submit within 72
hours a written explanation why they should not be terminated for having supported the illegal concerted
activities of NAMA-MCCH-NFL which has no legal personality as per DOLE records. In their collective
response/statement dated March 18, 1996, it was explained that the picketing employees wore armbands to
protest MCCHIs refusal to bargain; it was also contended that MCCHI cannot question the legal personality of
the union which had actively assisted in CBA negotiations and implementation.

San Beda College of Law 437


4S: 2015 - 2016
LABOR LAW REVIEW Atty. Joyrich Golangco

NAMA-MCCH-NFL filed a Notice of Strike but the same was deemed not filed for want of legal personality on
the part of the filer. Despite such rebuff, Nava and her group still conducted a strike vote during which an
overwhelming majority of union members approved the strike.

MCCHI issued a cease-and-desist order to the rest of the striking employees stressing that the wildcat
concerted activities spearheaded by the Nava group is illegal without a valid Notice of Strike and warning them
that non-compliance will compel management to impose disciplinary actions against them. For their continued
picketing activities despite the said warning, more than 100 striking employees were dismissed. Unfazed, the
striking union members held more mass actions. The means of ingress to and egress from the hospital were
blocked so that vehicles carrying patients and employees were barred from entering the premises.

With the volatile situation adversely affecting hospital operations and the condition of confined patients, MCCHI
filed a petition for injunction in the NLRC. A TRO was issued. MCCHIs petition was granted and a permanent
injunction was issued enjoining the Nava group from committing illegal acts mentioned in Article 264 of the
Labor Code.

The City Government of Cebu ordered the demolition of the structures and obstructions put up by the picketing
employees of MCCHI along the sidewalk, having determined the same as a public nuisance or nuisance per se.

Thereafter, several complaints for illegal dismissal and unfair labor practice were filed by the terminated
employees against MCCHI, Rev. Iyoy, UCCP and members of the Board of Trustees of MCCHI.

RULING OF THE LA:


Executive Labor Arbiter Belarmino dismissed the complaints for unfair labor practice filed by Nava and 90 other
complainants and found no basis for the charge of unfair labor practice and declared the strike and picketing
activities illegal having been conducted by NAMA-MCCH-NFL which is not a legitimate labor organization.

RULING OF THE NLRC:


The NLRC affirmed the decision of the LA.

ISSUE: Were the strike and picketing activities conducted by union officers and members illegal?

RULING: YES. Art. 263 (b) of the Labor Code, as amended, provides that workers shall have the right to
engage in concerted activities for purposes of collective bargaining or for their mutual benefit and protection.
However, no labor union may strike and no employer may declare a lockout on grounds involving inter-union
and intra-union disputes.

The same provision also provides that in cases of bargaining deadlocks, the duly certified or recognized
bargaining agent may file a notice of strike or the employer may file a notice of lockout with the Department at
least 30 days before the intended date thereof. In cases of unfair labor practice, the period of notice shall be 15
days and in the absence of a duly certified or recognized bargaining agent, the notice of strike may be
filed by any legitimate labor organization in behalf of its members.

Section 6, Rule XXII, Book V of the Omnibus Rules Implementing the Labor Code reads that any certified or
duly recognized bargaining representative may declare a strike in cases of bargaining deadlocks and unfair
labor practices. The employer may declare a lockout in the same cases. In the absence of a certified or duly
recognized bargaining representative, any legitimate labor organization in the establishment may declare a
strike but only on grounds of unfair labor practice.

As borne by the records, NAMA-MCCH-NFL was not a duly registered or an independently registered union at
the time it filed the notice of strike and when it conducted the strike vote. It could not then legally represent the
union members. Consequently, the mandatory notice of strike and the conduct of the strike vote report were
ineffective for having been filed and conducted by NAMA-MCCH-NFL which has no legal personality as a
legitimate labor organization, in violation of Art. 263 (c), (d) and (f) of the Labor Code and Rule XXII, Book V of
the Omnibus Rules Implementing the Labor Code. Furthermore, the strike was illegal due to the commission of
prohibited activities

San Beda College of Law 438


4S: 2015 - 2016
LABOR LAW REVIEW Atty. Joyrich Golangco

YSS EMPLOYEES UNIONPHILIPPINE TRANSPORT AND GENERAL WORKERS ORGANIZATION v. YSS
LABORATORIES, INC.,
G.R. No. 155125 December 4, 2009
CHICO-NAZARIO, J.

DOCTRINE: Art. 263. Strikes, picketing, and lockouts.


(g) When, in his opinion, there exists a labor dispute causing or likely to cause a strike or lockout in an industry
indispensable to the national interest, the Secretary of Labor and Employment may assume jurisdiction over the
dispute and decide it or certify the same to the Commission for compulsory arbitration. Such assumption or
certification shall have the effect of automatically enjoining the intended or impending strike or lockout as
specified in the assumption or certification order. If one has already taken place at the time of assumption or
certification, all striking or locked out employees shall immediately return to work and the employer shall
immediately resume operations and readmit all workers under the same terms and conditions prevailing
before the strike or lockout. The Secretary of Labor and Employment or the Commission may seek the
assistance of law enforcement agencies to ensure compliance with this provision as well as with such orders as
he may issue to enforce the same.

FACTS:
In order to arrest escalating business losses, YSS Laboratories, a domestic corporation engaged in
Pharmaceutical business implemented a retrenchment program which affected 11 employees purportedly
chosen in accordance with the reasonable standards established by the company. Of the 11 employees sought
to be retrenched, nine were officers and members of YSSEU, a duly registered labor organization and the sole
and exclusive bargaining representative of the rank-and-file employees of YSS. Claiming that YSS Laboratories
was guilty of discrimination and union-busting in carrying out the said retrenchment program, YSSEU decided to
hold a valid strike. A number of conciliation proceedings were made by the NCMB-NCR but still the dispute was
not resolved. This prompted the Sectary of Labor to intervene. Finding that the labor dispute was inimical to the
national interest, it certifies the case to the NLRC for compulsory arbitration and issued two orders: first,
directing all striking workers to return to work and for the Company to accept them back under the same terms
and conditions of employment prior to the strike; second, that the nine retrenched employees be included in the
return to work order. Aggrieved, YSS Laboratories filed a petition under Rule 65 before the Court of Appeals in
which the latter reversed the orders of the Secretary of Labor and granted the petition of YSS Laboratories. The
appellate court found that YSS Laboratories validly carried out its retrenchment program, which effectively
severed the concerned employees employment with the company. Hence, YSSEU comes to this petition.
ISSUE: Whether or not the retrenched employees should be excluded from the coverage of the return-to-work-
order.
HELD: No. The Orders of the Secretary of Labor, certifying the labor dispute involving the herein parties to the
NLRC for compulsory arbitration, and enjoining YSSEU to return to work and YSS Laboratories to admit them
under the same terms and conditions prevailing before the strike, were issued pursuant to Article 263(g) of the
Labor Code. Said provision reads:
Art. 263. Strikes, picketing, and lockouts.
xxxx
(g) When, in his opinion, there exists a labor dispute causing or likely to cause a strike
or lockout in an industry indispensable to the national interest, the Secretary of Labor and
Employment may assume jurisdiction over the dispute and decide it or certify the same to the
Commission for compulsory arbitration. Such assumption or certification shall have the effect of
automatically enjoining the intended or impending strike or lockout as specified in the
assumption or certification order. If one has already taken place at the time of assumption or
certification, all striking or locked out employees shall immediately return to work and the
employer shall immediately resume operations and readmit all workers under the same
terms and conditions prevailing before the strike or lockout.

It should be noted that the primary reason why the strike was conducted in the first place was to protest
the implementation of the retrenchment program, which clearly discriminated against union officers and
members. YSS Laboratories vigorous insistence on the exclusion of the retrenched employees from the
coverage of the return-to-work order seriously impairs the authority of the Secretary of Labor to forestall a labor
dispute that he deems inimical to the national economy. Accordingly, when the Secretary of Labor directed YSS
Laboratories to accept all the striking workers back to work, the Secretary did not exceed his jurisdiction, or
gravely abuse the same, said the Supreme Court. Hence, the petition is granted. The orders of the Secretary of
Labor and Employment are hereby reinstated.
San Beda College of Law 439
4S: 2015 - 2016
LABOR LAW REVIEW Atty. Joyrich Golangco

NATIONAL UNION OF WORKERS IN THE HOTEL RESTAURANT AND ALLIED INDUSTRIES (NUWHRAIN-
APL-IUF) DUSIT HOTEL NIKKO CHAPTER vs. THE HONORABLE COURT OF APPEALS (Former Eighth
Division), THE NATIONAL LABOR RELATIONS COMMISSION (NLRC), PHILIPPINE HOTELIERS INC.,
owner and operator of DUSIT HOTEL NIKKO and/or CHIYUKI FUJIMOTO, and ESPERANZA V. ALVEZ;
NUWHRAIN-DUSIT HOTEL NIKKO CHAPTER vs. SECRETARY OF LABOR AND EMPLOYMENT and
PHILIPPINE HOTELIERS, INC.
G.R. No. 163942 & G.R. No. 166295 November 11, 2008
VELASCO, JR., J.:

FACTS: Due to failed CBA negotiations, the union filed on December 20, 2001 a Notice of Strike. A Strike Vote
was conducted by the Union on January 14, 2002 on which it was decided that the Union would wage a strike.

Soon thereafter, the Union held a general assembly at its office, where some members sported closely cropped
hair or cleanly shaven heads. The next day, on January 18, 2002, more male Union members came to work
sporting the same hair style. The Hotel prevented these workers from entering the premises claiming that they
violated the Hotel's Grooming Standards. In view of the Hotel's action, the Union staged a picket outside the
Hotel premises. Later, other workers were also prevented from entering the Hotel causing them to join the
picket. For this reason the Hotel experienced a severe lack of manpower which forced them to temporarily
cease operations.

Subsequently, on January 20, 2002, the Hotel issued notices to Union members, preventively suspending them
and charging them with offenses. The next day, the Union filed with the NCMB a second Notice of Strike. On
January 26, 2002, the Hotel terminated the services of the Union officers and members; and suspended a
number of employees. On the same day, the Union declared a strike.

On January 31, 2002, the Union filed its third Notice of Strike with the NCMB. On the same day, the Secretary
assumed jurisdiction over the labor dispute and certified the case to the NLRC.

NLRC RULING: The NLRC held that the January 18, 2002 concerted action was an illegal strike and that the
strike violated the "No Strike, No Lockout" provision of the CBA. It was illegal because it failed to comply with
the mandatory 30-day cooling-off period and the seven-day strike ban, as the strike occurred only 29 days after
the submission of the notice of strike on December 20, 2001 and only four days after the submission of the
strike vote on January 14, 2002. In addition, it was attended by illegal acts committed by the Union officers and
members.

CA RULING: affirmed the rulings of the NLRC

ISSUE: Did the Union conduct an illegal strike?

SC RULING: YES.

First, the Union's violation of the Hotel's Grooming Standards was clearly a deliberate and concerted action to
undermine the authority of and to embarrass the Hotel and was, therefore, not a protected action. The decision
to violate the company rule on grooming was designed and calculated to place the Hotel management on its
heels and to force it to agree to the Union's proposals.

Second, the Union's concerted action which disrupted the Hotel's operations clearly violated the CBA's "No
Strike, No Lockout" provision.

Third, the Union's action to have their officers and members' heads shaved was manifestly calculated to
antagonize and embarrass the Hotel management and in doing so effectively disrupted the operations of the
Hotel and violated their duty to bargain collectively in good faith.

Fourth, the Union failed to observe the mandatory 30-day cooling-off period and the seven-day strike ban before
it conducted the strike on January 18, 2002.

Last, the Union committed illegal acts in the conduct of its strike. As shown by the pictures presented by the
Hotel, the Union officers and members formed human barricades and obstructed the driveway of the Hotel.

San Beda College of Law 440


4S: 2015 - 2016
LABOR LAW REVIEW Atty. Joyrich Golangco

Art. 263-264

JACKBILT INDUSTRIES, INC. vs. JACKBILT EMPLOYEES WORKERS UNION-NAFLU-KMU


G.R. Nos. 171618-19 March 20, 2009
CORONA, J.:

DOCTRINE: Pursuant to the principle of conclusiveness of judgment, the strike was ipso facto illegal. The filing
of a petition to declare the strike illegal was thus unnecessary.

FACTS: Petitioner decided to temporarily stop its business. Respondent Union immediately protested claiming
that the shutdown was motivated by anti-union sentiments. Accordingly respondent went on strike. Its officers
and members picketed petitioners main gates and deliberately prevented persons and vehicles from going into
and out of the compound. Petitioner filed a petition for injunction with a prayer for the issuance of a TRO in the
NLRC. The NLRC issued a TRO directing the respondents to refrain from preventing access to petitioners
property. Respondent union violated the order.

Meanwhile, petitioner sent individual memoranda to the officers and members of respondent who participated in
the strike ordering them to explain why they should not be dismissed for committing illegal acts in the course of
a strike. However, respondent repeatedly ignored petitioners memoranda. Thus petitioner dismissed the
concerned officers and members.

Aggrieved, respondent filed complaints for illegal lockout, runaway shop and damages, unfair labor practice,
illegal dismissal and attorneys fees, and refusal to bargain on behalf of its officers and members against
petitioner and its corporate officers. Petitioner, on the other hand, asserted that because respondent conducted
a strike without observing the procedural requirements provided in Article 263 of the Labor Code, the strike was
illegal. Furthermore, in view of the decision of the NLRC, petitioner validly dismissed respondents officers and
employees for committing illegal acts in the course of a strike.

LA RULING: Dismissed the complaints for illegal lockout and unfair labor practice for lack of merit. However,
because petitioner did not file a petition to declare the strike illegal before terminating respondents officers and
employees, it was found guilty of illegal dismissal.

NLRC RULING: Modified the decision of the labor arbiter. It held that only petitioner should be liable for
monetary awards granted to respondents officers and members.

CA RULING: The temporary shutdown was moved by anti-union sentiments. Petitioner was therefore guilty of
unfair labor practice.

ISSUE: Is the filing of a petition with the labor arbiter to declare a strike illegal a condition sine qua non for the
valid termination of employees who commit an illegal act in the course of such strike?

SC RULING: NO. Article 264(e) of the Labor Code prohibits any person engaged in picketing from obstructing
the free ingress to and egress from the employers premises. Since respondent was found in the decision of the
NLRC to have prevented the free entry into and exit of vehicles from petitioners compound, respondents officers
and employees clearly committed illegal acts in the course of the strike.

The use of unlawful means in the course of a strike renders such strike illegal. Therefore, pursuant to the
principle of conclusiveness of judgment, the strike was ipso facto illegal. The filing of a petition to declare the
strike illegal was thus unnecessary.

San Beda College of Law 441


4S: 2015 - 2016
LABOR LAW REVIEW Atty. Joyrich Golangco

Art. 263-264

AIRLINE PILOTS ASSOCIATION OF THE PHILIPPINES v. PHILIPPINE AIRLINES, INC.


G.R. No. 168382 June 6, 2011
DEL CASTILLO, J.:

FACTS:
Claiming that PAL committed unfair labor practice, ALPAP filed a notice of strike against respondent PAL with the DOLE.
Upon PALs petition and considering that its continued operation is impressed with public interest, the DOLE Secretary
assumed jurisdiction over the labor dispute. DOLE Secretary ordered that all strikes and lockouts at the Philippine Airlines,
Inc., whether actual or impending, are hereby strictly prohibited. The parties are also enjoined from committing any act
that may exacerbate the situation. However, ALPAP went on strike.

DOLE issue a return-to-work orderon June 7, 1998. However, it was only on June 26, 1998 when ALPAP officers and
members reported back to work as shown in a logbook signed by each of them. As a consequence, PAL refused to
accept the returning pilots for their failure to comply immediately with the return-to-work order.

DOLE declared the strike conducted by ALPAP illegal and pronouncing the loss of employment status of its officers and
members who participated in the strike in defiance of the June 7, 1998 return-to-work order. This was eventually elevated
to the SC and attained finality on August 29, 2002.

Despite such, on January 13, 2003, ALPAP filed before DOLE a Motion requesting to conduct an appropriate
legal proceeding to determine who among its officers and members should be reinstated or deemed to have lost
their employment with PAL for their actual participation in the strike. PAL, in its comment argued that the
motions cannot legally prosper since the DOLE Secretary has no authority to reopen or review a final judgment
of the Supreme Court.

DOLE concludes that the case has indeed been resolved with finality by the highest tribunal of the land, the
Supreme Court and being final and executory, DOLE has no authority to reopen an issue that has been passed
upon by the Supreme Court. DOLE merely noted the motions.\

This was affirmed by the CA.

ISSUE: Did the CA committed grave mistake in declaring that the resolution of the DOLE has already taken up
and resolved the issue of who among the ALPAP members are deemed to have lost their employment status?

RULING:
NO. There was no grave abuse of discretion on the part of DOLE in merely noting ALPAPs twin motions in due
deference to a final and immutable judgment rendered by the Supreme Court.

In the instant case, ALPAP seeks for a conduct of a proceeding to determine who among its members and
officers actually participated in the illegal strike because, it insists, the June 1, 1999 DOLE Resolution did not
make such determination. However, as correctly ruled by DOLE and affirmed by the CA, such proceeding would
entail a reopening of a final judgment which could not be permitted by this Court. Settled in law is that once a
decision has acquired finality, it becomes immutable and unalterable, thus can no longer be modified in any
respect. Subject to certain recognized exceptions, the principle of immutability leaves the judgment undisturbed
as nothing further can be done except to execute it.

True, the dispositive portion of the DOLE Resolution does not specifically enumerate the names of those who
actually participated in the strike but only mentions that those strikers who failed to heed the return-to-work
order are deemed to have lost their employment. This omission, however, cannot prevent an effective execution
of the decision. Any ambiguity may be clarified by reference primarily to the body of the decision or
supplementary to the pleadings previously filed in the case, as in this case.

A review of the records reveals that in NCMB NCR NS 12-514-97, the DOLE Secretary declared the ALPAP
officers and members to have lost their employment status based on either of two grounds, viz: their
participation in the illegal strike on June 5, 1998 or their defiance of the return-to-work order of the DOLE
Secretary. The records of the case unveil the names of each of these returning pilots. The logbook with the
heading Return To Work Compliance/ Returnees bears their individual signature signifying their conformity that
San Beda College of Law 442
4S: 2015 - 2016
LABOR LAW REVIEW Atty. Joyrich Golangco

they were among those workers who returned to work only on June 26, 1998 or after the deadline imposed by
DOLE.

A scrutiny of the records of the case discloses that these allegations were raised at a very late stage, that is,
after the judgment has finally decreed that the returning pilots termination was legal. Interestingly, these
defenses were not raised and discussed when the case was still pending before the DOLE Secretary, the CA or
even before this Court. No ALPAP member has claimed that he was unable to comply with the return-to-work
directive because he was either on leave, abroad or unable to report for some reason. These defenses were
raised in ALPAPs twin motions only after the Resolution in G.R. No. 152306 reached finality in its last ditch effort
to obtain a favorable ruling. It has been held that a proceeding may not be reopened upon grounds already
available to the parties during the pendency of such proceedings; otherwise, it may give way to vicious and
vexatious proceedings.[44] ALPAP was given all the opportunities to present its evidence and arguments. It
cannot now complain that it was denied due process

San Beda College of Law 443


4S: 2015 - 2016
LABOR LAW REVIEW Atty. Joyrich Golangco

Art. 263-264

DANILO ESCARIO,ET. AL. v. NLRC


G.R. No. 160302; September 27, 2010
BERSAMIN, J.:

FACTS:
In the morning of March 13, 1993, all the officers and some 200 members of the Union walked out of PINAs
premises and proceeded to the barangay office to show support for Juanito Caete, an officer of the Union
charged with oral defamation by Aurora Manor, PINAs personnel manager, and Yolanda Fabella, Manors
secretary.

INA filed a complaint for unfair labor practice (ULP) and damages. LA declared the incident an illegal walkout
constituting ULP and that all the Unions officers, except Caete, had thereby lost their employment.

However, the Union thereafter filed a notice of strike, claiming that PINA was guilty of union busting through the
constructive dismissal of its officers. The strike was thereafter held. PINA retaliated by charging the petitioners
with ULP and abandonment of work.

LA the again declared the strike to be illegal.

NLRC sustained the finding that the strike was illegal, but reversed the LAs ruling that there was abandonment.
Hence NLRC directed the Company to reinstate respondents to their former positions but without backwages.

NLRCs decision of not awarding backwages was assailed in the CA, however the CA affirmed the NLRC. In
denying the petitioners claim for full backwages, the CA applied the third paragraph of Article 264(a) instead of
Article 279 of the Labor Code.

ISSUE: Are petitioners entitled to full backwages from the date of dismissal until the date of actual reinstatement
due to their not being found to have abandoned their jobs in an illegal strike?

RULING:
No. The petitioners contend that they are entitled to full backwages by virtue of their reinstatement, and submit
that applicable to their situation is Article 279, not the third paragraph of Article 264(a), both of the Labor Code.

We do not agree with the petitioners.

Article 279 provides:


Article 279. Security of Tenure. In cases of regular employment, the employer shall not terminate the services of
an employee except for a just cause or when authorized by this Title. An employee who is unjustly
dismissed from work shall be entitled to reinstatement without loss of seniority rights and other privileges and
to his full backwages, inclusive of allowances, and to his other benefits or their monetary equivalent computed
from the time his compensation was withheld from him up to the time of his actual reinstatement.

By its use of the phrase unjustly dismissed, Article 279 refers to a dismissal that is unjustly done, that is, the
employer dismisses the employee without observing due process, either substantive or procedural. Substantive
due process requires the attendance of any of the just or authorized causes for terminating an employee as
provided under Article 278 (termination by employer), or Article 283 (closure of establishment and reduction of
personnel), or Article 284 (disease as ground for termination), all of the Labor Code; while procedural due
process demands compliance with the twin-notice requirement.[17]

In contrast, the third paragraph of Article 264(a) states:

Art. 264. Prohibited activities. (a) xxx


Any worker whose employment has been terminated as a consequence of an unlawful lockout shall be entitled
to reinstatement with full backwages. Any union officer who knowingly participates in an illegal strike and any
worker or union officer who knowingly participates in the commission of illegal acts during a strike may be
declared to have lost his employment status; Provided, That mere participation of a worker in a lawful strike
San Beda College of Law 444
4S: 2015 - 2016
LABOR LAW REVIEW Atty. Joyrich Golangco

shall not constitute sufficient ground for termination of his employment, even if a replacement had been hired by
the employer during such lawful strike.

Contemplating two causes for the dismissal of an employee, that is: (a) unlawful lockout; and (b) participation in
an illegal strike, the third paragraph of Article 264(a) authorizes the award of full backwages only when the
termination of employment is a consequence of an unlawful lockout. On the consequences of an illegal strike,
the provision distinguishes between a union officer and a union member participating in an illegal strike. A union
officer who knowingly participates in an illegal strike is deemed to have lost his employment status, but a union
member who is merely instigated or induced to participate in the illegal strike is more benignly treated. Part of
the explanation for the benign consideration for the union member is the policy of reinstating rank-and-file
workers who are misled into supporting illegal strikes, absent any finding that such workers committed illegal
acts during the period of the illegal strikes.[18]

The petitioners were terminated for joining a strike that was later declared to be illegal. The NLRC ordered their
reinstatement or, in lieu of reinstatement, the payment of their separation pay, because they were mere rank-
and-file workers whom the Unions officers had misled into joining the illegal strike. They were not unjustly
dismissed from work. Based on the text and intent of the two aforequoted provisions of the Labor Code,
therefore, it is plain that Article 264(a) is the applicable one.

Petitioners not entitled to backwages despite their reinstatement. A fair days wage for a fair days labor. That
backwages are not granted to employees participating in an illegal strike simply accords with the reality that they
do not render work for the employer during the period of the illegal strike.

San Beda College of Law 445


4S: 2015 - 2016
LABOR LAW REVIEW Atty. Joyrich Golangco

Art. 263-264

VISAYAS COMMUNITY MEDICAL CENTER (VCMC), formerly known as METRO CEBU COMMUNITY
HOSPITAL (MCCH) vs. ERMA YBALLE, NELIA ANGEL, ELEUTERIA CORTEZ and EVELYN ONG
G.R. No. 196156 January 15, 2014
VILLARAMA, Jr., J.

Doctrine: The law makes a distinction between union members and union officers. A worker merely
participating in an illegal strike may not be terminated from employment. It is only when he commits illegal acts
during a strike that he may be declared to have lost employment status. In contrast, a union officer may be
terminated from employment for knowingly participating in an illegal strike or participates in the commission of
illegal acts during a strike.

FACTS: Respondents were hired as staff nurses and midwives by petitioner. NFL is the exclusive bargaining
representatives of the rank-and-file employees of MCCH. Nava, president of Nagkahiusang Mamumuno sa
MCCH (NAMA-MCCH-NFL) wrote Rev. Iyoy, MCCH Administrator, expressing the unions desire to renew the
CBA attaching a statement of proposal. However, MCCHI returned the CBA proposal for Nava to secure first the
endorsement of the legal counsel of NFL as the official bargaining representative of MCCHI employees.
Meanwhile, Atty. Alforque informed MCCHI that the proposed CBA submitted by Nava was never referred to
NFL and that NFL has not authorized any other legal counsel or any person for collective bargaining
negotiations.

Atty. Alforque suspended the union membership of Nava, et al for serious violation of the Constition and By-
Laws. The next day, several union members led by Nava and her group launched a series of mass actions such
as wearing black and red armbands/headbands, marching around the hospital premises and putting up
placards, posters and streamers. Atty. Alforque immediately disowned the concerted activities being carried out
by union members which are not sanctioned by NFL.

DOLE RO issued certifications stating that there is nothing in their records which shows that NAMA-MCCH- NFL
is a registered labor organization. MCCHI then sent individual notices to all union members asking them to
submit within 72 hours a written explanation why they should not be terminated for having supported the illegal
concerted activities of NAMA-MCCH-NFL which has no legal personality as per DOLE records.

Nava group refused to submit to investigation so MCCH sent termination letter to union leaders and other
members who participated in the strike and picketing activities. With the intensified atmosphere of violence and
animosity within the hospital premises as a result of continued protest activities by union members, MCCHI
suffered heavy losses due to low patient admission rates. The hospitals suppliers also refused to make further
deliveries on credit. MCCHI filed a petition for injunction in the NLRC. Several complaints for illegal dismissal
and unfair labor practice were filed by the terminated employees against MCCHI.

LA RULING: LA dismissed the claim of unfair labor practice and illegal dismissal and declaring the termination
of the respondents as an offshoot of the illegal strike, but directed the Hospital to pay the respondents their
separation pay.

NLRC RULING: NLRC affirmed the decision of LA but deleted the award of separation pay.

CA RULING: CA initially affirmed decision of NLRC. But, on certiorari, CA reversed and set aside NLRCs
decision.

ISSUE: Were the respondents illegally dismissed?

RULING: Yes. The law makes a distinction between union members and union officers. A worker merely
participating in an illegal strike may not be terminated from employment. It is only when he commits illegal acts
during a strike that he may be declared to have lost employment status. In contrast, a union officer may be
terminated from employment for knowingly participating in an illegal strike or participates in the commission of
illegal acts during a strike.

San Beda College of Law 446


4S: 2015 - 2016
LABOR LAW REVIEW Atty. Joyrich Golangco

The CA found that respondents participation was limited to the wearing of armband and thus, declared
respondents termination as invalid in the absence of any evidence that they committed any illegal act during the
strike.

With respect to backwages, the principle of a "fair days wage for a fair days labor" remains as the basic factor
in determining the award thereof. If there is no work performed by the employee there can be no wage or pay
unless, of course, the laborer was able, willing and ready to work but was illegally locked out, suspended or
dismissed or otherwise illegally prevented from working.

The alternative relief for union members who were dismissed for having participated in an illegal strike is the
payment of separation pay in lieu of reinstatement under the following circumstances: (a) when reinstatement
can no longer be effected in view of the passage of a long period of time or because of the realities of the
situation; (b) reinstatement is inimical to the employers interest; (c) reinstatement is no longer feasible; (d)
reinstatement does not serve the best interests of the parties involved; (e) the employer is prejudiced by the
workers continued employment; (f) facts that make execution unjust or inequitable have supervened; or (g)
strained relations between the employer and employee.

Considering that 15 years had lapsed from the onset of this labor dispute, and in view of strained relations that
ensued, in addition to the reality of replacements already hired by the hospital which had apparently recovered
from its huge losses, and with many of the petitioners either employed elsewhere, already old and sickly, or
otherwise incapacitated, separation pay without back wages is the appropriate relief.

San Beda College of Law 447


4S: 2015 - 2016
LABOR LAW REVIEW Atty. Joyrich Golangco

Art. 263-264

TABANGAO SHELL REFINERY EMPLOYEES ASSOCIATION vs. PILIPINAS SHELL PETROLEUM


CORPORATION
G.R. No. 170007 April 7, 2014
LEONARDO-DE CASTRO, J.

Doctrine: While the purpose of collective bargaining is the reaching of an agreement between the employer and
the employees union resulting in a binding contract between the parties, the failure to reach an agreement after
negotiations continued for a reasonable period does not mean lack of good faith

FACTS: In anticipation of the expiration of the CBA between the petitioner and the respondent, the parties
started negotiations for a new CBA. The union proposed a 20% annual across-the-board basic salary increase
for the next three years that would be covered by the new CBA. In lieu of the annual salary increases, the
company made a counter-proposal to grant all covered employees a lump sum amount of P80,000.00 yearly for
the three-year period of the new CBA. The union requested the company to present its counter-proposal in full
detail, similar to the presentation by the union of its economic proposal. Not satisfied with the companys
explanation, the union asked for further justification of the lump sum amount offered by the company. When the
company refused to acknowledge any obligation to give further justification, the union rejected the companys
counter-proposal and maintained its proposal for a 20% annual increase in basic pay for the next three years.
The union lowered its proposal to 12% annual across-the-board increase for the next three years. For its part,
the company increased its counter-proposal to a yearly lump sum payment of P88,000.00 for the next three
years. However, the union remained unconvinced and asked for additional documents to justify the companys
counter-offer. Alleging failure on the part of the company to justify its offer, the union manifested that the
company was bargaining in bad faith. On the parties 41st meeting, the company proposed the declaration of a
deadlock and recommended that the help of a third party be sought. On that same day, however, the union filed
a Notice of Strike in the NCMB. The Secretary of Labor and Employment assumed jurisdiction over the labor
dispute.

ISSUE: Is the company guilty of bad faith bargaining?

RULING: NO. The final and executory DecisioN of the Secretary of Labor and Employment squarely addressed
the contention of the union that the company was guilty of bargaining in bad faith. The said Decision correctly
characterized the nature of the duty to bargain, that is, it does not compel any party to accept a proposal or to
make any concession. While the purpose of collective bargaining is the reaching of an agreement between the
employer and the employees union resulting in a binding contract between the parties, the failure to reach an
agreement after negotiations continued for a reasonable period does not mean lack of good faith. The laws
invite and contemplate a collective bargaining contract but do not compel one. For after all, a CBA, like any
contract is a product of mutual consent and not of compulsion. As such, the duty to bargain does not include the
obligation to reach an agreement. In this light, the corporations unswerving position on the matter of annual
lump sum payment in lieu of wage increase did not, by itself, constitute bad faith even if such position caused a
stalemate in the negotiations.

San Beda College of Law 448


4S: 2015 - 2016
LABOR LAW REVIEW Atty. Joyrich Golangco

Art. 263-264

ASIA BREWERY vs. TUNAY NA PAGKAKAISA NG MGA MANGGAGAWA SA ASIA (TPMA)


G.R. Nos. 171594-96 September 18, 2013
DEL CASTILLO, J.:

DOCTRINE: In cases of compulsory arbitration before the Secretary of Labor pursuant to Article 263(g) of the
Labor Code, the financial statements of the employer must be properly audited by an external and independent
auditor in order to be admissible in evidence for purposes of determining the proper wage award.

FACTS: Tunay Na Pagkakaisa ng mga Manggagawa sa Asia (TPMA) is a legitimate labor organization,
certified as the sole and exclusive bargaining agent of all regular rank and file employees of [petitioner
corporation] Asia Brewery, Incorporated (ABI), a company engaged in the manufacture, sale and distribution of
beer, shandy, glass and bottled water products.

Respondent union and petitioner had been negotiating for a new CBA for the years 2003-2006 for 18 sessions
and negotiations but to no avail due to their differences on their respective positions on most items, particularly
on wages and other economic benefits.

Respondent union subsequently declared a deadlock and conducted a strike after filing a notice for one.
Petitioner then petitioned the Secretary of Labor to assume jurisdiction over the parties labor dispute, invoking
Article 263 (g) of the Labor Code. In answer, Respondent union opposed the assumption of jurisdiction,
reasoning therein that the business of petitioner corporation is not in dispensable to the national interest.

In the meantime, the Secretary resolved the deadlock between the parties concerning the dispute on the wages.
The secretary granted wage increase under the opposition of respondent contending that the ruling lacks
evidentiary proof to sufficiently justify the same.

CA: The computation of wage increase should be remanded to the Secretary of Labor because the computation
was based on petitioner corporations unaudited financial statements, which have no probative value pursuant to
the ruling in Restaurante Las Conchas v. Llego, and was done in contravention of DOLE Advisory No. 1, Series
of 2004, which contained the guidelines in resolving bargaining deadlocks

ISSUE: Whether or not the secretary of labor was correct in adjusting the wage increase based on unaudited
financial statements of the petitioner corporation.

HELD: NO. Secretary of Labor gravely abused her discretion when she relied on the unaudited financial
statements of petitioner corporation in determining the wage award because such evidence is self-serving and
inadmissible. Not only did this violate the December 19, 2003 Order of the Secretary of Labor herself to
petitioner corporation to submit its complete audited financial statements, but this may have resulted to a wage
award that is based on an inaccurate and biased picture of petitioner corporation's capacity to pay one of the
more significant factors in making a wage award.

Petitioner corporation has offered no reason why it failed and/or refused to submit its audited financial
statements for the past five years relevant to this case. This only further casts doubt as to the veracity and
accuracy of the unaudited financial statements it submitted to the Secretary of Labor. Verily, we cannot
countenance this procedure because this could unduly deprive labor of its right to a just share in the fruits of
production and provide employers with a means to understate their profitability in order to defeat the right of
labor to a just wage.

San Beda College of Law 449


4S: 2015 - 2016
LABOR LAW REVIEW Atty. Joyrich Golangco

Art. 263-264

ESCARIO, ET. AL. vs NLRC AND PINAKAMASARAP CORPORATION


G.R. No. 160302 September 27, 2010
BERSAMIN, J.:

EMPLOYEES DISMISSED FOR JOINING AN ILLEGAL STRIKE ARE NOT ENTITLED TO BACKWAGES
FOR THE PERIOD OF THE STRIKE WHEN REINSTATED.

DOCTRINE: With respect to backwages, the principle of a fair days wage for a fair days labor remains as the
basic factor in determining the award thereof. If there is no work performed by the employee there can be no
wage or pay unless, of course, the laborer was able, willing and ready to work but was illegally locked out,
suspended or dismissed or otherwise illegally prevented from working.

FACTS: Petitioners(who were also members of the the Union Malayang Samagan ng mga Manggagawa sa
Balanced Foods) were regular employees of respondent Pinakamasarap Corporation (PINA), a corporation
engaged in manufacturing and selling food seasoning.

On March 13, 1990, all officers and some 200 members of the said union walked out of PINAs premises to
support an officer charged with defamation by PINA. As a result of the said walkout, PINA preventively
suspended all officers of the Union and terminated their employment after a month.

The Union subsequently held a strike filing a notice claiming that PINA was guilty of union busting through the
constructive dismissal of its officers.

PINA countered by charging petitioners with ULP and abandonment of work, violating the CBA relating to
strikes. The NLRC also granted their TRO ordering the Union to cease and desist with their strike.

Labor Arbiter: LA declared the strike illegal

NLRC: The NLRC affirmed said decision but reversed LAs ruling that there was abandonment. NLRC also
added that petitioners were not entitled to backwages.

CA: The CA affirmed the NLRCs decision ruling that the only instance under Article 264 when a dismissed
employee would be reinstated with full backwages was when he was dismissed by reason of an illegal lockout;
that Article 264 was silent on the award of backwages to employees participating in a lawful strike; and that a
reinstatement with full backwages would be granted only when the dismissal of the petitioners was not done in
accordance with Article 282 (dismissals with just causes) and Article 283 (dismissals with authorized causes) of
the Labor Code.

ISSUE: Are petitioners, as participants of a strike subsequently declared illegal, entitled to backwages in
addition to the grant of reinstatement.

SC: NO. Article 279 provides that: An employee who is unjustly dismissed from work shall be entitled to
reinstatement without loss of seniority rights and other privileges and to his full backwages . By its use of the
phrase unjustly dismissed, Article 279 refers to a dismissal that is unjustly done, that is, the employer dismisses
the employee without observing due process, either substantive or procedural.

Substantive due process requires the attendance of any of the just or authorized causes for terminating an
employee as provided under Article 278 (termination by employer), or Article 283 (closure of establishment and
reduction of personnel), or Article 284 (disease as ground for termination), all of the Labor Code; while
procedural due process demands compliance with the twin-notice requirement.

Article 264 on the other hand contemplates two causes for the dismissal of an employee, to wit: (a) unlawful
lockout; and (b) participation in an illegal strike, the third paragraph of Article 264(a) authorizes the award of
full backwages only when the termination of employment is a consequence of an unlawful lockout.

On the consequences of an illegal strike, the provision distinguishes between a union officer and a union
member participating in an illegal strike. A union officer who knowingly participates in an illegal strike is deemed
San Beda College of Law 450
4S: 2015 - 2016
LABOR LAW REVIEW Atty. Joyrich Golangco

to have lost his employment status, but a union member who is merely instigated or induced to participate
in the illegal strike is more benignly treated. Part of the explanation for the benign consideration for the union
member is the policy of reinstating rank-and-file workers who are misled into supporting illegal strikes, absent
any finding that such workers committed illegal acts during the period of the illegal strikes.

The petitioners were terminated for joining a strike that was later declared to be illegal. The NLRC ordered their
reinstatement or, in lieu of reinstatement, the payment of their separation pay, because they were mere rank-
and-file workers whom the Unions officers had misled into joining the illegal strike. They were not unjustly
dismissed from work. Based on the text and intent of the two aforequoted provisions of the Labor Code,
therefore, it is plain that Article 264(a) is the applicable one.

San Beda College of Law 451


4S: 2015 - 2016
LABOR LAW REVIEW Atty. Joyrich Golangco

UNIVERSITY OF SAN AGUSTIN V. CA
G.R. No. 169632 March 28, 2006
GARCIA, J

FACTS:
Sometime on 2000, the parties agreed on a 5-year CBA, the economic provisions of which are effective
for 3 years only. After the lapse of 3 years, the parties negotiated on the economic provisions but did not agree
on the terms during the remaining 2 years of the CBA and beyond.

The parties did not agree on the computation of tuition incremental proceeds (TIP) which shall be the
basis for the increase of salaries. Unresolved, the Union declared a bargaining deadlock and thereafter filed a
Notice of Strike at the NCMB, which was expectedly opposed by the Univ through a Motion to Strike-out Notice
of Strike and Refer the Dispute to Voluntary Arbitration, since the CBA contained a "no-strike, no-lockout"
provision, and a grievance machinery for settling disputes, including a voluntary arbitration mechanism should
the grievance machinery fail to settle the dispute. The NCMB, however, failed to resolved the Univ's Motion

Thereafter, both parties made a joint request for the Secretary of Labor and Employment (SOLE) to
assume jurisdiction over the dispute.

On September 18, 2003, he SOLE assumed jurisdication, and with such assumption of jurisdiction, any strike or
lockout was strictly enjoined.

The day after the SOLE assumed jurisdiction, and on the same day that the Assumption of Jurisdiction Order
(AJO) was supposedly served to both parties, the Union staged a strike. Union members refused to receive a
copy of the AJO assailing that only the Union President is authorized to receive the same. The Union filed a
Petition Declare Illegal Strike and Loss of Employment Status of the striking employees, which Petition was filed
at the NLRC. Such Petition was later on consolidated with the case pending before the SOLE, at the request of
the Univ.

The SOLE rendered a Decision resolving the various economic issues over which the parties had a deadlock in
the collective bargaining, and likewise dismissed the Petition to Declare Illegal Strike.

The CA ruled that the SOLE abused its discretion in resolving the economic issues on the ground that
said issues were proper subject of the grievance machinery as embodied in the parties' CBA. Consequently, the
CA directed the parties to refer the economic issues of the CBA to voluntary arbitration. The CA, however, stood
firm in its finding that the strike conducted by the petitioner Union was illegal and its officers were deemed to
have lost their employment status.

ISSUES:
1. Whether or not the strike was illegal
2. Whether or not the dispute on the economic provisions of the CBA should have been forwarded to
Voluntary Arbitration
HELD:
On the first issue, the SC ruled that ART. 263 of the Labor Code provides: ."..Such assumption or
certification (of the SOLE) shall have the effect of automatically enjoining the intended or impending
strike or lockout as specified in the assumption or certification order. If one has already taken place at
the time of assumption or certification, all striking or locked out employees shall immediately return to
work and the employer shall immediately resume operations and readmit all workers under the same
terms and conditions prevailing before the strike or lockout." The phrase "immediately return to work"
indicates an almost instantaneous or automatic compliance for a striker to return to work once an AJO has been
duly served. Therefore, the act of the striking employees is violative of the foregoing provision.

On the second issue, the Supreme Court ruled that economic benefits, which included the issue on the formula
in computing the TIP share of the employees, is one that arises from the interpretation or implementation
of the CBA, and these matters should be referred to a Voluntary Arbitrator, as provided in Art. 261 and 262 of
the Labor Code. The peculiar facts of the instant case show that the University was deprived of a remedy that
would have enjoined the Union strike and was left without any recourse except to invoke the jurisdiction of the
SOLE.

San Beda College of Law 452


4S: 2015 - 2016
LABOR LAW REVIEW Atty. Joyrich Golangco

Art. 263-264

PHILIPPINE DIAMOND HOTEL AND RESORT INC V. MANILA DIAMOND HOTEL UNION
G.R no. 158075 June 30 2006
CARPIO MORALES, J

FACTS:
On November 11, 1996, the union, which was registered on August 19, 1996 before the Department of Labor
and Employment (DOLE), filed a Petition for Certification Election before the DOLE-National Capital Region
(NCR) seeking certification as the exclusive bargaining representative of its members.

The DOLE-NCR denied the unions petition as it failed to comply with legal requirements and was seen to
fragment the employees of the petitioner.

Through its president Kimpo, the union later notified petitioner of its intention to negotiate, by Notice to Bargain,a
Collective Bargaining Agreement (CBA) for its members.
Acting on the notice, the Hotel, through its Human Resource Development Manager, advised the union that
since it was not certified by the DOLE as the exclusive bargaining agent, it could not be recognized as such.
On the ground that the employer refused to bargain collectively, the union staged a strike. During the
conciliation proceedings the union requested that a consent election be held to which the petitioner interposed
no objection provided they comply with the requirements of the law.
However, the union went again on strike wherein several supervisory employees were seen joining them. An
ocular inspection report by the NLRC representative further showed that the strikers obstructed the free ingress
and egress from the hotel.
Despite the TRO issued by the NLRC, the strikers refused to dismantle the tents they put up at the employees
entrance of the hotel prompting the hotel guards to dismantle the same, in which the said guards suffered
injuries as they were hit by rocks coming from the direction of the strikers
The NLRC declared the strike as illegal and union officers and members are deemed to have lost their
employment status. It also dismissed the supervisory employees who participated in the strike.

ISSUE:
1. whether or not the strike was legal
2. whether or not the officers and employee members should be dismissed

HELD: The strike was illegal.

Only the labor organization designated or selected by the majority of the employees in an appropriate collective
bargaining unit is the exclusive representative of the employees in such unit for the purpose of collective
bargaining.

The union (hereafter referred to as respondent) is admittedly not the exclusive representative of the majority of
the employees of petitioner, hence, it could not demand from petitioner the right to bargain collectively in their
behalf.

On respondents contention that it was bargaining in behalf only of its members, the appellate court, affirming the
NLRCs observation that the same would only fragment the employees of petitioner, held that what [respondent]
will be achieving is to divide the employees, more particularly, the rank-and-file employees of [petitioner] . . . the
other workers who are not members are at a serious disadvantage, because if the same shall be allowed,
employees who are non-union members will be economically impaired and will not be able to negotiate their
terms and conditions of work, thus defeating the very essence and reason of collective bargaining, which is an
effective safeguard against the evil schemes of employers in terms and conditions of work.
Even if the purpose of a strike is valid, the strike may still be held illegal where the means employed are
illegal. Thus, the employment of violence, intimidation, restraint or coercion in carrying out concerted activities
which are injurious to the rights to property renders a strike illegal. And so is picketing or the obstruction to the
free use of property or the comfortable enjoyment of life or property, when accompanied by intimidation, threats,
violence, and coercion as to constitute nuisance.
Only union officers, and those members who have participated in the illegal acts should be deemed
dismissed from their employment.
San Beda College of Law 453
4S: 2015 - 2016
LABOR LAW REVIEW Atty. Joyrich Golangco

the union officers should be dismissed for staging and participating in the illegal strike, following paragraph 3,
Article 264(a) of the Labor Code which provides that . . .[a]ny union officer who knowingly participates in an
illegal strike and any worker or union officer who knowingly participates in the commission of illegal acts
during strike may be declared to have lost his employment status . . .

An ordinary striking worker cannot, thus be dismissed for mere participation in an illegal strike. There must be
proof that he committed illegal acts during a strike, unlike a union officer who may be dismissed by mere
knowingly participating in an illegal strike and/or committing an illegal act during a strike.

This Court made a distinction between two types of employees involved in a ULP: those who are discriminatorily
dismissed for union activities, and those who voluntarily go on strike even if it is in protest of an
ULP. Discriminatorily dismissed employees were ordered entitled to backpay from the date of the act of
discrimination, that is, from the day of their discharge, whereas employees who struck as a voluntary act of
protest against what they considered a ULP of their employer were held generally not entitled to backpay.

Jurisprudential law, however, recognizes several exceptions to the no backwages rule, to wit: when the
employees were illegally locked to thus compel them to stage a strike; when the employer is guilty of the
grossest form of ULP; when the employer committed discrimination in the rehiring of strikers refusing to readmit
those against whom there were pending criminal cases while admitting nonstrikers who were also criminally
charged in court; or when the workers who staged a voluntary ULP strike offered to return to work
unconditionally but the employer refused to reinstate them. Not any of these or analogous instances is,
however, present in the instant case.

San Beda College of Law 454


4S: 2015 - 2016
LABOR LAW REVIEW Atty. Joyrich Golangco

SUKHOTHAI CUISINE and RESTAURANT v. COURT OF APPEALS, NATIONAL LABOR RELATIONS
COMMISSION, PHILIPPINE LABOR ALLIANCE COUNCIL (PLAC) Local 460 Sukhothai Restaurant
Chapter, et., al.
G.R. No. 150437 July 17, 2006
AUSTRIA-MARTINEZ, J.:

DOCTRINE:
Strikes staged in violation of agreements providing for arbitration are illegal, since these agreements must be
strictly adhered to and respected if their ends are to be achieved.
In case of alleged union busting, the three remaining requirements notice, strike vote, and seven-day report
period cannot be dispensed with.

FACTS:
PLAC Local 460 Sukhothai Restaurant Chapter (Union), filed a Notice of Strike with the NCMB on the ground of
ULP. Sukhothai agreed and guaranteed that there will be no termination of the employees during the pendency
of the case, with the reservation of the management prerogative to issue memos to erring employees violation
of company policies. On the following day, a Strike Vote was conducted and the results of the vote were
submitted to the NCMB.

Thereafter Sukhothai and the Union agreed to submit the issue of ULP, the subject matter of the foregoing
Notice of Strike and the Strike Vote, for voluntary arbitration. During the pendency of the voluntary arbitration
proceedings, Sukhothai, dismissed a union member, due to an alleged petty quarrel with a co-employee. In view
of this termination, the Union filed with the NLRC a complaint for illegal dismissal.

Subsequently, another union member, was relieved from his post, and his employment terminated.
Subsequently, the Union protested Lanorias dismissal. Shortly thereafter, respondents staged a wildcat strike.
A Notice of Strike was re-filed by the private respondents and the protest, according to the respondents, was
converted into a sit-down strike. On the next day, the same was transformed into an actual strike. Sukhothai
then filed a complaint for illegal strike with the NLRC against private respondents, seeking, among others, to
declare the strike illegal.

LA DECISION: Strike is illegal, the Notice of Strike and the Strike Vote referred to a prior dispute submitted for
VA thus cannot be applied to the strike stage six months later.

NLRC DECISION: Strike is legal, Sukhothai is guilty of Union Busting and the Notice of Strike and Strike vote
are applicable to the subsequent strike because the same issues of ULP were involved and that ULP are
continuing offenses

CA DECISION: Affirmed NLRC decision

ISSUE: Is the strike staged by the Union illegal?

SC RULING: Yes. Under Art. 264, No strike or lockout shall be declared after assumption of jurisdiction by the
President or the Secretary or after certification or submission of the dispute to compulsory or voluntary
arbitration or during the pendency of cases involving the same grounds for the strike or lockout. At the time the
strike was staged, voluntary arbitration between the parties was ongoing by virtue of the Submission
Agreement. The issue to be resolved under those proceedings pertained to the very same issues stated in the
Notice of Strike, which is the commission of ULP, such as acts of harassment, fault-finding, and union busting
through coercion and interference with union affairs.

Strikes staged in violation of agreements providing for arbitration are illegal, since these agreements must be
strictly adhered to and respected if their ends are to be achieved. The rationale of the prohibition under Art. 264
is that once jurisdiction over the labor dispute has been properly acquired by competent authority, that
jurisdiction should not be interfered with by the application of the coercive processes of a strike.

The alleged dismissal of the union members are not sufficient to justify the radical recourse on the part of the
private respondents. The questions that surround their dismissal, are connected to the alleged breach of the
guarantee by the Sukhothai not to dismiss its employees during the pendency of the arbitration case. These

San Beda College of Law 455


4S: 2015 - 2016
LABOR LAW REVIEW Atty. Joyrich Golangco

matters should have been raised and resolved in the voluntary arbitration proceedings that were commenced
precisely to address them.

On the other hand, if private respondents believed that the disciplinary measures had nothing to do with the
issues under arbitration, then they should have availed of the appropriate remedies under the LC, such as the
institution of cases of illegal dismissal, or the submission of the cases to the grievance machinery of the CBA, or
simply seek to terminate the pending voluntary arbitration case and complete the mandatory procedure for a
lawful strike.

With respect to the argument respecting the allegation of union busting, it cannot be taken into account because
in case of union busting where the existence of the union is threatened, it is only the 15-day cooling-off period
that may be dispensed with and the three remaining requirements; notice, strike vote, and seven-day report
period cannot be dispensed with.

San Beda College of Law 456


4S: 2015 - 2016
LABOR LAW REVIEW Atty. Joyrich Golangco

BIFLEX PHILS. INC. LABOR UNION (NAFLU) v. FILFLEX INDUSTRIAL AND MANUFACTURING
CORPORATION and BIFLEX (PHILS.), INC.
G.R. No. 155679 December 19, 2006
CARPIO MORALES, J.:

WELGA NG BAYAN/GENERAL STRIKE

DOCTRINE: Stoppage of work due to welga ng bayan is in the nature of a general strike, an extended sympathy
strike. It affects numerous employers including those who do not have a dispute with their employees regarding
their terms and conditions of employment. Employees who have no labor dispute with their employer but who,
on a day they are scheduled to work, refuse to work and instead join a welga ng bayan commit an illegal work
stoppage.

FACTS:
The labor sector staged a welga ng bayan to protest the accelerating prices of oil. On even date, petitioner-
unions, led by their officers, (Petitioners), staged a work stoppage which lasted for several days, prompting
Filflex and Biflex (Respondents) to file a petition to declare the work stoppage illegal for failure to comply with
procedural requirements.

Thereafter respondents resumed their operations but petitioners were not allowed to enter the
premises. Petitioners, claiming that they were illegally locked out by respondents, assert that aside from the fact
that the welga ng bayan rendered it difficult to get a ride and the apprehension that violence would erupt
between those participating in the welga and the authorities, respondents workers were prevented from
reporting for work.

Respondents maintain that the work stoppage was illegal since the following requirements for the staging of a
valid strike were not complied with: (1) filing of notice of strike; (2) securing a strike vote, and (3) submission of a
report of the strike vote to the DOLE.

LA DECISION: Strike is illegal; the union officers are deemed terminated from employment.

NLRC DECISION: There was no strike to speak of as no labor or industrial dispute existed between the parties.
It accordingly ordered respondents to reinstate petitioners to their former positions, without loss of seniority
rights, and with full backwages from the date of their termination.

CA DECISION: Reinstated LA Decision.

ISSUE: Whether the work stoppage is legal.

SC RULING: No. Stoppage of work due to welga ng bayan is in the nature of a general strike, an extended
sympathy strike. It affects numerous employers including those who do not have a dispute with their employees
regarding their terms and conditions of employment.

Employees who have no labor dispute with their employer but who, on a day they are scheduled to work, refuse
to work and instead join a welga ng bayan commit an illegal work stoppage.

Even if petitioners joining the welga ng bayan were considered merely as an exercise of their freedom of
expression, freedom of assembly or freedom to petition the government for redress of grievances, the exercise
of such rights is not absolute. For the protection of other significant state interests such as the right of
enterprises to reasonable returns on investments, and to expansion and growth enshrined in the Constitution
must also be considered, otherwise, oppression or self-destruction of capital in order to promote the interests of
labor would be sanctioned. And it would give imprimatur to workers joining demonstrations/rallies even before
affording the employer an opportunity to make the necessary arrangements to counteract the implications of the
work stoppage on the business, and ignore the novel principle of shared responsibility between workers and
employers aimed at fostering industrial peace.

There being no showing that petitioners notified respondents of their intention, or that they were allowed by
respondents, to join the welga ng bayan, their work stoppage is beyond legal protection.

San Beda College of Law 457


4S: 2015 - 2016
LABOR LAW REVIEW Atty. Joyrich Golangco

Even assuming arguendo that in staging the strike, petitioners had complied with legal formalities, the strike
would just the same be illegal, for by blocking the free ingress to and egress from the company premises, they
violated Article 264(e) of the Labor Code which provides that [n]o person engaged in picketing shall obstruct the
free ingress to or egress from the employers premises for lawful purposes, or obstruct public thoroughfares.

San Beda College of Law 458


4S: 2015 - 2016
LABOR LAW REVIEW Atty. Joyrich Golangco

SANTA ROSA COCA-COLA PLANT EMPLOYEES UNION, ET AL v. COCA-COLA BOTTLERS PHILS INC
G.R. Nos. 164302-03 January 24, 2007
CALLEJO, J. SR.

FACTS: The Sta. Rosa Coca-Cola Plant Employees Union (Union) is the sole and exclusive bargaining
representative of the regular daily paid workers and the monthly paid non-commission-earning employees of the
Coca-Cola Bottlers Philippines, Inc. (Company) in its Sta. Rosa, Laguna plant. The individual petitioners are
Union officers, directors, and shop stewards.

Upon the expiration of the CBA, the Union informed the Company of its desire to renegotiate its terms. The CBA
meetings commenced. The Union insisted that representatives from the Alyansa ng mga Unyon sa Coca-Cola
be allowed to sit down as observers in the CBA meetings. The Union officers and members also insisted that
their wages be based on their work shift rates. For its part, the Company was of the view that the members of
the Alyansa were not members of the bargaining unit. The Alyansa was a mere aggregate of employees of the
Company in its various plants; and is not a registered labor organization. Thus, an impasse ensued.

The Union filed a "Notice of Strike" with the NCMB. Due to the motion to dismiss filed by the Company, the
Union amended their notice of strike to (a) unfair labor practice for the companys refusal to bargain in good
faith; and (b) interference with the exercise of their right to self-organization.

Union decided to participate in a mass action organized by the Alyansa ng mga Unyon sa Coca-Cola in front of
the Companys premises. Union members, officers and members of the Board of Directors, and shop stewards,
individually filed applications for leave of absence the said date. The Company disapproved all leave
applications and notified the applicants accordingly. A day before the mass action, some Union members wore
gears, red tag cloths stating "YES KAMI SA STRIKE" as headgears and on the different parts of their uniform,
shoulders and chests.

The Office of the Mayor issued a permit to the Union, allowing it "to conduct a mass protest action within the
perimeter of the Coca-Cola plant

All of the 14 personnel of the Engineering Section of the Company did not report for work, and 71 production
personnel were also absent. As a result, only one of the three bottling lines operated during the day shift. The
volume of production for the day was short by 60,000 physical case[s] versus budget.

The Company filed a "Petition to Declare Strike Illegal".

Unions answer: not a strike but just a valid exercise of their right to picket, which is part of the right of free
expression as guaranteed by the Constitution

LABOR ARBITER: Granted the petition of the Company. He declared that the September 21, 1999 mass leave
was actually a strike under Article 212 of the Labor Code. The strike conducted by the Union was illegal since
there was no showing that the Union conducted a strike vote, observed the prescribed cooling-off period, much
less, submitted a strike vote to the DOLE within the required time. Consequently, for knowingly participating in
the illegal strike, the individual petitioners were considered to have lost their employment status.

NLRC: Affirmed the LA with the modification that Union Treasurer Charlita M. Abrigo, who was on bereavement
leave at the time, should be excluded from the list of those who participated in the illegal strike. She was thus
ordered reinstated to her former position with full backwages and benefits.

CA: Dismissed the petition for lack of merit.

Petitioners also point out that members belonging to the IBM-KMU at the San Fernando Coca-Cola bottling
plant staged simultaneous walkout from their work assignments for two consecutive days

SECRETARY OF LABOR: Declared that the walkout was considered a mass action, not a strike, and the
officers of the IBM-KMU were only meted a three-day suspension.

ISSUES:
1. Whether the mass action staged by the Union was a strike;
San Beda College of Law 459
4S: 2015 - 2016
LABOR LAW REVIEW Atty. Joyrich Golangco

a. If, in the affirmative, whether it was legal; and
2. Whether the individual officers and shop stewards of petitioner Union should be dismissed from their
employment.

RULING: Petition is denied for lack of merit.

1. Yes. It was a strike and not merely a picket.

Article 212(o) of the Labor Code defines strike as a temporary stoppage of work by the concerted action
of employees as a result of an industrial or labor dispute.

The term "strike" encompasses not only concerted work stoppages, but also slowdowns, mass leaves,
sit-downs, attempts to damage, destroy or sabotage plant equipment and facilities, and similar activities.

Picketing involves merely the marching to and fro at the premises of the employer, usually accompanied
by the display of placards and other signs making known the facts involved in a labor dispute.

Picket means the stationing of one or more persons to observe and attempt to observe. The purpose of
pickets is said to be a means of peaceable persuasion

The basic elements of a strike are present in this case:


a. 106 members of petitioner Union, whose respective applications for leave of absence on
September 21, 1999 were disapproved, opted not to report for work on said date, and gathered
in front of the company premises to hold a mass protest action.
b. Petitioners deliberately absented themselves and instead wore red ribbons, carried placards
with slogans such as: "YES KAMI SA STRIKE," "PROTESTA KAMI," "SAHOD, KARAPATAN
NG MANGGAGAWA IPAGLABAN," "CBA-WAG BABOYIN," "STOP UNION BUSTING."

The bare fact that petitioners were given a Mayors permit is not conclusive evidence that their
action/activity did not amount to a strike.
What is definitive of whether the action staged by petitioners is a strike and not merely a picket is the
totality of the circumstances surrounding the situation.

The right to strike as a means of attainment of social justice is never meant to oppress or destroy the
employer.

The law has provided limitations on the right to strike, otherwise: the strike is illegal. For a strike to be
valid, the following procedural requisites provided by Art. 263 of the Labor Code must be observed:
a. a notice of strike filed with the DOLE 30 days before the intended date thereof, or 15 days in case of
unfair labor practice;
b. strike vote approved by a majority of the total union membership in the bargaining unit concerned
obtained by secret ballot in a meeting called for that purpose; and
c. notice given to the DOLE of the results of the voting at least seven days before the intended strike.

1.a. It was an illegal strike. Records reveal that the said strike did not comply with the requirements of Article
263 (F) in relation to Article 264 of the Labor Code

There is no showing that respondents had observed the prescribed cooling-off period, conducted a
strike vote, much less submitted a strike vote report to the Department of Labor within the required time.

The intention of the law in requiring the strike notice and strike-vote report as mandatory requirements is
to reasonably regulate the right to strike which is essential to the attainment of legitimate policy
objectives embodied in the law. Verily, substantial compliance with a mandatory provision will not
suffice. Strict adherence to the mandate of the law is required.

2. Yes. CA affirming the decisions of the NLRC and the Labor Arbiter ordering the dismissal of the petitioners-
officers, directors and shop stewards of petitioner Union is correct.

San Beda College of Law 460


4S: 2015 - 2016
LABOR LAW REVIEW Atty. Joyrich Golangco

The law makes a distinction between union members and union officers. A worker merely participating
in an illegal strike may not be terminated from employment. It is only when he commits illegal acts
during a strike that he may be declared to have lost employment status. For knowingly participating in
an illegal strike or participates in the commission of illegal acts during a strike, the law provides that a
union officer may be terminated from employment. The law grants the employer the option of declaring
a union officer who participated in an illegal strike as having lost his employment. It possesses the right
and prerogative to terminate the union officers from service

Petitioners had known beforehand that their planned mass leave would definitely result in a stoppage of
the operations of the company for September 21, 1999. Still, they knowingly and deliberately proceeded
with their mass action, unmindful of the ill effects thereof on the business operations of the company.

Union officers are duty-bound to guide their members to respect the law. If instead of doing so, the
officers urge the members to violate the law and defy the duly constituted authorities, their dismissal
from the service is just penalty or sanction for their unlawful acts. The officers responsibility is greater
than that of the members.

San Beda College of Law 461


4S: 2015 - 2016
LABOR LAW REVIEW Atty. Joyrich Golangco

MANILA HOTEL EMPLOYEES ASSOCIATION v. MANILA HOTEL CORPORATION
G.R. No. 154591 March 5, 2007
CHICO-NAZARIO, J.:

FACTS: Manila Hotel Employees Association (MHEA) filed a Notice of Strike with the NCMB on the grounds of
unfair labor practices. The Secretary of Labor and Employment (SOLE) certified the labor dispute to the NLRC
for compulsory arbitration pursuant to Article 263(g) of the Labor Code. Specifically, the Order enjoined any
strike or lockout and the parties were ordered to cease and desist from committing any acts that may
exacerbate the situation.

During the mandatory conference, the parties were advised of the certification order, which prohibited them from
taking any action that would exacerbate the situation. MHEA conducted a strike despite the clear terms of the
Order issued by the SOLE. On the same day, Commissioner Rayala called for a mandatory conference. NLRC
sought to have both parties identify the issues and stipulate the facts, despite their reluctance. It also allowed
the parties sufficient time to file their position papers, with which both parties failed to comply.

Manila Hotel filed a complaint with Prayer for Injunction and/or Temporary Restraining Order alleging that MHEA
conducted an illegal strike, blocked all ingress and egress of the hotel premises, harassed and intimidated
company officers, non-striking employees, customers and suppliers. In addition, it sought a declaration that the
strike was illegal and that, consequently, the striking employees lost their employment.

The NLRC issued an Order directing the striking workers to return to work immediately and the hotel to accept
them back under the same terms and conditions of employment. NLRC received Compliance filed by Manila
Hotel on 14 February 2000, manifesting that only six striking employees complied with the return-to-work Order
and were reinstated. The other striking employees had openly defied the said Order.

MHEA filed an Urgent Manifestation and Motion to Set Aside Order questioning the validity of the Order of the
SOLE which certified the case to the NLCR, was still pending with the SOLE. The said motion had prevented
the said Order of the SOLE from becoming final and executory. Thus, it alleged that the NLRC had not acquired
jurisdiction over the labor dispute.

NLRC issued another order ordering MHEA to refrain from putting up a blockade or barricade or any mode of
preventing the free ingress to and egress from the hotel. Parenthetically, it also ordered Manila Hotel to respect
the right of the striking workers to peacefully picket in a designated area outside the hotel. Manila Hotel moved
for the Reconsideration of the said Order on the ground that the picket, which they were ordered to respect, was
an unlawful activity.

NLRC: strike held by MHEA was illegal for its defiance of the return-to-work order. However, it determined that
only the union officers were deemed to have lost their employment. It ruled that there was no evidence showing
who among the striking employees were actually notified of the return-to-work order, and therefore, such
employees have not forfeited their employment.

CA: granted Manila Hotels petition and ruled that both the incumbent officers and members of the Union
involved in the illegal strike are declared to have lost their employment status.

ISSUE: MHEA members seek their reinstatement after participating in an illegal strike, that is, a strike that was
conducted after receiving an Order of assumption by the SOLE certifying the dispute to the NLRC for
compulsory arbitration. Worse still, the strikers failed to comply with the 11 February 2000 return-to-work Order,
issued by the NLRC, despite receipt thereof.

RULING:

Procedural defect of the case: It has been ruled that it is the party-pleader, and not the counsel, who must
execute the certificate against forum shopping. The rationale for the rule is that the counsel may be unaware of
any similar actions pending with other courts on the same matter. In this case, Ferdinand Barles was no longer
an officer of the union at the time this petition was filed, and therefore was no longer privy to the cases that may
have been filed by MHEA. Absent the specific authorization from the MHEA members that he sought to
represent, any statement he may make cannot bind the MHEA herein named. For the foregoing reasons alone,
this petition should be dismissed.
San Beda College of Law 462
4S: 2015 - 2016
LABOR LAW REVIEW Atty. Joyrich Golangco

Substantial aspect of the case:

The assumption of jurisdiction by the SOLE over labor disputes causing or likely to cause a strike or lockout in
an industry indispensable to the national interest is in the nature of a police power measure.

The Court reiterated the rule that a return-to-work order is immediately executory notwithstanding the filing of a
motion for reconsideration. It must be strictly complied with even during the pendency of any petition questioning
its validity. Returning to work in this situation is not a matter of option or voluntariness but of obligation. The
worker must return to his job together with his co-workers so the operations of the company can be resumed
and it can continue serving the public and promoting its interest.

MHEA claims that the Court should consider as a mitigating circumstance the fact that they held the strike three
months after filing their notice of strike. Such detail is irrelevant. What is crucial is that they were apprised of the
assumption order of the SOLE wherein they were enjoined from carrying out a strike.

They, likewise, imply that they were not served a copy of the return-to-work order. Such allegation loses
credence because MHEA, in its Urgent Manifestation and Motion to Set Aside Order dated 14 February 2000,
and Motion for Reconsideration dated 11 April 2000, admitted that a copy of the return-to-work order was served
on the picket lines. Records show that their counsel was likewise served a copy thereof during the 11 February
2000 conference and that he refused to acknowledge receipt.

MHEA cannot lean on the doctrine in the case of PNOC Dockyard and Engineering Corporation v. National
Labor Relations Commission. The Court, in the aforecited case, ruled that there was no valid service of the
certification order which prohibited any strike or lockout since the said order was served on the guard on duty
instead of the president of the union who was authorized to receive the same. As a result, the strike undertaken
after the issuance of the said order was considered legal, hence cannot effectively terminate the employment of
workers who joined the strike. In the present case, not only were the union officers apprised of the order, a copy
of the same was served on the picket lines

MHEA propounds the theory that both parties had acted in pari delicto and, therefore, the dismissal of its
members who participated in the illegal strike, was unwarranted. In the present case, nothing in the records
shows that Manila Hotel was guilty of an illegal lockout. It readmitted the six (6) employees who complied with
the return-to-work order. MHEA made a vague reference to striking employees who complied with the return-to-
work order, but were nevertheless refused re-admittance by Manila Hotel. However, they failed to even identify
these employees. There is no allegation that MHEA filed any case for illegal lock-out against Manila Hotel.

San Beda College of Law 463


4S: 2015 - 2016
LABOR LAW REVIEW Atty. Joyrich Golangco

Art. 263-264

G&S TRANSPORT CORPORATION v. INFANTE


533 SCRA 289
TING, J.:

FACTS: - Petitioner was the exclusive coupon taxi concessionaire at the NAIA for 5 years. Under such contract
the taxi units were given a garage located at the Duty Free compound just opposite NAIA.

NAIA sent a letter to the NAIA Service Taxi Employees Union (Union) demanding the dismissal from
employment Ricardo Gonzales and Ephraim Alzaga (both drivers of herein petitioner) on the grounds of
disloyalty and unbecoming of a union member inimical to the interests of the Union. Petitioner terminated said
drivers. This caused several drivers to stop driving their taxis apparently to sympathize with their dismissed
colleagues. Petitioner claims that this brought stoppage to its business operation and is one that squarely falls
under the concept of an illegal strike at the work premises. Petitioner filed an action for illegal strike before the
LA against 37 drivers (among those included private respondents Infante, Borbo and Castaeda). The 37
drivers filed an illegal dismissal case but most of them filed their affidavits of desistance.

Defense of Infante and Borbo: they reported to work but found no taxis in their garage and found out that on
that same day the protest to sympathize with their colleagues ensued. The day after the protest they did not
report back to work, as it was their day-off. However, when they returned to work they were refused entry by the
security guard because their names did not appear in the list of drivers allowed to work by the petitioner.

Defense of Castaeda: He was on a sick leave. But when I reported back to work it was on the same day that
the strike occurred. The following day, he reported back to work but was refused entry by the guard.

Decision of LA: concerted action by the drivers is illegal strike. Drivers undertook those collective actions
without first filing a notice to strike and taking a strike vote, and in violation of the no strike no lock out clause
in the CBA. The LA further noted that when the strike occurred there was no labor dispute (as Article 212
mandates that such stoppage of work by employees be a result of an industrial of labor disputes). LA found
respondents PARTICIPATED in the illegal strike BUT did not affirm their dismissal. Instead, ordered that
separation pay be given in lieu of reinstatement but without backwages. NLRC affirmed in toto LAs ruling.

CAs Decision: It remanded it to the LA for the computation of backwages and other monetary benefits. It
further noted that the dismissal of respondents was illegal. CA relied on a certification from SEC that petitioner
was still operational and that the LA and NLRC ruling for separation pay instead of reinstatement amounted to
grave abuse of discretion.

ISSUES:
1. Was there an illegal strike?
2. Whether the order for the payment of separation pay, in lieu of reinstatement without backwages, was
proper.

HELD:
1. YES. Article 212 of the Labor Code defines strike as any temporary stoppage of work by the concerted action
of employees as a result of an industrial or labor dispute. A valid strike therefore presupposes the existence of a
labor dispute. The strike undertaken by respondents took the form of a sit-down strike, or more aptly termed as
a sympathetic strike, where the striking employees have no demands or grievances of their own, but they
strike for the purpose of directly or indirectly aiding others, without direct relation to the advancement of the
interest of the strikers. It is indubitable that an illegal strike in the form of a sit-down strike occurred in petitioners
premises, as a show of sympathy to the two employees who were dismissed by petitioner. The office telegram
sent to individual respondents informing them to return to work went unheeded. Respondents failed to
satisfactorily explain their conspicuous absence following the day of the purported illegal strike.

What is the effect of their participation? Article 264 of the Labor Code, in providing for the consequences of an
illegal strike, makes a distinction between union officers and members who participated therein. Thus, knowingly
participating in an illegal strike is a valid ground for termination of employment of a union officer. The law,
however, treats differently mere union members. Mere participation in an illegal strike is not a sufficient ground
for termination of the services of the union members. There must be proof that he committed illegal acts during
San Beda College of Law 464
4S: 2015 - 2016
LABOR LAW REVIEW Atty. Joyrich Golangco

the strike and the striker who participated in the commission of illegal act must be identified. Proof beyond
reasonable doubt is not required. Substantial evidence available under the attendant circumstances, which may
justify the imposition of the penalty of dismissal, may suffice.

In the case at bar, this Court is not convinced that the affidavits of petitioners witnesses constitute substantial
evidence to establish that illegal acts were committed by respondents. Nowhere in their affidavits did these
witnesses cite the particular illegal acts committed by each individual respondent during the strike. Notably, no
questions during the hearing were asked relative to the supposed illegal acts.

As adverted to earlier, no matter by what term the respondents complainants used in describing their concerted
action, i.e. [,] protest, sympathy or mere expression, their joint action have successfully paralyzed the operations
of G & S Transport, and this is considered a strike.

It can now therefore be concluded that the acts of respondents do not merit their dismissal from employment
because it has not been substantially proven that they committed any illegal act while participating in the illegal
strike.

2. YES. Respondents dismissal from work could not be any clearer than the refusal of petitioner to admit them
back as they signified their intention to go back to work..
With respect to backwages, the principle of a "fair days wage for a fair days labor" remains as the basic factor
in determining the award thereof. If there is no work performed by the employee there can be no wage or pay
unless, of course, the laborer was able, willing and ready to work but was illegally locked out, suspended or
dismissed or otherwise illegally prevented from working. While it was found that respondents expressed their
intention to report back to work, the latter exception cannot apply in this case. In Philippine Marine Officers
Guild v. Compaia Maritima, as affirmed in Philippine Diamond Hotel and Resort v. Manila Diamond Hotel
Employees Union, the Court stressed that for this exception to apply, it is required that the strike be legal, a
situation that does not obtain in the case at bar. Under the circumstances, respondents reinstatement without
backwages suffices for the appropriate relief. If reinstatement is no longer possible, given the lapse of
considerable time from the occurrence of the strike, the award of separation pay of one (1) month salary for
each year of service, in lieu of reinstatement, is in order.

Section 4, Rule I of the Rules Implementing Book VI of the Labor Code provides:
Reinstatement to former position.(a) An employee who is separated from work without just cause
shall be reinstated to his former position, unless such position no longer exists at the time of his
reinstatement, in which case he shall be given a substantially equivalent position in the same
establishment without loss of seniority rights.

The above-quoted rule enunciates reinstatement as the standard relief. However, in this case, seventeen (17)
years have elapsed since respondents were illegally dismissed. In Association of Independent Unions in the
Philippines v. NLRC, where more than eight (8) years have passed since the petitioners therein staged an illegal
strike and were found to have been unlawfully terminated, an award of separation pay equivalent to one (1)
month pay for every year of service, in lieu of reinstatement, was deemed more practical and appropriate to all
the parties concerned. We adopt the same tack in this case.

San Beda College of Law 465


4S: 2015 - 2016
LABOR LAW REVIEW Atty. Joyrich Golangco

Art. 263-264

STEEL CORPORATION OF THE PHILIPPINES, v. SCP EMPLOYEES UNION-NATIONAL FEDERATION OF


LABOR UNIONS.
551 SCRA 595
AZCUNA, J.:

FACTS: Petitioner Steel Corporation of the Philippines (SCP) is engaged in manufacturing construction
materials. SCP-Federated Union of the Energy Leaders-General and Allied Services (FUEL-GAS) filed a petition
for Certification Election in its bid to represent the rank-and-file employees of the petitioner. Respondent SCP
Employees Union-National Federation of Labor Unions (SCPEU-NAFLU) intervened, seeking to participate and
be voted for in such election but the same was denied for having been filed out of time.

A consent election was conducted, with "FUEL-GAS" and "NO UNION" as choices. Said election was however
declared a failure because less than a majority of the rank-and-file employees cast their votes. FUEL-GAS filed
an Election Protest claiming that the certification election of respondent was characterized by and replete with
irregularities.

NAFLU, the mother federation of respondent, filed a petition for Certification Election for and on behalf of its
affiliate, seeking to represent the rank-and-file employees of petitioner.

The Med-Arbiter denied the election protest of FUEL-GAS and granted the petition for certification election filed
by NAFLU and further ordered the conduct of the election with "NAFLU" and "NO UNION" as choices.

Petitioner and FUEL-GAS appealed to the Secretary of Labor. The DOLE Undersecretary ordered the conduct
of a certification election with "FUEL-GAS," respondent SCPEU-NAFLU and "NO UNION" as choices.
Subsequent motions for reconsideration were denied. Unsatisfied, petitioner and FUEL-GAS appealed to the CA
by way of certiorari.

On April 14, 2000, the certification election proceeded. FUEL-GAS participated without prejudice to the decision
of the CA in its pending petition. In said election, respondent SCPEU-NAFLU emerged as winner. FUEL-GAS
filed a second election protest.

The CA annulled and set aside the DOLE Usecs decision and resolution, and directed the holding of a
certification election with "FUEL-GAS" and "NO UNION" as choices, to the exclusion of SCPEU-NAFLU.
The Med-Arbiter dismissed FUEL-GAS' election protest but deferred the declaration of respondent as winner in
the certification election until final resolution of the pending petitions with the CA.

Respondent appealed to the Labor Secretary. It further filed a Manifestation before the CA pointing out that in
the certification election, it emerged as winner, and thus, the election should be considered as an intervening
event sufficient to bar another certification election. The CA, however, dismissed said manifestation.
Meanwhile, the Usec rendered a Decision certifying respondent as the exclusive bargaining agent of petitioner's
employees. Petitioner and FUEL-GAS filed MRs of the aforesaid decision.

As a consequence of its certification as the exclusive bargaining agent, respondent sent to petitioner CBA
proposals. Petitioner, however, held in abeyance any action on the proposals in view of its pending motion for
reconsideration.

Thus, respondent filed a Notice of Strike with NCMB raising the issue of unfair labor practice (ULP) allegedly
committed by petitioner for its refusal to bargain. The labor dispute was certified to the NLRC for compulsory
arbitration. Another Notice of Strike was filed by respondent which was later consolidated with the certified case.

NLRC held that petitioner has no obligation to recognize respondent as the certified bargaining agent;
dismissing the charge of ULP; declaring the strike as illegal; and declaring the loss of employment of the officers
of the union.

On May 20, 2002, respondent filed another Notice of Strike which was dismissed and respondent was enjoined
from holding a strike. On Jan. 7, 2003, it again filed another Notice of Strike and thereafter went on strike on
February 4, 2003. The Labor Secretary certified the dispute to the NLRC and directed the employees to return
San Beda College of Law 466
4S: 2015 - 2016
LABOR LAW REVIEW Atty. Joyrich Golangco

to work. NLRC ordered petitioner to bargain collectively with respondent as the duly certified bargaining agent. It
also ordered the reinstatement of the employees dismissed in connection with the strike.

Meantime, in the first certified case, the NLRC ordered the reinstatement of the union officers whom it previously
ordered terminated.

CA affirmed the order of reinstatement but nullified and set aside the second certification election. It added that
since petitioner did not recognize the second certification election where NAFLU won, respondent has no basis
for its claim and no right to demand that petitioner collectively bargain with it.

Petitioner filed MR which was denied; hence, this petition.


(Whether or not respondent is the recognized collective bargaining agent had been finally resolved in the
negative)

ISSUES: 1. Was the strike participated in by the officers of the respondent union is valid?
2. Was the respondent union officers termination from employment by reason of such participation valid?

HELD:
1, NO. The strike is a legitimate weapon in the human struggle for a decent existence. It is considered as the
most effective weapon in protecting the rights of the employees to improve the terms and conditions of their
employment. But to be valid, a strike must be pursued within legal bounds. The right to strike as a means for the
attainment of social justice is never meant to oppress or destroy the employer. The law provides limits for its
59
exercise.

In the instant case, the strike undertaken by the officers of respondent union is patently illegal for the following
reasons: (1) it is a union-recognition-strike which is not sanctioned by labor laws; (2) it was undertaken after the
dispute had been certified for compulsory arbitration; and (3) it was in violation of the Secretary's return-to-work
order.

Respondent's notices of strike were founded on petitioner's continued refusal to bargain with it. It thus staged
the strike to compel petitioner to recognize it as the collective bargaining agent, making it a union-recognition-
strike. As its legal designation implies, this kind of strike is calculated to compel the employer to recognize one's
union and not other contending groups, as the employees' bargaining representative to work out a collective
bargaining agreement despite the striking union's doubtful majority status to merit voluntary recognition and lack
of formal certification as the exclusive representative in the bargaining unit.

The certification election that was conducted where respondent emerged as winner, not having been recognized
as valid, it has no authority to represent the rank and file employees of petitioner. Thus, it could not ask
petitioner to bargain with it. As the issue of its identity had been the subject of a separate case which had been
settled by the court with finality, petitioner cannot, therefore, be faulted in refusing to bargain. Neither could this
Court sustain respondent's imputation of unfair labor practice and union busting against petitioner. With more
reason, this Court cannot sustain the validity of the strike staged on such basis.

Even if this Court were to uphold the validity of respondent's purpose or objective in staging a strike, still, the
strike would be declared illegal for having been conducted in utter defiance of the Secretary's return-to-work
order and after the dispute had been certified for compulsory arbitration. Although ostensibly there were several
notices of strike successively filed by respondent, these notices were founded on substantially the same
grounds petitioner's continued refusal to recognize it as the collective bargaining representative.

The powers granted to the Secretary under Article 263(g) of the Labor Code have been characterized as an
exercise of the police power of the State, aimed at promoting the public good. When the Secretary exercises
these powers, he is granted "great breadth of discretion" to find a solution to a labor dispute. The most obvious
of these powers is the automatic enjoining of an impending strike or lockout or its lifting if one has already taken
place.

The moment the Secretary of Labor assumes jurisdiction over a labor dispute in an industry indispensable to
national interest, such assumption shall have the effect of automatically enjoining the intended or impending
strike. It was not even necessary for the Secretary of Labor to issue another order directing a return to work.

San Beda College of Law 467


4S: 2015 - 2016
LABOR LAW REVIEW Atty. Joyrich Golangco

The mere issuance of an assumption order by the Secretary of Labor automatically carries with it a return-to-
work order, even if the directive to return to work is not expressly stated in the assumption order.

A return-to-work order imposes a duty that must be discharged more than it confers a right that may be waived.
While the workers may choose not to obey, they do so at the risk of severing their relationship with their
employer.

Returning to work in this situation is not a matter of option or voluntariness but of obligation. The worker must
return to his job together with his co-workers so that the operations of the company can be resumed and it can
continue serving the public and promoting its interest. This extraordinary authority given to the Secretary of
Labor is aimed at arriving at a peaceful and speedy solution to labor disputes, without jeopardizing national
interests. Regardless of their motives, or the validity of their claims, the striking workers must cease and/or
desist from any and all acts that undermine or tend to undermine this authority of the Secretary of Labor, once
an assumption and/or certification order is issued. They cannot, for instance, ignore return-to-work orders, citing
unfair labor practices on the part of the company, to justify their action.
xxx
The strike, having been staged after the dispute had been certified for arbitration and contrary to the return-to-
work order, became a prohibited activity, and was thus illegal.

2. YES. The law (Art. 264) makes a distinction between union members and union officers. A worker merely
participating in an illegal strike may not be terminated from employment. It is only when he commits illegal acts
during a strike that he may be declared to have lost employment status. For knowingly participating in an illegal
strike or participating in the commission of illegal acts during a strike, the law provides that a union officer may
be terminated from employment. The law grants the employer the option of declaring a union officer who
participated in an illegal strike as having lost his employment. It possesses the right and prerogative to terminate
the union officers from service. Otherwise, the workers will simply refuse to return to their work and cause a
standstill in the company operations while retaining the positions they refuse to discharge and preventing
management from filling up their positions.

San Beda College of Law 468


4S: 2015 - 2016
LABOR LAW REVIEW Atty. Joyrich Golangco

Art. 263-264

CHRIS GARMENTS CORPORATION vs. HON. PATRICIA A. STO. TOMAS and CHRIS GARMENTS
WORKERS UNION-PTGWO LOCAL CHAPTER No. 832
G.R. No. 167246 January 12, 2009
QUISUMBING, J.:

DOCTRINE: It is settled that the filing of MR is a prerequisite to the filing of a special civil action for certiorari to
give the lower court the opportunity to correct itself. This rule, however, admits of exceptions, such as when the
MR would be useless under the circumstances.

FACTS:
Chris Garmets Workers Union -PTGWO filed a petition for certification election with the med-arb, seeking to
represent petitioner's employees not covered by petitioner'sthe current CBA with SMCGC-SUPER. Petitioner
moved to dismiss because there is an existing CBA from July 1, 1999 to June 30, 2004 with SMCGC-SUPER
which bars any petition for certification election prior to the 60-day freedom period.

The med-arb dismissed the petition holding that the contract bar rule under Art. 232 applies and that there is no
Er-Ee relationship as it is the union itself which admitted that its members are agency employees. The Secretary
of Labor affirmed.

A second petition for certification was made by respondent union within the 60 day freedom period but was still
denied by the med-arb and affirmed by the secretary.

A third one was filed but was also dismissed due to the absence of Er-Ee relationship. The secretary granted
the petition this time.

Petitioner filed a petition for certiorari with the CA which was dismissed due to its failure to file a motion for
reconsideration of the secretary's decision prior to the petition.

A certification election was conducted among petitioner's rank and file employees and SMCGC-SUPER
emerged as the winner.

ISSUE: Was an MR necessary before petitioner could file a petition for certiorari of the decision of the
Secretary?

HELD: NO. It is settled that the filing of MR is a prerequisite to the filing of a special civil action for certiorari to
give the lower court the opportunity to correct itself. This rule, however, admits of exceptions, such as when the
MR would be useless under the circumstances. Furthermore, under DO 04-03 series of 2013, the decision of
the Secretary shall be final and executory after 10 days from receipt thereof by the parties and it shall not be
subject to an MR. Clearly, petitioner availed of the proper remedy since the DO expressly prohibits the filing of
an MR. Such motion becomes dispensable and not at all necessary.

ISSUE: Was the third case barred by res judicata?

HELD: No. The elements of res judicata are: a) the judgement sought to bar the new action must be final; b)
the judgment must have been rendered by a court having jurisdiction over the parties and the subject matter; c)
the disposition of the case must be a judgment on the merits; and d) identity of the parties, subject matter and
cause of action.

There is no identity of causes of action to speak of since in the first petition, the union has no cause of action
while in the third, a cause of action already exists for the union as they are now legally allowed to challenge the
status of SMCGC-SUPER as exclusive bargaining representative.

San Beda College of Law 469


4S: 2015 - 2016
LABOR LAW REVIEW Atty. Joyrich Golangco

Art. 277 (b)

ST. LUKES MEDICAL CENTER, INC. and ROBERT KUAN, Chairman vs. ESTRELITO NOTARIO
G.R. No. 152166 October 20, 2010
PERALTA, J.:

DOCTRINE: Where the dismissal was without just cause and there was no due process, Article 279 of the
Labor Code, as amended, mandates that the employee is entitled to reinstatement without loss of seniority
rights and other privileges and full backwages, inclusive of allowances and other benefits, or their monetary
equivalent computed from the time the compensation was not paid up to the time of actual reinstatement.

FACTS:
St. Lukes employed respondent as In-House Security Guard, mainly monitoring the CCTVs in the hospital.
In December 1996, a foreigner from Marshall Island, Justin Tibon, reported that he lost his mint green travelling
bag, placed inside the cabinet of the room where his daughter was confined. It contained, among others, two (2)
Continental Airlines tickets, two (2) passports, and some clothes. When he brought this incident to security, the
cameras failed to record any theft incident at room 257.

In his letter dated January 6, 1997, respondent explained that on the subject dates, he was the only personnel
on duty as nobody wanted to assist him. Because of this, he decided to focus the cameras on the Old and New
Maternity Units, as these two units have high incidence of crime. Finding the written explanation to be
unsatisfactory, St. Lukes served on respondent a copy of the Notice of Termination, dismissing him on the
ground of gross negligence/inefficiency under Section 1, Rule VII of its Code of Discipline.

Thus respondent filed a Complaint for illegal dismissal against petitioners, seeking reinstatement with payment
of full backwages from the time of his dismissal up to actual reinstatement, without loss of seniority rights and
other benefits.

LA dismissed respondents complaint, stating that a CCTV monitoring system is designed to focus on many
areas in a programmed and sequential manner and should not to be focused only on a specific area, unless the
situation requires it.

NLRC reversed the LA, stating that petitioners failed to submit proof that there was an existing Standard
Operating Procedure (SOP) in the CCTV monitoring system, particularly on the focusing procedure.
CA dismissed petitioners petition for certiorari, affirming that respondents negligence does not constitute
sufficient ground to terminate him. Moreover, it declared that petitioners failed to comply with the twin notice rule
and hearing as what they did was to require respondent to submit a written explanation, within 24 hours and,
thereafter, he was ordered dismissed, without affording him an opportunity to be heard.

ISSUE: WoN respondent was illegally dismissed.

HELD:
Contrary to the stance of petitioners, respondent was illegally dismissed without just cause and compliance with
the notice requirement.

Article 282 (b) of the Labor Code provides that an employer may terminate an employment for gross and
habitual neglect by the employee of his duties. To effectuate a valid dismissal, the Labor two requirements must
be met: (1) the dismissal must be for any of the causes provided in Article 282 of the Labor Code; and (2) the
employee must be given an opportunity to be heard and defend himself. An employer can terminate the
services of an employee only for valid and just causes which must be supported by clear and convincing
evidence.

A perusal of petitioner hospitals CCTV Monitoring Guidelines, disseminated to all in-house security personnel,
reveals that that there is no categorical provision requiring an in-house security personnel to observe a rotation
sequence procedure in focusing the cameras so that the security monitoring would cover as many areas as
possible.

Petitioners question the findings of the CA that there was no compliance with the twin-notice rule and hearing,
while respondent maintains that they violated his right to due process. The employee must be furnished two
San Beda College of Law 470
4S: 2015 - 2016
LABOR LAW REVIEW Atty. Joyrich Golangco

written notices: the first notice apprises the employee of the particular acts or omissions for which his dismissal
is sought, and the second is a subsequent notice, which informs the employee of the employer's decision
to dismiss him.

The facts showed that on January 6, 1997, petitioner hospital, through Abdul A. Karim, issued a Memorandum
to respondent, with the directive to require him to explain in writing, within 24 hours upon receipt thereof, why no
disciplinary action should be taken against him for violating the normal rotation or sequencing process of the
VCR which led to the loss of the traveling bag of Tibon, the patients father, at room 257. On the same day,
January 6, 1997, respondent submitted a written explanation. Later, on January 24, 1997, petitioner hospital
served a copy of the Notice of Termination upon the respondent for gross negligence/inefficiency.

Where the dismissal was without just cause and there was no due process, Article 279 of the Labor Code, as
amended, mandates that the employee is entitled to reinstatement without loss of seniority rights and other
privileges and full backwages, inclusive of allowances and other benefits, or their monetary equivalent computed
from the time the compensation was not paid up to the time of actual reinstatement.

Petitioners lack of just cause and non-compliance with the procedural requisites in terminating respondents
employment renders them guilty of illegal dismissal. Consequently, respondent is entitled to reinstatement to his
former position without loss of seniority rights and payment of backwages.

San Beda College of Law 471


4S: 2015 - 2016
LABOR LAW REVIEW Atty. Joyrich Golangco

Art. 277 (b)

ARMANDO ALILING v. JOSE B. FELICIANO, MANUEL F. SAN MATEO III, JOSEPH R. LARIOSA, and
WIDE WIDE WORLD EXPRESS CORPORATION
GR No. 185829 April 25, 2012
VELASCO, Jr. J.

PROCEDURAL DUE PROCESS

FACTS:
Respondent employed petitioner as Account Executive (Seafreight Sales). On June 11, 2004, Aliling and
WWWEC inked an Employment Contract providing the following terms:
Conversion to regular status shall be determined on the basis of work performance; and
Employment services may, at any time, be terminated for just cause or in accordance with the standards
defined at the time of engagement.

Instead of a Seafreight Sale assignment, WWWEC asked Aliling to handle a new company product. Marketing
this product and finding daily contracts for it formed the core of Alilings new assignment. San Mateo, WWWEC
Sales and Marketing Director, expressed dissatisfaction with Alilings performance.

Lariosa, Human Resources Manager of WWWEC, asked Aliling to explain his absence taken without leave.
Aliling denied being absent on the days in question. Alilings explanation came with a query regarding the
withholding of his salary.

In a separate letter, Aliling wrote San Mateo tendering his resignation. While WWWEC took no action on his
tender, Aliling nonetheless demanded reinstatement and a written apology, claiming in a subsequent letter to
management that San Mateo had forced him to resign.

Lariosa informed Aliling that his case was still in the process of being evaluated. On October 6, 2004, Lariosa
again wrote, this time to advise Aliling of the termination of his services effective as of that date owing to his
non-satisfactory performance during his probationary period.

Earlier, however, or on October 4, 2004, Aliling filed a Complaint for illegal dismissal due to forced resignation,
nonpayment of salaries as well as damages with the NLRC against WWWEC. As a defense, WWWEC attached
to its Position Paper a memo in which San Mateo asked Aliling to explain why he should not be terminated for
failure to meet the expected job performance. According to WWWEC, Aliling, instead of explaining himself,
simply submitted a resignation letter.

LA: Declared that Alilings termination was UNJUSTIFIED. The labor arbiter explained that Aliling cannot be
validly terminated for non-compliance with the quota threshold absent a prior advisory of the reasonable
standards upon which his performance would be evaluated.

NLRC: Affirmed LA

CA: Affirmed decision.

ISSUE: Was Alilings dismissal proper?

HELD: NO.
To effect a legal dismissal, the employer must show not only a valid ground therefor, but also that procedural
due process has properly been observed. When the Labor Code speaks of procedural due process, the
reference is usually to the two (2)-written notice rule envisaged in Section 2 (III), Rule XXIII, Book V of the
Omnibus Rules Implementing the Labor Code, which provides:

Section 2. Standard of due process: requirements of notice. In all cases of termination of employment, the
following standards of due process shall be substantially observed.

I. For termination of employment based on just causes as defined in Article 282 of the Code:

San Beda College of Law 472


4S: 2015 - 2016
LABOR LAW REVIEW Atty. Joyrich Golangco

(a) A written notice served on the employee specifying the ground or grounds for termination, and giving to said
employee reasonable opportunity within which to explain his side;

(b) A hearing or conference during which the employee concerned, with the assistance of counsel if the
employee so desires, is given opportunity to respond to the charge, present his evidence or rebut the evidence
presented against him; and

(c) A written notice [of] termination served on the employee indicating that upon due consideration of all the
circumstance, grounds have been established to justify his termination.

Here, the first and second notice requirements have not been properly observed, thus tainting petitioners
dismissal with illegality.

The adverted memo of WWWEC supposedly informing Aliling of the likelihood of his termination and directing
him to account for his failure to meet the expected job performance would have had constituted the charge
sheet, sufficient to answer for the first notice requirement, but for the fact that there is no proof such letter had
been sent to and received by him. In fact, in his December 13, 2004 Complainants Reply Affidavit, Aliling goes
on to tag such letter/memorandum as fabrication. WWWEC did not adduce proof to show that a copy of the
letter was duly served upon Aliling. Clearly enough, WWWEC did not comply with the first notice requirement.

Neither was there compliance with the imperatives of a hearing or conference. On the contrary, in its October 1,
2004 letter to Aliling, or barely five (5) days after it served the notice of termination, WWWEC acknowledged that
it was still evaluating his case. And the written notice of termination itself did not indicate all the circumstances
involving the charge to justify severance of employment.

San Beda College of Law 473


4S: 2015 - 2016
LABOR LAW REVIEW Atty. Joyrich Golangco

Art. 277 (b)

FELIX B. PEREZ and AMANTE G. DORIA v. PHILIPPINE TELEGRAPH AND TELEPHONE COMPANY and
JOSE LUIS SANTIAGO
GR No. 152048 April 7, 2009
CORONA J.:

HEARING NOT MANDATORY; DUE PROCESS

FACTS:
Petitioners were employed by respondent PT&T as shipping clerk and supervisor, in PT&Ts Shipping Section,
Materials Management Group.

Acting on an alleged unsigned letter regarding anomalous transactions at the Shipping Section, respondents
formed a special audit team to investigate the matter. It was discovered that the Shipping Section jacked up the
value of the freight costs for goods shipped and that the duplicates of the shipping documents allegedly showed
traces of tampering, alteration and superimposition.

On September 3, 1993, petitioners were placed on preventive suspension for 30 days for their alleged
involvement in the anomaly.Their suspension was extended for 15 days twice: first on October 3, 1993 and
second on October 18, 1993.

On October 29, 1993, a memorandum was issued by respondents dismissing petitioners from the service for
having falsified company documents.

On November 9, 1993, petitioners filed a complaint for illegal suspension and illegal dismissal. They alleged that
they were dismissed on November 8, 1993, the date they received the above-mentioned memorandum.

LA: Suspension and dismissal both ILLEGAL.

NLRC: REVERSED LAs decision.

CA: The CA affirmed the NLRC decision insofar as petitioners illegal suspension for 15 days and dismissal for
just cause were concerned. However, it found that petitioners were dismissed without due process.

ISSUE:
1. Is a hearing (or conference) mandatory in cases involving the dismissal of an employee?
2. Is there a valid cause for petitioners dismissal?
3. Is the dismissal valid?

HELD:
1. NO.
There is no need for a hearing or conference.
We note a marked difference in the standards of due process to be followed as prescribed in the Labor
Code and its implementing rules. The Labor Code, on one hand, provides that an employer must provide the
employee ample opportunity to be heard and to defend himself with the assistance of his representative if
he so desires. The omnibus rules implementing the Labor Code, on the other hand, require a hearing and
conference during which the employee concerned is given the opportunity to respond to the charge, present his
evidence or rebut the evidence presented against him.
In case of conflict, the law prevails over the administrative regulations implementing it.
A hearing means that a party should be given a chance to adduce his evidence to support his side of the
case and that the evidence should be taken into account in the adjudication of the controversy. To be heard
does not mean verbal argumentation alone inasmuch as one may be heard just as effectively through written
explanations, submissions or pleadings. Therefore, while the phrase ample opportunity to be heard may in fact
include an actual hearing, it is not limited to a formal hearing only. In other words, the existence of an actual,
formal trial-type hearing, although preferred, is not absolutely necessary to satisfy the employees right to be
heard.
It is enough that the parties are given a fair and reasonable opportunity to explain their respective sides of
the controversy and to present supporting evidence on which a fair decision can be based.
San Beda College of Law 474
4S: 2015 - 2016
LABOR LAW REVIEW Atty. Joyrich Golangco

2. NO.
Willful breach by the employee of the trust reposed in him by his employer or duly authorized
representative is a just cause for termination. However,loss of confidence should not be simulated. It should not
be used as a subterfuge for causes which are improper, illegal or unjustified. Loss of confidence may not be
arbitrarily asserted in the face of overwhelming evidence to the contrary. It must be genuine, not a mere
afterthought to justify an earlier action taken in bad faith.

The burden of proof rests on the employer to establish that the dismissal is for cause in view of the
security of tenure that employees enjoy under the Constitution and the Labor Code. The employers evidence
must clearly and convincingly show the facts on which the loss of confidence in the employee may be fairly
made to rest. It must be adequately proven by substantial evidence. Respondents failed to discharge this
burden.

3. NO.
Respondents illegal act of dismissing petitioners was aggravated by their failure to observe due process.
To meet the requirements of due process in the dismissal of an employee, an employer must furnish the worker
with two written notices: (1) a written notice specifying the grounds for termination and giving to said employee a
reasonable opportunity to explain his side and (2) another written notice indicating that, upon due consideration
of all circumstances, grounds have been established to justify the employer's decision to dismiss the employee.

Petitioners were neither apprised of the charges against them nor given a chance to defend themselves.
They were simply and arbitrarily separated from work and served notices of termination in total disregard of their
rights to due process and security of tenure. The labor arbiter and the CA correctly found that respondents failed
to comply with the two-notice requirement for terminating employees.

San Beda College of Law 475


4S: 2015 - 2016
LABOR LAW REVIEW Atty. Joyrich Golangco

JENNY M. AGABON and VIRGILIO C. AGABON vs. NLRC, RIVIERA HOME IMPROVEMENTS, INC. and
VICENTE ANGELES
G.R. No. 158693 November 17, 2004
YNARES-SANTIAGO, J.:

FACTS: Private respondent Riviera Home Improvements, Inc. is engaged in the business of selling and
installing ornamental and construction materials. It employed petitioners Virgilio Agabon and Jenny Agabon as
gypsum board and cornice installers on January 2, 1992 until February 23, 1999 when they were dismissed for
abandonment of work.
Petitioners then filed a complaint for illegal dismissal and payment of money claims.

LA: declared the dismissals illegal and ordered private respondent to pay the monetary claims.

NLRC: Reversed the Labor Arbiter because it found that the petitioners had abandoned their work, and were not
entitled to backwages and separation pay. The other money claims awarded by the Labor Arbiter were also
denied for lack of evidence.

CA: Ruled that the dismissal of the petitioners was not illegal because they had abandoned their employment
but ordered the payment of money claims.

Hence, this petition for review on the sole issue of whether petitioners were illegally dismissed.

Petitioners: assert that they were dismissed because the private respondent refused to give them assignments
unless they agreed to work on a pakyaw basis. They did not agree on this arrangement because it would mean
losing benefits as Social Security System (SSS) members. Petitioners also claim that private respondent did not
comply with the twin requirements of notice and hearing.

Private respondents: Petitioners were not dismissed but abandoned their work. Petitioners did not report for
work because they had subcontracted to perform installation work for another company.

RULING: To dismiss an employee, the law requires not only the existence of a just and valid cause but also
enjoins the employer to give the employee the opportunity to be heard and to defend himself. Article 282 of the
Labor Code enumerates the just causes for termination by the employer.

Abandonment is the deliberate and unjustified refusal of an employee to resume his employment. It is a form of
neglect of duty, hence, a just cause for termination of employment by the employer. For a valid finding of
abandonment, these two factors should be present: (1) the failure to report for work or absence without valid or
justifiable reason; and (2) a clear intention to sever employer-employee relationship, with the second as the
more determinative factor which is manifested by overt acts from which it may be deduced that the employees
has no more intention to work. The intent to discontinue the employment must be shown by clear proof that it
was deliberate and unjustified.

In February 1999, petitioners were frequently absent having subcontracted for an installation work for another
company. Subcontracting for another company clearly showed the intention to sever the employer-employee
relationship with private respondent. This was not the first time they did this. Private respondent at that time
warned petitioners that they would be dismissed if this happened again. Petitioners disregarded the warning and
exhibited a clear intention to sever their employer-employee relationship. The record of an employee is a
relevant consideration in determining the penalty that should be meted out to him.

In Sandoval Shipyard v. Clave, we held that an employee who deliberately absented from work without leave or
permission from his employer, for the purpose of looking for a job elsewhere, is considered to have abandoned
his job. We should apply that rule with more reason here where petitioners were absent because they were
already working in another company.

The dismissal should be upheld because it was established that the petitioners abandoned their jobs to work for
another company. Private respondent, however, did not follow the notice requirements and instead argued that
sending notices to the last known addresses would have been useless because they did not reside there
anymore. Unfortunately for the private respondent, this is not a valid excuse because the law mandates the twin
notice requirements to the employees last known address. Thus, it should be held liable for non-compliance with
San Beda College of Law 476
4S: 2015 - 2016
LABOR LAW REVIEW Atty. Joyrich Golangco

the procedural requirements of due process.

Where the employer had a valid reason to dismiss an employee but did not follow the due process requirement,
the dismissal may be upheld but the employer will be penalized to pay an indemnity to the employee. This
became known as the Wenphil or Belated Due Process Rule. However, this doctrine was changed in the case of
Serrano. It was held that held that the violation by the employer of the notice requirement in termination for just
or authorized causes was not a denial of due process that will nullify the termination. However, the dismissal is
ineffectual and the employer must pay full backwages from the time of termination until it is judicially declared
that the dismissal was for a just or authorized cause. The fact that the Serrano ruling can cause unfairness and
injustice which elicited strong dissent has prompted us to revisit the doctrine. After carefully analyzing the
consequences of the divergent doctrines in the law on employment termination, we believe that in cases
involving dismissals for cause but without observance of the twin requirements of notice and hearing, the better
rule is to abandon the Serrano doctrine and to follow Wenphil by holding that the dismissal was for just cause
but imposing sanctions on the employer.

The indemnity to be imposed should be stiffer to discourage the abhorrent practice of dismiss now, pay later,
which we sought to deter in the Serrano ruling. The sanction should be in the nature of indemnification or
penalty and should depend on the facts of each case, taking into special consideration the gravity of the due
process violation of the employer. The violation of the petitioners right to statutory due process by the private
respondent warrants the payment of indemnity in the form of nominal damages. The amount of such damages is
addressed to the sound discretion of the court, taking into account the relevant circumstances. Considering the
prevailing circumstances in the case at bar, we deem it proper to fix it at P30,000.00.

San Beda College of Law 477


4S: 2015 - 2016
LABOR LAW REVIEW Atty. Joyrich Golangco

JAKA FOOD PROCESSING CORPORATION, petitioner, vs. DARWIN PACOT, ROBERT PAROHINOG,
DAVID BISNAR, MARLON DOMINGO, RHOEL LESCANO and JONATHAN CAGABCAB, respondents.
G.R. No. 151378. March 28, 2005
GARCIA, J.:

FACTS: Respondents were earlier hired by petitioner JAKA until the latter terminated their employment on
August 29, 1997 because the corporation was in dire financial straits. It is not disputed, however, that the
termination was effected without JAKA complying with the requirement under Article 283 of the Labor Code
regarding the service of a written notice upon the employees and the Department of Labor and Employment at
least one (1) month before the intended date of termination.

In time, respondents separately filed with the regional Arbitration Branch of the NLRC complaints for illegal
th
dismissal, underpayment of wages and nonpayment of service incentive leave and 13 month pay against JAKA
and its HRD Manager, Rosana Castelo.

LA: declared the termination illegal and ordered JAKA and its HRD Manager to reinstate respondents with full
backwages, and separation pay if reinstatement is not possible.

NLRC: affirmed in toto the decision of the NLRC. JAKA filed a motion for reconsideration. Acting thereon, the
NLRC came out with another decision, reversing and setting aside the awards of backwages, service incentive
leave pay. Each of the complainants-appellees shall be entitled to a separation pay equivalent to one month. In
addition, respondents-appellants is (sic) ordered to pay each of the complainants-appellees the sum of
P2,000.00 as indemnification for its failure to observe due process in effecting the retrenchment.

CA: Applied the doctrine in Serrano vs NLRC, reversing and setting aside the decision of the NLRC.

ISSUE: what are the legal implications of a situation where an employee is dismissed for cause but such
dismissal was effected without the employers compliance with the notice requirement under the Labor Code.

RULING: The difference between Agabon and the instant case is that in the former, the dismissal was based on
a just cause under Article 282 of the Labor Code while in the present case, respondents were dismissed due to
retrenchment, which is one of the authorized causes under Article 283 of the same Code.

A dismissal for just cause under Article 282 implies that the employee concerned has committed, or is guilty of,
some violation against the employer, i.e. the employee has committed some serious misconduct, is guilty of
some fraud against the employer, or, as in Agabon, he has neglected his duties. Thus, it can be said that the
employee himself initiated the dismissal process.

On another breath, a dismissal for an authorized cause under Article 283 does not necessarily imply
delinquency or culpability on the part of the employee. Instead, the dismissal process is initiated by the
employers exercise of his management prerogative, i.e. when the employer opts to install labor saving devices,
when he decides to cease business operations or when, as in this case, he undertakes to implement a
retrenchment program.

In the first, payment of separation pay, as a rule, is not required, while in the second, the law requires payment
of separation pay.

Accordingly, it is wise to hold that: (1) if the dismissal is based on a just cause under Article 282 but the
employer failed to comply with the notice requirement, the sanction to be imposed upon him should be tempered
because the dismissal process was, in effect, initiated by an act imputable to the employee; and (2) if the
dismissal is based on an authorized cause under Article 283 but the employer failed to comply with the notice
requirement, the sanction should be stiffer because the dismissal process was initiated by the employers
exercise of his management prerogative.

The records before us reveal that, indeed, JAKA was suffering from serious business losses at the time it
terminated respondents employment. It is, therefore, established that there was ground for respondents
dismissal, i.e., retrenchment, which is one of the authorized causes enumerated under Article 283 of the Labor
Code. Likewise, it is established that JAKA failed to comply with the notice requirement under the same Article.
Considering the factual circumstances in the instant case and the above ratiocination, we, therefore, deem it
San Beda College of Law 478
4S: 2015 - 2016
LABOR LAW REVIEW Atty. Joyrich Golangco

proper to fix the indemnity at P50,000.00.

We likewise find the Court of Appeals to have been in error when it ordered JAKA to pay respondents
separation pay equivalent to one (1) month salary for every year of service. This is because in Reahs
Corporation vs. NLRC, we made the following declaration: The rule, therefore, is that in all cases of business
closure or cessation of operation or undertaking of the employer, the affected employee is entitled to separation
pay. This is consistent with the state policy of treating labor as a primary social economic force, affording full
protection to its rights as well as its welfare. The exception is when the closure of business or cessation of
operations is due to serious business losses or financial reverses; duly proved, in which case, the right of
affected employees to separation pay is lost for obvious reasons.

San Beda College of Law 479


4S: 2015 - 2016
LABOR LAW REVIEW Atty. Joyrich Golangco

NELSON A. CULILI v. EASTERN TELECOMMUNICATIONS PHILIPPINES, INC., SALVADOR HIZON
(President and Chief Executive Officer), EMILIANO JURADO (Chairman of the Board), VIRGILIO GARCIA
(Vice President), and STELLA GARCIA (Assitant Vice President)
G.R. No. 165381 February 9, 2011
LEONARDO-DE CASTRO

DOCTRINE:
Our laws, while recognizing the right of employers to terminate employees it cannot sustain, also recognize the
employees right to be properly informed of the impending severance of his ties with the company he is working
for. Where the dismissal is due to a just or authorized cause, but without observance of the due process
requirements, the dismissal may be upheld but the employer must pay an indemnity to the employee.

FACTS:
Nelson Culili was employed by Eastern Telecommunications Philippines, Inc. (ETPI) as a Techinician in its Field
Operations on January 27, 1981. Culili was promoted to Senior Technician in the Customer Premises
Equipment Management Unit of the Service Quality Department and his basic salary was increased.

In 1998, due to business troubles and losses, ETPI was compelled to implement a Right-Sizing Program which
consisted of two phases: the first phase involved the reduction of ETPIs workforce to only those employees that
were necessary and which ETPI could sustain; the second phase entailed a company-wide reorganization which
would result in the transfer, merger, absorption, or abolition of certain departments of ETPI.

As part of the first phase, ETPI offered to its employees who had rndered at least 15 years of service, the
Special Retirement Program, which consisted of the option to voluntarily retire at an earlier age and a retirement
package equivalent to 2 months salary for every year of service. This offer was initially rejected by the
Eastern Telecommunications Employees Union (ETEU), ETPIs duly recognizes bargaining agent, which
threatened to stage a strike. However, after explanation to the ETEU, of the 102 employees who qualified for the
program, only Culili rejected the offer. ETPI then proceeded with the second phase which necessitated the
abolition, transfer, and merger of a number of ETPIs departments including the Service Quality Department
(Cusomer Premises Equipment Management Unit) to which Culili belongs, thereby rendering the specialized
functions of a Senior Technician unnecessary. Culilis name was omitted in ETPIs New Table of Organization
and was later advised of his termination of employment due to economic crisis.

Hence, this complaint for illegal dismissal where Culili alleged, among others that neither he nor the DOLE were
formally notified of his termination since he only found out when he was handed the termination letter and was
thereafter barred from entering ETPIs premises by its armed security personnel when he tried to report for
work.

LA: ETPI is guilty of illegal dismissal and unfair labor practice.

NLRC: LA decision was affirmed.

CA: Culilis position was validly abolished due to redundancy. Hence, there was no illegal dismissal nor unfair
labor practice. Hence, this petition for review on certiorari.

NOTE: The SC ruled that there was no illegal dismissal nor unfair labor practice.

ISSUE:
After finding that the dissmissal of Culili was for a lawful cause and not an act of unfair labor practice, was there
proper observance of procedural due process?

HELD:
No. ETPI was remiss in its duty to observe procedural due process in effecting the termination of Culili.

There are two aspects which characterize the concept of due process under the Labor Code: one is substantive
whether the termination of employment was based on the provision of the Labor Code or in accordance with the
prevailing jurisprudence; the other is procedural the manner in which the dismissal was effected.

Section 2(d), Rule I, Book VI of the Rules Implementing the Labor Code provides:
San Beda College of Law 480
4S: 2015 - 2016
LABOR LAW REVIEW Atty. Joyrich Golangco

(d) In all cases of termination of employment, the following standards of due process shall be substantially
observed:

For termination of employment as defined in Article 283 of the Labor Code, the requirement of due process shall
be deemed complied with upon service of a written notice to the employee and the appropriate Regional Office
of the Department of Labor and Employment at least thirty days before effectivity of the termination, specifying
the ground or grounds for termination.

ETPI does not deny its failure to provide DOLE with a written notice regarding Culilis termination. Our laws,
while recognizing the right of employers to terminate employees it cannot sustain, also recognize the employees
right to be properly informed of the impending severance of his ties with the company he is working for. In the
case at bar, ETPI, in effecting Culilis termination, simply asked one of its guards to serve the required written
notice on Culili. Culili, on one hand, claims in his petition that this was handed to him by ETPIs vice president,
but previously testified before the Labor Arbiter that this was left on his table. Regardless of how this notice was
served on Culili, ETPI failed to properly notify Culili about his termination. Aside from the manner the written
notice was served, a reading of that notice shows that ETPI failed to properly inform Culili of the grounds for his
termination.

Where the dismissal is due to a just or authorized cause, but without observance of the due process
requirements, the dismissal may be upheld but the employer must pay an indemnity to the employee.

If the dismissal is based on a just cause under Article 282 but the employer failed to comply with the notice
requirement, the sanction to be imposed upon him should be tempered because the dismissal process was, in
effect, initiated by an act imputable to the employee. If the dismissal is based on an authorized cause under
Article 283 but the employer failed to comply with the notice requirement, the sanction should be stiffer because
the dismissal process was initiated by the employer's exercise of his management prerogative.

Since it has been established that Culilis termination was due to an authorized cause and cannot be considered
unfair labor practice on the part of ETPI, his dismissal is valid. However, in view of ETPIs failure to comply with
the notice requirements under the Labor Code, Culili is entitled to nominal damages in addition to his separation
pay.

San Beda College of Law 481


4S: 2015 - 2016
LABOR LAW REVIEW Atty. Joyrich Golangco

ANTONIO M. SERRANO v. GALLANT MARITIME SERVICES, INC. and MARLOW NAVIGATION CO., INC.
G.R. No. 167614, March 24, 2009
AUSTRIA-MARTINEZ

FACTS:
Antonio Serrano, a Filipino seafarer, was hired by Gallant Maritime Services, Inc. and Marlow Navigation Co.,
Ltd. under a POEA-Approved Contract of Employment with the ff. terms and conditions:
Duration of contract 12 months
Position Chief Officer
Basic monthly salary US$1,400.00
Hours of work 48.0 hours per week
Overtime US$700 per month
Vacation leave with pay 7.00 days per month

On March 19, 1998, his departure date, Serrano was constrained to accept a downgraded employement
contract for Second Officer with a monthly salary of US$1,0000.00, upon the assurance and representation of
Gallant and Marlow that he would be made Chief Officer by the end of April 1998.

Gallant and Marlow did not deliver. Hence, Serrano refused to stay on as Second Officer and was repatriated to
the Philippines on May 26, 1998, serving only two months and seven days of his contract, leaving an unexpired
portion of nine months and 23 days.

Serrano filed a complaint for constructive dismissal and payment of his money claims amounting to
US$26,442.73, as well as moral and exemplary damages and attorneys fees.

LA: Dismissal was illegal. Serrano was awarded US$8,770.00 representing Serranos salary for 3 months of the
unexpired portion of the contract + US$45.00 salary differential + 10% atty.s fees. This was based on the salary
rate of US$2,590.00 consisting of: basic salary, US$1,400.00/month + US$700.00/month, fixed overtime pay, +
US$490.00/month, vacation leave pay = US$2,590.00/compensation per month.

NLRC: Serrano was only awarded a total of $4,669.50 reducing the applicable salary rate to US$1400 since RA
8042 does not provide for the award of OT pay, which should be proven to have been actually performed, and
for vacation leave pay.

CA: NLRC affirmed applicable salary rate. MR was denied. Hence, this petition for review where Serrano
assails, among others, the constitutionality of the clause or for three (3) months for every year of the unexpired
term, whichever is less under Sec. 10 of Ra 8042 alleging that it is violative of Sec. 3, Art. XIII of the
Constitution.

ISSUE:
Can Section 3 of Artt. XIII of the Constitution, on its own, be a source of a positive enforceable right?

HELD:
The constitutional mandates of protection to labor and security of tenure may be deemed as self-executing in
the sense that these are automatically acknowledged and observed without need for any enabling legislation.
However, to declare that the constitutional provisions are enough to guarantee the full exercise of the rights
embodied therein, and the realization of ideals therein expressed, would be impractical, if not unrealistic. The
espousal of such view presents the dangerous tendency of being overbroad and exaggerated. The guarantees
of "full protection to labor" and "security of tenure", when examined in isolation, are facially unqualified, and the
broadest interpretation possible suggests a blanket shield in favor of labor against any form of removal
regardless of circumstance. This interpretation implies an unimpeachable right to continued employment-a
utopian notion, doubtless-but still hardly within the contemplation of the framers. Subsequent legislation is still
needed to define the parameters of these guaranteed rights to ensure the protection and promotion, not only the
rights of the labor sector, but of the employers' as well. Without specific and pertinent legislation, judicial bodies
will be at a loss, formulating their own conclusion to approximate at least the aims of the Constitution.

Ultimately, therefore, Section 3 of Article XIII cannot, on its own, be a source of a positive enforceable right to
stave off the dismissal of an employee for just cause owing to the failure to serve proper notice or hearing. As

San Beda College of Law 482


4S: 2015 - 2016
LABOR LAW REVIEW Atty. Joyrich Golangco

manifested by several framers of the 1987 Constitution, the provisions on social justice require legislative
enactments for their enforceability.

Section 3, Article XIII cannot be treated as a principal source of direct enforceable rights, for the violation of
which the questioned clause may be declared unconstitutional. It may unwittingly risk opening the floodgates of
litigation to every worker or union over every conceivable violation of so broad a concept as social justice for
labor.

Section 3, Article XIII does not directly bestow on the working class any actual enforceable right, but merely
clothes it with the status of a sector for whom the Constitution urges protection through executive or legislative
action and judicial recognition. Its utility is best limited to being an impetus not just for the executive and
legislative departments, but for the judiciary as well, to protect the welfare of the working class.

When the challenge to a statute is premised on the perpetuation of prejudice against persons favored by the
Constitution with special protection -- such as the working class or a section thereof -- the Court may recognize
the existence of a suspect classification and subject the same to strict judicial scrutiny.

Article XIII, by itself, without the application of the equal protection clause, has no life or force of its own.

Nonetheless, the subject clause violates Serrano's right to substantive due process, for it deprives him of
property, consisting of monetary benefits, without any existing valid governmental purpose.

There is nothing in the text of the law or the records of the deliberations leading to its enactment or the
pleadings of respondent that would indicate that there is an existing governmental purpose for the subject
clause, or even just a pretext of one.

The subject clause does not state or imply any definitive governmental purpose; and it is for that precise reason
that the clause violates not just petitioner's right to equal protection, but also her right to substantive due process
under Section 1 of Article III of the Constitution.

The subject clause being unconstitutional, petitioner is entitled to his salaries for the entire unexpired period of
nine months and 23 days of his employment contract, pursuant to law and jurisprudence prior to the enactment
of R.A. No. 8042.

San Beda College of Law 483


4S: 2015 - 2016
LABOR LAW REVIEW Atty. Joyrich Golangco

CLAUDIO S. YAP v. THENAMARIS SHIPS MANAGEMENT AND INTERMARE MARITIME AGENCIES INC.
G.R. No. 179532 May 30, 2011
NACHURA, J.:

FACTS:
Claudio S. Yap (Petitioner) was employed as electrician of the vessel, M/T SEASCOUT on 14 August 2001 by
Intermare Maritime Agencies, Inc. The contract of employment was for a duration of 12 months which
commenced on August 23, 2001. On or about November 8, 2001, the vessel was sold. The crew was informed
in an Advisory sent by Capt. Constatinou that if they wish to be transferred to other vessels they should be
informed accordingly to schedule them for re-embarkation.

Yap received his seniority bonus, vacation bonus, extra bonus along with the scrapping bonus. He refused
however the payment of his wages.

Petitoners Contention:
In the complaint filed for illegal dismissal, Yap refused the payment of his wage, because he was entitled to the
payment of the unexpired portion of his contract since he was illegally dismissed from employment. He alleged
that he opted for immediate transfer but none was made.

Respondents Contention:

Yap was not illegally dismissed and he signed off from the vessel on November 10, 2001 and was paid his
wages corresponding to the months he worked or until 10 November 2001 plus his seniority bonus, vacation
bonus and extra bonus. Also, Yaps employment contract was validly terminated due to the sale of the vessel
and no arrangement was made for Yaps transfer to Thenamaris other vessels

LAs decision:
Favoured the petitioner. It ruled that there was constructive and illegal dismissal by respondents. Moreover, LA
opined that since the unexpired portion of petitioners contract was less than one year, petitioner was entitled to
his salaries for the unexpired portion of his contract for a period of nine months.

NLRCs Ruling:
NLRC affirmed the LAs findings, however, it held that instead of an award of salaries corresponding to nine
months, petitioner was only entitled to salaries for three months as provided under Section 10 of Republic Act
(R.A.) No. 8042. It later modified it ruling granting the payment of the wages for the unexpired portion of the
contract upon the instance of petitioner.
.
CAs Ruling:
It affirmed the findings and ruling of the LA and the NLRC that petitioner was constructively and illegally
dismissed. However, the CA ruled that the NLRC erred in sustaining the LAs interpretation of Section 10 of R.A.
No. 8042. In this regard, the CA relied on the clause or for three months for every year of the unexpired term,
th
whichever is less provided in the 5 paragraph of Section 10 of R.A. No. 8042 and held that petitioner was only
entitled to three months of basic salary.

ISSUES:

1. Whether or not Section 10 of R.A. 8042, to the extent that it affords an illegally dismissed
migrant worker the lesser benefit of salaries for the unexpired portion of his employment
contract or for three (3) months for every year of the unexpired term, whichever is less is
constitutional?

2. Whether or not the tanker allowance should not be included in the computation of the lump-sum salary
to be awarded to petitioner which Respondents only raised as issue during the hearing of the present
case?

HELD:
1. NO. Verily, we have already declared in Serrano that the clause or for three months for every year
th
of the unexpired term, whichever is less provided in the 5 paragraph of Section 10 of R.A. No.
San Beda College of Law 484
4S: 2015 - 2016
LABOR LAW REVIEW Atty. Joyrich Golangco

8042 is unconstitutional for being violative of the rights of Overseas Filipino Workers (OFWs) to
equal protection of the laws.

Moreover, this Court held therein that the subject clause does not state or imply any definitive
governmental purpose; hence, the same violates not just therein petitioners right to equal
protection, but also his right to substantive due process under Section 1, Article III of the
[28]
Constitution. Consequently, petitioner therein was accorded his salaries for the entire unexpired
period of nine months and 23 days of his employment contract, pursuant to law and jurisprudence
prior to the enactment of R.A. No. 8042.

We have already spoken. Thus, this case should not be different from Serrano.

2. NO. In the same vein, we cannot subscribe to respondents postulation that the tanker allowance of
US$130.00 should not be included in the computation of the lump-sum salary to be awarded to
petitioner.

First. It is only at this late stage, more particularly in their Memorandum, that respondents are
raising this issue. It was not raised before the LA, the NLRC, and the CA.

Second. Respondents invocation of Serrano is unavailing. Indeed, we made the following


pronouncements in Serrano, to wit:

The word salaries in Section 10(5) does not include overtime and leave pay. For
seafarers like petitioner, DOLE Department Order No. 33, series 1996, provides a
Standard Employment Contract of Seafarers, in which salary is understood as the basic
wage, exclusive of overtime, leave pay and other bonuses; whereas overtime pay is
compensation for all work performed in excess of the regular eight hours, and holiday
[32]
pay is compensation for any work performed on designated rest days and holidays.

A close perusal of the contract reveals that the tanker allowance of US$130.00 was not categorized as
a bonus but was rather encapsulated in the basic salary clause, hence, forming part of the basic salary
of petitioner. Respondents themselves in their petition for certiorari before the CA averred that
petitioners basic salary, pursuant to the contract, was US$1,300.00 + US$130.00 tanker allowance. If
respondents intended it differently, the contract per se should have indicated that said allowance does
not form part of the basic salary or, simply, the contract should have separated it from the basic salary
clause.

San Beda College of Law 485


4S: 2015 - 2016
LABOR LAW REVIEW Atty. Joyrich Golangco

BANK OF LUBAO V. ROMEL MANABAT
G.R. No. 188722 February 1, 2012
REYES, J.:

FACTS:
Rommel J. Manabat (Respondent) was hired by Bank of Lubao (Petitioner), a rural bank, as a Market Collector.
Subsequently, the respondent was assigned as an encoder at the Bank of Lubaos Sta. Cruz Extension Office,
which he manned together with two other employees, teller Susan P. Lingad (Lingad). As an encoder, the
respondents primary duty is to encode the clients deposits on the banks computer after the same are received
by Lingad.

The initial audit on the Bank of Lubaos Sta. Cruz Extension Office conducted revealed however that that there
was a misappropriation of funds in the amount of P3,000,000.00, more or less. After an administrative hearing
was conducted by the banks investigating committee, where the respondent was further made to explain his
side, the investigating committee concluded that the respondent conspired with Lingad in making fraudulent
entries disguised as error corrections in the banks computer.

Thereafter, citing serious misconduct tantamount to willful breach of trust as ground, it terminated the
respondents employment.

Respondents Contention:
In the complaint for illegal dismissal, the respondent, averred that the charge against him for qualified theft was
dismissed for lack of sufficient basis to conclude that he conspired with Lingad. The respondent sought an
th
award for separation pay, full backwages, 13 month pay for 2004 and moral and exemplary damages.

Petitioners Contention:
It insists that the dismissal of the respondent is justified, asserting that the Audit Report confirmed the
participation of the respondent in the alleged misappropriations. It further asserted that the dismissal of the
qualified theft charge against the respondent is immaterial to the validity of the ground for the latters dismissal.

LAs Decision:
Favoured the claim of illegal dismissal ordering the petitioner to reinstate the respondent to his former position
th
and awarding the latter backwages in the amount of P111,960.00 and 13 month pay in the amount
of P6,220.00. The LA opined that the petitioner failed to adduce substantial evidence that there was a valid
ground for the respondents dismissal. Further, the Audit Report that was adduced by the petitioner in evidence
was disregarded by the LA since it was unsigned.

NLRCs Decision:
Affirmed the Decision of the LA. The NLRC held that it was sufficiently established that only Lingad was the one
responsible for the said misappropriations. Further, the NLRC asserted that the February 14, 2006 and April 30,
2007 audit reports presented by the petitioner could not be given evidentiary weight as the same were executed
after the respondent had already been dismissed.

CAs Decision:
The CA agreed with the LA and the NLRC that the petitioner failed to establish by substantial evidence that
there was indeed a valid ground for the respondents dismissal. Nevertheless, the CA held that the petitioner
should pay the respondent separation pay since the latter did not pray for reinstatement before the LA and that
the same would be in the best interest of the parties considering the animosity and antagonism that exist
between them.

ISSUES:
1. Whether the CA erred in ordering the petitioner to pay the respondent separation pay in lieu of
reinstatement?
2. Whether the respondent is entitled to payment of backwages?

HELD:

San Beda College of Law 486


4S: 2015 - 2016
LABOR LAW REVIEW Atty. Joyrich Golangco

1. NO. Under the law and prevailing jurisprudence, an illegally dismissed employee is entitled to
reinstatement as a matter of right. However, if reinstatement would only exacerbate the tension and
strained relations between the parties, or where the relationship between the employer and the
employee has been unduly strained by reason of their irreconcilable differences, particularly where
the illegally dismissed employee held a managerial or key position in the company, it would be more
prudent to order payment of separation pay instead of reinstatement.

Under the doctrine of strained relations, the payment of separation pay is considered an acceptable
alternative to reinstatement when the latter option is no longer desirable or viable.

Here, we agree with the CA that the relations between the parties had been already strained
thereby justifying the grant of separation pay in lieu of reinstatement in favor of the respondent.

2. YES. However, the backwages that should be awarded to the respondent should be
modified. Employees who are illegally dismissed are entitled to full backwages, inclusive of
allowances and other benefits or their monetary equivalent, computed from the time their actual
compensation was withheld from them up to the time of their actual reinstatement. But if
reinstatement is no longer possible, the backwages shall be computed from the time of their illegal
termination up to the finality of the decision.

Thus, when there is an order of reinstatement, the computation of backwages shall be reckoned
from the time of illegal dismissal up to the time that the employee is actually reinstated to his former
position.

San Beda College of Law 487


4S: 2015 - 2016
LABOR LAW REVIEW Atty. Joyrich Golangco

ST. MARYS ACADEMY OF DIPOLOG CITY vs. TERESITA PALACIO, MARIGEN CALIBOD, LEVIE LAQUIO,
ELAINE MARIE SANTANDER, ELIZA SAILE, and MA. DOLORES MONTEDERAMOS
G.R. No. 164913 September 8, 2010
DEL CASTILLO, J.:

POST-EMPLOYMENT; SECURITY OF TENURE

DOCTRINE:
The Court will not hesitate to defend the workers constitutional right to security of tenure. After all, the interest of
the workers is paramount as they are regarded with compassion under the policy of social justice.

FACTS:
On different dates in the late 1990s, petitioner hired respondents as classroom teachers and guidance
counselor. In separate letters dated March 31, 2000, however, petitioner informed them that their re-application
for school year 2000-2001 could not be accepted because they failed to pass the LET. According to petitioner,
as non-board passers, respondents could not continue practicing their teaching profession pursuant to the
DECS Memorandum No. 10, S. 1998 which requires incumbent teachers to register as professional teachers
pursuant to Sec. 27 of R.A. No. 7836, otherwise known as the Philippine Teachers Professionalization Act of
1994.
Respondents filed a complaint contesting their termination as highly irregular and premature. They argued that
their security of tenure could not simply be trampled upon for their failure to register with the PRC or to pass the
LET prior to the deadline (September 19, 2009) set by RA 7836.

LA RULING:
The LA adjudged petitioner guilty of illegal dismissal because it terminated the services of the respondents on
March 31, 2000 which was clearly prior to the September 19, 2000 deadline fixed by PRC for the registration of
teachers as professional teachers, in violation of the doctrine regarding the prospective application of laws.
Thus, petitioner was ordered to reinstate the respondents or to pay them separation pay at the rate of month
wage for every year of service, plus limited backwages covering the period from March 31, 2000 to September
30, 2000.

NLRC RULING:
The NLRC affirmed the decision of the LA.

CA RULING:
The CA affirmed the decision of the LA and the NLRC.

ISSUES: (1) Is the dismissal not valid even if the same was made in order to protect the interest of the
Employer (the petitioner justifies respondents termination by advancing that it would be difficult to hire licensed
teachers in the middle of the school year as respondents replacements)?
(2) Is the amount of separation pay awarded to respondents correct? (The petitioner contends that assuming
respondents were illegally dismissed, they are only entitled to an amount computed from the time of dismissal
up to September 19, 2000 only. After September 19, 2000, respondents, according to petitioner, are already
dismissible for cause for lack of the necessary license to teach)

SC RULING:
(1) YES, the dismissal is not valid. It is incumbent upon this Court to afford full protection to labor. Thus, while
we take cognizance of the employers right to protect its interest, the same should be exercised in a manner
which does not infringe on the workers right to security of tenure. Under the policy of social justice, the law
bends over backward to accommodate the interests of the working class on the humane justification that those
with less privilege in life should have more in law. To reiterate, this Court will not hesitate to defend respondents
right to security of tenure. The premature dismissal from the service of respondents is unwarranted.

(2) Yes, the amount of separation pay awarded to respondents is correct. Petitioner cannot possibly
presume that respondents could not timely comply with the requirements of the law. At any rate, we note that
petitioner only assailed the amount of backwages for the first time in its motion for reconsideration of the
Decision of the CA. Thus, the Court cannot entertain the issue for being belatedly raised. Hence, the award of
limited backwages covering the period from March 31, 2000 to September 30, 2000 as ruled by the Labor
Arbiter and affirmed by both the NLRC and CA is in order.
San Beda College of Law 488
4S: 2015 - 2016
LABOR LAW REVIEW Atty. Joyrich Golangco

TOYOTA MOTOR PHILS. CORP. WORKERS ASSOCIATION (TMPCWA) vs. NATIONAL LABOR
RELATIONS COMMISSION
TOYOTA MOTOR PHILIPPINES CORPORATION vs. TOYOTA MOTOR PHILIPPINES CORP. WORKERS
ASSOCIATION (TMPCWA)
G.R. Nos. 158786 &158789 October 19, 2007
DEL CASTILLO, J.:

POST-EMPLOYMENT; SECURITY OF TENURE

DOCTRINE:
Separation should not be awarded to the Union members who participated in the illegal strikes.

GR: When just causes for terminating the services of an employee under Art. 282 of the Labor Code exist, the
employee is not entitled to separation pay.
EXCEPTION: When the court finds justification in applying the principle of social justice well entrenched in
the 1987 Constitution (the cause of the dismissal is other than serious misconduct or that which
reflects adversely on the employees moral character)

FACTS:
The case started when the Toyota refused to negotiate CBAs with the Union pending an appeal filed in DOLE
where the former questioned the order certifying the Union as the sole and exclusive bargaining agent of the
Toyota rank and file employees. The Unions subsequent notice to strike was converted into a preventive
mediation case. The Union requested that its members be absent to attend in the hearing in the said mediation
case, but the same was denied. Despite said denial, more than 200 employees staged mass actions on 22 and
23 February in front of the BLR and DOLE offices, to protest the partisan and anti-union stance of Toyota. Due
to the loss of the said number of employees, Toyota experienced losses due to inability to meet production
goals. Soon thereafter, Toyota sent individual letters to some 360 employees requiring them to explain within
24 hours why they should not be dismissed for their obstinate defiance of the companys directives. The letters
specifically cited the Companys Code of Conduct wherein inciting or participating in riots, disorders, alleged
strikes, or concerted actions detrimental to Toyotas interest wherein the first offense would amount to
dismissal.

The Union members explained that their refusal to work on their scheduled work time for two consecutive days
was simply an exercise of their constitutional right to peaceably assemble and to petition the government for
redress of grievances. On 16 March 2001, Toyota terminated 227 employees for participation in concerted
actions in violation of its Code of Conduct and for misconduct under Article 282 of the Labor Code. In reaction
to the dismissal of its union members and officers, the Union went on strike where the Union intensified its strike
by barricading the gates of Toyotas Bicutan and Sta. Rosa plants.

NLRC RULING:
The NLRC declared the strikes staged by the Union as illegal. However, the Company is ordered to pay the 227
Union members, who participated in the illegal strike severance compensation in an amount equivalent to one
month salary for every year of service, as an alternative relief to continued employment.

CA RULING:
The CA affirmed the decision of the NLRC with a modification, however, of deleting the award of severance
compensation to the dismissed Union members. In justifying the recall of the severance compensation, the CA
considered the participation in illegal strikes as serious misconduct. It defined serious misconduct as a
transgression of some established and definite rule of action, a forbidden act, a dereliction of duty, willful in
character, and implies wrongful intent and not mere error in judgment.

However, in its Resolution, the CA modified its Decision by reinstating severance compensation to the
dismissed employees based on social justice.

ISSUE: Should separation pay be awarded to the Union members who participated in the illegal strikes?

SC RULING:
NO, separation should not be awarded to the Union members who participated in the illegal strikes.

San Beda College of Law 489


4S: 2015 - 2016
LABOR LAW REVIEW Atty. Joyrich Golangco

The general rule is that when just causes for terminating the services of an employee under Art. 282 of the
Labor Code exist, the employee is not entitled to separation pay. The apparent reason behind the forfeiture of
the right to termination pay is that lawbreakers should not benefit from their illegal acts. The dismissed
employee, however, is entitled to whatever rights, benefits and privileges [s/he] may have under the applicable
individual or collective bargaining agreement with the employer or voluntary employer policy or practice or under
the Labor Code and other existing laws. This means that the employee, despite the dismissal for a valid cause,
retains the right to receive from the employer benefits provided by law, like accrued service incentive leaves.
With respect to benefits granted by the CBA provisions and voluntary management policy or practice, the
entitlement of the dismissed employees to the benefits depends on the stipulations of the CBA or the company
rules and policies.

One exception where separation pay is given even though an employee is validly dismissed is when the court
finds justification in applying the principle of social justice well entrenched in the 1987 Constitution. Severance
compensation shall be allowed only when the cause of the dismissal is other than serious misconduct or that
which reflects adversely on the employees moral character.

There can be no good faith in intentionally incurring absences in a collective fashion from work just to attend
DOLE hearings. The Union members should know from common sense that the company will incur substantial
amounts of losses. In a slew of cases, the Court refrained from awarding separation pay or financial assistance
to union officers and members who were separated from service due to their participation in or commission of
illegal acts during strikes Therefore, the award of separation pay to the Union officials and members in the
instant petitions cannot be sustained.

San Beda College of Law 490


4S: 2015 - 2016
LABOR LAW REVIEW Atty. Joyrich Golangco

BRISTOL MYERS SQUIBB (PHILS.), INC. vs. RICHARD NIXON A. BABAN
G.R. No. 167449 December 17, 2008
REYES, R.T., J.:

DOCTRINES:
There are two requisites for a valid dismissal by the employer on the ground of breach of trust
and confidence under Article 297(c)(new numbering); namely:
1) The employee concerned must be one holding a position of trust and confidence.
Two classes:
(a) MANAGERIAL EMPLOYEES
They are defined as those vested with the powers or prerogatives to lay down
management policies and to hire, transfer suspend, lay-off, recall, discharge,
assign or discipline employees or effectively recommend such managerial
actions.
(b) CASHIERS, AUDITORS, PROPERTY CUSTODIANS, ETC
They are defined as those who in the normal and routine exercise of their
functions, regularly handle significant amounts of money or property.
2) There must be an act that would justify the loss of trust and confidence.
Loss of trust and confidence to be a valid cause for dismissal must be based on a willful breach
of trust and founded on clearly established facts. The basis for the dismissal must be clearly
and convincingly established but proof beyond reasonable doubt is not necessary.
FACTS:
In 1992, BMSI hired Baban as district manager of the company. Being a manager and a confidential
employee, he was assigned to handle the company's clients in Cagayan de Oro-Northern Mindanao area and its
immediate vicinities and to supervise territory managers detailed in his district. He is also tasked to promote the
nutritional products of BMSI to medical practitioners and to sell products the same to drug outlets.
Sometime in 1998, an auditor of BMSI found twenty (20) packs of "Mamacare" samples in the baggage
compartment of a company car with an accompanying note with political overtones. Apparently, Baban stapled
the political "thank you" note with the intention of distributing them to his father's supporters during the 1998
election.
In the course of the investigation, Baban admitted that he had caused the attachment of the notes to
the product samples. He argued that there was no unauthorized distribution of the samples since he intended to
give them only to doctors who requested them. He then admitted that he had committed a mistake and pleaded
for consideration for the lapse, insisting that he has not caused any damage nor injury to the image of the
company as the samples were not, in fact, distributed and that no gain was derived by him or his family.
Unconvinced with his explanation, Baban was dismissed, hence this case for illegal dismissal.
LA RULING: BABANS TERMINATION is VALID
Baban has violated company rules and regulations by his unauthorized use of its property. BMSI is
therefore justified to declare respondent unworthy of the trust and confidence formerly imposed in him.
BMSI was ordered to pay Baban P297,009.84 representing the admitted monetary liabilities.
NLRC RULING: AFFIRMED the LA
The NLRC further awarded financial assistance in favor of Baban by way of separation pay equivalent
to one (1) month pay for every year of service covering the period from the date of his regular employment up to
25 August 1998, a fraction of six (6) months being considered one (1) year
CA RULING: BABAN was ILLEGALLY DISMISSED
The CA apparently granted his plea for mercy when it ruled that his action while censurable did not
merit termination. The CA characterized his action as a mere lapse of human frailty considering the elections
were over. Moreover, the stapling of the thank you notes did not give rise to any undue advantage to respondent
or his father.
ISSUE: Was Baban validly dismissed on the ground of loss of trust and confidence?

SC RULING:
YES. The aforementioned requisites are present in this case.
st
1 Requisite: The employee concerned must be one holding a position of trust and confidence
In this case, Baban was employed as district manager for Cagayan de Oro-North Mindanao and its
immediate vicinities. It is not the job title but the actual work that the employee performs. He was employed to
handle products for distribution to medical practitioners and sale to drug outlets. As a result of his handling of
large amounts of petitioner's samples, respondent is, by law, an employee with a position of trust, falling under
San Beda College of Law 491
4S: 2015 - 2016
LABOR LAW REVIEW Atty. Joyrich Golangco

the second class.
nd
2 Requisite: There must be an act that would justify the loss of trust and confidence.
There is no doubt that Baban willfully breached the trust and confidence reposed in him by not asking
for permission before using company property for his own or another's benefit, as required in the Company
Standards of Business Conduct. Moreover, when Baban failed to turn over the samples left in his care and
stapled the political "thank you" note with the intention of distributing them to his father's supporters, he had, in
effect appropriated company property for personal gain and benefit.
However, the SC ruled that since Baban was validly dismissed for a cause other than serious
misconduct or those that negatively reflect on his moral character, the award of separation pay as an equitable
relief is justifiable. This award is merely to coat the bitter termination experienced by respondent with a little
social justice. Separation pay at the rate of one month salary for every year of service is proper.

San Beda College of Law 492


4S: 2015 - 2016
LABOR LAW REVIEW Atty. Joyrich Golangco

ARMANDO G. YRASUEGUI v. PHILIPPINE AIRLINES, INC
G.R. No. 168081 October 17, 2008
REYES, R.T., J.:

DOCTRINES:
Requisites for Bona Fide Occupational Qualification (BFOQ):
(1) the employment qualification is reasonably related to the essential operation of the job
involved; and
(2) that there is factual basis for believing that all or substantially all persons meeting the
qualification would be unable to properly perform the duties of the job.

The prescribed weight of PAL is a continuing BFOQ. On board an aircraft, the body weight and
size of a cabin attendant are important factors to consider in case of emergency. Aircrafts have
constricted cabin space, and narrow aisles and exit doors. It would d be absurd to require airline
companies to reconfigure the aircraft in order to widen the aisles and exit doors just to accommodate
overweight cabin attendants like ARMANDO. Thus, the failure to maintain such weight is a valid ground
for dismissal under the analogous cases of Article 297(e) of the Labor Code (new numbering).

As a rule, a legally dismissed employee is not entitled to separation pay. This may be deduced
from the language of Article 294 of the Labor Code (new numbering) that "[a]n employee who is unjustly
dismissed from work shall be entitled to x x x. Exceptionally, it is granted to a legally dismissed
employee as an act "social justice," or based on "equity." In both instances, it is required that the
dismissal (1) was not for serious misconduct; and (2) does not reflect on the moral character of the
employee.

FACTS:
Armando was a former international flight steward of PAL. He stands five feet and eight inches (58")
with a large body frame. The proper weight for a man of his height and body structure is from 147 to 166
pounds, the ideal weight being 166 pounds, as mandated by the Cabin and Crew Administration Manual of PAL.

In 1984, PAL gave Armando an extended vacation leave in order for him to lose weight. After meeting
the required weight, Armando was allowed to return to work. But Armandos weight problem recurred. He was
40 pounds beyond his ideal weight and his weight kept on increasing in the course of his leave without pay from
October 17, 1988 to February 1989. For a span of 5 years, PAL has been giving Armando vacation leave
without pay to give him time to get back in shape. PAL even scheduled several weight checks to monitor
Armandos weight. Armando, on the other hand, would always reschedule and would only report for a couple
of times. On the last 3 weight checks that he had attended, Armando still remained overweight.

On November 13, 1992, PAL finally served Armando a Notice of Administrative Charge for violation of
company standards on weight requirements. In his Answer, Armando did not deny being overweight but
claimed that his violation, if any, had already been condoned by PAL since "no action has been taken by the
company" regarding his case "since 1988. He also claimed that PAL discriminated against him because "the
company has not been fair in treating the cabin crew members who are similarly situated." During the
clarificatory hearing on December 1992, Armando manifested that he was undergoing a weight reduction
program to lose at least two (2) pounds per week so as to attain his ideal weight. Six months thereafter,
Armando was formally informed by PAL that due to his inability to attain his ideal weight, "and considering the
utmost leniency" extended to him "which spanned a period covering a total of almost five (5) years," his services
were considered terminated "effective immediately."

Armando then filed a complaint for illegal dismissal.

LA RULING: Armando was ILLEGALLY DISMISSED


The LA held that the weight standards of PAL are reasonable in view of the nature of the job of
Armando. However, the weight standards need not be complied with under pain of dismissal since his weight
did not hamper the performance of his duties. Assuming that it did, Armando could be transferred to other
positions where his weight would not be a negative factor. Notably, other overweight employees, i.e., Mr.
Palacios, Mr. Cui, and Mr. Barrios, were promoted instead of being disciplined.

PAL and Armando appealed to the NLRC.


San Beda College of Law 493
4S: 2015 - 2016
LABOR LAW REVIEW Atty. Joyrich Golangco

NLRC RULING: AFFIRMED the decision of the LA


According to the NLRC, "obesity, or the tendency to gain weight uncontrollably regardless of the amount
of food intake, is a disease in itself." As a consequence, there can be no intentional defiance or serious
misconduct by petitioner to the lawful order of PAL for him to lose weight.

CA RULING: Armando was VALIDLY DISMISSED


The CA found Armandos termination to be valid under Article 297(e) of the Labor Code (new
numbering), which covers the analogous cases. Thus, Armando was legally dismissed because he repeatedly
failed to meet the prescribed weight standards. It is obvious that the issue of discrimination was only invoked by
petitioner for purposes of escaping the result of his dismissal for being overweight.

Armando appealed to the SC. He claims that obesity is a physical abnormality and/or illness and that
as a disease, he has no control over it. Thus, his dismissal cannot fall under the analogous cases.

ISSUE: Was Armandos dismissal valid?

SC RULING:
YES. The obesity of Armando is a ground for dismissal under Article 297(e) of the Labor Code (new
numbering) since the weight standards of PAL constitute a continuing qualification of an employee in order to
keep the job.

The SC held that the obesity of Armando, when placed in the context of his work as flight attendant,
becomes an analogous cause under Article 297(e) of the Labor Code that justifies his dismissal from the
service. His obesity may not be unintended, but is nonetheless voluntary. As the CA correctly puts it,
"voluntariness basically means that the just cause is solely attributable to the employee without any external
force influencing or controlling his actions. This element runs through all just causes under Article 297, whether
they be in the nature of a wrongful action or omission. Gross and habitual neglect, a recognized just cause, is
considered voluntary although it lacks the element of intent found in Article 297(a), (c), and (d)."

By way of equity, the SC granted separation pay in favor of Armando equivalent to one-half (1/2)
months pay for every year of service. It includes regular allowances, which he might have been receiving.

San Beda College of Law 494


4S: 2015 - 2016
LABOR LAW REVIEW Atty. Joyrich Golangco

DREAMLAND HOTEL RESORT and WESTLEY J. PRENTICE vs. STEPHEN B. JOHNSON
G.R. No. 191455 March 12, 2014
REYES, J.:

SEPARATION PAY

DOCTRINE:
Under the doctrine of strained relations, the payment of separation pay is considered an acceptable
alternative to reinstatement when the latter option is no longer desirable or viable. On one hand, such payment
liberates the employee from what could be a highly oppressive work environment. On the other hand, it releases
the employer from the grossly unpalatable obligation of maintaining in its employ a worker it could no longer
trust.

FACTS: Prentice and Johnson entered into an Employment Agreement, which stipulates among others, that
Johnson shall serve as Operations Manager of Dreamland from August 1, 2007 and shall serve as such for a
period of three (3) years. From the start of August 2007, Johnson already reported for work. However, he was
instructed to supervise construction and speak with potential guests. He also undertook the overall preparation
of the guestrooms and staff for the opening of the hotel, even performing menial tasks. As he was remained
unpaid since August 2007 and he has loaned all his money to petitioners, he asked for his salary after the resort
was opened in October 2007 but the same was not given to him by petitioners. He became very alarmed with
the situation as it appears that there was no intention to pay him his salary, which he now depended on for his
living as he has been left penniless. He was also denied the benefits promised him as part of his compensation
such as service vehicles, meals and insurance. He was also not given the authority due to him as resort
manager. Prentice countermanded his orders to the staff at every opportunity. Worse, he would even be berated
and embarrassed in front of the staff. After another embarrassment was handed out by Prentice in front of the
staff, which highlighted his lack of real authority in the hotel and the disdain for him by petitioners, respondent
Johnson was forced to submit his resignation. In deference to the Employment Agreement signed, Johnson
stated that he was willing to continue work for the three month period stipulated therein. However, in an SMS or
text message sent by Prentice to him on the same day at around 8:20 pm, he was informed that his resignation
is considered immediate. Despite demand, petitioners refused to pay Johnson the salaries and benefits due
him. Johnson filed a Complaint for illegal dismissal and non-payment of salaries, separation pay, among others,
against the petitioners.

LA RULING: Dismissed Johnsons complaint for lack of merit with the finding that he voluntarily resigned from
his employment and was not illegally dismissed.

NLRC RULING: Reversed LA's ruling.

ISSUE: Is Johnson entitled to his claims of unpaid salaries and separation pay?

SC RULING:
YES. The petitioners contend that the employment of Johnson as operations manager commenced only on
October 8, 2007 and not on August 1, 2007. However, the employment contract categorically stated that the
"term of employment shall commence on August 1, 2007." Furthermore, the factual allegations of Johnson that
he actually worked from August 1, 2007 were neither sufficiently rebutted nor denied by the petitioners. For the
petitioners failure to disprove that Johnson started working on August 1, 2007, as stated on the employment
contract, payment of his salaries on said date, even prior to the opening of the hotel is warranted.

Johnson has adduced proof that as a permanent resident, he is exempted from the requirement of securing an
AEP as expressed under Department Order No. 75-06, Series of 2006 of the Department of Labor and
Employment (DOLE). Furthermore, Johnson submitted a Certification from DOLE Regional Office III, stating that
he is exempted from securing an AEP as a holder of Permanent Resident Visa. Consequently, the condition
imposed upon Johnsons employment, if there is any, is in truth without effect to its validity. Anent the
requirement of securing a TIN to make the contract of employment efficacious, records show that Johnson
secured his TIN only on December 2007 after his resignation as operations manager. Nevertheless, this does
not negate the fact that the contract of employment had already become effective even prior to such date. In
addition to the foregoing, there is no stipulation in the employment contract itself that the same shall only be
effective upon the submission of AEP and TIN.

San Beda College of Law 495


4S: 2015 - 2016
LABOR LAW REVIEW Atty. Joyrich Golangco

As regards the NLRC findings that Johnson was constructively dismissed and did not abandon his work, the
Court agrees. Even the most reasonable employee would consider quitting his job after working for three
months and receiving only an insignificant fraction of his salaries. There was, therefore, not an abandonment of
employment nor a resignation in the real sense, but a constructive dismissal, which is defined as an involuntary
resignation resorted to when continued employment is rendered impossible, unreasonable or unlikely.

There is constructive dismissal if an act of clear discrimination, insensibility, or disdain by an employer


becomes so unbearable on the part of the employee that it would foreclose any choice by him except to forego
his continued employment. It exists where there is cessation of work because continued employment is
rendered impossible, unreasonable or unlikely, as an offer involving a demotion in rank and a diminution in pay.
It is impossible, unreasonable or unlikely that any employee, such as Johnson would continue working for an
employer who does not pay him his salaries.

Since Johnson was constructively dismissed, he was illegally dismissed. Thus, an illegally dismissed employee
is entitled to two reliefs: backwages and reinstatement. The two reliefs provided are separate and distinct. The
normal consequences of respondents illegal dismissal, then, are reinstatement without loss of seniority rights,
and payment of backwages computed from the time compensation was withheld up to the date of actual
reinstatement. Where reinstatement is no longer viable as an option, separation pay equivalent to one (1) month
salary for every year of service should be awarded as an alternative. The payment of separation pay is in
addition to payment of backwages.

Under the doctrine of strained relations, the payment of separation pay is considered an acceptable
alternative to reinstatement when the latter option is no longer desirable or viable. On one hand, such payment
liberates the employee from what could be a highly oppressive work environment. On the other hand, it releases
the employer from the grossly unpalatable obligation of maintaining in its employ a worker it could no longer
trust.

Employment contract states that Johnson's employment is for a term of three years. Accordingly, the award of
backwages should be computed from November 3, 2007 to August 1, 2010 - which is three years from August
1, 2007. Furthermore, separation pay is computed from the commencement of employment up to the time of
termination, including the imputed service for which the employee is entitled to backwages. As one-month salary
is awarded as separation pay for every year of service, including imputed service, Johnson should be paid
separation pay equivalent to his three-month salary for the three-year contract.

Dreamland Hotel Resort and Westley Prentice are ORDERED to PAY Stephen Johnson backwages of
P60,000.00 per month which should be computed from November 3, 2007 to August 1, 2010 less the
P.7,200.00 already paid to him. Likewise, separation pay of P180.000.00, representing Stephen Johnson's
three-year contract should be awarded.

San Beda College of Law 496


4S: 2015 - 2016
LABOR LAW REVIEW Atty. Joyrich Golangco

MANILA WATER COMPANY vs. CARLITO DEL ROSARIO
G.R. No. 188747 January 29, 2014
PEREZ, J.:

SEPARATION PAY

DOCTRINE:
In exceptional cases, however, the Court has granted separation pay to a legally dismissed employee as an act
of "social justice" or on "equitable grounds." In both instances, it is required that the dismissal (1) was not for
serious misconduct; and (2) did not reflect on the moral character of the employee.

FACTS: On 22 October 1979, Del Rosario was employed as Instrument Technician by Metropolitan Waterworks
and Sewerage System (MWSS). Sometime in 1996, MWSS was reorganized pursuant to Republic Act No. 8041
or the National Water Crisis Act of 1995, and its implementing guidelines Executive Order No. 286. Because
of the reorganization, Manila Water absorbed some employees of MWSS including Del Rosario. On 1 August
1997, Del Rosario officially became an employee of Manila Water.

Sometime in May 2000, Manila Water discovered that 24 water meters were missing in its stockroom. Upon
investigation, it appeared that Del Rosario and his co-employee, Danilo Manguera, were involved in the
pilferage and the sale of water meters to the companys contractor. Consequently, Manila Water issued a
Memorandum, directing Del Rosario to explain in writing within 72 hours why he should not be dealt with
administratively for the loss of the said water meters. In his letter-explanation, Del Rosario confessed his
involvement in the act charged and pleaded for forgiveness, promising not to commit similar acts in the future.

On 29 June 2000, Manila Water conducted a hearing to afford Del Rosario the opportunity to personally defend
himself and to explain and clarify his defenses to the charge against him. During the formal investigation Del
Rosario was found responsible for the loss of the water meters and therefore liable for violating Section 11.1 of
the Companys Code of Conduct. Manila Water proceeded to dismiss Del Rosario from employment on 3 July
2000.

This prompted Del Rosario to file an action for illegal dismissal claiming that his severance from employment is
without just cause. He averred that his admission to the misconduct charged was not voluntary but was coerced
by the company. Such admission therefore, made without the assistance of a counsel, could not be made basis
in terminating his employment.

Invoking Section 11.1 of the Companys Code of Conduct, Manila Water averred that such act of stealing the
companys property is punishable by dismissal. The company invited the attention of this Court to the fact that
Del Rosario himself confessed his involvement to the loss of the water meters not only in his letter-explanation,
but also during the formal investigation, and in both instances, pleaded for his employers forgiveness.

LA RULING: Dismissed for lack of merit the complaint filed by Del Rosario who was, however, awarded
separation pay. According to the Labor Arbiter, Del Rosarios length of service for 21 years, without previous
derogatory record, warrants the award of separation pay.

NLRC RULING: It dismissed the appeal interposed by Manila Water for its failure to append a certification
against forum shopping in its Memorandum of Appeal. MR was also denied.

CA RULING: Reversed the NLRC Resolution and held that it committed a grave abuse of discretion when it
dismissed Manila Waters appeal on mere technicality. The appellate court, however, proceeded to affirm the
decision of the Labor Arbiter awarding separation pay to Del Rosario.

ISSUE: Is respondent Del Rosario entitled to separation pay contrary to Book VI, Rule 1, and Section 7 of the
Omnibus IRR of the Labor Code and prevailing jurisprudence that an employee separated from service due to
serious misconduct is not entitled to separation pay?

SC RULING:
NO. As a general rule, an employee who has been dismissed for any of the just causes enumerated under
Article 282 of the Labor Code is not entitled to a separation pay. Section 7, Rule I, Book VI of the Omnibus
Rules implementing the Labor Code provides:
San Beda College of Law 497
4S: 2015 - 2016
LABOR LAW REVIEW Atty. Joyrich Golangco

Sec. 7. Termination of employment by employer. The just causes for terminating the services of
an employee shall be those provided in Article 282 of the Code. The separation from work of an
employee for a just cause does not entitle him to the termination pay provided in the Code, without
prejudice, however, to whatever rights, benefits and privileges he may have under the applicable
individual or collective agreement with the employer or voluntary employer policy or practice.

In exceptional cases, however, the Court has granted separation pay to a legally dismissed employee as an act
of "social justice" or on "equitable grounds." In both instances, it is required that the dismissal (1) was not for
serious misconduct; and (2) did not reflect on the moral character of the employee. Where the reason for the
valid dismissal is, for example, habitual intoxication or an offense involving moral turpitude, like theft or illicit
sexual relations with a fellow worker, the employer may not be required to give the dismissed employee
separation pay, or financial assistance, or whatever other name it is called, on the ground of social justice.

A contrary rule would, as the petitioner correctly argues, have the effect, of rewarding rather than punishing the
erring employee for his offense. And we do not agree that the punishment is his dismissal only and that the
separation pay has nothing to do with the wrong he has committed. Of course it has. Indeed, if the employee
who steals from the company is granted separation pay even as he is validly dismissed, it is not unlikely that he
will commit a similar offense in his next employment because he thinks he can expect a like leniency if he is
again found out. This kind of misplaced compassion is not going to do labor in general any good as it will
encourage the infiltration of its ranks by those who do not deserve the protection and concern of the
Constitution.

The policy of social justice is not intended to countenance wrongdoing simply because it is committed by the
underprivileged. At best, it may mitigate the penalty but it certainly will not condone the offense. Compassion for
the poor is an imperative of every humane society but only when the recipient is not a rascal claiming an
undeserved privilege. Social justice cannot be permitted to be refuge of scoundrels any more than can equity be
an impediment to the punishment of the guilty. Those who invoke social justice may do so only if their hands are
clean and their motives blameless and not simply because they happen to be poor. This great policy of our
Constitution is not meant for the protection of those who have proved they are not worthy of it, like the workers
who have tainted the cause of labor with the blemishes of their own character.

That Del Rosario rendered 21 years of service to the company will not save the day for hm. To this case,
Central Pangasinan Electric Cooperative, Inc. v. NLRC is on all fours, thus:

Although long years of service might generally be considered for the award of separation benefits
or some form of financial assistance to mitigate the effects of termination, this case is not the
appropriate instance for generosity under the Labor Code nor under our prior decisions. The fact
that private respondent served petitioner for more than twenty years with no negative record prior
to his dismissal, in our view of this case, does not call for such award of benefits, since his violation
reflects a regrettable lack of loyalty and worse, betrayal of the company. If an employee's length of
service is to be regarded as a justification for moderating the penalty of dismissal, such gesture will
actually become a prize for disloyalty, distorting the meaning of social justice and undermining the
efforts of labor to cleanse its ranks of undesirables.

San Beda College of Law 498


4S: 2015 - 2016
LABOR LAW REVIEW Atty. Joyrich Golangco

DARIO NACAR vs. GALLERY FRAMES AND/OR FELIPE BORDEY, JR.
G.R. No. 189871 August 13, 2013
PERALTA, J.:

DOCTRINE:
In the absence of an express stipulation as to the rate of interest that would govern the parties, the rate of legal
interest for loans or forbearance of any money, goods or credits and the rate allowed in judgments shall no
longer be twelve percent (12%) per annum - as reflected in the case of Eastern Shipping Lines40 and
Subsection X305.1 of the Manual of Regulations for Banks and Sections 4305Q.1, 4305S.3 and 4303P.1 of the
Manual of Regulations for Non-Bank Financial Institutions, before its amendment by BSP-MB Circular No. 799 -
but will now be six percent (6%) per annum effective July 1, 2013. It should be noted, nonetheless, that the new
rate could only be applied prospectively and not retroactively. Consequently, the twelve percent (12%) per
annum legal interest shall apply only until June 30, 2013. Come July 1, 2013 the new rate of six percent (6%)
per annum shall be the prevailing rate of interest when applicable.

FACTS:
Petitioner Dario Nacar filed a complaint for constructive against respondents Gallery Frames (GF) and/or Felipe
Bordey, Jr. The LA ruled in favor of petitioner. An Entry of Judgment was later issued certifying that the
resolution became final and executory on May 27, 2002. The case was, thereafter, referred back to the Labor
Arbiter.

Petitioner filed a Motion for Correct Computation, praying that his backwages be computed from the
date of his dismissal on January 24, 1997 up to the finality of the Resolution of the Supreme Court on
May 27, 2002. Upon recomputation, the Computation and Examination Unit of the NLRC arrived at an updated
amount in the sum of P471,320.31.12On August 20, 2003, an Entry of Judgment was issued declaring the
Resolution of the NLRC to be final and executory. Consequently, another pre-execution conference was held,
but respondents failed to appear on time. Meanwhile, petitioner moved that an Alias Writ of Execution be issued
to enforce the earlier recomputed judgment award in the sum of P471,320.31.

The records of the case were again forwarded to the Computation and Examination Unit for recomputation,
where the judgment award of petitioner was reassessed to be in the total amount of only P147,560.19.

Petitioner then moved that a writ of execution be issued ordering respondents to pay him the original amount as
determined by the Labor Arbiter in his Decision dated October 15, 1998, pending the final computation of his
backwages and separation pay. On January 14, 2003, the Labor Arbiter issued an Alias Writ of Execution to
satisfy the judgment award that was due to petitioner in the amount of P147,560.19, which petitioner eventually
received.

Petitioner then filed a Manifestation and Motion praying for the re-computation of the monetary award to include
the appropriate interests. The Labor Arbiter issued an Order granting the motion, but only up to the amount of
P11,459.73. The Labor Arbiter reasoned that it is the October 15, 1998 Decision that should be enforced
considering that it was the one that became final and executory. However, the Labor Arbiter reasoned
that since the decision states that the separation pay and backwages are computed only up to the
promulgation of the said decision, it is the amount of P158,919.92 that should be executed. Thus, since
petitioner already received P147,560.19, he is only entitled to the balance of P11,459.73.

ISSUE: What is the applicable interest rate?

SC RULING:
Interest of twelve percent (12%) per annum of the total monetary awards, computed from May 27, 2002
to June 30, 2013 and six percent (6%) per annum from July 1, 2013 until their full satisfaction.

To recapitulate and for future guidance, the guidelines laid down in the case of Eastern Shipping Lines42 are
accordingly modified to embody BSP-MB Circular No. 799, as follows:

San Beda College of Law 499


4S: 2015 - 2016
LABOR LAW REVIEW Atty. Joyrich Golangco

I. When an obligation, regardless of its source, i.e., law, contracts, quasi-contracts, delicts or quasi-delicts is
breached, the contravenor can be held liable for damages. The provisions under Title XVIII on "Damages" of the
Civil Code govern in determining the measure of recoverable damages.1wphi1

II. With regard particularly to an award of interest in the concept of actual and compensatory damages, the rate
of interest, as well as the accrual thereof, is imposed, as follows:

When the obligation is breached, and it consists in the payment of a sum of money, i.e., a loan or forbearance of
money, the interest due should be that which may have been stipulated in writing. Furthermore, the interest due
shall itself earn legal interest from the time it is judicially demanded. In the absence of stipulation, the rate of
interest shall be 6% per annum to be computed from default, i.e., from judicial or extrajudicial demand under
and subject to the provisions of Article 1169 of the Civil Code.

When an obligation, not constituting a loan or forbearance of money, is breached, an interest on the amount of
damages awarded may be imposed at the discretion of the court at the rate of 6% per annum. No interest,
however, shall be adjudged on unliquidated claims or damages, except when or until the demand can be
established with reasonable certainty. Accordingly, where the demand is established with reasonable certainty,
the interest shall begin to run from the time the claim is made judicially or extrajudicially (Art. 1169, Civil Code),
but when such certainty cannot be so reasonably established at the time the demand is made, the interest shall
begin to run only from the date the judgment of the court is made (at which time the quantification of damages
may be deemed to have been reasonably ascertained). The actual base for the computation of legal interest
shall, in any case, be on the amount finally adjudged.

When the judgment of the court awarding a sum of money becomes final and executory, the rate of legal
interest, whether the case falls under paragraph 1 or paragraph 2, above, shall be 6% per annum from such
finality until its satisfaction, this interim period being deemed to be by then an equivalent to a forbearance of
credit.

And, in addition to the above, judgments that have become final and executory prior to July 1, 2013, shall not be
disturbed and shall continue to be implemented applying the rate of interest fixed therein.

San Beda College of Law 500


4S: 2015 - 2016
LABOR LAW REVIEW Atty. Joyrich Golangco

BANI RURAL BANK INC. ENOC THEATER I AND II and/or RAFAEL DE GUZMAN, Petitioners, vs. TERESA
DE GUZMAN, EDGAR C. TAN and TERESA G. TAN, Respondents.
G.R. No.170904 November 13, 2013
BRION, J.:

DOCTRINE:
Thus, the computation of the respondents' backwages must be from the time of the illegal dismissal from
employment until the finality of the decision ordering the payment of separation pay.

FACTS:
The respondents were employees of Bani Rural Bank, Inc. and ENOC Theatre I and II who filed a complaint for
illegal dismissal against the petitioners. The NLRC ordered the petitioners to: Reinstate the two complainants to
their former positions, without loss o seniority rights and other benefits and privileges, with backwages from the
time o their dismissal (constructive) until their actual reinstatement, less earnings elsewhere.

The March 17, 1995 resolution of the NLRC became final and executory and the computation of the awards was
remanded to the labor arbiter for execution purposes.

The first computation of the monetary award under the March 17, 1995 resolution of the NLRC

First, Labor Arbiter Gambito deducted the earnings derived by the respondents either from Bani Rural Bank, Inc.
or ENOC Theatre I and II. Second, Labor Arbiter Gambito fixed the period of backwages from the respondents'
illegal dismissal until August 25 1995 or the date when the respondents allegedly manifested that they no longer
wanted to be reinstated.

The respondents appealed Labor Arbiter Gambito's computation with the NLRC.
The NLRC modified the terms of the March 17, 1995 resolution insofar as it clarified the phrase less earnings
elsewhere. The NLRC additionally awarded the payment of separation pay, in lieu of reinstatement, under the
following terms: The decision of this Commission is hereby MODIFIED to the extent that: (1) the phrase
earnings elsewhere in its dispositive portion shall exclude the complainants' salaries from the Rural Bank of
Mangantarem; and (2) in lieu of reinstatement, the respondents are hereby ordered to pay the complainants
separation pay equivalent to one month salary for every year of service computed from the start of their
employment up to the date of the finality of the decision.

The respondents filed a motion for reconsideration on whether the award of backwages was still included in the
judgment. The NLRC dismissed the motion for having been filed out of time. On January 29, 1999, the July 31,
1998 decision of the NLRC lapsed to finality and became executory.

The second computation of the monetary awards under the July 31, 998 decision of the NLRC
Labor Arbiter Gambito computed the respondents backwages only up to August 25, 1995.

NLRC RULING:
The NLRC ruled that the computation of the respondents backwages should be until January 29, 1999 which
was the date when the July 31, 1998 decision attained finality:

ISSUE: What is the basis for the computation of backwages?

SC RULING:
The computation of the respondents' backwages must be from the time of the illegal dismissal from
employment until the finality of the decision ordering the payment of separation pay. It is only when the
NLRC rendered its July 31, 1998 decision ordering the payment of separation pay (which both parties no longer
questioned and which thereafter became final) that the issue of the respondents' employment with the
petitioners was decided with finality, effectively terminating it. The respondents' backwages, therefore, must be
computed from the time of their illegal dismissal until January 29, 1999, the date of finality of the NLRC's July
31, 1998 Decision. As a final point, the CA s ruling must be modified to include legal interest commencing from
the finality of the NLRC's July 31, 1998 decision. The CA failed to consider that the NLRC's July 31, 1998
decision, once final, becomes a judgment for money from which another consequence flows -the payment of
interest in case of delay. Under the circumstances, the payment of legal interest of six percent (6) upon the
finality of the judgment is proper. It is not barred by the principle of immutability of judgment as it is
San Beda College of Law 501
4S: 2015 - 2016
LABOR LAW REVIEW Atty. Joyrich Golangco

compensatory interest arising from the final judgment.

The computation of backwages depends on the final awards adjudged as a consequence of illegal dismissal, in
that: First, when reinstatement is ordered, the general concept under Article 279 of the Labor Code, as
amended, computes the backwages from the time of dismissal until the employees reinstatement. The
computation of backwages (and similar benefits considered part of the backwages) can even continue beyond
the decision of the labor arbiter or NLRC and ends only when the employee is actually reinstated.

Second, when separation pay is ordered in lieu of reinstatement (in the event that this aspect of the case is
disputed) or reinstatement is waived by the employee (in the event that the payment of separation pay, in lieu, is
not disputed), backwages is computed from the time of dismissal until the finality of the decision ordering
separation pay.

Third, when separation pay is ordered after the finality of the decision ordering the reinstatement by reason of a
supervening event that makes the award of reinstatement no longer possible (as in the case), backwages is
computed from the time of dismissal until the finality of the decision ordering separation pay.

The above computation of backwages, when separation pay is ordered, has been the Courts consistent ruling.
In Session Delights Ice Cream and Fast Foods v. Court Appeals Sixth Division, we explained that the finality of
the decision becomes the reckoning point because in allowing separation pay, the final decision effectively
declares that the employment relationship ended so that separation pay and backwages are to be computed up
to that point.

We may also view the proper computation of backwages (whether based on reinstatement or an order of
separation pay) in terms of the life of the employment relationship itself.

When reinstatement is ordered, the employment relationship continues. Once the illegally dismissed employee
is reinstated, any compensation and benefits thereafter received stem from the employee s continued
employment. In this instance, backwages are computed only up until the reinstatement of the employee since
after the reinstatement, the employee begins to receive compensation from his resumed employment.

When there is an order of separation pay (in lieu of reinstatement or when the reinstatement aspect is waived or
subsequently ordered in light of a supervening event making the award of reinstatement no longer possible), the
employment relationship is terminated only upon the finality of the decision ordering the separation pay. The
finality of the decision cuts-off the employment relationship and represents the final settlement of the rights and
obligations of the parties against each other. Hence, backwages no longer accumulate upon the finality of the
decision ordering the payment of separation pay since the employee is no longer entitled to any compensation
from the employer by reason of the severance of his employment.

The computation of the respondents backwages

As the records show, the contending parties did not dispute the NLRC s order of separation pay that replaced
the award of reinstatement on the ground of the supervening event arising from the newly-discovered strained
relations between the parties. The parties allowed the NLRCs July 31, 1998 decision to lapse into finality and
recognized, by their active participation in the second computation of the awards, the validity and binding effect
on them of the terms of the July 31, 1998 decision.

Under these circumstances, while there was no express modification on the period for computing backwages
stated in the dispositive portion of the July 31, 1998 decision of the NLRC, it is nevertheless clear that the award
of reinstatement under the March 17, 1995 resolution (to which the respondents backwages was initially
supposed to have been computed) was substituted by an award of separation pay. As earlier stated, the
awards of reinstatement and separation pay are exclusive remedies; the change of awards (from
reinstatement to separation pay) under the NLRC s July 31, 1998 not only modified the awards granted,
but also changed the manner the respondents backwages is to be computed. The respondents
backwages can no longer be computed up to the point of reinstatement as there is no longer any award
of reinstatement to speak of.

San Beda College of Law 502


4S: 2015 - 2016
LABOR LAW REVIEW Atty. Joyrich Golangco

UNIVERSAL ROBINA CORPORATION and LANCE Y. GOKONGWEI vs. WILFREDO Z. CASTILLO
G.R. No. 189686 July 10, 2013
PEREZ, J.:

AWARD OF SEPARATION PAY FOR VALIDLY DISMISSED EMPLOYEES

DOCTRINE:
As the rule now stands, the award of separation pay is authorized in the situations dealt with in Article 283 and
284 of the Labor Code, but not in terminations of employment based on instances enumerated in Article 282.

FACTS:
Petitioner Universal Robina Corporation (URC) terminated from employment as a Regional Sales Manager,
respondent Wilfredo Castillo (Castillo). He was dismissed for breach of trust and confidence and acts inimical to
the interest of URC after a finding that he signed 2 blank Charge Invoices despite full knowledge that the same
will be used by Lianas Supermarket to cause the deduction of P72,000.00 against the account of URC for the
gift certificate that Castillo unduly received. Castillo filed a complaint for illegal dismissal against URC.

LA declared Castillo to have been illegally dismissed and ordered the payment of backwages and separation
pay. The NLRC reversed the decision of LA after finding that URC had more than sufficient proof that
respondent violated its trust. MR of Castillo was denied. On appeal, the CA upheld his dismissal but awarded
him separation pay "as a form of equitable relief considering that he honorably served the company for about
23 years and this is his only and first offense.

ISSUE: Whether Castillo is entitled to separation pay?

HELD: NO. Castillo is not entitled to separation pay.

As the rule now stands, the award of separation pay is authorized in the situations dealt with in Article 283 and
284 of the Labor Code, but not in terminations of employment based on instances enumerated in Article 282.
Labor adjudicatory officials and the CA must demur the award of separation pay based on social justice when
an employees dismissal is based on serious misconduct or willful disobedience; gross and habitual neglect of
duty; fraud or willful breach of trust; or commission of a crime against the person of the employer or his
immediate family. They must be most judicious and circumspect in awarding separation pay or financial
assistance as the constitutional policy to provide full protection to labor is not meant to be an instrument to
oppress the employers. In fine, we should be more cautious in awarding financial assistance to the undeserving
and those who are unworthy of the liberality of the law.

Indeed, Castillo has committed acts constituting willful breach of trust and confidence reposed on him by URC.
The principal charge against petitioner Castillo was hinged upon "unauthorized arrangements" which he
allegedly entered into. His neglect in signing the blank charge invoices and his improper receipt of gift
certificates for his personal gain, when taken together constitute a breach of the trust and confidence reposed
on petitioner Castillo by URC.

As a Regional Sales Manager, petitioner Castillo occupied a position or responsibility and as such, he should
have known that he placed the interests of the company at a disadvantage by signing the blank charge invoices.
Because of such act, private respondent URC was prejudiced by no less than P72,000.00. This alone is
sufficient cause for breach of trust and loss of confidence.

San Beda College of Law 503


4S: 2015 - 2016
LABOR LAW REVIEW Atty. Joyrich Golangco

MINETTE BAPTISTA, BANNIE EDSEL SAN MIGUEL, and MA. FEDAYON vs. ROSARIO VILLANUEVA, et.
al.
G.R. No. 194709 July 31, 2013
MENDOZA, J.:

SUBSTANTIVE AND PROCEDURAL DUE PROCESS IN ADMINISTRATIVE PROCEEDINGS

DOCTRINE: In administrative proceedings, the filing of charges and giving reasonable opportunity for the
person so charged to answer the accusations against him constitute the minimum requirements of due process.
The essence of due process is simply to be heard, or as applied to administrative proceedings, an opportunity to
explain ones side, or an opportunity to seek a reconsideration of the action or ruling complained of. Mere
absence of a one-on-one confrontation between the petitioners and their complainants does not automatically
affect the validity of the proceedings before the Committee. Not all cases necessitate a trial-type hearing.

FACTS:
Petitioners were former union members of Radio Philippines Network Employees Union (RPNEU), a legitimate
labor organization and the sole and exclusive bargaining agent of the rank and file employees of Radio
Philippines Network (RPN), while the respondents were the unions elected officers and members.

On suspicion of union mismanagement, petitioners filed a complaint for impeachment of their union president,
Reynato Siozon, before the executive board of RPN, which was eventually abandoned. They later re-lodged the
impeachment complaint, this time, against all the union officers and members of RPNEU before the DOLE and
likewise filed various petitions for audit covering the period from 2000 to 2004.

Thereafter, complaints were filed before the Chairman of RPNEUs Committee on Grievance and Investigation
(the Committee) against petitioners citing as grounds the "commission of an act which violates RPNEU
Constitution and By-Laws, specifically, Article IX, Section 2.5 for urging or advocating that a member start an
action in any court of justice or external investigative body against the Union or its officer without first exhausting
all internal remedies open to him or available in accordance with the CBL.

The Committee submitted their recommendation of expulsion from the union to RPNEUs Board of Directors
which was affirmed by the RPNEUs Board of Directors. Through a Memorandum, petitioners were served an
expulsion notice from the union. Consequently, RPN informed petitioners of the termination of their employment
enforcing Article II, Section 2 also known as the union security clause of their current CBA.

Petitioners filed a complaint for ULP against the respondents arguing that that they were denied substantive and
procedural due process of law when they were expelled from the RPNEU. First, they posit that the procedure
that should have been followed by the respondents in resolving the charges against them should be that
provided in their Constitution, which requires that members put their grievance in writing to be submitted to their
union president, who shall strive to have the parties settle their differences amicably. Petitioners maintain that
any form of grievance would be referred only to the committee upon failure of the parties to settle amicably.
They also point out that they were not given the opportunity to personally face and confront their accusers,
which were violative of their right to examine the complainants and the supposed charges against them.

ISSUE: Were petitioners deprived of substantive and procedural due process in their expulsion from the union?

HELD: NO. As to substantive due process, the SC ruled that the expulsion of petitioners was meritorious. It is
well-settled that workers and employers organizations shall have the right to draw up their constitutions. In this
case, RPNEUs Constitution and By-Laws expressly mandate that before a party is allowed to seek the
intervention of the court, it is a pre-condition that he should have availed of all the internal remedies within the
organization. Petitioners were found to have violated such when they filed petitions for impeachment against
their union officers and for audit before the DOLE without first exhausting all internal remedies available within
their organization. This act is a ground for expulsion from union membership. Thus, petitioners expulsion from
the union was not a deliberate attempt to curtail or restrict their right to organize, but was triggered by the
commission of an act, expressly sanctioned by their Constitution.

Going now to the procedural aspect, the SC declared that the expulsion was free from infirmities. Under
RPNEUs Constitution, any charge against the member must be submitted to the Board of Directors and not to
the Committee. Although the respondents filed the charge before the Committee, this supposed procedural flaw
San Beda College of Law 504
4S: 2015 - 2016
LABOR LAW REVIEW Atty. Joyrich Golangco

was deemed cured when petitioners were given the opportunity to be heard. Due process, as a constitutional
precept, is satisfied when a person was notified of the charge against him and was given an opportunity to
explain or defend himself. In administrative proceedings, the filing of charges and giving reasonable opportunity
for the person so charged to answer the accusations against him constitute the minimum requirements of due
process. The essence of due process is simply to be heard, or as applied to administrative proceedings, an
opportunity to explain ones side, or an opportunity to seek a reconsideration of the action or ruling complained
of. It cannot be denied that petitioners were properly notified of the charges filed against them and were equally
afforded the opportunity to present their side.

San Beda College of Law 505


4S: 2015 - 2016
LABOR LAW REVIEW Atty. Joyrich Golangco

BPI EMPLOYEES UNION-DAVAO CITY-FUBU vs BANK OF THE PHILIPPINE ISLANDS
G.R. No. 174912 July 24, 2013
MENDOZA, J.:

FACTS:
BOMC was created as a subsidiary of BPI, operating and functioning as an entirely separate and distinct entity.
A service agreement between BPI and BOMC was implemented wherein BOMC undertook to provide certain
services such as check clearing, card production, operations accounting, and cash servicing.

Following a merger between BPI and FEBTC, a transfer of the functions of twelve Personnel engaged in BPIs
cashiering function and FEBTCs cashiering, distribution and bookkeeping functions to BOMC was effected. The
Union objected to the transfer of the functions to BOMC claiming that the functions rightfully belonged to the BPI
employees and that the Union was deprived of membership of former FEBTC personnel, who by virtue of the
merger, would have formed part of the bargaining unit represented by the Union pursuant to its union shop
provision in the CBA.

BPI invoked management prerogative stating that the creation of the BOMC was to preserve more jobs and to
designate it as an agency to place employees where they were most needed. On the other hand, the Union
charged that BOMC undermined the existence of the union since it reduced or divided the bargaining unit. While
BOMC employees perform BPI functions, they were beyond the bargaining units coverage. In contracting out
FEBTC functions to BOMC, BPI effectively deprived the union of the membership of employees handling said
functions as well as curtailed the right of those employees to join the union.

Thereafter, the Union demanded that the matter be submitted to the grievance machinery as the resort to the
LMC was unsuccessful. As BPI allegedly ignored the demand, the Union filed a notice of strike before the
National Conciliation and Mediation Board(NCMB) on the following grounds:cralavvonlinelawlibrary

a) Contracting out services/functions performed by union members that interfered


with, restrained and/or coerced the employees in the exercise of their right to
self-organization;

b) Violation of duty to bargain; and


9
c) Union busting.

BPI then filed a petition for assumption of jurisdiction/certification with the Secretary of the Department of Labor
and Employment (DOLE), who subsequently issued an order certifying the labor dispute to the NLRC for
compulsory arbitration. The DOLE Secretary directed the parties to cease and desist from committing any act
that might exacerbate the situation.

The NLRC ruled upholding the validity of the service agreement between BPI and BOMC and dismissing the
charge of ULP. The CA upheld the ruling of the NLRC, hence this petition.

ISSUE: the primordial issue in this case is whether or not the act of BPI to outsource the cashiering, distribution
and bookkeeping functions to BOMC is in violation of the law and the existing CBA. Particularly in dispute is the
validity of the transfer of twelve (12) former FEBTC employees to BOMC, instead of being absorbed in BPI after
the corporate merger.

RULING: No. The Supreme Court held that only gross violations of the economic provisions of the CBA are
treated as ULP. Otherwise, they are mere grievances.

In the present case, the alleged violation of the union shop agreement in the CBA, even assuming it was
malicious and flagrant, is not a violation of an economic provision in the agreement. The provisions relied upon
by the Union were those articles referring to the recognition of the union as the sole and exclusive bargaining
representative of all rank-and-file employees, as well as the articles on union security, specifically, the
maintenance of membership in good standing as a condition for continued employment and the union shop
26
clause. It failed to take into consideration its recognition of the banks exclusive rights and prerogatives,

San Beda College of Law 506


4S: 2015 - 2016
LABOR LAW REVIEW Atty. Joyrich Golangco

likewise provided in the CBA, which included the hiring of employees, promotion, transfers, and dismissals for
just cause and the maintenance of order, discipline and efficiency in its operations.

The Union, however, insists that jobs being outsourced to BOMC were included in the existing bargaining unit,
thus, resulting in a reduction of a number of positions in such unit. The reduction interfered with the employees
28
right to self-organization because the power of a union primarily depends on its strength in number.

It is incomprehensible how the reduction of positions in the collective bargaining unit interferes with the
employees right to self-organization because the employees themselves were neither transferred nor dismissed
from the service. As the NLRC clearly stated:cralavvonlinelawlibrary

In the case at hand, the union has not presented even an iota of evidence that
petitioner bank has started to terminate certain employees, members of the union. In
fact, what appears is that the Bank has exerted utmost diligence, care and effort to see
to it that no union member has been terminated. In the process of the consolidation or
merger of the two banks which resulted in increased diversification of functions, some
of these non-banking functions were merely transferred to the BOMC without affecting
the union membership.

As far as the twelve (12) former FEBTC employees are concerned, the Union failed to substantially prove that
their transfer, made to complete BOMCs service complement, was motivated by ill will, anti-unionism or bad
faith so as to affect or interfere with the employees right to self-organization.

San Beda College of Law 507


4S: 2015 - 2016
LABOR LAW REVIEW Atty. Joyrich Golangco

INTEGRATED MICROELECTRONICS, INC. vs PIONILLA
G.R. No. 200222 August 28, 2013
PERLAS-BERNABE, J.:

FACTS:
Respondent Adonis Pionilla was hired by petitioner IMI as its production worker. On May 5, 2005, Pionilla
received a notice from IMI requiring him to explain the incident which occurred the day before where he was
seen escorting a lady to board the company shuttle bus at the Alabang Terminal. It was reported by the bus
marshall that the lady was wearing a company identification card (ID) which serves as a free pass for shuttle
bus passengers even if she was just a job applicant at IMI. In this regard, Pionilla admitted that he lent his
temporary second ID to the lady who turned out to be his relative. He further intimated that he risked lending her
his ID to save on their transportation expenses. Nevertheless, he apologized for his actions.

A Committee was formed to investigate the matter, and found that Pionilla violated the Company Rules and
Regulations which prohibits the lending of company IDs since the same is considered a breach of its security
and carries the penalty of dismissal. Pionilla was dismissed from his job, prompting him to file a complaint for
illegal dismissal with damages against IMI.

The LA found Pionilla illegally dismissed, noting that the penalty of dismissal was too harsh dismissed and
ordered his reinstatement with backwages. The NLRC reversed the LA and upheld his dismissal. The CA
reinstated the ruling of the Labor Arbiter, agreeing that the penalty imposed upon Pionilla was too harsh.

ISSUE:
Whether or not Pionilla is entitled to reinstatement with full back wages.

RULING:
The Supreme Court partially granted the petition of IMI, holding that while Pionilla should be reinstated, he is not
entitled to backwages. As a general rule, an illegally dismissed employee is entitled to reinstatement (or
separation pay, if reinstatement is not viable) and payment of full backwages. In certain cases, however, the
Court has carved out an exception to the foregoing rule and thereby ordered the reinstatement of the employee
without backwages on account of the following: (a) the fact that dismissal of the employee would be too harsh of
a penalty; and (b) that the employer was in good faith in terminating the employee.

In this case, the Court observes that: (a) the penalty of dismissal was too harsh of a penalty to be imposed
against Pionilla for his infractions; and (b) IMI was in good faith when it dismissed Pionilla as his dereliction of its
policy on ID usage was honestly perceived to be a threat to the company's security. As such, Pionilla was
reinstated, but the award of backwages was deleted.

San Beda College of Law 508


4S: 2015 - 2016
LABOR LAW REVIEW Atty. Joyrich Golangco

GOLDEN ACE BUILDERS and ARNOLD U. AZUL vs. JOSE A. TALDE
G.R. No. 187200 May 5, 2010
CARPIO MORALES, J.

FACTS: Jose Talde was hired in 1990 as a carpenter by petitioner Golden Ace Builders of which its co-
petitioner Arnold Azul is the owner-manager. In February 1999, Azul, alleging the unavailability of construction
projects, stopped giving work assignments to respondent, prompting the latter to file a complaint for illegal
dismissal.

The Labor Arbiter ruled in favor of respondent and ordered his immediate reinstatement without loss of
seniority rights and other privileges, and with payment of full backwages. Pending their appeal to the NLRC and
in compliance with the LAs Decision, petitioners advised respondent to report for work in the construction site
within 10 days from receipt thereof. Respondent manifested, however, that actual animosities existed between
him and petitioners and there had been threats to his life and his familys safety, hence, he opted for the
payment of separation pay. Petitioners denied the existence of any such animosity.

Finding the amount awarded exorbitant, petitioners filed a motion for reconsideration with the NLRC,
contending that since respondent refused to report back to work, he should be considered to have abandoned
the same, hence, the recomputation of the wages and benefits due him should not be beyond May 15, 2001, the
date when he manifested his refusal to be reinstated.

NLRC Ruling: granted petitioners motion and accordingly vacated the computation.

CA Ruling: set aside the NLRC Resolutions, holding that respondent is entitled to both backwages and
separation pay, even if separation pay was not granted by the Labor Arbiter, the latter in view of the strained
relations between the parties.

ISSUE: Is respondent entitled to both backwages and separation pay, due to strained relations?

SC Ruling: YES. The basis for the payment of backwages is different from that for the award of separation pay.
Separation pay is granted where reinstatement is no longer advisable because of strained relations between the
employee and the employer. Backwages represent compensation that should have been earned but were not
collected because of the unjust dismissal. The basis for computing backwages is usually the length of the
employees service while that for separation pay is the actual period when the employee was unlawfully
prevented from working.

Under the doctrine of strained relations, the payment of separation pay is considered an acceptable
alternative to reinstatement when the latter option is no longer desirable or viable. On one hand, such payment
liberates the employee from what could be a highly oppressive work environment. On the other hand, it
releases the employer from the grossly unpalatable obligation of maintaining in its employ a worker it could no
longer trust.
In the present case, the Labor Arbiter found that actual animosity existed between petitioner Azul and
respondent as a result of the filing of the illegal dismissal case. Such finding, especially when affirmed by the
appellate court as in the case at bar, is binding upon the Court, consistent with the prevailing rules that this
Court will not try facts anew and that findings of facts of quasi-judicial bodies are accorded great respect, even
finality.

Clearly then, respondent is entitled to backwages and separation pay as his reinstatement has been
rendered impossible due to strained relations. As correctly held by the appellate court, the backwages due
respondent must be computed from the time he was unjustly dismissed until his actual reinstatement, or from
February 1999 until June 30, 2005 when his reinstatement was rendered impossible without fault on his part.

San Beda College of Law 509


4S: 2015 - 2016
LABOR LAW REVIEW Atty. Joyrich Golangco

METROGUARDS SECURITY AGENCY CORPORATION vs. ALBERTO N. HILONGO
G.R. No. 215630 March 09, 2015
VILLARAMA, JR., J.

FACTS: In the case, entitled Alberto Hilongo v. Bee Guards Corp., the Labor Arbiter ruled that herein
respondent Alberto N. Hilongo was illegally dismissed and ordered herein petitioners to pay Hilongo his
backwages from the date of dismissal to the date of the decision and separation pay of one month pay per year
of service, plus 10% thereof as attorneys fees.

On appeal, the NLRC reversed the ruling of the LA. Aggrieved, Hilongo filed a petition for certiorari before the
CA, which reversed the NLRC decision and reinstated the LAs Decision. Petitioners no longer appealed to the
Supreme Court.

Hilongo then filed a motion for entry of judgment and a motion for clarification of Decision/Resolution. The CA
granted the motion for entry of judgment and noted Hilongos motion for clarification of Decision/Resolution. The
CA held that when an appellate court affirms the Labor Arbiters ruling, it is understood that awards due to the
illegally dismissed employee shall be recomputed in order to account for the period of time that has lapsed from
the rendition of the Labor Arbiters decision up to its finality.

After the corresponding entry of judgment was issued, the case was remanded to the Labor Arbiter. On July 9,
2013, respondent Hilongo filed a motion for issuance of writ of execution alleging that the CA Resolution had
confirmed that the amount of P170,520.31 awarded by the Labor Arbiter is not sufficient, and that there is a
need to compute additional monetary awards reckoned from May 1, 2010 up to April 26, 2013 or the date
Hilongo presumed as the date of finality of the decision.

LA Ruling: directed the issuance of a writ of execution and ruled that the award of P170,520.31 as stated in the
LAs Decision prevails.

NLRC Ruling: dismissed the petition for extraordinary remedy.

CA Ruling: granted Hilongos petition and set aside the NLRC Decision and ordered the LA to re-compute
Hilongos monetary awards. The CA held that it is already settled that the computation of the monetary awards
due to the illegally dismissed employee must continue to run until the final termination of the case on appeal.

ISSUE: Whether the CA erred in ordering the re-computation of Hilongos monetary awards.

SC Ruling: NO. In Nacar v. Gallery Frames, the Court held that: xxx no essential change is made by a
recomputation as this step is a necessary consequence that flows from the nature of the illegality of dismissal
declared by the Labor Arbiter in that decision. A recomputation (or an original computation, if no previous
computation has been made) is a part of the law specifically, Article 279 of the Labor Code and the
established jurisprudence on this provision that is read into the decision. By the nature of an illegal dismissal
case, the reliefs continue to add up until full satisfaction, as expressed under Article 279 of the Labor Code. The
recomputation of the consequences of illegal dismissal upon execution of the decision does not constitute an
alteration or amendment of the final decision being implemented. The illegal dismissal ruling stands; only the
computation of monetary consequences of this dismissal is affected, and this is not a violation of the principle of
immutability of final judgments.

The Court thus cannot agree with petitioners contention that a decision that has acquired finality becomes
immutable and unalterable. The re-computation of the consequences of illegal dismissal upon execution of the
decision does not constitute an alteration or amendment of the final decision being implemented. The illegal
dismissal ruling stands; only the computation of monetary consequences of this dismissal is affected, and this is
not a violation of the principle of immutability of final judgments.

San Beda College of Law 510


4S: 2015 - 2016
LABOR LAW REVIEW Atty. Joyrich Golangco

MAERSK-FILIPINAS CREWING, INC., A.P. MOLLER SINGAPORE PTE. LIMITED, AND JESUS AGBAYANI
v. TORIBIO C. AVESTRUZ
G.R. No. 207010 February 18, 2015
PERLAS-BERNABE, J.:

ILLEGAL DISMISSAL; BURDEN OF PROOF

DOCTRINE: It is well-settled that the burden of proving that the termination of an employee was for a just or
authorized cause lies with the employer. If the employer fails to meet this burden, the conclusion would be that
the dismissal was unjustified and, therefore, illegal. In order to discharge this burden, the employer must present
substantial evidence, which is defined as that amount of relevant evidence which a reasonable mind might
accept as adequate to justify a conclusion,58 and not based on mere surmises or conjectures.

FACTS:
Petitioner Maersk, on behalf of its foreign principal, petitioner A.P. Moller, hired Avestruz as Chief Cook on
board the vessel M/V Nedlloyd Drake for a period of 6 months.

In the course of the weekly inspection of the vessels galley, Captain Woodward noticed that the cover of the
garbage bin in the kitchen near the washing area was oily. As part of Avestruzs job was to ensure the
cleanliness of the galley, Captain Woodward called Avestruz and asked him to stand near the garbage bin
where the former took the latters right hand and swiped it on the oily cover of the garbage bin, telling Avestruz
to feel it. Shocked, Avestruz remarked, Sir if you are looking for dirt, you can find it; the ship is big. Tell us if you
want to clean and we will clean it. Captain Woodward replied by shoving Avestruzs chest, to which the latter
complained and said, Dont touch me, causing an argument to ensue between them.

Later that afternoon, Captain Woodward summoned and required Avestruz to state in writing what transpired in
the galley that morning. Avestruz complied and submitted his written statement on that same day. Captain
Woodward likewise asked Messman Kong to submit his own written statement regarding the incident, to which
the latter immediately complied. On the very same day, Captain Woodward informed Avestruz that he would be
dismissed from service and be disembarked in India. Avestruz was disembarked in Colombo, Sri Lanka and
arrived in the Philippines. Subsequently, he filed a complaint for illegal dismissal.

LA Ruling:
The LA dismissed Avestruzs complaint for lack of merit. The LA found that he failed to perform his duty of
maintaining cleanliness in the galley, and that he also repeatedly failed to obey the directives of his superior,
which was tantamount to insubordination.

NLRC Ruling:
The NLRC sustained the validity of Avestruzs dismissal but found that petitioners failed to observe the
procedures hence, held petitioners liable to pay damages.

CA Ruling:
The CA reversed and set aside the rulings of the NLRC and instead, found Avestruz to have been illegally
dismissed.

ISSUE: Was Avestruz illegally dismissed?

HELD: Yes, Avestruz was illegally dismissed.

It is well-settled that the burden of proving that the termination of an employee was for a just or authorized
cause lies with the employer. If the employer fails to meet this burden, the conclusion would be that the
dismissal was unjustified and, therefore, illegal. In order to discharge this burden, the employer must present
substantial evidence, which is defined as that amount of relevant evidence which a reasonable mind might
accept as adequate to justify a conclusion, and not based on mere surmises or conjectures.

Insubordination, as a just cause for the dismissal of an employee, necessitates the concurrence of at least two
requisites: (1) the employees assailed conduct must have been willful, that is, characterized by a wrongful and
perverse attitude; and (2) the order violated must have been reasonable, lawful, made known to the employee,
and must pertain to the duties which he had been engaged to discharge.
San Beda College of Law 511
4S: 2015 - 2016
LABOR LAW REVIEW Atty. Joyrich Golangco

In this case, the contents of Captain Woodwards e-mails do not establish that Avestruzs conduct had been
willful, or characterized by a wrongful and perverse attitude. The Court concurs with the CAs observation that
Avestruzs statement62 regarding the incident in the galley deserves more credence, being corroborated by
Kong, a messman who witnessed the same.

Conversely, apart from Captain Woodwards e-mails, no other evidence was presented by the petitioners to
support their claims.

As in this case, it was incumbent upon the petitioners to present other substantial evidence to bolster their claim
that Avestruz committed acts that constitute insubordination as would warrant his dismissal. At the least, they
could have offered in evidence entries in the ships official logbook showing the infractions or acts of
insubordination purportedly committed by Avestruz, the ships logbook being the official repository of the day-to-
day transactions and occurrences on board the vessel. Having failed to do so, their position that Avestruz was
lawfully dismissed cannot be sustained.

San Beda College of Law 512


4S: 2015 - 2016
LABOR LAW REVIEW Atty. Joyrich Golangco

CONCEPCION A. VILLENA vs. BATANGAS II ELECTRIC COOPERATIVE, INC. and GEORGE A. DIN
G.R. No. 205735 February 4, 2015
PERLAS-BERNABE, J.:

FACTS:
Villena was hired by respondent Batangas II Electric Cooperative, Inc. (BATELEC II) as bookkeeper in 1978.
She rose from the ranks and was promoted as Finance Manager in 1985.In 1994, she was demoted to the
position of Auditor, which caused her to file a complaint for constructive dismissal before the LA. After the case
passed through LA and NLRC, the CA ruled that Villena was illegally dismissed and she was entitled among
others to any other benefits pertaining to the position of Finance Manager at the time she was removed
therefrom up to the date of her actual reinstatement." The case was then remanded to the NLRC for the
computation of the total amount due to Villena.

In the course thereof, the LA declared that Villena was entitled only to "salary differentials, 13th month pay,
unused sick leave, leave of absence" excluding from the computation claims for bonus, representation
allowance, transportation benefits, and attorneys fees. Moreover, her claim for separation pay in lieu of
reinstatement was denied.

The NLRC directed BATELEC II "to pay Villena her claim for separation pay in lieu of reinstatement, salary
differentials and other benefits, from the date of her dismissal up to the date of the payment of her separation
pay, and attorneys fees equivalent to 10% of the totality of her award." BATELEC II moved for reconsideration,
but the same was denied. The NLRC Resolution became final and executor.

Hence, Villena filed a motion for execution before the LA.

LA Ruling:
Acting on the motion for execution, the LA found Villena to be entitled to the following benefits: (a) salary
differentials; (b) 13th month pay; (c) 14th month pay; (d) bonus cash gift; (e) unused sick leave; (f) leave of
absence; (g) uniform allowance; (h) separation pay; (i) representation allowance; (j) transportation allowance; (k)
cellular phone allowance; (l) retirement pay; and (m) attorneys fees.

Respondents Contention: Villena was not entitled to salary differentials, allowances and benefits of a Finance
Manager, separation pay, and allowances for representation, transportation, and cellular phone usage.

NLRC Ruling:
The NLRC partly granted the appeal and excluded from the computation of monetary awards the sums for
representation, transportation, and cellular phone usage allowances, as well as retirement pay. It found that
Villena was not able to prove that she was qualified to receive representation allowance or that she was
authorized to travel. The NLRC likewise found no basis for the award of cellular phone allowance to Villena.

CA Ruling:
The CA affirmed the NLRC Resolution disallowing the inclusion of allowances for representation, transportation,
and cellular phone usage as Villena did not perform her duties as Finance Manager not being a certified public
accountant which is a required qualification for such position.

ISSUE: Whether or not (a) retirement pay, and (b) representation, transportation, and cellular phone usage
allowances should be awarded in favor of Villena.

HELD: The petition is partly meritorious.

As to Retirement Pay:

In order for her retirement pay claim to be considered, Villenas complaint should have contained substantial
allegations which would show that she (a) had applied for the same, and (b) her application squares with the
requirements of entitlement under the terms of the companys retirement plan. However, based on the records,
what she sought for in her illegal dismissal complaint were the reliefs of reinstatement, payment of salary
differentials, all benefits and allowances that she may have received as Finance Manager, attorneys fees, and
damages (which does not include retirement pay).

San Beda College of Law 513


4S: 2015 - 2016
LABOR LAW REVIEW Atty. Joyrich Golangco

Verily, the Court is not unaware of its rulings wherein it pronounced that retirement pay and separation pay are
not mutually exclusive (unless there is a specific prohibition in the collective bargaining agreement or retirement
plan against the payment of both benefits); however, with Villenas entitlement to retirement pay not included as
an issue in an illegal dismissal case which had already been finally decided, it is quite absurd for Villena to
submit a "contemporaneous" claim for retirement pay on the execution phase of these proceedings. In fine, the
plea to include retirement pay in the execution under the phrase "other benefits," cannot be granted.

As to Transportation, Representation, and Cellular Phone Usage Allowances:

It is clear from BATELEC IIs pleadings and submissions that representation allowance, transportation
allowance, and cellular phone usage allowance are given to the Finance Manager/Department Manager as part
of their benefits, unlike the separate entitlement to retirement pay which may be recovered only upon a
meritorious subsequent application when the employee decides to retire. Consequently, these allowances ought
to be included in the "other benefits pertaining to the position of Finance Manager" to which Villena is entitled to
and which were awarded to her under the final and executory CA Decision and NLRC Resolution. On this score,
therefore, the claim of Villena is granted.

San Beda College of Law 514


4S: 2015 - 2016
LABOR LAW REVIEW Atty. Joyrich Golangco

LYNVIL FISHING ENTERPRISES, INC. and/or ROSENDO S. DE BORJA v. ANDRES G. ARIOLA, JESSIE D.
ALCOVENDAS, JIMMY B. CALINAO AND LEOPOLDO G. SEBULLEN
G.R. No. 181974 February 1, 2012
PEREZ

FIXED-TERM EMPLOYEES

DOCTRINE: Jurisprudence, laid two conditions for the validity of a fixed-contract agreement between the
employer and employee: First, the fixed period of employment was knowingly and voluntarily agreed upon by
the parties without any force, duress, or improper pressure being brought to bear upon the employee and
absent any other circumstances vitiating his consent; or Second, it satisfactorily appears that the employer and
the employee dealt with each other on more or less equal terms with no moral dominance exercised by the
former or the latter.

FACTS: Lynvil is a company engaged in deep-sea fishing, operating along the shores of Palawan and other
outlying islands of the Philippines, operated and managed by Rosendo S. de Borja. Lynvil received a report from
Romanito Clarido, one of its employees, that on 31 July 1998, he witnessed that while on board the company
vessel Analyn VIII, Lynvil employees, namely: Andres G. Ariola, the captain; Jessie D. Alcovendas, Chief Mate;
Jimmy B. Calinao, Chief Engineer; Ismael G. Nubla, cook; Elorde Baez, oiler; and Leopoldo D. Sebullen,
bodegero, conspired with one another and stole eight (8) tubs of "pampano" and "tangigue" fish and delivered
them to another vessel, to the prejudice of Lynvil.

The said employees were engaged on a per trip basis or "por viaje" which terminates at the end of each trip.
Ariola, Alcovendas and Calinao were managerial field personnel while the rest of the crew were field personnel.

By reason of the report and after initial investigation, De Borja summoned respondents to explain within five (5)
7
days why they should not be dismissed from service. However, except for Alcovendas and Baez, the
respondents refused to sign the receipt of the notice. Failing to explain as required, respondents employment
was terminated. Lynvil filed a criminal complaint against the dismissed employees.

Aggrieved, the employees filed with the Arbitration Branch of NLRC a complaint for illegal dismissal with claims
for backwages, salary differential reinstatement, service incentive leave, holiday pay and its premium and 13th
month pay from 1996 to 1998. They also claimed for moral, exemplary damages and attorneys fees for their
dismissal with bad faith. They added that the unwarranted accusation of theft stemmed from their oral demand
of increase of salaries three months earlier and their request that they should not be required to sign a blank
payroll and vouchers.

LA DECISION: Found that complainants were illegally dismissed, ordering respondents to jointly and severally
pay complainants separation pay at one half month pay for every year of service; backwages; salary differential;
13th month pay; and attorneys fees. LA disregarded the Resolution of Assistant City Prosecutor on the theft
case. LA ruled that the contractual provision that the employment terminates upon the end of each trip does not
make the respondents dismissal legal.

NLRC DECISION: Reversed and set aside the Decision of LA.

CA DECISION: Reinstated the Decision of LA except as to the award of attorneys fees. The appellate court
held that the allegation of theft did not warrant the dismissal of the employees since there was no evidence to
prove the actual quantities of the missing kinds of fish loaded to Analyn VIII. It added that they were regular ones
performing activities which are usually necessary or desirable in the business and trade of Lynvil.

ISSUE: Were the respondents illegally dismissed? Are the respondents regular employees of the petitioner?

RULING: Lynvil cannot argue that since the Office of the Prosecutor found probable cause for theft the LA must
follow the finding as a valid reason for the termination of respondents employment. The proof required for
purposes that differ from one and the other are likewise different. Nonetheless, even without reliance on the
prosecutors finding, there was valid cause for respondents dismissal. In illegal dismissal cases, the employer
bears the burden of proving that the termination was for a valid or authorized cause. Just cause is required for a
valid dismissal. Breach of trust is present in this case.

San Beda College of Law 515


4S: 2015 - 2016
LABOR LAW REVIEW Atty. Joyrich Golangco

42
Jurisprudence, laid two conditions for the validity of a fixed-contract agreement between the employer and
employee:
First, the fixed period of employment was knowingly and voluntarily agreed upon by the parties without any
force, duress, or improper pressure being brought to bear upon the employee and absent any other
circumstances vitiating his consent; or
Second, it satisfactorily appears that the employer and the employee dealt with each other on more or less
equal terms with no moral dominance exercised by the former or the latter.

Textually, the provision that: "NA ako ay sumasang-ayon na maglingkod at gumawa ng mga gawain sang-ayon
sa patakarang "por viaje" na magmumula sa pagalis sa Navotas papunta sa pangisdaan at pagbabalik sa
pondohan ng lantsa sa Navotas, Metro Manila" is for a fixed period of employment. In the context, however, of
the facts that: (1) the respondents were doing tasks necessarily to Lynvils fishing business with positions
ranging from captain of the vessel to bodegero; (2) after the end of a trip, they will again be hired for another trip
with new contracts; and (3) this arrangement continued for more than ten years, the clear intention is to go
around the security of tenure of the respondents as regular employees. And respondents are so by the express
provisions of the second paragraph of Article 280, thus:
xxx Provided, That any employee who has rendered at least one year of service, whether such service is
continuous or broken, shall be considered a regular employee with respect to the activity in which he is
employed and his employment shall continue while such activity exists.

The same set of circumstances indicate clearly enough that it was the need for a continued source of income
that forced the employees acceptance of the "por viaje" provision.
Having found that respondents are regular employees who may be, however, dismissed for cause as we have
so found in this case, there is a need to look into the procedural requirement of due process in Section 2, Rule
XXIII, Book V of the Rules Implementing the Labor Code. It is required that the employer furnish the employee
with two written notices: (1) a written notice served on the employee specifying the ground or grounds for
termination, and giving to said employee reasonable opportunity within which to explain his side; and (2) a
written notice of termination served on the employee indicating that upon due consideration of all the
circumstances, grounds have been established to justify his termination.

From the records, there was only one written notice which required respondents to explain within five (5) days
why they should not be dismissed from the service.

The twin requirements of notice and hearing constitute the elements of [due] process in cases of employee's
dismissal. The requirement of notice is intended to inform the employee concerned of the employer's intent to
dismiss and the reason for the proposed dismissal. Upon the other hand, the requirement of hearing affords the
employee an opportunity to answer his employer's charges against him and accordingly, to defend himself
44
therefrom before dismissal is effected. Obviously, the second written notice, as indispensable as the first, is
intended to ensure the observance of due process.

Given the fact that their dismissal was for just cause, we cannot grant backwages and separation pay to
respondents. However, following the findings of the Labor Arbiter who with the expertise presided over the
proceedings below, which findings were affirmed by the Court of Appeals, we grant the 13th month pay and
salary differential of the dismissed employees.

San Beda College of Law 516


4S: 2015 - 2016
LABOR LAW REVIEW Atty. Joyrich Golangco

JOSE Y. SONZA vs. ABS-CBN BROADCASTING CORPORATION
GR NO. 138051 JUNE 10, 2004
CARPIO

INDEPENDENT CONTRACTOR

DOCTRINE: Being an exclusive talent does not by itself mean that one is an employee. In the broadcast
industry, exclusivity is not necessarily the same as control.

FACTS: ABS-CBN signed an Agreement with Mel and Jay Management and Development Corporation
(MJMDC), as AGENT of Sonza, as President and GM, and Carmela Tiangco, as EVP and Treasurer. MJMDC
agreed to provide Sonzas services exclusively to ABS-CBN as talent for radio and television.

On April 1, 1996, Sonza wrote to ABS-CBNs President Eugenio Lopez III serving a notice of rescission of the
Agreement at their instance effective as of date thereby waiving and renouncing recovery of the remaining
amount stipulated but reserving the right to seek recovery to other benefits pursuant to Sonzas resignation in
view of recent events concerning his programs and career due to ABS-CBNs act and breach violative of the
Agreement.

Sonza filed a complaint against ABS-CBN before the DOLE, NCR in QC on the ground that ABS-CBN did not
th
pay his salaries, separation pay, service incentive leave pay, 13 month pay, signing bonus, travel allowance,
and amounts due under the Employees Stock Option Plan (ESOP). ABS-CBN filed a Motion to Dismiss on the
ground that no employer-employee relationship existed between the parties. Meanwhile ABS-CBN continued to
remit Sonzas monthly talent fees.

LA DECISION: Denied the motion to dismiss and directed the parties to file their position papers ruling that
Sonza, for having invoked a claim that he was an employee and was not paid certain claims, is sufficient
enough to confer jurisdiction over the case. On February 24, 1997, the position papers were submitted.

Sonza filed a Reply with Motion to Expunge the affidavits of ABS-CBNs witnesses Soccoro Vidanes and
Rolando Cruz which stated that the prevailing practice in the tv and broadcast industry is to treat talents as
independent contractors. LA dismissed the complaint for lack of jurisdiction on the ground that a talent cannot
be considered as an employee by reason of the peculiar circumstances surrounding the engagement of his
services: (1) He was free to perform the services he undertook to render in accordance with his own style, his
benefits were very much higher than those given to employees, and he was not bound to render 8 hours of work
per day. (2) Whatever benefits Sonza enjoyed arose from specific agreement by the parties and not by reason
of the employer-employee relationship. All these benefits are merely talent fees and other contractual benefits
and should not be deemed as salaries, wages, and/or other remuneration accorded to an employee,
notwithstanding the nomenclature appended to these benefits. Apropos to this is the rule that the term or
nomenclature given to a stipulated benefit is not controlling, but the intent of parties to the Agreement
conferring such benefit. (3) The fact that Sonza was made subject to ABS-CBNs Rules and Regulations does
not detract from the absence of employer-employee relationship as such merely served as guidelines toward the
achievement of the mutually desired result without dictating the means or methods to be employed in attaining
it.

NLRC DECISION: Affirmed the LAs decision.

ISSUE: Is Sonza an employee of ABS-CBN?

RULING: NO.

Independent contractors often present themselves to possess unique skills, expertise, or talent to distinguish
them from ordinary employees. The specific selection and hiring of Sonza because of his unique skills, talent,
and celebrity status not possessed by ordinary employees is a circumstance indicative, but not conclusive, of an
independent contractual relationship. If SONZA did not possess such unique skills, talent and celebrity status,
ABS- CBN would not have entered into the Agreement with SONZA but would have hired him through its
personnel department just like any other employee.

San Beda College of Law 517


4S: 2015 - 2016
LABOR LAW REVIEW Atty. Joyrich Golangco

All the talent fees and benefits paid to Sonza were the result of negotiations that led to the Agreement. If Sonza
were ABS-CBNs employee, there would be no need for the parties to stipulate on the benefits as such are
automatically incorporated into every employer-employee contract by the law. Whatever benefits Sonza enjoyed
arose from contract and not because of an employer-employee relationship. The payment of talent fees directly
to SONZA and not to MJMDC does not negate the status of SONZA as an independent contractor. The parties
expressly agreed on such mode of payment.

Sonza failed to show that ABS-CBN could terminate his services on grounds other than breach of contract, such
as retrenchment to prevent losses as provided under labor laws. During the life of the Agreement, even if it
suffered severe business losses, ABS-CBN could not retrench Sonza because it remained obligation to pay
Sonzas talent fees. This circumstance indicates an independent contractual relationship between Sonza and
ABS-CBN. Even after ABS-CBN ceased broadcasting his programs, ABS-CBN still paid him his talent fees.
Furthermore, while Sonza did actually resign from ABS-CBN, he also, as president of MJMDC, rescinded the
Agreement.

The control test is the most important test the courts apply in distinguishing an employee from an independent
contractor. It is based on the extent of control the hirer exercises over a worker. The greater the supervision and
control the hirer exercises, the more likely the worker is deemed an employee. The converse holds true as well
the less control the hirer exercises, the more likely the worker is considered as an independent contractor.

(5) Control over the means and methods of work. ABS-CBN engaged SONZAs services specifically to co-
host the Mel & Jay programs. ABS-CBN did not assign any other work to SONZA. To perform his
work, SONZA only needed his skills and talent. How SONZA delivered his lines, appeared on
television, and sounded on radio were outside ABS-CBNs control. SONZA did not have to render
eight hours of work per day. The Agreement required SONZA to attend only rehearsals and tapings of
the shows, as well as pre- and post-production staff meetings, as well as pre- and post-production staff
meetings. ABS-CBN could not dictate the contents of SONZAs script. However, the Agreement
prohibited SONZA from criticizing in his shows ABS-CBN or its interests. The clear implication is that
SONZA had a free hand on what to say or discuss in his shows provided he did not attack ABS-CBN
or its interests. ABS-CBN was not involved in the actual performance that produced the finished
product of Sonzas work. ABS-CBN merely reserved the right to modify the program format and airtime
schedule for more effective programming. ABS-CBNs sole concern was the quality of the shows
and their standing in the ratings. Clearly, ABS- CBN did not exercise control over the means
and methods of performance of SONZAs work. A radio broadcast specialist who works under
minimal supervision is an independent contractor.

(6) Subjected to rules and standards of performance. The Agreement stipulates the Sonza shall abide with
the rules and standards of performance covering talents of ABS-CBN. One could still be an
independent contractor although the hirer reserved certain supervision to insure the attainment of the
desired result. The hirer, however, must not deprive the one hired from performing his services
according to his own initiative.

(7) Exclusivity clause. Being an exclusive talent does not by itself mean that SONZA is an employee of
ABS-CBN. In the broadcast industry, exclusivity is not necessarily the same as control. The hiring of
exclusive talents is a widespread and accepted practice in the entertainment industry since the
broadcast station normally spends substantial amounts of money, time and effort in building up its
talents and programs they appear in.

Sonzas claim that the practice to treat talents as independent contractors is void for violating the right of labor to
security of tenure. The right of labor to security of tenure as guaranteed in the Constitution arises only if
there is an employer- employee relationship under labor laws. Not every performance of services for a
fee creates an employer- employee relationship. To hold that every person who renders services to
another for a fee is an employee - to give meaning to the security of tenure clause - will lead to absurd
results. The Court will not interpret the right of labor to security of tenure to compel artists and talents to render
their services only as employees. If radio and television program hosts can render their services only as
employees, the station owners and managers can dictate to the radio and television hosts what they say in their
shows. This is not conducive to freedom of the press.

San Beda College of Law 518


4S: 2015 - 2016
LABOR LAW REVIEW Atty. Joyrich Golangco

CONSOLIDATED BROADCASTING SYSTEM, INC. v. DANNY OBERIO, ELNA DE PEDRO, LUISITO
VILLAMOR, WILMA SUGATON, RUFO DEITA, JR., EMILY DE GUZMAN, CAROLINE LADRILLO, JOSE
ROBERTO REGALADO, ROSEBEL NARCISO & ANANITA TANGETE
G.R. No. 168424 June 8, 2007
YNARES-SANTIAGO

REGULAR EMPLOYEES

DOCTRINE: The test to determine whether employment is regular or not is the reasonable connection between
the particular activity performed by the employee in relation to the usual business or trade of the employer. Also,
if the employee has been performing the job for at least one year, even if the performance is not continuous or
merely intermittent, the law deems the repeated and continuing need for its performance as sufficient evidence
of the necessity, if not indispensability of that activity to the business.

FACTS: Respondents alleged that they were employed as drama talents by DYWB-Bombo Radyo, a radio
station owned and operated by petitioner Consolidated Broadcasting System, Inc. They reported for work daily
for six days in a week and were required to record their drama production in advance. Some of them were
employed by petitioner since 1974, while the latest one was hired in 1997.

Sometime in August 1998, petitioner reduced the number of its drama productions from 14 to 11, but was
opposed by respondents. After the negotiations failed, the latter sought the intervention of DOLE conducted
through its RO, an inspection of DWYB station. The results thereof revealed that petitioner is guilty of violation of
labor standard laws, such as underpayment of wages, 13th month pay, non-payment of service incentive leave
pay, and non-coverage of respondents under SSS.

Petitioner contended that respondents are not its employees and refused to submit the payroll and daily time
records despite the subpoena.

Respondents Oberio and Delta were suspended for minor lapses and the payment of their salaries were
purportedly delayed. Eventually, pending the outcome of the inspection case, respondents were barred by
petitioner from reporting for work; thus, the former claimed constructive dismissal.

DOLE RD issued an order directing petitioner to pay respondents a total of P318,986.74 representing non-
6
payment/underpayment of the salary and benefits due them. However, RD certified the records of the case to
the NLRC, Regional Arbitration Branch VI, for determination of employer-employee relationship. Respondents
appealed.

Respondents filed a case for illegal dismissal, underpayment/non-payment of wages and benefits plus damages
against petitioner.

LA DECISION: Dismissed the case without prejudice while waiting for the decision of the Secretary of Labor on
the same issue of the existence of an employer-employee relationship between petitioner and respondents.

NLRC DECISION: Held that respondents were regular employees of petitioner who were illegally dismissed by
the latter.

ISSUE: Are the respondents employees of the petitioner?

RULING: YES. Respondents employment with petitioner passed the "four-fold test" on employer-employee
relations, namely: (1) the selection and engagement of the employee, or the power to hire; (2) the payment of
wages; (3) the power to dismiss; and (4) the power to control the employee.

Petitioner failed to controvert with substantial evidence the allegation of respondents that they were hired by the
former on various dates from 1974 to 1997. If petitioner did not hire respondents and if it was the director alone
who chose the talents, petitioner could have easily shown, being in possession of the records, a contract to such
effect. However, petitioner merely relied on its contention that respondents were piece rate contractors who
17
were paid by results. Note that under Policy Instruction No. 40, petitioner is obliged to execute the necessary
contract specifying the nature of the work to be performed, rates of pay, and the programs in which they will
work. Moreover, project or contractual employees are required to be apprised of the project they will undertake
San Beda College of Law 519
4S: 2015 - 2016
LABOR LAW REVIEW Atty. Joyrich Golangco

under a written contract. This was not complied with by the petitioner, justifying the reasonable conclusion that
no such contracts exist and that respondents were in fact regular employees.

Petitioner failed to adduce an iota proof that the requirements for program employment were even complied with
by it. It is basic that project or contractual employees are appraised of the project they will work under a written
contract, specifying, inter alia, the nature of work to be performed and the rates of pay and the program in which
they will work. Sadly, however, no such written contract was ever presented by the petitioner. Petitioner is in the
best of position to present these documents. And because none was presented, we have every reason to
surmise that no such written contract was ever accomplished by the parties, thereby belying petitioners posture.

Worse, there was no showing of compliance with the requirement that after every engagement or production of
a particular television series, the required reports were filed with the proper government agency, as provided no
less under the very Policy Instruction invoked by the petitioner, nor under the Omnibus Implementing Rules of
the Labor Code for project employees. This alone bolsters respondents contention that they were indeed
petitioners regular employees since their employment was not only for a particular program.

Moreover, the engagement of respondents for a period ranging from 2 to 25 years and the fact that their drama
programs were aired not only in Bacolod City but also in the sister stations of DYWB in the Visayas and
Mindanao areas, undoubtedly show that their work is necessary and indispensable to the usual business or
trade of petitioner.

The test to determine whether employment is regular or not is the reasonable connection between the particular
activity performed by the employee in relation to the usual business or trade of the employer. Also, if the
employee has been performing the job for at least one year, even if the performance is not continuous or merely
intermittent, the law deems the repeated and continuing need for its performance as sufficient evidence of the
necessity, if not indispensability of that activity to the business.

Thus, even assuming that respondents were initially hired as project/contractual employees who were paid per
drama or per project/contract, the engagement of their services for 2 to 25 years justify their classification as
regular employees, their services being deemed indispensable to the business of petitioner. The respondents
were illegally dismissed.

San Beda College of Law 520


4S: 2015 - 2016
LABOR LAW REVIEW Atty. Joyrich Golangco

WILHELMINA S. OROZCO v. C.A.
G.R. No. 155207 August 13, 2008
NACHURA, J.

A COLUMNIST IN A NEWSPAPER IS NOT A REGULAR EMPLOYEE OF THE NEWSPAPER WHICH


PUBLISHES THE COLUMN.

DOCTRINE: The newspapers power to approve or reject publication of any specific article she wrote for her
column cannot be the control contemplated in the "control test," as it is but logical that one who commissions
another to do a piece of work should have the right to accept or reject the product. The important factor to
consider in the "control test" is still the element of control over how the work itself is done, not just the end result
thereof.
the so-called control as to time, space, and discipline are dictated by the very nature of the newspaper
business itself.

FACTS:
In March 1990, PDI engaged the services of petitioner to write a weekly column for its Lifestyle section. She
religiously submitted her articles every week, except for a six-month stint in New York City when she,
nonetheless, sent several articles through mail.

After two years, petitioners column was terminated by PDI. Petitioner filed a complaint for illegal dismissal.
Petitioner argues that PDI exercised control over her work as evidenced by the fact that she had to strictly follow
deadlines, her column must strictly be related to the Lifestyle section and her article must conform to the space
allotted in the said section.

ISSUE: Whether a columnist in a newspaper subjected to the afore-stated conditions is a regular employee of
the newspaper?

HELD: NO. Petitioner has misconstrued the "control test," as did the Labor Arbiter and the NLRC.

Not all rules imposed by the hiring party on the hired party indicate that the latter is an employee of the former.
Rules which serve as general guidelines towards the achievement of the mutually desired result are not
indicative of the power of control.

It should, however, be obvious that not every form of control that the hiring party reserves to himself over the
conduct of the party hired in relation to the services rendered may be accorded the effect of establishing an
employer-employee relationship between them in the legal or technical sense of the term. A line must be drawn
somewhere, if the recognized distinction between an employee and an individual contractor is not to vanish
altogether. Realistically, it would be a rare contract of service that gives untrammelled freedom to the party hired
and eschews any intervention whatsoever in his performance of the engagement.

Logically, the line should be drawn between rules that merely serve as guidelines towards the achievement of
the mutually desired result without dictating the means or methods to be employed in attaining it, and those that
control or fix the methodology and bind or restrict the party hired to the use of such means. The first, which aim
only to promote the result, create no employer-employee relationship unlike the second, which address both the
result and the means used to achieve it.

A careful examination reveals that the factors enumerated by the petitioner are inherent conditions in running a
newspaper. In other words, the so-called control as to time, space, and discipline are dictated by the very nature
of the newspaper business itself.

The newspapers power to approve or reject publication of any specific article she wrote for her column cannot
be the control contemplated in the "control test," as it is but logical that one who commissions another to do a
piece of work should have the right to accept or reject the product. The important factor to consider in the
"control test" is still the element of control over how the work itself is done, not just the end result thereof.

In contrast, a regular reporter is not as independent in doing his or her work for the newspaper. We note the
common practice in the newspaper business of assigning its regular reporters to cover specific subjects,
geographical locations, government agencies, or areas of concern, more commonly referred to as "beats." A
San Beda College of Law 521
4S: 2015 - 2016
LABOR LAW REVIEW Atty. Joyrich Golangco

reporter must produce stories within his or her particular beat and cannot switch to another beat without
permission from the editor. In most newspapers also, a reporter must inform the editor about the story that he or
she is working on for the day. The story or article must also be submitted to the editor at a specified time.
Moreover, the editor can easily pull out a reporter from one beat and ask him or her to cover another beat, if the
need arises.

This is not the case for petitioner. Although petitioner had a weekly deadline to meet, she was not precluded
from submitting her column ahead of time or from submitting columns to be published at a later time. More
importantly, respondents did not dictate upon petitioner the subject matter of her columns, but only imposed the
general guideline that the article should conform to the standards of the newspaper and the general tone of the
particular section.

Where a person who works for another performs his job more or less at his own pleasure, in the manner he
sees fit, not subject to definite hours or conditions of work, and is compensated according to the result of his
efforts and not the amount thereof, no employer-employee relationship exists.

San Beda College of Law 522


4S: 2015 - 2016
LABOR LAW REVIEW Atty. Joyrich Golangco

WILLIAM UY CONSTRUCTION CORP. v. TRINIDAD
G.R. No. 183250 March 10, 2010
ABAD, J.:

REPEATED HIRING IN THE CONSTRUCTION BUSINESS OF A DRIVER DOES NOT RESULT INTO
REGULAR EMPLOYMENT.

DOCTRINE: Generally, length of service provides a fair yardstick for determining when an employee initially
hired on a temporary basis becomes a permanent one, entitled to the security and benefits of regularization. But
this standard will not be fair, if applied to the construction industry, simply because construction firms cannot
guarantee work and funding for its payrolls beyond the life of each project. And getting projects is not a matter of
course. Construction companies have no control over the decisions and resources of project proponents or
owners. There is no construction company that does not wish it has such control but the reality, understood by
construction workers, is that work depended on decisions and developments over which construction companies
have no say.

FACTS:
Trinidad claimed that he had been working with the latter company for 16 years since 1988 as driver of its
service vehicle, dump truck, and transit mixer. He had signed several employment contracts with the company
that identified him as a project employee although he had always been assigned to work on one project after
another with some intervals.

In 2004 petitioner company terminated him from work after it shut down operations because of lack of
projects. He learned later, however, that although it opened up a project in Batangas, it did not hire him back for
that project.

Trinidad filed a case for illegal dismissal.

ISSUE: Whether repeated hiring in a construction business, for 16 years, will result into a status of regular
employment?

HELD: NO. Generally, length of service provides a fair yardstick for determining when an employee initially
hired on a temporary basis becomes a permanent one, entitled to the security and benefits of regularization. But
this standard will not be fair, if applied to the construction industry, simply because construction firms cannot
guarantee work and funding for its payrolls beyond the life of each project. And getting projects is not a matter of
course. Construction companies have no control over the decisions and resources of project proponents or
owners. There is no construction company that does not wish it has such control but the reality, understood by
construction workers, is that work depended on decisions and developments over which construction companies
have no say.

For this reason, the Court held in Caseres v. Universal Robina Sugar Milling Corporation that the repeated and
successive rehiring of project employees do not qualify them as regular employees, as length of service is not
the controlling determinant of the employment tenure of a project employee, but whether the employment has
been fixed for a specific project or undertaking, its completion has been determined at the time of the
engagement of the employee.

In this case, respondent Trinidads series of employments with petitioner company were co-terminous with its
projects. When its Boni Serrano-Katipunan Interchange Project was finished in December 2004, Trinidads
employment ended with it. He was not dismissed. His employment contract simply ended with the project for
which he had signed up. His employment history belies the claim that he continuously worked for the
company. Intervals or gaps separated one contract from another.
Parenthetically, the Social Security System should be able to alleviate the temporary unemployment of
construction workers, a problem that is inherent in the nature of their work.

San Beda College of Law 523


4S: 2015 - 2016
LABOR LAW REVIEW Atty. Joyrich Golangco

D.M. CONSUNJI V. JAMIN
G.R. No. 192514. April, 18, 2012
BRION, J.:

REPEATED AND CONTINUOUS HIRING OF A CARPENTER IN A CONSTRUCTION BUSINESS, FOR 31


YEARS, RESULTS INTO REGULAR EMPLOYMENT.

DOCTRINE: Surely, length of time is not the controlling test for project employment. Nevertheless, it is vital in
determining if the employee was hired for a specific undertaking or tasked to perform functions vital, necessary
and indispensable to the usual business or trade of the employer. Here, [private] respondent had been a project
employee several times over. His employment ceased to be coterminous with specific projects when he was
repeatedly re-hired due to the demands of petitioners business.

FACTS:
On December 17, 1968, petitioner D.M. Consunji, Inc. (DMCI), a construction company, hired respondent
Estelito L. Jamin as a laborer. Sometime in 1975, Jamin became a helper carpenter. Since his initial hiring,
Jamins employment contract had been renewed a number of times. On March 20, 1999, his work at DMCI was
terminated due to the completion of the SM Manila project. This termination marked the end of his employment
with DMCI as he was not rehired again. He filed a complaint for illegal dismissal.

DMCI denied liability. It argued that it hired Jamin on a project-to-project basis, from the start of his engagement
in 1968 until the completion of its SM Manila project on March 20, 1999 where Jamin last worked. With the
completion of the project, it terminated Jamins employment. It alleged that it submitted a report to the
Department of Labor and Employment (DOLE) everytime it terminated Jamins services.

ISSUE: Whether continuous and repeated hiring of a carpenter in a construction business, for 38 years, will
result into regular employment?

HELD: YES. In Liganza v. RBL Shipyard Corporation, the Court held that [a]ssuming, without granting[,] that
[the] petitioner was initially hired for specific projects or undertakings, the repeated re-hiring and
continuing need for his services for over eight (8) years have undeniably made him a regular employee.

We reviewed Jamins employment contracts as the CA did and we noted that while the contracts indeed show
that Jamin had beenengaged as a project employee, there was an almost unbroken string of Jamins rehiring
from December 17, 1968 up to the termination of his employment on March 20, 1999.

To reiterate, Jamins employment history with DMCI stands out for his continuous, repeated and successive
rehiring in the companys construction projects. In all the 38 projects where DMCI engaged Jamins services, the
tasks he performed as a carpenter were indisputably necessary and desirable in DMCIs construction business.
He might not have been a member of a work pool as DMCI insisted that it does not maintain a work pool, but his
continuous rehiring and the nature of his work unmistakably made him a regular employee. In Maraguinot, Jr. v.
NLRC, the Court held that once a project or work pool employee has been: (1) continuously, as opposed to
intermittently, rehired by the same employer for the same tasks or nature of tasks; and (2) these tasks are vital,
necessary and indispensable to the usual business or trade of the employer, then the employee must be
deemed a regular employee.

San Beda College of Law 524


4S: 2015 - 2016
LABOR LAW REVIEW Atty. Joyrich Golangco

WILFREDO ARO, RONILO TIROL, JOSE PACALDO, PRIMITIVO CASQUEJO and MARCIAL ABGO VS.
NATIONAL LABOR RELATIONS COMMISSION AND BENTHEL DEVELOPMENT CORPORATION
G.R. No. 174792 March 7, 2012
PERALTA, J.:

PROJECT EMPLOYEES

FACTS:
Petitioners, employees of private respondent Benthel Development Corporation (BDC), filed a Complaint for
illegal dismissal with various money claims and prayer for damages against the latter. NLRC rendered a
decision finding private respondent guilty of illegal dismissal and ordering it to pay employees separation pay.
Upon appeal of petitioners, the decision was affirmed with modification that private respondent pay backwages
computed from the respective dates of dismissal until finality of the decision. BDC filed a motion for
reconsideration with the contention that, since it has been found by the Labor Arbiter and affirmed in the
assailed decision that the employees were project employees, the computation of backwages should be limited
to the date of the completion of the project and not to the finality of the decision. DENIED. Herein petitioner
sought for the execution of the decision, questioned by the BDC. In this petition respondent alleged that the
petitioners are regular employees and not project employees. This is because the NLRC ruled that the
petitioners are project employees. However in the order of the NLRC that petitioners are entitled to backwages
up to the finality of its decision, when as project employees, private respondents are only entitled to payment of
backwages until the date of the completion of the project. In a later resolution on private respondent's motion for
reconsideration of its January 12, 1999 decision, the NLRC changed its findings by ruling that petitioners herein
were regular employees and, therefore, entitled to full backwages, until finality of the decision, citing that
petitioners repeated rehiring over a long span of time made them regular employees.

ISSUE: From the confusing ruling and order of the NLRC, are the petitioners project employees?

RULING: Yes. The Court agrees with the findings of the CA that petitioners were project employees. It is not
disputed that petitioners were hired for the construction of the Cordova Reef Village Resort in Cordova,
Cebu. By the nature of the contract alone, it is clear that petitioners' employment was to carry out a specific
project. Hence, the CA did not commit grave abuse of discretion when it affirmed the findings of the Labor
Arbiter. Therefore, being project employees, petitioners are only entitled to full backwages, computed from the
date of the termination of their employment until the actual completion of the work. Illegally dismissed workers
are entitled to the payment of their salaries corresponding to the unexpired portion of their employment where
the employment is for a definite period. In this case, as found by the CA, the Cordova Reef Village Resort
project had been completed in October 1996 and private respondent herein had signified its willingness, by way
of concession to petitioners, to set the date of completion of the project as March 18, 1997; hence, the latter
date should be considered as the date of completion of the project for purposes of computing the full backwages
of petitioners.

San Beda College of Law 525


4S: 2015 - 2016
LABOR LAW REVIEW Atty. Joyrich Golangco

UNIVERSAL ROBINA SUGAR MILLING CORPORATION and RENE CABATI vs. FERDINAND ACIBO
G.R. No. 186439 January 15, 2014
BRION, J.:

REGULAR EMPLOYEES

FACTS:
PETITIONER Universal Robina Sugar Milling Corp. (URSUMCO) is engaged in the sugarcane milling business.
Respondents Ferdinand Acibo and 21 others were hired by the petitioner on various dates, in different
capacities. At the start of their engagements, they signed contracts of employment for a period of one month or
for a given season. URSUMCO repeatedly hired them to perform the same duties and for every engagement
required them to sign new employment contracts for the same duration.

Last Oct. 23, 2002, the respondents filed before the labor arbiter complaints for regularization, entitlement to the
benefits under the Collective Bargaining Agreement (CBA) and attorneys fees. The labor arbiter (LA) dismissed
the complaint for lack of merit.

The National Labor Relations Commission (NLRC) reversed the LAs ruling and declared respondents as
regular URSUMCO employees and granted their monetary claims under the CBA. The Court of Appeals (CA)
affirmed the NLRCs ruling finding respondents to be regular employees of URSUMCO but deleted the grant of
monetary benefits under the CBA. Did the CA err?

ISSUE: WON they are considered regular employees.

RULING: Yes. Under the system, the plantation workers or the mill employees do not work continuously for one
whole year but only for the duration of the growing of the sugarcane or the milling season. Their seasonal work,
however, does not detract from considering them in regular employment since in a litany of cases, this Court
has already settled that seasonal workers who are called to work from time to time and are temporarily laid off
during the offseason are not separated from the service in said period, but are merely considered on leave until
reemployment. Be this as it may, regular seasonal employees, like the respondents in this case, should not be
confused with the regular employees of the sugar mill such as the administrative or office personnel who
perform their tasks for the entire year regardless of the season. The NLRC, therefore, gravely erred when it
declared the respondents regular employees of URSUMCO without qualification and that they were entitled to
the benefits granted, under the CBA, to URSUMCOS regular employees.

San Beda College of Law 526


4S: 2015 - 2016
LABOR LAW REVIEW Atty. Joyrich Golangco

GMA NETWORK, INC., vs. CARLOS P. PABRIGA et.al
G.R. No. 176419 November 27, 2013
LEONARDO-DE CASTRO, J.:

REGULAR EMPLOYEES

FACTS: RESPONDENTS Carlos P. Pabriga and four others were engaged by petitioner GMA Network, Inc. to
perform the following activities: 1) manning of technical operations center: (a) responsibility for airing of local
commercials, and (b) logging/monitoring of national commercials (satellite); 2) acting as transmitter/VTR men:
(a) prepare tapes for local airing, (b) actual airing of commercials, (c) plugging of station promo, (d) logging of
transmitter reading, and (e) in case of power failure, start up generator set to resume program; 3) acting as
maintenance staff: (a) checking of equipment, (b) warming up of generator, (c) filling of oil, fuel, and water in
radiator, and 4) acting as cameramen.

In a complaint for illegal dismissal, the petitioner interchangeably characterized respondents employment as
project and fixed-period/fixed-term employment. The National Labor Relations Commission (NLRC), in reversing
the decision of the labor arbiter, held that they are regular employees.

ISSUE: Is Pabrigo a regular employee?

RULING: YES. In order to safeguard the rights of workers against the arbitrary use of the word project to
prevent employees from attaining the status of regular employees, employers claiming that their workers are
project employees should not only prove that the duration and scope of the employment was specified at the
time they were engaged, but also that there was indeed a project. As discussed above, the project could either
be (1) a particular job or undertaking that is within the regular or usual business of the employer company, but
which is distinct and separate, and identifiable as such, from the other undertakings of the company; or (2) a
particular job or undertaking that is not within the regular business of the corporation. As it was with regard to
the distinction between a regular and casual employee, the purpose of this requirement is to delineate whether
or not the employer is in constant need of the services of the specified employee. If the particular job or
undertaking is within the regular or usual business of the employer company and it is not identifiably distinct or
separate from the other undertakings of the company, there is clearly a constant necessity for the performance
of the task in question, and therefore said job or undertaking should not be considered a project.These jobs and
undertakings are clearly within the regular or usual business of the employer company and are not identifiably
distinct or separate from the other undertakings of the company. There is no denying that the manning of the
operations center to air commercials, acting as transmitter/VTR men, maintaining the equipment, and acting as
cameramen are not undertakings separate or distinct from the business of a broadcasting company.

San Beda College of Law 527


4S: 2015 - 2016
LABOR LAW REVIEW Atty. Joyrich Golangco

ROY D. PASOS vs. PHILIPPINE NATIONAL CONSTRUCTION CORPORATION
G.R. No. 192394 July 03, 2013
VILLARAMA, JR., J.:

DOCTRINE: The principal test used to determine whether employees are project employees is whether or not
the employees were assigned to carry out a specific project or undertaking, the duration or scope of which was
specified at the time the employees were engaged for that project.

FACTS: Petitioner Roy D. Pasos started working for respondent PNCC on April 26, 1996. He was designated
as "Clerk II (Accounting)" and was assigned to the "NAIA II Project." It is stated in his employment contract
that: his contract maybe terminated at anytime for cause as provided for by law and/or existing Company Policy.
This maybe terminated if services are unsatisfactory, or when it shall no longer needed, as determined by the
Company. If services are still needed beyond the validity of this contract, the Company shall extend your
services. After services are terminated, the employee shall be under no obligation to re-employ with the
Company nor shall the Company be obliged to re-employ the employee. Petitioners employment, however, did
not end on July 25, 1996 but was extended until August 4, 1998, or more than two years later, based on the
"Personnel Action Form Project Employment" dated July 7, 1998. Despite the termination of his employment
on October 19, 2000, petitioner claims that his superior instructed him to report for work the following day,
intimating to him that he will again be employed for the succeeding SM projects. For purposes of reemployment,
he then underwent a medical examination which allegedly revealed that he had pneumonitis. Petitioner was
advised by PNCCs physician, Dr. Arthur C. Obena, to take a 14-day sick leave. After serving his sick leave,
petitioner claims that he was again referred for medical examination where it was revealed that he contracted
13
Kochs disease. He was then required to take a 60-day leave of absence. The following day, he submitted his
application for sick leave but PNCCs Project Personnel Officer, Mr. R.S. Sanchez, told him that he was not
entitled to sick leave because he was not a regular employee. Petitioner claims that after he presented his
medical clearance to the Project Personnel Officer on even date, he was informed that his services were already
terminated on October 19, 2000 and he was already replaced due to expiration of his contract. This prompted
14
petitioner on February 18, 2003 to file a complaint for illegal dismissal against PNCC with a prayer for
reinstatement and back wages. He argued that he is deemed a regular employee of PNCC due to his prolonged
employment as a project employee as well as the failure on the part of PNCC to report his termination every
time a project is completed. He further contended that his termination without the benefit of an administrative
investigation was tantamount to an illegal dismissal.

LA RULING: complainant had attained regular employment thereby making his termination from employment
illegal since it was not for any valid or authorized causes.

Petitioner submits that the CA erroneously concluded that he was a project employee when there are indicators
which point otherwise. He contends that even if he was just hired for the NAIA 2 Project from April 26, 1996 to
July 25, 1996, he was made to work until August 4, 1998. He also avers the DOLE had certified that he was not
among the employees listed in the termination reports submitted by PNCC which belies the photocopies of
termination reports attached by PNCC to its pleadings listing petitioner as one of the affected employees

ISSUE: Whether petitioner is a regular employee

RULING: This Court is convinced however that although he started as a project employee, he eventually
became a regular employee of PNCC. The principal test used to determine whether employees are project
employees is whether or not the employees were assigned to carry out a specific project or undertaking, the
duration or scope of which was specified at the time the employees were engaged for that project. In the case at
bar, petitioner worked continuously for more than two years after the supposed three-month duration of his
project employment for the NAIA II Project. While his appointment for said project allowed such extension since
it specifically provided that in case his "services are still needed beyond the validity of the contract, the
Company shall extend his services," there was no subsequent contract or appointment that specified a particular
duration for the extension. His services were just extended indefinitely until "Personnel Action Form Project
Employment" dated July 7, 1998 was issued to him which provided that his employment will end a few weeks
later or on August 4, 1998. While for first three months, petitioner can be considered a project employee of
PNCC, his employment thereafter, when his services were extended without any specification of as to the
duration, made him a regular employee of PNCC. And his status as a regular employee was not affected by the
fact that he was assigned to several other projects and there were intervals in between said projects since he
enjoys security of tenure.
San Beda College of Law 528
4S: 2015 - 2016
LABOR LAW REVIEW Atty. Joyrich Golangco

JAIME N. GAPAYAO vs. ROSARIO FULO, SOCIAL SECURITY SYSTEM and SOCIAL SECURITY
COMMISSION
G.R. No. 193493 June 13, 2013
SERENO, CJ:

FACTS:
On 4 November 1997, Jaime Fulo (deceased) died of "acute renal failure secondary to 1st degree burn 70%
secondary electrocution"5 while doing repairs at the residence and business establishment of petitioner located
at San Julian, Irosin, Sorsogon.
Allegedly moved by his Christian faith, petitioner extended some financial assistance to private respondent. On
16 November 1997, the latter executed an Affidavit of Desistance6 stating that she was not holding them liable
for the death of her late husband, Jaime Fulo, and was thereby waiving her right and desisting from filing any
criminal or civil action against petitioner.

Both parties executed a Compromise Agreement wherein P40,000 was given as a complete and full payment
for all claims due the victim and that private respondent discharge the employer from any and all claims that
maybe due the victim in connection with the victims employment thereat.

Thereafter, private respondent filed a claim for social security benefits with the Social Security System (SSS)
Sorosogon Branch.8 However, upon verification and evaluation, it was discovered that the deceased was not a
registered member of the SSS.

Investigation by SSS showed that deceased is an employee of petitioner and hence held petitioner liable for
unpaid contributions due on behalf of deceased Fulo and penalties and surcharges

Petitioner claims that Fulo is not an employee but was rather an independent contractor whose tasks were not
subject to petitioners control and supervision.14 Assuming arguendo that the deceased was petitioners
employee, he was still not entitled to be paid his SSS premiums for the intervening period when he was not at
work, as he was an "intermittent worker who was only summoned every now and then as the need arose."15
Hence, petitioner insisted that he was under no obligation to report the formers demise to the SSS for social
security coverage.

ISSUE: Whether or not Fulo is an employee of petitioner hence making him liable for SSS contributions

HELD:
Jurisprudence has identified the three types of employees mentioned in the provision: (1) regular employees or
those who have been engaged to perform activities that are usually necessary or desirable in the usual business
or trade of the employer; (2) project employees or those whose employment has been fixed for a specific project
or undertaking, the completion or termination of which has been determined at the time of their engagement, or
those whose work or service is seasonal in nature and is performed for the duration of the season; and (3)
casual employees or those who are neither regular nor project employees.55

Farm workers generally fall under the definition of seasonal employees. We have consistently held that
seasonal employees may be considered as regular employees.56 Regular seasonal employees are those called
to work from time to time. The nature of their relationship with the employer is such that during the off season,
they are temporarily laid off; but reemployed during the summer season or when their services may be
needed.57 They are in regular employment because of the nature of their job,and not because of the length of
time they have worked.58

A reading of the records reveals that the deceased was indeed a farm worker who was in the regular employ of
petitioner. From year to year, starting January 1983 up until his death, the deceased had been working on
petitioners land by harvesting abaca and coconut, processing copra, and clearing weeds. His employment was
continuous in the sense that it was done for more than one harvesting season. Moreover, no amount of
reasoning could detract from the fact that these tasks were necessary or desirable in the usual business of
petitioner.

The other tasks allegedly done by the deceased outside his usual farm work only bolster the existence of an
employer-employee relationship. As found by the SSC, the deceased was a construction worker in the building

San Beda College of Law 529


4S: 2015 - 2016
LABOR LAW REVIEW Atty. Joyrich Golangco

and a helper in the bakery, grocery, hardware, and piggery all owned by petitioner.63 This fact only proves
that even during the off season, the deceased was still in the employ of petitioner.

The most telling indicia of this relationship is the Compromise Agreement executed by petitioner and private
respondent. It is a valid agreement as long as the consideration is reasonable and the employee signed the
waiver voluntarily, with a full understanding of what he or she was entering into. All that is required for the
compromise to be deemed voluntarily entered into is personal and specific individual consent. Once executed
by the workers or employees and their employers to settle their differences, and done in good faith, a
Compromise Agreement is deemed valid and binding among the parties.

Petitioner entered into the agreement with full knowledge that he was described as the employer of the
deceased. This knowledge cannot simply be denied by a statement that petitioner was merely forced or
threatened into such an agreement. His belated attempt to circumvent the agreement should not be given any
consideration or weight by this Court.

San Beda College of Law 530


4S: 2015 - 2016
LABOR LAW REVIEW Atty. Joyrich Golangco

MILLENNIUM ERECTORS CORPORATION vs. VIRGILIO MAGALLANES
G.R. No. 184362 November 15, 2010
CARPIO MORALES, J.:

REGULAR EMPLOYEE

DOCTRINE: A project employee is one whose "employment has been fixed for a specific project or undertaking,
the completion or termination of which has been determined at the time of the engagement of the employee or
where the work or service to be performed is seasonal in nature and the employment is for the duration of the
season."

FACTS: Respondent Virgilio Magallanes started working in 1988 as a utility man for Laurencito Tiu, Chief
Executive Officer of Millennium Erectors Corporation, Tius family, and Kenneth Construction Corporation. He
was assigned to different construction projects undertaken by petitioner in Metro Manila, the last of which was
for a building in Libis, Quezon City. In July of 2004 he was told not to report for work anymore allegedly due to
old age, prompting him to file on August 6, 2004 an illegal dismissal complaint1 before the Labor Arbiter.
Petitioner claimed that respondent was a project employee whom it hired for a building project in Libis on
January 30, 2003, to prove which it submitted the employment contract3 signed by him; that on August 3, 2004,
respondents services were terminated as the project was nearing completion; and he was given financial
assistance4 in the amount of P2,000, for which he signed a quitclaim and waiver.

Labor Arbiter ruled in favor of petitioner and dismissed the complaint, holding that respondent knew of the
nature of his employment as a project employee, he having executed an employment contract specifying therein
the name of and duration of the project from January 2003 until its completion

NLRC: respondent was a regular, not a project employee, as the employment contract he supposedly signed
contained the date of commencement but not a specific date when it would end, contrary to the rule that the
duration and scope of similar contracts should be clearly set forth therein; and that based on the payrolls8
petitioner submitted and contrary to its claim that respondent was hired in January 2003, he had been employed
in 2001, not 2003, lending weight to his claim that he had worked for petitioner for 16 years prior to the filing of
his complaint.

ISSUE: Is respondent a regular employee?

RULING: Yes, respondent was a regular, not a project employee. As the Court has consistently held, the
service of project employees are coterminus [sic] with the project and may be terminated upon the end or
completion of that project or project phase for which they were hired. Regular employees, in contrast, enjoy
security of tenure and are entitled to hold on to their work or position until their services are terminated by any of
the modes recognized under the Labor Code. Petitioners various payrolls dating as early as 2001 show that
respondent had been employed by it. As aptly observed by the appellate court, these documents, rather than
sustaining petitioners argument, only serve to support respondents contention that he had been employed in
various projects, if not for 16 years, at the very least two years prior to his dismissal. Assuming arguendo that
petitioner hired respondent initially on a per project basis, his continued rehiring, as shown by the sample
payrolls converted his status to that of a regular employee. Following Cocomangas Beach Hotel Resort v.
Visca,16 the repeated and continuing need for respondents services is sufficient evidence of the necessity, if
not indispensability, of his services to petitioner's business and, as a regular employee, he could only be
dismissed from employment for a just or authorized cause.

San Beda College of Law 531


4S: 2015 - 2016
LABOR LAW REVIEW Atty. Joyrich Golangco

CAPAROSO, QUINDIPAN VERSUS COURT OF APPEALS
G.R. No. 155505 February 15, 2007
CARPIO, J.:

DOCTRINE:
Art. 280. Regular and Casual Employment. - The provisions of written agreement to the contrary
notwithstanding and regardless of the oral agreement of the parties, an employment shall be deemed to be
regular where the employee has been engaged to perform activities which are usually necessary or desirable in
the usual business or trade of the employer, except where the employment has been fixed for a specific
project or undertaking the completion or termination of which has been determined at the time of the
engagement of the employee or where the work or services to be performed is seasonal in nature and
the employment is for the duration of the season.

FACTS:
Emilio Caparoso and Joeve Quindipan were Composite Enterprises Incorporated (Composite)s delivery men.
Caparoso was hired on November 8, 1998 while Quindipan was hired on an intermittent basis since 1997.
Quindipan alleged that he was working continuously with Composite since August 1998.

They were dismissed by Composite, alleging that Caparoso and Quindipan were hired for three months, and
then on a month-to-month basis. Their termination was a result from the expiration of their contracts of
employment.

LA: Ruled that petitioners are regular employees of Composite, rendering them illegally dismissed by the latter
and ordered to immediately reinstate them from their respective former positions without loss of seniority rights
and other privileges. LA ruled that by the nature of company engaged in the distribution of supply of
confectionaries, there is no doubt as to the status of the delivery men.

NLRC: Set aside the LAs decision and ruled that the mere fact that the employees duties are necessary or
desirable in the business or trade of the employer does not mean that they are forbidden from stipulating the
period of employment. The contracts are binding and valid between the contracting parties and shall be
considered as the law between them.

CA: The court dismissed the petition and affirmed the NLRC Decision and held that the manpower requirement
of Composite varies from month to month depending on the demand from their clients for their products. Their
manpower requirement determines the period of their employees services, addressing a temporary manpower
shortage.

ISSUE: Whether or not Caparoso and Quindipan were regular employees, subsequently dismissed illegally.

HELD: Under Article 280 of the Labor Code, a regular employee is (1) one who is engaged to perform activities
that are necessary or desirable in the usual trade or business of the employer, or (2) a casual employee who
has rendered at least one year of service, whether continuous or broken, with respect to the activity in which he
is employed. However, even if an employee is engaged to perform activities that are necessary or desirable in
the usual trade or business of the employer, it does not preclude the fixing of employment for a definite period.

Accordingly, and since the entire purpose behind the development of legislation culminating in the present
Article 280 of the Labor Code clearly appears to have been, as already observed, to prevent circumvention of
the employees right to be secure in his tenure, the clause in said article indiscriminately and completely ruling
out all written or oral agreements conflicting with the concept of regular employment as defined therein should
be construed to refer to the substantive evil that the Code itself has singled out: agreements entered into
precisely to circumvent security of tenure. It should have no application to instances where a fixed period of
employment was agreed upon knowingly and voluntarily by the parties, without any force, duress or improper
pressure being brought to bear upon the employee and absent any other circumstances vitiating his consent, or
where it satisfactorily appears that the employer and employee dealt with each other on more or less equal
terms with no moral dominance whatever being exercised by the former over the latter. Unless thus limited in its
purview, the law would be made to apply to purposes other than those explicitly stated by its framers; it thus
become pointless and arbitrary, unjust in its effects and apt to lead to absurd and unintended consequences.

San Beda College of Law 532


4S: 2015 - 2016
LABOR LAW REVIEW Atty. Joyrich Golangco

The Court thus laid down the criteria under which fixed-term employment could not be said to be in
circumvention of the law on security of tenure, thus:

1. The fixed period of employment was knowingly and voluntarily agreed upon by the parties without
any force, duress, or improper pressure being brought to bear upon the employee and absent any other
circumstances vitiating his consent; or

2. It satisfactorily appears that the employer and the employee dealt with each other on more or less
equal terms with no moral dominance exercised by the former or the latter.

The Court of Appeals noted that there was no indication of force, duress, or improper pressure exerted on
petitioners when they signed the contracts. Further, there was no proof that respondents were regularly
engaged in hiring workers for work for a minimum period of five months to prevent the regularization of their
employees. Petitioners Employment is akin to Probationary Employment. Their fixed-term employment
contracts had expired and they were not illegally dismissed.

San Beda College of Law 533


4S: 2015 - 2016
LABOR LAW REVIEW Atty. Joyrich Golangco

SPOUSES ALWYN ONG LIM and EVELYN LUKANG LIM v. LEGAZPI HOPE CHRISTIAN SCHOOL
G.R. No. 172818 March 31, 2009
J. QUISUMBING

DOCTRINE:
For a private school teacher to acquire permanent status in employment, the following requisites must concur:
(1) the teacher is a full-time teacher; (2) the teacher must have rendered three consecutive years of service; and
(3) such service must have been satisfactory.

FACTS:

Petitioners Alwyn Ong Lim and Evelyn Lukang Lim were hired in June 1999 as professors in Legazpi Hope
Christian School. Helen Sia, head teacher of the schools Chinese department, verbally informed them that their
employment were to be terminated, without giving reasons therefor. Spouses Lim filed for illegal dismissal and
monetary claims against the school. Ramon Sia, Vice Chairman of the Schools Board of Directors, sent a letter
to petitioners stating that their three-year probation had expired and that the school management had decided to
discontinue their employment.

Spouses Lim contend that they were not issued any formal written probationary contract and neither were they
informed of reasonable standards under which they would be evaluated or rated in connection with their
supposed probationary period of employment. In the absence of such, they became regular and permanent
teaching personnel of the school. Petitioners further claim that they are full-time, not part-time, teaching
personnel. They claim to have no other outside remunerative occupation requiring regular hours of work that will
conflict with the working hours of the respondent school and in addition to their teaching jobs, they were
performing non-teaching functions like preparing lesson plans, checking of notebooks and test papers, assisting
during enrolment period, attending to school programs and other tasks. They were required to report as early as
7 a.m. until their respective classes ended.

On the other hand, respondents argue that under the Manual of Regulations for Private Schools, a full-time
instructor is one who has a teaching load of at least 15 hours a week or is paid on a full salary basis, while a
part-time instructor is one who has a teaching load of less than 15 hours a week. Thus, according to
respondents, since petitioners have a teaching load that is less than 15 hours a week then they are only part-
time instructors and do not enjoy security of tenure.

LA: Ruled in favor of Spouses Lim and ordered for their reinstatement to their former position without loss of
seniority rights.

NLRC: Found that Spouses Lim were only part-time teachers who did not acquire permanent status, hence their
dismissal is legal.

CA: Affirmed the decision of the NLRC

ISSUE: Whether or not Spouses Lim are regular employees

HELD: In University of Sto. Tomas v. NLRC, we ruled that for a private school teacher to acquire permanent
status in employment, the following requisites must concur: (1) the teacher is a full-time teacher; (2) the teacher
must have rendered three consecutive years of service; and (3) such service must have been satisfactory.

The burden is on petitioners to prove their affirmative allegation that they are permanent teaching personnel.
However, there is not enough evidence on record to show that their total working day is devoted to the school.
There is no showing of what the regular work schedule of a regular teacher in respondent school is. What is
clear in the records is that Evelyn and Alwyn spent two hours and four hours, respectively, but not the entire
working day, at the respondent school. They do not meet requirement c of Section 45 of the Manual. Hence, we
sustain the findings of the Court of Appeals that the petitioners are part-time teachers. Being part-time teachers,
in accordance with University of Sto. Tomas v. NLRC, they cannot acquire permanent status.

San Beda College of Law 534


4S: 2015 - 2016
LABOR LAW REVIEW Atty. Joyrich Golangco

DM CONSUNJI v. GOBRES
G.R. No. 169170 August 8, 2010
J. PERALTA

DOCTRINE:
In cases of project employment or employment covered by legitimate contracting or sub-contracting
arrangements, no employee shall be dismissed prior to the completion of the project or phase thereof for which
the employee was engaged, or prior to the expiration of the contract between the principal and contractor,
unless the dismissal is for just or authorized cause subject to the requirements of due process or prior notice, or
is brought about by the completion of the phase of the project or contract for which the employee was engaged.

FACTS: Gobres, Dalisay, Paragsa, Aleta and Melo worked as carpenters for DM Consunji, on several
occasions or/at various times. Their termination was reported to DOLE in accordance with Policy Instruction No.
20, superseded by DO No. 19, series of 1993. Gobres, et al, saw their names in the Notice of Termination on
the bulletin board at the project premises in Makati. They filed a complaint for illegal dismissal, non-payment of
th
13th mon pay, 5 days service incentive leave pay, damages and attorneys fees.

DM Consunji, on the other hand, countered that they were employed per project and within varying estimated
periods indicated in their respective project employment contracts. They further averred that respondents
services were terminated when their phases of work for which their services were engaged were completed or
when the projects were completed.

LA: Dismissed the complaint and found that they were project employees and that the requirements of
termination of services to DOLE were in accordance with the requirements of law.

NLRC: Affirmed the decision of the Labor Arbiter

CA: Sustained the finding of NLRC in holding respondents as project employees stating that the latter correctly
applied Article 280 of the Labor Code when it ruled that petitioners employment, which is fixed for [a] specific
project and the completion of which has been determined at the time that their services were engaged, makes
them project employees. As could be gleaned from the last portion of Article 280 of the Labor Code, the nature
of employment of petitioners, which is fixed for a specific project and the completion of which has been
determined when they were hired, is excepted therefrom.

However, CA stated that although they were project employees, they were entitled to know the reason for their
dismissal and to be heard on whatever claims they might have. Citing Agabon versus NLRC, which held that
where the dismissal is for a just cause, the lack of statutory due process should not nullify the dismissal, or
render it illegal, or ineffectual, but the employer should indemnify the employee for the violation of his statutory
rights by paying nominal damages. Hence, the Court of Appeals ordered petitioner and David M. Consunji to
pay respondents P20,000.00 each as nominal damages for lack of advance notice of their termination.

ISSUE: Whether or not as project employees, they are entitled for nominal damages for lack of advance notice
of dismissal

HELD: Records show that respondents were dismissed after the expiration of their respective project
employment contracts, and due to the completion of the phases of work respondents were engaged for. Hence,
the cited provisions requirements of due process or prior notice when an employee is dismissed for just or
authorized cause (under Articles 282 and 283 of the Labor Code) prior to the completion of the project or phase
thereof for which the employee was engaged do not apply to this case.

In this case, the Labor Arbiter, the NLRC and the Court of Appeals all found that respondents were validly
terminated due to the completion of the phases of work for which respondents services were engaged. If the
termination is brought about by the completion of the contract or phase thereof, no prior notice is
required. Cioco, Jr. v. C.E. Construction Corporation explained that this is because completion of the work or
project automatically terminates the employment, in which case, the employer is, under the law, only obliged to
render a report to the DOLE on the termination of the employment.

Hence, prior or advance notice of termination is not part of procedural due process if the termination is brought
about by the completion of the contract or phase thereof for which the employee was engaged. Petitioner,
San Beda College of Law 535
4S: 2015 - 2016
LABOR LAW REVIEW Atty. Joyrich Golangco

therefore, did not violate any requirement of procedural due process by failing to give respondents advance
notice of their termination; thus, there is no basis for the payment of nominal damages.

San Beda College of Law 536


4S: 2015 - 2016
LABOR LAW REVIEW Atty. Joyrich Golangco

YOLANDA M. MERCADO, CHARITO S. DE LEON, DIANA R. LACHICA, MARGARITO M. ALBA, JR., and
FELIX A. TONOG v. AMA COMPUTER COLLEGE-PARAAQUE CITY, INC.
G.R. No. 183572 April 13, 2010
BRION, J.:

FACTS:
AMA Computer College-Paranaque City, Inc. (AMACC) is an educational institution engaged in computer-based
education in the country. The petitioners were faculty members who started teaching at AMACC on 25 May
1998. The petitioners executed individual Teachers Contracts for each of the trimesters they were engaged.
AMACC implemented a new faculty screening guidelines for the school year 2000-2001. The performance
standards under the new screening guidelines were also used to determine the present members entitlement to
salary increase. AMACC did not give the petitioners any salary increase because they failed to obtain a passing
rating based on the performance standards.

The petitioners filed a complaint with the Arbitration Branch of the NLRC for underpayment of wages non-
payment of overtime and overload compensation, 13th month pay, and for discriminatory practices. They were
informed by AMACC that with the expiration of their contract to teach, their contract would no longer be
renewed. The petitioners amended their complaint to include the charge of illegal dismissal. They claimed that
their dismissal was illegal because it was made in retaliation for their complaint for monetary benefits and
discriminatory practices of AMACC. AMACC contented that they worked under a contracted term under a non-
tenured appointment and were still within the 3-year probationary period for teachers. Their contracts were not
renewed because they failed to pass the Performance Appraisal System for Teachers (PAST).

The Labor Arbiter ruled that the petitioners were illegally dismissed. Art. 281 of the Labor Code on probationary
employment applied to the case; AMACC did not specify who among the petitioners failed to pass the PAST and
who among them did not comply with the requirements of regularization, promotion or increase in salary; and
that the petitioners dismissal could not be sustained on the basis of AMACCs vague and general allegation
without substantial factual basis. The NLRC ruled that Sec. 92 of the Manual of Regulations for Private Schools
(which mandated a probationary period of 9 consecutive trimesters of satisfactory service for academic
personnel in the tertiary level where collegiate courses are offered on a trimester basis and not Art. 281 of the
Labor Code. However, it sustained the LAs finding of illegal dismissal since the petitioners were terminated on
the basis of standards that were only introduced near the end of their probationary period. The CA reversed the
decision of the Labor Arbiter and the NLRC on the ground the petitioners were not dismissed but rather their
contract merely expired.

ISSUE: Were the petitioners illegally dismissed?

RULING: Yes. Given the clear constitutional and statutory intents, we cannot but conclude that in a situation
where the probationary status overlaps with a fixed-term contract not specifically used for the fixed term it offers,
Article 281 should assume primacy and the fixed-period character of the contract must give way. This
conclusion is immeasurably strengthened by the petitioners and the AMACCs hardly concealed expectation that
the employment on probation could lead to permanent status, and that the contracts are renewable unless the
petitioners fail to pass the schools standards.

If the school were to apply the probationary standards (as in fact it says it did in the present case), these
standards must not only be reasonable but must have also been communicated to the teachers at the start of
the probationary period, or at the very least, at the start of the period when they were to be applied. These
terms, in addition to those expressly provided by the Labor Code, would serve as the just cause for the
termination of the probationary contract. The details of this finding of just cause must be communicated to the
affected teachers as a matter of due process.

While we can grant that the standards were duly communicated to the petitioners and could be applied
beginning the 1st trimester of the school year 2000-2001, glaring and very basic gaps in the schools evidence
still exist. The exact terms of the standards were never introduced as evidence; neither does the evidence show
how these standards were applied to the petitioners. Without these pieces of evidence, there is nothing to
consider and pass upon as valid or invalid for each of the petitioners. Inevitably, the non-renewal or effectively,
the termination of employment of employees on probationary status lacks the supporting finding of just cause
that the law requires and, hence, is illegal.

San Beda College of Law 537


4S: 2015 - 2016
LABOR LAW REVIEW Atty. Joyrich Golangco

BRENT SCHOOL, INC., and REV. GABRIEL DIMACHE v. RONALDO ZAMORA, the Presidential Assistant
for Legal Affairs, Office of the President, and DOROTEO R. ALEGRE
G.R. No. L-48494 February 5, 1990
NARVASA, J.:

FIXED PERIOD EMPLOYEE

FACTS:
Doroteo Alegre was engaged as athletic director by Brent School, Inc. The contract fixed a specific term
for its existence at five years. Subsequent subsidiary agreements reiterated the same terms and conditions
including the expiry date as those contained in the original contract. Some three months before the expiration of
the stipulated period, Alegre was given a copy of the report by Brent School with the Department of Labor
advising of the termination of his services. The stated ground for termination was completion of contract and
expiration of the definite period. However, Alegre protested the announced termination of his services at the
investigation conducted by a Labor Conciliator. He argued that although his contract did stipulate that the same
would terminate after five years, since his services were necessary and desirable in the usual business of his
employer, and his employment had lasted for five years, he had acquired the status of a regular employee and
could not be removed except for valid cause. The Regional Director accepted the recommendation of the Labor
Conciliator and required the reinstatement of Alegre as a permanent employee. Brent School filed a motion for
reconsideration but was denied by the Regional Director. The Secretary of Labor sustained the Regional
Director. An appeal to the Office of the President was likewise dismissed.

ISSUE:
Is the fixed-period employment of Alegre valid?

RULING:
Yes. Since the entire purpose behind the development of legislation culminating in the present Article
280 of the Labor Code clearly appears to have been, as already observed, to prevent circumvention of the
employee's right to be secure in his tenure, the clause in said article indiscriminately and completely ruling out
all written or oral agreements conflicting with the concept of regular employment as defined therein should be
construed to refer to the substantive evil that the Code itself has singled out: agreements entered into precisely
to circumvent security of tenure. It should have no application to instances where a fixed period of employment
was agreed upon knowingly and voluntarily by the parties, without any force, duress or improper pressure being
brought to bear upon the employee and absent any other circumstances vitiating his consent, or where it
satisfactorily appears that the employer and employee dealt with each other on more or less equal terms with no
moral dominance whatever being exercised by the former over the latter. Unless thus limited in its purview, the
law would be made to apply to purposes other than those explicitly stated by its framers; it thus becomes
pointless and arbitrary, unjust in its effects and apt to lead to absurd and unintended consequences.

Alegre's employment was terminated upon the expiration of his last contract with Brent School without the
necessity of any notice. The advance written advice given the Department of Labor with copy to said petitioner
was a mere reminder of the impending expiration of his contract, not a letter of termination, nor an application
for clearance to terminate which needed the approval of the Department of Labor to make the termination of his
services effective. In any case, such clearance should properly have been given, not denied.

San Beda College of Law 538


4S: 2015 - 2016
LABOR LAW REVIEW Atty. Joyrich Golangco

PURE FOODS CORPORATON v. NATIONAL LABOR RELATIONS COMMISSION, RODOLFO CORDOVA,
VIOLETA CRUSIS, ET AL.
G.R. No. 122653 December 12, 1997
DAVIDE, JR., J.:

FIXED-PERIOD EMPLOYEES

FACTS:
The private respondents (numbering 906) were hired by petitioner Pure Foods Corporation to work for a
fixed period of five months at its tuna cannery plant. After the expiration of their respective contracts of
employment, their services were terminated. They forthwith executed a Release and Quitclaim stating that they
had no claim whatsoever against the petitioner. The private respondent filed before the NLRC Sub-Regional
Arbitration Branch a complaint for illegal dismissal against the petitioner and its plant manager.

The Labor Arbiter dismissed the complaint on the ground that the private respondents were mere
contractual workers, and not regular employees. The termination of their services by reason of the expiration of
their contracts of employment was justified. The NLRC affirmed the Labor Arbiters decision but reversed the
decision upon a motion for reconsideration. It declared that the contract of employment for five months was a
clandestine scheme employed by the petitioner to stifle private respondents right to security of tenure and
should therefore be struck down and disregarded for being contrary to law, public policy, and morals.

ISSUE:
Is the fixed-period employment of the private respondents valid?

RULING:
No. Brent vs. Zamora laid down the criteria under which term employment cannot be said to be in
circumvention of the law on security of tenure:
1. The fixed period of employment was knowingly and voluntarily agreed upon by the parties without any
force, duress, or improper pressure being brought to bear upon the employee and absent any other
circumstances vitiating his consent; or
2. It satisfactorily appears that the employer and the employee dealt with each other on more or less equal
terms with no moral dominance exercised by the former or the latter.

None of these criteria had been met in the present case. It could not be supposed that private respondents
KNOWINGLY and VOLUNTARILY agreed to the 5-month employment contract. Cannery workers are never on
equal terms with their employers. Almost always, they agree to any terms of an employment contract just to get
employed considering that it is difficult to find work given their ordinary qualifications. Their freedom to contract
is empty and hollow because theirs is the freedom to starve if they refuse to work as casual or contractual
workers. Indeed, to the unemployed, security of tenure has no value. It could not then be said that petitioner and
private respondents "dealt with each other on more or less equal terms with no moral dominance whatever
being exercised by the former over the latter.

The five-month period specified in private respondents employment contracts having been imposed precisely to
circumvent the constitutional guarantee on security of tenure should, therefore, be struck down or disregarded
as contrary to public policy or morals.

San Beda College of Law 539


4S: 2015 - 2016
LABOR LAW REVIEW Atty. Joyrich Golangco

LEYTE GEOTHERMAL POWER PROGRESSIVE EMPLOYEES UNION ALU TUCP vs. PHILIPPINE
NATIONAL OIL COMPANY ENERGY DEVELOPMENT CORPORATION
G.R. No. 170351 March 30, 2011
NACHURA, J.:

PROJECT EMPLOYEES

FACTS: Respondent is a GOCC while petitioner is a legitimate labor organization. Among [respondents]
geothermal projects is the Leyte Geothermal Power Project located at the Greater Tongonan Geothermal
Reservation in Leyte. Thus, the [respondent] hired and employed hundreds of employees on a contractual
basis, whereby, their employment was only good up to the completion or termination of the project and would
automatically expire upon the completion of such project.

Majority of the employees hired by [respondent] in its Leyte Geothermal Power Projects had become members
of petitioner. In view of that circumstance, the petitioner demands from the [respondent] for recognition of it as
the collective bargaining agent of said employees and for a CBA negotiation with it. However, the [respondent]
did not heed such demands of the petitioner. Sometime in 1998 when the project was about to be completed,
the [respondent] proceeded to serve Notices of Termination of Employment upon the employees who are
members of the petitioner.

On December 28, 1998, the petitioner filed a Notice of Strike with DOLE against the [respondent] on the ground
of purported commission by the latter of unfair labor practice for refusal to bargain collectively, union busting
and mass termination. On the same day, the petitioner declared a strike and staged such strike.

Secretary of Labor intervened and ordered all workers to return to work. However, petitioner did not abide.

NLRC: ruled that the employees are PROJECT EMPLOYEES, and the strike as ILLEGAL

Petitioner Union contends that its officers and members performed activities that were usually necessary and
desirable to respondents usual business.

ISSUE: WON they are project employees

HELD: They are PROJECT EMPLOYEES


Article 280 of the Labor Code contemplates four (4) kinds of employees:
(a) regular employees or those who have been engaged to perform activities which are usually necessary
or desirable in the usual business or trade of the employer;
(b) project employees or those whose employment has been fixed for a specific project or undertaking[,]
the completion or termination of which has been determined at the time of the engagement of the
employee;
(c) seasonal employees or those who work or perform services which are seasonal in nature, and the
employment is for the duration of the season; and
(d) casual employees or those who are not regular, project, or seasonal employees.

Jurisprudence has added a fifth kind a fixed-term employee.

By entering into such a contract, an employee is deemed to understand that his employment is coterminous with
the project. He may not expect to be employed continuously beyond the completion of the project. It is of judicial
notice that project employees engaged for manual services or those for special skills like those of carpenters or
masons, are, as a rule, unschooled. However, this fact alone is not a valid reason for bestowing special
treatment on them or for invalidating a contract of employment. Project employment contracts are not lopsided
agreements in favor of only one party thereto. The employers interest is equally important as that of the
employee[s] for theirs is the interest that propels economic activity. While it may be true that it is the employer
who drafts project employment contracts with its business interest as overriding consideration, such contracts
do not, of necessity, prejudice the employee. Neither is the employee left helpless by a prejudicial employment
contract. After all, under the law, the interest of the worker is paramount.

San Beda College of Law 540


4S: 2015 - 2016
LABOR LAW REVIEW Atty. Joyrich Golangco

Unions own admission, both parties had executed the contracts freely and voluntarily without force, duress or
acts tending to vitiate the worker[s] consent. Thus, we see no reason not to honor and give effect to the terms
and conditions stipulated therein.

The litmus test to determine whether an individual is a project employee lies in setting a fixed period of
employment involving a specific undertaking which completion or termination has been determined at the time of
the particular employees engagement.

San Beda College of Law 541


4S: 2015 - 2016
LABOR LAW REVIEW Atty. Joyrich Golangco

ENGINEER LEONCIO V. SALAZAR, vs. NATIONAL LABOR RELATIONS COMMISSION (2nd Division) and
H. L. CARLOS CONSTRUCTION, CO. INC.
G.R. No 109210 April 17, 1996
KAPUNAN, J.:

FACTS: On 17 April 1990, private respondent Carlos Construction, at a monthly salary of P4,500.00, employed
Salazar as construction/project engineer for the construction of a building in Cubao. Allegedly, by virtue of an
oral contract, petitioner would also receive a share in the profits after completion of the project and that
petitioners services in excess of 8 ours on regular days and services rendered on weekends and legal holidays
shall be compensable overtime.

On 16 April 1991, petitioner received a memorandum issued by private respondents project manager informing
him of the termination of his services effective on 30 April 1991.

On 13 September 1991, Salazar filed a complaint against private respondent for illegal dismissal, unfair labor
practice, illegal deduction, non-payment of wages, overtime rendered, service incentive leave pay, commission,
allowances, profit-sharing and separation pay with the NLRC-NCR Arbitration Branch, Manila.

The Labor Arbiter rendered a decision dismissing the instant case for lack of merit. Petitioner appealed to the
NLRC, where it affirmed in toto the decision of the Labor Arbiter. His MR was likewise dismissed. Hence the
instant petition.

ISSUES:
1) WON petitioner is entitled to overtime pay, premium pay for services rendered on rest days and
holidays and service incentive leave pay
2) WON petitioner is entitled to a share in the profits of the construction project;.
3) WON petitioner rendered services from 1 May to 15 May 1991 and is, therefore, entitled to unpaid
wages;
4) WON private respondent is liable to reimburse petitioners legal expenses and;
5) WON petitioner is entitled to separation pay.

HELD: The assailed decision is modified.


1. NO. Although petitioner cannot strictly be classified as a managerial employee, nonetheless he is still not
entitled to payment of the aforestated benefits because he falls squarely under another exempt category
officers or members of a managerial staff as defined under sec. 2(c) of the abovementioned implementing
rules:
Sec. 2. Exemption. The provisions of this Rule shall not apply to the following persons if they qualify for
exemption under the condition set forth herein:xxx
(c) Officers or members of a managerial staff xxx

That petitioner was paid overtime benefits does not automatically and necessarily denote that petitioner is
entitled to such benefits

2. NO. petitioner insists that private respondent promised him a share in the profits after completion of the
construction project. It is because of this oral agreement, petitioner elucidates, that he agreed to a monthly
salary of P4,500.00, an amount which he claims is too low for a professional civil engineer like him with the rank
of project engineer.

We cannot accede to petitioners demand. Nowhere in the disbursement vouchers can we find even the
remotest hint of a profit-sharing agreement between petitioner and private respondent. Petitioners
rationalization stretches the imagination way too far.
Also, as said by the Labor Arbiter:

As to the issue of profit sharing, we simply cannot grant the same on the mere basis of complainants allegation
that respondent verbally promised him that he is entitled to a share in the profits derive(d) from the projects.
Benefits or privileges of this nature (are) usually in writing, besides complainant failed to (establish) that said
benefits or privileges (have) been given to any of respondent(s) employees as a matter of practice or policy.

San Beda College of Law 542


4S: 2015 - 2016
LABOR LAW REVIEW Atty. Joyrich Golangco

3. YES. On April 30, he was advised by the Manager to continue supervising the finishing touches to the
building until May 15, the date appearing in the Certificate of Service as the date of the termination of the
contract between Salazar and the Company. But the Manager insists that Salazars services terminated at April
30 according to the Memorandum given the petitioner.
The purpose for which the said certificate was issued becomes irrelevant. The fact remains that private
respondent knowingly and voluntarily issued the certificate. Mere denials and self-serving statements to the
effect that petitioner allegedly promised not to use the certificate against private respondent are not sufficient to
overturn the same. Hence, private respondent is estopped from assailing the contents of its own certificate of
service.

4. YES. During the construction of the building, a criminal complaint for unjust vexation was filed against the
officers of the owner of the building. Petitioner avers that he was implicated in the complaint for the sole reason
that he was the construction engineer of the project.

Although not directly implicated in the criminal complaint, Carlos Construction is nonetheless obligated to defray
petitioners legal expenses. Petitioner was included in the complaint not in his personal capacity but in his
capacity as project engineer of private respondent and the case arose in connection with his work as such. At
the construction site, petitioner is the representative of private respondent being its employee and he acts for
and in behalf of private respondent. Hence, the inclusion of petitioner in the complaint for unjust vexation, which
was work-related, is equivalent to inclusion of private respondent itself.

5. NO. On the last issue, we rule that petitioner is a project employee and, therefore, not entitled to separation
pay.
The applicable provision is Article 280 of the Labor Code which defines the term project employee, thus:
Art. 280. Regular and Casual Employment. The provisions of written agreement to the contrary
notwithstanding and regardless of the oral agreement of the parties, an employment shall be deemed to be
regular where the employee has been engaged to perform activities which are usually necessary or desirable in
the usual business or trade of the employer, except where the employment has been fixed for a specific period
or undertaking the completion or termination of which has been determined at the time of the engagement of the
employee or where the work or services to be performed is seasonal in nature and the employment is for the
duration of the season. (Emphasis ours.)

In the case at bench, it was duly established that private respondent hired petitioner as project or construction
engineer specifically for its Monte de Piedad building project. Accordingly, as project employee, petitioners
services are deemed coterminous with the project, that is, petitioners services may be terminated as soon as
the project for which he was hired is completed. There can be no dispute that petitioners dismissal was due to
the completion of the construction of the building.

San Beda College of Law 543


4S: 2015 - 2016
LABOR LAW REVIEW Atty. Joyrich Golangco

FONTERA BRANDS PHIS, INC. vs. LEONARDO LARGADO AND TEOTIMO ESTRELLADO
G.R. No. 205300, March 18, 2015
VELASCO JR., J.

FACTS:
Fonterra contracted the services of Zytron Marketing and Promotions Corp. (Zytron) for the marketing of its milk
and dairy products. Pursuant to their contract, Zytron provided Fonterra with Largado (in 2003) and Estrellado
(in 2002) as trade merchandising representatives (TMRs).

Sometime in 2006 Fonterra terminated its promotions contract with Zytron and entered into a manpower supply
agreement with A.C. Sicat Marketing and Promotional Services (A.C. Sicat). It appears Largado and Estrellado
wanted to continue as TMRs for Fonterra and therefore they submitted their job applications with A.C. Sicat.
A.C. Sicat hired them for a term of five months.

After the expiration of the 5-month period, Largado and Estrellados contracts were not renewed. The latter filed
a case for illegal dismissal (among other claims) against Fonterra, Zytron and A.C. Sicat but did not assert any
claim against Zytron and A.C. Sicat.

LABOR ARBITERS RULING:


The Labor Arbiter found in favor of respondents.

NLRCs RULING:
The NLRC affirmed the Labor Arbiter.

CAs RULING:
But when the case reached the Court of Appeals the CA said respondents (i.e., complainants in the original
case before the LA) were illegally dismissed as early as June 6, 2006 because Zytron was in its view not a
legitimate job contractor (thus making Fonterra the real employer and the one liable for respondents alleged
illegal dismissal) and respondents transfer to A.C. Sicat is tantamount to a completely new engagement by
another employer.

ISSUES:
1. Whether or not Zytron is a legitimate job contractor
2. Whether or not respondents are illegally dismissed
3. Whether or not respondents are project employees

HELD:
1. The SC also found A.C. Sicat to be a legitimate job contractor.

2. Whether Zytron is a legitimate job contractor or not is immaterial to the resolution of the illegal dismissal
issue. The Supreme Court found that respondents (i.e., complainants in the original case before the LA) refused
to renew their contracts with Zytron because apparently they wanted to continue their assignment in Fonterra,
which could not happen because of the end of Zytrons contract with Fonterra. They therefore applied with A.C.
Sicat, Fonterras new manpower supplier, in the hope that they will be able to continue rendering services as
TMRs at Fonterra. They are deemed to have voluntarily resigned. Worth quoting is the SCs definition of
voluntary resignation:
Resignation is the voluntary act of employees who are compelled by personal reasons to dissociate themselves
from their employment, done with the intention of relinquishing an office, accompanied by the act of
abandonment.

3. The SC found that respondents were hired by A.C. Sicat as project employees for a fixed term. According to
the SC
As held by this Court, fixed-term employment contracts are not limited, as they are under the present Labor
Code, to those by nature seasonal or for specific projects with predetermined dates of completion; they also
include those to which the parties by free choice have assigned a specific date of termination. The determining
factor of such contracts is not the duty of the employee but the day certain agreed upon by the parties for the
commencement and termination of the employment relationship.

San Beda College of Law 544


4S: 2015 - 2016
LABOR LAW REVIEW Atty. Joyrich Golangco

The SC agreed with the CAs finding that the termination of respondents employment with A.C. Sicat was
simply due to the expiration of their employment contracts, which A.C. Sicat could refuse to renew as a matter of
managerial prerogative. What is interesting is that the SC seems to consider respondents contracts as fixed-
term contracts, yet at the same time it said,
In the case at bar, it is clear that respondents were employed by A.C. Sicat as project employees.

Hence, not all fixed-term employees are project employees, although it would seem that all project employees
are at the same time fixed-term employees. In any event, what is interesting here is that we now have precedent
for considering trade merchandising representatives hired for a specific project, such as the promotion of certain
products, as fixed-term employees whose contracts, once expired, their employer can refuse to renew as a
matter of managerial prerogative. To quote the SC:
Respondents, by accepting the conditions of the contract with A.C. Sicat, were well aware of and even acceded
to the condition that their employment thereat will end on said pre-determined date of termination. They cannot
now argue that they were illegally dismissed by the latter when it refused to renew their contracts after its
expiration. This is so since the non-renewal of their contracts by A.C. Sicat is a management prerogative, and
failure of respondents to prove that such was done in bad faith militates against their contention that they were
illegally dismissed. The expiration of their contract with A.C. Sicat simply caused the natural cessation of their
fixed-term employment thereat.

San Beda College of Law 545


4S: 2015 - 2016
LABOR LAW REVIEW Atty. Joyrich Golangco

ROMEO BASAN, DANILO DIZON, JAIME L. TUMABIAO, JR., ROBERTO DELA RAMA, JR., RICKY S.
NICOLAS, CRISPULO D. DONOR, GALO FALGUERA, AND NATIONAL LABOR RELATIONS
COMMISSION v. COCA-COLA BOTTLERS PHILIPPINES
G.R NO. 174365-66 February 4, 2015
PERALTA, J:

2 KINDS OF REGULAR EMPLOYEES; MAGSALIN DOCTRINE; BRENT vs. ZAMORA GUIDELINES

FACTS:Petitioners Romeo Basan, et al. filed a complaint for illegal dismissal with money claims against
respondent Coca-Cola Bottlers Philippines, alleging that respondent dismissed them without just cause and
prior written notice.

Coca Cola however, countered that it hired petitioners as temporary route helpers to act as substitutes for its
absent regular route helpers merely for a fixed period in anticipation of the high volume of work in its plants or
sales offices.As such, petitioners claims have no basis for they knew that their assignment as route helpers was
temporary in duration.

LA: Illegally dismissed. Since they were performing activities necessary and desirable to the usual business of
petitioner for more than the period for regularization, petitioners are considered as regular employees, and thus,
their dismissal was done contrary to law in the absence of just cause and prior written notice.

NLRC: affirmed and rejected respondents contention that petitioners were merely employed for a specific
project or undertaking the completion or termination of which has been determined at the time of their
engagement. It stressed that nowhere in the records of the case was it shown that petitioners were hired as
project or seasonal employees, respondent having failed to submit any contract of project or other similar proof
thereof. It also noted that neither can petitioners be considered as probationary employees for the fact that they
had performed their services for more than six (6) months.

CA: Reversed rulings of LA and NLRC.


That the petitioners performed duties which are necessary or desirable in the usual trade or business of Coca-
Cola, is of no moment. This is not the only standard for determining the status of ones employment. Such fact
does not prevent them from being considered as fixed term employees of Coca-Cola whose engagement was
fixed for a specific period. The petitioners repeated hiring for various periods (ranging from more than six
months for private respondent Basan to eight years in the case of private respondent Dizon) would not
automatically categorize them as REGULAR EMPLOYEES.cralawred

Following Brent School, Inc. vs. Zamora, petitioners must be considered as fixed term employees whose
seasonal employment or employment for a period have been set down.

Petitioners essentially maintain that contrary to the findings of the CA, they were continuously hired by
respondent company to perform duties necessary and desirable in the usual trade or business and are,
therefore, regular employees. They allege that if their services had really been engaged for fixed specific
periods, respondent should have at least provided the contracts of employment evidencing the same.
Coca cola disagrees with the contention that petitioners, as route helpers, were performing functions necessary
or desirable to its business.

ISSUE: WON petitioners shall be considered as regular employees of Coca Cola

HELD: YES. (Magsalin v NLRC: MAGSALIN DOCTRINE) the nature of work of route helpers hired by Coca
Cola Bottlers Philippines, Inc. is necessary and desirable in its usual business or trade thereby qualifying them
as regular employees, to wit:
Coca-Cola Bottlers Phils., Inc., is one of the leading and largest manufacturers of softdrinks in the country.
Respondent workers have long been in the service of petitioner company. Respondent workers, when hired,
would go with route salesmen on board delivery trucks and undertake the laborious task of loading and
unloading softdrink products of petitioner company to its various delivery points.

In determining whether an employment should be considered regular or non-regular, the applicable test is the
reasonable connection between the particular activity performed by the employee in relation to the usual
business or trade of the employer. The standard, supplied by the law itself, is whether the work undertaken is
San Beda College of Law 546
4S: 2015 - 2016
LABOR LAW REVIEW Atty. Joyrich Golangco

necessary or desirable in the usual business or trade of the employer, a fact that can be assessed by looking
into the nature of the services rendered and its relation to the general scheme under which the business or trade
is pursued in the usual course. It is distinguished from a specific undertaking that is divorced from the normal
activities required in carrying on the particular business or trade. But, although the work to be performed is only
for a specific project or seasonal, where a person thus engaged has been performing the job for at least one
year, even if the performance is not continuous or is merely intermittent, the law deems the repeated and
continuing need for its performance as being sufficient to indicate the necessity or desirability of that activity to
the business or trade of the employer. The employment of such person is also then deemed to be regular with
respect to such activity and while such activity exists.

The argument of Coca Cola that its usual business or trade is softdrink manufacturing and that the work
assigned to workers as sales route helpers so involves merely "postproduction activities," one which is
not indispensable in the manufacture of its products, scarcely can be persuasive. If, as so argued by
company, only those whose work are directly involved in the production of softdrinks may be held
performing functions necessary and desirable in its usual business or trade, there would have then
been no need for it to even maintain regular truck sales route helpers. The nature of the work performed
must be viewed from a perspective of the business or trade in its entirety and not on a confined scope.

The repeated rehiring of respondent workers and the continuing need for their services clearly attest to the
necessity or desirability of their services in the regular conduct of the business or trade of petitioner
company. While this Court, in Brent School, Inc. vs. Zamora, has upheld the legality of a fixed-term
employment, it has done so, however, with a stern admonition that where from the circumstances it is
apparent that the period has been imposed to preclude the acquisition of tenurial security by the
employee, then it should be struck down as being contrary to law, morals, good customs, public order
and public policy. The pernicious practice of having employees, workers and laborers, engaged for a
fixed period of few months, short of the normal six-month probationary period of employment, and,
thereafter, to be hired on a day-to-day basis, mocks the law. Any obvious circumvention of the law
cannot be countenanced. A contract of employment is impressed with public interest. The provisions of
applicable statutes are deemed written into the contract, and "the parties are not at liberty to insulate themselves
and their relationships from the impact of labor laws and regulations by simply contracting with each other."

Pursuant to Art. 280 of the Labor Code, regular employees are classified into:
(1) regular employees by nature of work; (refers to those employees who perform a particular activity which is
necessary or desirable in the usual business or trade of the employer, regardless of their length of service)
(2) regular employees by years of service (refers to those employees who have been performing the job,
regardless of the nature thereof, for at least a year.)awlawlibrary

Petitioners, in this case, fall under the first kind of regular employee above. As route helpers who are engaged in
the service of loading and unloading softdrink products of respondent company to its various delivery points,
which is necessary or desirable in its usual business or trade, petitioners are considered as regular employees.
That they merely rendered services for periods of less than a year is of no moment.
There is, on the other hand, the Civil Code, which has always recognized, and continues to recognize, the
validity and propriety of contracts and obligations with a fixed or definite period, and imposes no restraints on
the freedom of the parties to fix the duration of a contract, whatever its object, be it specie, goods or services,
except the general admonition against stipulations contrary to law, morals, good customs, public order or public
policy. Under the Civil Code, therefore, and as a general proposition, fixed-term employment contracts
are not limited, as they are under the present Labor Code, to those by nature seasonal or for specific
projects with pre-determined dates of completion; they also include those to which the parties by free
choice have assigned a specific date of termination.

Accordingly, and since the entire purpose behind the development of legislation culminating in the present
Article 280 of the Labor Code clearly appears to have been, as already observed, to prevent circumvention of
the employee's right to be secure in his tenure, the clause in said article indiscriminately and completely ruling
out all written or oral agreements conflicting with the concept of regular employment as defined therein should
be construed to refer to the substantive evil that the Code itself has singled out: agreements entered into
precisely to circumvent security of tenure. It should have no application to instances where a fixed period
of employment was agreed upon knowingly and voluntarily by the parties, without any force, duress or
improper pressure being brought to bear upon the employee and absent any other circumstances
vitiating his consent, or where it satisfactorily appears that the employer and employee dealt with each
San Beda College of Law 547
4S: 2015 - 2016
LABOR LAW REVIEW Atty. Joyrich Golangco

other on more or less equal terms with no moral dominance whatever being exercised by the former
over the latter. Unless thus limited in its purview, the law would be made to apply to purposes other than those
explicitly stated by its framers; it thus becomes pointless and arbitrary, unjust in its effects and apt to lead to
27
absurd and unintended consequences.
ChanRoblesVirtualawlibrary
Thus, under the above Brent doctrine, while it was not expressly mentioned in the Labor Code, this Court has
recognized a fixed-term type of employment embodied in a contract specifying that the services of the employee
shall be engaged only for a definite period, the termination of which occurs upon the expiration of said period
irrespective of the existence of just cause and regardless of the activity the employee is called upon to
28
perform. Considering, however, the possibility of abuse by employers in the utilization of fixed-term
employment contracts, this Court, in Brent, laid down the following criteria to prevent the circumvention of the
employees security of tenure:chanRoblesvirtualLawlibrary
1) The fixed period of employment was knowingly and voluntarily agreed upon by the parties without any
force, duress, or improper pressure being brought to bear upon the employee and absent any other
circumstances vitiating his consent; or
2) It satisfactorily appears that the employer and the employee dealt with each other on more or less equal
29
terms with no moral dominance exercised by the former or the latter.
ChanRoblesVirtualawlibrary
Unfortunately, however, the records of this case is bereft of any proof which will show that petitioners freely
entered into agreements with respondent to perform services for a specified length of time. In fact, there is
nothing in the records to show that there was any agreement at all, the contracts of employment not having
been presented. While respondent company persistently asserted that petitioners knowingly agreed upon a
fixed period of employment and repeatedly made reference to their contracts of employment, the expiration
thereof being made known to petitioners at the time of their engagement, respondent failed to present the same
in spite of all the opportunities to do so.

While fixed term employment is not per se illegal or against public policy, the criteria above must first be
established to the satisfaction of this Court. Yet, the records of this case reveal that for years, petitioners were
repeatedly engaged to perform functions necessary to respondents business for fixed periods short of the six-
month probationary period of employment. If there was really no intent to circumvent security of tenure,
respondent should have made it clear to petitioners that they were being hired only for fixed periods in an
agreement freely entered into by the parties. To this Court, respondents act of hiring and re-hiring petitioners for
periods short of the legal probationary period evidences its intent to thwart petitioners security of tenure,
especially in view of an awareness that ordinary workers, such as petitioners herein, are never on equal terms
with their employers. Consequently, for lack of any clear, valid, and just or authorized cause in terminating
petitioners employment, We find respondent guilty of illegal dismissal.

San Beda College of Law 548


4S: 2015 - 2016
LABOR LAW REVIEW Atty. Joyrich Golangco

ZUELLIG FREIGHT AND CARGO SYSTEMS, vs. NATIONAL LABOR RELATIONS COMMISSION AND
RONALDO V. SAN MIGUEL
G.R. No. 157900 July 22, 2013
BERSAMIN, J.:

DOCTRINE: The mere change in the corporate name is not considered under the law as the creation of a new
corporation; hence, the renamed corporation remains liable for the illegal dismissal of its employee separated
under that guise.

FACTS: Ronaldo San Miguel brought a complaint for unfair labor practice, illegal dismissal, non-payment of
salaries and moral damages against Zuellig Freight and Cargo Systems, formerly known as Zeta Brokerage
2
Corporation (Zeta). He alleged that he had been a checker/customs representative of Zeta since December 16,
1985; that in January 1994, he and other employees of Zeta were informed that Zeta would cease operations,
and that all affected employees, including him, would be separated; Zeta informed him of his termination.

San Miguel contended that the amendments of the articles of incorporation of Zeta were for the purpose of
changing the corporate name, broadening the primary functions, and increasing the capital stock; and that such
3
amendments could not mean that Zeta had been thereby dissolved.

Zeta countered that San Miguels termination from Zeta had been for a cause authorized by the Labor Code;
that its non-acceptance of him had not been by any means irregular or discriminatory; that it had no obligation to
employ San Miguel in the exercise of its valid management prerogative;

LA: San Miguel illegally dismissed. LA believes that there was merely a change of business name and primary
purpose and upgrading of stocks of the corporation. Zuellig and Zeta are therefore legally the same person and
entity.

NLRC: affirmed.

CA: affirmed

Good faith was not established by mere registration with the Securities and Exchange Commission (SEC) of the
Amended Articles of Incorporation and ByLaws. The factual milleu of the case, considered in its totality, shows
that there was no closure to speak of. The termination of services allegedly due to cessation of business
operations of Zeta was illegal.

There being no valid closure of business operations, the dismissal of private respondent San Miguel on alleged
authorized cause of cessation of business pursuant to Article 283 of the Labor Code, was utterly illegal.

ISSUE: WON the cessation of business by Zeta was a bona fide closure to be regarded as a valid ground for
the termination of employment of San Miguel

HELD: NO.
Article 283. Closure of establishment and reduction of personnel. The employer may also terminate the
employment of any employee due to the installation of labor-saving devices, redundancy, retrenchment to
prevent losses or the closing or cessation of operation of the establishment or undertaking unless the closing is
for the purpose of circumventing the provisions of this Title, x x x.

Verily, the amendments of the articles of incorporation of Zeta to change the corporate name to Zuellig Freight
and Cargo Systems, Inc. did not produce the dissolution of the former as a corporation. For sure, the
Corporation Code defined and delineated the different modes of dissolving a corporation, and amendment of the
articles of incorporation was not one of such modes.

Zeta and petitioner remained one and the same corporation. The change of name did not give petitioner the
license to terminate employees of Zeta like San Miguel without just or authorized cause. Petitioner, despite its
new name, was the mere continuation of Zeta's corporate being, and still held the obligation to honor all of
Zeta's obligations, one of which was to respect San Miguel's security of tenure. The dismissal of San Miguel
from employment on the pretext that petitioner, being a different corporation, had no obligation to accept him as
its employee, was illegal and ineffectual.
San Beda College of Law 549
4S: 2015 - 2016
LABOR LAW REVIEW Atty. Joyrich Golangco

JENNY F. PECKSON vs. ROBINSONS SUPERMARKET CORPORATION, JODY GADIA, ROENA SARTE,
and RUBY ALEX
G.R. No. 198534 July 3, 2013
REYES, J.:

MANAGEMENT PREROGATIVE; REASSIGNMENT; RURAL BANK OF CANTILA INC V. JULVE


JURISPRUDENTIAL GUIDELINES

DOCTRINE: It is the employers prerogative, based on its assessment and perception of its employees
qualifications, aptitudes, and competence, to move them around in the various areas of its business operations
in order to ascertain where they will function with maximum benefit to the company. An employees right to
security of tenure does not give him such a vested right in his position as would deprive the company of its
prerogative to change his assignment or transfer him where he will be most useful.

FACTS: Jenny Peckson first joined the Robinsons Supermarket Corporation (RSC) as a Sales Clerk holding the
position of Category Buyer when respondent Roena Sarte Assistant Vice-President, reassigned her to the
position of Provincial Coordinator. Claiming that her new assignment was a demotion because it was non-
supervisory and clerical in nature, the petitioner refused to turn over her responsibilities to the new Category
Buyer, or to accept her new responsibilities as Provincial Coordinator.

RSC, through Sarte, demanded an explanation from her for her refusal to accept her new assignment despite
written and verbal demands. Sarte cited a company rule, Offenses Subject to Disciplinary Action No. 4.07, which
provided that "[d]isobedience, refusal or failure to do assigned task or to obey superiors/officials
orders/instructions, or to follow established procedures or practices without valid reason" would be meted the
penalty of suspension.

The petitioner ignored the 48-hour deadline to explain. Peckson stated in reply that she could not accept the
position of Provincial Coordinator since she saw it as a demotion. As it turned out, however, Peckson had
already filed a complaint for constructive dismissal.

Several tasked were asked of Peckson by Sarte which Peckson refused to heed.

Peckson argued before the LA that the true organizational chart of the RSC showed that the position of
Category Buyer was one level above that of the Provincial Coordinator.

The respondents in their position paper denied the correctness of the organizational chart presented by the
petitioner. They maintained that her transfer was not a demotion since the Provincial Coordinator occupied a
"Level 5" position like the Category Buyer, with the same work conditions, salary and benefits. The respondents
also raised the petitioners record of habitual tardiness as far back as 1999, as well as poor performance rating.
Also, that the petitioner was suspended for seven (7) days in September and October 2005 for deliberately
violating a company policy after she was seen having lunch with a company supplier.

Sarte denied that the reassignment of the petitioner as Provincial Coordinator was motivated by a desire to
besmirch the name of the latter. She asserted that it was made in the exercise of management prerogative and
sound discretion.

LA: ruled that job reassignment or classification is a strict prerogative of the employer, and that the petitioner
cannot refuse her transfer since both positions commanded the same salary structure, high degree of
responsibility and impeccable honesty and integrity. LA found no showing of any illegal motive on the part of the
respondents in reassigning the petitioner.

A month after the above ruling, Peckson tendered her written "forced" resignation, wherein she complained that
she was being subjected to ridicule by clients and co-employees alike on account of her floating status since the
time she refused to accept her transfer. She likewise claimed that she was being compelled to accept the
position of Provincial Coordinator without due process.

NLRC: sustained the findings of the LA.

CA: also affirmed


San Beda College of Law 550
4S: 2015 - 2016
LABOR LAW REVIEW Atty. Joyrich Golangco

ISSUE: WON the reassignment of Peckson amounted to demotion thus it was not a valid exercise of
management prerogative.

HELD: VALID exercise of management prerogative. This Court has consistently refused to interfere with the
exercise by management of its prerogative to regulate the employees work assignments, the working methods
and the place and manner of work.

Labor laws discourage interference with an employers judgment in the conduct of his business.

Under the doctrine of management prerogative, every employer has the inherent right to regulate, according to
his own discretion and judgment, all aspects of employment, including hiring, work assignments, working
methods, the time, place and manner of work, work supervision, transfer of employees, lay-off of workers, and
discipline, dismissal, and recall of employees. The only limitations to the exercise of this prerogative are those
imposed by labor laws and the principles of equity and substantial justice.

While the law imposes many obligations upon the employer, nonetheless, it also protects the employers right to
expect from its employees not only good performance, adequate work, and diligence, but also good conduct and
loyalty.

(Rural Bank of Cantila Inc v. Julve Jurisprudential Guidelines)


Concerning the transfer of employees, these are the following jurisprudential guidelines:
(a) a transfer is a movement from one position to another of equivalent rank, level or salary without break in
the service or a lateral movement from one position to another of equivalent rank or salary;
(b) the employer has the inherent right to transfer or reassign an employee for legitimate business
purposes;
(c) a transfer becomes unlawful where it is motivated by discrimination or bad faith or is effected as a form
of punishment or is a demotion without sufficient cause;
(d) the employer must be able to show that the transfer is not unreasonable, inconvenient, or prejudicial to
the employee.

It is the employers prerogative, based on its assessment and perception of its employees qualifications,
aptitudes, and competence, to move them around in the various areas of its business operations in order to
ascertain where they will function with maximum benefit to the company. An employees right to security of
tenure does not give him such a vested right in his position as would deprive the company of its prerogative to
change his assignment or transfer him where he will be most useful.

We agree with the appellate court that the respondents are justified in moving the petitioner to another
equivalent position, which presumably would be less affected by her habitual tardiness or inconsistent
attendance than if she continued as a Category Buyer, a "frontline position" in the day-to-day business
operations of a supermarket such as Robinsons.

The managerial prerogative to transfer personnel must be exercised without grave abuse of discretion, bearing
in mind the basic elements of justice and fair play. Having the right should not be confused with the manner in
which that right is exercised. Thus, it cannot be used as a subterfuge by the employer to rid himself of an
undesirable worker.

There is substantial showing that the transfer of the petitioner from Category Buyer to Provincial Coordinator
was not unreasonable, inconvenient, or prejudicial to her. The petitioner failed to dispute that the job
classifications of Category Buyer and Provincial Coordinator are similar, or that they command a similar salary
structure and responsibilities. We agree with the NLRC that the Provincial Coordinators position does not
involve mere clerical functions but requires the exercise of discretion from time to time, as well as independent
judgment, since the Provincial Coordinator gives appropriate recommendations to management and ensures the
faithful implementation of policies and programs of the company. It even has influence over a Category Buyer
because of its recommendatory function that enables the Category Buyer to make right decisions on
assortment, price and quantity of the items to be sold by the store.

San Beda College of Law 551


4S: 2015 - 2016
LABOR LAW REVIEW Atty. Joyrich Golangco

We also cannot sustain the petitioners claim that she was not accorded due process. In addition to verbal
reminders from Sarte, the petitioner was asked in writing twice to explain within 48 hours her refusal to accept
her transfer. To her discredit, her defiance constituted a neglect of duty, or an act of insubordination, per the LA.

San Beda College of Law 552


4S: 2015 - 2016
LABOR LAW REVIEW Atty. Joyrich Golangco

RENATO S. GATBONTON v. NATIONAL LABOR RELATIONS COMMISSION, MAPUA INSTITUTE OF
TECHNOLOGY and JOSE CALDERON
G.R. NO. 146779 January 23, 2006
AUSTRIA-MARTINEZ, J.:

MANAGEMENT PREROGATIVE; PREVENTIVE SUSPENSION

DOCTRINE: Preventive suspension is a disciplinary measure for the protection of the companys property
pending investigation of any alleged malfeasance or misfeasance committed by the employee. The employer
may place the worker concerned under preventive suspension if his continued employment poses a serious and
imminent threat to the life or property of the employer or of his co-workers. However, when it is determined that
there is no sufficient basis to justify an employees preventive suspension; the latter is entitled to the payment of
salaries during the time of preventive suspension.

FACTS:
Petitioner is an associate professor of respondent Mapua Institute of Technology (MIT), Faculty of Civil
Engineering. A civil engineering student of respondent MIT filed a letter-complaint against petitioner for
unfair/unjust grading system, sexual harassment and conduct unbecoming of an academician. Pending
investigation of the complaint, respondent MIT, through its Committee on Decorum and Investigation placed
petitioner under a 30-day preventive suspension. The committee believed that petitioners continued stay during
the investigation affects his performance as a faculty member, as well as the students learning; and that the
suspension will allow petitioner to prepare himself for the investigation and will prevent his influences to other
members of the community.

Thus, petitioner filed with the NLRC a complaint for illegal suspension, damages and attorneys fees.

LA Ruling:
The 30-day preventive suspension is illegal but dismissed petitioners claim for damages.

NLRC and CA Ruling:


The NLRC and LA ruled that the 30-day preventive suspension is legal.

ISSUE: Is the 30-day preventive suspension legal?

HELD: NO, the 30-day preventive suspension is illegal.

Preventive suspension is a disciplinary measure for the protection of the companys property pending
investigation of any alleged malfeasance or misfeasance committed by the employee. The employer may place
the worker concerned under preventive suspension if his continued employment poses a serious and imminent
threat to the life or property of the employer or of his co-workers. However, when it is determined that there is no
sufficient basis to justify an employees preventive suspension; the latter is entitled to the payment of salaries
during the time of preventive suspension.

Under the Labor Code, petitioners preventive suspension finds no valid justification. As provided in Section 8,
Rule XXIII, Book V of the Omnibus Rules Implementing the Labor Code:

Sec. 8. Preventive Suspension. The employer may place the worker concerned under
preventive suspension if his continued employment poses a serious threat to the life or property
of the employer or of his co-workers.

There is nothing on record which shows that respondent MIT imposed the preventive suspension on
petitioner as his continued employment poses a serious threat to the life or property of the employer or of his co-
workers; therefore, his preventive suspension is not justified. Consequently, the payment of wages during his
30-day preventive suspension is in order.

San Beda College of Law 553


4S: 2015 - 2016
LABOR LAW REVIEW Atty. Joyrich Golangco

TAMSONS ENTERPRISES, INC. VS CA
G.R. No. 192881 November 16, 2011
MENDOZA, J.

FACTS:
Sy was hired by Tamsons as Assistant to the President. Four days before she completed her sixth
month of working in Tamsons, Ng, the Sales Project Manager, called her to a meeting with him and Lee. During
the meeting, they informed Sy that her services would be terminated due to inefficiency. She was asked to sign
a letter of resignation and quitclaim.
During her pre-employment interview, Lee had nice comments about her good work experience and
educational background. She was assured of a long-term employment with benefits. Throughout her
employment, she earnestly performed her duties, had a perfect attendance record, worked even during
brownouts and typhoons, and would often work overtime just to finish her work.
Sy claimed that the remarks of her superiors about her alleged inefficiency were ill-motivated and made
without any basis. She had been rendering services for almost six (6) months before she was arbitrarily and
summarily dismissed. Her dismissal was highly suspicious as it took place barely four (4) days prior to the
completion of her six-month probationary period. The petitioners did not show her any evaluation or appraisal
report regarding her alleged inefficient performance. As she was terminated without an evaluation on her
performance, she was deprived of the opportunity to be regularly part of the company and to be entitled to the
benefits and privileges of a regular employee. Worse, she was deprived of her only means of livelihood.
For their part, the petitioners asserted that before Sy was hired, she was apprised that she was being
hired as a probationary employee for six months, subject to extension as a regular employee conditioned on her
meeting the standards of permanent employment set by the company. Her work performance was thereafter
monitored and evaluated. She was formally informed that her employment would end because she failed to
meet the companys standards. From then on, Sy started threatening the families of the petitioners with bodily
harm. They pointed out that the unpredictable attitude of Sy was one of the reasons for her not being considered
for regular employment.
The foregoing circumstances prompted Sy to file a case for illegal dismissal with claims for back wages,
th
unpaid salary, service incentive leave, overtime pay, 13 month pay, and moral and exemplary damages, and
attorneys fees.

ISSUE: Whether the termination of Sy, a probationary employee, was valid or not.

HELD:
No. The pertinent law governing the present case is Article 281 of the Labor Code which provides as
follows:

Art. 281. Probationary employment. Probationary employment shall not exceed six months from
the date the employee started working, unless it is covered by an apprenticeship agreement
stipulating a longer period. The services of an employee who has been engaged in a
probationary basis may be terminated for a just cause or when he fails to qualify as a regular
employee in accordance with reasonable standards made known by the employer to the
employee at the time of his engagement. An employee who is allowed to work after a
probationary period shall be considered a regular employee.

There is probationary employment where the employee upon his engagement is made to undergo a trial
period during which the employer determines his fitness to qualify for regular employment based on reasonable
[14]
standards made known to him at the time of engagement. The probationary employment is intended to afford
the employer an opportunity to observe the fitness of a probationary employee while at work, and to ascertain
whether he will become an efficient and productive employee. While the employer observes the fitness,
propriety and efficiency of a probationer to ascertain whether he is qualified for permanent employment, the
probationer, on the other hand, seeks to prove to the employer that he has the qualifications to meet the
reasonable standards for permanent employment. Thus, the word probationary, as used to describe the period
of employment, implies the purpose of the term or period, not its length.
It is settled that even if probationary employees do not enjoy permanent status, they are accorded the
constitutional protection of security of tenure. This means they may only be terminated for a just cause or when
they otherwise fail to qualify as regular employees in accordance with reasonable standards made known to
them by the employer at the time of their engagement.

San Beda College of Law 554


4S: 2015 - 2016
LABOR LAW REVIEW Atty. Joyrich Golangco

In this case, the justification given by the petitioners for Sys dismissal was her alleged failure to qualify
by the companys standard. Other than the general allegation that said standards were made known to her at
the time of her employment, however, no evidence, documentary or otherwise, was presented to substantiate
the same. Neither was there any performance evaluation presented to prove that indeed hers was
unsatisfactory.
For failure of the petitioners to support their claim of unsatisfactory performance by Sy, this Court shares
the view of the CA that Sys employment was unjustly terminated to prevent her from acquiring a regular status
in circumvention of the law on security of tenure.As the Court previously stated, this is a common and
convenient practice of unscrupulous employers to circumvent the law on security of tenure. Security of tenure,
which is a right of paramount value guaranteed by the Constitution, should not be denied to the workers by such
a stratagem. The Court cannot permit such a subterfuge, if it is to be true to the law and social justice.
Even on the assumption that Sy indeed failed to meet the standards set by them and made known to
the former at the time of her engagement, still, the termination was flawed for failure to give the required notice
to Sy. Section 2, Rule I, Book VI of the Implementing Rules provides:

Section 2. Security of tenure.


xxx
If the termination is brought about by the completion of a contract or phase thereof,
or by failure of an employee to meet the standards of the employer in the case of probationary
employment, it shall be sufficient that a written notice is served the employee, within a
reasonable time from the effective date of termination. [Emphasis and Underscoring supplied]

In this case, the petitioners failed to comply with the requirement of a written notice. Notably, Sy was
merely verbally informed that her employment would be terminated, as admitted by the petitioners. Considering
that the petitioners failed to observe due process in dismissing her, the dismissal had no legal sanction. It bears
stressing that a workers employment is property in the constitutional sense.

San Beda College of Law 555


4S: 2015 - 2016
LABOR LAW REVIEW Atty. Joyrich Golangco

HACIENDA PRIMERA DEVELOPMENT CORPORATION and ANNA KATRINA E. HERNANDEZ v. MICHAEL
S. VILLEGAS
G.R. No. 186243 April 11, 2011
NACHURA, J.:

ART. 281 [Now ART. 296: PROBATIONARY EMPLOYMENT]

DOCTRINE: Due process dictates that a probationary employee be apprised beforehand of the condition of his
employment and of the terms of advancement therein, unless he is deemed to have been hired from day one as
a regular employee.

FACTS:
Petitioner Hacienda hired respondent Villegas as General Manager of Amorita Resort. He was hired as a
probationary employee for three (3) months. Respondent started working for petitioner on January 1, 2007. On
March 14, 2007, he received a call from Paramount Consultancy and Management telling him to report back to
Manila. There, he learned that his services were terminated. He, thus, asked for a written notice of termination,
but did not receive any. Hence, the complaint for illegal dismissal.

Petitioner Hacienda, on the other hand, stated that respondent was hired as probationary employee. It explained
that respondents services were terminated because he failed to qualify for regular employment. Specifically, it
claimed that respondent failed to conceptualize and complete financial budgets, sales projection, room rates,
website development, and marketing plan in coordination with the Sales and Marketing Manager.

LA Ruling:
The LA found that respondent Villegas was illegally dismissed.

NLRC Ruling:
The NLRC dismissed the claim of illegal dismissal.

CA Ruling:
The CA reinstated the ruling of the LA, declaring that respondent Villegas was illegally dismissed.

ISSUE: Is respondent Villegas legally dismissed?

HELD: NO, respondent Villegas is illegally dismissed.

A probationary employee or probationer is one who is on trial for an employer, during which the latter
determines whether or not he is qualified for permanent employment. The probationary employment is intended
to afford the employer an opportunity to observe the fitness of a probationary employee while at work, and to
ascertain whether he will become an efficient and productive employee. While the employer observes the
fitness, propriety and efficiency of a probationer to ascertain whether he is qualified for permanent employment,
the probationer, on the other hand, seeks to prove to the employer that he has the qualifications to meet the
reasonable standards for permanent employment. Thus, the word probationary, as used to describe the period
of employment, implies the purpose of the term or period, not its length.

It can be gleaned from the provisions of law and jurisprudential pronouncement that there are two grounds to
legally terminate a probationary employee. It may be done either: a) for a just cause; or b) when the employee
fails to qualify as a regular employee in accordance with reasonable standards made known by the employer to
the employee at the start of the employment.

In this case, petitioner Hacienda fails to specify the reasonable standards by which respondents alleged poor
performance was evaluated, much less to prove that such standards were made known to him at the start of his
employment. Thus, he is deemed to have been hired from day one as a regular employee. Due process dictates
that an employee be apprised beforehand of the condition of his employment and of the terms of advancement
therein.

San Beda College of Law 556


4S: 2015 - 2016
LABOR LAW REVIEW Atty. Joyrich Golangco

UNIVERSIDAD DE STA. ISABEL vs MARVIN-JULIAN L. SAMBAJON, JR.
G.R. Nos. 196280 & 196286 April 2, 2014
VILLARAMA, JR., J.:

ART. 281 [Now ART. 296: PROBATIONARY EMPLOYEMENT]

FACTS:
Universidad de Sta. Isabel (petitioner) is a non-stock, non-profit religious educational institution in Naga City.
Petitioner hired Marvin-Julian L. Sambajon, Jr. (respondent) as a full-time college faculty member with the rank
4
of Assistant Professor on probationary status, as evidenced by an Appointment Contract dated November 1,
2002, effective November 1, 2002 up to March 30, 2003.

After the aforesaid contract expired, petitioner continued to give teaching loads to respondent who remained a
full-time faculty member of the Department of Religious Education for the two semesters of school-year (SY)
2003-2004 (June 1, 2003 to March 31, 2004); and two semesters of SY 2004-2005 (June 2004 to March 31,
5
2005).

Sometime in June 2003, respondent submitted his masters degree for the purpose of salary
adjustment/increase. Subsequently, respondents salary was increased, as reflected in his pay slips starting
October 1-15, 2004. In a letter dated October 15, 2004 addressed to the President of petitioner, Sr. Ma.
Asuncion G. Evidente, D.C., respondent vigorously argued that his salary increase should be made effective as
of June 2003 and demanded the payment of his salary differential.

The school replied that The Faculty Manual as revised for school year 2002-2003 provides (page 38) "Re-
ranking is done every two years, hence the personnel hold their present rank for two years. It also argued that
personnel file shows that respondent were hired as a probationary teacher in the second semester of school
year 2002-2003. By October 2004, he will be completing four (4) semesters (two school years) of service. Even
permanent teachers are re-ranked only every two years, and he is not even a permanent teacher.

ISSUES:
1. Whether or not respondent is a probationary employee
2. Whether or not he was illegally dismissed.

HELD:
1. A probationary employee is one who is on trial by the employer during which the employer determines
whether or not said employee is qualified for permanent employment. A probationary appointment is made to
afford the employer an opportunity to observe the fitness of a probationary employee while at work, and to
ascertain whether he will become a proper and efficient employee. The word probationary as used to describe
the period of employment implies the purpose of the term or period, but not its length

ART. 281. Probationary Employment.Probationary employment shall not exceed six (6) months from the date
the employee started working, unless it is covered by an apprenticeship agreement stipulating a longer period.
The services of an employee who has been engaged on a probationary basis may be terminated for a just
cause or when he fails to qualify as a regular employee in accordance with reasonable standards made known
by the employer to the employee at the time of his engagement. An employee who is allowed to work after a
probationary period shall be considered a regular employee.

The probationary employment of teachers in private schools is not governed purely by the Labor Code. The
Labor Code is supplemented with respect to the period of probation by special rules found in the Manual of
24
Regulations for Private Schools. On the matter of probationary period, Section 92 of the 1992 Manual of
Regulations for Private Schools regulations states:

Section 92. Probationary Period. Subject in all instances to compliance with the Department and school
requirements, the probationary period for academic personnel shall not be more than three (3) consecutive
years of satisfactory service for those in the elementary and secondary levels, six (6) consecutive regular
semesters of satisfactory service for those in the tertiary level, and nine (9) consecutive trimesters of satisfactory
service for those in the tertiary level where collegiate courses are offered on a trimester basis. (Emphasis
supplied.)

San Beda College of Law 557


4S: 2015 - 2016
LABOR LAW REVIEW Atty. Joyrich Golangco

Thus, it is the Manual of Regulations for Private Schools, and not the Labor Code, that determines whether or
25 26
not a faculty member in an educational institution has attained regular or permanent status. Section 93 of the
1992 Manual of Regulations for Private Schools provides that full-time teachers who have satisfactorily
completed their probationary period shall be considered regular or permanent.

There can be no dispute that the period of probation may be reduced if the employer, convinced of the fitness
and efficiency of a probationary employee, voluntarily extends a permanent appointment even before the three-
year period ends. Conversely, if the purpose sought by the employer is neither attained nor attainable within the
said period, the law does not preclude the employer from terminating the probationary employment on justifiable
ground; or, a shorter probationary period may be incorporated in a collective bargaining agreement. But absent
any circumstances which unmistakably show that an abbreviated probationary period has been agreed upon,
32
the three-year probationary term governs.

The circumstance that respondents services were hired on semester basis did not negate the applicable
probationary period, which is three school years or six consecutive semesters.

2. Notwithstanding the limited engagement of probationary employees, they are entitled to constitutional
41
protection of security of tenure during and before the end of the probationary period. The services of an
employee who has been engaged on probationary basis may be terminated for any of the following: (a) a just or
(b) an authorized cause; and (c) when he fails to qualify as a regular employee in accordance with reasonable
42
standards prescribed by the employer.

Thus, while no vested right to a permanent appointment had as yet accrued in favor of respondent since he had
not completed the prerequisite three-year period (six consecutive semesters) necessary for the acquisition of
43 44
permanent status as required by the Manual of Regulations for Private Schools -- which has the force of law
-- he enjoys a limited tenure. During the said probationary period, he cannot be terminated except for just or
authorized causes, or if he fails to qualify in accordance with reasonable standards prescribed by petitioner for
the acquisition of permanent status of its teaching personnel.

In a letter dated February 26, 2005, petitioner terminated the services of respondent stating that his probationary
employment as teacher will no longer be renewed upon its expiry on March 31, 2005, respondents fifth
semester of teaching. No just or authorized cause was given by petitioner. Prior to this, respondent had
consistently achieved above average rating based on evaluation by petitioners officials and students. He had
also been promoted to the rank of Associate Professor after finishing his masters degree course on his third
semester of teaching. Clearly, respondents termination after five semesters of satisfactory service was illegal.

Respondent therefore is entitled to continue his three-year probationary period, such that from March 31, 2005,
his probationary employment is deemed renewed for the following semester (1st semester of SY 2005-2006).
However, given the discordant relations that had arisen from the parties dispute, it can be inferred with certainty
that petitioner had opted not to retain respondent in its employ beyond the three-year period.

San Beda College of Law 558


4S: 2015 - 2016
LABOR LAW REVIEW Atty. Joyrich Golangco

UNIVAC DEVELOPMENT, INC. vs. WILLIAM M. SORIANO
G.R. No. 182072 June 19, 2013
PERALTA, J.:

FACTS:
5
The case stemmed from the Complaint for Illegal Dismissal filed by respondent against petitioner, the
companys Chairperson Sadamu Watanabe (Watanabe), and the Head of the Engineering Department Johnny
Castro (Castro). Admittedly, respondent was hired on August 23, 2004 by petitioner on probationary basis as
6
legal assistant of the company with a monthly salary of P15,000.00. Respondent claimed that on February 15,
2005, or eight (8) days prior to the completion of his six months probationary period, Castro allegedly informed
him that he was being terminated from employment due to the companys cost-cutting measures. He allegedly
8
asked for a thirty-day notice but his termination was ordered to be effective immediately. Thus, he was left with
9
no choice but to leave the company.

Petitioner, on the other hand, denied the allegations of respondent and claimed instead that prior to his
employment, respondent was informed of the standards required for regularization. Petitioner also supposedly
informed him of his duties and obligations which included safekeeping of case folders, proper coordination with
10
the companys lawyers, and monitoring of the status of the cases filed by or against the company. Petitioner
recalled that on January 5, 2005, a company meeting was held where respondent allegedly expressed his
intention to leave the company because he wanted to review for the bar examinations. It was also in that
meeting where he was informed of his unsatisfactory performance in the company. Thus, when respondent did
not report for work on February 16, 2005, petitioner assumed that he pushed through with his plan to leave the
11
company. In other words, petitioner claimed that respondent was not illegally dismissed from employment,
rather, he in fact abandoned his job by his failure to report for work.

ISSUE: Whether or not Soriano was illegally dismissed

HELD:
Art. 281. Probationary Employment. Probationary employment shall not exceed six (6) months from the date
the employee started working, unless it is covered by an apprenticeship agreement stipulating a longer period.
The services of an employee who has been engaged on a probationary basis may be terminated for a just
cause or when he fails to qualify as a regular employee in accordance with reasonable standards made known
by the employer to the employee at the time of his engagement. An employee who is allowed to work after a
probationary period shall be considered a regular employee.

Probationary employment shall be governed by the following rules:


xxxx

(c) The services of an employee who has been engaged on probationary basis may be terminated only for a just
or authorized cause, when he fails to qualify as a regular employee in accordance with the reasonable
standards prescribed by the employer.
(d) In all cases of probationary employment, the employer shall make known to the employee the standards
under which he will qualify as a regular employee at the time of his engagement. Where no standards are made
known to the employee at that time, he shall be deemed a regular employee.

It is undisputed that respondent was hired as a probationary employee.1wphi1 As such, he did not enjoy a
permanent status. Nevertheless, he is accorded the constitutional protection of security of tenure which means
that he can only be dismissed from employment for a just cause or when he fails to qualify as a regular
employee in accordance with reasonable standards made known to him by the employer at the time of his
30
engagement.

It is primordial that at the start of the probationary period, the standards for regularization be made known to the
31
probationary employee. In this case, as held by the CA, petitioner failed to present adequate evidence to
substantiate its claim that respondent was apprised of said standards.

Indeed, the power of the employer to terminate a probationary employee is subject to three limitations, namely:
(1) it must be exercised in accordance with the specific requirements of the contract; (2) the dissatisfaction on
the part of the employer must be real and in good faith, not feigned so as to circumvent the contract or the law;
32
and (3) there must be no unlawful discrimination in the dismissal. In this case, not only did petitioner fail to
San Beda College of Law 559
4S: 2015 - 2016
LABOR LAW REVIEW Atty. Joyrich Golangco

show that respondent was apprised of the standards for regularization but it was likewise not shown how these
standards had been applied in his case.

Pursuant to well-settled doctrine, petitioners failure to specify the reasonable standards by which respondents
alleged poor performance was evaluated as well as to prove that such standards were made known to him at
the start of his employment, makes respondent a regular employee. In other words, because of this omission on
33
the part of petitioner, respondent is deemed to have been hired from day one as a regular employee.

To justify the dismissal of an employee, the employer must, as a rule, prove that the dismissal was for a just
34
cause and that the employee was afforded due process prior to dismissal. We find no reason to depart from
the CA conclusion that respondents termination from employment is without just and valid ground. Neither was
due process observed, making his termination illegal. He is, therefore, entitled to the twin relief of reinstatement
35
and backwages granted under the Labor Code. However, as aptly held by the CA, considering the strained
relations between petitioner and respondent, separation pay should be awarded in lieu of reinstatement. This
Court has consistently ruled that if reinstatement is no longer feasible, backwages shall be computed from the
36
time of illegal dismissal until the date the decision becomes final. Separation pay, on the other hand, is
equivalent to at least one month pay, or one month pay for every year of service, whichever is higher (with a
37
fraction of at least six months being considered as one whole year), computed from the time of employment or
38
engagement up to the finality of the decision.

San Beda College of Law 560


4S: 2015 - 2016
LABOR LAW REVIEW Atty. Joyrich Golangco

ABBOTT LABORATORIES, PHILIPPINES, CECILLE A. TERRIBLE, EDWIN D. FEIST, MARIA OLIVIA T.
YABUTMISA, TERESITA C. BERNARDO, AND ALLAN G. ALMAZAR vs. PEARLIE ANN F. ALCARAZ
G.R. No. 192571 July 23, 2013
PERLAS-BERNABE, J.:

FACTS:
(Abbott) caused the publication in a major broadsheet newspaper of its need for a Medical and Regulatory
Affairs Manager (Regulatory Affairs Manager) who would: (a) be responsible for drug safety surveillance
operations, staffing, and budget; (b) lead the development and implementation of standard operating
procedures/policies for drug safety surveillance and vigilance; and (c) act as the primary interface with internal
4
and external customers regarding safety operations and queries. Alcaraz - who was then a Regulatory Affairs
and Information Manager at Aventis Pasteur Philippines, Incorporated (another pharmaceutical company like
5
Abbott) showed interest and submitted her application on October 4, 2004.

On December 7, 2004, Abbott formally offered Alcaraz the abovementioned position which was an item under
6 7
the companys Hospira Affiliate Local Surveillance Unit (ALSU) department. In Abbotts offer sheet. it was
8
stated that Alcaraz was to be employed on a probationary basis. Later that day, she accepted the said offer and
received an electronic mail (e-mail) from Abbotts Recruitment Officer, petitioner Teresita C. Bernardo
(Bernardo), confirming the same. Attached to Bernardos e-mail were Abbotts organizational chart and a job
9
description of Alcarazs work.

On February 12, 2005, Alcaraz signed an employment contract which stated, inter alia, that she was to be
placed on probation for a period of six (6) months beginning February 15, 2005 to August 14, 2005.

On March 3, 2005, petitioner Maria Olivia T. Yabut-Misa (Misa), Abbotts Human Resources (HR) Director, sent
Alcaraz an e-mail which contained an explanation of the procedure for evaluating the performance of
probationary employees and further indicated that Abbott had only one evaluation system for all of its
employees. Alcaraz was also given copies of Abbotts Code of Conduct and Probationary Performance
Standards and Evaluation (PPSE) and Performance Excellence Orientation Modules (Performance Modules)
12
which she had to apply in line with her task of evaluating the Hospira ALSU staff.

Abbotts PPSE procedure mandates that the job performance of a probationary employee should be formally
reviewed and discussed with the employee at least twice: first on the third month and second on the fifth month
from the date of employment. The necessary Performance Improvement Plan should also be made during the
third-month review in case of a gap between the employees performance and the standards set. These
performance standards should be discussed in detail with the employee within the first two (2) weeks on the job.
It was equally required that a signed copy of the PPSE form must be submitted to Abbotts Human Resources
Department (HRD) and shall serve as documentation of the employees performance during his/her
probationary period. This shall form the basis for recommending the confirmation or termination of the
13
probationary employment.

On May 23, 2005, Walsh, Almazar, and Bernardo personally handed to Alcaraz a letter stating that her services
21
had been terminated effective May 19, 2005. The letter detailed the reasons for Alcarazs termination
particularly, that Alcaraz: (a) did not manage her time effectively; (b) failed to gain the trust of her staff and to
build an effective rapport with them; (c) failed to train her staff effectively; and (d) was not able to obtain the
knowledge and ability to make sound judgments on case processing and article review which were necessary
22
for the proper performance of her duties. On May 27, 2005, Alcaraz received another copy of the said
23
termination letter via registered mail.

Alcaraz felt that she was unjustly terminated from her employment and thus, filed a complaint for illegal
dismissal and damages against Abbott and its officers, namely, Misa, Bernardo, Almazar, Walsh, Terrible, and
24
Feist. She claimed that she should have already been considered as a regular and not a probationary
employee given Abbotts failure to inform her of the reasonable standards for her regularization upon her
25
engagement as required under Article 295 of the Labor Code.

ISSUE: Whether or not respondent was properly dismissed from work

HELD: The employer is made to comply with two (2) requirements when dealing with a probationary employee:
first, the employer must communicate the regularization standards to the probationary employee; and second,
San Beda College of Law 561
4S: 2015 - 2016
LABOR LAW REVIEW Atty. Joyrich Golangco

the employer must make such communication at the time of the probationary employees engagement. If the
employer fails to comply with either, the employee is deemed as a regular and not a probationary employee.

Keeping with these rules, an employer is deemed to have made known the standards that would qualify a
probationary employee to be a regular employee when it has exerted reasonable efforts to apprise the
employee of what he is expected to do or accomplish during the trial period of probation. This goes without
saying that the employee is sufficiently made aware of his probationary status as well as the length of time of
the probation.

The exception to the foregoing is when the job is self-descriptive in nature, for instance, in the case of maids,
61 62
cooks, drivers, or messengers. Also, in Aberdeen Court, Inc. v. Agustin, it has been held that the rule on
notifying a probationary employee of the standards of regularization should not be used to exculpate an
employee who acts in a manner contrary to basic knowledge and common sense in regard to which there is no
need to spell out a policy or standard to be met. In the same light, an employees failure to perform the duties
and responsibilities which have been clearly made known to him constitutes a justifiable basis for a probationary
employees non-regularization.

A punctilious examination of the records reveals that Abbott had indeed complied with the above-stated
requirements. This conclusion is largely impelled by the fact that Abbott clearly conveyed to Alcaraz her duties
and responsibilities as Regulatory Affairs Manager prior to, during the time of her engagement, and the incipient
stages of her employment.

As the records show, Alcaraz's dismissal was effected through a letter dated May 19, 2005 which she received
on May 23, 2005 and again on May 27, 2005. Stated therein were the reasons for her termination, i.e., that after
proper evaluation, Abbott determined that she failed to meet the reasonable standards for her regularization
considering her lack of time and people management and decision-making skills, which are necessary in the
66
performance of her functions as Regulatory Affairs Manager. Undeniably, this written notice sufficiently meets
the criteria set forth above, thereby legitimizing the cause and manner of Alcarazs dismissal as a probationary
67
employee under the parameters set by the Labor Code.

Nonetheless, despite the existence of a sufficient ground to terminate Alcarazs employment and Abbotts
compliance with the Labor Code termination procedure, it is readily apparent that Abbott breached its
contractual obligation to Alcaraz when it failed to abide by its own procedure in evaluating the performance of a
probationary employee.
Records show that Abbotts PPSE procedure mandates, inter alia, that the job performance of a probationary
employee should be formally reviewed and discussed with the employee at least twice: first on the third month
and second on the fifth month from the date of employment. Abbott is also required to come up with a
Performance Improvement Plan during the third month review to bridge the gap between the employees
69
performance and the standards set, if any. In addition, a signed copy of the PPSE form should be submitted to
Abbotts HRD as the same would serve as basis for recommending the confirmation or termination of the
70
probationary employment.

In this case, it is apparent that Abbott failed to follow the above-stated procedure in evaluating Alcaraz. For one,
there lies a hiatus of evidence that a signed copy of Alcarazs PPSE form was submitted to the HRD. It was not
even shown that a PPSE form was completed to formally assess her performance. Neither was the performance
evaluation discussed with her during the third and fifth months of her employment. Nor did Abbott come up with
the necessary Performance Improvement Plan to properly gauge Alcarazs performance with the set company
standards.

San Beda College of Law 562


4S: 2015 - 2016
LABOR LAW REVIEW Atty. Joyrich Golangco

COLEGIO DEL SANTISIMO ROSARIO v. ROJO
G.R. No. 170388, September 04, 2013
DEL CASTILLO, J.

FACTS:
Petitioner Colegio del Santisimo Rosario (CSR) hired respondent as a high school teacher on
7
probationary basis for the school years 1992-1993, 1993-1994 and 1994-1995. On April 5, 1995, CSR, through
petitioner Mofada, decided not to renew respondents services. Thus, respondent filed a Complaint for illegal
dismissal. He alleged that since he had served three consecutive school years which is the maximum number of
terms allowed for probationary employment, he should be extended permanent employment. Citing paragraph
75 of the 1970 Manual of Regulations for Private Schools (1970 Manual), respondent asserted that full- time
teachers who have rendered three (3) consecutive years of satisfactory services shall be considered
permanent.
On the other hand, petitioners argued that respondent knew that his Teachers Contract for school year
1994-1995 with CSR would expire on March 31, 1995. Accordingly, respondent was not dismissed but his
probationary contract merely expired and was not renewed. Petitioners also claimed that the three years
14
mentioned in paragraph 75 of the 1970 Manual refer to 36 months, not three school years. And since
respondent served for only three school years of 10 months each or 30 months, then he had not yet served the
three years or 36 months mentioned in paragraph 75 of the 1970 Manual.

ISSUE: Whether respondent has attained the status of a regular employee

HELD:
Yes. Cases dealing with employment on probationary status of teaching personnel are not governed
solely by the Labor Code as the law is supplemented, with respect to the period of probation, by special rules
found in the Manual of Regulations for Private Schools (the Manual). With regard to the probationary period,
Section 92 of the 1992 Manual provides:
Section 92. Probationary Period. Subject in all instances to compliance with the Department and
school requirements, the probationary period for academic personnel shall not be more than
three (3) consecutive years of satisfactory service for those in the elementary and secondary
levels, six (6) consecutive regular semesters of satisfactory service for those in the tertiary level, and
nine (9) consecutive trimesters of satisfactory service for those in the tertiary level where collegiate
courses are offered on a trimester basis.
In this case, petitioners teachers who were on probationary employment were made to enter into a
contract effective for one school year. Thereafter, it may be renewed for another school year, and the
probationary employment continues. At the end of the second fixed period of probationary employment, the
contract may again be renewed for the last time. Such employment for fixed terms during the teachers
probationary period is an accepted practice in the teaching profession.
However, this scheme of fixed-term contract is a system that operates during the probationary period
and for this reason is subject to Article 281 of the Labor Code, which provides:brary
x x x The services of an employee who has been engaged on a probationary basis may be
terminated for a just cause or when he fails to qualify as a regular employee in accordance
with reasonable standards made known by the employer to the employee at the time of his
engagement. An employee who is allowed to work after a probationary period shall be considered a
regular employee.
Unless this reconciliation is made, the requirements of [Article 281] on probationary status would be fully
negated as the school may freely choose not to renew contracts simply because their terms have expired.
That teachers on probationary employment also enjoy the protection afforded by Article 281 of the
Labor Code is supported by Section 93 of the 1992 Manual which provides:
Sec. 93. Regular or Permanent Status. - Those who have served the probationary period shall be made
regular or permanent. Full-time teachers who have satisfactorily completed their probationary
period shall be considered regular or permanent.
The above provision clearly provides that full-time teachers become regular or permanent employees
once they have satisfactorily completed the probationary period of three school years. The use of the
term satisfactorily necessarily connotes the requirement for schools to set reasonable standards to be followed
by teachers on probationary employment.
For teachers on probationary employment, it is incumbent upon the school to have not only set
reasonable standards to be followed by said teachers in determining qualification for regular
employment, the same must have also been communicated to the teachers at the start of the
San Beda College of Law 563
4S: 2015 - 2016
LABOR LAW REVIEW Atty. Joyrich Golangco

probationary period, or at the very least, at the start of the period when they were to be applied. These
terms, in addition to those expressly provided by the Labor Code, would serve as the just cause for the
termination of the probationary contract. The specific details of this finding of just cause must be communicated
to the affected teachers as a matter of due process. Corollarily, should the teachers not have been apprised
of such reasonable standards at the time specified above, they shall be deemed regular employees.
Where no standards are made known to the employee at that time, he shall be deemed a regular employee.
In this case, glaringly absent from petitioners evidence are the reasonable standards that respondent
was expected to meet that could have served as proper guidelines for purposes of evaluating his performance.
Nowhere in the Teachers Contract could such standards be found. Neither was it mentioned that the same
were ever conveyed to respondent. Even assuming that respondent failed to meet the standards set forth by
CSR and made known to the former at the time he was engaged as a teacher on probationary status, still, the
termination was flawed for failure to give the required notice to respondent. This is because Book VI, Rule I,
Section 2 of the IRR of the Labor Code provides:
xxxx
If the termination is brought about by the completion of a contract or phase thereof, or by failure of
an employee to meet the standards of the employer in the case of probationary employment, it
shall be sufficient that a written notice is served the employee, within a reasonable time from the
effective date of termination.
It should be pointed out that absent any showing of unsatisfactory performance on the part of
respondent, it can be presumed that his performance was satisfactory, especially taking into consideration the
fact that even while he was still more than a year into his probationary employment, he was already designated
Prefect of Discipline.

San Beda College of Law 564


4S: 2015 - 2016
LABOR LAW REVIEW Atty. Joyrich Golangco

PHILIPPINE DAILY INQUIRER, INC. v. MAGTIBAY
G.R. No. 164532 July 24, 2007
GARCIA, J.

FACTS:
PDI hired Magtibay, on contractual basis, to assist, for a period of five months, the regular phone
operator. Before the expiration of Magtibays contractual employment, he and PDI agreed to a fifteen-day
contract extension, under the same conditions as the existing contract.
After the expiration of Magtibays contractual employment, as extended, PDI announced the creation
and availability of a new position for a second telephone operator who would undergo probationary
employment.
After the usual interview for the second telephone operator slot, PDI chose to hire Magtibay on a
probationary basis for a period of six (6) months. The signing of a written contract of employment followed.
A week before the end the agreed 6-month probationary period, PDI officer Del Rosario handed
Magtibay his termination paper, grounded on his alleged failure to meet company standards. Aggrieved,
Magtibay immediately filed a complaint for illegal dismissal and damages before the Labor Arbiter.
Magtibay anchored his case principally on the postulate that he had become a regular employee by
operation of law, considering that he had been employed by and had worked for PDI for a total period of ten
months, i.e., four months more than the maximum six-month period provided for by law on probationary
employment. He also claimed that he was not apprised at the beginning of his employment of the performance
standards of the company, hence, there was no basis for his dismissal. Finally, he described his dismissal as
tainted with bad faith and effected without due process.
PDI, for its part, denied all the factual allegations of Magtibay, adding that his previous contractual
employment was validly terminated upon the expiration of the period stated therein. Pressing the point, PDI
alleged that the period covered by the contractual employment cannot be counted with or tacked to the period
for probation, inasmuch as there is no basis to consider Magtibay a regular employee. PDI additionally claimed
that Magtibay was dismissed for violation of company rules and policies, such as allowing his lover to enter and
linger inside the telephone operators booth and for failure to meet prescribed company standards which were
allegedly made known to him at the start through an orientation seminar conducted by the company.

ISSUE: Whether respondent was illegally dismissed.

HELD:
No. The same Labor Code also gives the employer a period within which to determine whether a
particular employee is fit to work for him or not. This employers prerogative is spelled out in the following
provision:
Art. 281. Probationary employment. Probationary employment shall not exceed six (6)
months from the date the employee started working, unless it is covered by an apprenticeship
agreement stipulating a longer period. The services of an employee who has been engaged on
a probationary basis may be terminated for a just cause or when he fails to qualify as a regular
employee in accordance with reasonable standards made known by the employer to the
employee at the time of his engagement. An employee who is allowed to work after a
probationary period shall be considered a regular employee.
Within the limited legal six-month probationary period, probationary employees are still entitled
to security of tenure. It is expressly provided in the afore-quoted Article 281 that a probationary employee may
be terminated only on two grounds: (a) for just cause, or (b) when he fails to qualify as a regular employee in
accordance with reasonable standards made known by the employer to the employee at the time of his
engagement.
PDI invokes the second ground under the premises. In claiming that it had adequately apprised
Magtibay of the reasonable standards against which his performance will be gauged for purposes of permanent
employment, PDI cited the one-on-one seminar between Magtibay and its Personnel Assistant, Ms. Rachel Isip-
Cuzio. PDI also pointed to Magtibays direct superior, Benita del Rosario, who diligently briefed him about his
responsibilities in PDI. These factual assertions were never denied nor controverted by Magtibay. Neither did he
belie the existence of a specific rule prohibiting unauthorized persons from entering the telephone operators
booth and that he violated that prohibition. This notwithstanding, the NLRC and the CA proceeded nonetheless
to rule that the records of the case are bereft of any evidence showing that these rules and regulations form part
of the so-called company standards.
It is on record that Magtibay committed obstinate infractions of company rules and regulations, which in
turn constitute sufficient manifestations of his inadequacy to meet reasonable employment norms. The
San Beda College of Law 565
4S: 2015 - 2016
LABOR LAW REVIEW Atty. Joyrich Golangco

suggestion that Magtibay ought to have been made to understand during his briefing and orientation that he is
expected to obey and comply with company rules and regulations strains credulity for acceptance. The CAs
observation that nowhere can it be found in the list of Basic Responsibility and Specific Duties and
Responsibilities of respondent Magtibay that he has to abide by the duties, rules and regulations that he has
allegedly violated is a strained rationalization of an unacceptable conduct of an employee. Common industry
practice and ordinary human experience do not support the CAs posture. All employees, be they regular or
probationary, are expected to comply with company-imposed rules and regulations, else why establish them in
the first place. Probationary employees unwilling to abide by such rules have no right to expect, much less
demand, permanent employment. We, therefore find sufficient factual and legal basis, duly established by
substantial evidence, for PDI to legally terminate Magtibays probationary employment effective upon the end of
the 6-month probationary period.
It is undisputed that PDI apprised Magtibay of the ground of his termination, i.e., he failed to qualify as a
regular employee in accordance with reasonable standards made known to him at the time of engagement, only
a week before the expiration of the six-month probationary period.
Unlike under the first ground for the valid termination of probationary employment which is for just
cause, the second ground does not require notice and hearing. Due process of law for this second ground
consists of making the reasonable standards expected of the employee during his probationary period known to
him at the time of his probationary employment. By the very nature of a probationary employment, the employee
knows from the very start that he will be under close observation and his performance of his assigned duties
and functions would be under continuous scrutiny by his superiors. It is in apprising him of the standards
against which his performance shall be continuously assessed where due process regarding the second ground
lies, and not in notice and hearing as in the case of the first ground.
PDI was only exercising its statutory hiring prerogative when it refused to hire Magtibay on a permanent
basis upon the expiration of the six-month probationary period. This was established during the proceedings
before the labor arbiter and borne out by the records and the pleadings before the Court. When the NLRC
disregarded the substantial evidence establishing the legal termination of Magtibays probationary employment
and rendered judgment grossly and directly contradicting such clear evidence, the NLRC commits grave abuse
of discretion amounting to lack or excess of jurisdiction. It was, therefore, reversible error on the part of the
appellate court not to annul and set aside such void judgment of the NLRC.

San Beda College of Law 566


4S: 2015 - 2016
LABOR LAW REVIEW Atty. Joyrich Golangco

RADIN C. ALCIRA vs. NATIONAL LABOR RELATIONS COMMISSION, MIDDLEBY PHILIPPINES
CORPORATION/FRANK THOMAS, XAVIER G. PEA and TRIFONA F. MAMARADLO
G.R. No. 149859 June 9, 2004
CORONA, J.:

DOCTRINES (Art. 281 [Now Art. 296: Probationary Employment]):

- The computation of the 6-month probationary period is reckoned from the date of appointment up to the same
calendar date of the 6th month following.

- Apprising the employee that he will be subjected to a performance evaluation on a particular date after his
hiring is substantial compliance with the legal requirement of making known to the probationary employee the
standards that would qualify the employee for regular employment.

- Even if probationary employees do not enjoy permanent status, they are accorded the constitutional protection
of security of tenure. But this constitutional protection ends on the expiration of the probationary period. On that
date, the parties are free to either renew or terminate their contract of employment.

FACTS:

Middleby Philippines Corporation hired Alciras on a probationary basis for six months. The parties in this case,
presenting their respective copies of Alciras appointment paper, claimed conflicting starting dates of
employment: May 20, 1996 according to Alciras and May 27, 1996 according to Middleby. Both documents
indicated Alciras employment status as probationary (6 mos.) and a remark that after five months Alciras
performance shall be evaluated and any adjustment in salary shall depend on his work performance. On
November 20, 1996, apparently unhappy with Alciras performance, Middleby terminated his services.

Considering this as a dismissal after the lapse of his probationary employment, Alciras filed on November 21,
1996 a complaint in the NLRC against Middleby contending that he had already become a regular employee as
of the date he was illegally dismissed. In their defense, Middleby claim that Alciras failed to meet company
standards during his probationary employment.

Rebuffed by both the NLRC and the CA, Alciras sought relief from the SC claiming that under the terms of his
contract, his probationary employment was only for five months; that he already attained the status of a regular
employee when he was dismissed on November 20, 1996 because, having started work on May 20, 1996, the
six-month probationary period ended on November 16, 1996; and that Middleby never informed him of the
standards for regularization at the start of his employment.

ISSUES:

1. Did the six-month probationary period lapse earlier than November 20, 1996?
2. Did Middleby comply with the legal mandate to inform the probationary employee of the standards for
regularization?
3. Did Alciras become a regular employee justifying his claim of illegal dismissal?

RULINGS:

1. NO. In CALS Poultry Supply Corp. v. Roco, it was held that the computation of the 6-month probationary
period is reckoned from the date of appointment up to the same calendar date of the 6th month following.

In short, since the number of days in each particular month was irrelevant, Alciras was still a probationary
employee when respondent Middleby opted not to regularize him on November 20, 1996.

2. YES. Apprising the employee that he will be subjected to a performance evaluation on a particular date after
his hiring is substantial compliance with the legal requirement of making known to the probationary employee
the standards that would qualify the employee for regular employment.

Here, Middleby substantially notified Alciras of the standards to qualify as a regular employee when it apprised
him, at the start of his employment, that it would evaluate his supervisory skills after five months.
San Beda College of Law 567
4S: 2015 - 2016
LABOR LAW REVIEW Atty. Joyrich Golangco

3. NO. It is settled that even if probationary employees do not enjoy permanent status, they are accorded the
constitutional protection of security of tenure. But this constitutional protection ends on the expiration of the
probationary period. On that date, the parties are free to either renew or terminate their contract of employment.

Here, respondent Middleby exercised its option not to renew the contract when it informed petitioner on the last
day of his probationary employment that it did not intend to grant him a regular status.

San Beda College of Law 568


4S: 2015 - 2016
LABOR LAW REVIEW Atty. Joyrich Golangco

YOLANDA M. MERCADO, CHARITO S. DE LEON, DIANA R. LACHICA, MARGARITO M. ALBA, JR., and
FELIX A. TONOG v. AMA COMPUTER COLLEGE-PARAAQUE CITY, INC.
G.R. No. 183572 April 13, 2010
BRION, J.:

DOCTRINE (Art. 281 [Now Art. 296: Probationary Employment]):


The probationary status of teaching personnel they are not governed purely by the Labor Code. The Labor Code
is supplemented with respect to the period of probation by special rules found in the Manual of Regulations for
Private Schools. It is important to note that in a situation where the probationary status overlaps with a fixed-
term contract not specifically used for the fixed term it offers, Article 281 should assume primacy and the fixed-
period character of the contract must give way.

FACTS:

Yolanda M. Mercado et al. are all former faculty members of AMA Computer College (AMACC) who started
teaching there on May 25, 1998. They executed individual Teachers Contracts for each of the trimesters that
they were engaged to teach, with the following common stipulation:

1. POSITION. The TEACHER has agreed to accept a non-tenured appointment to work in the
College of xxx effective xxx to xxx or for the duration of the last term that the TEACHER is given a
teaching load based on the assignment duly approved by the DEAN/SAVP-COO.

For the school year 2000-2001, AMACC implemented new Performance Appraisal System for Teachers
(PAST). Under the new screening guidelines, teachers were to be hired or maintained based on extensive
teaching experience, capability, potential, high academic qualifications and research background. The
petitioners failed to obtain a passing rating based on the performance standards. On September 7, 2000,
Mercado et al. individually received a memorandum from AMACC, informing them that with the expiration of
their contract to teach, their contract would no longer be renewed.

Mercado et al. now claim that their dismissal was illegal contending, among others, that AMACC failed to give
them adequate notice; hence, their dismissal was ineffectual. In response, AMACC contended that they worked
under a contracted term under a non-tenured appointment and were still within the three-year probationary
period for teachers. Their contracts were not renewed for the following term because they failed to pass the
PAST while others failed to comply with the other requirements for regularization, promotion, or increase in
salary. This move, according to AMACC, was justified since the school has to maintain its high academic
standards.

ISSUE:

Did AMACC validly terminate Mercado et al.s teaching services?

RULING:

NO. The probationary status of teaching personnel they are not governed purely by the Labor Code. The Labor
Code is supplemented with respect to the period of probation by special rules found in the Manual of
Regulations for Private Schools. It provides that the probationary period for academic personnel shall not be
more than xxx nine (9) consecutive trimesters of satisfactory service xxx. It is important to note that in a situation
where the probationary status overlaps with a fixed-term contract not specifically used for the fixed term it
offers,2 Article 281 should assume primacy and the fixed-period character of the contract must give way.


2 To highlight what we mean by a fixed-term contract specifically used for the fixed term it offers, a replacement
teacher, for example, may be contracted for a period of one year to temporarily take the place of a permanent teacher on
a one-year study leave. The expiration of the replacement teachers contracted term, under the circumstances, leads to no
probationary status implications as she was never employed on probationary basis; her employment is for a specific
purpose with particular focus on the term and with every intent to end her teaching relationship with the school upon
expiration of this term (The Court, p. 29 of the decision).
San Beda College of Law 569
4S: 2015 - 2016
LABOR LAW REVIEW Atty. Joyrich Golangco

In the case at bar, Mercado et al were found to be not merely fixed term employees as their fixed term contracts
were not specifically used for the fixed term it offered. Their termination effected well-within their probationary
period and the belated notice of the standards for their regularization (PAST), runs afoul the security of tenure
granted by the Constitution to probationary employees.

Evidently, Mercado et al. were illegally dismissed.

San Beda College of Law 570


4S: 2015 - 2016
LABOR LAW REVIEW Atty. Joyrich Golangco

LOLITA S. CONCEPCION v. MINEX IMPORT CORPORATION/MINERAMA CORPORATION, KENNETH
MEYERS, SYLVIA P. MARIANO, and VINA MARIANO
G.R. No. 153569 January 24, 2012
BERSAMIN, J.:

TERMINATION BY EMPLOYER: Loss of Trust and Confidence

DOCTRINE: The employer may validly dismiss for loss of trust and confidence an employee who commits an
act of fraud prejudicial to the interest of the employer. Neither a criminal prosecution nor a conviction beyond
reasonable doubt for the crime is a requisite for the validity of the dismissal. Nonetheless, the dismissal for a just
or lawful cause must still be made upon compliance with the requirements of due process under the Labor
Code; otherwise, the employer is liable to pay nominal damages as indemnity to the dismissed employee.

FACTS: Minex Import-Export Corporation (Minex) engaged in the retail of semi-precious stones, selling them in
kiosks or stalls installed in various shopping centers within Metro Manila. It employed Concepcion initially as a
salesgirl. It made her a supervisor in July 1997. On October 23, 1997, Vina Mariano, an Assistant Manager of
Minex, assigned the petitioner to the SM Harrison Plaza kiosk with the instruction to hold the keys of the kiosk.

Concepcion, however was later on implicated in the theft of more than P50,000.00 in cash sales. After
undergoing initial investigation, she pleaded her superiors to allow her to return to work but one of them told her
that they do not want to see her face anymore. She later on filed a complaint for illegal dismissal.

Thereafter, a complaint for qualified theft was lodged against her. The prosecutors found probable cause
against her and an information was filed.

ISSUES:
1. Was Concepcions dismissal valid?
2. Was Concepcion accorded due process as regards her dismissal?

RULINGS:
1. YES. The employer may validly dismiss for loss of trust and confidence an employee who commits an act of
fraud prejudicial to the interest of the employer. Neither a criminal prosecution nor a conviction beyond
reasonable doubt for the crime is a requisite for the validity of the dismissal. It is unfair to require an employer to
first be morally certain of the guilt of the employee by awaiting a conviction before terminating him when there is
already sufficient showing of the wrongdoing. Requiring that certainty may prove too late for the employer,
whose loss may potentially be beyond repair.

Here, no less than the DOJ Secretary found probable cause for qualified theft against the petitioner. That finding
was enough to justify her termination for loss of confidence.

2. NO. An employees right to procedural due process prior to dismissal cannot be denied. The law requires that
compliance with the two notice rule and that the employee be given an opportunity to be heard and defend
herself.

Here, Concepcion plainly demonstrated how quickly and summarily her dismissal was carried out without first
requiring her to explain anything in her defense. Instead, her superiors forthwith had her arrested and
investigated by the police authorities for qualified theft. This was a denial of her right to due process of law.

For such disregard of Concepcions right, the employers must pay her P30,000.00 in nominal damages.

San Beda College of Law 571


4S: 2015 - 2016
LABOR LAW REVIEW Atty. Joyrich Golangco

DREAMLAND HOTEL RESORT and WESTLEY J. PRENTICE vs. STEPHEN B. JOHNSON
G.R. No. 191455 March 12, 2014
REYES, J.:

TERMINATION BY EMPLOYER: Constructive Dismissal

DOCTRINE: There is constructive dismissal if an act of clear discrimination, insensibility, or disdain by an


employer becomes so unbearable on the part of the employee that it would foreclose any choice by him except
to forego his continued employment. It exists where there is cessation of work because continued employment
is rendered impossible, unreasonable or unlikely, as an offer involving a demotion in rank and a diminution in
pay.

FACTS: Dreamland is a corporation while Prentice is its current President and Chief Executive Officer.
Respondent Stephen B. Johnson is an Australian citizen who came to the Philippines as a
businessman/investor without the authority to be employed as the employee/officer of any business as he was
not able to secure his Alien Employment Permit AEP. Prentice alleged that Johnson invest in Dreamland and at
the same time provide his services as Operations Manager of Dreamland with a promise that he will secure an
AEP and TIN.The employment contract shall commence on Aug. 1, 2007. However the resort opened on Oct 8,
2007. During the interim Johnson was tasked to supervise the construction. Another argument posited by the
petitioners is that the employment contract executed by the parties is inefficacious because the employment
contract is subject to the presentation of Johnson of his Alien Employment Permit (AEP) and Tax Identification
Number (TIN).

Johnson, alleged that it was Prentice who made the advertisement. It was in response to these advertisements
that private respondent Johnson contacted petitioners to inquire on the terms for employment offered. It was
Prentice who offered employment and convinced Johnson to give out a loan, purportedly so the resort can be
completed and operational by August 2007. Johnson accepted the employment as Resort Manager and loaned
money to petitioners his retirement pay in the amount of (USD 100,000.00) to finish construction of the resort.
He found out that it was not yet finished. As [Johnson] remained unpaid since August 2007 and he has loaned
all his money to petitioners, he asked for his salary after the resort was opened in October 2007 but the same
was not given to him by petitioners. [Johnson] was also not given the authority due to him as resort manager.
Respondent Johnson was forced to submit his resignation because of the act of Prentice humiliating him in front
of the staff. Ultimately, Johnson filed a Complaint for illegal dismissal and non-payment of salaries, among
others, against the petitioners. Johnson has adduced proof that as a permanent resident, he is exempted from
the requirement of securing an AEP as expressed under Department Order No. 75-06, Series of 2006 of the
Department of Labor and Employment (DOLE). Furthermore, Johnson submitted a Certification from DOLE
Regional Office III, stating that he is exempted from securing an AEP as a holder of Permanent Resident Visa.

ISSUES:
(1) WON Johnson is an employee of Dreamland.
(2) WON he was constructively dismissed.

RULING:
(1) Yes. This is evidenced by their employment contract. The fact that Johnson did not furnished AEP and TIN
number will not invalidate the contract since nowhere in the contract did it state that failure to submit the said
document would invalidate the employment. Another, failure of submission of AEP will not invalidate the contract
because Johnson is exempted from such requirement.

(2) Yes. The SC upheld the findings of NLRC that theres no abandonment of work but a constructive
dismissal, which is defined as an involuntary resignation resorted to when continued employment is rendered
impossible, unreasonable or unlikely.

"There is constructive dismissal if an act of clear discrimination, insensibility, or disdain by an employer


becomes so unbearable on the part of the employee that it would foreclose any choice by him except to forego
his continued employment. It exists where there is cessation of work because continued employment is
rendered impossible, unreasonable or unlikely, as an offer involving a demotion in rank and a diminution in pay."

It is impossible, unreasonable or unlikely that any employee, such as Johnson would continue working for an
employer who does not pay him his salaries. The Court construes that the act of the petitioners in not paying
San Beda College of Law 572
4S: 2015 - 2016
LABOR LAW REVIEW Atty. Joyrich Golangco

Johnson his salaries for three months has become unbearable on the latters part that he had no choice but to
cede his employment with them. While it was Johnson who tendered his resignation, it was due to the
petitioners acts that he was constrained to resign. The petitioners cannot expect Johnson to tolerate working for
them without any compensation.

Since Johnson was constructively dismissed, he was illegally dismissed. Thus, an illegally dismissed employee
is entitled to two reliefs: backwages and reinstatement. The two reliefs provided are separate and distinct. In
instances where reinstatement is no longer feasible because of strained relations between the employee and
the employer, separation pay is granted. In effect, an illegally dismissed employee is entitled to either
reinstatement, if viable, or separation pay if reinstatement is no longer viable, and backwages.

The normal consequences of respondents illegal dismissal, then, are reinstatement without loss of seniority
rights, and payment of backwages computed from the time compensation was withheld up to the date of actual
reinstatement. But in this case, NLRC found that due to the strained relations between the parties, separation
pay is to be awarded to Johnson in lieu of his reinstatement.

The NLRC held that Johnson is entitled to backwages up to the finality of the decision; separation pay
equivalent to one month salary; and unpaid salaries.Separation pay is computed from the commencement of
employment up to the time of termination, including the imputed service for which the employee is entitled to
backwages. As one-month salary is awarded as separation pay for every year of service, including imputed
service, Johnson should be paid separation pay equivalent to his three-month salary for the three-year contract.

San Beda College of Law 573


4S: 2015 - 2016
LABOR LAW REVIEW Atty. Joyrich Golangco

GRAND ASIAN SHIPPING LINES, INC., EDUARDO P. FRANCISCO and WILLIAM HOW vs. GALVEZ et.al.
G.R. No. 17818 January 29, 2014
DEL CASTILLO, J.:

TERMINATION BY EMPLOYER: Burden of Proof

DOCTRINE: The rule that the employer bears the burden of proof in illegal dismissal cases finds no application
when the employer denies having dismissed the employee. The employee must first establish by substantial
evidence the fact of dismissal before shifting to the employer the burden of proving the validity of such
dismissal.

FACTS: Galvez was employed as Captain and Gruta as Chief engineer. On the other hand, Arguelles,
Batayola, Fresnillo, Noble, Dominico, Nilmao and Austral were employed as second engineers. (All
respondents)

On 2000, one of the vessels Oilers, Abis, reported to the Manager an alleged illegal activity being committed by
respondents aboard the vessel. Abis revealed that after about four to five voyages a week, a substantial volume
of fuel oil is unconsumed and stored in the vessels fuel tanks. However, Gruta would misdeclare it as
consumed fuel in the Engineers Voyage Reports. Then, the saved fuel oil is siphoned and sold to other vessels
out at sea usually at nighttime. Respondents would then divide among themselves the proceeds of the sale. An
investigation on the alleged pilferage was conducted. After audit and examination Certification of Overstatement
of Fuel Oil Consumption was submitted to the management. Then a formal complaint for qualified theft was
filed. Meanwhile, respondentswere placed under preventive suspension. After conducting administrative
hearings, petitioners decided to terminate respondents from employment. Respondents (except Sales) were
thus served with notices informing them of their termination for serious misconduct, willful breach of trust, and
commission of a crime or offense against their employer.

ISSUE: WON there is a valid termination as to


(a) Galvez and Gruta
(b) Second engineers Batayola et al.
(c) Sales

Ruling: Galvez and Gruta were validly dismissed on the ground of loss of trust and confidence; there were no
valid grounds for the dismissal of Arguelles, Batayola, Fresnillo, Noble, Dominico, Nilmao and Austral; Sales
was not dismissed in the first place, hence not illegally dismissed.

(a) Galvez and Grutas termination, which is loss of trust and confidence, distinction should be made between
managerial and rank and file employees. "[W]ith respect to rank-and-file personnel, loss of trust and confidence,
as ground for valid dismissal, requires proof of involvement in the alleged events [while for] managerial
employees, the mere existence of a basis for believing that such employee has breached the trust of his
employer would suffice for his dismissal."

In the case before us, Galvez, as the ship captain, is considered a managerial employee since his duties involve
the governance, care and management of the vessel. Gruta, as chief engineer, is also a managerial employee
for he is tasked to take complete charge of the technical operations of the vessel. As captain and as chief
engineer, Galvez and Gruta perform functions vested with authority to execute management policies and
thereby hold positions of responsibility over the activities in the vessel. Indeed, their position requires the full
trust and confidence of their employer for they are entrusted with the custody, handling and care of company
property and exercise authority over it.

Thus, we find that there is some basis for the loss of confidence reposed on Galvez and Gruta. The certification
issued by De la Rama stated that there is an overstatement of fuel consumption. Notably, while respondents
made self-serving allegations that the computation made therein is erroneous, they never questioned the
competence of De la Rama to make such certification. Neither did they question the authenticity and validity of
the certification. Thus, the fact that there was an overstatement of fuel consumption and that there was loss of a
considerable amount of diesel fuel oil remained unrefuted. Their failure to account for this loss of company
property betrays the trust reposed and expected of them. They had violated petitioners trust and for which their
dismissal is justified on the ground of breach of confidence.

San Beda College of Law 574


4S: 2015 - 2016
LABOR LAW REVIEW Atty. Joyrich Golangco

(b) As for Arguelles, Batayola, Fresnillo, Noble, Dominico, Nilmao and Austral, proof of involvement in the loss
of the vessels fuel as well as their participation in the alleged theft is required for they are ordinary rank and file
employees. And as discussed above, no substantial evidence exists in the records that would establish their
participation in the offense charged. This renders their dismissal illegal, thus, entitling them to reinstatement plus
full backwages, inclusive of allowances and other benefits, computed from the time of their dismissal up to the
time of actual reinstatement.

(c) The rule that the employer bears the burden of proof in illegal dismissal cases finds no application when the
employer denies having dismissed the employee. The employee must first establish by substantial evidence the
fact of dismissal before shifting to the employer the burden of proving the validity of such dismissal.

We give credence to petitioners claim that Sales was not dismissed from employment. Unlike the other
respondents, we find no evidence in the records to show that Sales was preventively suspended, that he was
summoned and subjected to any administrative hearing and that he was given termination notice. It is likewise
worth noting that in the Supplemental Complaint Affidavit of Montegrico, Sales was not included in the list of
those employees who were accused of having knowledge of the alleged pilferage. This only shows that he was
never subjected to any accusation or investigation as a prelude to termination. Hence, it would be pointless to
determine the legality or illegality of his dismissal because, in the first place, he was not dismissed from
employment.

San Beda College of Law 575


4S: 2015 - 2016
LABOR LAW REVIEW Atty. Joyrich Golangco

MIRANT (PHILIPPINES) CORPORATION AND EDGARDO A. BAUTISTA vs. JOSELITO A. CARO
G.R. No. 181490 April 23, 2014
VILLARAMA, JR., J.:

TERMINATION BY EMPLOYER: Reasonableness of Termination

DOCTRINE: The [petitioner corporations] Anti-Drug Policy is excessive in terminating an employee for his
"unjustified refusal" to subject himself to the random drug test on first offense, without clearly defining what
amounts to an "unjustified refusal and that for the ten-year period that respondent had been employed by
petitioner corporation, he did not have any record of a violation of its company policies.

FACTS: Caro was hired by petitioner corporation as its Logistics Officer. At the time of the filing of the
complaint, respondent was already a Supervisor at the Logistics and Purchasing Department.

Petitioner Corporation conducted a random drug test where Caro was randomly chosen among its employees
who would be tested for illegal drug use. The same day, he received a phone call from his wifes colleague who
informed him that a bombing incident occurred near his wifes work station in Tel Aviv, Israel where his wife was
then working as a caregiver. He proceeded to the Israeli Embassy to confirm the news on the alleged bombing
incident.

On that same day, Caro returned to Petitioner Corporations office. He immediately called up to explain the
reasons for his failure to submit himself to the random drug test that day. He also proposed that he would submit
to a drug test the following day at his own expense.

Caro received a Show Cause Notice from the corporation through Jaime Dulot (Dulot), his immediate
supervisor, requiring him to explain in writing why he should not be charged with "unjustified refusal to submit to
random drug testing." The Investigating Panel issued an Investigating Report finding respondent guilty of
"unjustified refusal to submit to random drug testing" and recommended a penalty of four working weeks
suspension without pay, instead of termination, due to the presence of mitigating circumstances. The Vice
President for Material Management Department recommended that respondent be terminated from employment
instead of merely being suspended. Lamela argued that even if respondent did not outrightly refuse to take the
random drug test, he avoided the same. Lamela averred that "avoidance" was synonymous with "refusal." He
was subsequently terminated on the same day. Caro demanded that he was not accorded due process.

ISSUE: Won Caro was illegally terminated.

RULING: Yes. Caro was illegally terminated.

While the adoption and enforcement by Petitioner Corporation of its Anti-Drugs Policy is recognized as a valid
exercise of its management prerogative as an employer, such exercise is not absolute and unbridled.
Managerial prerogatives are subject to limitations provided by law, collective bargaining agreements, and the
general principles of fair play and justice. In the exercise of its management prerogative, an employer must
therefore ensure that the policies, rules and regulations on work-related activities of the employees must always
be fair and reasonable and the corresponding penalties, when prescribed, commensurate to the offense
involved and to the degree of the infraction. The Anti-Drugs Policy of Mirant fell short of these requirements.

Petitioner corporations subject Anti-Drugs Policy fell short of being fair and reasonable.
First. The policy was not clear on what constitutes "unjustified refusal" when the subject drug policy prescribed
that an employees "unjustified refusal" to submit to a random drug test shall be punishable by the penalty of
termination for the first offense. To be sure, the term "unjustified refusal" could not possibly cover all forms of
"refusal" as the employees resistance, to be punishable by termination, must be "unjustified." To the mind of the
Court, it is on this area where petitioner corporation had fallen short of making it clear to its employees as well
as to management as to what types of acts would fall under the purview of "unjustified refusal."

The fact that petitioner corporations own Investigating Panel and its Vice President for Operations, Sliman,
differed in their recommendations regarding respondents case are first-hand proof that there, indeed, is
ambiguity in the interpretation and application of the subject drug policy. The fact that petitioner corporations
own personnel had to dissect the intended meaning of "unjustified refusal" is further proof that it is not clear on
what context the term "unjustified refusal" applies to. It is therefore not a surprise that the Labor Arbiter, the
San Beda College of Law 576
4S: 2015 - 2016
LABOR LAW REVIEW Atty. Joyrich Golangco

NLRC and the CA have perceived the term "unjustified refusal" on different prisms due to the lack of parameters
as to what comes under its purview. To be sure, the fact that the courts and entities involved in this case had to
engage in semantics and come up with different constructions is yet another glaring proof that the subject
policy is not clear creating doubt that respondents dismissal was a result of petitioner corporations valid
exercise of its management prerogative.

It is not a mere jurisprudential principle, but an enshrined provision of law, that all doubts shall be resolved in
favor of labor. Thus, in Article 4 of the Labor Code, as amended, "[a]ll doubts in the implementation and
interpretation of the provisions of [the Labor] Code, including its implementing rules and regulations, shall be
resolved in favor of labor." In Article 1702 of the New Civil Code, a similar provision states that "[i]n case of
doubt, all labor legislation and all labor contracts shall be construed in favor of the safety and decent living for
the laborer." Applying these provisions of law to the circumstances in the case at bar, it is not fair for this Court
to allow an ambiguous policy to prejudice the rights of an employee against illegal dismissal. To hold otherwise
and sustain the stance of petitioner corporation would be to adopt an interpretation that goes against the very
grain of labor protection in this jurisdiction. As correctly stated by the Labor Arbiter, "when a conflicting interest
of labor and capital are weighed on the scales of social justice, the heavier influence of the latter must be
counter-balanced by the sympathy and compassion the law must accord the underprivileged worker."

Second. The penalty of termination imposed by petitioner corporation upon respondent fell short of being
reasonable. Company policies and regulations are generally valid and binding between the employer and the
employee unless shown to be grossly oppressive or contrary to law as in the case at bar. Recognizing the
ambiguity in the subject policy, the CA was more inclined to adopt the recommendation of petitioner
corporations own Investigating Panel over that of Sliman and the NLRC. The appellate court succinctly but
incisively pointed out, viz.:

x x x We find, as correctly pointed out by the investigating panel, that the [petitioner corporations] Anti-Drug
Policy is excessive in terminating an employee for his "unjustified refusal" to subject himself to the random drug
test on first offense, without clearly defining what amounts to an "unjustified refusal."

To be sure, the unreasonableness of the penalty of termination as imposed in this case is further highlighted by
a fact admitted by petitioner corporation itself: that for the ten-year period that respondent had been employed
by petitioner corporation, he did not have any record of a violation of its company policies.

San Beda College of Law 577


4S: 2015 - 2016
LABOR LAW REVIEW Atty. Joyrich Golangco

BLUER THAN BLUE JOINT VENTURES COMPANY/MARY ANN DELA VEGA vs. GLYZA ESTEBAN
G.R. No. 192582 April 7, 2014
REYES, J.:

TERMINATION BY EMPLOYER: Loss of Trust and Confidence

DOCTRINE: It is not the job title but the actual work that the employee performs that determines whether he or
she occupies a position of trust and confidence.

FACTS: Glyza Esteban was employed in January 2004 as Sales Clerk, and assigned at Bluer Than Blue Joint
Ventures Company's EGG boutique in SM City Marilao, Bulacan, beginning the year 2006. Part of her primary
tasks were attending to all customer needs, ensuring efficient inventory, coordinating orders from clients,
cashiering and reporting to the accounting department.

In November 2006, the Esteban received a report that several employees have access to its point-of-sale (POS)
system through a universal password given by Elmer Flores. Upon investigation, it was discovered that it was
Esteban who gave Flores the password. BTB sent a letter memorandum to Esteban, asking her to explain in
writing why she should not be disciplinary dealt with for tampering with the companys POS system through the
use of an unauthorized password. Esteban was also placed under preventive suspension for ten days.

Esteban admitted that she used the universal password three times on the same day in December 2005, in
good faith, as she was told by two other employees browsing sales inquiry that they used the "123456"
password. A notice of termination was sent to her, finding her explanation unsatisfactory and terminating her
employment immediately on the ground of loss of trust and confidence. Esteban filed a complaint for illegal
dismissal, illegal suspension, holiday pay, rest day and separation pay.

Labor Arbiter: ruled in favor of Esteban and found that she was illegally dismissed.
NLRC: reversed the decision of the LA and dismissed the case for illegal dismissal.
CA: granted Estebans petition and reinstated the LA decision.

ISSUE: Whether Estebans acts constitute just cause to terminate her employment with the company on the
ground of loss of trust and confidence. NO

HELD: NO. Esteban is, no doubt, a rank-and-file employee. Among the fiduciary rank-and-file employees are
cashiers, auditors, property custodians, or those who, in the normal exercise of their functions, regularly handle
significant amounts of money or property. These employees, though rank-and-file, are routinely charged with the
care and custody of the employers money or property, and are thus classified as occupying positions of trust
and confidence. Esteban is considered as a rank-and-file employee occupying a position of trust and
confidence.

Loss of trust and confidence to be a valid cause for dismissal must be work related such as would show the
employee concerned to be unfit to continue working for the employer and it must be based on a wilful breach of
trust and founded on clearly established facts. In this case, the Court finds that the acts committed by Esteban
do not amount to a wilful breach of trust. To the Courts mind, Estebans lapse is, at best, a careless act that
does not merit the imposition of the penalty of dismissal.

San Beda College of Law 578


4S: 2015 - 2016
LABOR LAW REVIEW Atty. Joyrich Golangco

MANILA JOCKEY CLUB, INC. vs. AIMEE O. TRAJANO
G.R. No. 160982 June 26, 2013
BERSAMIN, J.:

TERMINATION BY EMPLOYER: Loss of Trust and Confidence

DOCTRINE: A breach is willful if it is done intentionally, knowingly and purposely, without justifiable excuse, as
distinguished from an act done carelessly, thoughtlessly, heedlessly or inadvertently. It must rest on substantial
grounds and not on the employers arbitrariness, whims, caprices or suspicion; otherwise, the employee would
eternally remain at the mercy of the employer. An ordinary breach is not enough.

FACTS:
Manila Jockey Club, Inc. (MJCI) employed Aimee Trajano as a selling teller of betting tickets since November
1989. In 1998, two regular bettors gave her their respective lists of bets (rota) and money for the bets for Race
14. She mistakenly cancelled the ticket of the second bettor when the first bettor requested for cancellation. To
complicate things, it was also the same bet that had won Race 14. At that point, she realized her mistake, and
explained to the second bettor that the cancellation of his ticket had not been intentional, but the result of an
honest mistake on her part. She offered to personally pay the dividends should the second bettor win Race 15,
which the latter accepted.

To her surprise, the reliever-supervisor later approached Trajano and told her to submit a written explanation
about the ticket cancellation incident. The next day, she submitted the handwritten explanation. Later that day,
she was informed that she was being placed under preventive suspension for an unstated period of time. At the
end of thirty days of her suspension, Trajano reported for work. But she was no longer admitted. She then
learned that she had been dismissed, as posted in a selling station of MJCI.

Trajano instituted a complaint for illegal dismissal against MJCI in DOLE. MJCI maintained that Trajanos
dismissal was justified because the unauthorized cancellation of the ticket had constituted a serious violation of
company policy amounting to dishonesty.

Labor Arbiter: dismissed the complaint for illegal dismissal upon finding that Trajanos gross negligence in the
performance of her job warranted the termination of her employment.
NLRC: reversed and set aside the decision of the Labor Arbiter and declared Trajano to have been illegally
dismissed by MJCI without just or authorized cause and without due process of law.
CA: upheld the NLRC.

ISSUES:
Whether there was just cause when MJCI dismissed Trajano from the service.

HELD:
NO. Loss of the employers trust and confidence is a just cause under Article 282 (c) of the Labor Code, a
provision that ideally applies only to cases involving an employee occupying a position of trust and confidence,
or to a situation where the employee has been routinely charged with the care and custody of the employers
money or property. But the loss of trust and confidence, to be a valid ground for dismissal, must be based on a
willful breach of trust and confidence founded on clearly established facts. A breach is willful if it is done
intentionally, knowingly and purposely, without justifiable excuse, as distinguished from an act done carelessly,
thoughtlessly, heedlessly or inadvertently. It must rest on substantial grounds and not on the employers
arbitrariness, whims, caprices or suspicion; otherwise, the employee would eternally remain at the mercy of the
employer. An ordinary breach is not enough. Moreover, the loss of trust and confidence must be related to the
employees performance of duties.

As a selling teller, Trajano held a position of trust and confidence. The nature of her employment required her to
handle and keep in custody the tickets issued and the bets made in her assigned selling station. The bets were
funds belonging to her employer. MJCI did not establish that the cancellation of the ticket was intentional,
knowing and purposeful on her part in order for her to have breached the trust and confidence reposed in her by
MJCI, instead of being only out of an honest mistake. MJCIs prejudice remained speculative and unrealized. To
dismiss an employee based on speculation as to the damage the employer could have suffered would be an
injustice.

San Beda College of Law 579


4S: 2015 - 2016
LABOR LAW REVIEW Atty. Joyrich Golangco

The Court concludes that the reinstatement of Trajano is no longer feasible. More than 14 years have already
passed since she initiated her complaint for illegal dismissal in 1998. Consequently, an award of separation pay
has become the practical alternative, computed at one month pay for every year of service. Trajano is entitled to
full backwages, inclusive of allowances and other benefits or their monetary equivalent.

San Beda College of Law 580


4S: 2015 - 2016
LABOR LAW REVIEW Atty. Joyrich Golangco

REXIE A. HORMILLOSA vs. COCA-COLA BOTTLERS PHILS., INC., represented by its Iloilo Plant Human
Resource Head, ROBERTO RICHARD H. DOLAR
G.R. No. 198699 October 9, 2013
MENDOZA, J.:

TERMINATION BY EMPLOYER: Loss of Trust and Confidence

DOCTRINE: The first requisite for dismissal on the ground of loss of trust and confidence is that the employee
concerned must be one holding a position of trust and confidence. The second requisite is that there must be an
act that would justify the loss of trust and confidence. It must be based on a willful breach of trust and founded
on clearly established facts.

FACTS:
In 1996, Hormillosa was employed as a route salesman by Coca-Cola Bottlers Phils., Inc. (CBPI). His duties
included, among others, selling CBPIs soft drink products, either on cash or on credit basis; receiving payments
from proceeds of the sale or payments of past due or current accounts; issuing sales invoices; and receiving
empty bottles and cases of soft drinks. Due to their delicate position, route salesmen, like Hormillosa, were
given a handbook entitled, CCBPI Employee Code of Disciplinary Rules and Regulations.

In 1999, the then CBPI District Sales Supervisor, Raul S. Tiosayco III, conducted a verification and audit of the
accounts handled by Hormillosa. He discovered transactions in violation of the handbook, specifically "Fictitious
sales transactions; Falsification of company records, etc., among others. Hormillosa was then placed on
grounded status and investigated. He was subsequently terminated. In addition to his termination, CBPI also
filed several criminal cases against him citing his fraudulent acts. Even after the termination, more anomalies
committed by Hormillosa were discovered.

Hormillosa averred in his position paper that prior to his dismissal, he was a member of the Board of Directors of
CBPIs employees union and he became its secretary. As secretary, he sent a copy of the new list of union
officers to the management with a warning that if CBPI would not stop harassing the members of the union, it
would declare a strike.

Labor Arbiter: dismissed Hormillosas complaint for illegal dismissal, ruling that his termination was proper.
NLRC: ordered the remand of the case to give Hormillosa the opportunity to confront the witnesses and
evidence against him.
CA: nullified and set aside the NLRC decision and held that the dismissal of Hormillosa was valid.

ISSUE:
Whether Hormillosa was validly dismissed. YES

HELD:
YES. Article 282 of the Labor Code enumerates the just causes for the termination of employment of an
employee by the employer. The CA was correct when it ruled that Hormillosas employment was validly
terminated under paragraph (c). There was substantial evidence to justify his dismissal. It is clear that Article
282(c) of the Labor Code allows an employer to terminate the services of an employee for loss of trust and
confidence.

The first requisite for dismissal on the ground of loss of trust and confidence is that the employee concerned
must be one holding a position of trust and confidence. There are two classes of positions of trust. The first
class consists of managerial employees. They are defined as those vested with the powers or prerogatives to
lay down management policies and to hire, transfer suspend, lay-off, recall, discharge, assign or discipline
employees or effectively recommend such managerial actions. The second class consists of cashiers, auditors,
property custodians, etc. They are defined as those who in the normal and routine exercise of their functions,
regularly handle significant amounts of money or property.

The second requisite is that there must be an act that would justify the loss of trust and confidence. It must be
based on a willful breach of trust and founded on clearly established facts. The basis for the dismissal must be
clearly and convincingly established but proof beyond reasonable doubt is not necessary.

San Beda College of Law 581


4S: 2015 - 2016
LABOR LAW REVIEW Atty. Joyrich Golangco

Hormillosa, being a route salesman, falls under the second class. Salesmen are highly individualistic personnel
who have to be trusted and left essentially on their own. A high degree of confidence is reposed on them
because they are entrusted with funds or properties of their employer. Clearly, Hormillosa occupies a position of
trust. Hormillosa committed acts which warranted his dismissal from employment. The falsified invoices
remained unexplained by him. Evidently, Hormillosa disregarded and disobeyed the company rules. The breach
was willful. The tampering of the invoices were clear enough.

In the case at bench, the cause for the dismissal from employment of Hormillosa clearly falls under Article 282
of the Labor Code. Therefore, he is not entitled to any separation pay.

San Beda College of Law 582


4S: 2015 - 2016
LABOR LAW REVIEW Atty. Joyrich Golangco

ERIC V. CHUANICO vs. LEGACY CONSOLIDATED PLANS, INC.
G.R. No. 181852 October 9, 2013
ABAD, J.:

TERMINATION BY EMPLOYER: Loss of Trust and Confidence

DOCTRINE: Settled is the rule, however, that under Article 282( c) of the Labor Code, the breach of trust must
be willful. Ordinary breach will not be enough. A breach is willful if it is done intentionally and knowingly without
any justifiable excuse, as distinguished from an act done carelessly, thoughtlessly or inadvertently.

FACTS:
Legacy Plans hired petitioner Atty. Chuanico as Asst Vice- President for legal services. He was to serve as in-
house counsel for the company and its subsidiaries under the supervision Atty. Cruz, the Senior Vice-Pres for
Legal Affairs. In the same year, Legacy Plans merged with Consolidated Plans Philippines, Inc. to become
Legacy Consolidated, the respondent in this case. Its legal services unit served all its affiliates.

Subsequently Atty. Cruz wrote Atty. Chuanico a memorandum, requiring him to explain why no administrative
action should be taken against him for mishandling two cases.

First case: he was supposed to draft an answer to a complaint for Bank of East Asia (a Legacy Consolidated
affiliate) but he belatedly drafted a haphazard one that he gave to the handling lawyers without coursing it to his
superior.

His defense: that he was given only one day within which to finish the draft. While admitting that his superior
had no opportunity to review it for lack of time, he denied that the answer had been haphazardly done

Second case: Atty. Chuanico was required to prepare a complaint-affidavit for the Rural Bank of Paraaque
(also an affiliate) against a certain De Rama but he failed to do so.

His defense: that the case had not actually been turned over to him. It was originally assigned to Atty. Dennis
Amparo who later said that the complaint-affidavit could not be prepared because the Rural Bank had no
witness.

Legacy Consolidated dismissed Atty. Chuanico with for serious misconduct, willful disobedience to lawful
orders, gross and habitual neglect of duties, and willful breach of trust. He filed complaint for illegal dismissal
with claims for his unpaid salary and 13th-month pay plus damages and attorneys fees

LA: Legacy Consolidated is guilty of illegal dismissal; entitled to full backwages, separation pay and 13th-month
pay

Findings (first charge): that Atty. Chuanico actually drafted an answer for Bank of East Asia but the companys
two new lawyers did not like it and chose to file one that they themselves prepared; Legacy Consolidated neither
bothered to present Atty. Chuanicos draft answer nor explained why it regarded the same as haphazardly done
nor present evidence that the bank filed a late answer on account of Atty. Chuanicos fault

Second charge: gave credence to Atty. Dennis Amparos sworn statement that it was to Atty. Cruz, not to Atty.
Chuanico, that he personally turned over the cases he was handling.

NLRC: affirmed the LAs Decision; Legacy Consolidated failed to present evidence to prove that Atty. Chuanico
violated some company rules or his superiors order; employer gave him no notice of these alleged violations

CA: NLRC committed grave abuse of discretion in holding Legacy guilty of illegal dismissal but affirmed, the
award to of13th-month pay. Atty. Chuanico moved for reconsideration but the CA denied his motion hence this
petition.

ISSUE: Whether or not the CA erred in holding that the NLRC committed grave abuse of discretion in finding
that Legacy Consolidated illegally dismissed Atty. Chuanico for mishandling the two cases alleged to have been
assigned to him.

San Beda College of Law 583


4S: 2015 - 2016
LABOR LAW REVIEW Atty. Joyrich Golangco

RULING: No. Atty. Chuanico was not a mere rank-and-file employee but an in-house counsel. Thus, Legacy
Consolidated enjoyed wide latitude in evaluating his work and attitude and in terminating his employment on the
ground of loss of trust and confidence. His mishandling of the cases assigned to him shows that he had been
unfit to continue working for his employer.

But these are broad principles that do not themselves show when, where, and how Atty. Chuanico betrayed the
trust that Legacy Consolidated gave him as in-house counsel. To be a valid cause for dismissal, the loss of trust
must be based on a willful breach of such trust and founded on clearly established facts. The company charged
him with having mishandled two things that were assigned to him, the drafting of an answer in one and the
preparation of a complaint affidavit in the other. It failed to present proof, however, of such mishandling.

In the first case: Legacy Consolidated did not bother to present the draft-answer Atty. Chuanico prepared and
demonstrate why it regarded the same as haphazardly done. Besides, as Atty. Chuanico said, he was given
only one day within which to finish the draft-answer and Legacy Consolidated did not contest this fact.
Consequently, he could not be expected to do more than an adequate pleading.

Second case: Legacy Consolidated did not bother to present some note or logbook to refute this denial. It only
presented the sworn statement of the office secretary, supposedly competent, who relied merely on her memory
for ascertaining individual work assignments in a law practice that served a number of affiliated companies.

Besides, Atty. Amparo, the former handling lawyer of the Rural Bank case said in his sworn statement that he
had been unable to prepare the required complaint-affidavit because the bank could not produce a witness
against De Rama. Atty. Amparo further added that it was to Atty. Cruz, not to Atty. Chuanico, that he turned over
the Rural Bank's case.

The Court held in CAPANELA v. National Labor Relations Commission that the factual findings of quasi-judicial
bodies, which are triers of facts on matters within their expertise, should be considered, when supported by
substantial evidence, binding and conclusive on appellate courts. Here the LA and the NLRC were in better
positions to assess and evaluate the credibility of the parties' claims and the weight to which the irrespective
evidence is entitled.

Lastly Atty. Chuanico was dismissed due to willful breach of trust. Settled is the rule, however, that under Article
282( c) of the Labor Code, the breach of trust must be willful. Ordinary breach will not be enough. A breach is
willful if it is done intentionally and knowingly without any justifiable excuse, as distinguished from an act done
carelessly, thoughtlessly or inadvertently. Willful breach was not proved in this case.

San Beda College of Law 584


4S: 2015 - 2016
LABOR LAW REVIEW Atty. Joyrich Golangco

DUNCAN ASSOCIATION OF DETAILMAN-PTGWO and PEDRO A. TECSON vs. GLAXO WELLCOME
PHILIPPINES, INC.
G.R. No. 162994 September 17, 2004
TINGA, J.

TERMINATION BY EMPLOYER: Marital Relationship

DOCTRINE: The prohibition against personal or marital relationships with employees of competitor companies
upon Glaxos employees is reasonable under the circumstances because relationships of that nature might
compromise the interests of the company. In laying down the assailed company policy, Glaxo only aims to
protect its interests against the possibility that a competitor company will gain access to its secrets and
procedures.

FACTS:
Petitioner Tecson was hired by respondent Glaxo Wellcome Philippines, Inc. (Glaxo) as medical
representative.He signed a contract of employment which stipulates, among others, that he agrees to study and
abide by existing company rules; to disclose to management any existing or future relationship by consanguinity
or affinity with co-employees or employees of competing drug companies and should management find that
such relationship poses a possible conflict of interest, to resign from the company.

The Employee Code of Conduct of Glaxo similarly provides that if management perceives a conflict of interest or
a potential conflict between such relationship and the employees employment with the company, the
management and the employee will explore the possibility of a "transfer to another department in a non-
counterchecking position" or preparation for employment outside the company after six months.

Tecson was initially assigned to market Glaxos products in the Camarines Sur-Camarines Norte sales area.
Subsequently, Tecson entered into a romantic relationship with Bettsy, an employee of Astra
Pharmaceuticals(Astra), a competitor of Glaxo. Bettsy was Astras Branch Coordinator in Albay. She supervised
the district managers and medical representatives of her company and prepared marketing strategies for Astra
in that area. Tecson and bettsy got martied.

Tecsons superiors informed him that his marriage to Bettsy gave rise to a conflict of interest. Tecsons superiors
reminded him that he and Bettsy should decide which one of them would resign from their jobs, although they
told him that they wanted to retain him as much as possible because he was performing his job well. Tecson
requested for time to comply with the company policy against entering into a relationship with an employee of a
competitor company.

Glaxo transferred Tecson to the Butuan City-Surigao City-Agusan del Sur sales area. Tecson asked Glaxo to
reconsider its decision, but his request was denied.

Tecson defied the transfer order and continued acting as medical representative in the Camarines Sur-
Camarines Norte sales area.

During the pendency of the grievance proceedings, Tecson was paid his salary, but was not issued samples of
products which were competing with similar products manufactured by Astra. He was also not included in
product conferences regarding such products.

For failure to resolve the issue at the grievance machinery level, they submitted the matter for voluntary
arbitration.

NCMB: declared as valid Glaxos policy on relationships between its employees and persons employed with
competitor companies, and affirming Glaxos right to transfer Tecson to another sales territory.

CA: affirmed NCMB; held that it is a valid exercise of its management prerogatives. Petitioners contend that
Glaxos policy against employees marrying employees of competitor companies violates the equal protection
clause of the Constitution because it creates invalid distinctions among employees on account only of marriage.
They claim that the policy restricts the employees right to marry

San Beda College of Law 585


4S: 2015 - 2016
LABOR LAW REVIEW Atty. Joyrich Golangco

ISSUE: Whether the Court of Appeals erred in ruling that Glaxos policy against its employees marrying
employees from competitor companies is valid, and in not holding that said policy violates the equal protection
clause of the Constitution

RULING: No. The stipulation in Tecsons contract of employment with Glaxo being questioned by petitioners
provides:
10. You agree to disclose to management any existing or future relationship you may have, either
by consanguinity or affinity with co-employees or employees of competing drug companies.
Should it pose a possible conflict of interest in management discretion, you agree to resign
voluntarily from the Company as a matter of Company policy.

The same contract also stipulates that Tescon agrees to abide by the existing company rules of
Glaxo, and to study and become acquainted with such policies. In this regard, the Employee
Handbook of Glaxo expressly informs its employees of its rules regarding conflict of interest:
1. Conflict of Interest
Employees should avoid any activity, investment relationship, or interest that may run counter to
the responsibilities which they owe Glaxo Wellcome.
Specifically, this means that employees are expected:
a. To avoid having personal or family interest, financial or otherwise, in any competitor supplier or
other businesses which may consciously or unconsciously influence their actions or decisions
and thus deprive Glaxo Wellcome of legitimate profit.

1.1. Employee Relationships


Employees with existing or future relationships either by consanguinity or affinity with co-
employees of competing drug companies are expected to disclose such relationship to the
Management. If management perceives a conflict or potential conflict of interest, every effort shall
be made, together by management and the employee, to arrive at a solution within six (6)
months, either by transfer to another department in a non-counter checking position, or by career
preparation toward outside employment after Glaxo Wellcome. Employees must be prepared for
possible resignation within six (6) months, if no other solution is feasible.

Glaxo has a right to guard its trade secrets, manufacturing formulas, marketing strategies and other confidential
programs and information from competitors, especially so that it and Astra are rival companies in the highly
competitive pharmaceutical industry.

The prohibition against personal or marital relationships with employees of competitor companies upon Glaxos
employees is reasonable under the circumstances because relationships of that nature might compromise the
interests of the company. In laying down the assailed company policy, Glaxo only aims to protect its interests
against the possibility that a competitor company will gain access to its secrets and procedures.
That Glaxo possesses the right to protect its economic interests cannot be denied. No less than the Constitution
recognizes the right of enterprises to adopt and enforce such a policy to protect its right to reasonable returns on
investments and to expansion and growth.

The challenged company policy does not violate the equal protection clause. Significantly, the company actually
enforced the policy after repeated requests to the employee to comply with the policy. Indeed, the application of
the policy was made in an impartial and even-handed manner, with due regard for the lot of the employee.

In any event, it is clear that Glaxo does not impose an absolute prohibition against relationships between its
employees and those of competitor companies. Its employees are free to cultivate relationships with and marry
persons of their own choosing. What the company merely seeks to avoid is a conflict of interest between the
employee and the company that may arise out of such relationships. Further, Employee Code of Conduct and of
its contracts with its employees, such as that signed by Tescon, was made known to him prior to his
employment. Tecson, therefore, was aware of that restriction when he signed his employment contract and
when he entered into a relationship with Bettsy. Since Tecson knowingly and voluntarily entered into a contract
of employment with Glaxo, the stipulations therein have the force of law between them and, thus, should be
complied with in good faith. He is therefore estopped from questioning said policy.

San Beda College of Law 586


4S: 2015 - 2016
LABOR LAW REVIEW Atty. Joyrich Golangco

STAR PAPER CORPORATION, JOSEPHINE ONGSITCO & SEBASTIAN CHUA vs. RONALDO D. SIMBOL,
WILFREDA N. COMIA & LORNA E. ESTRELLA
G.R. No. 164774 April 12, 2006
PUNO, J.:

TERMINATION BY EMPLOYER: Marital Relationship (Bona fide Occupational Qualification)

DOCTRINE: To justify a bona fide occupational qualification, the employer must prove two factors: (1) that the
employment qualification is reasonably related to the essential operation of the job involved; and, (2) that there
is a factual basis for believing that all or substantially all persons meeting the qualification would be unable to
properly perform the duties of the job.

FACTS: Petitioner Star Paper Corporation (Star) is a corporation engaged in trading principally of paper
products. Ongsitco is its Manager of the Personnel and Administration Department while Sebastian Chua is its
Managing Director. Respondents Simbol, Comia and Estrella were all regular employees of the company.

Version of the Company: Simbol was employed by the company where he met Alma Dayrit, also an employee of
the company, whom he married. Prior to the marriage, Ongsitco advised the couple that should they decide to
get married, one of them should resign pursuant to a company policy promulgated:
2. In case of two of our employees (both singles [sic], one male and another female) developed a
friendly relationship during the course of their employment and then decided to get married, one of them
should resign to preserve the policy stated above

Simbol resigned pursuant to the company policy.

Comia was hired by the company where she met Howard Comia, a co-employee, whom she married. Ongsitco
likewise reminded them hence Comia resigned.

Estrella met Zuiga, also a co-worker. Petitioners stated that Zuiga, a married man, got Estrella pregnant. The
company allegedly could have terminated her services due to immorality but she opted to resign.

The respondents each signed a Release and Confirmation Agreement stating that they have no money and
property accountabilities in the company and that they release the latter of any claim or demand of whatever
nature.

Version of Respondents: Simbol and Comia allege that they did not resign voluntarily but were compelled to
resign in view of an illegal company policy. As to respondent Estrella, she alleges that she had a relationship
with co-worker Zuiga who misrepresented himself as a married but separated man. After he got her pregnant,
she discovered that he was not separated. Thus, she severed her relationship with him to avoid dismissal due to
the company policy. Subsequently, she met an accident and was advised by the doctor at the Orthopedic
Hospital to recuperate for 21 days. She returned to work but she was denied entry. The staff handed her a
memorandum stating that she was being dismissed for immoral conduct which she refused to sign because she
has not been given a chance to explain. The management asked her to write an explanation. However, after
submission of the explanation, she was nonetheless dismissed by the company. Due to her urgent need for
money, she later submitted a letter of resignation in exchange for her thirteenth month pay

Respondents filed a complaint for ULP, constructive dismissal, separation pay and attorneys fees; averred that
the aforementioned company policy is illegal and contravenes Article 136 of the Labor Code; also contended
that they were dismissed due to their union membership.

LA: Dismissed the complaint for lack of merit: valid management prerogative

NLRC: Affirmed LA

CA: Reversed the NLRC; holding that that policy/regulation is violative of the constitutional rights towards
marriage and the family of employees and of Article 136 of the Labor Code; and respondents resignations were
far from voluntary.

San Beda College of Law 587


4S: 2015 - 2016
LABOR LAW REVIEW Atty. Joyrich Golangco

ISSUE: Whether the policy of the employer banning spouses from working in the same company violates the
rights of the employee under the Constitution and the Labor Code or is a valid exercise of management
prerogative.

RULING: The courts that have broadly construed the term "marital status" rule that it encompassed the identity,
occupation and employment of one's spouse. They strike down the no-spouse employment policies based on
the broad legislative intent of the state statute. They reason that the no-spouse employment policy violate the
marital status provision because it arbitrarily discriminates against all spouses of present employees without
regard to the actual effect on the individual's qualifications or work performance. These courts also find the no-
spouse employment policy invalid for failure of the employer to present any evidence of business
necessity other than the general perception that spouses in the same workplace might adversely affect the
business. They hold that the absence of such a bona fide occupational qualification invalidates a rule
denying employment to one spouse due to the current employment of the other spouse in the same office. Thus,
they rule that unless the employer can prove that the reasonable demands of the business require a distinction
based on marital status and there is no better available or acceptable policy which would better accomplish the
business purpose, an employer may not discriminate against an employee based on the identity of the
employees spouse. This is known as the bona fide occupational qualification exception.

We note that since the finding of a bona fide occupational qualification justifies an employers no-spouse rule,
the exception is interpreted strictly and narrowly by these state courts. There must be a compelling business
necessity for which no alternative exists other than the discriminatory practice. To justify a bona fide
occupational qualification, the employer must prove two factors: (1) that the employment qualification is
reasonably related to the essential operation of the job involved; and, (2) that there is a factual basis for
believing that all or substantially all persons meeting the qualification would be unable to properly perform the
duties of the job.

We do not find a reasonable business necessity in the case at bar.

Petitioners sole contention that "the company did not just want to have two (2) or more of its employees related
between the third degree by affinity and/or consanguinity" is lame. That the second paragraph was meant to
give teeth to the first paragraph of the questioned rule is evidently not the valid reasonable business necessity
required by the law.

It is significant to note that in the case at bar, respondents were hired after they were found fit for the job, but
were asked to resign when they married a co-employee. Petitioners failed to show how the marriage could be
detrimental to its business operations.The policy is premised on the mere fear that employees married to each
other will be less efficient. If we uphold the questioned rule without valid justification, the employer can create
policies based on an unproven presumption of a perceived danger at the expense of an employees right to
security of tenure.

The questioned policy may not facially violate Article 136 of the Labor Code but it creates a disproportionate
effect and under the disparate impact theory, the only way it could pass judicial scrutiny is a showing that it
is reasonable despite the discriminatory, albeit disproportionate, effect. The failure of petitioners to prove a
legitimate business concern in imposing the questioned policy cannot prejudice the employees right to be free
from arbitrary discrimination based upon stereotypes of married persons working together in one company.

San Beda College of Law 588


4S: 2015 - 2016
LABOR LAW REVIEW Atty. Joyrich Golangco

ACE PROMOTION AND MARKETING CORPORATION v. REYNALDO URSABIA
G.R. No. 171703, September 22, 2006
YNARES-SANTIAGO, J.:

TERMINATION BY EMPLOYER: Willful Disobedience

DOCTRINE: To be validly dismissed on the ground of willful disobedience requires the concurrence of at least
two requisites:
(1) the employees assailed conduct must have been willful or intentional, the willfulness being
characterized by a wrongful and perverse attitude; and
(2) the order violated must have been reasonable, lawful, made known to the employee and must pertain to
the duties which he had been engaged to discharge.

Intentional failure/refusal to answer a memorandum validly executed by the employer shows a wrongful and
perverse attitude to defy the reasonable orders which undoubtedly pertain to his duties as an employee of
petitioner.

FACTS:
Sometime in August, 1994, petitioner Ace Promotion and Marketing Corporation, hired respondent
Reynaldo Ursabia as a company driver assigned to pick up the products of Nestle Philippines, Inc., for
promotion and marketing. On July 6, 2001, respondent failed to report for work. Petitioner, through its area
supervisor, Gerry Garcia, issued a Memorandum asking the latter why he should not be placed under
disciplinary action for violating a company rule. Ursabia must explain within 24 hours upon receipt of the
memorandum. Ursabia reported back to work on July 09, 2001. He was personally served with the foregoing
memorandum but refused to acknowledge the same, hence the memorandum was sent through registered mail
to his last known address.

The following day, Garcia noticed some damage on the vehicle assigned to Ursabia, hence another
memorandum was issued which requires that Ursabia must explain within 24 hours to explain his side. Failure to
comply, the company will terminate him. The second memorandum was sent through registered mail.

Sometime in July 2001, the company received an anonymous note which states Be careful and save
your life because theres a time to come everybody will die. The PNP Crime Laboratory allegedly showed that
the handwriting of Ursabia has significant similarities with the handwritten on the note.

On August 6, 2001, Ursabia went to the petitioners office and was served with a termination letter.
Again, respondent refused to receive the same prompting petitioner to serve it by registered mail to respondents
last known address.

Meanwhile, the petitioner filed two criminal cases for Malicious Mischief and Grave Threats against the
respondent. Displeased with his termination, respondent filed a complaint for illegal dismissal and non-payment
of other monetary benefits.

On May 9, 2002, Labor Arbiter Jose G. Gutierrez rendered a decision in favor of respondent. On appeal,
the NLRC rendered a decision reversing the decision of the Labor Arbiter.

ISSUES:
1. Whether or not there exists a just cause to dismiss respondent
2. Whether or not he was accorded procedural due process

HELD:

1. YES. We agree with the Court of Appeals that respondent cannot be dismissed for abandonment. To
constitute a just and valid ground for dismissal, abandonment requires the deliberate and unjustified refusal of
the employee to resume his employment. Two elements must be present, namely: (1) the failure to report for
work or absence without valid or justifiable reason, and (2) a clear intention to sever the employer-employee
relationship. In the instant case, the subsequent conduct of respondent after he failed to report for work on July
6, 2001, shows that he had no intention to sever his employment with petitioner.

San Beda College of Law 589


4S: 2015 - 2016
LABOR LAW REVIEW Atty. Joyrich Golangco

The foregoing, notwithstanding, we find that respondent should be dismissed for willful
disobedience of the memoranda issued by petitioner. To be validly dismissed on the ground of willful
disobedience requires the concurrence of at least two requisites: (1) the employees assailed conduct must have
been willful or intentional, the willfulness being characterized by a wrongful and perverse attitude; and (2) the
order violated must have been reasonable, lawful, made known to the employee and must pertain to the duties
which he had been engaged to discharge.

In the instant case, the failure of respondent to answer the July 9 and 10, 2001 memoranda of petitioner
is clearly intentional. He reported to and loitered outside petitioners premises but never made any oral or written
reply to the said memoranda. This shows respondents wrongful and perverse attitude to defy the reasonable
orders which undoubtedly pertain to his duties as an employee of petitioner.

2. NO. the Court finds that respondent was not afforded his procedural due process rights. In dismissing
an employee, the employer has the burden of proving that the former worker has been served two notices: (1)
one to apprise him of the particular acts or omissions for which his dismissal is sought, and (2) the other to
inform him of his employers decision to dismiss him. The first notice must state that dismissal is sought for the
act or omission charged against the employee, otherwise, the notice cannot be considered sufficient compliance
with the rules.

In the instant case, the just cause to terminate respondent was his willful disobedience to the July 9 and
July 10, 2001 memoranda of petitioner. However, he was not given sufficient notice that his services will be
terminated on such grounds. Respondent defied two memoranda of petitioner. Hence, it is necessary that he be
furnished with a third memorandum informing him that his disobedience to the previous two memoranda may
cause his dismissal.

Moreover, the final notice of termination of respondent failed to specify the ground for his dismissal. It
vaguely stated that he is being terminated for violation of company rules which were not specified by
petitioner. Following the Agabon doctrine, the dismissal is valid, but the company is liable to pay Ursabia, a
nominal damage amounting to 30,000.

San Beda College of Law 590


4S: 2015 - 2016
LABOR LAW REVIEW Atty. Joyrich Golangco

LOLITA S. CONCEPCION v. MINEX IMPORT CORPORATION/MINERAMA CORPORATION, KENNETH
MEYERS, SYLVIA P. MARIANO, and VINA MARIANO
G.R. No. 153569, January 24, 2012
BERSAMIN, J.:

TERMINATION BY EMPLOYER: Loss of Trust and Confidence

DOCTRINE: It is not necessary that the employer should await the employees final conviction in the criminal
case involving such fraud or breach of trust before it can terminate the employees services. In fact, even the
dropping of the charges or an acquittal of the employee therefrom does not preclude the dismissal of an
employee for acts inimical to the interests of the employer.

FACTS:
Respondent Vina Mariano assigned petitioner Lolita Concepcion to the SM Harrison Plaza kiosk with
the instruction to hold the keys of the kiosk. At the close of business, they conducted a cash-count of their sales
proceeds amounting to P50,912.00. The next morning, the petitioner phoned Vina Mariano to report that the
P50,912.00 was missing. Later, while the petitioner was giving a detailed statement on the theft to the security
investigator of Harrison Plaza, Vina and Sylvia Mariano, her superiors, arrived with a policeman who
immediately placed the petitioner under arrest, the police investigated her. She was detained for a day being
released only because the inquest prosecutor instructed so.

The petitioner complained against the respondents for illegal dismissal in the Department of Labor and
Employment. Minex, through Vina, filed a complaint for qualified theft against the petitioner in the Office of the
City Prosecutor in Manila.

Labor Arbiter decided in favor of Lolita, declaring the dismissal of the latter from work illegal and
ordering her reinstatement to her former work position with full backwages counted from November 10, 1997
until her actual reinstatement without loss of seniority or other employees rights and benefits.

The NLRC reversed the decision of the Labor Arbiter declaring that the petitioner had not been
dismissed, but had abandoned her job after being found to have stolen the proceeds of the sales; and holding
that even if she had been dismissed, her dismissal would be justifiable for loss of trust and confidence in the
light of the finding of probable cause by the DOJ and the City Prosecutor and the filing of the information for
qualified theft against her.

The CA sustained the NLRC mainly because of the DOJ Secretarys finding of probable cause for
qualified theft. The CA denied the petitioners motion for reconsideration. Hence, the instant petition.

ISSUE: Whether or not the petitioner was terminated for a just and valid cause.

HELD: YES.
To dismiss an employee, the law requires the existence of a just and valid cause. The NLRC held that
the termination of the petitioner was due to loss of trust and confidence. Sustaining the NLRC, the CA stated:
With the finding of probable cause not only by the investigating prosecutor but by the Secretary of
Justice no less, it cannot be validly claimed, as the Petitioner does, in her Petition at bench, that there is no
lawful cause for her dismissal xxx

Admittedly, there is no direct evidence that the Petitioner took the money from the drawer in the cabinet
in the Kiosk. But direct evidence that the Petitioner took the money is not required for the Petitioner to be
lawfully dismissed for the loss of the money of the Private Respondent corporation. If circumstantial evidence is
sufficient on which to anchor a judgment of conviction in criminal cases under Section 4, Rule 133 of the
Revised Rules of Evidence, there is no cogent reason why circumstantial evidence is not sufficient on which to
anchor a factual basis for the dismissal of the Petitioner for loss of confidence.

The petitioner still argues, however, that there was no evidence at all upon which Minex could validly
dismiss her considering that she had not yet been found guilty beyond reasonable doubt of the crime of qualified
theft.

San Beda College of Law 591


4S: 2015 - 2016
LABOR LAW REVIEW Atty. Joyrich Golangco

Fraud or willful breach of trust reposed upon an employee by his employer is a recognized cause for
termination of employment and it is not necessary that the employer should await the employees final
conviction in the criminal case involving such fraud or breach of trust before it can terminate the
employees services. In fact, even the dropping of the charges or an acquittal of the employee therefrom
does not preclude the dismissal of an employee for acts inimical to the interests of the employer.

Yet, even as we now say that the respondents had a just or valid cause for terminating the petitioner, it
becomes unavoidable to ask whether or not they complied with the requirements of due process. The
petitioner plainly demonstrated how quickly and summarily her dismissal was carried out without first requiring
her to explain anything in her defense as demanded under Section 2 (d) of Rule I of the Implementing Rules of
Book VI of the Labor Code. Instead, the respondents forthwith had her arrested and investigated by the police
authorities for qualified theft. This, we think, was a denial of her right to due process of law, consisting in the
opportunity to be heard and to defend herself.

In view of the foregoing, we impose on the respondents the obligation to pay to the petitioner an
indemnity in the form of nominal damages of P30,000.00, conformably with Agabon v. NLRC. Where the
dismissal is for a just cause, as in the instant case, the lack of statutory due process should not nullify
the dismissal, or render it illegal, or ineffectual. However, the employer should indemnify the employee
for the violation of his statutory rights, as ruled in Reta v. National Labor Relations Commission.

San Beda College of Law 592


4S: 2015 - 2016
LABOR LAW REVIEW Atty. Joyrich Golangco

ST. LUKES MEDICAL CENTER, INC. and ROBERT KUAN, Chairman v. ESTRELITO NOTARIO
G.R. No. 152166, October 20, 2010
PERALTA, J.:

TERMINATION BY EMPLOYER: Gross and Habitual Neglect of Duty

DOCTRINE: Neglect of duty, to be a ground for dismissal, must be both gross and habitual. Gross
negligence connotes want of care in the performance of ones duties. Habitual neglect implies repeated failure to
perform ones duties for a period of time, depending upon the circumstances. A single or isolated act of
negligence does not constitute a just cause for the dismissal of the employee.

FACTS:
On June 23, 1995, St. Lukes Medical Center, Inc., QC (petitioner hospital), employed respondent as In-
House Security Guard. A CCTV system was installed in the premises of petitioner hospital to enhance its
security measures and conducted an orientation seminar for the in-house security personnel on the proper way
of monitoring video cameras.

On December 30, 1996, respondent was on duty from 6:00 p.m. to 6:00 a.m. of the following day. His
work consisted mainly of monitoring the video cameras. In the evening of December 30, 1996, Justin Tibon, a
foreigner then attending to his 3-year-old daughter, who was admitted at room 257 of petitioners hospital,
reported to the management of petitioner hospital about the loss of his mint green traveling bag, which was
placed inside the cabinet containing valuables.

Acting on the complaint of Tibon, the Security Department of petitioner hospital conducted an
investigation. When the tapes of video camera recorder (VCR) no. 3 covering the subject period were reviewed,
it was found out that the cameras failed to record any incident of theft at the subject room.

On January 6, 1997, petitioner hospital, issued Memorandum to respondent, the CCTV monitoring staff
on duty, directing him to explain in writing, why no disciplinary action should be taken against him for violating
the normal rotation/sequencing process of the VCR and, consequently, failed to capture the theft of Tibon's
traveling bag at room 257.

In his letter, respondent explained that on the subject dates, he was the only personnel on duty as
nobody wanted to assist him. Because of this, he decided to focus the cameras on the Old and New Maternity
Units, as these two units have high incidence of crime. Finding the written explanation of respondent to be
unsatisfactory, petitioner hospital, served on respondent a copy of the Notice of Termination, dismissing him on
the ground of gross negligence/inefficiency under Section 1, Rule VII of its Code of Discipline.

Thus, on March 19, 1997, respondent filed a Complaint for illegal dismissal against petitioner hospital
and its Chairman, Robert Kuan, seeking reinstatement with payment of full backwages from the time of his
dismissal up to actual reinstatement, without of loss of seniority rights and other benefits.

The Labor Arbiter dismissed respondents complaint for illegal dismissal against petitioners. He said
that respondents infraction exposed petitioners to the possibility of a damage suit that may be filed against them
arising from the theft.

The NLRC reversed decision of the LA. t observed that respondent was not negligent when he focused
the cameras on the Old and New Maternity Units, as they were located near the stairways and elevators, which
were frequented by many visitors and, thus, there is the likelihood that untoward incidents may arise. If at all, it
treated the matter as a single or isolated act of simple negligence which did not constitute a just cause for the
dismissal of an employee.

The CA affirmed NLRC ruling.

ISSUE: Whether or not Notario was illegally dismissed

HELD: YES.

San Beda College of Law 593


4S: 2015 - 2016
LABOR LAW REVIEW Atty. Joyrich Golangco

Contrary to the stance of petitioners, respondent was illegally dismissed without just cause and
compliance with the notice requirement. To effectuate a valid dismissal from employment by the employer, the
Labor Code has set twin requirements, namely: (1) the dismissal must be for any of the causes provided in
Article 282 of the Labor Code; and (2) the employee must be given an opportunity to be heard and defend
himself. This first requisite is referred to as the substantive aspect, while the second is deemed as the
procedural aspect.

Under Article 282 (b) of the Labor Code, an employer may terminate an employee for gross and
habitual neglect of duties. Neglect of duty, to be a ground for dismissal, must be both gross and
habitual. Gross negligence connotes want of care in the performance of ones duties. Habitual neglect implies
repeated failure to perform ones duties for a period of time, depending upon the circumstances. A single or
isolated act of negligence does not constitute a just cause for the dismissal of the employee.

Under the prevailing circumstances, respondent exercised his best judgment in monitoring the CCTV
cameras so as to ensure the security within the hospital premises. Verily, assuming arguendo that respondent
was negligent, although this Court finds otherwise, the lapse or inaction could only be regarded as a single or
isolated act of negligence that cannot be categorized as habitual and, hence, not a just cause for his dismissal.

Petitioners question the findings of the CA that there was no compliance with the twin-notice rule
and hearing, while respondent maintains that they violated his right to due process.

The employee must be furnished two written notices: the first notice apprises the employee of the
particular acts or omissions for which his dismissal is sought, and the second is a subsequent notice, which
informs the employee of the employer's decision to dismiss him. The CA found that petitioner hospital failed to
comply with the rule on twin notice and hearing as it merely required respondent to give his written explanation
within 24 hours and, thereafter, ordered his dismissal.

Petitioners lack of just cause and non-compliance with the procedural requisites in terminating
respondents employment renders them guilty of illegal dismissal. Consequently, respondent is entitled to
reinstatement to his former position without loss of seniority rights and payment of backwages. However, if such
reinstatement proves impracticable, and hardly in the best interest of the parties, perhaps due to the lapse of
time since his dismissal, or if he decides not to be reinstated, respondent should be awarded separation pay in
lieu of reinstatement.

San Beda College of Law 594


4S: 2015 - 2016
LABOR LAW REVIEW Atty. Joyrich Golangco

THE COCA-COLA EXPORT CORPORATION vs. CLARITA P. GACAYAN
G.R. No. 149433 December 15, 2010
LEONARDO-DE CASTRO, J.:

TERMINATION BY EMPLOYER: Loss of Trust and Confidence

DOCTRINE: The guidelines for the application of the doctrine of loss of confidence are:
(a) Loss of confidence should not be simulated;
(b) It should not be used as a subterfuge for causes which are improper, illegal or unjustified;
(c) It may not be arbitrarily asserted in the face of overwhelming evidence to the contrary; and
(d) It must be genuine, not a mere afterthought to justify earlier action taken in bad faith.

FACTS: Gacayans employment was terminated, for alleged loss of trust and confidence; respondent was
holding the position of Senior Financial Accountant. Respondent allegedly altered three receipts which she
submitted to support her claim for reimbursement of meal expenses. Respondent filed a claim for illegal
dismissal against petitioner.

Respondent averred that the dismissal was too harsh a penalty. Petitioner claims that in terminating the services
of an employee for breach of trust, it is enough that the misconduct of the employee tends to prejudice the
employers interest since it would be unreasonable to require the employer to wait until he is materially injured
before removing the cause of the impending evil.

LA Dismissed the complaint. NLRC affirmed LA. CA reversed NLRC

ISSUE: Was respondent validly dismissed for breach of trust and confidence?

RULING: NO. At the outset, it is important to note that the term trust and confidence is restricted to managerial
employees. In Nokom v. National Labor Relations Commission, this Court set the guidelines for the application
of the doctrine of loss of confidence:
(a) Loss of confidence should not be simulated;
(b) It should not be used as a subterfuge for causes which are improper, illegal or unjustified;
(c) It may not be arbitrarily asserted in the face of overwhelming evidence to the contrary; and
(d) It must be genuine, not a mere afterthought to justify earlier action taken in bad faith.

In the instant case, the basis for terminating the employment of respondent was for gross violation of the
companys rules and regulations. Evidently, no mention was made regarding petitioners alleged loss of trust and
confidence in respondent. Neither was there any explanation nor discussion of the alleged sensitive and delicate
position of respondent requiring the utmost trust of petitioner.

It bears emphasizing that the right of an employer to dismiss its employees on the ground of loss of trust and
confidence must not be exercised arbitrarily. For loss of trust and confidence to be a valid ground for dismissal,
it must be substantial and founded on clearly established facts. Loss of confidence must not be used as a
subterfuge for causes which are improper, illegal or unjustified; it must be genuine, not a mere afterthought, to
justify earlier action taken in bad faith. Because of its subjective nature, this Court has been very scrutinizing in
cases of dismissal based on loss of trust and confidence because the same can easily be concocted by an
abusive employer. Thus, when the breach of trust or loss of confidence theorized upon is not borne by clearly
established facts, as in the instant case, such dismissal on the ground of loss and confidence cannot be
countenanced.

San Beda College of Law 595


4S: 2015 - 2016
LABOR LAW REVIEW Atty. Joyrich Golangco

MA. LOURDES T. DOMINGO vs. ROGELIO I. RAYALA;
ROGELIO I. RAYALA vs OFFICE OF THE PRESIDENT;
The REPUBLIC OF THE PHILIPPINES vs ROGELIO I. RAYALA
G.R. No. 155831; G.R. No. 155840; G.R. No. 158700 February 18, 2008
NACHURA, J.:

TERMINATION BY EMPLOYER (Presidential Appointee): Disgraceful and Immoral Conduct

DOCTRINE: The dismissal of a Presidential Appointee such as the NLRC Chairman to be valid must be for a
cause as provided by law. The imposable penalty for the first offense of either the administrative offense of
sexual harassment or for disgraceful and immoral conduct is suspension of six (6) months and one (1) day to
one (1) year, and not dismissal.

FACTS: Ma. Lourdes T. Domingo (Domingo), then Stenographic Reporter III at the NLRC, filed a Complaint for
sexual harassment against Rayala before Secretary Bienvenido Laguesma of the DOLE. Upon receipt of the
Complaint, the DOLE Secretary referred the Complaint to the OP, Rayala being a presidential appointee. It
found Rayala guilty of the offense charged and imposed the penalty of dismissal.

On petition for certiorari, the CA held that there was sufficient evidence on record to create moral certainty that
Rayala committed the acts he was charged with. It also held that Rayalas dismissal was proper. The CA
pointed out that Rayala was dismissed for disgraceful and immoral conduct in violation of RA 6713, the Code of
Conduct and Ethical Standards for Public Officials and Employees. It held that the OP was correct in concluding
that Rayalas acts violated RA 6713. But after filing of Motion for Reconsideration, the CA deleted the penalty of
dismissal and only imposed a suspension.

Petitioner argues that there is no legal basis for the CAs reduction of the penalty imposed by the OP. Rayalas
dismissal is valid and warranted under the circumstances. The power to remove the NLRC Chairman solely
rests upon the President, limited only by the requirements under the law and the due process clause.

Respondent, on the other hand, alleges that under the pertinent Civil Service Rules, disgraceful and immoral
conduct is punishable by suspension for a period of six (6) months and one (1) day to one (1) year. He also
argues that since he is charged administratively, aggravating or mitigating circumstances cannot be appreciated
for purposes of imposing the penalty.

ISSUE: Whether the disimissal is the correct penalty to be imposed?

RULING: NO. It is noteworthy that under AO 250, sexual harassment amounts to disgraceful and immoral
conduct. Thus, any finding of liability for sexual harassment may also be the basis of culpability for disgraceful
and immoral conduct.

It is the President of the Philippines, as the proper disciplining authority, who would determine whether there is a
valid cause for the removal of Rayala as NLRC Chairman. This power, however, is qualified by the phrase "for
cause as provided by law." Thus, when the President found that Rayala was indeed guilty of disgraceful and
immoral conduct, the Chief Executive did not have unfettered discretion to impose a penalty other than the
penalty provided by law for such offense. The imposable penalty for the first offense of either the administrative
offense of sexual harassment or for disgraceful and immoral conduct is suspension of six (6) months and one
(1) day to one (1) year. Accordingly, it was error for the Office of the President to impose upon Rayala the
penalty of dismissal from the service, a penalty which can only be imposed upon commission of a second
offense.

Even if the OP properly considered the fact that Rayala took advantage of his high government position, it still
could not validly dismiss him from the service. Under the Revised Uniform Rules on Administrative Cases in the
Civil Service, taking undue advantage of a subordinate may be considered as an aggravating circumstance and
where only aggravating and no mitigating circumstances are present, the maximum penalty shall be imposed.
Hence, the maximum penalty that can be imposed on Rayala is suspension for one (1) year.

San Beda College of Law 596


4S: 2015 - 2016
LABOR LAW REVIEW Atty. Joyrich Golangco

PHILIPPINE AEOLUS AUTOMOTIVE UNITED CORPORATION and/or FRANCIS CHUA vs. NATIONAL
LABOR RELATIONS COMMISSION and ROSALINDA C. CORTEZ
G.R. No. 124617 April 28, 2000
BELLOSILLO, J.:

TERMINATION BY EMPLOYER: Serious Misconduct

DOCTRINE: For misconduct or improper behavior to be a just cause for dismissal


(a) it must be serious;
(b) must relate to the performance of the employees duties; and,
(c) must show that the employee has become unfit to continue working for the employer.

In order to consider the throwing of a stapler and uttering abusive language upon the manager a serious
misconduct that would justify dismissal under the law, it must have been done in relation to the performance of
her duties as would show her to be unfit to continue working for her employer.

FACTS: A memorandum was issued to private respondent Rosalinda C. Cortez requiring her to explain why no
disciplinary action should be taken against her (a) for throwing a stapler at Plant Manager William Chua, her
superior, and uttering invectives against him; (b) for losing the amount of P1,488.00 entrusted to her by Plant
Manager Chua; and, (c) for asking a co-employee to punch-in her time card. The memorandum however was
refused by private respondent although it was read to her and discussed with her by a co-employee. Another
memorandum was issued by petitioner corporation giving her seventy-two (72) hours to explain why no
disciplinary action should be taken against her for allegedly failing to process the ATM applications of her nine
(9) co-employees with the Allied Banking Corporation. Private respondent also refused to receive the second
memorandum although it was read to her by a co-employee.

A third memorandum was issued to private respondent, this time informing her of her termination from the
service effective on grounds of gross and habitual neglect of duties, serious misconduct and fraud or willful
breach of trust. Private respondent filed with the Labor Arbiter a complaint for illegal dismissal.

LA found the dismissal valid. NLRC reversed LA.

ISSUE: Whether the dismissal was valid?

RULING: NO. For misconduct or improper behavior to be a just cause for dismissal (a) it must be serious; (b)
must relate to the performance of the employees duties; and, (c) must show that the employee has become unfit
to continue working for the employer. The act of private respondent in throwing a stapler and uttering abusive
language upon the person of the plant manager may be considered, from a lay man's perspective, as a serious
misconduct. However, in order to consider it a serious misconduct that would justify dismissal under the law, it
must have been done in relation to the performance of her duties as would show her to be unfit to continue
working for her employer. The acts complained of, under the circumstances they were done, did not in any way
pertain to her duties as a nurse.

As regards the third alleged infraction, i.e., the act of private respondent in asking a co-employee to punch-in
her time card, although a violation of company rules, likewise does not constitute serious misconduct. Firstly, it
was done by her in good faith considering that she was asked by an officer to perform a task outside the office,
which was for the benefit of the company, with the consent of the plant manager. Secondly, it was her first time
to commit such infraction during her five (5)-year service in the company. Finally, the company did not lose
anything by reason thereof as the offense was immediately known and corrected.

Gross negligence implies a want or absence of or failure to exercise slight care or diligence, or the entire
absence of care. It evinces a thoughtless disregard of consequences without exerting any effort to avoid them.
The negligence, to warrant removal from service, should not merely be gross but also habitual. Likewise, the
ground "willful breach by the employee of the trust reposed in him by his employer" must be founded on facts
established by the employer who must clearly and convincingly prove by substantial evidence the facts and
incidents upon which loss of confidence in the employee may fairly be made to rest. All these requirements
prescribed by law and jurisprudence are wanting in the case at bar.

San Beda College of Law 597


4S: 2015 - 2016
LABOR LAW REVIEW Atty. Joyrich Golangco

PHARMACIA and UPJOHN, INC. v. RICARDO P. ALBAYDA, JR.
G.R. No. 172724 August 23, 2010
PERALTA, J.:

TERMINATION BY EMPLOYER: Willful Disobedience (Transfer)

DOCTRINE: Court has long stated that the objection to the transfer being grounded solely upon the personal
inconvenience or hardship that will be caused to the employee by reason of the transfer is not a valid reason to
disobey an order of transfer. Such being the case, respondent cannot adamantly refuse to abide by the order of
transfer without exposing himself to the risk of being dismissed. Hence, his dismissal was for just cause in
accordance with Article 282(a) of the Labor Code.

FACTS: Ricardo P. Albayda, Jr. (respondent) was an employee of Upjohn, Inc. (Upjohn) in 1978 and continued
working there until 1996 when a merger between Pharmacia and Upjohn was created. After the merger,
respondent was designated by petitioner Pharmacia and Upjohn (Pharmacia) as District Sales Manager
assigned to District XI in the Western Visayas area. During the period of his assignment, respondent settled in
Bacolod City.

However, in December 1999, respondent received a Memorandum announcing the sales force structure for the
year 2000. In the said memorandum, respondent was reassigned as District Sales Manager to District XII in the
Northern Mindanao area. One of the key areas covered in District XII is Cagayan de Oro City.

In response to the memorandum, respondent wrote a letter to Aleda Chu. Respondent said that he has always
been assigned to the Western Visayas area and that he felt that he could not improve the sales of products if he
was assigned to an unfamiliar territory. Respondent concluded that his transfer might be a way for his managers
to dismiss him from employment. Respondent added that he could not possibly accept his new assignment in
Cagayan de Oro City because he will be dislocated from his family; his wife runs an established business in
Bacolod City; his eleven- year-old daughter is studying in Bacolod City; and his two-year-old son is under his
and his wifes direct care.

Chu explained to respondent that they are moving him to Cagayan de Oro City, because of their need of
respondents expertise to build the business there. Chu added that the district performed dismally in 1999 and,
therefore, they were confident that under respondents leadership, he can implement new ways and develop the
sales force to become better and more productive. Moreover, since respondent has been already in Bacolod
and Iloilo for 22 years, Chu said that exposure to a different market environment and new challenges will
contribute to respondents development as a manager. Finally, Chu stressed that the decision to transfer
respondent was purely a business decision.

Thereafter, Chu reassigned respondent to Manila, and he was directed to report to work to Manila within 5 days,
but this was unheeded. Because of this, petitioner dismissed him from work through a letter due to AWOL and
Insubordination.

Respondent filed a case for constructive dismissal with the Labor arbiter.

ISSUE: Is the dismissal valid?

HELD: Yes. Jurisprudence recognizes the exercise of management prerogative to transfer or assign employees
from one office or area of operation to another, provided there is no demotion in rank or diminution of salary,
benefits, and other privileges, and the action is not motivated by discrimination, made in bad faith, or effected as
a form of punishment or demotion without sufficient cause. In this case, the transfer from one district to another
is duly explained.

Moreover, the allegation of complainant that respondents familys income will be affected because his wife who
is doing business in Bacolod City and earns P50,000.00, if true, should not be taken in consideration of his
transfer. What is contemplated here is the diminution of the salary of the complainant but not his wife. Besides,
even if complainant may accept his new assignment in Cagayan de Oro or in Metro Manila, his wife may still
continue to do her business in Bacolod City. Anyway, Bacolod City and Manila is just one (1) hour travel by
plane.

San Beda College of Law 598


4S: 2015 - 2016
LABOR LAW REVIEW Atty. Joyrich Golangco

Also, in respondents contract of employment, he agreed to be assigned to any work or workplace as may be
determined by the company whenever the operations require such assignment. As such, he is bound thereby. It
is also in the nature of sales force to be transferred whenever deemed fit by the company.

Court has long stated that the objection to the transfer being grounded solely upon the personal inconvenience
or hardship that will be caused to the employee by reason of the transfer is not a valid reason to disobey an
order of transfer. Such being the case, respondent cannot adamantly refuse to abide by the order of transfer
without exposing himself to the risk of being dismissed. Hence, his dismissal was for just cause in accordance
with Article 282(a) of the Labor Code.

ON THE ISSUE OF SEPARATION PAY:

In the instant case, this Court rules that an award to respondent of separation pay by way of financial
assistance, equivalent to one-half (1/2) months pay for every year of service, is equitable. Although respondent's
actions constituted a valid ground to terminate his services, the same is to this Court's mind not so
reprehensible as to warrant complete disregard of his long years of service (22 years). It also appears that the
same is respondent's first offense. While it may be expected that petitioners will argue that respondent has only
been in their service for four years since the merger of Pharmacia and Upjohn took place in 1996, equity
considerations dictate that respondent's tenure be computed from 1978, the year when respondent started
working for Upjohn.

San Beda College of Law 599


4S: 2015 - 2016
LABOR LAW REVIEW Atty. Joyrich Golangco

JAMES BEN L. JERUSALEM vs. HOCK, SUNNY YAP and JOSEFINA PICART
G.R. No. 169564 April 6, 2011
DEL CASTILLO, J.:

TERMINATION BY EMPLOYER: Loss of Trust and Confidence

DOCTRINE: For breach of trust and confidence to become a valid ground for the dismissal of an employee, the
cause of loss of trust and confidence must be related to the performance of the employees duties.

FACTS: James Ben L. Jerusalem (James) was employed by Keppel Monte Bank (Keppel) as Assistant Vice-
President as was assigned as the head of the VISA Credit card department. Later, he was reassigned as the
head of the Marketing and Operations of the Jewelry Department.

During his leadership in the new assignment, James received from Jorge Javier (Jorge) a sealed envelope said
to be containing VISA Card application forms. Jorge is a Keppel Visa Card Holder since December 1998. James
immediately handed over the envelope with accomplished application forms to the VISA Credit Card Unit. All in
all, the VISA credit card applications referred by Jorge which James forwarded to the VISA Credit Card Unit
numbered 67, all of which were subsequently approved. As it turned out, all the accounts under these approved
applications became past due.

Due to this, the bank suffered a loss of P7,961,619.82. James upon knowing the status of the accounts referred
by Jorge, sent a Memorandum6 to Roberto (head of the VISA Credit Card Department) recommending the filing
of a criminal case for estafa against Jorge. He further recommended that coordination with the other banks
where Jorge has deposits should be made promptly so that they can ask said banks to freeze Jorges accounts.
James even warned Keppel that immediate action should be taken while Jorge is still in the country.

However thereafter, James received a Notice to Explain from Keppels Vice President for Operations, Sunny
Yap (Sunny), why no disciplinary action should be taken against him for referring/endorsing fictitious VISA card
applicants.

James submitted his written explanation to Sunny. He pointed out that he had no participation in the processing
of the VISA card applications since he was no longer connected with the VISA Credit Card Unit at the time of
such transactions. He explained that he can only endorse the applications referred by Jorge to the VISA Credit
Card Unit because he was already transferred to Jewelry Department, as Head.

Despite such explanation, the Manager for Human Resources Department, Josefina Picart, handed to James a
Notice of Termination informing the latter that he was found guilty of breach of trust and confidence for
knowingly and maliciously referring, endorsing and vouching for VISA card applicants who later turned out to be
impostors resulting in financial loss to Keppel.

This prompted James to file before the Labor Arbiter a complaint for illegal dismissal.

ISSUE: Is the dismissal of James on the ground of loss of trust and confidence valid?

HELD: NO. "Loss of confidence as a just cause for termination of employment is premised on the fact that the
employee concerned holds a position of responsibility or trust and confidence. He must be invested with
confidence on delicate matters, such as custody handling or care and protection of the property and assets of
the employer. And, in order to constitute a just cause for dismissal, the act complained of must be work-related
and shows that the employee concerned is unfit to continue to work for the employer."

The first requisite for dismissal on the ground of loss of trust and confidence is that the employee concerned
must be holding a position of trust and confidence. In this case, there is no doubt that James held a position of
trust and confidence as Assistant Vice-President of the Jewelry Department.

The second requisite is that there must be an act that would justify the loss of trust and confidence. Loss of trust
and confidence, to be a valid cause for dismissal, must be based on a wilful breach of trust and founded on
clearly established facts. The basis for the dismissal must be clearly and convincingly established but proof
beyond reasonable doubt is not necessary. Keppels evidence against James fails to meet this standard.

San Beda College of Law 600


4S: 2015 - 2016
LABOR LAW REVIEW Atty. Joyrich Golangco

The bank has not been able to show any concrete proof that indeed complainant had participated in the
approval of the questioned VISA CARD accounts. The records [are] bereft of any concrete showing that
complainant directed Ms. Gerena to approve the applications without passing through the process. The alleged
marginal notations in the applications were admittedly scribbled by Ms. Gerena. Even assuming that there are
such notations on the applications i.e., "c/o James Jerusalem", still, such notations to us can not be construed
as a directive coming from complainant to specifically do away with existing policy on the approval of
applications for VISA Card.

Of course, we concede to the fact that respondent had sustained losses on account of the so-called "credit card
scam" in the amount of P7,961,619.82 all coming from the accounts referred x x x by Mr. Jorge Javier, but no
amount of mind boggling can we infer that the mere act of handing the already accomplished forms for VISA
CREDIT Card could be interpreted as "Favorable endorsement" with instructions not to conduct the usual credit
investigation/verification of applicants. To lay the blame upon the complainant would be at the height of
injustice considering that at that time, he no longer has the authority to pass upon such applications. To
attribute such huge financial losses to one who is no longer connected with the VISA Card department
would be stretching too far, the import of the term "some basis." We simply could not see our way through
how respondent bank could have inferred that complainant made such instruction upon Ms. Gerena to forego
the usual process and have the applications approved without any direct evidence showing to be so.

San Beda College of Law 601


4S: 2015 - 2016
LABOR LAW REVIEW Atty. Joyrich Golangco

RENO FOODS, INC., VICENTE KHU v. Nagkakaisang Lakas ng Manggagawa (NLM) - KATIPUNAN on
behalf of its member, NENITA CAPOR
G.R. No. 164016 March 15, 2010
DEL CASTILLO, J.:

TERMINATION BY EMPLOYER: Serious Misconduct

DOCTRINE:
There is no legal or equitable justification for awarding financial assistance to an employee who was
dismissed for stealing company property. Social justice and equity are not magical formulas to erase the
unjust acts committed by the employee against his employer. While compassion for the poor is
desirable, it is not meant to coddle those who are unworthy of such consideration.

A criminal conviction is not necessary to find just cause for employment termination.

FACTS:
It is a standard operating procedure of petitioner-company to subject all its employees to reasonable search of
their belongings upon leaving the company premises. On October 19, 1998, the guard on duty found six Reno
canned goods wrapped in nylon leggings inside Capors fabric clutch bag. The only other contents of the bag
were money bills and a small plastic medicine container.

After giving Capor several opportunities to explain her side, she was dismissed from work due to serious
misconduct.

Petitioners herein filed a case of qualified theft against her although later dismissed because of reasonable
doubt.

On the other hand, NLM-Katipunan assisted Capor to file an illegal dismissal case against the company but the
labor arbiter and the NLRC find substantial evidence to support the alleged company theft.

Moreover, while the labor arbiter did not give financial assistance to Capor, NLRC awards such financial
assistance.

ISSUES:
1. Is the acquittal of Capor in the criminal case amounts to reversal of the findings with the labor arbiter
and NLRC?
2. Is an employee who was validly dismissed for theft of company property entitled to financial assistance?

HELD:
BOTH NO.

1. A criminal conviction is not necessary to find just cause for employment termination. Otherwise stated,
an employees acquittal in a criminal case, especially one that is grounded on the existence of
reasonable doubt, will not preclude a determination in a labor case that he is guilty of acts inimical to the
employers interests. Criminal cases require proof beyond reasonable doubt while labor disputes require
only substantial evidence, which means such relevant evidence as a reasonable mind might accept as
adequate to justify a conclusion.

The evidence in this case was reviewed by the appellate court and two labor tribunals endowed with expertise
on the matter the Labor Arbiter and the NLRC. They all found substantial evidence to conclude that Capor had
been validly dismissed for dishonesty or serious misconduct. It is settled that factual findings of quasi-judicial
agencies are generally accorded respect and finality so long as these are supported by substantial evidence. In
the instant case, we find no compelling reason to doubt the common findings of the three reviewing bodies.

2. Separation pay is only warranted when the cause for termination is not attributable to the employees
fault, such as those provided in Articles 283 and 284 of the Labor Code, as well as in cases of illegal
dismissal in which reinstatement is no longer feasible. It is not allowed when an employee is dismissed
for just cause, such as serious misconduct.

San Beda College of Law 602


4S: 2015 - 2016
LABOR LAW REVIEW Atty. Joyrich Golangco

Length of service and a previously clean employment record cannot simply erase the gravity of the betrayal
exhibited by a malfeasant employee. Length of service is not a bargaining chip that can simply be stacked
against the employer. After all, an employer-employee relationship is symbiotic where both parties benefit from
mutual loyalty and dedicated service. If an employer had treated his employee well, has accorded him fairness
and adequate compensation as determined by law, it is only fair to expect a long-time employee to return such
fairness with at least some respect and honesty. Thus, it may be said that betrayal by a long-time employee is
more insulting and odious for a fair employer.

While we sympathize with Capors plight, being of retirement age and having served petitioners for 39 years, we
cannot award any financial assistance in her favor because it is not only against the law but also a retrogressive
public policy.

San Beda College of Law 603


4S: 2015 - 2016
LABOR LAW REVIEW Atty. Joyrich Golangco

FE LA ROSA, OFELIA VELEZ, CELY DOMINGO, JONA NATIVIDAD and EDGAR DE LEON vs.
AMBASSADOR HOTEL
G.R. NO. 177059 March 13, 2009
CARPIO MORALES, J.

TERMINATION BY EMPLOYER: Constructive Dismissal/Abandonment

DOCTRINE: Respondent's sudden, arbitrary and unfounded adoption of the two-day work scheme which greatly
reduced petitioners' salaries renders it liable for constructive dismissal.

FACTS: Employees of Ambassador Hotel filed before the NLRC several complaints for illegal dismissal, illegal
suspension, and illegal deductions against the hotel. They alleged that, following their filing of complaints with
the Department of Labor and Employment-NCR which prompted an inspection of the hotel's premises by a labor
inspector, respondent was found to have been violating labor standards laws and was thus ordered to pay them
some money claims. This purportedly angered respondent's management which retaliated by suspending and/or
constructively dismissing them by drastically reducing their work days (two-day work scheme) through the
adoption of a work reduction/rotation scheme.

LA Ruling: found respondent and its manager Yolanda L. Chan guilty of illegal dismissal and ordered them to
pay petitioners' separation pay.

NLRC Ruling: affirmed the labor arbiter's ruling with the modification. The petitioners were directed to report
back to work, and respondent was directed to accept them without having to pay them backwages.

CA Ruling: reversed the NLRC decision and dismissed petitioners' complaints, holding that there was no
constructive dismissal because petitioners "simply disappeared from work" upon learning of the work
reduction/rotation scheme. The appellate court went on to hold that respondent's adoption of the work
reduction/rotation scheme, as well as its reassignment of petitioners, was a valid exercise of management
prerogative, absent any showing that the same was done out of vengeance.

ISSUE: Was there a valid dismissal?

RULING: No.

No proof to justify 2-day work scheme


The records fail, however, to show any documentary proof that the work reduction scheme was adopted due to
respondent's business reverses. Respondent's memorandum informing petitioners of the adoption of a two-day
work scheme made no mention why such scheme was being adopted. Neither do the records show any
documentary proof that respondent suffered financial losses to justify its adoption of the said scheme to stabilize
its operations.

Constructive dismissal
Case law holds that constructive dismissal occurs when there is cessation of work because continued
employment is rendered impossible, unreasonable or unlikely; when there is a demotion in rank or diminution in
pay or both; or when a clear discrimination, insensibility, or disdain by an employer becomes unbearable to the
employee. Respondent's sudden, arbitrary and unfounded adoption of the two-day work scheme which greatly
reduced petitioners' salaries renders it liable for constructive dismissal.

Abandonment was not proved


Absence must be accompanied by overt acts unerringly pointing to the fact that the employee simply does not
want to work anymore. And the burden of proof to show that there was unjustified refusal to go back to
work rests on the employer. For abandonment to exist, two requisites must concur: first, the employee must
have failed to report for work or must have been absent without valid or justifiable reason; and second, there
must have been a clear intention on the part of the employee to sever the employer-employee relationship as
manifested by some overt acts.

Relief
As for the appellate court's ruling that petitioners are not entitled to reinstatement because they did not pray for it
in their complaints, the same does not lie. In all the pro-forma complaints filed by petitioners before the NLRC,
San Beda College of Law 604
4S: 2015 - 2016
LABOR LAW REVIEW Atty. Joyrich Golangco

they prayed for reinstatement or, in the alternative, for the award to them of separation pay.

Besides, under Article 279 of the Labor Code and based on settled jurisprudence, an employee dismissed
without just cause and without due process, like petitioners herein, are entitled to reinstatement and backwages
or payment of separation pay.

San Beda College of Law 605


4S: 2015 - 2016
LABOR LAW REVIEW Atty. Joyrich Golangco

MARIBAGO BLUEWATER BEACH RESORT, INC. vs. NITO DUAL
G.R. No. 180660, July 20, 2010
PEREZ, J.

TERMINATION BY EMPLOYER: Serious Misconduct

DOCTRINE: Theft committed by an employee constitutes serious misconduct which is a valid reason for his
dismissal by the employer.

FACTS: Petitioner hired respondent as waiter and promoted him later as outlet cashier of its Poolbar/Allegro
Restaurant. A group of Japanese guests and their companions dined at Allegro. Pursuant to the order slip,
fourteen (14) sets of dinner were prepared by the chef and were served to the guests. After dinner, at around
9:00 p.m., the guests asked for their bill. Since Hiyas (attending waiter) was attending to other guests, he gave a
signal to Mission (another waiter) to give the bill. Mission asked respondent Dual for the sales transaction
receipt and presented this to the guests. The guests paid the amount indicated on the receipt and thereafter left
in a hurry. The receipt printed at 10:40 p.m. shows that only P3,036.00 was remitted by cashier Dual
corresponding to six (6) sets of dinner. In view of the discrepancy between the order slip and the receipt issued,
petitioner Maribago, through its HRD manager, issued memoranda requiring respondents and the other waiters
to explain why they should not be penalized for violating House Rule 4.1 (dishonesty in any nature). After the
investigation, respondent Dual was found guilty of dishonesty for his fabricated statements and for asking one of
the waiters (Mission) to corroborate his allegations.
th
Dual filed a complaint for unfair labor practice, illegal dismissal, non-payment of 13 month and
separation pay, and damages before the NLRC.

LA Ruling: found that respondents termination was without valid cause and ruled that respondent is entitled to
separation pay.

NLRC Ruling: set aside the Labor Arbiters decision and dismissed the complaint. It ruled that complainants act
of depriving respondent of its lawful revenue is tantamount to fraud against the company which warrants
dismissal from the service.

CA Ruling: reversed the decision and resolution of the NLRC. Finding no sufficient valid cause to justify
respondents dismissal, the Court of Appeals ordered petitioner to pay respondent full backwages and
separation pay.

ISSUE: Was the respondent illegally dismissed?

RULING: No. Petitioners evidence proved that respondent is guilty of dishonesty and of stealing money
entrusted to him as cashier. Instead of reporting P10,100.00 as payment by the guests for their dinner,
respondent cashier only reported P3,036.00 (for 6 sets of meals) as shown by the receipt which he admitted to
have issued. The receipt which bears his name NITO was printed at 22:40 (10:40 p.m.) or 1 hour and 40
minutes after the guests had left at 9:00 p.m. Two other receipts were issued for the same amount at 22:39:55
and 22:40:01.

The allegation of Dual that six (6) dinner sets were indeed cancelled as evidenced by the dishes he
allegedly saw in the utensil station is negated by the testimonies of the kitchen staff that twelve (12) set meals
were served and consumed. These testimonies coincide with the claim of waiters Hiyas and Mission that
fourteen (14) sets of dinner were served. The serving of food eliminates the argument of cancellation. The alibi
of cancellation has no leg to stand on. The standard operating procedure of Maribago dictates that in cases of
cancellation, the order slip has to be countersigned by the attending waiter (which in this case should have been
Chief Waiter Hiyas) but such was not so in this case. The foregoing facts explain why Dual tried twice to
convince Mission to cover up their crime. They even asked Mission to take the fall by asking him to admit that
he altered the order slip from twelve (12) sets of dinner to six (6) sets.

Respondents acts constitute serious misconduct which is a just cause for termination under the law.
Theft committed by an employee is a valid reason for his dismissal by the employer.

San Beda College of Law 606


4S: 2015 - 2016
LABOR LAW REVIEW Atty. Joyrich Golangco

CENTURY CANNING CORPORATION vs. VICENTE RANDY RAMIL
G.R. No. 171630 August 8, 2010
PERALTA, J.

TERMINATION BY EMPLOYER: Loss of Trust and Confidence

DOCTRINE: Loss of trust and confidence, to be a valid cause for dismissal, must be based on a willful breach of
trust and founded on clearly established facts. The basis for the dismissal must be clearly and convincingly
established, but proof beyond reasonable doubt is not necessary.

FACTS: Respondent is a technical specialist in the petitioner company. Respondent prepared a capital
expenditure (CAPEX) form for external fax modems and terminal server, per order of Technical Operations
Manage and endorsed it Secretary of Executive Vice-President Ricardo T. Po, for the latter's signature. With the
form apparently signed by Po, respondent transmitted it to Purchasing Officer Lorena Paz. Paz processed the
paper and found that some details in the CAPEX form were left blank. She also doubted the genuineness of the
signature of Po, as appearing in the form. Paz then transmitted the CAPEX form to Purchasing Manager and
informed her of the questionable signature of Po. Suspecting him to have committed forgery, respondent was
asked to explain in writing the events surrounding the incident. He vehemently denied any participation in the
alleged forgery. Respondent was, thereafter, suspended. Subsequently, he received a Notice of Termination for
loss of trust and confidence. Respondent filed a Complaint for illegal dismissal, among others.

LA Ruling: dismissing the complaint for lack of merit.

NLRC Ruling: The NLRC declared respondent's dismissal to be illegal and directed petitioner to reinstate
respondent with full backwages and seniority rights and privileges. It found that petitioner failed to show clear
and convincing evidence that respondent was responsible for the forgery of the signature of Po in the CAPEX
form. But NLRC reversed itself upon petitioners Motion for Reconsideration.

CA Ruling: rendered judgment in favor of respondent and reinstated the earlier decision of the NLRC

ISSUE: Was the allegation of loss of trust and confidence a valid cause of dismissal in this case?

RULING: No. The law mandates that the burden of proving the validity of the termination of employment rests
with the employer. Failure to discharge this evidentiary burden would necessarily mean that the dismissal was
not justified and, therefore, illegal.

The termination letter of the respondent provides that the company conducted inquiries from persons concerned
to get more information in this forgery. However, this information which petitioner allegedly obtained from the
persons concerned was not backed-up by any affidavit or proof. Petitioner did not even bother to name these
resource persons.

Petitioner based respondent's dismissal on its unsubstantiated suspicions and conclusion that since respondent
was the custodian and the one who prepared the CAPEX forms, he had the motive to commit the forgery.
However, as correctly found by the NLRC in its original Decision, respondent would not be benefited by the
purchase of the subject equipment. The equipment would be for the use of petitioner company.

The right of an employer to dismiss an employee on the ground that it has lost its trust and confidence in him
must not be exercised arbitrarily and without just cause. Loss of trust and confidence, to be a valid cause for
dismissal, must be based on a willful breach of trust and founded on clearly established facts. The basis for the
dismissal must be clearly and convincingly established, but proof beyond reasonable doubt is not necessary.

The Court finds that it would be best to award separation pay instead of reinstatement, in view of the strained
relations between petitioner and respondent.

San Beda College of Law 607


4S: 2015 - 2016
LABOR LAW REVIEW Atty. Joyrich Golangco

TONGKO VS. MANUFACTURERS OF LIFE INSURANCE CO. AND DE DIOS
G.R. No. 167622 November 07, 2008
VELASCO, JR., R.

TERMINATION BY EMPLOYER: Willful Disobedience

DOCTRINE: The law mandates that the burden of proving the validity of the termination of employment rests
with the employer. Failure to discharge this evidentiary burden would necessarily mean that the dismissal was
not justified, and, therefore, illegal. Unsubstantiated suspicions, accusations and conclusions of employers do
not provide for legal justification for dismissing employees.

FACTS: Petitioner, Gregorio Tongko, an agent who started his professional relationship with respondent
Manufacturers of Life Insurance(Manulife) on 1977 entered into an agreement stipulating among others that: 1.
Agent (Tongko) is an independent contractor and no employee-employer relationship between them exist; 2.
That the company may terminate this agreement for any breach or violation of any of the provisions hereof by
the Agent.

Through the years, Tongko was promoted into becoming a Branch Manager. However in 2001, Manulife through
De Dios (Manulifes President and CEO) wrote letters to Tongko informing him of the companys dissatisfaction
of his performance. Manulife subsequently issued letters of instructions and guidelines purportedly given for
Tongko to follow and comply in the hope that both of their directions would align toward the Managements
avowed agency growth policy.

However, after a short span of time, Manulife terminated the services of Tongko alleging that Tongko wilfully
disobeyed its orders. Tongko then filed a case against Manulife for illegal dismissal.

In establishing the ER-EE relationship, Tongko alleged that Manulife thru De Dios gave him specific directives
on how to manage his area of responsibility as evidenced by the letters of instruction given. Tongko further
argued that he was dismissed without basis.

Manulife countered that Tongko is not an employee and it did not exercise control over him. Manulife added that
even if Tongko is considered as its employee, his employment was validly terminated on the ground of gross
and habitual neglect of duties, inefficiency, as well as willful disobedience of the lawful orders of Manulife.
Manulife claimed that Tongko, despite the written reminder from Mr. De Dios, refused to shape up and
altogether disregarded the latter's advice resulting in his laggard performance clearly indicative of his willful
disobedience of the lawful orders of his superior.

LA: The LA dismissed Tongkos action for lack of an employer-employee relationship.


NLRC: The NLRC reversed the LAs decision and held Manulife liable for illegal dismissal.
CA: The CA reversed the NLRCs decision for lack of said relationship.

ISSUES:
1. Was there an employer-employee relationship?
2. If yes, was Manulife guilty of illegal dismissal?

SC RULING:
1. YES. If the specific rules and regulations that are enforced against insurance agents or managers are such
that would directly affect the mean and methods by which such agents or managers would achieve the
objectives set by the insurance company, they are employees of the insurance company.

In the instant case, Manulife had the power of control over Tongko that would make him its employee.

2. YES. It is readily evident from Manulife's petition that it failed to cite a single iota of evidence to support its
claims. Manulife did not even point out which order or rule that Tongko disobeyed. More importantly, Manulife
did not point out the specific acts that Tongko was guilty of that would constitute gross and habitual neglect of
duty or disobedience. Manulife merely cited Tongko's alleged "laggard performance," without substantiating
such claim, and equated the same to disobedience and neglect of duty.

San Beda College of Law 608


4S: 2015 - 2016
LABOR LAW REVIEW Atty. Joyrich Golangco

When there is no showing of a clear, valid and legal cause for the termination of employment, the law considers
the matter a case of illegal dismissal and the burden is on the employer to prove that the termination was for a
valid or authorized cause.

The law mandates that the burden of proving the validity of the termination of employment rests with the
employer. Failure to discharge this evidentiary burden would necessarily mean that the dismissal was not
justified, and, therefore, illegal.

San Beda College of Law 609


4S: 2015 - 2016
LABOR LAW REVIEW Atty. Joyrich Golangco

SCHOOL OF HOLY SPIRIT OF QUEZON CITY VS. TAGUIAM
G.R. No. 1165565. July 14, 2008
QUISUMBING, J.

TERMINATION BY EMPLOYER: Gross and Habitual Neglect;

WHEN THE NEGLECT IS SO SUBSTANTIAL TO CAUSE CONSIDERABLE DAMAGE TO THE EMPLOYER,


IT DOES NOT NEED TO BE HABITUAL

DOCTRINE:
In view of the considerable resultant damage, however, we are in agreement that the cause is sufficient to
dismiss respondent. This is not the first time that we have departed from the requirements laid down by the law
that neglect of duties must be both gross and habitual.

Otherwise stated, the GENERAL RULE is that the neglect must be both gross and habitual. Hence, a single or
isolated act of negligence does not constitute a just cause for the dismissal of the employee. However, as an
EXCEPTION, when the neglect was so substantial to cause considerable damage to the employer, it does not
need to be habitual. (Disclaimer: The first par is found in the case but the second is not but of my own
paraphrasing. So is the tickler/heading)

FACTS:
Respondent, Corazon Taguiam, worked as a school teacher and class adviser for petitioner School of Holy
Spirit (Holy Spirit). As a year-end celebration, the principal allowed the students who sought the latters
permission thru its president, to use the school grounds including the swimming pool.

While Corazon went away to check on the other two students, Chiara Mae, one of her students, drowned and
was declared dead upon arrival at the hospital. It was later found out that Corazon permitted Chiara Mae to join
the activity despite the fact that Chiara Maes permit was not signed by her parents.

Holy Spirit then issued a notice of Administrative Charge to Corazon for alleged gross negligence and required
her to submit a written explanation which she did. Holy Spirit then dismissed Corazon on the ground of gross
negligence resulting to loss of trust and confidence.

Corazon in turn filed a case for illegal dismissal against Holy Spirit.

LA: The LA dismissed the complaint ruling that dismissal was justified because Corazon left the pupils without
any adult supervision considering the absence of adequate facilities. The LA added that although the negligence
was not habitual, the same warranted her dismissal since death resulted therefrom.

NLRC: The NLRC affirmed the LA.

CA: The CA reversed the decision ruling that there was insufficient proof that Corazons negligence was both
gross and habitual.

ISSUE: Was respondents dismissal on the ground of gross negligence resulting to loss of trust and confidence
valid?

SC RULING:
Under Article 282 of the Labor Code, gross and habitual neglect of duties is a valid ground for an employer to
terminate an employee. Gross negligence implies a want or absence of or a failure to exercise slight care or
diligence, or the entire absence of care. It evinces a thoughtless disregard of consequences without exerting
any effort to avoid them. Habitual neglect implies repeated failure to perform ones duties for a period of time,
depending upon the circumstances.

The court ruled that Corazon had been grossly negligent as evidenced by the following facts: First, it is
undisputed that Chiara Maes permit form was unsigned; and Second, it was respondents responsibility as
Class Adviser to supervise her class in all activities sanctioned by the school.

San Beda College of Law 610


4S: 2015 - 2016
LABOR LAW REVIEW Atty. Joyrich Golangco

The court added that, respondents negligence, although gross, was not habitual. In view of the considerable
resultant damage, however, the court ruled that the cause is sufficient to dismiss respondent.

As a result of gross negligence in the present case, petitioners lost its trust and confidence in respondent. Loss
of trust and confidence to be a valid ground for dismissal must be based on a willful breach of trust and founded
on clearly established facts. A breach is willful if it is done intentionally, knowingly and purposely, without
justifiable excuse, as distinguished from an act done carelessly, thoughtlessly, heedlessly or inadvertently.
There must, therefore, be an actual breach of duty committed by the employee which must be established by
substantial evidence.

San Beda College of Law 611


4S: 2015 - 2016
LABOR LAW REVIEW Atty. Joyrich Golangco

YRASUEGUI VS. PAL
G.R. No. 168081 October 17, 2008
REYES, R.T., J.

TERMINATION OF EMPLOYMENT: Analogous Cause (Obesity);

OBESITY AS A GROUND FOR DISMISSAL UNDER ARTICLE 282(E) OF THE LABOR CODE.

DOCTRINE: The failure to meet the employer's qualifying standards is in fact a ground that falls under Article
282(e) - the "other causes analogous to the foregoing."

By its nature, these "qualifying standards" are norms that apply prior to and after an employee is hired. They
apply prior to employment because these are the standards a job applicant must initially meet in order to be
hired. They apply after hiring because an employee must continue to meet these standards while on the job in
order to keep his job.

FACTS: Armando G. Yrasuegui was an international flight steward of Philippine Airlines, Inc. (PAL). He stands
five feet and eight inches (5'8") with a large body frame. The proper weight for a man of his height and body
structure is from 147 to 166 pounds, the ideal weight being 166 pounds, as mandated by the Cabin and Crew
Administration Manual of PAL.

Yraseguis weight problem: Yrasegui encountered a great deal of weight problem. From meeting the
required weight limit of 166 pounds, he ballooned, reaching the weights up to 215 to 217 pounds. PAL was
lenient enough to grant him a number of vacation leaves to address his weight concerns. He was thus required
to report for periodical weight checks. However, instead of losing, he gained more weight. It was when he
reached 217 pounds where he made a commitment in writing to lose 17 pounds in 90 days and continue
reducing from then on. He remained overweight despite the 90-day period given him to reach his ideal weight.
Time and again, Yrasuegui refused to comply with the weight checks. When he finally tipped the scale, he
weighed at 212 pounds. Clearly, he was still way over his ideal weight of 166 pounds. From then on, nothing
was heard from petitioner until he followed up his case requesting for leniency. He continuously gained weight.
On November 13, 1992, PAL finally served petitioner a Notice of Administrative Charge for violation of company
standards on weight requirements. Ultimately, Yrasuegui was formally informed by PAL that due to his inability
to attain his ideal weight, "and considering the utmost leniency" extended to him "which spanned a period
covering a total of almost five years," his services were considered terminated "effective immediately."

LA: The Labor Arbiter ruled in favor of Yrasuegui stating that although reasonable, the weight standards need
not be complied with under pain of dismissal since his weight did not hamper the performance of his duties.

NLRC: The NLRC AFFIRMED the decision of the LA. According to the NLRC, "obesity, or the tendency to
gain weight uncontrollably regardless of the amount of food intake, is a disease in itself. As a consequence,
there can be no intentional defiance or serious misconduct by petitioner to the lawful order of PAL for him to lose
weight.

CA: The CA reversed the NLRC opining that the weight standards of PAL are meant to be a continuing
qualification for an employee's position. The failure to adhere to the weight standards is an analogous cause for
the dismissal of an employee under Article 282(e) of the Labor Code in relation to Article 282(a).

ISSUE: Is obesity a just cause in terminating the employees services?

SC RULING:
YES, the obesity of petitioner is a ground for dismissal under Article 282(e) of the Labor Code. It is a continuing
qualification. Tersely put, an employee may be dismissed the moment he is unable to comply with his ideal
weight as prescribed by the weight standards. The dismissal of the employee would thus fall under Article
282(e) of the Labor Code. The standards violated in this case were not mere "orders" of the employer; they
were the "prescribed weights" that a cabin crew must maintain in order to qualify for and keep his or her
position in the company.

In this sense, the failure to maintain these standards does not fall under Article 282(a) whose express terms
require the element of willfulness in order to be a ground for dismissal. The failure to meet the employer's
San Beda College of Law 612
4S: 2015 - 2016
LABOR LAW REVIEW Atty. Joyrich Golangco

qualifying standards is in fact a ground that falls under Article 282(e) - the "other causes analogous to the
foregoing."

By its nature, these "qualifying standards" are norms that apply prior to and after an employee is hired. They
apply prior to employment because these are the standards a job applicant must initially meet in order to be
hired. They apply after hiring because an employee must continue to meet these standards while on the job in
order to keep his job. Under this perspective, a violation is not one of the faults for which an employee can be
dismissed pursuant to pars. (a) to (d) of Article 282; the employee can be dismissed simply because he no
longer "qualifies" for his job irrespective of whether or not the failure to qualify was willful or intentional.

The court held that the obesity of petitioner, when placed in the context of his work as flight attendant, becomes
an analogous cause under Article 282(e) of the Labor Code that justifies his dismissal from the service. His
obesity may not be unintended, but is nonetheless voluntary. As the CA correctly puts it, voluntariness basically
means that the just cause is solely attributable to the employee without any external force influencing or
controlling his actions. This element runs through all just causes under Article 282, whether they be in the nature
of a wrongful action or omission. Gross and habitual neglect, a recognized just cause, is considered voluntary
although it lacks the element of intent found in Article 282(a), (c), and (d).

San Beda College of Law 613


4S: 2015 - 2016
LABOR LAW REVIEW Atty. Joyrich Golangco

JOHN HANCOCK LIFE INSURANCE CORPORATION and/or MICHAEL PLAXTON v. JOANNA CANTRE
DAVIS
G.R. No. 169549 G.R. No. 169549
CORONA, J.:

TERMINATION BY EMPLOYER: Analogous Causes Theft committed against co-employee

DOCTRINE: Theft committed by an employee against a person other than his employer (such as to his co-
employee), if proven by substantial evidence, is a cause analogous to serious misconduct.

FACTS: Respondent Joanna Cantre Davis was agency administration officer of petitioner John Hancock Life
Insurance Corporation.

Patricia Yuseco, petitioners corporate affairs manager, discovered that her wallet was missing. She immediately
reported the loss of her credit cards to AIG and BPI Express. To her surprise, she was informed that Patricia
Yuseco had just made substantial purchases using her credit cards in various stores in the City of Manila. She
was also told that a proposed transaction in Abensons-Robinsons Place was disapproved because she gave the
wrong information upon verification.

Because loss of personal property among its employees had become rampant in its office, petitioner sought the
assistance of the NBI. The NBI, in the course of its investigation, obtained a security video from Abensons
showing the person who used Yusecos credit cards. Yuseco and other witnesses positively identified the person
in the video as respondent. Consequently, the NBI and Yuseco filed a complaint for qualified theft against
respondent in the office of the Manila city prosecutor. But because the affidavits presented by the NBI
(identifying respondent as the culprit) were not properly verified, the city prosecutor dismissed the complaint due
to insufficiency of evidence.

Meanwhile, petitioner placed respondent under preventive suspension and instructed her to cooperate with its
ongoing investigation. Instead of doing so, however, respondent filed a complaint for illegal dismissal alleging
that petitioner terminated her employment without cause.

LAs Ruling: Respondent committed serious misconduct (she was the principal suspect for qualified theft
committed inside petitioners office during work hours). There was a valid cause for her dismissal. Thus, the
complaint was dismissed for lack of merit.

NLRCs Ruling: NLRC affirmed the decision of the LA.

CAs Ruling: CA granted the petition of Respondent Davis.

Petitioners Argument: The ground for an employees dismissal need only be proven by substantial evidence.
Thus, the dropping of charges against an employee (especially on a technicality such as lack of proper
verification) or his subsequent acquittal does not preclude an employer from dismissing him due to serious
misconduct.

ISSUE: Is respondent validly terminated?

HELD: YES, respondent is validly terminated.

Misconduct involves the transgression of some established and definite rule of action, forbidden act, a
dereliction of duty, willful in character, and implies wrongful intent and not mere error in judgment. For
misconduct to be serious and therefore a valid ground for dismissal, it must be:
1. Of grave and aggravated character and not merely trivial or unimportant; and
2. Connected with the work of the employee.

In this case, petitioner dismissed respondent based on the NBIs finding that the latter stole and used Yusecos
credit cards. But since the theft was not committed against petitioner itself but against one of its employees,
respondents misconduct was not work-related and therefore, she could not be dismissed for serious
misconduct.

San Beda College of Law 614


4S: 2015 - 2016
LABOR LAW REVIEW Atty. Joyrich Golangco

Nonetheless, Article 282(e) of the Labor Code talks of other analogous causes or those which are susceptible of
comparison to another in general or in specific detail. For an employee to be validly dismissed for a cause
analogous to those enumerated in Article 282, the cause must involve a voluntary and/or willful act or omission
of the employee.

A cause analogous to serious misconduct is a voluntary and/or willful act or omission attesting to an employees
moral depravity. Theft committed by an employee against a person other than his employer, if proven by
substantial evidence, is a cause analogous to serious misconduct.

San Beda College of Law 615


4S: 2015 - 2016
LABOR LAW REVIEW Atty. Joyrich Golangco

HOCHENG PHILIPPINES CORPORATION v. ANTONIO M. FARRALES
G.R. No. 211497 March 18, 2015
REYES, J.:

TERMINATION BY EMPLOYER: Reasonableness of Termination

DOCTRINE: The Court held that to be lawful, the cause for termination must be a serious and grave
malfeasance to justify the deprivation of a means of livelihood. The penalty imposed on the erring employee
ought to be proportionate to the offense, taking into account its nature and surrounding circumstances.

FACTS: Farrales was first employed by HPC on May 12, 1998 as Production Operator, followed by promotions
as (1) Leadman in 2004, (2) Acting Assistant Unit Chief in 2007, and (3) Assistant Unit Chief of Production in
2008, a supervisory position. He was a consistent recipient of citations for outstanding performance, as well as
appraisal and year-end bonuses.7chanroblesvirtuallawlibrary

On December 2, 2009, a report reached HPC management that a motorcycle helmet of an employee, Reymar
Solas, was stolen at the parking lot within its premises. Security Officer Francisco Paragas III confirmed a video
sequence recorded on CCTV showing Farrales taking the missing helmet from a parked motorcycle. Later that
day, HPC sent Farrales a notice to explain his involvement in the alleged theft. Farrales sent an explanation.

After a hearing was conducted, the HPC issued a Notice of Termination to Farrales dismissing him for violation
of Article 69, Class A, Item No. 29 of the HPC Code of Discipline, which provides that stealing from the
company, its employees and officials, or from its contractors, visitors or clients, is akin to serious misconduct
and fraud or willful breach by the employee of the trust reposed in him by his employer or duly authorized
representative, which are just causes for termination of employment under Article 282 of the Labor Code.

Farrales filed a complaint for illegal dismissal.

LAs Ruling: HPC is guilty of illegal dismissal.

NLRCs Ruling: NLRC reversed the decision of LA finding the dismissal lega.

CAs Ruling: HPC is guilty of illegal dismissal. Farrales act of taking Reymars helmet did not amount to theft,
holding that HPC failed to prove that Farrales conduct was induced by a perverse and wrongful intent to gain, in
light of the admission of Eric that he did let Farrales borrow one of his two helmets, only that Farrales mistook
Reymars helmet as the one belonging to him.

ISSUE: Is the dismissal legal?

HELD: NO, the dismissal is illegal.

The Court held that [t]o be lawful, the cause for termination must be a serious and grave malfeasance to justify
the deprivation of a means of livelihood. This is merely in keeping with the spirit of our Constitution and laws
which lean over backwards in favor of the working class, and mandate that every doubt must be resolved in their
favor. Moreover, the penalty imposed on the erring employee ought to be proportionate to the offense, taking
into account its nature and surrounding circumstances.

The Court agrees with the CA that Farrales committed no serious or willful misconduct or disobedience to
warrant his dismissal. It is not disputed that Farrales lost no time in returning the helmet to Reymar the moment
he was apprised of his mistake by Eric, which proves, according to the CA, that he was not possessed of a
depravity of conduct as would justify HPCs claimed loss of trust in him. Farrales immediately admitted his error
to the company guard and sought help to find the owner of the yellow helmet, and this, the appellate court said,
only shows that Farrales did indeed mistakenly think that the helmet he took belonged to Eric.

Theft committed by an employee against a person other than his employer, if proven by substantial evidence, is
a cause analogous to serious misconduct. Misconduct is improper or wrong conduct, it is the transgression of
some established and definite rule of action, a forbidden act, a dereliction of duty, willful in character, and
implies wrongful intent and not mere error in judgment. The misconduct to be serious must be of such grave and

San Beda College of Law 616


4S: 2015 - 2016
LABOR LAW REVIEW Atty. Joyrich Golangco

aggravated character and not merely trivial or unimportant. Such misconduct, however serious, must,
nevertheless, be in connection with the employees work to constitute just cause for his separation.

But where there is no showing of a clear, valid and legal cause for termination of employment, the law considers
the case a matter of illegal dismissal. If doubts exist between the evidence presented by the employer and that
of the employee, the scales of justice must be tilted in favor of the latter. The employer must affirmatively show
rationally adequate evidence that the dismissal was for a justifiable cause.

San Beda College of Law 617


4S: 2015 - 2016
LABOR LAW REVIEW Atty. Joyrich Golangco

ROQUE B. BENITEZ and SANTA FE LABOR UNION-FEDERATION OF FREE WORKERS vs. SANTA FE
MOVING AND RELOCATION SERVICESNEDIT KURANGIL
G.R. No. 208163 April 20, 2015
BRION, J.:

TERMINATION BY EMPLOYER: Serious Misconduct Insulting and Offensive Language

DOCTRINE: Display of insolent and disrespectful behavior, in utter disregard of the time and place of its
occurrence constituted a serious misconduct as defined by law.

FACTS: Petitioners filed a complaint for ULP and illegal dismissal, with money claims against respondents. The
company is engaged in providing relocation and moving services, including visa, immigration and real estate
services. Benitez (the unions Vice-President at the time), was its former packing and moving operator (crew
leader) since June 2001.

Benitez alleged that the company served him a memorandum advising him not to report for work effective
immediately, thereby terminating his employment, supposedly on grounds of serious misconduct or willful
disobedience. He allegedly uttered abusive words against Kurangil during the companys Christmas Party. He
bewailed that he was not given the opportunity to defend himself. Benitez denies the accusation against him and
argued that his dismissal constituted ULP as he was a union officer and that it was undertaken to derail the
conclusion of a collective bargaining agreement with the company. He further argued that the penalty of
dismissal is disproportionate to his alleged offense, considering that it was committed during a casual gathering
and had no connection to his work.

The company required Benitez to explain in writing why he should not be disciplined for serious misconduct and
willful disobedience of its lawful orders in connection with the incident. Benitez failed to comply and neither did
he show remorse for what he did. In view of Benitezs failure to explain his side, the company issued a
memorandum to Benitez, terminating his employment effective on the same day, for clear violation of "Santa Fe
Policy and Procedure under Conduct and Behavior as well as Labor Code of the Philippines under Art. 282
Serious misconduct or willful disobedience by the employee of the lawful orders of his employer x x x."

LAs Decision: Benitez is legally dismissed. Benitez, who was holding a position of trust and confidence as
packing and moving operator, committed a serious misconduct at the companys Christmas Party by "hurling
obscene, insulting or offensive language against a superior," thereby losing the trust and confidence of his
employer.

NLRCs Decision: Benitez is legally dismissed.

CAs Decision: Benitez is legally dismissed.

Petitioners Argument: Even if Benitez committed the offense for which he was charged, it was not a serious
misconduct that would warrant his dismissal under the law.

ISSUE: Is Benitez legally dismissed?

HELD: YES, Benitez is legally dismissed.

The present case is distinguished with the case of Samson v. NLRC, where the SC held that the employee was
illegally dismissed.

In the case of Samson v. NLRC, the Court opined:


The instant case [Samson v. NLRC] should be distinguished from the previous cases where we
held that the use of insulting and offensive language constituted gross misconduct justifying an
employees dismissal. In De la Cruz vs. NLRC, the dismissed employee shouted "sayang ang
pagka-professional mo!" and "putang ina mo" at the company physician when the latter refused to
give him a referral slip. In Autobus Workers Union (AWU) v. NLRC, the dismissed employee
called his supervisor "gago ka" and taunted the latter by saying "bakit anong gusto mo tang ina
mo." In these cases, the dismissed employees personally subjected their respective superiors to
the foregoing verbal abuses. The utter lack of respect for their superiors was patent. In contrast,
San Beda College of Law 618
4S: 2015 - 2016
LABOR LAW REVIEW Atty. Joyrich Golangco

when petitioner was heard to have uttered the alleged offensive words against respondent
companys president and general manager, the latter was not around.

Further, it appears that in Samson, the company was ambivalent for a while on what to do with Samsons
offense as it took several weeks after the last incident on January 3, 1994 before it asked him to explain.
Moreover, the company official maligned merely admonished Samson during a meeting on January 4, 1994.

In contrast, the company acted swiftly, and decisively in Benitez's case, obviously and understandably, because
of the gravity and high visibility of his offense, which not only constituted a frontal verbal, and nearly physical
(the attempted beer bottle throwing), assault against Kurangil. Needless to say, Benitez's outburst also caused
grave embarrassment for the audience who witnessed the incident, including company officials whom he
likewise maligned, as well as company clients and guests.

Under the foregoing circumstances, we are convinced - as the Labor Arbiter, the NLRC and the CA had been -
that Benitez's offense constituted a serious misconduct as defined by law. His display of insolent and
disrespectful behavior, in utter disregard of the time and place of its occurrence, had very much to do with his
work. He set a bad example as a union officer and as a crew leader of a vital division of the company. His
actuations during the company's Christmas Party, to our mind, could have had negative repercussions for his
employer had he been allowed to stay on the job. His standing before those clients who witnessed the incident
and those who would hear of it would surely be diminished, to the detriment of the company.

San Beda College of Law 619


4S: 2015 - 2016
LABOR LAW REVIEW Atty. Joyrich Golangco

ST. LUKES MEDICAL CENTER, INC. v. MARIA THERESA V. SANCHEZ
G.R. No. 212054 March 11, 2015
PERLAS-BERNABE, J.

TERMINATION BY EMPLOYER: Willful Disobedience of Lawful Orders

DOCTRINE: Doubtless, the deliberate disregard or disobedience of rules by the employee cannot be
countenanced as it may encourage him or her to do even worse and will render a mockery of the rules of
discipline that employees are required to observe.

FACTS: Sanchez was hired by petitioner St. Lukes Medical Center, Inc. (SLMC) as a Staff Nurse, and was
eventually assigned at SLMC, Quezon Citys Pediatric Unit until her termination for her purported violation of
SLMCs Code of Discipline, particularly Section 1, Rule 1 on Acts of Dishonesty, i.e., Robbery, Theft, Pilferage,
and Misappropriation of Funds.

Records reveal that at the end of her shift on May 29, 2011, Sanchez passed through the SLMC Centralization
Entrance/Exit where she was subjected to the standard inspection procedure by the security personnel. In the
course thereof, the Security Guard on-duty, SG Manzanade, noticed a pouch in her bag and asked her to open
the same. When opened, said pouch contained the following assortment of medical stocks which were
subsequently confiscated. Sanchez asked SG Manzanade if she could just return the pouch inside the treatment
room; however, she was not allowed to do so. Instead, she was brought to the SLMC In-House Security
Department (IHSD) where she was directed to write an Incident Report explaining why she had the questioned
items in her possession. She complied with the directive and also submitted an undated handwritten letter of
apology.

An initial investigation was also conducted by the SLMC Division of Nursing which thereafter served Sanchez a
notice to explain. Sanchez submitted an Incident Report Addendum, explaining that the questioned items came
from the medication drawers of patients who had already been discharged, and, as similarly practiced by the
other staff members, she started saving these items as excess stocks in her pouch, along with other basic items
that she uses during her shift. She then put the pouch inside the lowest drawer of the bedside table in the
treatment room for use in immediate procedures in case replenishment of stocks gets delayed. However, on the
day of the incident, she failed to return the pouch inside the medication drawer upon getting her tri-colored pen
and calculator and, instead, placed it inside her bag. Eventually, she forgot about the same as she got caught
up in work, until it was noticed by the guard on duty on her way out of SMLCs premises.

Consequently, Sanchez was placed under preventive suspension until the conclusion of the investigation by
SLMCs Employee and Labor Relations Department (ELRD) which, thereafter, required her to explain why she
should not be terminated from service for acts of dishonesty due to her possession of the questioned items in
violation of Section 1, Rule I of the SLMC Code of Discipline. In response, she submitted a letter which merely
reiterated her claims in her previous letter. She likewise requested for a case conference, which SLMC granted.
After hearing her side, SLMC informed Sanchez of its decision to terminate her employment.

LAs Ruling: Sanchez was validly dismissed for intentionally taking the property of SLMCs clients for her own
personal benefit, which constitutes an act of dishonesty as provided under SLMCs Code of Discipline.

NLRCs Ruling: Sanchez was illegally dismissed. It declared that the alleged violation of Sanchez was a unique
case, considering that keeping excess hospital stocks or hoarding was an admitted practice amongst nurses in
the Pediatric Unit which had been tolerated by SLMC management for a long time. The NLRC held that while
Sanchez expressed remorse for her misconduct in her handwritten letter, she manifested that she only
hoarded the questioned items for future use in case their medical supplies are depleted, and not for her
personal benefit.

CAs Ruling: Sanchez was illegally dismissed. It ruled that Sanchezs offense did not qualify as serious
misconduct, given that: (a) the questioned items found in her possession were not SLMC property since said
items were paid for by discharged patients, thus discounting any material or economic damage on SLMCs part;
(b) the retention of excess medical supplies was an admitted practice amongst nurses in the Pediatric Unit
which was tolerated by SLMC; (c) it was illogical for Sanchez to leave the pouch in her bag since she would be
subjected to a routine inspection; (d) Sanchezs lack of intention to bring out the pouch was manifested by her
composed demeanor upon apprehension and offer to return the pouch to the treatment room; and (e) had
San Beda College of Law 620
4S: 2015 - 2016
LABOR LAW REVIEW Atty. Joyrich Golangco

SLMC honestly believed that Sanchez committed theft or pilferage, it should have filed the appropriate criminal
case, but failed to do so.
ISSUE: Was Sanchez illegally dismissed?

HELD: NO, Sanchez was legally dismissed.

Among the employers management prerogatives is the right to prescribe reasonable rules and regulations
necessary or proper for the conduct of its business or concern, to provide certain disciplinary measures to
implement said rules and to assure that the same would be complied with. At the same time, the employee has
the corollary duty to obey all reasonable rules, orders, and instructions of the employer; and willful or intentional
disobedience thereto, as a general rule, justifies termination of the contract of service and the dismissal of the
employee. Article 296 (formerly Article 282) of the Labor Code provides:
Article 296. Termination by Employer. - An employer may terminate an employment for any of
the following causes:
(a) Serious misconduct or willful disobedience by the employee of the lawful orders of his
employer or his representative in connection with his work;
xxxx

Note that for an employee to be validly dismissed on this ground, the employers orders, regulations, or
instructions must be: (1) reasonable and lawful, (2) sufficiently known to the employee, and (3) in connection
with the duties which the employee has been engaged to discharge.

The act of Sanchez is obviously connected with her work, who, as a staff nurse, is tasked with the proper
stewardship of medical supplies. Significantly, records show that Sanchez made a categorical admission in her
handwritten letter i.e., [k]ahit alam kong bawal ay nagawa kong [makapag-uwi] ng gamit that despite her
knowledge of its express prohibition under the SLMC Code of Discipline, she still knowingly brought out the
subject medical items with her. It is apt to clarify that SLMC cannot be faulted in construing the taking of the
questioned items as an act of dishonesty (particularly, as theft, pilferage, or its attempt in any form or manner)
considering that the intent to gain may be reasonably presumed from the furtive taking of useful property
appertaining to another. Note that Section 1, Rule 1 of the SLMC Code of Discipline is further supplemented by
the company policy requiring the turn-over of excess medical supplies/items for proper handling and providing a
restriction on taking and bringing such items out of the SLMC premises without the proper authorization or
pass from the official concerned, which Sanchez was equally aware thereof. Nevertheless, Sanchez failed to
turn-over the questioned items and, instead, hoarded them, as purportedly practiced by the other staff
members in the Pediatric Unit. As it is clear that the company policies subject of this case are reasonable and
lawful, sufficiently known to the employee, and evidently connected with the latters work, the Court concludes
that SLMC dismissed Sanchez for a just cause.

On a related point, the Court observes that there lies no competent basis to support the common observation of
the NLRC and the CA that the retention of excess medical supplies was a tolerated practice among the nurses
at the Pediatric Unit. While there were previous incidents of hoarding, it appears that such acts were in
similar fashion furtively made and the items secretly kept, as any excess items found in the concerned nurses
possession would have to be confiscated. Hence, the fact that no one was caught and/or sanctioned for
transgressing the prohibition therefor does not mean that the so-called hoarding practice was tolerated by
SLMC. Besides, whatever maybe the justification behind the violation of the company rules regarding excess
medical supplies is immaterial since it has been established that an infraction was deliberately committed.
Doubtless, the deliberate disregard or disobedience of rules by the employee cannot be countenanced as it may
encourage him or her to do even worse and will render a mockery of the rules of discipline that employees are
required to observe.

San Beda College of Law 621


4S: 2015 - 2016
LABOR LAW REVIEW Atty. Joyrich Golangco

SPI TECHNOLOGIES, INC. and LEA VILLANUEVA vs. VICTORIA K. MAPUA
G.R. No. 191154 April 7, 2014
REYES, J.:

AUTHORIZED CAUSE: Redundancy

DOCTRINE: In cases of redundancy, the management should adduce evidence and prove that a position which
was created in place of a previous one should pertain to functions which are dissimilar and incongruous to the
abolished office.
Presentation of the new table of the organization and the certification of the Human Resources
Supervisor that the positions occupied by the retrenched employees are redundant are inadequate as evidence
to support the colleges redundancy program.

FACTS: Mapua alleged that she was hired by SPI and was the Corporate Developments Research/Business
Intelligence Unit Head and Manager of the company. Subsequently, the then Vice President and Corporate
Development Head, Maquera hired Nolan as Mapuas supervisor.

Nolan informed Mapua that she was realigning Mapuas position to become a subordinate of co-manager Raina
due to her missing a work deadline (despite Mapuas report that her laptop crashed and lost her files). Nolan
also disclosed that Mapuas colleagues were "demotivated" because she was "taking things easy while they
were working very hard," and that she was "frequently absent, under timing, and coming in late every time
Maquera goes on leave or on vacation."

Mapua noticed that her colleagues began to ostracize and avoid her. Nolan and Raina started giving out
majority of her research work and other duties under Healthcare and Legal Division to the rank-and-file staff.
Mapua lost about 95% of her work projects and job responsibilities.

Mapua consulted these work problems with SPIs Human Resource Director and asked if she can be transferred
to another department within SPI. Subsequently, Mapua was informed that there is an intra-office opening and
that she would schedule an exploratory interview for her. However, due to postponements not made by Mapua,
the interview did not materialize.

Then, Mapua allegedly saw the new table of organization of the Corporate Development Division which would
be renamed as the Marketing Division. The new structure showed that Mapuas level will be again downgraded
because a new manager will be hired and positioned between her rank and Rainas.

On March 21, 2007, Raina informed Mapua over the phone that her position was considered redundant and that
she is terminated from employment effective immediately. Mapua was notified that she should cease reporting
for work the next day. Her laptop computer and company mobile phone were taken right away and her office
phone ceased to function.

Mapua filed a complaint for illegal dismissal, claiming reinstatement or if deemed impossible, for separation pay.

LAs Ruling: Mapuas termination was illegal, the redundancy of Mapuas position being in want of factual
basis.

NLRCs Ruling: Mapuas termination was legal.

CAs Ruling: Mapuas termination was illegal.

SPIs Argument: An employer has the unbridled right to conduct its own business in order to achieve the results
it desires. To prove that Villanuevas functions are redundant, SPI submitted an Inter-Office Memorandum and
affidavit executed by its Human Resources Director, Villanueva.

Mapuas Argument: Her position as Corporate Development Manager is not redundant. She cited that SPI was
in fact actively looking for her replacement after she was terminated. Furthermore, SPI violated her right to
procedural due process when her termination was made effective on the same day she was notified of it.

ISSUE: Was Mapuas termination legal?


San Beda College of Law 622
4S: 2015 - 2016
LABOR LAW REVIEW Atty. Joyrich Golangco

HELD: NO, Mapuas termination was illegal.

This Court in Asian Alcohol Corporation v. NLRC pronounced that for a valid implementation of a redundancy
program, the employer must comply with the following requisites: (1) written notice served on both the employee
and the DOLE at least one month prior to the intended date of termination; (2) payment of separation pay
equivalent to at least one month pay or at least one month pay for every year of service, whichever is higher; (3)
good faith in abolishing the redundant position; and (4) fair and reasonable criteria in ascertaining what positions
are to be declared redundant.

Anent the first requirement which is written notice served on both the employee and the DOLE at least one
month prior to the intended date of termination, SPI had discharged the burden of proving that it submitted a
notice to the DOLE, stating therein the effective date of termination.

In AMA Computer College, Inc. v. Garcia, et al., the Court held that the presentation of the new table of the
organization and the certification of the Human Resources Supervisor that the positions occupied by the
retrenched employees are redundant are inadequate as evidence to support the colleges redundancy program.
The Court quotes the related portion of its ruling:
In the case at bar, ACC attempted to establish its streamlining program by presenting its new
table of organization. ACC also submitted a certification by its Human Resources Supervisor, Ma.
Jazmin Reginaldo, that the functions and duties of many rank and file employees, including the
positions of Garcia and Balla as Library Aide and Guidance Assistant, respectively, are now being
performed by the supervisory employees. These, however, do not satisfy the requirement of
substantial evidence that a reasonable mind might accept as adequate to support a conclusion.
As they are, they are grossly inadequate and mainly self-serving. More compelling evidence
would have been a comparison of the old and new staffing patterns, a description of the
abolished and newly created positions, and proof of the set business targets and failure to attain
the same which necessitated the reorganization or streamlining.

Also connected with the evidence negating redundancy was SPIs publication of job vacancies after Mapua was
terminated from employment. SPI maintained that the CA erred when it considered Mapuas self-serving
affidavit as regards the Prime Manpower advertisement because the allegations therein were based on Mapuas
unfounded suspicions. Also, the failure of Mapua to present a sworn statement of Dimatulac renders the
formers statements hearsay.

Even if we disregard Mapuas affidavit as regards the Prime Manpower advertisement, SPI admitted that it
caused the Inquirer advertisement for a Marketing Communications Manager position. Mapua alleged that this
advertisement belied the claim of SPI that her position is redundant because the Corporate Development
division was only renamed to Marketing division.

A change in the job title is not synonymous to a change in the functions. A position cannot be abolished by a
mere change of job title. In cases of redundancy, the management should adduce evidence and prove that a
position which was created in place of a previous one should pertain to functions which are dissimilar and
incongruous to the abolished office.

Thus, in Caltex (Phils.), Inc. (now Chevron Phils., Inc.) v. NLRC, the Court dismissed the employers claim of
redundancy because it was shown that after declaring the employees position of Senior Accounting Analyst as
redundant, the company opened other accounting positions (Terminal Accountant and Internal Auditor) for
hiring. There was no showing that the private respondent therein could not perform the functions demanded of
the vacant positions, to which he could be transferred to instead of being dismissed.

San Beda College of Law 623


4S: 2015 - 2016
LABOR LAW REVIEW Atty. Joyrich Golangco

EUGENE S. ARABIT, EDGARDO C. SADSAD, LOWELL C. FUNTANOZ, GERARDO F. PUNZALAN,
FREDDIE M. MENDOZA, EMILIO B. BELEN, VIOLETA C. DIUMANO and MB FINANCE EMPLOYEES
ASSOCIATION FFW CHAPTER (FEDERATION OF FREE WORKERS) vs. JARDINE PACIFIC FINANCE,
INC. (FORMERLY MB FINANCE)
G.R. No. 181719 April 21, 2014
BRION, J.:

AUTHORIZED CAUSE: Redundancy

DOCTRINE: To dismiss the petitioners and hire new contractual employees as replacements necessarily give
rise to the sound conclusion that the petitioners services have not really become in excess of what the
employers business requires.

FACTS: Petitioners were former regular employees of respondent Jardine. The petitioners were also officers
and members of a legitimate labor union and the sole exclusive bargaining agent of the employees of Jardine.

On the claim of financial losses, Jardine decided to reorganize and implement a redundancy program among its
employees. The petitioners were among those affected by the redundancy program. Jardine thereafter hired
contractual employees to undertake the functions these employees used to perform.

Negotiations ensued between the Union and Jardine under the auspices of the NCMB, and both parties
eventually reached an amicable settlement. In the settlement, the petitioners accepted their redundancy pay
without prejudice to their right to question the legality of their dismissal with the NLRC. Jardine paid the
petitioners a separation package composed of their severance pay, plus their grossed up transportation
allowance.

The petitioners and the Union filed a complaint against Jardine with the NLRC for illegal dismissal and ULP.

Jardine argued in its defense that the company had been incurring substantial business losses from 1996 to
1998. Because of these serious business losses, Jardine asserted that it had to lay-off some of its employees
and reorganize its ranks to eliminate positions that were in excess of what its business required. Jardine,
however, admitted that it hired contractual employees to replace petitioners in their previous posts. Jardine
reasoned out that no bad faith took place since the hiring of contractual employees was a valid exercise of its
management prerogative. Jardine argued that the distinction between redundancy and retrenchment is not
material; an employer resorts to retrenchment or redundancy for the same reason, namely the economics of
business. Since Jardine successfully established that it incurred serious business losses, then termination of
employment of the petitioners was valid for all intents and purposes.

LAs Ruling: Petitioners were illegally dismissed. The hiring of contractual employees to replace the petitioners
directly contradicts the concept of redundancy which involves the trimming down of the workforce because a
task is being carried out by too many people.

NLRCs Ruling: Petitioners were illegally dismissed.

CAs Ruling: Petitioners were legally dismissed. The CA found that Jardines act of hiring contractual
employees in replacement of the petitioners does not run counter to the argument that their positions are
already superfluous. The hiring of contractual employees is a management prerogative that Jardine has the right
to exercise. In the absence of any showing of malice or arbitrariness on the part of Jardine in implementing its
redundancy program, the courts must not interfere with the companys exercise of a bona fide management
decision.

ISSUE: Were petitioners legally dismissed?

HELD: NO, Petitioners were illegally dismissed.

Redundancy in contrast with retrenchment

We cannot accept Jardines shallow understanding of the concepts of redundancy and retrenchment in
determining the validity of the severance of an employer-employee relationship. The fact that they are found
San Beda College of Law 624
4S: 2015 - 2016
LABOR LAW REVIEW Atty. Joyrich Golangco

together in just one provision does not necessarily give rise to the conclusion that the difference between them
is immaterial. This Court has already ruled before that retrenchment and redundancy are two different concepts;
they are not synonymous; thus, they should not be used interchangeably.50 The clear distinction between these
two concepts was discussed in Andrada, et al., v. NLRC, citing the case of Sebuguero v. NLRC, where this
Court clarified:

Redundancy exists where the services of an employee are in excess of what is reasonably demanded by the
actual requirements of the enterprise. A position is redundant where it is superfluous, and superfluity of a
position or positions may be the outcome of a number of factors, such as over hiring of workers, decreased
volume of business, or dropping of a particular product line or service activity previously manufactured or
undertaken by the enterprise.

Retrenchment, on the other hand, is used interchangeably with the term "lay-off." It is the termination of
employment initiated by the employer through no fault of the employees and without prejudice to the latter,
resorted to by management during periods of business recession, industrial depression, or seasonal
fluctuations, or during lulls occasioned by lack of orders, shortage of materials, conversion of the plant for a new
production program or the introduction of new methods or more efficient machinery, or of automation. Simply
put, it is an act of the employer of dismissing employees because of losses in the operation of a business, lack
of work, and considerable reduction on the volume of his business, a right consistently recognized and affirmed
by this Court.

These rulings appropriately clarify that redundancy does not need to be always triggered by a decline in the
business. Primarily, employers resort to redundancy when the functions of an employee have already become
superfluous or in excess of what the business requires. Thus, even if a business is doing well, an employer can
still validly dismiss an employee from the service due to redundancy if that employees position has already
become in excess of what the employers enterprise requires.

From this perspective, it is illogical for Jardine to terminate the petitioners employment and replace them with
contractual employees. The replacement effectively belies Jardines claim that the petitioners positions were
abolished due to superfluity. Redundancy could have been justified if the functions of the petitioners were
transferred to other existing employees of the company.

To dismiss the petitioners and hire new contractual employees as replacements necessarily give rise to the
sound conclusion that the petitioners services have not really become in excess of what Jardines business
requires. To replace the petitioners who were all regular employees with contractual ones would amount to a
violation of their right to security of tenure.

Guidelines in implementing redundancy

The Court, in Asian Alcohol Corp. v. NLRC, laid down guidelines for redundancy to be characterized as validly
undertaken by the employer. The Court ruled:

For the implementation of a redundancy program to be valid, the employer must comply with the following
requisites: (1) written notice served on both the employees and the Department of Labor and Employment at
least one month prior to the intended date of retrenchment; (2) payment of separation pay equivalent to at least
one month pay or at least one month pay for every year of service, whichever is higher; (3) good faith in
abolishing the redundant positions; and (4) fair and reasonable criteria in ascertaining what positions are to be
declared redundant and accordingly abolished.

Admittedly, Jardine complied with guidelines 1 and 2 of the guidelines in Asian Alcohol. Jardine informed the
Department of Labor and Employment of the petitioners separation from the service due to redundancy on April
30, 1999, one month before their terminations effectivity. Also, the petitioners were given their individual
separation packages, composed of their severance pay, plus their grossed up transportation allowance.

Guidelines 3 and 4 of Asian Alcohol, however, are different matters. These last two guidelines are interrelated to
ensure good faith in abolishing redundant positions; the employer must clearly show that it used fair and
reasonable criteria in ascertaining what positions are to be declared redundant.

San Beda College of Law 625


4S: 2015 - 2016
LABOR LAW REVIEW Atty. Joyrich Golangco

In this cited case, the employer took pains to discuss and elaborate on the reasons why the position of the
private respondent was the one chosen by the employer to be abolished.

Jardine never undertook what the employer in Asian Alcohol did. Jardine was never able to explain in any of its
pleadings why the petitioners positions were redundant. It never even attempted to discuss the attendant facts
and circumstances that led to the conclusion that the petitioners positions had become superfluous and
unnecessary to Jardines business requirements. Thus, we can only speculate on what actually happened.

To sum up, based on the guidelines set by the Court in the cases of Golden Thread and Asian Alcohol, we find
that at two levels, Jardine failed to set the required fair and reasonable criteria in the termination of the
petitioners employment, leading to the conclusion that the termination from the service was arbitrary and in bad
faith.

San Beda College of Law 626


4S: 2015 - 2016
LABOR LAW REVIEW Atty. Joyrich Golangco

PHILIPPINE CARPET MANUFACTURING CORPORATION, PACIFIC CARPET MANUFACTURING
CORPORATION, MR. PATRICIO LIM and MR. DAVID LIM vs. IGNACIO B. TAGYAMON,PABLITO L. LUNA,
FE B. BADA YOS, GRACE B. MARCOS, ROGELIO C. NEMIS, ROBERTO B. ILAO, ANICIA D. DELA CRUZ
and CYNTHIA L. COMANDAO
G.R. No. 191475 December 11, 2013
PERALTA, J.:

AUTHORIZED CAUSE: Retrenchment

DOCTRINE: The purchase of machinery and equipment after the termination, the declaration of cash dividends
to stockholders, the hiring of 100 new employees after the retrenchment, and the authorization of full blast
overtime work for six hours daily are inconsistent with petitioners claim that there was a slump in the demand
for its products which compelled them to implement the termination programs.

FACTS: Petitioner Philippine Carpet Manufacturing Corporation (PCMC) is a corporation registered in the
Philippines engaged in the business of manufacturing wool and yarn carpets and rugs. Respondents were its
regular and permanent employees, but were affected by petitioners retrenchment and voluntary retirement
programs.

On March 15, 2004, respondents received a uniformly worded Memorandum of dismissal due to the
implementation of a retrenchment program.

Claiming that they were aggrieved by PCMCs decision to terminate their employment, respondents filed
separate complaints for illegal dismissal against PCMC, Pacific Carpet Manufacturing Corporation, Mr. Patricio
Lim and Mr. David Lim. These cases were later consolidated. Respondents primarily relied on the Supreme
Courts decision in Philippine Carpet Employees Association (PHILCEA) v. Hon. Sto. Tomas (Philcea case), as
to the validity of the companys retrenchment program. They further explained that PCMC did not, in fact, suffer
losses shown by its acts prior to and subsequent to their termination. They also insisted that their acceptance of
separation pay and signing of quitclaim is not a bar to the pursuit of illegal dismissal case.

PCMC, for its part, defended its decision to terminate the services of respondents being a necessary
management prerogative. It pointed out that as an employer, it had no obligation to keep in its employ more
workers than are necessary for the operation of his business. Thus, there was an authorized cause for
dismissal.

LAs Ruling: The complaint was dismissed.

NLRCs Ruling: The NLRC sustained the decision of the LA.

CAs Ruling: Respondents were illegally dismissed.

ISSUE: Were the respondents illegally dismissed?

HELD: YES, respondents were illegally dismissed.

The illegality of the basis of the implementation of both voluntary retirement and retrenchment programs of
petitioners had been thoroughly ruled upon by the Court in the Philcea case. It discussed the requisites of both
retrenchment and redundancy as authorized causes of termination and that petitioners failed to substantiate
them. In ascertaining the bases of the termination of employees, it took into consideration petitioners claim of
business losses; the purchase of machinery and equipment after the termination, the declaration of cash
dividends to stockholders, the hiring of 100 new employees after the retrenchment, and the authorization of full
blast overtime work for six hours daily. These, said the Court, are inconsistent with petitioners claim that there
was a slump in the demand for its products which compelled them to implement the termination programs. In
arriving at its conclusions, the Court took note of petitioners net sales, gross and net profits, as well as net
income. The Court, thus, reached the conclusion that the retrenchment effected by PCMC is invalid due to a
substantive defect. We quote hereunder the Courts pronouncement in the Philcea case, to wit:

San Beda College of Law 627


4S: 2015 - 2016
LABOR LAW REVIEW Atty. Joyrich Golangco

Respondents failed to adduce clear and convincing evidence to prove the confluence of the essential requisites
for a valid retrenchment of its employees. We believe that respondents acted in bad faith in terminating the
employment of the members of petitioner Union.

Contrary to the claim of respondents that the Corporation was experiencing business losses, respondent
Corporation, in fact, amassed substantial earnings from 1999 to 2003. It found no need to appropriate its
retained earnings except on March 23, 2001, when it appropriated P60,000,000.00 to increase production
capacity. x x x

If respondent Corporation were to be believed that it had to retrench employees due to the debilitating slump in
demand for its products resulting in severe losses, how could it justify the purchase of P20,000,000.00 worth of
machinery and equipment? There is likewise no justification for the hiring of more than 100 new employees,
more than the number of those who were retrenched, as well as the order authorizing full blast overtime work for
six hours daily. All these are inconsistent with the intransigent claim that respondent Corporation was impelled
to retrench its employees precisely because of low demand for its products and other external causes.

xxxx

That respondents acted in bad faith in retrenching the 77 members of petitioner is buttressed by the fact that
Diaz issued his Memorandum announcing the cost-reduction program on March 9, 2004, after receipt of the
February 10, 2004 letter of the Union president which included the proposal for additional benefits and wage
increases to be incorporated in the CBA for the ensuing year. Petitioner and its members had no inkling, before
February 10, 2004, that respondent Corporation would terminate their employment. Moreover, respondent
Corporation failed to exhaust all other means to avoid further losses without retrenching its employees, such as
utilizing the latter's respective forced vacation leaves. Respondents also failed to use fair and reasonable criteria
in implementing the retrenchment program, and instead chose to retrench 77 of the members of petitioner out of
the dismissed 88 employees. Worse, respondent Corporation hired new employees and even rehired the others
who had been "retrenched."

As shown by the SGV & Co. Audit Report, as of year end December 31, 2003, respondent Corporation
increased its net sales by more than P8,000,000.00. Respondents failed to prove that there was a drastic or
severe decrease in the product sales or that it suffered severe business losses within an interval of three (3)
months from January 2004 to March 9, 2004 when Diaz issued said Memorandum. Such claim of a depressed
market as of March 9, 2004 was only a pretext to retaliate against petitioner Union and thereby frustrate its
demands for more monetary benefits and, at the same time, justify the dismissal of the 77 Union members.

xxxx

In contrast, in this case, the retrenchment effected by respondent Corporation is invalid due to a substantive
defect, non-compliance with the substantial requirements to effect a valid retrenchment; it necessarily follows
that the termination of the employment of petitioner Union's members on such ground is, likewise, illegal. As
such, they (petitioner Union's members) are entitled to reinstatement with full backwages.

San Beda College of Law 628


4S: 2015 - 2016
LABOR LAW REVIEW Atty. Joyrich Golangco

SANOH FULTON PHILS., INC. AND MR. EDDIE JOSE v. EMMANUEL BERNARDO AND SAMUEL TAGHOY
G.R. No. 187214 August 14, 2013
PEREZ, J.:

AUTHORIZED CAUSE: Retrenchment

DOCTRINE: Losses in the operation of the enterprise, lack of work, or considerable reduction on the volume of
business may justify an employer to reduce the work force. But a lull caused by lack of orders or shortage of
materials must be of such nature as would severely affect the continued business operations of the employer to
the detriment of all and sundry if not properly addressed.

FACTS: Sanoh is a domestic corporation engaged in the manufacture of automotive parts and wire condensers
for home appliances. Its Wire Condenser Department employed 61 employees. Respondents belonged to this
department.

In view of job order cancellations relating to the manufacture of wire condensers by Matsushita, Sanyo and
National Panasonic, Sanoh decided to phase out the Wire Condenser Department. On 22 December 2003, the
Human Resources Manager of Sanoh informed the 17 employees, 16 of whom belonged to the Wire Condenser
Department, of retrenchment effective 22 January 2004. All 17 employees are union members.

A grievance conference was held where the affected employees were informed of the following grounds for
retrenchment:
1) Lack of local market.
2) Competition from imported products.
3) Phasing out of Wire Condenser Department.

Two succeeding conciliation conferences were likewise held but the parties failed to reach an amicable
settlement. Hence, a case of illegal dismissal was filed.

Sanoh on its part, filed a petition for declaration of the partial closure of its Wire Condenser Department and
valid retrenchment of the 17 employees.

The complainants alleged that there was no valid cause for retrenchment and in effecting retrenchment, there
was a violation of the first in-last out and last in-first out (LIFO) policy embodied in the CBA

Sanoh, on the other hand, asserted that retrenchment was a valid exercise of management prerogative. Sanoh
averred that some employees who were hired much later were either assigned to other departments or were
bound by the terms of their job training agreement to stay with the company for 3 years.

LAs Ruling: The complaint for illegal dismissal was dismissed.

NLRCs Ruling: The Decision of the LA was affirmed.

CAs Ruling: The CA overturned the findings of the Labor Arbiter and the NLRC, and ruled that Sanoh failed to
prove the existence of substantial losses that would justify a valid retrenchment.

Petitioners Argument: It is the prerogative of management to effect retrenchment as long as it is done in good
faith. Sanoh relies on letters from its customers showing cancellation of job orders to prove that it is suffering
from serious losses. In addition, Sanoh claims that it had, in fact, closed down the Wire Condenser Department
in view of serious business losses.

Respondents Argument: The Wire Condenser Department was not phased out and there was no need to
retrench the personnel. Respondents point out that Sanoh even made the retained employees render
substantial overtime work. Respondents refute the allegation of serious business losses by producing
documentary evidence to the contrary.

ISSUE: Was there a valid retrenchment?

HELD: NO, there was no valid retrenchment. Nor there was a valid closure of business.
San Beda College of Law 629
4S: 2015 - 2016
LABOR LAW REVIEW Atty. Joyrich Golangco

We are mindful of the principle that losses in the operation of the enterprise, lack of work, or considerable
reduction on the volume of business may justify an employer to reduce the work force. But a lull caused by lack
of orders or shortage of materials must be of such nature as would severely affect the continued business
operations of the employer to the detriment of all and sundry if not properly addressed.

We held in Lambert Pawnbrokers and Jewelry Corporation v. Binamira, that the losses must be supported by
sufficient and convincing evidence and the normal method of discharging this is by the submission of financial
statements duly audited by independent external auditors. It was aptly observed by the appellate court that no
financial statements or documents were presented to substantiate Sanohs claim of loss of P7 million per month.
And a business lull caused by lack of orders which could have justified retrenchment was not shown by
petitioner. As observed once more by the Court of Appeals, petitioner failed to present proof of the extent of the
reduced order and its contribution to the sustainability of its business.

As the Wire Condenser Department is still in operation and no business losses were proven by Sanoh, the
dismissal of respondents was unlawful.

San Beda College of Law 630


4S: 2015 - 2016
LABOR LAW REVIEW Atty. Joyrich Golangco

RUBEN L. ANDRADA, BERNALDO V. DELOS SANTOS, JOVEN M. PABUSTAN, FILAMER ALFONSO,
VICENTE A. MANTALA, JR., HARVEY D. CAYETANO, and JOVENCIO L. POBLETE vs NATIONAL LABOR
RELATIONS COMMISSION, SUBIC LEGEND RESORTS AND CASINO, INC., and/or MR. HWA PUAY, MS.
FLORDELIZA MARIA REYES RAYEL, and its CORPORATE OFFICERS
G.R. No. 173231 December 28, 2007
VELASCO, JR., J.

AUTHORIZED CAUSE: Retrenchment

DOCTRINE: Failure to show financial conditions prior to and at the time of enforcement of retrenchment
program and failure to submit audited financial statements regarding alleged financial losses will result to illegal
retrenchment for not establishing it by substantial evidence.

FACTS: The petitioners in this case, whose names appear in the title, are employees of Subic legend Resorts
and Casino, Inc. who were hired on various dates from 1995 up to 1997 working as architects, draftsmen,
operators, engineers, and surveyors in the Project Development Division on various projects. Herein
respondents Mr. Hwa Puay and Ms. Flordeliza Maria Reyes Rayel are impleaded in this case in their official
capacities as officers of said resort and casino.

On January 6, 1998, Subic Legend Resorts and Casino, Inc. decided to retrench and terminate the employment
of some thirty four (34) employees including herein petitioners sending notice of such action to the Department
of Labor and Employment. Legend has undertaken this action on the strength of the updated status report of its
Project Development Division, as follows:
1. shelving of the condotel project until economic conditions in the Philippines improve.
2. completion of the temporary casino in Cubi by Mid-February 1998
3. subcontracting the super structure work of Grand Legend to a third party
4. completion of the rectification work at the Legenda hotel
5. completion of the temporary casino in Cubi
6. abolition of the Personnel and Administrative department of the Project Development division and
transfer of its function back to Legends Human Resource Department.

For the same reasons, Legend sent to the 34 employees the notices of retrenchment and offering them
retrenchment options. After they have chosen their options they signed a quitclaim reserving the right to sue
should their separation benefits not be settled by January 30, 2008.

On that same day, Labor and Employment Center of Subic Bay Metropolitan Authority advertised that Legend
International Resorts, Inc. was in need of employees for the positions which were very much similar to those
vacated by the petitioners.

On March 3, 1998, fourteen (14) of the 34 retrenched employees filed before the Regional Arbitration Branch of
the NLRC in San Fernando, Pampanga a complaint for illegal dismissal based on the advertisement made by
the Subic Bay Metropolitan Authority saying that Legend have created positions similar to those which they have
vacated. Legend however, invoked management prerogatives.

The Labor Arbiter decided that the petitioners were illegally dismissed on the ground of insufficiency of
documents showing that legend had suffered actual losses or that there were redundancy of positions as
occupied by the petitioners. Legend then appealed the decision to the National Labor Relations which reversed
the decision.

Herein petitioners appealed the NLRC decision to the Court of Appeals. The latter however sustained the CAs
ruling and held that the employees were validly terminated from employment due to redundancy and not
retrenchment. Aggrieved with the CAs decision, the above petitioners appealed to the Supreme Court.

ISSUE: Whether or not the complainants were illegally dismissed? Corollary, was there a valid retrenchment?

COURT RULING: The Supreme Court favors the petitioners.

San Beda College of Law 631


4S: 2015 - 2016
LABOR LAW REVIEW Atty. Joyrich Golangco

It held that a companys prerogative is not absolute. It cannot exercise its prerogative in a cruel, repressive, or
despotic manner. In the case of Ariola vs. Philex Mining Corporation, the following are requirements of a valid
retrenchment:
1. It is undertaken to prevent losses, which are not merely de minimis, but substantial, serious, actual
and real, or if only expected, are reasonably imminent as perceived objectively and in good faith by
the employer
2. The employer serves written notice both to the employees and the DOLE at least one month prior to
the intended date of retrenchment
3. The employer pays the retrenched employees separation pay equivalent to one month pay or at least
month pay for every year of service, whichever is higher.

The court further added that the employer must use fair and reasonable criteria in ascertaining who would be
dismissed and retained among the employees and that the retrenchment must be undertaken in good faith.

Supreme Court held that in the instant case, Legend glaringly failed to show its financial conditions prior to and
at the time it enforced its retrenchment program. Furthermore, it failed to submit audited financial statements
regarding its alleged financial losses. Thus, the retrenchment was illegal.

On the issue of redundancy, Supreme Court held that Legend failed to establish it as such. It further explains
that retrenchment and redundancy are two different concepts. The difference of which were clearly elaborated in
the case of Sebuguero vs. NLRC. To simply put it, redundancy according to the Supreme Court exists when the
number of employees is in excess of what is reasonably necessary to operate the business.

Thus, the Supreme Court finally held, that the basis for retrenchment was not established by substantial
evidence and it also ruled that Legend failed to establish by the same quantum of proof the fact of redundancy;
hence the petitioners termination from employment was illegal.

San Beda College of Law 632


4S: 2015 - 2016
LABOR LAW REVIEW Atty. Joyrich Golangco

NELSON A. CULILI v. EASTERN TELECOMMUNICATIONS PHILIPPINES, INC., SALVADOR HIZON
(President and Chief Executive Officer), EMILIANO JURADO (Chairman of the Board), VIRGILIO GARCIA
(Vice President) and STELLA GARCIA (Assistant Vice President)
G.R. No. 165381 February 9, 2011
LEONARDO-DE CASTRO, J.:

AUTHORIZED CAUSE: Redundancy

DOCTRINE: The following evidence may be proffered to substantiate redundancy: the new staffing pattern,
feasibility studies/ proposal on the viability of the newly created positions, job description and the approval by
the management of the restructuring.

FACTS: Culili was a Senior Technician in the Customer Premises Equipment Management Unit of the Service
Quality Department of Eastern Telecom (ETPI). Due to business troubles and losses, ETPI was compelled to
implement a Right-Sizing Program which consisted of two phases: the first phase involved the reduction of
ETPIs workforce to only those employees that were necessary and which ETPI could sustain; the second
phase entailed a company-wide reorganization which would result in the transfer, merger, absorption or abolition
of certain departments of ETPI. ETPI offered the Special Retirement Program and the corresponding retirement
package to the 102 employees who qualified for the program. Of all the employees who qualified to avail of the
program, only Culili rejected the offer. After the successful implementation of the first phase of the Right-Sizing
Program, ETPI proceeded with the second phase which necessitated the abolition, transfer and merger of a
number of ETPIs departments. The specialized functions of a Senior Technician unnecessary. As a result,
Culilis position was abolished due to redundancy and his functions were absorbed by another employee. Later,
he was informed of his termination from employment through a letter which was similar to the memo shown to
Culili by the union president weeks before Culili was dismissed. The memo was advising him of his dismissal
effective due to the Right-Sizing Program ETPI was going to implement to cut costs and avoid losses. Culili
alleged that neither he nor the Department of Labor and Employment (DOLE) were formally notified of his
termination. Culili claimed that he only found out about it after he was barred from entering ETPIs premises by
its armed security personnel when he tried to report for work. Culili asserted that ETPI had contracted out the
services he used to perform to a labor-only contractor which not only proved that his functions had not become
unnecessary, but which also violated their CBA and the Labor Code. ETPI denied singling Culili out for
termination. ETPI averred that since Culili did not avail of the Special Retirement Program and his position was
subsequently declared redundant, it had no choice but to terminate Culili.

LA: He rendered a decision finding, among others, ETPI guilty of illegal dismissal and unfair labor practice, and
ordering it to reinstate Culili with backwages and such other benefits due him, and damages and attorneys fees.

NLRC: Affirmed the LAs decision but modified the amount when it failed to properly notify both Culili and the
DOLE of Culilis termination.

CA: partially granted ETPIs petition and deleted the award for damages. CA found that Culilis position was
validly abolished due to redundancy

ISSUES:
1. Is Culili validly dismissed?
2. Did ETPI fail to properly comply with the procedural due process requirement in terminating Culili?

HELD:
1. Yes. There is redundancy when the service capability of the workforce is greater than what is reasonably
required to meet the demands of the business enterprise. A position becomes redundant when it is rendered
superfluous by any number of factors such as over-hiring of workers, decrease in volume of business, or
dropping a particular product line or service activity previously manufactured or undertaken by the enterprise.

This Court has been consistent in holding that the determination of whether or not an employees services are
still needed or sustainable properly belongs to the employer. Provided there is no violation of law or a showing
that the employer was prompted by an arbitrary or malicious act, the soundness or wisdom of this exercise of
business judgment is not subject to the discretionary review of the Labor Arbiter and the NLRC.

San Beda College of Law 633


4S: 2015 - 2016
LABOR LAW REVIEW Atty. Joyrich Golangco

However, an employer cannot simply declare that it has become overmanned and dismiss its employees without
producing adequate proof to sustain its claim of redundancy. Among the requisites of a valid redundancy
program are: (1) the good faith of the employer in abolishing the redundant position; and (2) fair and reasonable
criteria in ascertaining what positions are to be declared redundant, such as but not limited to: preferred status,
efficiency, and seniority.

This Court also held that the following evidence may be proffered to substantiate redundancy: the new staffing
pattern, feasibility studies/ proposal on the viability of the newly created positions, job description and the
approval by the management of the restructuring.

In the case at bar, ETPI was upfront with its employees about its plan to implement a Right-Sizing Program.
Even in the face of initial opposition from and rejection of the said program by ETEU, ETPI patiently negotiated
with ETEUs officers to make them understand ETPIs business dilemma and its need to reduce its workforce and
streamline its organization. This evidently rules out bad faith on the part of ETPI.

In deciding which positions to retain and which to abolish, ETPI chose on the basis of efficiency, economy,
versatility and flexibility. It needed to reduce its workforce to a sustainable level while maintaining functions
necessary to keep it operating. The records show that ETPI had sufficiently established not only its need to
reduce its workforce and streamline its organization, but also the existence of redundancy in the position of a
Senior Technician. ETPI explained how it failed to meet its business targets and the factors that caused this,
and how this necessitated it to reduce its workforce and streamline its organization. ETPI also submitted its old
and new tables of organization and sufficiently described how limited the functions of the abolished position of a
Senior Technician were and how it decided on whom to absorb these functions.

In the new table of organization that the management approved, one hundred twelve (112) employees were
redeployed and nine (9) positions were declared redundant. It is inconceivable that ETPI would effect a
company-wide reorganization of this scale for the mere purpose of singling out Culili and terminating him. If
Culilis position were indeed indispensable to ETPI, then it would be absurd for ETPI, which was then trying to
save its operations, to abolish that one position which it needed the most. Contrary to Culilis assertions that
ETPI could not do away with his functions as long as it is in the telecommunications industry, ETPI did not
abolish the functions performed by Culili as a Senior Technician. What ETPI did was to abolish the position itself
for being too specialized and limited. The functions of that position were then added to another employee whose
functions were broad enough to absorb the tasks of a Senior Technician.

2. Yes. ETPI, in effecting Culilis termination, simply asked one of its guards to serve the required written notice
on Culili. Culili, on one hand, claims in his petition that this was handed to him by ETPIs vice president, but
previously testified before the Labor Arbiter that this was left on his table. Regardless of how this notice was
served on Culili, this Court believes that ETPI failed to properly notify Culili about his termination. Aside from
the manner the written notice was served, a reading of that notice shows that ETPI failed to properly inform
Culili of the grounds for his termination. Since it has been established that Culilis termination was due
to an authorized cause and cannot be considered unfair labor practice on the part of ETPI, his dismissal
is valid. However, in view of ETPIs failure to comply with the notice requirements under the Labor Code,
Culili is entitled to nominal damages in addition to his separation pay.

As for the award of moral and exemplary damages in illegal dismissal cases, moral damages are awarded only
where the dismissal was attended by bad faith or fraud, or constituted an act oppressive to labor, or was done in
a manner contrary to morals, good customs or public policy. Exemplary damages may avail if the dismissal was
effected in a wanton, oppressive or malevolent manner to warrant an award for exemplary damages. It is our
considered view that Culili has failed to prove that his dismissal was orchestrated by the individual respondents
herein (ETPI officers) for the mere purpose of getting rid of him. In fact, most of them have not even dealt with
Culili personally. Moreover, it has been established that his termination was for an authorized cause, and that
there was no bad faith on the part of ETPI in implementing its Right-Sizing Program, which involved abolishing
certain positions and departments for redundancy. It is not enough that ETPI failed to comply with the due
process requirements to warrant an award of damages, there being no showing that the companys and its
officers acts were attended with bad faith or were done oppressively.

San Beda College of Law 634


4S: 2015 - 2016
LABOR LAW REVIEW Atty. Joyrich Golangco

BONIFACIO ASUFRIN, JR. vs. SAN MIGUEL CORPORATION and the COURT OF APPEALS
G.R. No. 156658 March 10, 2004
YNARES-SANTIAGO, J.:

AUTHORIZED CAUSE: Redundancy

DOCTRINE: It is not enough for a company to merely declare that it has become over manned. It must produce
adequate proof that such is the actual situation to justify the dismissal of the affected employees for redundancy.

FACTS: Coca Cola Plant, then a department of respondent San Miguel Beer Corporation (SMC), hired
petitioner as utility/miscellaneous worker in February 1972. On November 1, 1973, he became a regular
employee paid on daily basis as a Forklift Operator. On November 16, 1981, he became a monthly paid
employee promoted as Stock Clerk.

Sometime in 1984, the sales office and operations at the Sum-ag, Bacolod City Sales Office were reorganized.
Several positions were abolished including petitioners position as Stock Clerk. After reviewing petitioners
qualifications, he was designated warehouse checker at the Sum-ag Sales Office.

On April 1, 1996, respondent SMC implemented a new marketing system known as the pre-selling scheme at
the Sum-ag Beer Sales Office. As a consequence, all positions of route sales and warehouse personnel were
declared redundant. Respondent notified the DOLE Director of Region VI that 22 personnel of the Sales
Department of the Negros Operations Center would be retired effective March 31, 1995.

Respondent SMC thereafter wrote a letter to petitioner informing him that, owing to the implementation of the
pre-selling operations scheme, all positions of route and warehouse personnel will be declared redundant and
the Sum-ag Sales Office will be closed effective April 30, 1996. Thus, from April 1, 1996 to May 15, 1996,
petitioner reported to respondents Personnel Department at the Sta. Fe Brewery, pursuant to a previous
directive.

Thereafter, the employees of Sum-ag sales force were informed that they can avail of respondents early
retirement package pursuant to the retrenchment program, while those who will not avail of early retirement
would be redeployed or absorbed at the Brewery or other sales offices. Petitioner opted to remain and
manifested to Acting Personnel Manager Salvador Abadesco his willingness to be assigned to any job,
considering that he had three children in college.

Petitioner was surprised when he was informed by the Acting Personnel Manager that his name was included in
the list of employees who availed of the early retirement package. Petitioners request that he be given an
assignment in the company was ignored by the Acting Personnel Manager. Petitioner thus filed a complaint for
illegal dismissal.

ISSUE: Was the dismissal of petitioner based on a just and authorized cause.

HELD: NO. In the case at bar, petitioner was dismissed on the ground of redundancy, one of the authorized
causes for dismissal. The determination that employees services are no longer necessary or sustainable and,
therefore, properly terminable is an exercise of business judgment of the employer. The wisdom or soundness
of this judgment is not subject to discretionary review of the Labor Arbiter and the NLRC, provided there is no
violation of law and no showing that it was prompted by an arbitrary or malicious act. In other words, it is not
enough for a company to merely declare that it has become over manned. It must produce adequate proof that
such is the actual situation to justify the dismissal of the affected employees for redundancy.

Persuasive as the explanation proffered by respondent may be to justify the dismissal of petitioner, a number of
disturbing circumstances, however, leave us unconvinced.

First, of the 23 SMC employees assigned at the Sum-ag Sales Office/Warehouse, 9 accepted the offer of SMC
to avail of the early retirement whose separation benefits was computed at 250% of their regular pay. The rest,
including petitioner, did not accept the offer. Out of the remaining 14, only petitioner clearly manifested, through
several letters, his desire to be redeployed to the Sta. Fe Brewery or any sales office and for any position not
necessarily limited to that of a warehouse checker. In short, he was even willing to accept a demotion just to

San Beda College of Law 635


4S: 2015 - 2016
LABOR LAW REVIEW Atty. Joyrich Golangco

continue his employment. Meanwhile, other employees who did not even write a letter to SMC were redeployed
to the Sta. Fe Brewery or absorbed by other offices/outlets outside Bacolod City.

Second, petitioner was in the payroll of the Sta. Fe Brewery and assigned to the Materials Section, Logistics
Department, although he was actually posted at the Sum-ag Warehouse. Thus, even assuming that his position
in the Sum-ag Warehouse became redundant, he should have been returned to the Sta. Fe Brewery where he
was actually assigned and where there were vacant positions to accommodate him.

Third, it appears that despite respondents allegation that it ceased and closed down its warehousing operations
at the Sum-ag Sales Office, actually it is still used for warehousing activities and as a transit point where buyers
and dealers get their stocks. Indeed, the Sum-ag Office is strategically situated on the southern part of Bacolod
City making it convenient for dealers from the southern towns of Negros Occidental to get their stocks and
deposit their empty bottles in the said warehouse, thereby decongesting the business activities at the Sta. Fe
Brewery.

Fourth, in selecting employees to be dismissed, a fair and reasonable criteria must be used, such as but not
limited to (a) less preferred status, e.g. temporary employee; (b) efficiency; and (c) seniority. In the case at bar,
no criterion whatsoever was adopted by respondent in dismissing petitioner. Furthermore, as correctly observed
by the NLRC, respondent has not shown how the cessation of operations of the Sum-ag Sales Office
contributed to the ways and means of improving effectiveness of the organization with the end in view of
efficiency and cutting distribution overhead and other related costs. Respondent, thus, clearly resorted to
sweeping generalization[s] in dismissing complainant. Indeed, petitioners predicament may have something to
do with an incident where he incurred the ire of an immediate superior in the Sales Logistics Unit for exposing
certain irregularities committed by the latter.

San Beda College of Law 636


4S: 2015 - 2016
LABOR LAW REVIEW Atty. Joyrich Golangco

VICENTE SY, TRINIDAD PAULINO, 6BS TRUCKING CORPORATION, and SBT TRUCKING
CORPORATION vs. HON. COURT OF APPEALS and JAIME SAHOT
G.R. No. 142293 February 27, 2003
QUISUMBING, J.:.

AUTHORIZED CAUSE: Disease Medical Certificate Requirement

DOCTRINE: The requirement for a medical certificate under Article 284 of the Labor Code cannot be dispensed
with; otherwise, it would sanction the unilateral and arbitrary determination by the employer of the gravity or
extent of the employees illness and thus defeat the public policy in the protection of labor.

FACTS: Sometime in 1958, private respondent Jaime Sahot5 started working as a truck helper for petitioners
family-owned trucking business named Vicente Sy Trucking. In 1965, he became a truck driver of the same
family business, renamed T. Paulino Trucking Service, later 6Bs Trucking Corporation in 1985, and thereafter
known as SBT Trucking Corporation since 1994. For 36 years, private respondent continuously served the
trucking business of petitioners.

In April 1994, Sahot then 59 years old, incurred absences as he was suffering from various ailments, particularly
the pain in his left thigh, which greatly affected the performance of his task. He inquired about his medical and
retirement benefits with the (SSS) but discovered that his premium payments had not been remitted by his
employer.
In May 1994, he filed a week-long leave. On May 27th, he was medically examined and treated for EOR,
presleyopia, hypertensive retinopathy G II, HPM, UTI, Osteoarthritis and heart enlargement. On said grounds,
Belen Paulino of the SBT Trucking Service management told him to file a formal request for extension of his
leave. Sahot applied for extension of his leave for the whole month of June, 1994. It was at this time when
petitioners allegedly threatened to terminate his employment should he refuse to go back to work.
Subsequently, he was dismissed from work, effective June 30, 1994.

Sahot filed a complaint for illegal dismissal with prayer for the recovery of separation pay and attorneys fees
against Vicente Sy and Trinidad Paulino-Sy, Belen Paulino, Vicente Sy Trucking, T. Paulino Trucking Service,
6Bs Trucking and SBT Trucking.
Petitioners contended that private respondent was not illegally dismissed as a driver because he was in fact
petitioners industrial partner. They add that it was not until the year 1994, when SBT Trucking Corporation was
established, and only then did respondent Sahot become an employee of the company.

Petitioners further claimed that from the expiration of his leave until June 30, private respondent never reported
back to work nor did he file an extension of his leave. Instead, he filed the complaint for illegal dismissal against
the trucking company and its owners.

LA: held that there was no illegal dismissal in Sahots case. Private respondent had failed to report to work. He
also held that petitioners and private respondent were industrial partners before January 1994. He ordered
petitioners to pay "financial assistance" of P15,000 to Sahot for having served the company as a regular
employee since January 1994.

NLRC: declared that private respondent was an employee, not an industrial partner, since the start. It held that
Sahot did not abandon his job but was terminated on account of his illness, pursuant to Article 284. It ordered
petitioners to pay private respondent separation pay at the rate of P2,080 per year for 29 years of service.

CA: held that private respondent was indeed an employee of petitioners since 1958. It also increased the
amount of separation pay at the rate of P2,080 per year for 36 years of service from 1958 to 1994.

ISSUES:
1. Was there was an employer-employee relationship?
2. Was private respondent validly dismissed by petitioners?
3. Is private respondent entitled to separation pay?

HELD:

San Beda College of Law 637


4S: 2015 - 2016
LABOR LAW REVIEW Atty. Joyrich Golangco

1. YES. SC did not entertain petitioners contention that complainant was an industrial partner, since a
computation of the complainants age shows that he was only 23 years old when he started working
with respondent as truck helper Moreover, not one of the circumstances in a contract of partnership is
present in this case. In contrast, the requisites for an Er-Ee relationship exist.)

2. NO, Article 284 of the Labor Code authorizes an employer to terminate an employee on the ground of
disease. However, in order to validly terminate employment on this ground, Book VI, Rule I, Section 8 of
the Omnibus Implementing Rules of the Labor Code requires:

Sec. 8. Disease as a ground for dismissal- Where the employee suffers from a disease and his
continued employment is prohibited by law or prejudicial to his health or to the health of his co-
employees, the employer shall not terminate his employment unless there is a certification by
competent public health authority that the disease is of such nature or at such a stage that it cannot be
cured within a period of six (6) months even with proper medical treatment. If the disease or ailment can
be cured within the period, the employer shall not terminate the employee but shall ask the employee to
take a leave. The employer shall reinstate such employee to his former position immediately upon the
restoration of his normal health. (Italics supplied).

As this Court stated in Triple Eight integrated Services, Inc. vs. NLRC the requirement for a medical
certificate under Article 284 of the Labor Code cannot be dispensed with; otherwise, it would sanction
the unilateral and arbitrary determination by the employer of the gravity or extent of the employees
illness and thus defeat the public policy in the protection of labor.

In the case at bar, the employer clearly did not comply with the medical certificate requirement before
Sahots dismissal was effected. In the same case of Sevillana vs. I.T. (International) Corp., we ruled:

Since the burden of proving the validity of the dismissal of the employee rests on the employer, the
latter should likewise bear the burden of showing that the requisites for a valid dismissal due to a
disease have been complied with. In the absence of the required certification by a competent public
health authority, this Court has ruled against the validity of the employees dismissal. It is therefore
incumbent upon the private respondents to prove by the quantum of evidence required by law that
petitioner was not dismissed, or if dismissed, that the dismissal was not illegal; otherwise, the dismissal
would be unjustified. This Court will not sanction a dismissal premised on mere conjectures and
suspicions, the evidence must be substantial and not arbitrary and must be founded on clearly
established facts sufficient to warrant his separation from work.

In addition, petitioners failed to furnish private respondent of the two written notices prior to dismissal,
even only for record purposes. What management did was to threaten the employee with dismissal,
then actually implement the threat when the occasion presented itself because of private respondents
painful left thigh. All told, both the substantive and procedural aspects of due process were violated.
Clearly, therefore, Sahots dismissal is tainted with invalidity.

3. YES. An employee who is terminated because of disease is entitled to "separation pay equivalent to at
least one month salary or to one-half month salary for every year of service, whichever is greater
xxx."34 Following the formula set in Art. 284 of the Labor Code, his separation pay was computed by
the appellate court at P2,080 times 36 years (1958 to 1994) or P74,880. We agree with the
computation, after noting that his last monthly salary was P4,160.00 so that one-half thereof is
P2,080.00. Finding no reversible error nor grave abuse of discretion on the part of appellate court, we
are constrained to sustain its decision. To avoid further delay in the payment due the separated worker,
whose claim was filed way back in 1994, this decision is immediately executory. Otherwise, six percent
(6%) interest per annum should be charged thereon, for any delay, pursuant to provisions of the Civil
Code.

San Beda College of Law 638


4S: 2015 - 2016
LABOR LAW REVIEW Atty. Joyrich Golangco

UNION MOTOR CORPORATION vs. NATIONAL LABOR RELATIONS COMMISSION and ALEJANDRO A.
ETIS
G.R. No. 159738 December 9 2004
CALLEJO, SR., J.:

AUTHORIZED CAUSE: Disease

FACTS: On October 23, 1993, the respondent was hired by the petitioner as an automotive mechanic at the
service department in the latters Paco Branch. In 1994, he was transferred to the Caloocan City Branch, where
his latest monthly salary was P6,330.00. During his employment, he was awarded the Top Technician for the
month of May in 1995 and Technician of the Year (1995). He also became a member of the Exclusive
P40,000.00 Club and received the Model Employee Award in the same year.

On September 22, 1997, the respondent made a phone call to Rosita dela Cruz, the company nurse, and
informed her that he had to take a sick leave as he had a painful and unbearable toothache. The next day, he
again phoned Dela Cruz and told her that he could not report for work because he still had to consult a doctor.
Finding that the respondents ailment was due to a tooth inflammation, the doctor referred him to a dentist for
further management. Dr. Rodolfo Pamor, a dentist, then scheduled the respondents tooth extraction on
September 27, 1997, hoping that, by that time, the inflammation would have subsided.

On September 27, 1997, Dr. Pamor rescheduled the respondents tooth extraction on October 4, 1997 because
the inflammation had not yet subsided and recommended that he rest. Thus, the respondent was not able to
report for work due to the painful and unbearable toothache.

On October 2, 1997, the petitioner issued an Inter Office Memorandum37[3] through Angelo B. Nicolas, the
manager of its Human Resources Department, terminating the services of the respondent for having incurred
more than five (5) consecutive absences without proper notification. The petitioner considered the consecutive
absences of the respondent as abandonment of office under Section 6.1.1, Article III of the Company Rules.

On October 4, 1997, Dr. Pamor successfully extracted the respondents tooth. As soon as he had recovered, the
respondent reported for work, but was denied entry into the companys premises. He was also informed that his
employment had already been terminated.

Left with no other recourse, the respondent filed, on May 18, 1999, a complaint for illegal dismissal before the
arbitration branch of the NLRC against the petitioner and/or Benito Cua.

ISSUE: Whether or not respondent was validly dismissed

HELD: NO. The termination by respondent-appellee of complainants service despite knowledge of


complainants ailment, as shown by the telephone calls made by the latter to the company nurse and the actual
confirmation made by respondents company guard, who personally visited complainants residence, clearly
establishes the illegality of complainants dismissal.

We agree with the rulings of the NLRC and the CA. We note that the company rules do not require that the
notice of an employees absence and the reasons therefor be in writing and for such notice to be given to any
specific office and/or employee of the petitioner. Hence, the notice may be verbal; it is enough then that an
officer or employee of the petitioner, competent and responsible enough to receive such notice for and in behalf
of the petitioner, was informed of such absence and the corresponding reason.

While the records do not reveal that the respondent filed the required leave of absence for the period during
which he suffered from a toothache, he immediately reported for work upon recovery, armed with medical
certificates to attest to the cause of his absence. The respondent could not have anticipated the cause of his
illness, thus, to require prior approval would be unreasonable. While it is true that the petitioner had objected to
the veracity of the medical certificates because of lack of notarization, it has been said that verification of
documents is not necessary in order that the said documents could be considered as substantial evidence. The
medical certificates were properly signed by the physicians; hence, they bear all the earmarks of regularity in
their issuance and are entitled to full probative weight.

San Beda College of Law 639


4S: 2015 - 2016
LABOR LAW REVIEW Atty. Joyrich Golangco

ROMEO VILLARUEL v. YEO HAN GUAN, doing business under the name and style YUHANS
ENTERPRISES
G.R. No. 169191 June 1 2011
PERALTA, J.:

AUTHORIZED CAUSE: Disease

FACTS: On February 15, 1999, herein petitioner filed with the NLRC, National Capital Region, Quezon City a
Complaint for payment of separation pay against Yuhans Enterprises.

Petitioner alleged that in June 1963, he was employed as a machine operator by Ribonette Manufacturing
Company, an enterprise engaged in the business of manufacturing and selling PVC pipes and is owned and
managed by herein respondent Yeo Han Guan. Over a period of almost twenty (20) years, the company
changed its name four times. Starting in 1993 up to the time of the filing of petitioner's complaint in 1999, the
company was operating under the name of Yuhans Enterprises. Despite the changes in the company's name,
petitioner remained in the employ of respondent. Petitioner further alleged that on October 5, 1998, he got sick
and was confined in a hospital; on December 12, 1998, he reported for work but was no longer permitted to go
back because of his illness; he asked that respondent allow him to continue working but be assigned a lighter
kind of work but his request was denied; instead, he was offered a sum of P15,000.00 as his separation pay;
however, the said amount corresponds only to the period between 1993 and 1999; petitioner prayed that he be
granted separation pay computed from his first day of employment in June 1963, but respondent refused. Aside
from separation pay, petitioner prayed for the payment of service incentive leave for three years as well as
attorney's fees.

On the other hand, respondent averred in his Position Paper that petitioner was hired as machine operator from
March 1, 1993 until he stopped working sometime in February 1999 on the ground that he was suffering from
illness; after his recovery, petitioner was directed to report for work, but he never showed up. Respondent was
later caught by surprise when petitioner filed the instant case for recovery of separation pay. Respondent
claimed that he never terminated the services of petitioner and that during their mandatory conference, he even
told the latter that he could go back to work anytime but petitioner clearly manifested that he was no longer
interested in returning to work and instead asked for separation pay.

ISSUE: Whether or not Petitioner is entitled to his separation pay

HELD: NO. The Court agrees with the CA in its observation of the following circumstances as proof that
respondent did not terminate petitioner's employment: first, the only cause of action in petitioner's original
complaint is that he was offered a very low separation pay; second, there was no allegation of illegal dismissal,
both in petitioner's original and amended complaints and position paper; and, third, there was no prayer for
reinstatement.

In consonance with the above findings, the Court finds that petitioner was the one who initiated the severance of
his employment relations with respondent. It is evident from the various pleadings filed by petitioner that he
never intended to return to his employment with respondent on the ground that his health is failing. Indeed,
petitioner did not ask for reinstatement. In fact, he rejected respondent's offer for him to return to work. This is
tantamount to resignation.

Resignation is defined as the voluntary act of an employee who finds himself in a situation where he believes
that personal reasons cannot be sacrificed in favor of the exigency of the service and he has no other choice but
to disassociate himself from his employment.

There is no provision in the Labor Code which grants separation pay to voluntarily resigning employees. In fact,
the rule is that an employee who voluntarily resigns from employment is not entitled to separation pay, except
when it is stipulated in the employment contract or CBA, or it is sanctioned by established employer practice or
policy. In the present case, neither the abovementioned provisions of the Labor Code and its implementing rules
and regulations nor the exceptions apply because petitioner was not dismissed from his employment and there
is no evidence to show that payment of separation pay is stipulated in his employment contract or sanctioned by
established practice or policy of herein respondent, his employer.

San Beda College of Law 640


4S: 2015 - 2016
LABOR LAW REVIEW Atty. Joyrich Golangco

Since petitioner was not terminated from his employment and, instead, is deemed to have resigned therefrom,
he is not entitled to separation pay under the provisions of the Labor Code.

The foregoing notwithstanding, this Court, in a number of cases, has granted financial assistance to separated
employees as a measure of social and compassionate justice and as an equitable concession. Taking into
consideration the factual circumstances obtaining in the present case, the Court finds that petitioner is entitled to
this kind of assistance.

The Court finds no cogent reason not to employ the same guiding principle of compassionate justice applied by
the Court, taking into consideration the factual circumstances obtaining in the present case. In this regard, the
Court finds credence in petitioner's contention that he is in the employ of respondent for more than 35 years. In
the absence of a substantial refutation on the part of respondent, the Court agrees with the findings of the Labor
Arbiter and the NLRC that respondent company is not distinct from its predecessors but, in fact, merely
continued the operation of the latter under the same owners and the same business venture. The Court further
notes that there is no evidence on record to show that petitioner has any derogatory record during his long years
of service with respondent and that his employment was severed not by reason of any infraction on his part but
because of his failing physical condition. Add to this the willingness of respondent to give him financial
assistance. Hence, based on the foregoing, the Court finds that the award of P50,000.00 to petitioner as
financial assistance is deemed equitable under the circumstances.

San Beda College of Law 641


4S: 2015 - 2016
LABOR LAW REVIEW Atty. Joyrich Golangco

ELSA S. MALIG-ON vs. EQUITABLE GENERAL SERVICES, INC.
G.R. No. 185269 June 29 2010
ABAD, J.:

TERMINATION BY EMPLOYEE

FACTS: Petitioner Elsa Malig-on (Malig-on) claimed that on March 4, 1996 respondent Equitable General
Services, Inc. (the company) hired her as janitress in its janitorial services. The company paid her P250.00 per
day for a nine-hour work. After six years or on February 15, 2002 Malig-ons immediate supervisor told her that
the company would be assigning her to another client. But it never did despite several follow-ups that she made.
Eight months later or on October 15, 2002 the company told Malig-on that she had to file a resignation letter
before it would reassign her. She complied but the company reneged on its undertaking, prompting Malig-on to
file a complaint against it for illegal dismissal.

The company denied Malig-ons allegations. It claimed that she just stopped reporting for work on February 16,
2002 without giving any reason. Consequently, the company wrote her two letters, first on August 23, 2002 and
again on September 2, 2002, asking her to explain her continued absence. On October 15, 2002 Malig-on
showed up at the companys office and submitted her resignation letter.

ISSUE: Whether or not the CA erred in holding that petitioner Malig-on abandoned her work and eventually
resigned from it rather than that respondent company constructively dismissed her.

HELD: YES, Malig-on did not abandon but rather was constructively dismissed by the employer.

According to the company, Malig-on simply dropped out of sight one day on February 16, 2002 for no reason at
all. Eight months later or on October 15, 2002 she appeared at the companys office and tendered her
resignation. To the companys surprise, three days later or on October 18, 2002 she went to the NLRC office
and filed her complaint against the company for illegal dismissal. Clearly, however, these circumstances do not
sound consistent with resignation freely made.

First, when Malig-on reportedly dropped out of sight and the company had no idea about the reason for it, the
natural and right thing for it to do was investigate why she had suddenly vanished. Indeed, the company needed
to write Malig-on immediately and ask her to explain in writing why she should not be considered to have
abandoned her job so the company may be cleared of its responsibility as employer. This did not happen here.

Second, if Malig-on had abandoned her work and had no further interest in it, there was no reason for her to
suddenly show up at her former place of work after eight months and file her resignation letter. Her action would
make sense only if, as she claimed, she had been on floating status for over six months and the company
promised to give her a new assignment if she would go through the process of resigning and reapplying.

And, third, that Malig-on went to the NLRC to file a complaint for unjust dismissal just three days after she filed
her alleged resignation letter is inconsistent with genuine resignation. It would make sense only if, as Malig-on
claims, the company tricked her into filing for resignation upon a promise to give her a new work assignment
and failed to deliver such promise.

The company evidently placed Malig-on on floating status after being relieved as janitress in a clients
workplace. But, as the Court has repeatedly ruled, such act of off-detailing Malig-on was not the equivalent of
dismissal so long as her floating status did not continue beyond a reasonable time. But, when it ran up to more
than six months, the company may be considered to have constructively dismissed her from work, that is, as of
August 16, 2002. Thus, her purported resignation on October 15, 2002 could not have been legally possible.

The company of course claims that it gave Malig-on notices on August 23, 2002 and September 2, 2002, asking
her to explain her failure to report for work and informing her that the company would treat such failure as lack of
interest in it, respectively. But these notices cannot possibly take the place of the notices required by law. They
came more than six months after the company placed her on floating status and, consequently, the company
gave her those notices after it had constructively dismissed her from work.

San Beda College of Law 642


4S: 2015 - 2016
LABOR LAW REVIEW Atty. Joyrich Golangco

CHIANG KAI SHEK COLLEGE and CARMELITA ESPINO vs. ROSALINDA M. TORRES
G.R. No.189456 April 2, 2014
PEREZ, J.:

TERMINATION BY EMPLOYEE

DOCTRINE: There is constructive dismissal when there is cessation of work, because continued
employment is rendered impossible, unreasonable or unlikely, as an offer involving a demotion in rank
or a diminution in pay and other benefits. Aptly called a dismissal in disguise or an act amounting to
dismissal but made to appear as if it were not, constructive dismissal may, likewise, exist if an act of
clear discrimination, insensibility, or disdain by an employer becomes so unbearable on the part of the
employee that it could foreclose any choice by him except to forego his continued employment.

FACTS: Petitioner Chiang Kai Shek College is a private educational institution. Carmelita Espino is the Vice-
President of the school. Respondent had been employed as a grade school teacher of the school from
July 1970 until 31 May 2003.

Respondent was accused of leaking a copy of a special quiz given to Grade 5 students.

Mrs. Koo, who was in charge of the HEKASI area, confronted respondent, who had initially denied
leaking the test paper but later on admitted that she gave the test paper to Mrs. Anduyan, her co-teacher
and the mother of Aileen, the student from whom the copy of the quiz was found. Respondent and Mrs.
Anduyan were both directed to submit their written statement on the incident.

An administrative hearing was conducted wherein respondent and Mrs. Anduyan were asked questions
by the Investigating Committee relative to the leakage of test paper. The Investigating Committee held a
meeting and found respondent and Mrs. Anduyan guilty of committing a grave offense of the school
policies by leaking a special quiz. As shown in the Minutes of the Meeting on 30 August 2002, the
Committee decided to impose the penalty of one-month suspension without pay on respondent and
forfeiture of all the benefits scheduled to be given on Teachers Day. According to petitioners, their
Investigating Committee had actually decided to terminate respondent and had in fact prepared a
memorandum of termination, but respondent allegedly pleaded for a change of punishment in a short
letter.

Petitioners acceded to the request and suspended respondent and Mrs. Anduyan effective 16
September to October 2002. The duo was directed to report to work on 4 November 2002.
Respondent continued her employment from 4 November 2002 until the end of the school year on 26
March 2003. On 14 February 2003 however, respondents counsel sent a letter to petitioners containing
the demands to pay her backwages, bonus and other monetary benefits.

Petitioners, through counsel, wrote to respondents counsel asserting that respondent was being
terminated but the latter requested that she be suspended instead on condition that she will tender
her voluntary resignation at the end of the school year. On 10 June 2003, respondent filed a complaint
for constructive dismissal and illegal suspension with the Labor Arbiter.

In her Position Paper, respondent alleged that she was forced and pressured to submit the written
request for a change of penalty and commitment to resign at the end of the school year. She was
threatened by the school management with immediate dismissal from service if she did not submit the
written statement. She claimed that she was not formally charged with any offense and she was not
served a copy of the notice of the schools decision to terminate her services. Petitioners insisted that
respondent voluntarily resigned. Petitioners averred that respondent was accorded her right to due
process prior to her termination.

LA RULING: NO CONSTRUCTIVE DISMISSAL. Respondent was not coerced nor pressured to write her
resignation letter.

NLRC RULING: AFFIRMED but ordered petitioners to pay respondent separation pay.

CA RULING: REVERSED. Respondent constructively dismissed.


San Beda College of Law 643
4S: 2015 - 2016
LABOR LAW REVIEW Atty. Joyrich Golangco

ISSUE: Was Torres constructively dismissed?

SC RULING:
NO. There is constructive dismissal when there is cessation of work, because continued employment is
rendered impossible, unreasonable or unlikely, as an offer involving a demotion in rank or a diminution
in pay and other benefits. Aptly called a dismissal in disguise or an act amounting to dismissal but
made to appear as if it were not, constructive dismissal may, likewise, exist if an act of clear
discrimination, insensibility, or disdain by an employer becomes so unbearable on the part of the
employee that it could foreclose any choice by him except to forego his continued employment.

There was here no discrimination committed by petitioners. While respondent did not tender her
resignation wholeheartedly, circumstances of her own making did not give her any other option. With
due process, she was found to have committed the grave offense of leaking test questions. Dismissal
from employment was the justified equivalent penalty. Having realized that, she asked for, and was
granted, not just a deferred imposition of, but also an acceptable cover for the penalty.

Respondent should not be rewarded for reneging on her promise to resign at the end of the school
year. Otherwise, employers placed in similar situations would no longer extend compassion to
employees. Compromise agreements, like that in the instant case, which lean towards desired liberality
that favor labor, would be discouraged.
Resignation is the voluntary act of an employee who is in a situation where one believes that personal
reasons cannot be sacrificed for the favor of employment, and opts to leave rather than stay employed.
It is a formal pronouncement or relinquishment of an office, with the intention of relinquishing the office
accompanied by the act of relinquishment. As the intent to relinquish must concur with the overt act of
relinquishment, the acts of the employee before and after the alleged resignation must be considered in
determining whether, he or she, in fact, intended to sever his or her employment.

San Beda College of Law 644


4S: 2015 - 2016
LABOR LAW REVIEW Atty. Joyrich Golangco

VICTORINO OPINALDO vs. NARCISA RAVINA
G.R. No.196573 October 16, 2013
VILLARAMA, JR., J.:

TERMINATION BY EMPLOYEE

DOCTRINE: Abandonment is the deliberate and unjustified refusal of an employee to resume his employment.
To constitute abandonment of work, two elements must concur: (1) the employee must have failed to report for
work or must have been absent without valid or justifiable reason; and, (2) there must have been a clear
intention on the part of the employee to sever the employer-employee relationship manifested by some overt
act.

FACTS: Respondent is the general manager and sole proprietor of St. Louisse Security Agency. Petitioner is
a security guard who had worked for the Agency until his alleged illegal dismissal by respondent on
December 22, 2006. The Agency hired the services of petitioner and detailed him to PAIJR Furniture
Accessories (PAIJR).

The owner of PAIJR submitted a written complaint to respondent stating that Opinaldo was no longer physically
fit to perform his duties and responsibilities as a company guard because of his health condition.

Acceding to PAIJRs request, respondent relieved petitioner from his work. Respondent also required petitioner
to submit a medical certificate to prove that he is physically and mentally fit for work as security guard. On
September 6, 2006, respondent reassigned petitioner to Gomez Construction. After working for a period of two
weeks for Gomez Construction and upon receipt of his salary for services rendered within the said two-week
period, petitioner ceased to report for work. The records show that petitioners post at Gomez Construction was
the last assignment given to him by respondent.

On November 7, 2006, petitioner filed a complaint against respondent for underpayment of salary and
nonpayment of other labor standard benefits. The parties agreed to settle and reached a compromise
agreement. On November 27, 2006, petitioner signed a Quitclaim and Release before the DOLE Regional
Office in Cebu City for the amount of P 5,000. After almost four weeks from the settlement of the case,
petitioner returned to respondents office on December 22, 2006. Petitioner claims that when he asked
respondent to sign an SSS Sickness Notification which he was going to use in order to avail of the discounted
fees for a medical checkup, respondent allegedly refused and informed him that he was no longer an employee
of the Agency. Respondent allegedly told him that when he signed the quitclaim and release form at the
DOLE Regional Office, she already considered him to have quit his employment. Respondent, on the other
hand, counterclaims that she did not illegally dismiss petitioner and that it was a valid exercise of management
prerogative that he was not given any assignment pending the submission of the required medical certificate of
his fitness to work.

On January 26, 2007, petitioner filed a Complaint for Illegal Dismissal with a prayer for the payment of
separation pay in lieu of reinstatement against respondent and the Agency before the NLRC Regional
Arbitration Branch.

LA RULING: ILLEGALLY DISMISSED. Respondent and the Agency to pay petitioner separation pay and back
wages.

NLRC RULING: AFFIRMED LA

CA RULING: REVERSED. It was petitioner himself who failed to report for work and therefore severed his
employment with the Agency. The CA further held that petitioners claims relative to his alleged illegal
dismissal were not substantiated.

ISSUE:
1. Was Opinaldo illegally dismissed?
2. Did Opinaldo abandon his work?

SC RULING:
1. YES. While it is a management prerogative to require petitioner to submit a medical certificate, we hold
San Beda College of Law 645
4S: 2015 - 2016
LABOR LAW REVIEW Atty. Joyrich Golangco

that respondent cannot withhold petitioners employment without observing the principles of due process and
fair play.

It is a basic principle of labor protection in this jurisdiction that a worker cannot be deprived of his job without
satisfying the requirements of due process. Labor is property and the right to make it available is next in
importance to the rights of life and liberty. As enshrined under the Bill of Rights, no person shall be deprived of
life, liberty or property without due process of law. The due process requirement in the deprivation of ones
employment is transcendental that it limits the exercise of the management prerogative of the employer to
control and regulate the affairs of the business. In the case at bar, all that respondent employer needed to
prove was that petitioner employee was notified that his failure to submit the required medical certificate will
result in his lack of work assignment and eventually the termination of his employment as a security guard.
There is no iota of evidence in the records, save for the bare allegations of respondent, that petitioner was
notified of such consequence for nonsubmission.

2. NO. Abandonment is the deliberate and unjustified refusal of an employee to resume his employment.
To constitute abandonment of work, two elements must concur: (1) the employee must have failed to report for
work or must have been absent without valid or justifiable reason; and, (2) there must have been a clear
intention on the part of the employee to sever the employer-employee relationship manifested by some overt
act. None of these elements is present in the case at bar.

It is clear that they prevented petitioners continued employment with them unless the latter presents a medical
certificate that he is physically and mentally fit for work. Moreover, if it was really true that complainant
abandoned his work, then why have not respondents sent him a notice to report back for work? It is evident then
that respondents found an excuse to decline complainants continued stay with them on the pretext that he has
to submit first a medical certificate before he could be allowed to resume employment.

San Beda College of Law 646


4S: 2015 - 2016
LABOR LAW REVIEW Atty. Joyrich Golangco

WILLI HAHN ENTERPRISES and/or WILLI HAHN vs. LILIA R. MAGHUYOP
G.R. No.160348 April 2, 2014
YNARES-SANTIAGO, J.:

TERMINATION BY EMPLOYEE

DOCTRINE: The failure of petitioner to pursue the termination proceedings against respondent and to make her
pay for the shortage incurred did not cast doubt on the voluntary nature of her resignation. A decision to give a
graceful exit to an employee rather than to file an action for redress is perfectly within the discretion of an
employer.

FACTS:
Sometime in 1982, respondent Lilia Maghuyop was hired by petitioner Willi Hahn as nanny of one of his sons. In
1986, she was employed as salesclerk of Willi Hahn Enterprises (Ali Mall, Cubao branch), an authorized dealer
of sporting goods, guns and ammunitions. In 1996, she was promoted as store manager of its branch in Shoe
Mart (SM) Cebu.

On February 25, 1998, petitioner conducted an Inventory Report and discovered that its SM Cebu branch
incurred stock shortages and non-remittances. In the latter part of July 1998, petitioner decided to terminate the
services of respondent, however, before he could do so, the latter tendered her resignation. Believing the good
faith of respondent in resigning, petitioner decided not to file charges against her anymore.

On the other hand, respondent claimed that on July 22, 1998, while she was in SM Cebu branch, she was
approached by Tony Abu and Cesar Araneta who ordered her to close shop and to write a letter to Mr. and Mrs.
Hahn thanking them for the years she had been in their employ and for all the benefits she received from them.
She refused to obey the order, but Tony Abu typed the letter of resignation and asked her to sign the same.
Respondent admitted that she read and affixed her signature on the letter. Thereafter, she was allegedly told to
pack her belongings and to vacate the housing unit provided by the company for her family.

On August 25, 1998, respondent filed a complaint with the NLRC, alleging that she should be awarded
backwages, separation pay, salary for July 16-22, 1998 which was withheld by petitioner, proportionate 13th
month pay, damages and attorneys fees.

LA RULING: NO Illegal dismissal but Ordered petitioner to pay respondent her financial assistance, one week
unpaid wages, and proportionate 13th month pay, or a total sum of P14,727.49, plus ten percent (10%)
attorneys fees as to the last two amounts.

NLRC RULING: denied for lack of merit

CA RULING: REVERSED decision of LA/ NLRC. Petitioner was ordered to pay respondent her unpaid salary
from July 16 to 22, 1998, full back wages (inclusive of basic pay, 13th month pay, allowances and monetary
value of all benefits) computed from July 22, 1998 up to the finality of this decision, separation pay equivalent to
one (1) month pay for every year of service, moral damages of P10,000.00 and 10% attorneys fees.

ISSUE: Is the resignation of Maghuyop voluntary?

SC RULING:
YES. The letter is simple, candid and direct to the point. We find no merit in respondents claim that being a
mere clerk, she did not realize the consequences of her resignation.

The rule that the filing of a complaint for illegal dismissal is inconsistent with resignation, is not applicable to the
instant case. The filing of an illegal dismissal case by respondent was evidently a mere afterthought. It was filed
not because she wanted to return to work but to claim separation pay and backwages.

Respondent had no motive to resign because the charges of dishonesty were not fully substantiated has no
basis. Had the separation of respondent been for dismissal due to loss of trust and confidence, substantial
evidence of the shortages and non-remittances would have been indispensable. Such, is not the case here
considering her voluntary resignation.

San Beda College of Law 647


4S: 2015 - 2016
LABOR LAW REVIEW Atty. Joyrich Golangco

The failure of petitioner to pursue the termination proceedings against respondent and to make her pay for the
shortage incurred did not cast doubt on the voluntary nature of her resignation. A decision to give a graceful exit
to an employee rather than to file an action for redress is perfectly within the discretion of an employer. It is not
uncommon that an employee is permitted to resign to save face after the exposure of her malfeasance. Under
the circumstances, the failure of petitioner to file action against the respondent should be considered as an act
of compassion for one who used to be a trusted employee and a close member of the household.

Having tendered a voluntary resignation was not illegally dismissed.

San Beda College of Law 648


4S: 2015 - 2016
LABOR LAW REVIEW Atty. Joyrich Golangco

SKIPPERS UNITED PACIFIC, INC. and SKIPPERS MARITIME SERVICES, INC., LTD. vs. NATHANIEL
DOZA, NAPOLEON DE GRACIA, ISIDRO L. LATA, and CHARLIE APROSTA
G.R. No. 175558 February 8, 2012
CARPIO, J.:

TERMINATION BY EMPLOYEE

DOCTRINE: Article 285 of the Labor Code recognizes termination by the employee of the employment contract
by serving written notice on the employer at least one (1) month in advance. Given that provision, the law
contemplates the requirement of a written notice of resignation. In the absence of a written resignation, it is safe
to presume that the employer terminated the seafarers.

FACTS: This is a consolidated labor case filed by respondents against local manning agency Skippers United
Pacific, Inc. and its foreign principal Skippers Maritime Services, Inc., Ltd.for unremitted home allotment for the
month of December 1998, salaries for the unexpired portion of their employment contracts, moral damages,
exemplary damages, and attorneys fees.

Respondents were deployed by petitioner to work on board the vessel MV Wisdom Star. Paragraph 2 of all the
employment contracts stated that: The terms and conditions of the Revised Employment Contract Governing the
Employment of All Seafarers approved per Department Order No. 33 and Memorandum Circular No. 55, both
series of 1996 shall be strictly and faithfully observed.

Subsequently, after airing their grievances with the Romanian Seafarers Free Union, respondents were
unceremoniously discharged and immediately repatriated. Respondents, then, filed a complaint for illegal
dismissal.

LA: dismissed the case for lack of merit because the seafarers voluntarily pre-terminated their employment
contracts by demanding for immediate repatriation due to dissatisfaction with the ship.
NLRC: affirmed the LA.
CA: reversed the LA and NLRC and awarded to respondents their unremitted home allotment, three months
salary each representing the unexpired portion of their employment contracts and attorneys fees.

ISSUE: Whether or not the workers dismissal is valid

HELD: NO. For a workers dismissal to be considered valid, it must comply with both procedural and substantive
due process. The legality of the manner of dismissal constitutes procedural due process, while the legality of the
act of dismissal constitutes substantive due process.

Procedural due process in dismissal cases consists of the twin requirements of notice and hearing. The
employer must furnish the employee with two written notices before the termination of employment can be
effected: (1) the first notice apprises the employee of the particular acts or omissions for which his dismissal is
sought; and (2) the second notice informs the employee of the employers decision to dismiss him. Before the
issuance of the second notice, the requirement of a hearing must be complied with by giving the worker an
opportunity to be heard. It is not necessary that an actual hearing be conducted.

Substantive due process, on the other hand, requires that dismissal by the employer be made under a just or
authorized causes.

In this case, there was no written notice furnished to De Gracia, et al. regarding the cause of their dismissal.
Cosmoship furnished a written notice (telex) to Skippers, the local manning agency, claiming that De Gracia, et
al. were repatriated because the latter voluntarily pre-terminated their contracts. This telex was given credibility
and weight by the Labor Arbiter and NLRC in deciding that there was pre-termination of the employment
contract akin to resignation and no illegal dismissal. However, as correctly ruled by the CA, the telex message is
a biased and self-serving document that does not satisfy the requirement of substantial evidence. If, indeed, De
Gracia, et al. voluntarily pre-terminated their contracts, then DeGracia, et al. should have submitted their written
resignations.

Article 285 of the Labor Code recognizes termination by the employee of the employment contract by serving
written notice on the employer at least one (1) month in advance. Given that provision, the law contemplates the
San Beda College of Law 649
4S: 2015 - 2016
LABOR LAW REVIEW Atty. Joyrich Golangco

requirement of a written notice of resignation. In the absence of a written resignation, it is safe to presume that
the employer terminated the seafarers. In addition, the telex message relied upon by the Labor Arbiter and
NLRC bore conflicting dates of 22 January 1998 and 22 January 1999, giving doubt to the veracity and
authenticity of the document. In 22 January 1998, De Gracia, et al. were not even employed yet by the foreign
principal. For these reasons, the dismissal of De Gracia, et al. was illegal.

San Beda College of Law 650


4S: 2015 - 2016
LABOR LAW REVIEW Atty. Joyrich Golangco

JONATHAN V. MORALES vs. HARBOUR CENTRE PORT TERMINAL, INC.
G.R. No. 174208 January 25, 2012
PEREZ, J.:

TERMINATION BY EMPLOYEE

FACTS: On May 16, 2000, petitioner was hired by HCPTI as an Accountant and Acting Finance Officer, with a
monthly salary of P18,000.00. Regularized on November 17, 2000, Morales was promoted to Division Manager
of the Accounting Department. Subsequent to HCPTIs transfer to its new offices at Vitas, Tondo, Manila,
Morales received an inter-office memorandum reassigning him to Operations Cost Accounting, tasked with the
duty of monitoring and evaluating all consumables requests, gears and equipment related to the corporations
operations and of interacting with its sub-contractor, Bulk Fleet Marine Corporation.

On 31 March 2003, Morales wrote Singson, protesting that his reassignment was a clear demotion since the
position to which he was transferred was not even included in HCPTIs plantilla. In response to Morales
grievance that he had been effectively placed on floating status, Singson issued an inter-office memorandum to
the effect that transfer of employees is a management prerogative and that HCPTI had the right and
responsibility to find the perfect balance between the skills and abilities of employees to the needs of the
business. For the whole of the ensuing month Morales was absent from work and/or tardy. Singson issued to
Morales an inter-office memorandum denominated as a First Warning. The memorandum reminded Morales
that, as an employee of HCPTI, he was subject to its rules and regulations and could be disciplinarily dealt with
pursuant to its Code of Conduct. In view of the absences Morales continued to incur, HCPTI issued a Second
Warning dated 6 May 2003 and a Notice to Report for Work and Final Warning.

Morales then filed before the NLRC a complaint for constructive dismissal, moral and exemplary damages as
well as attorneys fees. HCPTI argued that Morales abandoned his employment and was not constructively
dismissed.

LA: Morales reassignment was a valid exercise of HCPTIs management prerogative which cannot be
construed as constructive dismissal absent showing that the same was done in bad faith and resulted in the
diminution of his salary and benefits.

NLRC: reversed the LA. Morales reassignment was a clear demotion despite lack of showing of diminution of
salaries and benefits.

CA: reversed the NLRC.

ISSUE: Whether or not Morales was constructively dismissed

HELD: Yes. In cases of a transfer of an employee, the rule is settled that the employer is charged with the
burden of proving that its conduct and action are for valid and legitimate grounds such as genuine business
necessity and that the transfer is not unreasonable, inconvenient or prejudicial to the employee. If the employer
cannot overcome this burden of proof, the employees transfer shall be tantamount to unlawful constructive
dismissal.

While there was a lack of showing that the transfer or reassignment entailed a diminution of salary and benefits,
one fact that must not be lost sight of was that Morales was already occupying the position of Division Manager.
Concurrently appointed as member of HCPTIs Management Committee (MANCOM), Morales was
subsequently reassigned by HCPTI from managerial accounting to Operations Cost Accounting without any
mention of the position to which he was actually being transferred. That the reassignment was a demotion is,
however, evident from Morales new duties which, far from being managerial in nature, were very simply and
vaguely described as inclusive of monitoring and evaluating all consumables requests, gears and equipments
related to [HCPTIs] operations as well as close interaction with [its] sub-contractor Bulk Fleet Marine
Corporation. Morales demotion is evident from the fact that his reassignment entailed a transfer from a
managerial position to one which was not even included in the corporations plantilla.

Morales did not abandon his employment. As a just and valid ground for dismissal, at any rate, abandonment
requires the deliberate, unjustified refusal of the employee to resume his employment, without any intention of
returning. Since an employee like Morales who takes steps to protest his dismissal cannot logically be said to
San Beda College of Law 651
4S: 2015 - 2016
LABOR LAW REVIEW Atty. Joyrich Golangco

have abandoned his work, it is a settled doctrine that the filing of a complaint for illegal dismissal is inconsistent
with abandonment of employment.

San Beda College of Law 652


4S: 2015 - 2016
LABOR LAW REVIEW Atty. Joyrich Golangco

SHS PERFORATED MATERIALS, INC., WINFRIED HARTMANNSHENN, and HINRICH JOHANN
SCHUMACHER vs. MANUEL F. DIAZ
G.R. No. 185814 October 13, 2010
MENDOZA, J.:

TERMINATION BY EMPLOYEE

FACTS: SHS is a start-up corporation organized and existing under the laws of the Republic of the Philippines
and registered with the PEZA. HARTMANNSHENN is its President, in which capacity he determines the
administration and direction of the day-to-day business affairs of SHS.Schumacher is the treasurer and one of
the Board of Directors.

Respondent was hired by SHS as Manager for Business Development on probationary status from July 18,
2005 to January 18, 2006.

During respondents employment, Hartmannshenn was often abroad and, because of business exigencies, his
instructions to respondent were either sent by electronic mail or relayed through telephone or mobile phone.
When he would be in the Philippines, he and the respondent held meetings. As to respondents work, there was
no close supervision by him.

During meetings with the respondent, Hartmannshenn expressed his dissatisfaction over respondents poor
performance. Respondent allegedly failed to make any concrete business proposal or implement any specific
measure to improve the productivity of the SHS office and plant or deliver sales except for a meagre P2,500.00
for a sample product. In numerous electronic mail messages, respondent acknowledged his poor performance
and offered to resign from the company.Respondent denied sending such messages. Respondent also failed to
communicate with Hartmannshenn upon the latters arrival in the Philippines.

On November 29, 2005, Hartmannshenn instructed Taguiang not to release respondents salary. Later that
afternoon, respondent called and inquired about his salary. Taguiang informed him that it was being withheld
and that he had to immediately communicate with Hartmannshenn. Again, respondent denied having received
such directive.

The next day, respondent served on SHS a demand letter and resignation letter. Respondent then filed a
complaint for illegal dismissal and non payment of his salaries.

LA: Respondent was constructively dismissed because the withholding of his salary was illegal. He had no other
alternative but to resign because he could not be expected to continue working for an employer who withheld
wages without valid cause. The LA also held that respondents probationary employment was deemed
regularized because petitioners failed to conduct a prior evaluation of his performance and to give notice two
days prior to his termination as required by the Probationary Contract of Employment and Article 281 of the
Labor Code. Petitioners contention that they lost trust and confidence in respondent as a managerial employee
was not given credence for lack of notice to explain the supposed loss of trust and confidence and absence of
an evaluation of respondents performance.

NLRC: reversed the LA.

CA: reversed the NLRC.

ISSUE: Whether or not respondent was constructively dismissed

HELD: Yes, he was forced to resign and was thus, constructively dismissed. What made it impossible,
unreasonable or unlikely for respondent to continue working for SHS was the unlawful withholding of his salary.
For said reason, he was forced to resign. Although respondent was a probationary employee, he was still
entitled to security of tenure.

It is worthy to note that in his resignation letter, respondent cited petitioners illegal and unfair labor practice as
his cause for resignation. As correctly noted by the CA, respondent lost no time in submitting his resignation
letter and eventually filing a complaint for illegal dismissal just a few days after his salary was withheld. These
circumstances are inconsistent with voluntary resignation and bolster the finding of constructive dismissal.
San Beda College of Law 653
4S: 2015 - 2016
LABOR LAW REVIEW Atty. Joyrich Golangco

This Court has held that probationary employees who are unjustly dismissed during the probationary period are
entitled to reinstatement and payment of full backwages and other benefits and privileges from the time they
were dismissed up to their actual reinstatement. Respondents reinstatement, however, is no longer feasible as
antagonism has caused a severe strain in their working relationship. Under the doctrine of strained relations, the
payment of separation pay is considered an acceptable alternative to reinstatement when the latter option is no
longer desirable or viable. Therefore, a more equitable disposition would be an award of separation pay
equivalent to at least one month pay, in addition to his full backwages, allowances and other benefits.

San Beda College of Law 654


4S: 2015 - 2016
LABOR LAW REVIEW Atty. Joyrich Golangco

SAN MIGUEL PROPERTIES PHILIPPINES INC. v. GWENDELLYN ROSE S. GUCABAN
G.R. No. 153982, July 18, 2011
PERALTA

TERMINATION BY EMPLOYEE

DOCTRINE: Resignation the formal pronouncement or relinquishment of a position or office is the voluntary act
of an employee who is in a situation where he believes that personal reasons cannot be sacrificed in favor of the
exigency of the service, and he has then no other choice but to disassociate himself from employment. The
intent to relinquish must concur with the overt act of relinquishment; hence, the acts of the employee before and
after the alleged resignation must be considered in determining whether he in fact intended to terminate his
employment. In illegal dismissal cases, fundamental is the rule that when an employer interposes the defense of
resignation, on him necessarily rests the burden to prove that the employee indeed voluntarily resigned.

FACTS: Gwendellyn Rose Gucaban (Gucaban) was well into the tenth year of her career as a licensed civil
engineer when she joined the workforce of petitioner San Miguel Properties Philippines, Inc. (SMPI) in 1991.
Initially engaged as a construction management specialist, she, by her satisfactory performance on the job, was
promoted in 1994 and 1995, respectively, to the position of technical services manager, and then of project
development manager. As project development manager, she also sat as a member of the companys
management committee. She had been in continuous service in the latter capacity until her severance from the
company in February 1998.

On June 26, 1998, she filed a complaint for illegal dismissal alleging that her separation from service was
practically forced upon her by management claiming that on Jan. 27, 1998, she was informed by SMPIs
Preseidnet and CEO Federico Gonzalez that the company was planning to reorganize its manpower in order to
cut on costs and that she must file for resignation or otherwise face termination. 3 days later, the HR Dept.
allegedly furnished her a blank resignation form which she refused to sign. From then on, she had been
hounded by Gonzales to sign and submit her resignation letter.

Gucaban complained of ugly treatment and claimed that she had been kept off from all the meetings of the
management committee. On Feb. 12, 1998, she received an evaluation report signed by Gonzalez showing that
she had been negligent and unsatisfactory in the performance of her duties which she found unfounded and
unfair because the VP for Property Management Manuel Torres, in a subsequent memorandum, had actually
vouched for her competence and efficiency on the job. It was supposedly the extreme humiliation and alienation
that impelled her to submit a signed resignation letter on Feb. 18, 1998.

Gucaban also alleged that she had been merely tricked since SMPI never actualized its reorganization and
streamlining plan but SMPI actually expanded its employee population and made new appointments and
promotions to varous other positions. She felt that she had been dismissed without cause and hence prayed for
reinstatement and payment of backwages.

SMPI argued that it truly encountered a steep market decline in 1997 that necessitated cost-cutting measures
and streamlining of its employee structure which, in turn, would require the abolition of certain job positions;
Gucabans post as project development manager was one of such positions. As a measure of generosity, it
allegedly proposed to Gucaban that she voluntarily resign from office in consideration of a financial package an
offer for which Gucaban was supposedly given the first week of February 1998 to evaluate. Gucaban, however,
did not communicate her acceptance of the offer and, instead, she allegedly conferred with the Human
Resource Department and negotiated to augment her benefits package.

LA: Complaint was dismissed.

NLRC: There was illegal dismissal. MR was denied.

CA: NLRC was affirmed. MR was denied.

Hence, this petition.

ISSUES:
1. Was there illegal dismissal?
San Beda College of Law 655
4S: 2015 - 2016
LABOR LAW REVIEW Atty. Joyrich Golangco

2. Did Gucaban voluntarily resign?

HELD:
(1) Yes. Resignation the formal pronouncement or relinquishment of a position or office is the voluntary act of an
employee who is in a situation where he believes that personal reasons cannot be sacrificed in favor of the
exigency of the service, and he has then no other choice but to disassociate himself from employment. The
intent to relinquish must concur with the overt act of relinquishment; hence, the acts of the employee before and
after the alleged resignation must be considered in determining whether he in fact intended to terminate his
employment. In illegal dismissal cases, fundamental is the rule that when an employer interposes the defense of
resignation, on him necessarily rests the burden to prove that the employee indeed voluntarily resigned. Guided
by these principles, SMPI was unable to discharge this burden.

Shortly prior to and at the time of Gucabans alleged resignation, there was actually no genuine corporate
restructuring plan in place as yet. In other words, although the company might have been suffering from losses
due to market decline as alleged, there was still no concrete plan for a corporate reorganization at the time
Gonzalez presented to Gucaban the seemingly last available alternative options of voluntary resignation and
termination by abolition of her office. Certainly, inasmuch as the necessity of corporate reorganization generally
lies within the exclusive prerogative of management, Gucaban at that point had no facility to ascertain the truth
behind it, and neither was she in a position to question it right then and there. Indeed, she could not have
chosen to file for resignation had SMPI not broached to her the possibility of her being terminated from service
on account of the supposed reorganization.

It is then understandable for Gucaban, considering the attractive financial package which SMPI admittedly
offered to her, to opt for resignation instead of suffer termination a consequence the certainty of which she was
made to believe.

There was no actual reorganization plan in place when Gucaban was induced to resign, and that she had been
excluded from the meetings of the management committee since she refused to sign her resignation letter
followed by the soured treatment that caused her humiliation and alienation, are matters which SMPI has not
directly addressed and successfully refuted.

(2) No. Gucabans separation from the company was the confluence of the fraudulent representation to her that
her office would be declared redundant, coupled with the subsequent alienation which she suffered from the
company by reason of her refusal to tender resignation. The element of voluntariness in her resignation is,
therefore, missing. She had been constructively and, hence, illegally dismissed as indeed her continued
employment is rendered impossible, unreasonable or unlikely under the circumstances.

The petition was denied.

San Beda College of Law 656


4S: 2015 - 2016
LABOR LAW REVIEW Atty. Joyrich Golangco

BMG RECORDS (PHILS.), INC. and JOSE YAP, JR. v. AIDA C. APARECIO and NATIONAL LABOR
RELATIONS COMMISSION
G.R. NO. 153290, September 5, 2007
AZCUNA

TERMINATION BY EMPLOYEE

DOCTRINE: The acceptance by the employer of the employees resignation rendered the same effective. Upon
such acceptance, it may not be unilaterally withdrawn without the consent of petitioners. When the employee
later signified the intention of continuing his or her work, it was already up to the employer to accept the
withdrawal of his or her resignation. The mere fact that the withdrawal was not accepted does not constitute
illegal dismissal, the acceptance of the withdrawal of the resignation being the employer's sole prerogative.

FACTS: BMG Records (Phils) Inc. hried Aida Aparecio as one of the promo girls in its Cenu branch. For working
from Monday to Sunday, she received a salary of P181/day.

On May 25, 1998, Aparecio filed a complaint against BMG and its branch manager Jose Yap, Jr. for illegal
th
dismissal and non-payment of overtime pay, holiday pay, premium pay for rest day, 13 month pay, SIL pay,
and separation pay alleging that:
She was illegally dismissed or terminated from employment on April 30, 1998.
Before said date, she was asked by BMG to resign and will be paid all her benefits due and to execute a
letter of resignation.
In view of BMGs insistence to prepare and execute a letter-resignation, even without proper accounting of
any accountability, Aparecio was lured, induced, and compelled to submit a letter of resignation believing on
BMGs promise and assurance to pay all the benefits due her.
After executing the resignation letter, BMG did not make good its promise and instead did an accounting by
themselves in the absence of Aparecio and arrived on a computation that Aparecios liability per their
accounting reached P8,000.
Since they offered to pay a separation pay of only P12,000 minus the P8,000, they are ready to pay the
balance thereof any time;
Aparecio was under BMGs employ for 7 years, 7 months, and 28 days when illegally terminated from her
employment.

BMG, on the other hand, alleges that:


Aparecio was initially performing well as an employee but as years passed by she seemed to be complacent
in the performance of her job and had been comparing the salaries of promo girls in other companies.
It appeared that Aparecio was no longer interested in her job.
In 1998, Aparecio and two other promogirls Soco and Mutya intimated to their supervisor that they were
intending to resign and were requesting for some financial assistance.BMG made it clear that, as a
company policy, an employee who resigns from service is not enetitled to financial assistance, but
considering the length of their service and due to humanitarian consieration, it would accede to the request
after they secure their respective clearances.
The 3 employees tendered their resignations, which were accepted.
When they processed the required individual clearance, it was found out that they incurred some shortages
after inventory which were then deducted from the amounts due them.
Soco and Mutya received their last salary and executed their releases and quitclaims.
Except for the financial assistance, Aparecio also obtained the same yet refused to sign the release and
quitclaim, protesting P9,170.12 deducted from the financial assistance.

LA: Complaint was dismissed.

NLRC: There was illegal dismissal. MR was denied.

CA: NLRC was affirmed in toto. MR was denied.

Hence, this petiiton.

ISSUE: Was there fraud, undue influence, intimidation, and/or mistake upon Aparecios resignation from BMG,
thereby making BMG and Yap guilty of illegal dismissal?
San Beda College of Law 657
4S: 2015 - 2016
LABOR LAW REVIEW Atty. Joyrich Golangco

HELD: No. Fraud exists only when, through insidious words or machinations, the other party is induced to act
and without which, the latter would not have agreed to. Circumstances evidencing fraud and misrepresentation
are as varied as the people who perpetrate it, each assuming different shapes and forms and may be committed
in as many different ways. Fraud and misrepresentation are, therefore, never presumed; it must be proved by
clear and convincing evidence and not mere preponderance of evidence.

Aparecio alleged that her resignation was wrongfully obtained when BMG and Yap did not keep the promise of
giving her employment benefits and financial assistance without any deductions. Without a showing of the
nature and extent of such "inducement," however, such submission fails to establish that there was in fact a
deception on the part of BMG and Yap. Even if it is considered that there was an assurance given by BMG and
Yap and that they later reneged on their promise, there is no injustice made since Aparecio, who only
questioned the manner by which the inventory was conducted that it was held without her presence but did
not categorically deny her accountabilities with BMG, would unjustly be enriched without the deduction.

Aparecio did not adduce any competent evidence to prove that force or threat was applied by petitioners. For
intimidation to vitiate consent, the following requisites must be present: (1) that the intimidation caused the
consent to be given; (2) that the threatened act be unjust or unlawful; (3) that the threat be real or serious, there
being evident disproportion between the evil and the resistance which all men can offer, leading to the choice of
doing the act which is forced on the person to do as the lesser evil; and (4) that it produces a well-grounded fear
from the fact that the person from whom it comes has the necessary means or ability to inflict the threatened
injury to his person or property. In this case, not one of these essential elements was amply proven by Aparecio.
Bare allegations of threat or force do not constitute substantial evidence to support a finding of forced
resignation.

That there was a "strong and irresistible economic pressure originating from BMG and Yap if only to push
Aparecio into accepting the offer" is not supported by any evidence in the records but is merely based on
conjectures and guesswork. There is no concrete evidence, direct or circumstantial, showing that undue
influence was used by BMG and Yap in such a way that it took improper advantage of its power over the will of
Aparecio and deprived the latter of a reasonable freedom of choice.

Resignation is the voluntary act of an employee who is in a situation where one believes that personal reasons
cannot be sacrificed in favor of the exigency of the service, and one has no other choice but to dissociate
oneself from employment. It is a formal pronouncement or relinquishment of an office, with the intention of
relinquishing the office accompanied by the act of relinquishment. As the intent to relinquish must concur with
the overt act of relinquishment, the acts of the employee before and after the alleged resignation must be
considered in determining whether in fact, he or she intended to sever from his or her employment.

Circumstances surrounding Aparecios resignation showed her intent to resign:


Aparecio already communicated to other people that she was about to resign to look for a better paying job
since she had been complaining that employees like her in other companies were earning much more.
Prior to the submission of her resignation letter, Aparecio and two other promo girls, Soco and Mutya,
approached their supervisor, intimated their desire to resign, and requested that they be given financial
assistance, which petitioners granted on the condition that deductions would be made in case of shortage
after inventory.
Aparecio, Soco, and Mutya submitted their duly signed resignation letters, which were accepted by
petitioners.
Aparecio already initiated the processing of her clearance; thus, she was able to receive her last salary,
13th month pay, and tax refund but refused to receive the financial assistance less the deductions made.

The acceptance by petitioners of Aparecio's resignation rendered the same effective. Upon such acceptance, it
may not be unilaterally withdrawn without the consent of petitioners. When the employee later signified the
intention of continuing his or her work, it was already up to the employer to accept the withdrawal of his or her
resignation. The mere fact that the withdrawal was not accepted does not constitute illegal dismissal, the
acceptance of the withdrawal of the resignation being the employer's sole prerogative.

Once an employee resigns and his resignation is accepted, he no longer has any right to the job. If the
employee later changes his mind, he must ask for approval of the withdrawal of his resignation from his
employer, as if he were re-applying for the job. It will then be up to the employer to determine whether or not his
San Beda College of Law 658
4S: 2015 - 2016
LABOR LAW REVIEW Atty. Joyrich Golangco

service would be continued. If the employer accepts said withdrawal, the employee retains his job. If the
employer does not, the employee cannot claim illegal dismissal for the employer has the right to determine who
his employees will be. To say that an employee who has resigned is illegally dismissed, is to encroach upon the
right of employers to hire persons who will be of service to them.

What transpired here was caused by an employee's error of judgment and not by the employer's application of
means vitiating the consent to resign. It would be utterly unfair to attribute to petitioners the commission of illegal
dismissal and to impose upon them the burden of accepting back Aparecio who unequivocally manifested her
intent and willingness to sever her employment ties.

The petition was granted.

San Beda College of Law 659


4S: 2015 - 2016
LABOR LAW REVIEW Atty. Joyrich Golangco

SKM ART CRAFT CORPORATION v. EFREN BAUCA, ET AL.
G.R. No. 171282, November 27, 2013
VILLARAMA, JR.

ART. 301: WHEN EMPLOYMENT NOT DEEMED TERMINATED

DOCTRINE: Under Article 286 of the Labor Code, the employment will not be deemed terminated if the bona
fide suspension of operations does not exceed six months. But if the suspension of operations exceeds six
months, the employment will be considered terminated.

FACTS: Efren Bauca, et al. (23 total) were employed by SKM Art Craft Corporation which is engaged in the
handicraft business.

On April 18, 2000, at around 1:12 am, a fire occurred at the inspection and receiving/repair/packing area of
SKMs premises in Intramuros, Manila. The fire investigation report stated that the structure and the beach
rubber building were totally damaged. Also burned were four container vans and a trailer truck. The estimated
damage was P22 million.

On May 8, 2000, SKM informed Bauca, et al. that it will suspend its operations for 6 months, effective May 9,
2000.

On May 16. 2000, only 8 days after receiving notice of the suspension of SKMs operations, Bauca filed a
complaint for illegal dismissal alleging that there was discrimination in choosing the workers to be laid off and
that SKM had discovered that most of them were members of a newly-organized union.

SKM denied the claim and said that Art. 286 (old numbering) of the Labor Code allows the bona fide suspension
of a business or undertaking for a period not exceeding 6 months and that the fire cost it millions in losses and
that it is impossible to resume its normal operations for a significant period of time.

LA: There was illegal dimissal.

NLRC: There was no illegal dismissal. MR was denied.

CA: LA decision was reinstated. MR was denied.

Hence, this petition.

ISSUE: Was there illegal dismissal?

HELD: Yes. While SKMs suspension of operations is valid because the fire caused substantial losses to SKM
and damaged its factory, Bauca, et al. were illegally dismissed since they were not recalled 6 months after the
bona fide suspension of SKMs operations.

Under Article 286 of the Labor Code, the bona fide suspension of the operations of a business or undertaking
for a period not exceeding six months shall not terminate employment.

As a general rule, a complaint for illegal dismissal filed prior to the lapse of said six months is generally
considered as prematurely filed.

In this case, however, Bauca, et al. were already considered illegally dismissed since SKM failed to recall them
after 6 months, when its bona fide suspension of operations lapsed. Under Article 286 of the Labor Code, the
employment will not be deemed terminated if the bona fide suspension of operations does not exceed six
months. But if the suspension of operations exceeds six months, the employment will be considered terminated.

Under Article 286 of the Labor Code, the bona fide suspension of the operation of a business or undertaking for
a period not exceeding six months shall not terminate employment. Consequently, when the bona fide
suspension of the operation of a business or undertaking exceeds six months, then the employment of the
employee shall be deemed terminated. By the same token and applying said rule by analogy, if the employee

San Beda College of Law 660


4S: 2015 - 2016
LABOR LAW REVIEW Atty. Joyrich Golangco

was forced to remain without work or assignment for a period exceeding six months, then he is in effect
constructively dismissed.

SKMs manifestatioon that it is willing to admit Bauca, et al. if they return to work was belatedly made, almost
one year after SKMs suspension of operations had already expired. SKM no longer recalled, nor wanted to
recall Bauca, et al. after 6 months.

The petition was denied.

San Beda College of Law 661


4S: 2015 - 2016
LABOR LAW REVIEW Atty. Joyrich Golangco

EMERITUS SECURITY AND MAINTENANCE SYSTEMS, INC v. JANRIE C. DAILIG
G.R. No. 204761 April 2, 2014
CARPIO, J.:

ART. 301: WHEN EMPLOYMENT NOT DEEMED TERMINATED

FACTS: In August 2000, Emeritus Security and Maintenance Systems (Emeritus) hired Janrie Dalig (Dalig) as
one of its security guards. During his employment, Dalig was assigned to Emeritus various clients. On 10
December 2005, he was relieved from his post. On various dates in December 2005 and from January to May
2006, he went to Emeritus office to follow-up his next assignment. After more than six months since his last
assignment, still Dalig was not given a new assignment. Dalig argued that if an employee is on floating status for
more than six months, such employee is deemed illegally dismissed.

On 27 January 2006, respondent filed a complaint for underpayment of wages, non-payment of legal and
special holiday pay, premium pay for rest day and underpayment of ECOLA before the Department of Labor and
Employment, National Capital Region. The hearing officer recommended the dismissal of the complaint since
the claims were already paid.

On 16 June 2006, Dalig filed a complaint for illegal dismissal and payment of separation pay against petitioner
before the Conciliation and Mediation Center of the NLRC. On 14 July 2006, he filed another complaint for
illegal dismissal, underpayment of salaries and non-payment of full backwages before the NLRC.

Petitioners claim:
Petitioner claimed that on 27 January 2006 it sent respondent a notice to his last known address requiring him
to report to the head office within 72 hours from receipt of the said notice. Petitioner further alleged that it had
informed respondent that he had been absent without official leave for the month of January 2006, and that his
failure to report within 72 hours from receipt of the notice would mean that he was no longer interested to
continue his employment.
Petitioner also claimed that there was no showing that respondent was prevented from returning to his work and
that it had consistently manifested its willingness to reinstate him to his former position. In addition, the fact that
there was no termination letter sent to respondent purportedly proved that respondent was not dismissed.

LAs Ruling:
Dalig was ordered to reinstate complainant and to pay him backwages from the time his compensation was
withheld by reason of his illegal dismissal until actual reinstatement. His claim for underpayment is hereby
denied for lack of merit.

CAs Ruling:
The Court of Appeals affirmed the finding of the Labor Arbiter and the NLRC that respondent was illegally
dismissed by petitioner. However, the Court of Appeals set aside the Labor Arbiter and the NLRCs
reinstatement order. Instead, the Court of Appeals ordered the payment of separation pay, invoking the doctrine
of strained relations between the parties.

ISSUES:
1. Whether respondent was illegally dismissed by respondent?
2. Whether respondent is entitled to separation pay, instead of reinstatement?

HELD:
1. YES. Petitioner admits relieving respondent from his post as security guard on 10 December 2005. There is
also no dispute that respondent remained on floating status at the time he filed his complaint for illegal
dismissal on 16 June 2006. In other words, respondent was on floating status from 10 December 2005 to 16
June 2006 or more than six months. Petitioners allegation of sending respondent a notice sometime in
January 2006, requiring him to report for work, is unsubstantiated, and thus, self-serving.

The Court agrees with the ruling of the Labor Arbiter, NLRC and Court of Appeals that a floating status of a
security guard, such as respondent, for more than six months constitutes constructive dismissal. In
Nationwide Security and Allied Services, Inc. v. Valderama, the Court held:

San Beda College of Law 662


4S: 2015 - 2016
LABOR LAW REVIEW Atty. Joyrich Golangco

x x x the temporary inactivity or "floating status" of security guards should continue only for six months.
Otherwise, the security agency concerned could be liable for constructive dismissal. The failure of petitioner
to give respondent a work assignment beyond the reasonable six-month period makes it liable for
constructive dismissal. x x x.

2. NO. Article 279 of the Labor Code of the Philippines mandates the reinstatement of an illegally dismissed
employee, to wit:

Security of Tenure. - x x x An employee who is unjustly dismissed from work shall be entitled to
reinstatement without loss of seniority rights and other privileges and to his full back wages, inclusive of
allowances, and to his other benefits or their monetary equivalent computed from the time his compensation
was withheld from him up to the time of his actual reinstatement.

Thus, reinstatement is the general rule, while the award of separation pay is the exception. The
circumstances warranting the grant of separation pay, in lieu of reinstatement, are laid down by the Court in
Globe-Mackay Cable and Radio Corporation v. National Labor Relations Commission, thus:

Over time, the following reasons have been advanced by the Court for denying reinstatement under the
facts of the case and the law applicable thereto; that reinstatement can no longer be effected in view of the
long passage of time (22 years of litigation) or because of the realities of the situation; or that it would be
inimical to the employers interest; or that reinstatement may no longer be feasible; or, that it will not serve
the best interests of the parties involved; or that the company would be prejudiced by the workers continued
employment; or that it will not serve any prudent purpose as when supervening facts have transpired which
make execution on that score unjust or inequitable or, to an increasing extent, due to the resultant
atmosphere of antipathy and antagonism or strained relations or irretrievable estrangement between the
employer and the employee.

In this case, petitioner claims that it complied with the reinstatement order of the Labor Arbiter.1wphi1 On
23 January 2008, petitioner sent respondent a notice informing him of the Labor Arbiters decision to
reinstate him. Accordingly, in February 2008, respondent was assigned by petitioner to Canlubang Sugar
Estate, Inc. in Canlubang, Laguna, and to various posts thereafter. At the time of the filing of the petition,
respondent was assigned by petitioner to MD Distripark Manila, Inc. in Bian, Laguna.

Respondent admits receiving a reinstatement notice from petitioner. Thereafter, respondent was assigned to
one of petitioner's clients. However, respondent points out that he was not reinstated by petitioner Emeritus
Security and Maintenance Systems, Inc. but was employed by another company, Emme Security and
Maintenance Systems, Inc. (Emme). Thus, according to respondent, he was not reinstated at all.

Petitioner counters that Emeritus and Emme are sister companies with the same Board of Directors and
officers, arguing that Emeritus and Emme are in effect one and the same corporation.

Considering petitioner's undisputed claim that Emeritus and Emme are one and the same, there is no basis
in respondent's allegation that he was not reinstated to his previous employment. Besides, respondent
assails the corporate personalities of Emeritus and Emme only in his Comment filed before this Court.
Further, respondent did not appeal the Labor Arbiter's reinstatement order.

Contrary to the Court of Appeals' ruling, there is nothing in the records showing any strained relations
between the parties to warrant the award of separation pay. There is neither allegation nor proof that such
animosity existed between petitioner and respondent. In fact, petitioner complied with the Labor Arbiter's
reinstatement order.

Considering that (1) petitioner reinstated respondent in compliance with the Labor Arbiter's decision, and (2)
there is no ground, particularly strained relations between the parties, to justify the grant of separation pay,
the Court of Appeals erred in ordering the payment thereof, in lieu of reinstatement.

However, the backwages should be computed from 10 June 2006 when respondent was illegally dismissed
up to the time he was reinstated in February 2008.

San Beda College of Law 663


4S: 2015 - 2016
LABOR LAW REVIEW Atty. Joyrich Golangco

NIPPON HOUSING PHIL. INC. ET. AL v. LEYNES
G.R. No. 177816 August 3, 2011
PEREZ, J.:

ART. 302: RETIREMENT

FACTS:
Nippon Housing Philippines, Inc. (NHPI) hired respondent Maiah Angela Leynes (Leynes) on 26 March 2001 for
the position of Property Manager.

On 6 February 2002, Leynes had a misunderstanding with Engr. Honesto Cantuba (Cantuba), the Building
Engineer assigned at the Project. Aside from instructing the security guards to bar Engr. Cantuba from entry
into the Project and to tell him to report to the NHPIs main office in Makati, Leynes also sent a letter dated 8
February 2002 by telefax to Joel Reyes (Reyes), NHPIs Human Resources Department (HRD) Head, apprising
the latter of said Building Engineers supposed insubordination and disrespectful conduct. Engr. Cantubas in turn
accused Leynes of pride, conceit and poor managerial skills, Hiroshi Takada (Takada), NHPIs Vice President,
went on to issue the 12 February 2002 memorandum, attributing the incident to simple personal differences and
directing Leynes to allow Engr. Cantuba to report back for work.

Disappointed with the foregoing management decision, Leynes submitted to Tadashi Ota, NHPIs President, a
letter dated 12 February 2002, asking for an emergency leave of absence for the supposed purpose of
coordinating with her lawyer regarding her resignation letter. While NHPI offered the Property Manager position
to Engr. Carlos Jose on 13 February 2002 as a consequence Leynes signification of her intention to resign, it
also appears that Leynes sent another letter to Reyes by telefax on the same day, expressing her intention to
return to work on 15 February 2002 and to call off her planned resignation upon the advice of her lawyer. Having
subsequently reported back for work and resumed performance of her assigned functions, Leynes was
constrained to send out a 20 February 2002 written protest regarding the verbal information she supposedly
received from Reyes that a substitute has already been hired for her position. On 22 February 2002, Leynes
was further served by petitioner Yasuhiro Kawata and Noboyushi Hisada, NHPIs Senior Manager and Janitorial
Manager, with a letter and memorandum from Reyes, relieving her from her position and directing her to report
to NHPIs main office while she was on floating status.

Aggrieved, Leynes lost no time in filing against NHPI and its above-named officers the 22 February 2002
complaint for illegal dismissal, unpaid salaries, benefits, damages and attorneys fees docketed before the
arbitral level of NLRC.

LAs Ruling:
Labor Arbiter rendered a decision, finding that NHPIs act of putting Leynes on floating status was equivalent to
termination from employment without just cause and compliance with the twin requirements of notice and
hearing. Likewise finding that NHPIs officers acted with bad faith in effecting Leynes termination.

NLRCs Ruling:
On appeal, the foregoing decision was reversed and set aside in the 30 September 2003 ordering the dismissal
of the complaint for lack of merit.

CAs Ruling:
NLRCs decision was reversed and set aside in the herein assailed 23 November 2006 decision, upon the
following findings and conclusions: (a) absent showing that there was a bona fide suspension of NHPIs
business operations, Leynes relief from her position even though requested by the client was tantamount to a
constructive dismissal; (b) the bad faith of NHPI and its officers is evident from the hiring of Engr. Jose as
Leynes replacement on 13 February 2002 or prior to her being relieved from her position on 22 February 2002;
and, (c) the failure of NHPI and its officers to prove a just cause for Leynes termination, the redundancy of her
services and their compliance with the requirements of due process renders them liable for illegal dismissal.

ISSUE: Whether or Not the CA erred in ruling that the petitioners decision to place respondent on floating
status is tantamount to constructive dismissal is contrary to law and settled jurisprudence?

HELD: YES. Petitioners argue that the CA erred in finding that Leynes was constructively dismissed when she
was placed on floating status prior to her termination from employment on the ground of
San Beda College of Law 664
4S: 2015 - 2016
LABOR LAW REVIEW Atty. Joyrich Golangco

redundancy. Maintaining that the employees right to security of tenure does not give him a vested right thereto
as would deprive the employer of its prerogative to change his assignment or transfer him to where he will be
most useful, petitioners call our attention to the supposed fact that Leynes was unacceptable to BGCC which
had a contractually guaranteed right to ask for her relief. Rather than outrightly terminating Leynes employment
as a consequence of her threats to resign from her position, moreover, petitioners claim that she was validly
placed on floating status pursuant to Article 286 of the Labor Code of the Philippines .

Although the CA correctly found that the record is bereft of any showing that Leynes was unacceptable to BGCC,
the evidence the parties adduced a quo clearly indicates that petitioners were not in bad faith when they placed
the former under floating status. Disgruntled by NHPIs countermanding of her decision to bar Engr. Cantuba from
the Project, Leynes twice signified her intention to resign from her position to Ota on 12 February 2002. Upon
receiving the copy of the memorandum issued for Engr. Cantubas return to work, Leynes inscribed thereon the
following handwritten note addressed to Ota, Good Morning! Im sorry but I would like to report to you my plan of
resigning as your Prop. Manager. Thank You. In her application letter for an immediate emergency leave, Leynes
also distinctly expressed her dissatisfaction over NHPIs resolution of her dispute with Engr. Cantuba and
announced her plan of coordinating with her lawyer regarding her resignation letter.

In view of the sensitive nature of Leynes position and the critical stage of the Projects business development,
NHPI was constrained to relay the situation to BGCC which, in turn, requested the immediate adoption of
remedial measures from Takada, including the appointment of a new Property Manager for the Project. Upon
BGCCs recommendation, NHPI consequently hired Engr. Jose on 13 February 2002 as Leynes replacement.
Far from being the indication of bad faith the CA construed the same to be, these factual antecedents suggest
that NHPIs immediate hiring of Engr. Jose as the new Property Manager for the Project was brought about by
Leynes own rash announcement of her intention to resign from her position. Although she subsequently
changed her mind and sent Reyes a letter by telefax on 13 February 2002 announcing the reconsideration of
her planned resignation and her intention to return to work on 15 February 2002, Leynes evidently had only
herself to blame for precipitately setting in motion the events which led to NHPIs hiring of her own replacement.

Acting on Leynes 20 February 2002 letter protesting against the hiring of her replacement and reiterating her
lack of intention to resign from her position, the record, moreover, shows that NHPI simply placed her on floating
status until such time that another project could be secured for her. Traditionally invoked by security agencies
when guards are temporarily sidelined from duty while waiting to be transferred or assigned to a new post or
client, Article 286 of the Labor Code has been applied to other industries when, as a consequence of the bona
fide suspension of the operation of a business or undertaking, an employer is constrained to put employees on
floating status for a period not exceeding six months. In brushing aside respondents reliance on said provision
to justify the act of putting Leynes on floating status, the CA ruled that no evidence was adduced to show that
there was a bona fide suspension of NHPIs business. What said court clearly overlooked, however, is the fact
that NHPI had belatedly ventured into building management and, with BGCC as its only client in said
undertaking, had no other Property Manager position available to Leynes.

Considering that even labor laws discourage intrusion in the employers judgment concerning the conduct of
their business, courts often decline to interfere in their legitimate business decisions, absent showing of
illegality, bad faith or arbitrariness. Indeed, the right of employees to security of tenure does not give them
vested rights to their positions to the extent of depriving management of its prerogative to change their
assignments or to transfer them. The record shows that Leynes filed the complaint for actual illegal dismissal
from which the case originated on 22 February 2002 or immediately upon being placed on floating status as a
consequence of NHPIs hiring of a new Property Manager for the Project. The rule is settled, however, that "off-
detailing" is not equivalent to dismissal, so long as such status does not continue beyond a reasonable time and
that it is only when such a "floating status" lasts for more than six months that the employee may be considered
to have been constructively dismissed. A complaint for illegal dismissal filed prior to the lapse of said six-month
and/or the actual dismissal of the employee is generally considered as prematurely filed.

Viewed in the light of the foregoing factual antecedents, we find that the CA reversibly erred in holding
petitioners liable for constructively dismissing Leynes from her employment. There is said to be constructive
dismissal when an act of clear discrimination, insensitivity or disdain on the part of the employer has become so
unbearable as to leave an employee with no choice but to forego continued employment. Constructive dismissal
exists where there is cessation of work because continued employment is rendered impossible, unreasonable or
unlikely, as an offer involving a demotion in rank and a diminution in pay. Stated otherwise, it is a dismissal in
disguise or an act amounting to dismissal but made to appear as if it were not. In constructive dismissal cases,
San Beda College of Law 665
4S: 2015 - 2016
LABOR LAW REVIEW Atty. Joyrich Golangco

the employer is, concededly, charged with the burden of proving that its conduct and action or the transfer of an
employee are for valid and legitimate grounds such as genuine business necessity. To our mind, respondents
have more than amply discharged this burden with proof of the circumstances surrounding Engr. Carlos
employment as Property Manager for the Project and the consequent unavailability of a similar position for
Leynes.

San Beda College of Law 666


4S: 2015 - 2016
LABOR LAW REVIEW Atty. Joyrich Golangco

MAYON HOTEL AND RESTAURANT v. ROLANDO ADANA ET AL
G.R. No. 157634 May 16, 2005
PUNO, J.:

ART. 302: RETIREMENT

FACTS:
Mayon Hotel & Restaurant is a single proprietor business registered in the name of petitioner Pacita O.
Po,whose mother, petitioner Josefa Po Lam, manages the establishment.The hotel and restaurant employed
about sixteen (16) employees.

Records show that on various dates starting in 1981, petitioner hotel and restaurant hired the respondents
herein. Due to the expiration and non-renewal of the lease contract for the rented space occupied by the said
hotel and restaurant at Rizal Street, the hotel operations of the business were suspended on March 31, 1997.
The operation of the restaurant was continued in its new location at Elizondo Street, Legazpi City, while waiting
for the construction of a new Mayon Hotel & Restaurant at Pearanda Street, Legazpi City. Only nine (9) of the
sixteen (16) employees continued working in the Mayon Restaurant at its new site.

On various dates of April and May 1997, the 16 employees filed complaints for underpayment of wages and
other money claims against petitioners, including separation pay. Of the sixteen employees, only the following
filed a case for illegal dismissal: respondents Loveres, Llarena, Nicerio, Macandog, Guades, Atractivo and
Broola.

LAs Ruling:
The Labor Arbiter found that there was illegal dismissal, and granted separation pay to respondents Loveres,
Macandog and Llarena. As respondents Guades, Nicerio and Alamares were already 79, 66 and 65 years old
respectively at the time of the dismissal, the Labor Arbiter granted retirement benefits pursuant to Article 287 of
the Labor Code as amended.The Labor Arbiter ruled that respondent Atractivo was not entitled to separation
pay because he had been transferred to work in the restaurant operations in Elizondo Street, but awarded him
damages. Respondents Loveres, Llarena, Nicerio, Macandog and Guades were also awarded damages.

NLRCs Ruling:
The NLRC reversed the Labor Arbiter, finding that no clear act of termination is attendant in the case at bar and
that respondents did not submit any evidence to that effect, but the finding and conclusion of the Labor Arbiter
are merely based on his own surmises and conjectures.

CAs Ruling:
The NLRC was reversed by the CA.

Petitioners Claim:
It is petitioners contention that the CA should have sustained the NLRC finding that none of the above-named
respondents were illegally dismissed, or entitled to separation or retirement pay. According to petitioners, even
the Labor Arbiter and the CA admit that when the illegal dismissal case was filed by respondents on April 1997,
they had as yet no cause of action. Petitioners therefore conclude that the filing by respondents of the illegal
dismissal case was premature and should have been dismissed outright by the Labor Arbiter. Petitioners also
claim that since the validity of respondents dismissal is a factual question, it is not for the reviewing court to
weigh the conflicting evidence.

ISSUE: Were respondents Loveres, Guades, Macandog, Atractivo, Llarena and Nicerio illegally dismissed?

HELD: YES. Whether respondents are still working for petitioners is a factual question. And the records are
unequivocal that since April 1997, when petitioner Mayon Hotel & Restaurant suspended its hotel operations
and transferred its restaurant operations in Elizondo Street, respondents Loveres, Macandog, Llarena, Guades
and Nicerio have not been permitted to work for petitioners. Respondent Alamares, on the other hand, was also
laid-off when the Elizondo Street operations closed, as were all the other respondents. Since then, respondents
have not been permitted to work nor recalled, even after the construction of the new premises at Pearanda
Street and the reopening of the hotel operations with the restaurant in this new site. As stated by the Joint
Decision of the Labor Arbiter on July 2000, or more than three (3) years after the complaint was filed:

San Beda College of Law 667


4S: 2015 - 2016
LABOR LAW REVIEW Atty. Joyrich Golangco

From the records, more than six months had lapsed without [petitioner] having resumed operation of the
hotel. After more than one year from the temporary closure of Mayon Hotel and the temporary transfer to
another site of Mayon Restaurant, the building which [petitioner] Josefa allege[d] w[h]ere the hotel and
restaurant will be transferred has been finally constructed and the same is operated as a hotel with bar and
restaurant nevertheless, none of [respondents] herein who were employed at Mayon Hotel and Restaurant
which was also closed on April 30, 1998 was/were recalled by [petitioner] to continue their services...

Parenthetically, the Labor Arbiter did not grant separation pay to the other respondents as they had not filed an
amended complaint to question the cessation of their employment after the closure of Mayon Hotel &
Restaurant on March 31, 1997.

The above factual finding of the Labor Arbiter was never refuted by petitioners in their appeal with the NLRC. It
confounds us, therefore, how the NLRC could have so cavalierly treated this uncontroverted factual finding by
ruling that respondents have not introduced any evidence to show that they were illegally dismissed, and that
the Labor Arbiters finding was based on conjecture.It was a serious error that the NLRC did not inquire as to
the legality of the cessation of employment. Article 286 of the Labor Code is clear there is termination of
employment when an otherwise bona fide suspension of work exceeds six (6) months.The cessation of
employment for more than six months was patent and the employer has the burden of proving that the
termination was for a just or authorized cause.

Moreover, we are not impressed by any of petitioners attempts to exculpate themselves from the charges. First,
in the proceedings with the Labor Arbiter, they claimed that it could not be illegal dismissal because the lay-off
was merely temporary (and due to the expiration of the lease contract over the old premises of the
hotel). They specifically invoked Article 286 of the Labor Code to argue that the claim for separation pay was
premature and without legal and factual basis.Then, because the Labor Arbiter had ruled that there was already
illegal dismissal when the lay-off had exceeded the six-month period provided for in Article 286, petitioners raise
this novel argument, to wit:

It is the firm but respectful submission of petitioners that reliance on Article 286 of the Labor Code is misplaced,
considering that the reason why private respondents were out of work was not due to the fault of
petitioners. The failure of petitioners to reinstate the private respondents to their former positions should not
likewise be attributable to said petitioners as the private respondents did not submit any evidence to prove their
alleged illegal dismissal. The petitioners cannot discern why they should be made liable to the private
respondents for their failure to be reinstated considering that the fact that they were out of work was not due to
the fault of petitioners but due to circumstances beyond the control of petitioners, which are the termination and
non-renewal of the lease contract over the subject premises. Private respondents, however, argue in their
Comment that petitioners themselves sought the application of Article 286 of the Labor Code in their case in
their Position Paper filed before the Labor Arbiter. In refutation, petitioners humbly submit that even if they
invoke Article 286 of the Labor Code, still the fact remains, and this bears stress and emphasis, that the
temporary suspension of the operations of the establishment arising from the non-renewal of the lease contract
did not result in the termination of employment of private respondents and, therefore, the petitioners cannot be
faulted if said private respondents were out of work, and consequently, they are not entitled to their money
claims against the petitioners.

It is confounding how petitioners have fashioned their arguments. After having admitted, in effect, that
respondents have been laid-off since April 1997, they would have this Court excuse their refusal to reinstate
respondents or grant them separation pay because these same respondents purportedly have not proven the
illegality of their dismissal.

Petitioners arguments reflect their lack of candor and the blatant attempt to use technicalities to muddle the
issues and defeat the lawful claims of their employees. First, petitioners admit that since April 1997, when
hotel operations were suspended due to the termination of the lease of the old premises, respondents Loveres,
Macandog, Llarena, Nicerio and Guadeshave not been permitted to work. Second, even after six months of
what should have been just a temporary lay-off, the same respondents were still not recalled to work. As a
matter of fact, the Labor Arbiter even found that as of the time when he rendered his Joint Decision on July 2000
or more than three (3) years after the supposed temporary lay-off, the employment of all of the respondents
with petitioners had ceased, notwithstanding that the new premises had been completed and the same
operated as a hotel with bar and restaurant. This is clearly dismissal or the permanent severance or complete
separation of the worker from the service on the initiative of the employer regardless of the reasons therefor.
San Beda College of Law 668
4S: 2015 - 2016
LABOR LAW REVIEW Atty. Joyrich Golangco

On this point, we note that the Labor Arbiter and the CA are in accord that at the time of the filing of the
complaint, respondents had no cause of action to file the case for illegal dismissal. According to the CA and the
Labor Arbiter, the lay-off of the respondents was merely temporary, pending construction of the new building
at Pearanda Street.

While the closure of the hotel operations in April of 1997 may have been temporary, we hold that the evidence
on record belie any claim of petitioners that the lay-off of respondents on that same date was merely temporary.
On the contrary, we find substantial evidence that petitioners intended the termination to be permanent. First,
respondents Loveres, Macandog, Llarena, Guades, Nicerio and Alamares filed the complaint for illegal
dismissal immediately after the closure of the hotel operations in Rizal Street, notwithstanding the alleged
temporary nature of the closure of the hotel operations, and petitioners allegations that the employees assigned
to the hotel operations knew about this beforehand. Second, in their position paper submitted to the Labor
Arbiter, petitioners invoked Article 286 of the Labor Code to assert that the employer-employee relationship was
merely suspended, and therefore the claim for separation pay was premature and without legal or factual
basis. But they made no mention of any intent to recall these respondents to work upon completion of
the new premises. Third, the various pleadings on record show that petitioners held respondents, particularly
Loveres, as responsible for mismanagement of the establishment and for abuse of trust and
confidence. Petitioner Josefa Po Lams affidavit on July 21, 1998, for example, squarely blamed respondents,
specifically Loveres, Bumalay and Camigla, for abusing her leniency and causing petitioner Mayon Hotel &
Restaurant to sustain continuous losses until it is closed. She then asserts that respondents are not entitled to
separation pay for they were not terminated and if ever the business ceased to operate it was because of
losses. Again, petitioners make the same allegation in their memorandum on appeal with the NLRC, where they
alleged that three (3) years prior to the expiration of the lease in 1997, the operation of the Hotel had been
sustaining consistent losses, and these were solely attributed to respondents, but most especially due to
Loveress mismanagement and abuse of petitioners trust and confidence. Even the petition filed in this court
made reference to the separation of the respondents due to severe financial losses and reverses, again
imputing it to respondents mismanagement. The vehemence of petitioners accusation of mismanagement
against respondents, especially against Loveres, is inconsistent with the desire to recall them to work. Fourth,
petitioners memorandum on appeal also averred that the case was filed not because of the business being
operated by them or that they were supposedly not receiving benefits from the Labor Code which is true, but
because of the fact that the source of their livelihood, whether legal or immoral, was stopped on March
31, 1997, when the owner of the building terminated the Lease Contract. Fifth, petitioners had inconsistencies in
their pleadings (with the NLRC, CA and with this Court) in referring to the closure, i.e., in the petition filed with
this court, they assert that there is no illegal dismissal because there was only a temporary cessation or
suspension of operations of the hotel and restaurant due to circumstances beyond the control of petitioners, and
that is, the non-renewal of the lease contract.. And yet, in the same petition, they also assert that: (a) the
separation of respondents was due to severe financial losses and reverses leading to the closure of the
business; and (b) petitioner Pacita Po had to close shop and was bankrupt and has no liquidity to put up her
own building to house Mayon Hotel & Restaurant. Sixth, and finally, the uncontroverted finding of the Labor
Arbiter that petitioners terminated all the other respondents, by not employing them when the Hotel and
Restaurant transferred to its new site on Pearanda Street. Indeed, in this same memorandum, petitioners
referred to all respondents as former employees of Mayon Hotel & Restaurant.

These factors may be inconclusive individually, but when taken together, they lead us to conclude that
petitioners really intended to dismiss all respondents and merely used the termination of the lease (on Rizal
Street premises) as a means by which they could terminate their employees.

Moreover, even assuming arguendo that the cessation of employment on April 1997 was merely temporary,
it became dismissal by operation of law when petitioners failed to reinstate respondents after the lapse of six (6)
months, pursuant to Article 286 of the Labor Code.

We are not impressed by petitioners claim that severe business losses justified their failure to reinstate
respondents. The evidence to prove this fact is inconclusive. But more important, serious business losses do
not excuse the employer from complying with the clearance or report required under Article 283 of the Labor
Code and its implementing rules before terminating the employment of its workers.In the absence of justifying
circumstances, the failure of petitioners to observe the procedural requirements set out under Article 284, taints
their actuations with bad faith, especially since they claimed that they have been experiencing losses in the
three years before 1997. To say the least, if it were true that the lay-off was temporary but then serious business
San Beda College of Law 669
4S: 2015 - 2016
LABOR LAW REVIEW Atty. Joyrich Golangco

losses prevented the reinstatement of respondents, then petitioners should have complied with the requirements
of written notice. The requirement of law mandating the giving of notices was intended not only to enable the
employees to look for another employment and therefore ease the impact of the loss of their jobs and the
corresponding income, but more importantly, to give the Department of Labor and Employment (DOLE) the
opportunity to ascertain the verity of the alleged authorized cause of termination.

And even assuming that the closure was due to a reason beyond the control of the employer, it still has to
accord its employees some relief in the form of severance pay.

While we recognize the right of the employer to terminate the services of an employee for a just or authorized
cause, the dismissal of employees must be made within the parameters of law and pursuant to the tenets of fair
play. And in termination disputes, the burden of proof is always on the employer to prove that the dismissal was
for a just or authorized cause.Where there is no showing of a clear, valid and legal cause for termination of
employment, the law considers the case a matter of illegal dismissal.

Under these circumstances, the award of damages was proper. As a rule, moral damages are recoverable
where the dismissal of the employee was attended by bad faith or fraud or constituted an act oppressive to
labor, or was done in a manner contrary to morals, good customs or public policy. We believe that the dismissal
of the respondents was attended with bad faith and meant to evade the lawful obligations imposed upon an
employer.

To rule otherwise would lead to the anomaly of respondents being terminated from employment in 1997 as a
matter of fact, but without legal redress. This runs counter to notions of fair play, substantial justice and the
constitutional mandate that labor rights should be respected. If doubts exist between the evidence presented
by the employer and the employee, the scales of justice must be tilted in favor of the latter the employer must
affirmatively show rationally adequate evidence that the dismissal was for a justifiable cause.It is a time-honored
rule that in controversies between a laborer and his master, doubts reasonably arising from the evidence, or in
the interpretation of agreements and writing should be resolved in the formers favor. The policy is to extend the
doctrine to a greater number of employees who can avail of the benefits under the law, which is in consonance
with the avowed policy of the State to give maximum aid and protection of labor.

San Beda College of Law 670


4S: 2015 - 2016
LABOR LAW REVIEW Atty. Joyrich Golangco

JEROME M. DAABAY v. COCA-COLA BOTTLERS PHILS., INC.
G.R. No. 199890 August 19, 2013
REYES, J.:

ART. 302: RETIREMENT - Retirement Benefits to a Legally Dismissed Employee (Serious Misconduct)

DOCTRINE:
Financial assistance, or whatever name it is called (such as retirement benefits), as a measure of social justice
is allowed only in instances where the employee is validly dismissed for causes other than serious misconduct
or those reflecting on his moral character.

FACTS:
The case stems from a complaint for illegal dismissal, illegal suspension, unfair labor practice and monetary
claims filed by Daabay against respondent Coca-Cola Bottlers Phils., Inc. and three officers of the company.
The records indicate that the employment of Daabay with Coca-Cola as Sales Logistics Checker was terminated
by the company following receipt of information from one Cesar Sorin that Daabay was part of a conspiracy that
allowed the pilferage of company property. A formal investigation on the matter ensued. Eventually, Coca-Cola
served upon Daabay a Notice of Termination that cited pilferage, serious misconduct and loss of trust and
confidence as grounds.

LA RULING:
Daabay was illegally dismissed because his participation in the alleged conspiracy was not proved by
substantial evidence. In lieu of reinstatement and considering the already strained relations between the parties,
ELA Magbanua ordered the payment to Daabay of backwages and separation pay or retirement benefits, as
may be applicable

NLRC RULING:
The NLRC reversed the finding of illegal dismissal. Notwithstanding its ruling on the legality of the dismissal, the
NLRC awarded retirement benefits in favor of Daabay. The NLRC explained that there was a need to humanize
the severe effects of dismissal and tilt the scales of justice in favor of labor as a measure of equity and
compassionate social justice.

CA RULING:
The CA agreed with Coca-Cola that the award of retirement benefits lacked basis considering that Daabay was
dismissed for just cause.

ISSUE: Should retirement benefits be awarded to a legally dismissed employee based on serious misconduct?

SC RULING:
NO, retirement benefits should not be awarded to a legally dismissed employee based on serious
misconduct.

Being intended as a mere measure of equity and social justice, the NLRCs award was then akin to a financial
assistance or separation pay that is granted to a dismissed employee notwithstanding the legality of his
dismissal. Jurisprudence on such financial assistance and separation pay then equally apply to this case. The
Court has ruled, time and again, that financial assistance, or whatever name it is called, as a measure of social
justice is allowed only in instances where the employee is validly dismissed for causes other than serious
misconduct or those reflecting on his moral character. We explained in Philippine Long Distance Telephone
Company v. NLRC:
[S]eparation pay shall be allowed as a measure of social justice only in those instances
where the employee is validly dismissed for causes other than serious misconduct or
those reflecting on his moral character. Where the reason for the valid dismissal is, for
example, habitual intoxication or an offense involving moral turpitude, like theft or illicit
sexual relations with a fellow worker, the employer may not be required to give the
dismissed employee separation pay, or financial assistance, or whatever other
name it is called, on the ground of social justice.

Clearly, considering that Daabay was dismissed on the grounds of serious misconduct, breach of trust and loss
of confidence, the award based on equity was unwarranted.
San Beda College of Law 671
4S: 2015 - 2016
LABOR LAW REVIEW Atty. Joyrich Golangco

ROSENDO PIERO, DUMAGUETE CATHEDRAL COLLEGE FACULTY AND STAFF ASSOCIATION
(DUCACOFSA) and NATIONAL FEDERATION OF TEACHERS AND EMPLOYEES UNION (NAFTEU) v.
NATIONAL LABOR RELATIONS COMMISSION, FOURTH DIVISION, CEBU CITY and DUMAGUETE
CATHEDRAL COLLEGE, INC.
G.R. No. 149610 August 20, 2004
YNARES-SANTIAGO, J.

ART. 302: RETIREMENT - Retirement Benefits to a Legally Dismissed Employee (Illegal Strike)

DOCTRINE:
An employee who is dismissed for cause is generally not entitled to any financial assistance. Equity
considerations, however, provide an exception such as the long years of service (24 years in this case and 2
years short for Piero to retire).

FACTS:
Upon the expiration of the CBA between petitioner and respondents, the parties failed to conclude another CBA
which led petitioner to conduct a strike in the premises of private respondent without submitting to the DOLE the
required results of the strike vote obtained from the members of the union. Private respondent filed with the
DOLE a complaint to declare the strike illegal and to dismiss the officers of petitioner union.

LA RULING:
The LA declared the strike illegal and declared the petitioner union officers to have lost their employment status
effective on the date of this decision.

NLRC and CA RULING:


The NLRC and CA both affirmed the decision of LA finding the strike illegal.

ISSUE: Should retirement benefits be awarded to a legally dismissed employee and a union officer based on
the conduct of illegal strike?

SC RULING:
YES, retirement benefits should be awarded to a legally dismissed employee/union officer based on the
conduct of illegal strike.

The Court notes that petitioner Piero turned 60 years old and retired on March 1, 1996 (2 years from the day of
decision of the NLRC) after 29 years of service, rendering his dismissal from service moot and academic.
However, in view of the propriety of his termination as a consequence of the illegal strike, he is no longer
entitled to payment of retirement benefits because he lost his employment status effective as of the date of the
decision of the Labor Arbiter.

An employee who is dismissed for cause is generally not entitled to any financial assistance. Equity
considerations, however, provide an exception. Although meriting termination of employment, Pieros infraction
is not so reprehensible nor unscrupulous as to warrant complete disregard of his long years of service.
Moreover, he has no previous derogatory records. Weighed on the scales of justice, conscience and reason tip
in favor of granting financial assistance to support him in the twilight of his life after long years of service.

Under the circumstances, social and compassionate justice dictate that petitioner Piero be awarded financial
assistance equivalent to one-half (1/2) months pay for every year of service computed from his date of
employment up to the date when he was declared to have lost his employment status. Indeed, equities of this
case should be accorded due weight because labor law determinations are not only secundum rationem but
also secundum caritatem.

San Beda College of Law 672


4S: 2015 - 2016
LABOR LAW REVIEW Atty. Joyrich Golangco

STA. CATALINA COLLEGE and SR. LORETA ORANZA vs. NATIONAL LABOR RELATIONS COMMISSION
and HILARIA G. TERCERO
G.R. No. 144483 November 19, 2003
CARPIO MORALES, J.:

ART, 302: RETIREMENT BENEFITS in relation to RE-EMPLOYMENT and GRATUITY PAY

DOCTRINE:
An employees re-employment as a new employee would mean a demotion in rank and privileges, and as to the
computation of retirement benefits, his entire previous years of service would simply be considered as non-
existent.

Gratuity pay is separate and distinct from retirement benefits. Hence, the former should not be deducted to the
latter.

FACTS:
In 1955, Hilaria was hired as an elementary school teacher at the Sta. Catalina College. In 1970, she applied for
and was granted a one year leave of absence without pay on account of the illness of her mother. After the
expiration in 1971 of her leave of absence, she had not been heard from by petitioner school. In the meantime,
she was employed as a teacher in another school for the years 1980-1982.

In 1982, she applied anew at petitioner school which hired her with a monthly salary of P6,567.95. During the
51st Commencement Exercises of petitioner school, Hilaria was awarded a Plaque of Appreciation for thirty
years of service and P12,000.00 as gratuity pay.

On May 31, 1997, Hilaria reached the compulsory retirement age of 65. Retiring pursuant to Article 287 of the
Labor Code, as amended by Republic Act 7641, petitioner school pegged her retirement benefits computed on
the basis of fifteen years of service from 1982 to 1997. Her service from 1955 to 1970 was excluded in the
computation, petitioner school having asserted that she had, in 1971, abandoned her employment. From the
retirement benefits was deducted the amount representing reimbursement of the employers contribution to her
retirement benefits under the Private Education Retirement Annuity Association (PERAA) which Hilaria had
already received. Deducted too was the amount of P12,000.00 representing the gratuity pay which was given to
her.

Hilaria insisted, however, that her retirement benefits should be computed on the basis of her thirty years of
service, inclusive of the period from 1955 to 1970; and that the gratuity pay earlier given to her should not be
deducted therefrom.

LA RULING:
The LA ruled in favor of the petitioner and applied petitioners computation.

NLRC RULING:
The NLRC set aside the LAs decision and ordered respondent-appellee to pay the total amount of P85,287.72
computed as follows: P3,935.89 (total computation of the retirement components) MULTIPLIED by 29 (number
of years in service) EQUALS P114,140.81 (total retirement package) LESS P28,287.72 (representing
respondent-appellees contribution with the PERAA proven to have already been received by complainant-
appellant). However, the gratuity pay earlier already given shall not be deducted from the retirement package.

CA RULING:
The CA affirmed the decision of the NLRC awarding retirement benefits in the amount of P85,287.72 to private
respondent Hilaria.

ISSUES:
1. Should retirement pay of Hilaria include the years of 1955 to 1982, the years she did not work for
petitioner?
2. Should the gratuity pay already received by Hilaria be deducted from her retirement pay?

SC RULING:
1. NO, retirement benefits should not include the years of 1955 to 1982, the years she did not work for
San Beda College of Law 673
4S: 2015 - 2016
LABOR LAW REVIEW Atty. Joyrich Golangco

petitioner.

Hilaria cannot be credited for her services in 1955-1970 in the determination of her retirement benefits. For, after
her one year leave of absence expired in 1971 without her requesting for extension thereof as in fact she had
not been heard from until she resurfaced in 1982 when she reapplied with petitioner school, she abandoned her
teaching position as in fact she was employed elsewhere in the interim and effectively relinquished the
retirement benefits accumulated during the said period. Abandonment of work being a just cause for terminating
the services of Hilaria, petitioner school was under no obligation to serve a written notice to her.

That Hilaria was in 1997 given a plaque of appreciation for thirty years of service to the school and awarded
P12,000.00 as gratuity pay should not be taken against petitioners, for acknowledgment of the total number of
years of her service, which was discontinuous, should not obliterate the fact that she abandoned her
employment in 1971, albeit she was rehired in 1982.

As Hilaria was considered a new employee when she rejoined petitioner school upon re-applying in 1982, her
retirement benefits should thus be computed only on the basis of her years of service from 1982 to 1997. This is
what JAM Transportation Co., Inc. v. Flores teaches:

Private respondents re-employment as a new employee x x x would mean a demotion in rank and
privileges, retirement benefits, for example, as his entire previous eighteen (18) years of service
with petitioner, would simply be considered as non-existent.

2. No, the gratuity pay already received by Hilaria should not be deducted from her retirement pay.

Gratuity pay is separate and distinct from retirement benefits. It is paid purely out of generosity. Hence, should
not be deducted in the retirement benefits of Hilaria. So Republic Planters Bank v. NLRC holds:

Gratuity pay x x x is paid to the beneficiary for the past services or favor rendered purely out of
the generosity of the giver or grantor. Gratuity, therefore, is not intended to pay a worker for
actual services rendered or for actual performance. It is a money benefit or bounty given to the
worker, the purpose of which is to reward employees who have rendered satisfactory service to
the company.

Retirement benefits, on the other hand, are intended to help the employee enjoy the remaining
years of his life, releasing him from the burden of worrying for his financial support, and are a
form of reward for his loyalty to the employer.

San Beda College of Law 674


4S: 2015 - 2016
LABOR LAW REVIEW Atty. Joyrich Golangco

PANTRANCO NORTH EXPRESS, INC v. NLRC and URBANO SUIGA
G.R. No. 95940 July 24, 1996
PANGANIBAN, J.:

ISSUES AS TO THE COMPULSORY RETIREMENT SCHEME RAISED BY A TERMINATED EMPLOYEE IS


A TERMINATION DISPUTE THUS, LA HAS JURISDICTION; EARLY RETIREMENT SCHEME DOES NOT
CONSTITUTE DIMINUTION OF BENEFITS

DOCTRINE: Article 287, as amended by the Retirement Pay Law, provides that an employee may be retired
upon reaching the retirement age established in the collective bargaining agreement or other applicable
employment contract. This provision makes clear the intention of spirit of the law to give employers and
employees a free hand to determine and agree upon the terms and conditions of retirement. Providing in a CBA
for compulsory retirement of employees after twenty-five (25) years of service is legal and enforceable so long
as the parties agree to be governed by such CBA. The law presumes that employees know what they want and
what is good for them absent any showing that fraud or intimidation was employed to secure their consent
thereto.

FACTS:
Suiga was hired by Pantranco in 1964 as a bus conductor. He eventually joined the Pantranco
Employees Association-PTGWO. He continued the Pantrancos employ until August 12, 1989, when he was
retired at the age of fifty-two (52) after having rendered 25 years of service. The basis of his retirement was the
compulsory retirement provision of the collective bargaining agreement between the Pantranco and the
aforenamed union. Suiga received P49,300.00 as retirement pay.

On February 15, 1990, Suiga filed a complaint for illegal dismissal against Pantranco with the LA The
complaint was consolidated with two other cases of illegal dismissal having similar facts and issues, filed by the
other employees, non-union members.

Pantranco contends that the LA had no jurisdiction because the dispute concerns a provision of the
CBA and its interpretation.

LA RULING: Suiga was ILLEGALLY DISMISSED; Reinstated; Money received shall be considered as
an advance to his retirement pay
The LA has jurisdiction. The interpretation of the CBA or enforcement of the company policy is only
corollary to the complaint of illegal dismissal. Otherwise, an employee who was on AWOL, or who committed
offenses contrary to the personnel policies can no longer file a case of illegal dismissal because the discharge is
premised on the interpretation or enforcement of the company policies.
Suiga is entitled to:
P 27,375.00 Backwages, Aug. 16/89 to March 31/90 (P3,650.00 x 7.5 mos.)
1,368.75 13th month pay for 1989 (P16,425.00 over 12)

P 28,743.75
2,874.37 10% attorney's fees

P 31,618.12 Total as of March 31/90 plus additional backwages and other benefits but not to exceed
3 years
and the corresponding attorney's fees.

The amounts already received by complainants shall be considered as advanced payment of their
retirement pay which shall be deducted when they shall actually retire or (be) separated from the service.

NLRC RULING: AFFIRMED the decision of the LA in toto

ISSUES:
(a) Is a Collective Bargaining Agreement provision allowing compulsory retirement before age 60 but
after 25 years of service legal and enforceable?
(b) Who has jurisdiction over a case involving such a question the labor arbiter or arbitrators
authorized by such CBA?
SC RULING:
San Beda College of Law 675
4S: 2015 - 2016
LABOR LAW REVIEW Atty. Joyrich Golangco

(a) YES. Art. 287 of the Labor Code as worded permits employers and employees to fix the applicable
retirement age at below 60 years. Moreover, providing for early retirement does not constitute diminution of
benefits.

Further, a CBA incorporates the agreement reached after negotiations between employer and
bargaining agent with respect to terms and conditions of employment. A CBA is not an ordinary contract. It is
impressed with public interest which must be construed liberally rather than narrowly and technically, and the
courts must place a practical and realistic construction upon it, giving due consideration to the context in which it
is negotiated and purpose which it is intended to serve.

Being a product of negotiation, the CBA between Pantranco and the union intended the provision on
compulsory retirement to be beneficial to the employees-union members, including Suiga. When Suiga
ratified the CBA with the union, he not only agreed to the CBA but also agreed to conform to and abide by its
provisions. Thus, it cannot be said that he was illegally dismissed when the CBA provision on compulsory
retirement was applied to his case.

(b) The LA has jurisdiction over the matter


It cannot be said that the "dispute" is between the union and Pantranco because both have previously
agreed upon the provision on "compulsory retirement" as embodied in the CBA. Also, it was only Suiga on his
own who questioned the compulsory retirement. Thus, the case is properly denominated as a "termination
dispute" which comes under the jurisdiction of labor arbiters.

San Beda College of Law 676


4S: 2015 - 2016
LABOR LAW REVIEW Atty. Joyrich Golangco

R&E TRANSPORT, INC., and HONORIO ENRIQUEZ v. AVELINA P. LATAG, representing her deceased
husband, PEDRO M. LATAG
G.R. No. 155214 February 13, 2004
PANGANIBAN, J.:

RETIREMENT BENEFITS - VALIDITY OF QUITCALIM

DOCTRINE: Courts have stepped in to annul questionable transactions, especially where there is clear proof
that a waiver, for instance, was wangled from an unsuspecting or a gullible person; or where the agreement or
settlement was "unconscionable on its face." A quitclaim is ineffective in barring recovery of the full measure of
a workers rights, and the acceptance of benefits therefrom does not amount to estoppel. Moreover, a quitclaim
in which the consideration is "scandalously low and inequitable" cannot be an obstacle to the pursuit of a
workers legitimate claim.

FACTS:
Latag was a regular employee of La Mallorca Taxi since March 1, 1961. When La Mallorca ceased from
business operations, Latag transferred to R&E Transport, Inc. He was receiving an average daily salary of
P500.00 as a taxi driver.

Latag got sick in January 1995 and was forced to apply for partial disability with the SSS, which was
granted. When he recovered, he reported for work in September 1998 but was no longer allowed to continue
working on account of his old age.

Latag thus asked Felix Fabros, the administrative officer of R&E for his retirement pay pursuant to
Republic Act 7641 but he was ignored. Thus, on December 21, 1998, Latag filed a case for payment of his
retirement pay before the NLRC. Latag however died on April 30, 1999. Subsequently, his wife substituted him.
R&E contends that the 23 years of employment of Latag with La Mallorca Taxi should not be added to
his 14 years with R & E Transport, Inc.

LA RULING: Ordered R&E to PAY; Added years of service of Latag with La Mallorca and R&E
The LA ordered La Mallorca Taxi, R&E and their owner/chief executive officer Honorio Enriquez to pay
P277,500.00 to jointly and severally pay Mrs. Latag by way of retirement pay. Som
On January 21, 2000, Mrs. Latag with her then counsel, was invited to the office of R&Es counsel and
was offered the amount of P38,500.00, which she accepted. Mrs. Latag was also asked to sign an already
prepared quitclaim and release and a joint motion to dismiss the case, which were later on filed with the LA.

NLRC RULING: DISMISSED for failure to post bond

CA RULING: REFERRED R&E to the decision of the LA


Since R&E failed to post an appeal bond, the decision of the LA has become final and executory.

ISSUES:
(a) Should the years of service of Latag with La Mallorca be added with his years of service with R&E for
purposes of his retirement benefits?
(b) Is the quitclaim executed by Mrs. Latag valid?

SC RULING:
(a) NO. No evidence was presented to show La Mallorca is an alter ego of R&E. Thus, the basis for computing
Latags retirement pay should only be 14 years.
(b) NO. It is accepted that taxi drivers do not receive fixed wages, but retain only those sums in excess of the
"boundary" or fee they pay to the owners or operators of their vehicles. Thus, the basis for computing their
benefits should be the average daily income. In this case, the CA found that Pedro was earning an average of
five hundred pesos (P500) per day. We thus compute his retirement pay as follows: P500 x 15 days x 14 years
of service equals P105,000. Compared with this amount, the P38,850 he received, which represented just over
one third of what was legally due him, was unconscionable.

San Beda College of Law 677


4S: 2015 - 2016
LABOR LAW REVIEW Atty. Joyrich Golangco

RODOLFO J. SERRANO v. SEVERINO SANTOS TRANSIT and/or SEVERINO SANTOS
G.R. No. 187698 August 9, 2010
CARPIO MORALES, J.:

AS REGULAR EMPLOYEES AND NOT FIELD PERSONNEL, RETIREMENT BENEFITS OF BUS


TH
CONDUCTORS INCLUDE SIL AND 13 MONTH PAY

DOCTRINES: As a rule, the Retirement Pay Law, which amended Article 287 of the Labor Code shall apply to
all employees in the private sector, regardless of their position, designation or status and irrespective of the
method by which their wages are paid.
Exceptions:
1) Employees of the National Government and its political subdivisions, including Government-
owned and/or controlled corporations, if they are covered by the Civil Service Law and its
regulations.
2) Domestic helpers and persons in the personal service of another.
3) Employees of retail, service and agricultural establishment or operations regularly employing
not more than ten (10) employees. As used in this sub-section
xxxx
SECTION 5 Retirement Benefits.
5.1 In the absence of an applicable agreement or retirement plan, an employee who retires pursuant to the
Act shall be entitled to retirement pay equivalent to at least one-half () month salary for every year of service,
a fraction of at least six (6) months being considered as one whole year.

5.2 Components of One-half () Month Salary. For the purpose of determining the minimum retirement
pay due an employee under this Rule, the term "one-half month salary" shall include all of the following:
(a) Fifteen (15) days salary of the employee based on his latest salary rate. As used herein, the term
"salary" includes all remunerations paid by an employer to his employees for services rendered during
normal working days and hours, whether such payments are fixed or ascertained on a time, task, piece
of commission basis, or other method of calculating the same, and includes the fair and reasonable
value, as determined by the Secretary of Labor and Employment, of food, lodging or other facilities
customarily furnished by the employer to his employees. The term does not include cost of living
allowances, profit- sharing payments and other monetary benefits which are not considered as part of or
integrated into the regular salary of the employees.
(b) The cash equivalent of not more than five (5) days of service incentive leave;
(c) One-twelfth of the 13th month pay due the employee.
(d) All other benefits that the employer and employee may agree upon that should be included in the
computation of the employees retirement pay.

FACTS:
Serrano was hired by SST as a bus conductor. After 14 years of service, Serrano applied for optional
retirement from the company whose representative advised him that he must first sign the already prepared
Quitclaim before his retirement pay could be released. As Serranos request to first go over the computation of
his retirement pay was denied, he signed the Quitclaim on which he wrote "U.P." (under protest) after his
signature, indicating his protest to the amount of P75,277.45 which he received, computed by the company at
15 days per year of service.

Serrano soon after filed a complaint before the Labor Arbiter, alleging that SST erred in its computation
since under Republic Act No. 7641, otherwise known as the Retirement Pay Law, his retirement pay should
have been computed at 22.5 days per year of service to include the cash equivalent of the 5-day service
incentive leave (SIL) and 1/12 of the 13th month pay which the company did not.

SST maintained, however, that the Quitclaim signed by Serrano barred his claim and, in any event, its
computation was correct since Serrano was not entitled to the 5-day SIL and pro-rated 13th month pay for, as a
bus conductor, he was paid on commission basis.

LA RULING: SIL and pro-rated 13th month pay are INCLUDED


The LA ruled in favor of Serrano, awarding him P116,135.45 as retirement pay differential, and 10% of
the total monetary award as attorneys fees.
San Beda College of Law 678
4S: 2015 - 2016
LABOR LAW REVIEW Atty. Joyrich Golangco

In arriving at such computation, the Labor Arbiter ratiocinated that the law is explicit that "one-half month
salary shall mean fifteen (15) days plus one-twelfth (1/12) of the 13th month pay and the cash equivalent of not
more than five (5) days service incentive leaves" unless the parties provide for broader inclusions. Thus, 22.5
days pay per year of service is the correct formula in arriving at the complete retirement pay of Serrano, the
salary being that fair and rational amount of his daily earning.

NLRC RULING: REVERSED the decision of the LA


Since Serrano was paid on purely commission basis, he is excluded from the coverage of the laws on
13th month pay and SIL pay, hence, the 1/12 of the 13th month pay and the 5-day SIL should not be factored in
the computation of his retirement pay.

ISSUE:
th
Are SIL and half of the 13 month pay included in computing for Serranos retirement benefits?

SC RULING:
YES. Admittedly, Serrano worked for 14 years for the bus company, which did not adopt any retirement
scheme. Even if Serrano as bus conductor was paid on commission basis then, he falls within the coverage of
R.A. 7641 and its implementing rules. As thus correctly ruled by the LA, Serranos retirement pay should
include the cash equivalent of the 5-day SIL and 1/12 of the 13th month pay.

It bears emphasis that under the SIL Law, the exclusion from its coverage of workers who are paid on a purely
commission basis is only with respect to field personnel. The more recent case of Auto Bus Transport Systems,
Inc., v. Bautista clarifies that an employee who is paid on purely commission basis is entitled to SIL.

San Beda College of Law 679


4S: 2015 - 2016
LABOR LAW REVIEW Atty. Joyrich Golangco

OBUSAN v. PHILIPPINE NATIONAL BANK
G.R. No. 181178 July 26, 2010
NACHURA, J.:

DOCTRINE:
Article 287 of the Labor Code, as amended, applies only to a situation where (1) there is no CBA or other
applicable employment contract providing for retirement benefits for an employee; or (2) there is a collective
bargaining agreement or other applicable employment contract providing for retirement benefits for an
employee, but it is below the requirement set by law.

FACTS:
Back in 1979, respondent Philippine National Bank (PNB) hired petitioner Amelia R. Obusan, who eventually
became the Manager of the PNB Medical Office. At that time, PNB was a government-owned or controlled
corporation, whose retirement program for its employees was administered by the GSIS, pursuant to the
Revised Government Service Insurance Act of 1977 (PD No. 1146).

On May 27, 1996, PNB was privatized pursuant to the Revised Charter of the PNB (EO No. 80).

Consequent to the privatization, all PNB employees, including Obusan, were deemed retired from the
government service. The GSIS confirmed Obusans retirement from the government service, and accordingly
paid her retirement gratuity in the net amount of P390,633.76. Thereafter, Obusan continued to be an employee
of PNB.

Later, the PNB Board of Directors, through Resolution No. 30 dated December 22, 2000, as amended,
approved the PNB Regular Retirement Plan (PNB-RRP). Section 1, Article VI of which provides

Normal Retirement. The normal retirement date of a Member shall be the day he attains sixty (60)
years of age, regardless of length of service or has rendered thirty (30) years of service, regardless
of age, whichever of the said conditions comes first. A Member who has reached the normal
retirement date shall have to compulsorily retire and shall be entitled to receive the retirement
benefits under the Plan.

PNB informed its officers and employees of the terms and conditions of the PNB-RRP, along with its
implementing guidelines.

Subsequently, the PNB-RRP was registered with the BIR. Later, the Philnabank Employees Association, the
union of PNB rank-and-file employees, recognized the PNB-RRP in the Collective Bargaining Agreement (CBA)
it entered with PNB.

PNB informed Obusan that her last day of employment would be on March 3, 2002, as she would reach the
mandatory retirement age of 60 years on March 4, 2002. Obusan questioned her compulsory retirement and
even threatened to take legal action against PNB for illegal dismissal and unfair labor practice in the form of
union busting, Obusan being then the President of the PNB Supervisors and Officers Association.

PNB replied to Obusan, explaining that compulsory retirement under the PNB-RRP is not contrary to law and
does not constitute union busting. Dissatisfied with PNBs explanation, Obusan filed before the Labor Arbiter a
complaint for illegal dismissal and unfair labor practice, claiming that PNB could not compulsorily retire her at
the age of 60 years, with her having a vested right to be retired only at 65 years old pursuant to civil service
regulations.

LA RULING: LA dismissed Obusan's complaint as he upheld the validity of the PNB-RRP and its provisions on
compulsory retirement upon reaching the age of 60 years.

NLRC RULING: NLRC dismissed Obusan's appeal, and affirmed the assailed decision in toto. Obusan's motion
for reconsideration of this resolution was later denied.

CA RULING: CA dismissed the petition ratiocinating that the PNB-RRPs lowering the compulsory retirement
age to 60 years is not violative of Article 287 of the Labor Code of the Philippines, as amended, despite the

San Beda College of Law 680


4S: 2015 - 2016
LABOR LAW REVIEW Atty. Joyrich Golangco

issuance of the plan years after Obusan was hired. Obusans motion for reconsideration of this Decision was
subsequently denied.

ISSUE: Can PNB unilaterally lower the compulsory retirement age to 60 years without violating Article 287 of
the Labor Code and Obusans alleged right to retire at the age of 65 years?

SC RULING:
Yes. Article 287 of the Labor Code, as amended by RA No. 7641, which took effect on January 7, 1993,
provides:

ART. 287. Retirement. Any employee may be retired upon reaching the retirement age established
in the collective bargaining agreement or other applicable employment contract.

In case of retirement, the employee shall be entitled to receive such retirement benefits as he may
have earned under existing laws and any collective bargaining agreement and other agreements:
Provided, however, That an employees retirement benefits under any collective bargaining
agreement and other agreements shall not be less than those provided herein.

In the absence of a retirement plan or agreement providing for retirement benefits of employees in
the establishment, an employee upon reaching the age of sixty (60) years or more, but not beyond
sixty-five (65) years which is hereby declared the compulsory retirement age, who has served at
least five (5) years in the said establishment, may retire and shall be entitled to retirement pay
equivalent to at least one-half (1/2) month salary for every year of service, a fraction of at least six
(6) months being considered as one whole year.

Unless the parties provide for broader inclusions, the term one-half (1/2) month salary shall mean
fifteen (15) days plus one-twelfth (1/12) of the 13th month pay and the cash equivalent of not more
than five (5) days of service incentive leaves.

Undoubtedly, under this provision, the retirement age is primarily determined by the existing agreement or
employment contract. Absent such an agreement, the retirement age shall be fixed by law. The above-cited law
mandates that the compulsory retirement age is at 65 years, while the minimum age for optional retirement is
set at 60 years. Moreover, Article 287 of the Labor Code, as amended, applies only to a situation where (1)
there is no CBA or other applicable employment contract providing for retirement benefits for an employee; or
(2) there is a collective bargaining agreement or other applicable employment contract providing for retirement
benefits for an employee, but it is below the requirement set by law. The rationale for the first situation is to
prevent the absurd situation where an employee, deserving to receive retirement benefits, is denied them
through the nefarious scheme of employers to deprive employees of the benefits due them under existing labor
laws. The rationale for the second situation is to prevent private contracts from derogating from the public law.

Retirement plans allowing employers to retire employees who have not yet reached the compulsory retirement
age of 65 years are not per se repugnant to the constitutional guaranty of security of tenure. By its express
language, the Labor Code permits employers and employees to fix the applicable retirement age at 60 years or
below, provided that the employees retirement benefits under any CBA and other agreements shall not be less
than those provided therein. By this yardstick, the PNB-RRP complies.

San Beda College of Law 681


4S: 2015 - 2016
LABOR LAW REVIEW Atty. Joyrich Golangco

KIMBERLY CLARK PHILS. v. DIMAYUGA, et. al.
G.R. No. 177705 September 18, 2009
CARPIO-MORALES, J.:

DOCTRINE:
Entitlement of employees to retirement benefits must specifically be granted under existing laws, a collective
bargaining agreement or employment contract, or an established employer policy.

FACTS:
Respondents were employees of Kimberly-Clark Philippines, Inc. Nora Dimayuga was Cost Accounting
Supervisor, Rosemarie Gloria was Business Analyst, and Maricar de Guia was General Accounting Manager.

On September 19, 2002, Nora tendered her resignation effective October 21, 2002. On October 7, 2002,
Rosemarie tendered her resignation, also effective October 21, 2002.

As petitioner had been experiencing a downward trend in its sales, it created a tax-free early retirement package
for its employees as a cost-cutting and streamlining measure. Twenty-four of its employees availed of the offer
that was made available from November 10-30, 2002.

Despite their resignation before the early retirement package was offered, Nora and Rosemarie pleaded with
petitioner that they be retroactively extended the benefits thereunder, to which petitioner acceded. Hence, Nora
received a total of P1,025,113.73 while Rosemarie received a total of P1,006,493.94, in consideration of which
they executed release and quitclaim deeds.

On November 4, 2002, Maricar tendered her resignation effective December 1, 2002 citing career advancement
as the reason. As at the time of her resignation the early retirement package was still effective, she received a
total of P523,540.13 for which she signed a release and quitclaim.

On November 28, 2002, petitioner announced that in lieu of the merit increase which it did not give that year, it
would provide economic assistance, to be released the following day, to all monthly-paid employees on regular
status as of November 16, 2002.

Still later or on January 16, 2003, petitioner announced that it would grant a lump sum retirement pay in the
amount of P200,000, in addition to the early retirement package benefit, to those who signed up for early
retirement and who would sign up until January 22, 2003.

On May 23, 2003, respondents filed a Complaint, before the NLRC-RAB against petitioner and its Finance
Manager Fernando B. Gomez whom respondents alleged to be responsible for the withholding of their additional
retirement benefits, claiming entitlement to the P200,000 lump sum retirement pay. Respondents Nora and
Rosemarie additionally claimed entitlement to the economic assistance.

LA RULING: LA dismissed the claims of Nora and Rosemarie, holding that they were not entitled to the
P200,000 lump sum retirement pay, they having ceased to be employees of petitioner at the time it was offered.
He, however, granted Maricar's claim for the same pay, holding that she was entitled to it because at the time
she resigned from the company effective December 1, 2002, such pay was already offered.

NLRC RULING: On appeal by both parties, NLRC modified the LA Decision by ordering petitioner to pay Nora
P200,000 additional bonus and P2,880 economic assistance, and to pay Rosemarie P200,000 additional bonus
and P2,656 economic assistance. It affirmed Maricars entitlement to the lump sum retirement pay.

CA RULING: Affirmed the NLRC.

ISSUE: Were Nora and Rosemarie entitled to the economic assistance and lump sum pay considering that they
were already retired and have in fact executed quitclaims and waivers?

SC RULING:
No. It is settled that entitlement of employees to retirement benefits must specifically be granted under existing
laws, a collective bargaining agreement or employment contract, or an established employer policy. No law or
collective bargaining agreement or other applicable contract, or an established company policy was existing
San Beda College of Law 682
4S: 2015 - 2016
LABOR LAW REVIEW Atty. Joyrich Golangco

during respondents employment entitling them to the P200,000 lump-sum retirement pay. Petitioner was not
thus obliged to grant them such pay.

Respondents reliance on Businessday case is misplaced. The factual milieu in Businessday case is markedly
different from that of the present case. That case involved the retrenched employees separation pay to which
they are entitled under Article 283 of the Labor Code. In the present case, Nora and Rosemarie resigned prior to
petitioners offer of the lump sum retirement pay as an incentive to those employees who would voluntarily avail
of its early retirement scheme as a cost-cutting and streamlining measure. That respondents resigned, and not
retrenched, is clear from their respective letters to petitioner. And nowhere in the letters is there any allegation
that they resigned in view of the company's downward trend in sales which necessitated downsizing or
streamlining.

The appellate courts finding that petitioners inclusion of Nora and Rosemarie in the termination report submitted
to the DOLE and its grant to them of the early retirement benefits made them full retirees to thus entitle them to
the same benefits offered to those who would voluntarily resign after November 16, 2003 does not lie. Neither
are they entitled to the economic assistance which petitioner awarded to all monthly employees who are under
regular status as of November 16, 2002, they having resigned earlier or on October 21, 2002.

Again, contrary to the appellate courts ruling that Nora and Rosemarie already earned the economic assistance,
the same having been given in lieu of the performance-based annual salary increase, the Court finds that the
economic assistance was a bonus over and above the employees salaries and allowances. The grant of
economic assistance to all monthly employees under regular status as of November 16, 2002 was thus well
within petitioners prerogatives.

While quitclaims executed by employees are commonly frowned upon as being contrary to public policy and are
ineffective to bar claims for the full measure of their legal rights, where the person making the waiver has done
so voluntarily, with a full understanding thereof, and the consideration for the quitclaim is credible and
reasonable, the transaction must be recognized as being a valid and binding undertaking. In the case at bar,
Nora and Rosemarie are Accounting graduates. They have not alleged having been compelled to sign the
quitclaims, nor that the considerations are unconscionable.

As for Maricar's claim to the lump sum retirement pay, the Court finds that, like Nora and Rosemarie, she is not
entitled to it. Although the incentive was offered when she was still connected with petitioner, she resigned from
employment, citing career advancement as the reason therefor. Indubitably, the incentive was addressed to
those employees who, without prior plans of resigning, opted to terminate their employment in light of the
downsizing being undertaken by petitioner. In other words, Maricar resigned from petitioner in order to find
gainful employment elsewhere a reason which has no bearing on the financial viability of petitioner.

San Beda College of Law 683


4S: 2015 - 2016
LABOR LAW REVIEW Atty. Joyrich Golangco

MAGDADARO v. PHILIPPINE NATIONAL BANK
G.R. No. 166198 July 17, 2009
CARPIO, J.

DOCTRINE: Retirement is the result of a bilateral act of the parties, a voluntary agreement between the
employer and the employee whereby the latter, after reaching a certain age, agrees to sever his or her
employment with the former.

FACTS:
Marcelino A. Magdadaro was employed by Philippine National Bank since 8 January 1968. On 21 September
1998, petitioner filed his application for early retirement under respondents Special Separation Incentive
Program (SSIP). Petitioner was then holding the position of Senior Assistant Manager of respondents Branch
Operations and Consumer Finance Division for the Visayas. Petitioner stated in his application that 31
December 1999 was his preferred effective date of retirement.

Respondent approved petitioners application for early retirement but made it effective on 31 December 1998.
Petitioner protested the acceleration of his retirement. He received, under protest, his retirement and separation
benefits amounting to P908,950.44. On 18 October 1999, petitioner filed a complaint for illegal dismissal and
payment of moral, exemplary and actual damages against respondent before the NLRC-RAB, Cebu City.

LA RULING: LA ruled that respondents insistence on the date of effectivity of petitioners retirement was not
tantamount to illegal dismissal. LA ruled that there was no dismissal to speak of because petitioner voluntarily
availed of the SSIP. Still, the LA granted petitioners preferred date of retirement and awarded him additional
retirement benefits.

NLRC RULING: NLRC affirmed the LA Decision. However, it considered petitioner's retirement on 31 December
1998 as tantamount to illegal dismissal. It ruled that while it recognized respondents prerogative to change
petitioners retirement date, management prerogative should be exercised with prudence and without malice.

CA RULING: CA ruled that the NLRC acted with grave abuse of discretion in affirming the decision of the LA.
Petitioner could not claim to have been illegally dismissed just because the date of effectivity of his retirement
did not conform to his preferred retirement date.

ISSUE: Was petitioner illegally dismissed from employment?

SC RULING:
No. Retirement is the result of a bilateral act of the parties, a voluntary agreement between the employer and
the employee whereby the latter, after reaching a certain age, agrees to sever his or her employment with the
former. Retirement is provided for under Article 287 of the Labor Code, as amended by Republic Act No. 7641,
or is determined by an existing agreement between the employer and the employee.

In this case, respondent offered the SSIP to overhaul the bank structure and to allow it to effectively compete
with local peer and foreign banks. SSIP was not compulsory on employees. Employees who wished to avail of
the SSIP were required to accomplish a form for availment of separation benefits under the SSIP and to submit
the accomplished form to the Personnel Administration and Industrial Relations Division (PAIRD) for approval.

Petitioner voluntarily availed of the SSIP. He accomplished the application form and submitted it to the PAIRD.
He only questioned the approval of his retirement on a date earlier than his preferred retirement date.

Whether petitioners early retirement within the SSIP period will improve or impair the delivery of bank services is
a business decision properly within the exercise of management prerogative. More importantly, the SSIP
provides:

7. Management shall have the discretion and prerogative in approving the applications filed under
the Plan, as well as in setting the effectivity dates for separation within the implementation
period of the Plan. (Emphasis supplied)

It is clear that it is within respondents prerogative to set the date of effectivity of retirement and it may not be
necessarily what is stated in the application. We see no grave abuse of discretion on the part of respondent in
San Beda College of Law 684
4S: 2015 - 2016
LABOR LAW REVIEW Atty. Joyrich Golangco

the exercise of this management prerogative. The exercise of management prerogative is valid provided it is not
performed in a malicious, harsh, oppressive, vindictive or wanton manner or out of malice or spite. In this case,
the NLRCs finding that petitioner received a rating of 70.5% in his working and business relations is not enough
reason to ascribe bad faith on the part of respondent in accelerating the date of effectivity of petitioners
retirement.

San Beda College of Law 685


4S: 2015 - 2016
LABOR LAW REVIEW Atty. Joyrich Golangco

PHILIPPINE LONGDISTANCE TELEPHONE COMPANY [PLDT] v. ROBERTO R. PINGOL
G.R. No. 182622 September 8, 2010
MENDOZA, J.:

ART. 306: MONEY CLAIMS

DOCTRINE: Article 291. Money Claims. All money claims arising from employer-employee relations accruing
during the effectivity of this Code shall be filed within three (3) years from the time the cause of action accrued;
otherwise they shall be barred forever. Although the Constitution is committed to the policy of social justice and
the protection of the working class, it does not necessary follow that every labor dispute will be automatically
decided in favor of labor. The management also has its own rights.

FACTS:
Pingol was hired by PLDT as a maintenance technician. In 1999, Pingol was admitted at The Medical City,
Mandaluyong City, for paranoid personality disorder due to financial and marital problems. He was later
discharged from the hospital. Thereafter, he reported for work but frequently absented himself due to his poor
mental condition.

From September 16, 1999 to December 31, 1999, Pingol was absent from work without official leave. Notices
were sent to him with a stern warning that he would be dismissed from employment if he continued to be absent
without official leave pursuant. Despite the warning, he failed to show up for work. On January 1, 2000, PLDT
terminated his services on the grounds of unauthorized absences and abandonment of office.

On March 29, 2004, four years later, Pingol filed a Complaint for Constructive Dismissal and Monetary Claims
against PLDT. PLDT filed a motion to dismiss claiming, among others, that respondents cause of action had
already prescribed as the complaint was filed four (4) years and three (3) months after his dismissal.

Pingol, however, countered that in computing the prescriptive period, the years 2001 to 2003 must not be taken
into account. He explained that from 2001 to 2003, he was inquiring from PLDT about the financial benefits due
him as an employee who was no longer allowed to do his work, but he merely got empty promises. It could not,
therefore, result in abandonment of his claim.

LA RULING: Granted petitioners Motion to Dismiss on the ground of prescription.


NLRC RULING: The NLRC in its November 15, 2006 Resolution reversed the LAs resolution and favored
Pingol.

ISSUE: Was Pingols complaint filed within the prescriptive period?

SC RULING:
No. Art. 1146. The following actions must be instituted within four years:(1) Upon an injury to the rights of the
plaintiff. As this Court stated in Callanta v. Carnation, when one is arbitrarily and unjustly deprived of his job or
means of livelihood, the action instituted to contest the legality of one's dismissal from employment constitutes,
in essence, an action predicated "upon an injury to the rights of the plaintiff," as contemplated under Art. 1146 of
the New Civil Code, which must be brought within four (4) years.

With regard to the prescriptive period for money claims, Article 291 of the Labor Code states: Article 291.
Money Claims. All money claims arising from employer-employee relations accruing during the effectivity of this
Code shall be filed within three (3) years from the time the cause of action accrued; otherwise they shall be
barred forever.

In the case at bench, Pingol himself alleged the date January 1, 2000 as the date of his dismissal in his
complaint filed on March 29, 2004, exactly four (4) years and three (3) months later. Respondent never denied
making such admission or raised palpable mistake as the reason therefor. Thus, the petitioner correctly relied
on such allegation in the complaint to move for the dismissal of the case on the ground of prescription.

The Labor Code has no specific provision on when a claim for illegal dismissal or a monetary claim accrues.
Thus, the general law on prescription applies. Article 1150 of the Civil Code states: Article 1150. The time for
prescription for all kinds of actions, when there is no special provision which ordains otherwise, shall be counted
from the day they may be brought. (Emphasis supplied)
San Beda College of Law 686
4S: 2015 - 2016
LABOR LAW REVIEW Atty. Joyrich Golangco

The day the action may be brought is the day a claim starts as a legal possibility. In the present case, January 1,
2000 was the date that respondent Pingol was not allowed to perform his usual and regular job as a
maintenance technician.

Unfortunately, respondent Pingol has no one but himself to blame for his own predicament. By his own
allegations in his complaint, he has barred his remedy and extinguished his right of action. Although the
Constitution is committed to the policy of social justice and the protection of the working class, it does not
necessary follow that every labor dispute will be automatically decided in favor of labor. The management also
has its own rights. Out of Its concern for the less privileged in life, this Court, has more often than not inclined, to
uphold the cause of the worker in his conflict with the employer. Such leaning, however, does not blind the
Court to the rule that justice is in every case for the deserving, to be dispensed in the light of the established
facts and applicable law and doctrine

San Beda College of Law 687


4S: 2015 - 2016
LABOR LAW REVIEW Atty. Joyrich Golangco

ROBERTO R. SERRANO vs. COURT OF APPEALS, NATIONAL LABOR RELATIONS COMMISSION,
MAERSK-FILIPINAS CREWING, INC. and A.P. MOLLER.
G.R. No. 139420 August 15, 2001
PUNO, J.:

ART. 306: MONEY CLAIMS

DOCTRINE: Petitioners cause of action accrued only upon respondent A.P. Moller's definite denial of his claim
in November 1993. Having filed his action five (5) months thereafter or in April 1994, we hold that it was filed
within the three-year (3) prescriptive period provided in Article 291 of the Labor Code.

FACTS:
From 1974 to 1991, respondent Maersk-Filipinas Crewing, Inc., the local agent of respondent foreign
corporation A.P. Moller, deployed petitioner Serrano as a seaman to Liberian, British and Danish ships. As
petitioner was on board a ship most of the time, respondent Maersk offered to send portions of petitioners salary
to his family in the Philippines. The amounts would be sent by money order. Petitioner agreed and from 1977 to
1978, he instructed respondent Maersk to send money orders to his family. Respondent Maersk deducted the
amounts of these money orders totaling HK$4,600.00 and 1,050.00 Sterling Pounds from petitioner's salary.
Respondent Maersk, it is also alleged, deducted various amounts from his salary for Danish Social Security
System (SSS), welfare contributions, ship club, and SSS Medicare.

Petitioner's family failed to receive the money orders petitioner sent through respondent Maersk. Upon learning
this in 1978, petitioner demanded that respondent Maersk pay him the amounts the latter deducted from his
salary. Respondent Maersk assured him that they would look into the matter, then assigned him again to board
one of their vessels.

Whenever he returned to the Philippines, petitioner would go to the office of respondent Maersk to follow up his
money claims but he would be told to return after several weeks as respondent Maersk needed time to verify its
records and to bring up the matter with its principal employer, respondent A.P. Moller. Meantime, respondent
Maersk would hire him again to board another one of their vessels for about a year.

Finally, in October 1993, petitioner wrote to respondent Maersk demanding immediate payment to him of the
total amount of the money orders deducted from his salary from 1977 to 1978. On November 11, 1993,
respondent A.P. Moller replied to petitioner that they keep accounting documents only for a certain number of
years, thus data on his money claims from 1977 to 1978 were no longer available. Likewise, it was claimed that
it had no outstanding money orders. A.P. Moller declined petitioner's demand for payment.

In April 1994, petitioner filed a complaint for collection of the total amount of the unsent money orders and illegal
salary deductions against the respondent Maersk in the POEA. The case was transferred to the NLRC.

LA RULING: Granted the claim for the amount of unsent money orders.
NLRC RULINGThe NLRC reversed and set side Labor Arbiter Amansec's decision and dismissed the case on
the ground of prescription.
CA: The appellate court dismissed his petition for having been filed out of time

ISSUE: Was the claim of petitioner filed on time?

SC RULING:
YES. Petitioner repeatedly demanded payment from respondent Maersk but similar to the actuations of Baliwag
Transit in the above cited case, respondent Maersk warded off these demands by saying that it would look into
the matter until years passed by. In October 1993, Serrano finally demanded in writing payment of the unsent
money orders. Then and only then was the claim categorically denied by respondent A.P. Moller in its letter
dated November 22, 1993. Following the Baliwag Transit ruling, petitioners cause of action accrued only upon
respondent A.P. Moller's definite denial of his claim in November 1993. Having filed his action five (5) months
thereafter or in April 1994, we hold that it was filed within the three-year (3) prescriptive period provided in
Article 291 of the Labor Code.

San Beda College of Law 688


4S: 2015 - 2016
LABOR LAW REVIEW Atty. Joyrich Golangco

INTERCONTINENTAL BROADCASTING CORPORATION v.IRENEO PANGANIBAN
G.R. No. 151407 February 6, 2007
AUSTRIA-MARTINEZ, J.:

ART. 306: MONEY CLAIMS

DOCTRINE: The prescription of an action is interrupted by (a) the filing of an action, (b) a written extrajudicial
demand by the creditor, and (c) a written acknowledgment of the debt by the debtor. On this point, the Court
ruled that although the commencement of a civil action stops the running of the statute of prescription or
limitations, its dismissal or voluntary abandonment by plaintiff leaves the parties in exactly the same position as
though no action had been commenced at all.

FACTS:
Ireneo Panganiban was employed as Assistant General Manager of IBC from May 1986 until his preventive
suspension on August 26, 1988. Respondent resigned from his employment on September 2, 1988. On April 12,
1989, respondent filed with the RTC against petitioner alleging, among others, non-payment of his unpaid
commissions. A motion to dismiss was filed by Joselito Santiago, one of the defendants, on the ground of lack of
jurisdiction, as respondent's claim was a labor money claim, but this was denied by the RTC (CIVIL CASE).
Thus, Santiago filed a petition for certiorari with the CA, the CA granted Santiago's petition for lack of
jurisdiction.

Thereafter, respondent was elected by the BOA as Vice-President for Marketing in July 1992. He resigned in
April 1993.On July 24, 1996, respondent filed against petitioner a complaint for illegal dismissal, separation pay,
retirement benefits, unpaid commissions, and damages. The LA ordered respondent's reinstatement with full
backwages, and the payment of his unpaid commission in the amount of P2,521,769.77, damages and
attorney's fees.

NLRC RULING: Due to petitioner's failure to post a bond, the appeal was dismissed on February 26, 1998, in a
Decision that was deemed final and executory.

ISSUE: Had respondent's claim for unpaid commissions in the amount of P2,521,769.77 already prescribed?

SC RULING:
YES. Like other causes of action, the prescriptive period for money claims is subject to interruption, and in the
absence of an equivalent Labor Code provision for determining whether the said period may be interrupted,
Article 1155 of the Civil Code may be applied, to wit:

ART. 1155. The prescription of actions is interrupted when they are filed before the Court, when there is a
written extrajudicial demand by the creditors, and when there is any written acknowledgment of the debt by the
debtor.

Thus, the prescription of an action is interrupted by (a) the filing of an action, (b) a written extrajudicial demand
by the creditor, and (c) a written acknowledgment of the debt by the debtor. On this point, the Court ruled that
although the commencement of a civil action stops the running of the statute of prescription or limitations, its
dismissal or voluntary abandonment by plaintiff leaves the parties in exactly the same position as though no
action had been commenced at all.

Hence, while the filing of Civil Case No. Q-89-2244 could have interrupted the running of the three-year
prescriptive period, its consequent dismissal by the CA due to lack of jurisdiction effectively canceled the tolling
of the prescriptive period within which to file his money claim, leaving respondent in exactly the same position as
though no civil case had been filed at all. The running of the three-year prescriptive period not having been
interrupted by the filing of Civil Case No. Q-89-2244, respondent's cause of action had already prescribed on
September 2, 1991, three years after his cessation of employment on September 2, 1988. Consequently, when
respondent filed his complaint for illegal dismissal, separation pay, retirement benefits, and damages in July 24,
1996, his claim, clearly, had already been barred by prescription.

San Beda College of Law 689


4S: 2015 - 2016
LABOR LAW REVIEW Atty. Joyrich Golangco

ACCESSORIES SPECIALIST INC vs. ERLINDA B. ALABANZA, for and in behalf of her deceased
husband, JONES B. ALABANZA
G.R. No. 168985 July 23, 2008
NACHURA, J.:

PROMISSORY ESTOPPEL AS AN EXCEPTION TO ARTICLE 291 OF LC

DOCTRINE: The principle of promissory estoppel is a recognized exception to the three-year prescriptive period
enunciated in Article 291 of the Labor Code. In order to make out a claim of promissory estoppel, a party bears
the burden of establishing the following elements: (1) a promise was reasonably expected to induce action or
forbearance; (2) such promise did, in fact, induce such action or forbearance; and (3) the party suffered
detriment as a result.

FACTS:
Erlinda Alabanza (Erlinda) for and in behalf of her husband Jones Alabanza (Jones) filed a complaint against
th
petitioners Accessories Specialists, Inc. (ASI) for non-payment of salaries, separation pay, and 13 month pay.
Erlinda alleged that ASI compelled Jones to file his involuntary resignation on the ground that ASI allegedly
suffered losses. Jones demanded payment of said money claims upon resignation but ASI informed him that it
would just settle first the money claims of the rank- and-file employees. Knowing the predicament of the
company, Jones patiently waited for his turn to be paid. Several demands were made by Jones but ASI just kept
on assuring him that he will be paid his monetary claims. Jones died on August 5, 2002 and failed to receive the
same.

ASI contended that Jones voluntarily resigned on October 31, 1997. Thus, Erlindas cause of action has already
prescribed and is forever barred on the ground that under Article 291 of the Labor Code, all money claims
arising from an employer-employee relationship shall be filed within three (3) years from the time the cause of
action accrues. Since the complaint was filed only on September 27, 2002, or almost five (5) years from the date
of the alleged illegal dismissal of her husband Jones, Erlindas complaint is now barred.

ISSUE:
Whether the cause of action of respondents has already prescribed?

HELD:
NO. The action has not yet prescribed. The principle of promissory estoppel is a recognized exception to the
three-year prescriptive period enunciated in Article 291 of the Labor Code. Promissory estoppel may arise from
the making of a promise, even though without consideration, if it was intended that the promise should be relied
upon, as in fact it was relied upon, and if a refusal to enforce it would virtually sanction the perpetration of fraud
or would result in other injustice. Promissory estoppel presupposes the existence of a promise on the part of
one against whom estoppel is claimed. The promise must be plain and unambiguous and sufficiently specific so
that the court can understand the obligation assumed and enforce the promise according to its terms.

In order to make out a claim of promissory estoppel, a party bears the burden of establishing the following
elements: (1) a promise was reasonably expected to induce action or forbearance; (2) such promise did, in fact,
induce such action or forbearance; and (3) the party suffered detriment as a result.

All the requisites of promissory estoppel are present in this case. Jones relied on the promise of ASI that he
would be paid as soon as the claims of all the rank-and-file employees had been paid. If not for this promise that
he had held on to until the time of his death, we see no reason why he would delay filing the complaint before
the LA. Thus, we find ample justification not to follow the prescriptive period imposed under Article 291 of the
Labor Code. Great injustice will be committed if we will brush aside the employees claims on a mere
technicality, especially when it was petitioners own action that prevented respondent from interposing the claims
within the required period.

San Beda College of Law 690


4S: 2015 - 2016
LABOR LAW REVIEW Atty. Joyrich Golangco

AUTO BUS TRANSPORT SYSTEMS, INC. vs. ANTONIO BAUTISTA
G.R. No. 156367 May 16, 2005
CHICO-NAZARIO, J.:

PRESCRIPTIVE PERIOD FOR CLAIMS OF SERVICE INCENTIVE LEAVE PAY

DOCTRINE: Applying Article 291 of the Labor Code in light of this peculiarity of the SIL, we can conclude that
the 3-year prescriptive period commences, not at the end of the year when the employee becomes entitled to
the commutation of his SIL, but from the time when the employer refuses to pay its monetary equivalent after
demand of commutation or upon termination of the employees services, as the case may be.

FACTS:
Antonio Bautista has been employed by Autobus as driver-conductor. After figuring in a vehicular accident while
performing his duties, Autobus dismissed Bautista from service. Thus, a month from his dismissal or on 02
th
February 2000, Bautista instituted a Complaint for Illegal Dismissal with Money Claims for non-payment of 13
month pay and service incentive leave pay against Autobus.

The LA upheld the validity of the dismissal but ordered Autobus to pay the money claims of Bautista. The NLRC
th
affirmed the ruling but deleted the award of 13 month pay and maintained the award of SIL. On appeal,
Autobus argued that the claim for SIL was barred by the 3-year prescriptive period under Article 291 of LC
considering that the period commences at the end of the year when the employee becomes entitled to the
commutation of his service incentive leave and not upon Bautistas dismissal from service.

ISSUE:
Whether the claim for SIL has already prescribed?

HELD: NO. In cases of nonpayment of allowances and other monetary benefits, if it is established that the
benefits being claimed have been withheld from the employee for a period longer than three (3) years, the
amount pertaining to the period beyond the three-year prescriptive period is therefore barred by prescription.
The amount that can only be demanded by the aggrieved employee shall be limited to the amount of the
benefits withheld within three (3) years before the filing of the complaint.

It is essential at this point, however, to recognize that SIL is a curious animal in relation to other benefits granted
by the law to every employee. In the case of SIL, the employee may choose to either use his leave credits or
commute it to its monetary equivalent if not exhausted at the end of the year. Furthermore, if the employee
entitled to SIL does not use or commute the same, he is entitled upon his resignation or separation from work to
the commutation of his accrued SIL.

Applying Article 291 of the Labor Code in light of this peculiarity of the SIL, we can conclude that the 3-year
prescriptive period commences, not at the end of the year when the employee becomes entitled to the
commutation of his SIL, but from the time when the employer refuses to pay its monetary equivalent after
demand of commutation or upon termination of the employees services, as the case may be.

In the case at bar, Bautista had not made use of his SIL nor demanded for its commutation until his employment
was terminated by Autobus. Neither did Autobus compensate his SIL pay at the time of his dismissal. It was only
upon his filing of a complaint for illegal dismissal, one month from the time of his dismissal, that he demanded
from Autobus the commutation of his accumulated leave credits. Therefore, the prescriptive period with respect
to his claim for SIL pay only commenced from the time Autobus failed to compensate his accumulated SIL pay
at the time of his dismissal. Since Bautista had filed his money claim after only one month from the time of his
dismissal, necessarily, his money claim was filed within the prescriptive period provided for by Article 291 of the
Labor Code.

San Beda College of Law 691


4S: 2015 - 2016
LABOR LAW REVIEW Atty. Joyrich Golangco

ONOFRE V. MONTERO, et. al. vs. TIMES TRANSPORTATION CO., INC.
G.R. No. 190828 March 16, 2015
REYES, J.:

INTERRUPTION OF THE RUNNING OF THE PRESCRIPTIVE PERIOD IN ILLEGAL DISMISSAL CASES

DOCTRINE: While the filing of the complaint for illegal dismissal before the LA interrupted the running of the
prescriptive period, its voluntary withdrawal left the petitioners in exactly the same position as though no
complaint had been filed at all. The withdrawal of their complaint effectively erased the tolling of the
reglementary period.

FACTS:
Respondent Times Transportation Co., Inc., (TTCI) employed herein 21 petitioners as bus drivers, conductors,
mechanics, welders, security guards and utility personnel. Upon TTCIs adoption of a company-wide
retrenchment program, petitioners received two notices of termination dated October 26, 1997 and November
24, 1997 from TCCI.

On May 14, 1998, 10 out of 21 petitioners filed several complaints against TTCI before the NLRC but this case
was withdrawn on March 4, 1999 upon motion by the petitioners.

Four years later, several complaints for unfair labor practice and illegal dismissal with money claims were filed
against TTCI before the LA. TTCI asserted that the petitioners cause of action had already been barred by
prescription because the complaints were filed only in June 2002 or after almost five years from the date of their
dismissal.

LA ruled that the complaints of these 10 petitioners were timely filed in June 2002 because the 8-month period
during which their cases were pending should be excluded from the four-year prescriptive period. NLRC held
that all complaints have been barred by prescription, which decision was affirmed by the CA. The petitioners
contend that the period when they filed a labor case on May 14, 1998, but withdrawn on March 22, 1999 should
be excluded from the computation of the four-year prescriptive period for illegal dismissal cases.

ISSUE: Whether or not the 8-month period when the case was pending should be excluded from computing the
4-year prescriptive period for illegal dismissal cases?

HELD: NO. SC ruled that the prescriptive period continues even after the withdrawal of the case as though no
action has been filed at all. The applicability of Article 1155 of the Civil Code in labor cases was upheld in the
case of Intercontinental Broadcasting Corporation v. Panganiban where the Court held that although the
commencement of a civil action stops the running of the statute of prescription or limitations, its dismissal or
voluntary abandonment by plaintiff leaves the parties in exactly the same position as though no action had been
commenced at all.

In the case at bar, the prescription was due to the petitioners withdrawal of their labor case. Hence, while the
filing of the said case could have interrupted the running of the four-year prescriptive period, the voluntary
withdrawal of the petitioners effectively cancelled the tolling of the prescriptive period within which to file their
illegal dismissal case, leaving them in exactly the same position as though no labor case had been filed at all.
The running of the four-year prescriptive period not having been interrupted by the filing of the said case, the
petitioners cause of action had already prescribed in four years after their cessation of employment on October
26, 1997 and November 24, 1997. Consequently, when the petitioners filed their complaint for illegal dismissal
in 2002, their claim, clearly, had already been barred by prescription.

San Beda College of Law 692


4S: 2015 - 2016
LABOR LAW REVIEW Atty. Joyrich Golangco

PENAFRANCIA TOURS AND TRAVEL TRANSPORT, INC. vs. JOSELITO P. SARMIENTO
and RICARDO S. CATIMBANG
G.R. No. 178397 October 20, 2010
NACHURA, J.:

EFFECTS OF CHANGE OF OWNERSHIP OF A BUSINESS

FACTS:
Respondents Joselito Sarmiento and Ricardo Catimbang were employed by Petitioner Penafrancia Tours as
bus inspectors. On October 30th, 2002, both Sarmiento and Catimbang were dismissed from work on the
ground of irreversible business losses. Prior to their dismissal, they were introduced to one Alfredo Perez, the
owner of ALPS Transportation, who allegedly bought Penafrancia and was now its new owner. Sarmiento and
Catimbang filed complaints for illegal dismissal, underpayment of wages, and unfair labor practice.

Traversing the complaint, petitioner admitted that respondents were among its bus inspectors. It asseverated,
however, that due to severe business losses, petitioner made the painful decision to stop its operation and sell
the business enterprise to the Perez family of ALPS Transportation. It alleged that due notice was given to the
Department of Labor and Employment, and that all its employees were duly notified and were paid their
th
corresponding separation pay, as well as their 13 month pay. The new owners maintained the business name
of petitioner, and the management of petitioner was entrusted to the new owners in October 2002, with Edilberto
Perez as Vice-President for Finance and Operations. Subsequently, several memoranda were issued by
Edilberto Perez in behalf of petitioner. Petitioner argued that the matter of rehiring respondents rested on the
sound discretion of its new owners, and the latter could not be compelled to absorb petitioners former
employees since the same was not part of the deal. Petitioner alleged that respondents submitted their
application for reemployment but, after evaluation, the new owners opted not to hire respondents.

During the pendency of the illegal dismissal case, the new owner (Perez) issued a notice that the management
of the company shall revert back to its former owners because of a rescission of sale.

The NLRC and the CA both found that petitioner failed to establish its allegation that it was suffering from
business losses, and that petitioner did not actually sell its business to the Perez family. As such, both ruled in
favour of the respondents.

ISSUE: Whether or not respondents were legally terminated from employment by reason of the sale of the
business enterprise and the consequent change or transfer of ownership/management.

RULING: NO. The Supreme Court held that the respondents were illegally dismissed. Closure of business is the
reversal of fortune of the employer whereby there is a complete cessation of business operations and/or an
actual locking-up of the doors of the establishment, usually due to financial losses. Closure of business, as an
authorized cause for termination of employment, aims to prevent further financial drain upon an employer who
can no longer pay his employees since business has already stopped.

On this ground, petitioner terminated the employment of respondents. However, what petitioner apparently
made was a transfer of ownership. It is true that, as invoked by petitioner, in Manlimos, et al. v. NLRC, et al., we
held that a change of ownership in a business concern is not proscribed by law. Lest petitioner forget, however,
we also held therein that the sale or disposition must be motivated by good faith as a condition for exemption
from liability. Thus, where the charge of ownership is done in bad faith, or is used to defeat the rights of labor,
the successor-employer is deemed to have absorbed the employees and is held liable for the transgressions of
his or her predecessor.

But, in this case, there is no successor-employer because there was no actual change of ownership. We sustain
the uniform factual finding of both the NLRC and the CA that no actual sale transpired and, as such, there is no
closure or cessation of business that can serve as an authorized cause for the dismissal of respondents.

San Beda College of Law 693


4S: 2015 - 2016
LABOR LAW REVIEW Atty. Joyrich Golangco

ANTONIO C. CARAG vs. NATIONAL LABOR RELATIONS COMMISSION, ISABEL G. PANGANIBAN-
ORTIGUERRA, as Executive Labor Arbiter, NAFLU, and MARIVELES APPAREL CORPORATION LABOR
UNION
G.R. No. 147590 April 2, 2007
CARPIO, J.:

LIABILITY OF CORPORATE OFFICERS

FACTS: National Federation of Labor Unions (NAFLU) and Mariveles Apparel Corporation Labor Union
(MACLU) (collectively, complainants), on behalf of all of MAC's rank and file employees, filed a complaint
against MAC for illegal dismissal brought about by its illegal closure of business. In their complaint dated 12
August 1993, complainants alleged the following:

1. That on July 8, 1993, without notice of any kind filed in accordance with pertinent provisions of the
Labor Code, [MAC], for reasons known only by herself [sic] ceased operations with the intention of
completely closing its shop or factory. Such intentions [sic] was manifested in a letter, allegedly claimed
by [MAC] as its notice filed only on the same day that the operations closed.
2. That at the time of closure, employees who have rendered one to two weeks work were not paid their
corresponding salaries/wages, which remain unpaid until time [sic] of this writing.
3. That there are other benefits than those above-mentioned which have been unpaid by [MAC] at the time
it decided to cease operations, benefits gained by the workers both by and under the CBA and by
operations [sic] of law.
4. That the closure made by [MAC] in the manner and style done is perce [sic] illegal, and had caused
tremendous prejudice to all of the employees, who suffered both mental and financial anguish and who
in view thereof merits [sic] award of all damages (actual, exemplary and moral), [illegible] to set [an]
example to firms who in the future will [illegible] the idea of simply prematurely closing without
complying [with] the basic requirement of Notice of Closure.

The complainants sought to implead Petitioner Antonio C. Carag in his official capacity as Chairman of the
Board as party respondents in order to guarantee satisfaction of any judgment award in favour of the illegally
dismissed employees. Carag claims that he was not afforded due process because he was not summoned to
the mandatory conference as required by the NLRC rules.

The Labor Arbiter ruled in favour of the Union, Carag was ordered to pay more than P50 million to the
employees of MAC. Upon appeal, the CA also ruled that Carag was solidarily liable with MAC.

ISSUES:
1) Whether or not Petitioners right to due process was violated.
2) Whether or not Petitioner, as a corporate officer, can be held solidarily liable with the Corporation for the
illegal dismissal of MACs employees.

RULING:
1) Yes. The SC held that It is clear from the narration in Arbiter Ortiguerra's Decision that she only summoned
complainants and MAC, and not Carag, to a conference for possible settlement. In her Decision, Arbiter
Ortiguerra stated that she scheduled the conference "upon receipt of the record of the case." At the time of
the conference, complainants had not yet submitted their position paper which contained the motion to
implead Carag. Complainants could not have submitted their position paper before the conference since
procedurally the Arbiter directs the submission of position papers only after the conference. Complainants
submitted their position paper only on 10 January 1994, five months after filing the complaint. In short, at the
time of the conference, Carag was not yet a party to the case. Thus, Arbiter Ortiguerra could not have
possibly summoned Carag to the conference. Carag vigorously denied receiving summons to the
conference, and complainants have not produced any order of Arbiter Ortiguerra summoning Carag to the
conference. A thorough search of the records of this case fails to show any order of Arbiter Ortiguerra
directing Carag to attend the conference. Clearly, Arbiter Ortiguerra did not summon Carag to the
conference.

2) To hold a director personally liable for debts of the corporation, and thus pierce the veil of corporate fiction,
the bad faith or wrongdoing of the director must be established clearly and convincingly. Bad faith is never
presumed. Bad faith does not connote bad judgment or negligence. Bad faith imports a dishonest purpose.
San Beda College of Law 694
4S: 2015 - 2016
LABOR LAW REVIEW Atty. Joyrich Golangco

Bad faith means breach of a known duty through some ill motive or interest. Bad faith partakes of the nature
of fraud.

Neither does bad faith arise automatically just because a corporation fails to comply with the notice
requirement of labor laws on company closure or dismissal of employees. The failure to give notice is not an
unlawful act because the law does not define such failure as unlawful. Such failure to give notice is a
violation of procedural due process but does not amount to an unlawful or criminal act. Such procedural
defect is called illegal dismissal because it fails to comply with mandatory procedural requirements, but it is
not illegal in the sense that it constitutes an unlawful or criminal act.

For a wrongdoing to make a director personally liable for debts of the corporation, the wrongdoing approved
or assented to by the director must be a patently unlawful act. Mere failure to comply with the notice
requirement of labor laws on company closure or dismissal of employees does not amount to a patently
unlawful act. Patently unlawful acts are those declared unlawful by law which imposes penalties for
commission of such unlawful acts. There must be a law declaring the act unlawful and penalizing the act.

In this case, Article 283 of the Labor Code, requiring a one-month prior notice to employees and the
Department of Labor and Employment before any permanent closure of a company, does not state that non-
compliance with the notice is an unlawful act punishable under the Code. There is no provision in any other
Article of the Labor Code declaring failure to give such notice an unlawful act and providing for its penalty.

Complainants did not allege or prove, and Arbiter Ortiguerra did not make any finding, that Carag approved
or assented to any patently unlawful act to which the law attaches a penalty for its commission. On this
score alone, Carag cannot be held personally liable for the separation pay of complainants.

San Beda College of Law 695


4S: 2015 - 2016
LABOR LAW REVIEW Atty. Joyrich Golangco

BPI EMPLOYEES UNION-DAVAO CITY-FUBU vs BANK OF THE PHILIPPINE ISLANDS
G.R. No. 174912, July 24, 2013
MENDOZA, J.:

EFFECT OF COMPANY MERGER ON UNION SHOP CLAUSE

DOCTRINE:

FACTS:
BOMC was created as a subsidiary of BPI, operating and functioning as an entirely separate and distinct entity.
A service agreement between BPI and BOMC was implemented wherein BOMC undertook to provide certain
services such as check clearing, card production, operations accounting, and cash servicing.

Following a merger between BPI and FEBTC, a transfer of the functions of twelve Personnel engaged in BPIs
cashiering function and FEBTCs cashiering, distribution and bookkeeping functions to BOMC was effected. The
Union objected to the transfer of the functions to BOMC claiming that the functions rightfully belonged to the BPI
employees and that the Union was deprived of membership of former FEBTC personnel, who by virtue of the
merger, would have formed part of the bargaining unit represented by the Union pursuant to its union shop
provision in the CBA.

BPI invoked management prerogative stating that the creation of the BOMC was to preserve more jobs and to
designate it as an agency to place employees where they were most needed. On the other hand, the Union
charged that BOMC undermined the existence of the union since it reduced or divided the bargaining unit. While
BOMC employees perform BPI functions, they were beyond the bargaining units coverage. In contracting out
FEBTC functions to BOMC, BPI effectively deprived the union of the membership of employees handling said
functions as well as curtailed the right of those employees to join the union.

Thereafter, the Union demanded that the matter be submitted to the grievance machinery as the resort to the
LMC was unsuccessful. As BPI allegedly ignored the demand, the Union filed a notice of strike before the
National Conciliation and Mediation Board(NCMB) on the following grounds:
a) Contracting out services/functions performed by union members that interfered
with, restrained and/or coerced the employees in the exercise of their right to
self-organization;

b) Violation of duty to bargain; and


9
c) Union busting.

BPI then filed a petition for assumption of jurisdiction/certification with the Secretary of the Department of Labor
and Employment (DOLE), who subsequently issued an order certifying the labor dispute to the NLRC for
compulsory arbitration. The DOLE Secretary directed the parties to cease and desist from committing any act
that might exacerbate the situation.

The NLRC ruled upholding the validity of the service agreement between BPI and BOMC and dismissing the
charge of ULP. The CA upheld the ruling of the NLRC, hence this petition.

ISSUE: the primordial issue in this case is whether or not the act of BPI to outsource the cashiering, distribution
and bookkeeping functions to BOMC is in violation of the law and the existing CBA. Particularly in dispute is the
validity of the transfer of twelve (12) former FEBTC employees to BOMC, instead of being absorbed in BPI after
the corporate merger.

RULING: No. The Supreme Court held that only gross violations of the economic provisions of the CBA are
treated as ULP. Otherwise, they are mere grievances.

In the present case, the alleged violation of the union shop agreement in the CBA, even assuming it was
malicious and flagrant, is not a violation of an economic provision in the agreement. The provisions relied upon
by the Union were those articles referring to the recognition of the union as the sole and exclusive bargaining
San Beda College of Law 696
4S: 2015 - 2016
LABOR LAW REVIEW Atty. Joyrich Golangco

representative of all rank-and-file employees, as well as the articles on union security, specifically, the
maintenance of membership in good standing as a condition for continued employment and the union shop
26
clause. It failed to take into consideration its recognition of the banks exclusive rights and prerogatives,
likewise provided in the CBA, which included the hiring of employees, promotion, transfers, and dismissals for
just cause and the maintenance of order, discipline and efficiency in its operations.

The Union, however, insists that jobs being outsourced to BOMC were included in the existing bargaining unit,
thus, resulting in a reduction of a number of positions in such unit. The reduction interfered with the employees
28
right to self-organization because the power of a union primarily depends on its strength in number.

It is incomprehensible how the reduction of positions in the collective bargaining unit interferes with the
employees right to self-organization because the employees themselves were neither transferred nor dismissed
from the service. As the NLRC clearly stated:

In the case at hand, the union has not presented even an iota of evidence that
petitioner bank has started to terminate certain employees, members of the union. In
fact, what appears is that the Bank has exerted utmost diligence, care and effort to see
to it that no union member has been terminated. In the process of the consolidation or
merger of the two banks which resulted in increased diversification of functions, some
of these non-banking functions were merely transferred to the BOMC without affecting
the union membership.

As far as the twelve (12) former FEBTC employees are concerned, the Union failed to substantially prove that
their transfer, made to complete BOMCs service complement, was motivated by ill will, anti-unionism or bad
faith so as to affect or interfere with the employees right to self-organization.

San Beda College of Law 697


4S: 2015 - 2016
LABOR LAW REVIEW Atty. Joyrich Golangco

GILDA C. FERNANDEZ AND BERNADETTE A. BELTRAN v. NEWFIELD STAFF SOLUTIONS, INC. and
ARNOLD "JAY" LOPEZ, JR.
G.R. No. 201979; July 10, 2013
VILLARAMA, JR., J.

CORPORATE LIABILITIES

DOCTRINE:

FACTS: Respondent Newfield Staff Solutions, Inc. hired Fernandez as Recruitment Manager starting
September 30, 2008 with a salary of P50,000 and an allowance of P6,000 per month. Newfield also hired
Beltran as probationary Recruitment Specialist starting October 7, 2008 with a salary of P15,000 and an
allowance of P2,000 per month.

On October 17, 2008, respondent Arnold "Jay" Lopez, Jr., Newfields General Manager, asked petitioners to
come to his office and terminated their employment on the ground that they failed to perform satisfactorily.
Lopez, Jr. ordered them to immediately turn over the records in their possession to their successors.

Petitioners filed a complaint for illegal dismissal, non-payment of salary and overtime pay, reimbursement of cell
phone billing, moral and exemplary damages and attorneys fees against respondents.

LA RULING: ruled that petitioners dismissal was illegal and held Lopez, Jr. solidarily liable with Newfield.
NLRC RULING: affirmed the Labor Arbiters decision.
CA RULING: reversed the NLRC and dismissed petitioners complaint for illegal dismissal.

ISSUES: a. Were the petitioners illegally dismissed?


b. Is Lopez,Jr., as General Manager, solidarily liable with Newfield?

SC RULING:
a. YES. Petitioners were illegally dismissed since there is no just cause for their dismissal.

b. NO. There is solidary liability when the obligation expressly so states, when the law so provides, or
when the nature of the obligation so requires. In MAM Realty Development Corporation v. NLRC, the
solidary liability of corporate officers in labor disputes was discussed in this wise:

"A corporation, being a juridical entity, may act only through its directors, officers and employees.
Obligations incurred by them, acting as such corporate agents, are not theirs but the direct accountabilities of
the corporation they represent. True, solidary liability may at times be incurred but only when exceptional
circumstances warrant such as, generally, in the following cases:

1. When directors and trustees or, in appropriate cases, the officers of a corporation -
(a) vote for or assent to patently unlawful acts of the corporation;
(b) act in bad faith or with gross negligence in directing the corporate affairs;
xxxx

In labor cases, for instance, the Court has held corporate directors and officers solidarily liable with the
corporation for the termination of employment of employees done with malice or in bad faith."

Bad faith does not connote bad judgment or negligence; It Imports dishonest purpose or some moral obliquity
and conscious doing of wrong; it means breach of a known duty through some motive or interest or ill will; it
partakes of the nature of fraud. To sustain such a finding, there should be evidence on record that an officer or
director acted maliciously or in bad faith in terminating the employee. But here, the Labor Arbiter and NLRC
have not found Lopez, Jr. guilty of malice or bad faith. Thus, there is no basis to hold Lopez, Jr. solidarily liable
with Newfield. Payment of the judgment award is the direct accountability of Newfield.

San Beda College of Law 698


4S: 2015 - 2016
LABOR LAW REVIEW Atty. Joyrich Golangco

SME BANK INC., ABELARDO P. SAMSON, OLGA SAMSON and AURELIO VILLAFLOR, JR. v. PEREGRIN
T. DE GUZMAN,EDUARDO M. AGUSTIN, JR., ELICERIO GASPAR, , RICARDO GASPAR JR., EUFEMIA
ROSETE, FIDEL ESPIRITU, SIMEONESPIRITU, JR., and LIBERATO MANGOBA
G.R. No. 184517 October 8, 2013
SERENO, CJ.

CHANGE OF EQUITY COMPOSITION OF CORPORATIONS

DOCTRINE: Security of tenure is a constitutionally guaranteed right. Employees may not be terminated from
their regular employment except for just or authorized causes under the Labor Code and other pertinent laws. A
mere change in the equity composition of a corporation is neither a just nor an authorized cause that would
legally permit the dismissal of the corporations employees en masse.

FACTS: Respondent employees were employees of Small and Medium Enterprise Bank, Incorporated (SME
Bank).Originally, the principal shareholders and corporate directors of the bank were Eduardo M. Agustin, Jr.
and Peregrin de Guzman, Jr. In June 2001, SME Bank experienced financial difficulties. To remedy the
situation, the bank officials sold 86.365% of the shares of stock of SME Bank to spouses Abelardo and Olga
Samson.

Espiritu, then the general manager of SME Bank, held a meeting with all the employees of SME Bank and
persuaded them to tender their resignations, with the promise that they would be rehired upon reapplication.
Relying on such representation, respondent tendered their resignations. As it turned out, respondent employees
were not rehired. Respondent-employees demanded the payment of their respective separation pays, but their
requests were denied.

Aggrieved by the loss of their jobs, respondent employees filed a Complaint and sued SME Bank, spouses
Abelardo and Olga Samson and Aurelio Villaflor (the Samson Group) for unfair labor practice; illegal dismissal;
illegal deductions; underpayment; and non-payment of allowances, separation pay and 13th month pay.
Subsequently, they amended their Complaint to include Agustin and De Guzman as respondents to the case.

Petitioner bank argued that, there being a transfer of the business establishment, the innocent transferees no
longer have any obligation to continue employing respondent employees, and that the most that they can do is
to give preference to the qualified separated employees; hence, the employees were validly dismissed

LA RULING: found that respondent employees were illegally dismissed, because they had involuntarily
executed their resignation letters after relying on representations that they would be given their separation
benefits and rehired by the new management.

NLRC RULING: found that there was only a mere transfer of shares and therefore, a mere change of
management from Agustin and De Guzman to the Samson Group. As the change of management was not a
valid ground to terminate respondent bank employees, the NLRC ruled that they had indeed been illegally
dismissed.

CA RULING: affirmed the NLRC.

ISSUE: Were the respondent employees validly dismissed on the ground of change of equity composition of the
petitioner corporation?

SC RULING: NO. Contrary to petitioner banks argument, there was no transfer of the business establishment
to speak of, but merely a change in the new majority shareholders of the corporation.

There are two types of corporate acquisitions: asset sales and stock sales. In asset sales, the corporate entity
sells all or substantially all of its assets to another entity. In stock sales, the individual or corporate shareholders
sell a controlling block of stock to new or existing shareholders.

In asset sales, the rule is that the seller in good faith is authorized to dismiss the affected employees, but is
liable for the payment of separation pay under the law. The buyer in good faith, on the other hand, is not obliged
to absorb the employees affected by the sale, nor is it liable for the payment of their claims. The most that it may

San Beda College of Law 699


4S: 2015 - 2016
LABOR LAW REVIEW Atty. Joyrich Golangco

do, for reasons of public policy and social justice, is to give preference to the qualified separated personnel of
the selling firm.

In contrast with asset sales, in which the assets of the selling corporation are transferred to another entity, the
transaction in stock sales takes place at the shareholder level. Because the corporation possesses a personality
separate and distinct from that of its shareholders, a shift in the composition of its shareholders will not affect its
existence and continuity. Thus, notwithstanding the stock sale, the corporation continues to be the employer of
its people and continues to be liable for the payment of their just claims. Furthermore, the corporation or its new
majority shareholders are not entitled to lawfully dismiss corporate employees absent a just or authorized cause.

In the case at bar, the Letter Agreements show that their main object is the acquisition by the Samson Group of
86.365% of the shares of stock of SME Bank. Hence, this case involves a stock sale, whereby the transferee
acquires the controlling shares of stock of the corporation. Thus, following the rule in stock sales, respondent
employees may not be dismissed except for just or authorized causes under the Labor Code.

San Beda College of Law 700


4S: 2015 - 2016

You might also like